Download as pdf or txt
Download as pdf or txt
You are on page 1of 390

1

ALLIGATION OR MIXTURE
NATIONALISED BANKS (LIC Assistant Administrative
& IBPS PO/MT/SO Officer (AAO) Exam. 2006)
1. In a mix tu re of mil k and w ater the 2. A man purchased 35 kg of rice at the rate
proportion of water by weight was 75% . If of Rs. 9.50 per kg and 30 kg at the rate of
in the 60 gms mixture 15 gms water was Rs. 10.50 per kg. He mixed the two.
added, what would be the percentage of Approximately, at what price (in Rupees)
water in the new mixture ? per kg should he sell the mixture to make
(1) 75% (2) 88% 35 per cent profit in the transaction ?
(3) 90% (4) 100% (1) 12 (2) 12.50
(5) None of these (3) 13 (4) 13.50
(Indian Overseas Bank PO (LIC Assistant Administrative
Exam. 05.04.2009) Officer (AAO) Exam. 07.06.2009)
3. To m litres of a m% solution of acid, x litres
of water is mixed to yield (m- 10)% solution
SBI PO EXAMS
of acid. If m > 25, then x equals
1. In 1kg mixture of sand and iron, 20% is
iron. How much sand should be added, so 10m 5m
(1) (2)
that the proportion of iron becomes 5% ? m  10 m  10
(1) 3 kg (2) 4 gms
(3) 5 gms (4) 6 kg 2m m
(3) (4)
(5) None of these m  10 m  10
(SBI Associate Banks (New India Assurance
PO Exam. 14.02.1999) AO Exam. 25.10.2009)
2. The wheat sold by a grocer contained 10% 4. When one litre of water is added to a
low quality wheat. What quantity of good mixture of acid and water, the new mixture
quality wheat should be added to 150 kgs contains 20% acid. When one litre of acid
of wheat so that the percentage of low is added to the new mixture, then the
quality wheat becomes 5% ?
1
(1) 150 kgs (2) 135 kgs resulting mixture contains 33 % acid. The
(3) 50 kgs (4) 85 kgs 3
(5) None of these percentage of acid in the original mixture
(SBI Associate Banks PO was
Exam. 21.07.2002) (1) 20% (2) 22%
3. A grocer purchased 2 kg. of rice at the rate (3) 24% (4) 25%
of Rs. 15 per kg. and 3 kg. of rice at the (New India Insurance AAO
rate of Rs. 13 per kg. At what price per kg Exam. 22.05.2011)
5. Two barrels contain a mixture of ethanol
1 and gasoline. The content of the ethanol is
should he sell the mixture to earn 33 %
3 60% in the first barrel and 30% in the
profit on the cost price ? second barrel. In what ratio must the
(1) Rs. 28.00 (2) Rs. 20.00 mixtures from the first and the second
(3) Rs. 18.40 (4) Rs. 17.40 barrel s be take n to form a mix tu re
(5) None of these containing 50% ethanol ?
(SBI PO Exam. 26.11.2006) (1) 1 : 2 (2) 2 : 1
(3) 2 : 3 (4) 3 : 2
INSURANCE EXAMS (General Insurance Corporation
1. The wheat sold by a grocer contained 10% AAO Exam. 11.12.2011)
low quality wheat. What quantity of good 6. To x litres of an x% solution of acid, y litres
quality wheat should be added to 150 kgs of water is added to get (x -10)% solution of
of wheat so that the percentage of low acid. If x > 20, then value of y is
quality wheat becomes 5% ?
x2 10 x
(1) 150 kgs (2) 135 kgs (1) (2)
(3) 50 kgs (4) 85 kgs 100 x  10
(5) None of these

LEARN MATHS FROM S.K. RAJU (9811549822, 9811649822)


2
8. A container has 30 litres of water. If 3 litres
10 x 10 x 2 of water is replaced by 3 litres of spirit and
(3) (4)
x  10 x  10 this operation is repeated twice, what will be
(Oriental Insurance Company the quantity of water in the new mixture ?
AAO Exam. 08.04.2012) (1) 24 litres (2) 23 litres
7. A and B are two alloys of gold and copper (3) 24.3 litres (4) 23.3 litres
prepared by mixing metals in the ratio 7 : 2 (5) None of these
and 7 : 11 respectively. If equal quantities (LIC Assistant Administrative
of the alloys are melted to form a third alloy Officer (AAO) Exam. 12.05.2013)
C, the ratio of gold and copper in C will be :
(1) 5 : 7 (2) 5 : 9
(3) 7 : 5 (4) 9 : 5
(Oriental Insurance Company
AAO Exam. 08.04.2012)

LEARN MATHS FROM S.K. RAJU (9811549822, 9811649822)


3
SHORT ANSWERS 2000
 x= = 4000 gms
5
NATIONALISED BANKS  Required answer = 4000 gms - 1000 gms
& IBPS PO/MT/SO = 3000 gms = 3 kg.
1. (5) 2. (1) Good quality content in 150 kgs of wheat
= 90% of 150 = 135 kg.
SBI PO EXAMS In new mixture, low quality wheat is 5% ,
l. (1) 2. (1) so good quality wheat 95%
3. (3)  5% of the new mixture = 15 kg,
15  100
INSURANCE EXAMS  New mixture = = 300 kg
5
1. (1) 2. (4)
 Good quality of wheat added
3. (1) 4. (4) = (300 - 150)kg. = 150 kg.
5. (2) 6. (2) 3. (3) Mixture : 2 kg of rice at Rs.
7. (3) 8. (3) 15/kg + 3 kg of rice at Rs. 13/kg
Total weight = 2 + 3 = 5 kg
EXPLANATIONS Total cost price = (2 × 15) + (3 × 13)
NATIONALISED BANKS = 30 + 39 = Rs. 69
& IBPS PO/MT/SO Cost price per kg of the mixture
1. (5) In 60 gm of mixture, Quantity of water 69
= = Rs.13.80
75 5
= 60 × = 45gm
100 1
Quantity of milk = 15 gm Selling price to get 33 % profit
3
After mixing 15 gm of more water, Quantity
of water in the new mixture = 45 + 15 1
10  33
= 60 gm 3 × Rs 13.80
=
 Quantity of water in 75 gm of mixture 100
= 60 gm
 100 gm of of mixture will contain 400
= × Rs 13.80
60 3  100
= × 100 = 80% of water
75 4
= × Rs 13.80 = Rs 18.40
SBI PO EXAMS 3
1. (1) Amount of iron in 1 kg mixture = 20%
of 1000 gms INSURANCE EXAMS
20  1000 1. (1) Good quality content in 150 kgs of wheat
= gms = 90% of 150 = 135 kg.
100
In new mixture, low quality wheat is 5% ,
= 200 gms so good quality wheat 95%
 Amount of sand in mixture  5% of the new mixture =15 kg
= (1000 - 200) gms = 800 gms
Now, let the total mixture is x kg in which 15  100
iron is 20%  New mixture = 5
= 300 kg
 According to the question,  Good quality of wheat added
5% of x = 200 gm = (300 - 150) kg. = 150 kg.
 5% of x = 200 2. (4) CP of 65 kg of the mixture
5 x = Rs. (35 × 9.50 + 30 × 10.50)
 = 200 = Rs. (332.5 + 315) = Rs. 647.5
100
 647.5 
200 100  Rate per kg of the mixture = Rs  65 
 x= gms  
5
647.5 135
 Required rate = ×
65 100
LEARN MATHS FROM S.K. RAJU (9811549822, 9811649822)
4
 Rs 13.50/kg
3. (i) Tricky approach x2
Acid =
100
m m2
Acid = m =
100 100 x2
Water = x 
100
m 2 100
 
New percentage = m  x 100 x2
 100 × 100 = x - 10
m2 x y
 = m - 10
m x
x2
 m2 - 10m + mx - 10x = m2  x  y = x - 10
 x (m - 10) = 10m
 x2 = y2 - 10x - y (x - 10)
10m
x= 10 x
m  10 10 x
y= =
4. (4) If there be 1 litre of acid in 4 litres of x  10 x  10
mixture, then in case I. because x > 20
Percentage of acid 7. (3) In 1 kg of alloy A,
1 7
= × 100 = 20% Gold =
4 1 18
Case II,
2
Percentage of acid Copper =
9
2 100 1 In 1 kg of alloy B,
= × 100 = = 33 %
6 3 3
7 11
 Percentage of acid in original mixture Gold = Copper =
= 25% 18 18
5. (2) By alligation,  Ratio of gold and copper in alloy C
Mixture-1 Mixture -II 7 7 2 11
= + : + = 21 : 15 = 7 : 5
3 3 9 18 9 18
Ethanol- Ethanol-
5 10 8. (3) Suppose a container contains x units of
liquid from which y units are taken out and
replaced by water. After n operations, the
quantity of pure liquid
n
 y
= x 1   units
 x
Remaining water
1 3 2 1 3 1 1
- = = - = 2
2 10 10 5 5 2 10  3 
= 30  1  
 Required ratio  30 
1 1 30  9  9
= : =2:1 = = 24.3 litres
5 10 100
6. (2) In litres of solution,

LEARN MATHS FROM S.K. RAJU (9811549822, 9811649822)


5
MODEL EXERCISES
1. several litres of acid were drawn off a 54 L 50% alcohol, to a lotion containing 30%
vessel full of acid and an equal amount of alcohol is :
water added, Again, the same volume of the (1) 5 mL (2) 4 mL
mixture was drawn off and replaced by (3) 3 mL (4) 6 mL
water. As a result the vessel contained 24 (5) None of these
L of pure acid. How much of the acid was 8. A person has a chemical of Rs 25 per litre.
drawn off initially ? In what ratio should water be mixed in that
(1) 12 L (2) 16 L chemical, so that after selling the mixture at
(3) 18 L (4) 24 L Rs 20 per litre he may get a profit of 25% ?
(5) None of these (1) 13 : 16 (2) 12 : 15
2. 37.85% and 92% alcoholic solutions are (3) 16 : 9 (4) 19 : 22
mixed to get 35 L of an 89% alcoholic (5) None of these
solution. How many litres of each solution 9. 50 g of an alloy of gold and silver contains
are there in the new mixture ? 80% gold (by weight). The quantity of gold,
(1) 10 of the first and 25 of the second that is to be mixed up with this alloy, so
(2) 20 of the first and 15 of the second that it may contain 95% gold, is
(3) 15 of the first and 20 of the second (1) 200 g (2) 50 g
(4) None of the above (3) 150 g (4) 10 g
(5) None of these (5) None of these
3. Three containers, A, B and C are having 10. A bottle contains 3/4 of milk and the rest
mixtures of milk and water in the ratio 1 : water. How much of the mixture must be
5, 3 : 5 and 5 : 7 respectively. If the taken away and replaced by an equal
capacities of the containers are in the ratio quantity of water so that the mixture has
5 : 4 : 5, then find the ratio of the milk to half milk and half water ?
the water, if the mixtures of all the three
1
containers are mixed together. (1) 25% (2) 33 %
(1) 51 : 115 (2) 52 : 115 3
(3) 53 : 115 (4) 54 : 115 (3) 45% (4) 50%
(5) None of these (5) None of these
4. A chemist has 10 L of a solution that is 10% 11. A mixture of 45 L of spirit and water
nitric acid by volume. He wants to dilute contains 20% of water in it. How much
the solution to 40% strength by adding water must be added to it make the water
water. How many litres of water must be 25% in the new mixture ?
add ? (1) 5 L (2) 3 L
(1) 15 (2) 20 (3) 4 L (4) 6 L
(3) 18 (4) 25 (5) None of these
(5) None of these 12. In a mixture of 60 L, the ratio of milk and
5. 5 L of water is added to a certain quantity water is 2 : 1. If this ratio is to be 1 : 2, then
of pure milk costing Rs 3/L. If by selling the quantity of water to be further added is
the mixture at the same price as before, a (1) 30 L (2) 20 L
profit of 20% is made, then what is the (3) 40 L (4) 60 L
amount of pure milk in the mixture ? (5) None of these
(1) 20 L (2) 30 L 13. One test tube contains some acid and
(3) 25 L (4) 35 L another test tube contains an equal
(5) None of these quantity of water. To prepare a solution, 20
6. How many kg of sugar costing Rs 5.75 per g of the acid is poured into the second test
kg should be mixed with 75 kg of cheaper tube. Then, two-thirds of the so formed
sugar costing Rs 4.50 per kg so that the solution is poured from the second tube
mixture is worth Rs 5.50 per kg ? into the first. If the fluid in the first test
(1) 350 kg (2) 300 kg tube is four times that in the second, what
(3) 250 kg (4) 325 kg quantity of water was taken initially ?
(5) None of these (1) 80 g (2) 60 g
7. The amount of water (in ml.) that should (3) 40 g (4) None of these
be added to reduce 9 mL lotion, containing (5) All of these

LEARN MATHS FROM S.K. RAJU (9811549822, 9811649822)


6
14. One type of liquid contains 25% of milk, 17. Vijay purchased two different kinds of
the other contains 30% of milk. A container alcohol. In the first mixture the ratio of
is filled with 6 parts of the first liquid and 4 alcohol to water is 3 : 4 and in the second
parts of the second liquid. The percentage mixture it is 5 : 6. If he mixes the two given
of milk in the mixture is— mixtures and makes a third mixture of 18
(1) 27% (2) 31% litres in which the ratio of alcohol to water
(3) 29% (4) 33% is 4 : 5, the quantity of the first mixture
(5) None of these (whose ratio is 3 : 4) that is required to make
15. A solution of sugar syrup has 15% sugar. 18 litres of the third kind of mixture is
Another solution has 5% sugar. How many (1) 6 (2) 7
litres of the second solution must be added (3) 8 (4) 9
to 20 L of the first solution to make a (5) None of these
solution of 20% sugar. 18. Two casks of 48 litres and 42 litres are filled
(1) 10 L (2) 5 L with mixtures of milk and water; the
(3) 15 L (4) 20 L proportions in the two casks being respec-
(5) None of these tively, 13 : 7 and 18 : 17. If the contents of
16. Mixture of milk and water has been kept in the two casks be mixed, and 20 litres of
two separate containers. Ratio of milk to water be added to the whole, what will be
water in one of the containers is 5 : 1 and the proportion of milk and water in the
that in the other container is 7 : 2. In that resulting mixture ?
ratio the mixture of these two containes (1) 5 : 12 (2) 7 : 13
should be added together, so that the quan- (3) 12 : 13 (4) 8 : 15
tity of milk in the new mixture may become (5) None of these
80% ?
(1) 2 : 3 (2) 3 : 2
(3) 4 : 5 (4) 3 : 4
(5) None of these

LEARN MATHS FROM S.K. RAJU (9811549822, 9811649822)


7
SHORT ANSWERS 4
l. (3) 2. (4) Then, (10 + x) × =1
100
3. (3) 4. (1)  x = 15 L
5. (3) 6. (2) 5. (3) Let the quantity of pure milk be x L.
7. (4) 8. (3) If 5 L of water is added to it, then cost of
9. (3) 10. (2) (5 + x)L
11. (2) 12. (4) = Rs (3x + 5)
13. (4) 14. (1)
 Rs Profit = Rs 15
15. (4) 16. (1) Given, 20% of 3x =15
17. (2) 18. (3)
EXPLANATIONS 3x
 = 15
1. (3) According to question, if a container 5
contains y units of liquid and x units of  3x = 75
liquid is taken out. If this operation is  x = 25 L
repeated n times. The final quantity of the  The amount of pure milk in the mixture
luquid in the container is was 25 L.
n
 x 6. (2) Using Alligation Method,
1
x  y  Sugar I Sugar II
5.75 4.50
2
 x 
 24 = 54  1  54 
 
2
 x  24 4
 1   = =
 54  54 9
2
 x  2 5.50 - 4.50 5.75 - 5.50
 1   =
 54  3 1.00 0.25
i.e., 4 : 1
x 1 Hence, the required quantity of Sugar I
 =
54 3
75
 x = 18 L =
1
× 4 = 300 kg
2. (4) Let x L of 37.85% of alcoholic solution
7. (4) Required quantity of water
and (35 - x)L of 92% alcoholic solution are
required to get 35 L of 89% solution. 9   50  30 
9  20
= = = 6 mL
 37.85% of x + 92% of (35 - x) 30 30
= 89% of 35 8. (3) Selling price of mixture = Rs 20
 x = 1.94 Cost price of mixture
 1.94 L of 1st solution and 33.06 L of Ilnd
100
solution are there in the new mixture. = × 20 = Rs16
3. (3) By question ratio of milk and water 125
By the rule of alligation,
1 3 5  5 5 7 
=   5   4   5 :   5   4   5 
6 8 12   6 8 12 
= 106 : 230 = 53 : 115
4. (1) According to question, quantity of nitric
acid
1
= 10 × =1L So, required ratio = 16 : 9
10
Water = 10 - 1 = 9 L 9. (3) According to question, quantity of gold
Let x L of water be added. 80
in alloy = 50 × = 40 g
100
Let x g of gold is mixed with this alloy.
LEARN MATHS FROM S.K. RAJU (9811549822, 9811649822)
8
Quantity of water in mixture
40  x
× 100 = 95
50  x 1
= × 60 = 20 L
3
40  x 95
 = Let quantity of water to be further added
50  x 100 be x.
 4000 + 100x = 4750 + 95x
40 1
Then, =
750 20  x 2
x= = 150 g
5  20 + x = 80
10. (2) According to question,  x = 60 L
Ratio of milk and water = 3 : 1 13. (4) Initially, let x g of water and Acid was
1 taken. Initially 1st process
Water = × 100 = 25 First test tube = (x - 20) g
4
Let x L of mixture is taken away, then Second test tube = (x + 20) g
2nd process
 3x 
quantity of milk left =  3   2
 4  First test tube = (x - 20) + (x + 20) ×
3
 x
and water left =  1   + x 1
 4 Second test tube = (x + 20) ×
3
3x x A/q,
Given, 3 - =1- +x
4 4 2 1
(x - 20) + (x + 20) = 4× (x + 20)
3x x 3 3
3- 1= - +x
4 4 2
 x - 20 = (x + 20)
6x 3
2=
4  3x - 60 = 2x - 40
4  x = 100 g
x= 14. (1) According to question, milk in mixture
3
25 30 270
4 1 6 × + 4 × =
Required percentage = × 100 = 33 % 100 100 100
3 4 3
11. (2) According to the question, 270  10
For 10 parts = = 27%
Quantity of water in the mixture 100
20 15. (4) Let x L of second solution must be added.
= × 45 = 9 L
100 15  20  5  x
Then, = 10
Let x L of water must be added to make the 20  x
water 25% in the  300 + 5x = 200 + 10x
9 x 100
 × 100 = 25 x = = 20L
45  x 5
9 x 25 16. (1) Percentage of milk in first mixture
 =
45  x 100 5 250
= ×100 = %
 36 + 4x = 45 + x 6 3
 3x = 9 Percentage of milk in second mixture
x = 3L 7 700
12. (4) By question quantity of milk in mixture = × 100 = %
9 9
2 Using Alligation method,
= × 60 = 40 L
3

LEARN MATHS FROM S.K. RAJU (9811549822, 9811649822)


9
18. (3) In mixture A of 48 litres,
13 156
Milk= × 48 = litres
20 5
7 84
Water = × 48 = litres
20 10 20 5
So, required ratio = : =2:3 In mixture B of 42 litres,
9 3
17. (2) Using Alligation method, 18 108
Milk =   × 42 = litres
35 5
17 102
Water = × 42 = litres
35 5
In the final mixture,
156 108 264
Milk = + = litres
5 4 1 4 3 1 5 5 5
- = - =
11 9 99 9 7 63
84 102 186
Water = + = litres
1 1 1 1 5 5 5
 Ratio = 99 : 63 = : 7
11  Required ratio
= 7:11 264 186
 Required quantity =
5
:
5
+ 20
7 264 : 286 = 12 : 13
= × 18 = 7 litres
18

LEARN MATHS FROM S.K. RAJU (9811549822, 9811649822)


1
APPROXIMATE VALUE
NATIONALISED BANKS Directions (11-15): What approximate
& IBPS PO/MT/SO value should come in place of the question mark
Directions (1-5): Find out the approximate (?) in the following questions ? (You are not
value which should replace the question mark (?) expected to calculate the exact value).
in the following questions. (You are not expected (Union Bank of India PO Exam. 27.11.2005)
to find out the exact value) 11. 63.9872 × 9449.8780  243.0034 = (?)2
(Canara Bank PO Exam. 09.02.2003) (1) 2489 (2) 2500
1. 953.7  950.9989 = 95? (3) 50 (4) 45
(1) 1.9 (2) 3 (5) 150
(3) 2.99 (4) 3.6 12. 5237.897 - 6629.010 + 7153.999 -
(5) 2.7 2205.102 = ?
(1) 6340 (2) 4688
3.001
2. 10000 + 4.987 of 1891.992 = ? (3) 5240 (4) 3558
(5) 6290
(1) 2500 (2) 1230 13. 4985.0346  215.987 - 3768.112 
(3) 1640 (4) 1525 206.868 = ?
(5) 2130 (1) 8 (2) 5
3. 0.0004  0.0001 × 36.000009 = ? (3)18 (4)11
(1) 0.10 (2) 1.45 (5) 15
(3) 145 (4) 14.5
(5) 1450 14. 956240 = ?
4. 137% of 12345 = ? (1) 979 (2) 864
(1) 17000 (2) 15000 (3) 1009 (4) 647
(3) 1500 (4) 14300 (5) 783
(5) 6300 15. 459% of 849.947 + 266% of 6284.012 -
5. 3739 + 164 × 27 = ? 1486.002 = ?
(1) 102400 (2) 4000 (1) 20330 (2) 12640
(3) 8200 (4) 690 (3)15000 (4) 22160
(5) 6300 (5) 19130
Directions (6-10): What approx imate Directions (16-20) : What aproximate
value will come in place of the question mark (?) value should come in place of the question-mark
in the following equations ? (?) in the followlg questions ? (You are not expected
(Syndicate Bank PO Exam, 10.10.2004) calculate the exact value).
6. 447.75  28 × 4.99 = ? (Corporation Bank PO
(1) 60 (2) 70 Exam. 9.07.2006)
(3) 72 (4) 80 16. 6,23,898 × 99 = ? × 60,000
(5) 75 (1) 1000 (2) 1030
7. (3.5)2 × 19.25 + ? = 275 (3) 1050 (4) 1065
(1) 15 (2) 20 (5) 1010
(3) 30 (4) 28 4 3 6 5
(5) 40 17. ×   =?
5 7 7 9
8. 85% of 225 + 32.91 × 5.01 = ?
(1) 340 (2) 355 9 20
(3) 375 (4) 345 (1) (2)
17 49
(5) 370
9. (15.96)2 + 75% of 285 = ? 18 1
(3) (4)
(1) 435 (2) 485 25 2
(3) 440 (4) 420
(5) 470 4
(5)
10. 1679  14.95 × 5.02 = ? 7
(1) 540 (2) 525 18. (399.98)2 = ?
(3) 545 (4) 565 (1) 160000 (2) 15999
(5) 520 (3) 1600 (4) 1599
(5) 16000

LEARN MATHS FROM S.K. RAJU (9811549822, 9811649822)


2
1 (3) 556 (4) 716
19. 624.9995 + (4.9989) = ?  4.9900865
2
(5) 586
29. (444% of 531)  972 = ?
(1) 4.5 (2) 0.5
(1) 6 (2) 50 (3) 2.5 (4) 8.5
(3) 10 (4) 125 (5) 6.5
(5) 15 30. (9321 + 5406 + 1001)  (498 + 929 + 660) =
20. 989.001 + 1.00982 × 76.792 = ? ?
(1) 1000 (2) 1100 (1) 13.5 (2) 4.5
(3) 1065 (4) 110 (3) 16.5 (4) 7.5
(5) 100 (5) 10.5
Directions (21-25) : What approximate Directions (31-35) : What approximate
value should come in place of the question mark value should come in place of question mark (?) in
(?) in the following question ? (You are not expected the folowing questions ?(You are not expected to
to calculate the exact value). calculate the exact value.)
(Bank Of Maharashtra PO (Andhra Bank Po Exam. 14.09.2008)
Exam. 25.05.2008 31. (11.49)4 = ?
3 4 2 (1) 15544 (2) 16729
21. × × × 3719 = ? (3) 17430 (4) 18443
7 9 5
(1) 341 (2) 283 (5) 19031
(3) 274 (4) 301 32. (2198 - 1347 - 403)  (159 - 113 - 27) = ?
(5) 288 (1) 15 (2) 24
22. 0.008 + 6.009  (0.7)2 = ? (3) 37 (4) 49
(1) 21 (2) 6 (5) 53
(3) 12 (4) 8 33. (825 % of 330)  507 = ?
(5) 18 (1) 5 12) 11
(3) 17 (4) 23
23.  3
795657  7   (3.8 × 5.5) = ? (5) 27
(1) 48 (2) 22 34. 888888 × 1.486 = ?
(3) 43 (4) 26 (1) 1200 (2) 1000
(5) 31 (3) 1600 (4) 1400
24. 98 × 785  (285)2 = ? (5) 800
(1) 0.3 (2) 1.8 35. 564.666 + 82.5091 × 44.581 - 34.111 = ?
(3) 2.2 (4) 0.9 (1) 28450 (2) 4000
(5) 0.08 (3) 1600 (4) 14225
25. 749 × 0.56 + 14.38 = ? (5) 4210
(1) 30 (2) 35 Directions (36-40) : What approximate
(3) 42 (4) 25 value will come in place of the question mark (?) in
(5) 45 the following questions ?
Directions (26-30) : What approximate (Bank Of Baroda Specialist
value should come in place question mark (?) in Officer Exam. 05.10.2008
the following questions ? (You are not expected to 36. (47% of 1442 - 36% of 1412)  63 = ?
calculate the exact value.) (1) 4 (2) 5
(Indian Overseas Bank PO (3) 3 (4) 6
Exam. 15.06.2008) (5) 1

 
26. 459.008 + 3.0056 × 88.862 = ? 1
(1) 738 (2) 725 37. 7921  2070.25 × =?
4
(3) 695 (4) 752 (1) 11 (2) 14
(5) 666 (3) 15 (4) 9
27. (621.52)2 = ? (5) 13
(1) 386300 (2) 379300 38. (341789 + 265108)  (8936 - 3578) = ?
(3) 398300 (4) 365300 (1) 150 (2) 113
(5) 356300 (3) 135 (4) 100
28. 561204 × 58 = ? × 55555 (5) 125
(1) 606 (2) 646
LEARN MATHS FROM S.K. RAJU (9811549822, 9811649822)
3
39. 29% of 725 = 60% of 315 + ? (1) 110 (2) 70
(1) 28 (2) 30 (3) 30 (4) 20
(3) 15 (4) 18 (5) 50
(5) 21 50. 16.8% of 222 × 12.1% of 923 = ?
40. 1595  25 × 36.5 = ? (1) 3325 (2) 5085
(1) 2459 (2) 2329 (3) 2925 (4) 4165
(3) 2359 (4) 2429 (5) 6245
(5) 2349 Directions (51-55) : What approximate
Directions (41-45) : What approximate value should come in place of question mark (?) in
value should come in place of the question Mark the following questions ?(You are not expected to
(?) in the following questions ? (You are not calculate the exact value.)
expected to calculate the exact value.) (Canara Bank PO Exam. 15.03.2009)
(Oriental Bank of Commerce 51. (4576 + 3286 + 5639)  (712 + 415 + 212) =
PO Exam. 21.12.2008) ?
41. 63251 × 82 = ? × 42105 (1) 18 (2) 22
(1) 101 (2) 123 (3) 34 (4) 10
(3) 147 (4) 165 (5) 46
(5) 189 52. 675.456 + 12.492 × 55.671 = ?
42. 84111 = ? (1) 971 (2) 1071
(1) 240 (2) 270 (3) 1171 (4) 1271
(3) 330 (4) 290 (5) 1371
(5) 310 53. (447.22)2 = ?
(1) 200000 (2) 210000
43. (54.78)2 = ?
(3) 220000 (4) 230000
(1) 3000 (2) 3300
(5) 240000
(3) 3500 (4) 3700
54. 4374562 × 64 = ? × 7777
(5) 3900
(1) 360 (2) 3600
44. (7171 + 3854 + 1195)  (892 + 214 + 543) =
(3) 36000 (4) 360000
?
(5) 3600000
(1) 13 (2) 18
55. (872% of 659)  543 = ?
(3) 3 (4) 26
(1) 17 (2) 11
(5) 7
(3) 21 (4) 27
45. (562% of 816 ) + 1449 = ? (5) 31
(1) 4145 (2) 5675 Directions (56-60) : What approximate
(3) 6035 (4) 7325 value should come in place of the question mark
(5) 8885
(?) in the following questions ? (Note: You are not
Directions (46-50) : What approximate
expected to calculate the exact value.)
value should come in place of the question mark
(UCO Bank PO Exam. 22.03.2009)
(?) in the following question ?
(PNB Agriculture Officer 56. 3
1500 = ?
Exam. 04.01.2009) (1) 11 (2) 6
46. 888888  88  8 = ? (3) 15 (4) 19
(1) 80800 (2) 1047 (5) 4
(3) 1263 (4) 70600 3 1 1
(5) 1526 57. 1 ×2 ×7 =?
5 7 3
47. 193.999 + 228.008 + ? + 422.005 = 1168.01 (1) 17 (2) 13
(1) 226 (2) 484 (3) 9 (4) 29
(3) 168 (4) 196 (5) 25
(5) 324 58. 18.999 × 12.005 × 25.998 = ?
48. 27.8 × 28.74 × 17.3 = ? (1) 4860 (2) 6470
(1) 13822 (2) 12546 (3) 3320 (4) 5930
(3) 10228 (4) 15183 (5) 4590
(5) 14995 59. 11.5% of 666 × 18.3% of 888 = ?
5 12 8 (1) 15608 (2) 12446
49. 1 ×6 ×5 =? (3) 10520 (4) 18338
7 13 9
(5) 11542
LEARN MATHS FROM S.K. RAJU (9811549822, 9811649822)
4
60. 2898  22  2 = ? Directions (71-75) : What approximate
(1) 278 (2) 52 value will come in place of the question mark (?) in
(3) 66 (4) 43 the following questions? (You are not expected to
(5) 263 calculate exact value).
Directions (61-65) : What approximate (PNB Specialist Officer’s
value will come in place of the question mark (?) in Exam. 16.08.2009)
the following questions ? 71. 1580.05 × 23.98 = ?
(United Bank of India PO (1) 36900 (2) 36800
Exam. 21.06.2009) (3) 37500 (4) 37900
61. 175 × 28 + 275 × 27.98 = ? (5) 37200
(1) 11800 (2) 12600 72. 77.077  7.07 × 6.08 = ?
(3) 12800 (4) 11600 (1) 57 (2) 46
(5) 12200 (3) 48 (4) 77
62. 324.995 × 15.98  4.002 + 36.88 = ? (5) 66
(1) 1300 (2) 1230 73. (16.01)2 - (8.99)2 = ?
(3) 1440 (4) 1380 (1) 175 (2) 180
(5) 1340 (3) 170 (4) 165
63. 1164 × 128  8.008 + 969.007 = ? (5) 185
(1) 18800 (2) 19000 74. 171% of 399 = ?
(3) 19600 (4) 19200 (1) 740 (2) 720
(5) 18600 (3) 680 (4) 640
64. (5) 620
624.98 + 729.25 = ?
(1) 58 (2) 56 75. 224 × 785 = ?
(3) 52 (4) 63 (1) 400 (2) 420
(5) 61 (3) 440 (4) 405
65. 69.008% of 699.998 + 32.99% of 399.999 ? (5) 435
(1) 615 (2) 645 Directions (76-80) : What approximate
(3) 675 (4) 715 value should come in place of question mark (?) in
(5) 725 the following questions ? (Note : You are not
Directions (66 - 70): What approximate expected to calculate the exact value.)
value will come in place of the question mark (?) in (Corporation Bank PO
the following questions ? (You are not expected to Exam. 22.11.2009)
calculate exact value). 76. 23.999 × 9.004 × 16.997 = ?
(Andhra Bank PO Exam. 05.07.2009) (1) 3200 (2) 4100
66. 7999.99 + 72 × 49.99 = ? (3) 2700 (4) 3700
(1) 12000 (2) 12600 (5) 4500
(3) 12500 (4) 11600
7 4 2
(5) 11000 77. 5 ×8 ×9 =?
67. (25.01)2 - (15.99)2 = ? 9 5 3
(1) 361 (2) 381 (1) 490 (2) 590
(3) 369 (4) 375 (3) 540 (4) 460
(5) 356 (5) 520
68. 380 × 12.25 - 365  15 = ? 78. 5940  28  6 = ?
(1) 4500 (2) 4550 (1) 40 (2) 35
(3) 4800 (4) 4850 (3) 46 (4) 52
(5) 4630 (5) 27
69. 180% of 25501 + 50% of 28999 = ? 79. 15.5% of 850 + 24.8% of 650 = ?
(1) 62400 (2) 64000 (1) 295 (2) 330
(3) 60400 (4) 64200 (3) 270 (4) 375
(5) 61600 (5) 220
70. 171.995 × 14.995  25 = ? 80. 2230 = ?
(1) 105 (2) 115 (1) 54 (2) 59
(3) 110 (4) 125 (3) 41 (4) 37
(5) 120 (5) 47

LEARN MATHS FROM S.K. RAJU (9811549822, 9811649822)


5
Directions (81-85) : What approximate Directions (91-95) : What approximate
value should come in the place of the question value should come in place of the question mark
mark (?) in the following questions ? (You are not (?) in the following questions ? (Note : You are not
expected to calculate the exact value.) expected to calculate the exact value)
(Indian Bank Rural Marketing (Bank Of India Banking Officer
Officer Exam. 03.01.2010) Exam. 24.01.2010)
81. 15.5% of 323 - 20.8% of 198 = ? 91. 8399.999  375.002 × 14.996 = ?
(1) 12 (2) 5 (1) 565 (2) 225
(3) 15 (4) 3 (3) 335 (4) 625
(5) 90 (5) 455
82. 3058  27 × 3 = ? 92. ? = 37.0005
(1) 360 (2) 348
(1) 1150 (2) 1220
(3) 340 (4) 330
(3) 1570 (4) 1480
(5) 321
(5) 1370
83. (3.58)2 × (1.75)2 = ?
93. 14.998% of 619.999 = ?
(1) 25 (2) 40
(1) 95 (2) 80
(3) 30 (4) 35
(3) 115 (4) 105
(5) 50
(5) 75
84. 5138  36 = ? 94. 11.003 × 19.998 × 9.010 = ?
(1) 21 (2) 6 (1) 1710 (2) 1680
(3) 12 (4) 18 (3) 1800 (4) 1980
(5) 26 (5) 1750
85. 37.5 × 34.9  2.75 = ? 95. 1088.88 + 1800.08 + 1880.80 = ?
(1) 476 (2) 491 (1) 3950 (2) 4770
(3) 464 (4) 453 (3) 4620 (4) 5040
(5) 486 (5) 6810
i Directions (86-90) : What aparoximate Directions (96-100): What approximate
value should come in place the question mark (?) value should come in place of the question mark
in the following questions ? (You are not expected (?) in each of the following questions ? (You are
to calculate the exact value.) not expected to find out the exact value.)
(Indian Bank PO Exam. 17.10.2010) (Allahabad Bank PO Exam. 21.02.2010)
86. 18% of 609 + 27.5% of 450 = ? 96. 1548.45 + 3065.15  15.058 = ?
(1) 220 (2) 233 (1) 1700 (2) 1650
(3) 267 (4) 248 (3) 1840 (4) 1750
(5) 274 (5) 1950
87. 3942  64  3 = ?
2
(1) 29 (2) 32 97. 6 of 248.65 = ? of 2398.59
(3) 21 (4) 17 5
(5) 11 2 1
(1) (2)
3 6 1 5 4
88. 2 ×4 ×7 =?
10 7 2 1 1
(1) 68 (2) 72 (3) (4)
2 3
(3) 93 (4) 84
(5) 101 2
89. 12.564 × 22.009 × 17.932 = ? (5)
3
(1) 4901 (2) 4895 98. 39% of 695 = 10% of ?
(3) 4800 (4) 4959 (1) 2800 (2) 2400
(5) 4350 (3) 3200 (4) 31000
90. 16.978 + 27.007 + 36.984 - 12.969 - 9.003 (5) 2500
=?
(1) 72 (2) 42 99. 6 2 + 14.275 = ? of 196.35
(2) 60 (4) 51 1 1
(5) 65 (1) (2)
3 4

LEARN MATHS FROM S.K. RAJU (9811549822, 9811649822)


6
(1) 27 (2) 9
1 1
(3) (4) (3) 4 (4) 16
8 5 (5) 21
1 109. 133.008 × 2.97 - 111.87 + 74.13 = ?
(5) (1) 311 (2) 234
2
(3) 357 (4) 290
100. 1524.79 × 19.92 + 495.26 = ?
(5) 399
(1) 33,000 (2) 78,535
110. 32.1 × 2799  549 + 120 = ?
(3) 31,000 (4) 26,575
(1) 220 (2) 284
(5) 34,000
(3) 375 (4) 505
Directions (101-105): What approximate
(5) 190
value should come in place of the question mark
Directions (111-115): What approximate
(?) in the following questions ? (You are not
value will come in place of the question mark (?) in
expected to calculate the exact value).
the following questions ? (You are not expected to
(Corporation Bank PO
calculate the exact value).
Exam. 09.05.2010)
(Bank Of Baroda PO Exam. 30.05.2010)
101. 57% of 394 - 2.5% of 996 = ?
(1) 215 (2) 175 ?
111. 21.7% of 514.9 -43.44 =
(3) 200 (4) 180 5.5
(5) 205 (1) 320 (2) 335
102. 96.996 × 9.669 + 0.96 = ? (3) 475 (4) 375
(1) 860 (2) 870 (5) 420
(3) 1020 (4) 940 112. 1599 × 199  49 - 1398 + 3877 = ?
(5) 1100 (1) 9400 (2) 9000
3 1125 (3) 8700 (4) 8400
103. × ×7=? (5) 9200
5 1228
113. 4433.764 - 2211.993 - 1133.667 +
(1) 7 (2) 12
3377.442 = ?
(3) 9 (4) 4
(1) 4466 (2) 4377
(5) 15
(3) 3633 (4) 4144
104.  
339  25  30 = ?
114.
(5) 3344
(13.96)2 - (15.03)2 + (18.09)2 - 32.65 = ?
(1) 12 (2) 15 (1) 223 (2) 264
(3) 24 (4) 21 (3) 334 (4) 354
(5) 9 (5) 201
105. (638 + 9709 - 216)  26 = ?
(1) 275 (2) 365 7 7 1
115. 7 ×6 9
(3) 420 (4) 300 12 19 3
(5) 390 (1) 9 (2) 11
Directions (106-110) : What approximate (3) 2 (4) 5
value should come in place of the question mark (5) 13
(?) in the following questions ? (You are not Directions (116-120): What approximate
ex­pected to calculate the exact value.) value will come in place of question mark (?) in the
(Punjab & Sind Bank PO following questions ? (You are not expected to
Exam. 16.05.2010) calculate the exact value.)
106. 2
8938 × (5.96) = ? (Central Bank Of India PO
(1) 3050 (2) 3780 Exam. 25.07.2010)
(3) 2340 (4) 3400 116. 9228.789 - 5021.832 + 1496.989 = ?
(5) 3950 (1) 6500 (2) 6000
107. 4734.96 - 3454.03 - 1612.86 = ? - 1611.43 (3) 6300 (4) 5700
(1) 1280 (2) 2290 (5) 5100
(3) 1020 (4) 18150 117. 1002  49 × 99 - 1299 = ?
(5) 1040 (1) 700 (2) 600
(3) 900 (4) 250
323 971 56 (5) 400
108. × × =?
55 251 61 118. 29.8% of 260 + 60.01% of 510 - 103.57 = ?
LEARN MATHS FROM S.K. RAJU (9811549822, 9811649822)
7
(1) 450 (2) 320 128. 572  1755 × 12 = ?
(3) 210 (4) 280
(1) 150 (2) 170
(5) 350
(3) 155 (4) 165
119. (21.98)2 - (25.02)2 + (13.03)2 = ?
(5) 175
(1) 25 (2) 120
(3) 10 (4) 65 129. 925  3 350 = ?
(5) 140 (1) 4 (2) 8
120. 2498 × 626  99 = ? (3) 12 (4) 15
(5) 6
(1) 110 (2) 90
130. 12.36 × 4.26 + 13.38 = ?
(3) 200 (4) 160
(1) 72 (2) 66
(5) 125
(3) 58 (4) 52
Directions (121- 125) : What approximate
(5) 80
value will come in place of the question mark (?) in
Directions (131-135) : What approximate
the following questions ? (You are not expected to
value will come in place of the question mark (?) in
calculate the exact value.)
the following questions ? (You are not expectec to
(Syndicate Bank PO Exam. 29.08.2010)
calculate the exact value.)
150 199 16 (Bank Of India PO Exam. 31.10.2010;
121. ×  = ? 131. 465.84 + 764.86 - 211.99 = ?
17 13 91
(1) 650 (2) 700 (1) 1100 (2) 1080
(3) 770 (4) 820 (3) 1000 (4) 1020
(5) 850 (5) 1060
122. 151.011 - 419.999 + 649.991 = ? 132. 149.9% of 149.9 + 149.9 = ?
(1) 380 (2) 420 (1) 375 (2) 400
(3) 350 (4) 410 (3) 350 (4) 425
(5) 360 (5) 450
123. 1299  19.99 × 25.01 + 400.01 = ? 133. 3001 × 749  1001 - 1399 = ?
(1) 2025 (2) 2300 (1) 650 (2) 700
(3) 1925 (4) 2200 (3) 950 (4) 850
(5) 1700 (5) 1000
124. 30.06% of 499 + 39.99% of 799 134. 2642 - 1156 + 459 = ?
(1) 420 (2) 380 (1) 50 (2) 90
(3) 440 (4) 470 (3) 40 (4) 20
(5) 510 (5) 30
125. (14.99)2 - (7.01)2 + (4.99)3 = ?
(1) 250 (2) 200 901 91 51
135. ×
29 301 599
 =?
(3) 150 (4) 300
(5) 350 (1) 140 (2) 120
Directions (126-130): What approximate (3) 60 (4) 80
value will come in place of the question mark (?) in (5) 110
the following questions ? (You are not expected to Directions (136-140): What approximate
calculate the exact value.) value should come in place of the question mark
(Punjab National Bank Specialist (?) in the following questions? (Note : you are not
Officer Exam, 24.10.2010) expected to calculate the exact value.)
3.5  1.35  4.5 (United Bank Of India PO
126. =? Exam. 14.11.2010)
0.5
136. 18.505% of 550.010 = ?
(1) 35 (2) 20 (1) 135 (2) 85
(3) 40 (4) 50 (3) 100 (4) 120
(5) 55 (5) 90
127. (128.4 + 11.101 + 35.025)  ? = 12 137. 969.69 + 996.96 + 966.66 = ?
(1) 8 (2) 10 (1) 2560 (2) 2870
(3) 18 (4) 14 (3) 2930 (4) 2390
(5) 20 (5) 2900

LEARN MATHS FROM S.K. RAJU (9811549822, 9811649822)


8
138. 148. (31.33)2 + (3.96)3 - (12.02)2 = ?
1599 = ?
(1) 800 (2) 900
(1) 40 (2) 45
(3) 950 (4) 980
(3) 35 (4) 30
(5) 1000
(5) 50
139. 24.996 × 13.005 × 17.080 = ? 149. 3178 × 1330  360 = ?
(1) 6225 (2) 5525 (1) 130 (2) 110
(3) 5405 (4) 5875 (3) 140 (4) 160
(5) 6025 (5) 90
140. 8599.999  420.002 × 14.996 = ? 150. 39% of 405 + 62% of 610 - 183.57 = ?
(1) 250 (2) 325 (1) 450 (2) 300
(3) 275 (4) 300 (3) 230 (4) 280
(5) 350 (5) 350
Directions (141-145): What approximate Directions (151-155): What approximate
value should come in place the question mark (?) value should come in place of the question mark
in the following questions? (You are not expected (?) in the following questions? (Note : you are not
to calculate the exact value) expected to calculate the exact value.)
(PNB Management Trainee (Oriental Bank Of Commerce
Exam. 28.11.2010) PO Exam. 26.12.2010 (1st Sitting)
141. 31.85  3.90 × 15 = ? 151. 20.06% of 599 + 10.01% of 901 = ?
(1) 120 (2) 90 (1) 150 (2) 210
(3) 80 (4) 140 (3) 250 (4) 280
(5) 160 (5) 300
142. 4.99 × 12.865 + 599 = ?
249 299 14
(1) 650 (2) 655 152. ×  =?
1(3) 665 (4) 670 15 19 99
(5) 675 (1) 1850 (2) 1700
143. 21 + 63  17 = ? (3) 1750 (4) 1900
(1) 35 (2) 40 (5) 2000
(3) 10 (4) 25 153. (11.99)2 - (8.01)2 + (5.99)3 = ?
(5) 15 (1) 250 (2) 450
144. 1562  24% of 356 = ? (3) 300 (4) 400
(1) 24 (2) 18 (5) 350
(3) 12 (4) 28 154. 1201  14.99 × 19.91 + 400.01 = ?
(5) 8 (1) 1700 (2) 1850
(3) 1800 (4) 1950
145. 5986  364 × 220 = ?
(5) 2000
(1) 250 (2) 245 155. 251.01 - 429.99 + 549.99 = ?
(3) 230 (4) 235 (1) 370 (2) 420
(5) 255 (3) 340 (4) 410
Directions (146 -150): What approximate (5) 320
value will come in place of Ihe question mark (?) in Directions (156-160): What approximate
the following qestions ? (You are not expected to value should come in place of the question mark
calculate the exact value.) (?) in the following questions ? (Note : You are not
(Bank Of Maharashtra expected to calculate the exact value.)
Exam. 19.12.2010) (Indian Bank PO Exam. 02.01.2011)
146. 5682  63 × 36 = ? × 19 156. 12959.998  18.010 = ?
(1) 170 (2) 190 (1) 840 (2) 990
(3) 210 (4) 240 (3) 570 (4) 680
(5) 140 (5) 720
340 43 113 157. 40.005% of 439.998 + ?% of 655.011 =
147.  × =? 228.5
33 510 93
(1) 8 (2) 17
(1) 150 (2) 120
(3) 12 (4) 20
(3) 210 (4) 240
(5) 5
(5) 170
158. 6894.986 + 5025.005 + 600.020 = ?

LEARN MATHS FROM S.K. RAJU (9811549822, 9811649822)


9
(1) 12170 (2) 13540 (1) 520 (2) 360
(3) 12950 (4) 11560 (3) 460 (4) 500
(5) 12520 (5) 420
159. 31.999 × 12.001 × 17.5001 = ? 169. 441.01 - 232.99 + 1649.99 = ? + 1225.92
(1) 6600 (2) 6720 (1) 600 (2) 630
(3) 6480 (4) 6070 (3) 660 (4) 690
(5) 6270 (5) 720
160. (10.998)3 = ? 170. (21.5% of 999)1/3 + (42% of 601)1/2 = ?
(1) 1440 (2) 1730 (1) 18 (2) 22
(3) 1330 (4) 1640 (3) 26 (4) 30
(5) 1000 (5) 33
Directions (161-165): What approximate Directions (171-175): What approximate
value will come in place of the question mark (?) in value should come in place of the questions mark
the following questions ? ( You are not expected to (?) in the following questions ? (Note : You are not
calculate the exact value). expected to calculate the exact value.)
(Union Bank Of India (Punjab & Sind Bank PO
PO Exam. 09.01.2001) Exam. 23.01.2011)
161. (41.33)2 + (7.96)2 - (22.02)2 = ? 171. 5554.999  50.007 = ?
(1) 1280 (2) 1440 (1) 110 (2) 150
(3) 1580 (4) 1540 (3) 200 (4) 50
(5) 1380 (5) 125
162. 41% of 601 - 250.17 = ? - 77% of 910 172. (18.001)3 = ?
(1) 800 (2) 500 (1) 5832 (2) 5500
(3) 700 (4) 650 (3) 6000 (4) 6480
(5) 550 (5) 5240
163. 52001  61 × 29 = ? × 41 173. 23.001 × 18.999 × 7.998 = ?
(1) 700 (2) 600 (1) 4200 (2) 3000
(3) 500 (4) 550 (3) 3500 (4) 4000
(5) 680 (5) 2500
174. 9999  99  9 = ?
701 11 112
164.  × =? (1) 18 (2) 15
52 699 107 (3) 6 (4) 11
(1) 700 (2) 850 (5) 20
(3) 900 (4) 800 175. 22.005% of 449.999 = ?
(5) 650 (1) 85 (2) 100
165. 6378 × 3330  360 = ? (3) 125 (4) 75
(1) 200 (2) 250 (5) 150
(3) 300 (4) 225 Directions (176-180): What approximate
(5) 325 value will come in place of the question mark (?) in
Directions (166-170): What aproximate the following questions? (You are not expected to
value will come in place of the question mark (?) in calculate the exact value.)
the following questions? (You are not expected to (UCO Bank PO Exam. 30.01.2011)
calculate the exact value) 176. 73.99 % of 1299 + 9.98% of 1899 = ?
(Corporation Bank PO (1) 1250 (2) 1230
Exam. 16.01.2011) (3) 1150 (4) 1180
166. 499.99 + 1999  39.99 × 50.01 = ? (5) 1200
(1) 3200 (2) 2700 177. 5891  14.99 + 589.01 - 111.99 = ?
(3) 3000 (4) 2500 (1) 870 (2) 920
(5) 2400 (3) 840 (4) 810
167. [(7.99)2 - (13.001)2 + (4.01)3]2 = ? (5) 770
(1) -1800 (2) 1450 178. (9.979)3 - (23.99)2 + (1.99)5 = ?
(3) -1660 (4) 1660 (1) 350 (2) 490
(5) -1450 (3) 390 (4) 420
(5) 450
601 399 29
168. ×  =?
49 81 201
LEARN MATHS FROM S.K. RAJU (9811549822, 9811649822)
10
2 (1) 760 (2) 800
 18  455 61 (3) 690 (4) 870
179.   × 19  799 = ?
 4  (5) 780
(1) 6320 (2) 6350 189. 6999  70.005 × 94.998 = ? × 19.999
(3) 6400 (4) 6430 (1) 475 (2) 420
(5) 6490 (3) 320 (4) 540
180. 2439.97 - 1234.01 + 401.99 = ? + 989.99 (5) 525
(1) 620 (2) 650 190. (49.99)2 - (8.9)2 - (15.9)2 = ?
(3) 680 (4) 700 (1) 2165 (2) 2000
(5) 600 (3) 1965 (4) 1920
Directions (181-185): What approximate (5) 1885
value will come in place of the question mark (?) in Directions (191-195): What approximate
the following questions ? (You are not expected to value will come in place of the question mark (?) in
calculate the exact value.) the following questions? (You are not expected to
(Bank Of Baroda PO Exam. 13.03.2011) calculate the exact value.)
2 (Indian Overseas Bank PO
 24  399 41 Exam. 22.05.2011)
181.   × 39  899 = ?
 9  191. 7441  34 × 12 = ? × 9 + 110
(1) 1600 (2) 1650 (1) 420 (2) 280
(3) 1700 (4) 1550 (3) 590 (4) 350
(5) 1750 (5) 220
182. 67.99% of 1401 - 13.99% of 1299 = ? 989 65 515
(1) 700 (2) 720 192.  × =?
34 869 207
(3) 770 (4) 800
(5) 740 (1) 840 (2) 920
183. 5466.97 - 3245.01 + 1122.99 = ? + 2309.99 (3) 970 (4) 780
(1) 1130 (2) 1000 (5) 1000
(3) 1100 (4) 1030 193. (32.13)2 + (23.96)2 - (17.11)2 = ?
(5) 1060 (1) 1270 (2) 1420
184. 5998  9.98 + 670.99 - 139.99 = ? (3) 1450 (4) 1360
(1) 1080 (2) 1280 (5) 1310
(3) 1180 (4) 1130 194. 5456 × 2120  460 = ?
(5) 1230 (1) 120 (2) 140
185. - (4.99)3 + (29.98)2 - (3.01)4 = ? (3) 160 (4) 180
(1) 550 (2) 590 (5) 200
(3) 620 (4) 650 195. 67% of 801 - 231.17 = ? - 23% of 789
(5) 690 (1) 490 (2) 440
Directions (186-170): What approximate (3) 540 (4) 520
value will come in place of the question mark (?) in (5) 590
the following questions ? (You are not expected to Directions (196-200): What approximate
calculate the exact value.) value should come in place of the question mark
(Allahabad Bank PO Exam. 17.04.2011) (?) in the following questions ?
186. 3100 × 567  250 = ?  8 (IBPS Bank PO/MT CWE 18.09.2011)
(1) 620 (2) 670 (Note : You are not expected to calculate
(3) 770 (4) 750 the exact value.)
(5) 700 196. 39.897% of 4331 + 58.779% of 5003 = ?
187. 89.988% of 699.9 + 50.002% of 999.99 - (1) 4200 (2) 4600
170.015 = ? (3) 4700 (4) 4800
(1) 990 (2) 900 (5) 5200
(3) 920 (4) 960 197. 43931.03  2111.02 × 401.04 = ?
(5) 860 (1) 8900 (2) 6600
(3) 6400 (4) 8000
340 29.997 179.909 (5) 8300
188.
20.002
 510
×
59.919
=?
198. 3178 × 1330  360 = ?

LEARN MATHS FROM S.K. RAJU (9811549822, 9811649822)


11
(1) 130 (2) 110 (3) 2000 (4) 1800
(3) 140 (4) 160 (5) 2100
(5) 90 208. 55.003 × 54.998 + 5.001 = ?
(1) 3500 (2) 3630
199. 3
4663  349 = ?  21.003
(3) 2540 (4) 3030
(1) 6700 (2) 8640
(5) 2750
(3) 6680 (4) 9520
209. 50.001% of 99.99  49.999 = ?
(5) 7680
(1) 1 (2) 0.1
200. 5682  63 × 36 = ? × 19
(3) 0.01 (4) 0.02
(1) 170 (2) 190
(5) None of these
(3) 210 (4) 240
210. 999.0001 + 899.999 - 349.88 = ?
(5) 140
(1) 1549 (2) 1560
Directions (201-205): What approximate
(3) 1449 (4) 1460
value should come in place of the quesiton mark
(5) None of these
(?) in the following questions ?
211. (2.0001)3 × (1.999)-2  (3.999)-4 = ?
(IBPS Bank PO/MT CWE 17.06.2012)
(1) 32 (2) 16
(Note : You are not expected to calculate
(3) 64 (4) 256
the exact value.)
(5) 512
201. 8787  343 × 50 = ? Directions (212-216): What approximate
(1) 250 (2) 140 value should come in place of the question mark
(3) 180 (4) 100 (?) in the following questions ?
(5) 280 (NOTE : You are not expected to calculate
the exact value)
202. 3
54821 × (303  8) = (?)2
(IBPS RRBs Office Assistant
(1) 48 (2) 38
CWE Exam. 09.09.2012)
(3) 28 (4) 18
212. (10.97)2 + (4.13)3 × 3.79 = ?
(5) 58
(1) 428 (2) 376
5 7 (3) 197 (4) 204
203. of 4011.33 + of 3411.22 = ? (5) 302
8 10
(1) 4810 (2) 4980 213. 12.13% of 935.81 + 1498% of 25.85 = ?
(3) 4890 (4) 4930 (1) 500 (2) 550
(5) 4850 (3) 478 (4) 341
204. 23% of 6783 + 57% of 8431 = ? (5) 596
(1) 6460 (2) 6420 214. 3
65 × 23.93 - 31.04 = ?
(3) 6320 (4) 6630 (1) 98 (2) 65
(5) 6360 (3) 102 (4) 35
205. 335.01 × 244.99  55 = ? (5) 79
(1) 1490 (2) 1550 215. 1624.12 × 3.891 = ?
(3) 1420 (4) 1590 (1) 6100 (2) 6900
(5) 1400 (3) 6000 (4) 6400
206. 24% of 4568  8% of 246 is approximately (5) 6500
equal to 216. 3018.19  2.87 - 841.02 = ?
(1) 32 (2) 43 (1) 365 (2) 90
(3) 89 (4) 78 (3) 387 (4) 1000
(4) 55 (5) 200
(IDBI Bank Officer Exam. 16.09.2012) Directions (217-226) : What approximate
Directions (207-211): What approximate value will come in place of the question mark (?) in
value should come in the following questions at the following qestions ? (You are not required to
the questions places. find the exact value).
(IBPS Specialist Officer (Indian Overseas Bank PO
CWE 17.03.2013) Online Exam. 01.09.2013)
(You are not required to calculate the exact 217. 2371  6 + (43 × 4.35) = ?
value) (1) 582 (2) 590
207. (13.001)3 = ? (3) 600 (4) 570
(1) 1900 (2) 2200 (5) 595
LEARN MATHS FROM S.K. RAJU (9811549822, 9811649822)
12
218. following questions ? (You are not expected to
3
3380 + 1300 = ?
calculate the exact value)
(1) 56 (2) 51
(IBPS Bank PO/MT
(3) 53 (4) 54
CWE-III 26.10.2013)
(5) 55
227. 21 + 3.9 × 2.9 + 8.99 = ?
219. (4.989)2 + (21.012)3 + 1090 = ? (1) 42 (2) 46
(1) 9219 (2) 9391 (3) 44 (4) 34
(3) 9319 (4) 9129 (5) 36
(5) None of these 228. 22.9889 + 0.002  ? = 23
(1) 23 (2) 1
13
220. 7020  2.99 × =? (3) 232 (4) 24
29 (5) None of these
(1) 1040 (2) 1100
229. 1000000.000001 = ?
(3) 1060 (4) 1050
(5) None of these (1) 1000 (2) 100
221. 24.99% of 5001 - 65.01% of 2999 = ? (3) 1000.001 (4) 10000
(1) 840 (2) 500 (5) 999
(3) 700 (4) -500 230. 134% of 3894 + 38.94 of 134 = ?
(5) -700 (1) 11452 (2) 10000
1 2
(3)10452 (4)1100
 81 -  64   3 = ?

222. 2 (5) None of these
231. 103 × 1003 + 999999999 = 10? + 10?
3 1 (1) 6 (2) 9
(1) (2)
19 16 (3) 7 (4) 10
(5) 12
7 1 Directions (232-236): What approximate
(3) (4)
144 9 value will come in place of the question mark (?) in
(5) None of these the following questions ? (You are not required to
223. 331.8  23.7 + (-21)2 - 94 = (?)2 calculate the exact value).
(1) 15 (2) 16 (Corporation Bank Specialist Officer
(3) 18 (4) 19 (Marketing) Exam 22.12.2014)
(5) 17
3 5
224. 34% of 576 + 18% of 842 = ?% of 400 + 232. 4× × 952 - 128 = ?
83.4 13 7
(1) 75 (2) 72 (1) 823 (2) 840
(3) 62 (4) 65 (3) 835 (4) 839
(5) 66 (5) 845
3
29241 2 233. 105.01% of 8451 - % of 5006 + 9.999 = ?
225. ×5 =? 7
361 9
(1) 8879 (2) 8860
(1) 47 (2) 49 (3) 8850 (4) 8760
(3) 46 (4) 45 (5) None of these
(5) 61 234. 103 × 1003 + 999999999 = 10? + 10?
1 2 3 (1) 6, 9 (2) 9, 9
226. 3 + 6 + ? = 13 (3) 6, 12 (4) 16, 9
4 7 28
(5) 6, 18
2 4 235. 21 + 3.7 × 2.9 = ?
(1) 3 (2) 3
7 7 (1) 74 (2) 70
(3) 27 (4) 32
3 5
(3) 3 (4) 3 (5) 44
7 7 236. 22.9782 + 9.002 - ? = 23.001
6 (1) 9 (2) 8
(5) 3 (3) 6 (4) 11
7
(5) 12
Directions (227-231) : What value will
come in place of the question mark (?) in the
LEARN MATHS FROM S.K. RAJU (9811549822, 9811649822)
13
SBI PO EXAMS (1) 7395 (2) 7490
1. The radius of the cylinder is half of its height (3) 7510 (4) 7375
and area of the inner part is 616sq. metres. (5) 7415
Approximately how many litres of milk can 8. 127.001 × 7.998 + 6.05 × 4.001 = ?
it contains ? (1) 1440 (2) 1400
(1) 1.4 (2) 1.5 (3) 1000 (4) 1040
(3) 1.9 (4) 1.7 (5) 1140
(5) 2.2 9. 1010  36 + 187 × 20.05 = ?
(SBI Associate Banks PO (1) 3650 (2) 3770
Exam. 16.07.2000) (3) 3825 (4) 3800
2. In the following equation what appro- (5) 3700
ximate value will come in place of the 10. What approximate value will come in place
question mark (?) ? of the question mark (?) in the following
85.147 + 34.912 × 6.2 + ? = 802.293 equation ?
(1) 400 (2) 450 1
(3) 550 (4) 600 33 % of 768.9 + 25% of 161.2 - 58.12 = ?
3
(5) 500
(SBI Banks PO Exam. 20.08.2000) (1) 230 (2) 225
3. In the foll ow in g equatio n wh at (3) 235 (4) 220
approximate value will come in place of (5) 240
the question mark (?)? (SBI PO Exam. 26.11.2006)
248.251  12.62 × 20.52 = ? D irection s (1 1 -1 5 ) : Fin d ou t th e
(1) 400 (2) 450 approximate value which should reolace the
(3) 600 (4) 350 question mark (?) in the following questions. (You
(5) 375 are not expected to find out the exact value)
(SBI Banks PO Exam. 20.08.2000) (SBI Associate Banks PO
4. In the foll ow in g equatio n wh at Exam. 07.01.2007)
approximate value will come in place of 11. 953.7  950.9989 = 95?
the question mark (?) (1) 1.9 (2) 3
158.25 × 4.6 + 21% of 847 + ? = 950.93 (3) 2.99 (4) 3.6
(1) 35 (2) 40 (5) 2.7
(3) 25 (4) 50 3.001
(5) 45 12. 10000 + 4.987 of 1891.992 = ?
(SBI Banks PO Exam. 20.08.2000) (1) 2500 (2) 1230
5. What approximate value should come in (3) 1640 (4) 1525
place of the question mark (?) in the (5)2130
following equation ? 13. 0.0004  0.0001 × 36.000009 = ?
39.05 × 14.95 - 27.99 × 10.12 = (36 + ?) × 5 (1) 0.10 (2) 1.45
(1) 22 (2) 29 (3) 145 (4) 14.5
(3) 34 (4) 32 (5) 1450
(5) 25 14. 137% of 12345 = ?
(SBI Banks PO Exam. 11.02.2001) (1) 17000 (2) 15000
6. What approximate value should come in (3) 1500 (4) 14300
place of ? in the following equation ? (5) 900
9876  24.96 + 215.005 - ? = 309.99 15. 3739 + 164 × 27 = ?
(1) 395 (2) 295 (1) 105400 (2) 4000
(3) 300 (4)315 (3) 8200 (4) 690
(5) 310 (5) 6300
(SBI Associate Banks PO 16 What approximate value should come in
Exam. 21.07.2002) place of the question mark(?) in the
Directions (7-9): What approximate value following question ?
will come in place of the question mark (?) in the 6523  544 × 1.2 = ?
following equations ? (1) 21 (2) 33
(SBI PO Exam. 09.01.2005) (3) 14 (4) 8
7. 125% of 4875 + 88.005 × 14.995 = ? (5) 28
LEARN MATHS FROM S.K. RAJU (9811549822, 9811649822)
14
(SBI PO Preliminary (Tire-I) 26. 25.05 × 123.95 + 388.999 × 15.001 = ?
Exam. 27.04.2008) (1) 900 (2) 8950
17. If an amount of Rs. 74,336 is equally divided (3) 8935 (4) 8975
amon gst 150 pe ople , ho w mu ch (5) 8995
approximate amount would each person 27. 561  35.05 × 19.99 = ?
get ? (1) 320 (2) 330
(1) Rs.522 (2) Rs.485 (3) 315 (4) 325
(3) Rs.496 (4) Rs.488 (5) 335
(5) Rs.510 28. (15.01)2 × ?
730 =
(SBI PO Preliminary (Tire-I) Exam.
(1) 6125 (2) 6225
27.04.2008)
(3) 6200 (4) 6075
Directions (18 - 23): What approximate
(5) 6250
value will come in place of the question mark (?) in
the following questions ?
(SBI PO Preliminary (Tire-I) RBI GRADE-B OFFICER EXAMS
Exam. 27.07.2008) Directions (1-5): What approximate value
18. 2959.85  16.001 - 34.99 = ? will come in place of the question mark (?) in the
(1) 160 (2) 150 following questions ? (You are not expected to
(3) 140 (4) 180 calculate exact value.)
(5) 170 (RBI Grade-B Officer
19. (1702  68) × 136.05 = ? Exam. 11.10.2009)
(1) 3500 (2) 3550 1. (32.51)2 - (17.45)2 = ?
(3) 3450 (4) 3400 (1) 780 (2) 850
(5) 3525 (3) 680 (4) 820
20. 2950  12.25 + 160 = ? (5) 750
(1) 440 (2) 350 2. 88.25% of 450 = ?% of 530
(3) 380 (4) 360 (1) 70 (2) 68
(5) 400 (3) 75 (4) 80
21. 25.05% of 2845 + 14.95 × 2400 = ? (5) 65
2
(1) 36,700 (2) 36,500 3. 898 × (12.005) + ? = 5000
(3) 35,800 (4) 35,600 (1) 680 (2) 720
(5) 36,200 (3) 750 (4) 620
22. (186 × 270.99)  40 = ? (5) 630
(1) 1160 (2) 1200 4. 3745  24.05 × 17.98 = ?
(3) 1300 (4) 1180 (1) 2860 (2) 2800
(5) 1260 (3) 2760 (4) 2720
23. 424.99 × 23.95  8.05 = ? (5) 2840
(1) 1300 (2) 1225 5. 117.95 × 8.017 × 4.98 = ?
(3) 1325 (4) 1275 (1) 4670 (2) 4780
(5) 1375 (3) 4840 (4) 4720
Directions (24-28) : What approximate (5) 4800
value will come in place of the question mark (?) in
the following questions ? INSURANCE EXAMS
(SBI Associate Banks PO 1. What approximate value will come in place
Exam. 07.08.2011) of question mark (?) in the following
24. 32.05% of 259.99 = ? equation ?
(1) 92 (2) 88 21.0091 - 6.085 + 13.24 = (35 + ?) × 2
(3) 78 (4) 90 (1) 6.5 (2) 10.5
(5) 83 (3) 15.5 (4) 20.5
1 2 3 (5) 24.5
25. of of of 1715 = ?
8 3 5 (LIC Assistant Administrative
Officer (AAO) Exam. 24.04.2005)
(1) 80 (2) 85
2. What approximate value should come in
(3) 90 (4) 95
place of ? in the following equation ?
(5) 75
9876  24.96 + 215.005 - ? = 309.99
LEARN MATHS FROM S.K. RAJU (9811549822, 9811649822)
15
(1) 395 (2) 295 10. (9.99)3 + (30.01)2 - (17.01)2 = ?
(3) 300 (4) 315 (1) 1610 (2) 1630
(5) 310 (3) 1580 (4) 1540
(LIC Assistant Administrative (5) 1510
Officer (AAO) Exam. 2006) 11. 3444 × 2121  1111 × 10 = ?
Directions ( 3 -7) : What approximate
value should come in place of the question mark (1) 720 (2) 740
(?) in the following questions ? (Note : You are not (3) 810 (4) 840
expected to calculate the exact value) (5) 760
(New India Assurance 12. 79% of 801 - 259.99 = ? - 66% of 499
AO Exam. 25.10.2009) (1) 800 (2) 700
3. 8537.986 - 2416.005 -221.996 = ? (3) 500 (4) 650
(1) 6500 (2) 5900 (5) 550
(3) 4300 (4) 3900 Directions (13-17) : What approximate
(5) 5050 value will come in place of the question mark (?) in
4. 1019.999  60.007 = ? the following questions ? (you are not required to
find the exact value).
(1) 11 (2) 23
(LIC Assistant Administrative
(3) 17 (4) 27
Officer (AAO) Exam. 12.05.2013)
(5) 13
5. 111111  1111  11 = ? 25 16
(1) 1180 (2) 15 13. × × 91 = ?
9 53
(3) 1100 (4) 9 (1) 65 (2) 75
(5) 2 (3) 80 (4) 85
6. 3
5000 = ? (5) None of these
(1) 15 (2) 9 4 8
(3) 29 (4) 32 14. × 5671 - × 2524 = ?
9 15
(5) 17
7. 16.001 × 30.999 × 8.998 = ? (1) 1200 (2) 1120
(1) 4450 (2) 4800 (3) 1100 (4) 1175
(3) 4100 (4) 3900 (5) None of these
(5) 5000 15. 4568.6531 - 2431.3178 + 134.675 = ?
Directions (8-12) : What approximate (1) 2272 (2) 2372
value will come in place of the question mark (?) in (3) 2172 (4) 2200
the following questions ? (You are not expected to (5) None of these
calculate the exact value.) 16. 24.9% of 5679 + 44.9% of 4301 = ?
(United India Insurance AO (1) 3455 (2) 3355
Exam. 27.03.2011) (3) 3255 (4) 3555
8. 17001  81 × 19 = ? × 29 (5) None of these
(1) 100 (2) 110 17. (6.99)2 + (8.01)2 - 85 = ?
(3) 140 (4) 170 (1) 95 (2) 115
(5) 130 (3)110 (4) 104
901 21 101 (5) None of these
9.  × =?
51 1201 301
(1) 320 (2) 350
(3) 400 (4) 410
(5) 430

LEARN MATHS FROM S.K. RAJU (9811549822, 9811649822)


16
SHORT ANSWERS 105. (5) 106. (4)
NATIONALISED BANKS 107. (1) 108. (5)
& IBPS PO/MT/SO 109. (3) 110. (2)
1. (5) 2. (2) 111. (4) 112. (2)
3. (3) 4. (1) 113. (1) 114. (2)
5. (3) 6. (4) 115. (4) 116. (4)
7. (3) 8. (2) 117. (1) 118. (4)
9. (5) 10. (4) 119. (1) 120. (5)
11. (3) 12. (4) 121. (3) 122. (1)
13. (2) 14. (1) 123. (1) 124. (4)
15. (5) 16. (2) 125. (4) 126. (2)
17. (3) 18. (1) 127. (4) 128. (4)
19. (3) 20. (3) 129. (1) 130. (2)
21. (2) 22. (3) 131. (4) 132. (1)
23. (5) 24. (4) 133. (4) 134. (3)
25. (1) 26. (2) 135. (5) 136. (3)
27. (1) 28. (5) 137. (3) 138. (1)
29. (3) 30. (4) 139. (2) 140. (4)
31. (3) 32. (2) 141. (1) 142. (3)
33. (1) 34. (4) 143. (4) 144. (2)
35. (5) 36. (3) 145. (2) 146. (1)
37. (1) 38. (2) 147. (1) 148. (2)
39. (5) 40. (2) 149. (2) 150. (5)
41. (2) 42. (4) 151. (2) 152. (3)
43. (1) 44. (5) 153. (3) 154. (5)
45. (3) 46. (3) 155. (1) 156. (5)
47. (5) 48. (1) 157. (1) 158. (5)
49. (2) 50. (4) 159. (2) 160. (3)
51. (4) 52. (5) 161. (1) 162. (3)
53. (1) 54. (3) 163. (2) 164. (3)
55. (2) 56. (1) 165. (4) 166. (3)
57. (5) 58. (4) 167. (4) 168. (5)
59. (2) 60. (3) 169. (2) 170. (2)
61. (2) 62. (5) 171. (1) 172. (1)
63. (3) 64. (3) 173. (3) 174. (4)
65. (1) 66. (4) 175. (2) 176. (3)
67. (3) 68. (5) 177. (1) 178. (5)
69. (3) 70. (1) 179. (1) 180. (1)
71. (4) 72. (5) 181. (1) 182. (3)
73. (1) 74. (3) 183. (4) 184. (4)
75. (2) 76. (4) 185. (5) 186. (2)
77. (1) 78. (2) 187. (4) 188. (4)
79. (1) 80. (5) 189. (1) 190. (1)
81. (5) 82. (3) 191. (2) 192. (3)
83. (2) 84. (3) 193. (5) 194. (3)
85. (1) 86. (2) 195. (1) 196. (3)
87. (3) 88. (4) 197. (5) 198. (2)
89. (4) 90. (3) 199. (5) 200. (1)
91. (3) 92. (5) 201. (3) 202. (2)
93. (1) 94. (4) 203. (3) 204. [5)
95. (2) 96. (4) 205. (1) 206. (5)
97. (5) 98. (1) 207. (2) 208. (4)
99. (3) 100. (3) 209. (1) 210. (1)
101. (3) 102. (4) 211. (5) 212. (2)
103. (4) 104. (4) 213. (1) 214. (2)

LEARN MATHS FROM S.K. RAJU (9811549822, 9811649822)


17
215. (5) 216. (5) = 1235.2  1230
217. (1) 218. (2)
0.0004
219. (3) 220. (4) 3. (3) ?  × 36 = 4 × 36
221. (5) 222. (3) 0.0001
223. (4) 224. (5) = 144  145
225. (1) 226. (2) 137
227. (1) 228. (2) 4. (1) ? = 12345 ×
100
229. (1) 230. (3)
= 16912.65  17000
231. (2) 232. (1)
5. (3) ? = 3739 + 164 × 27
233. (2) 234. (2)
= 3739 + 4428
235. (4) 236. (1)
= 8167  8200
6. (4) Taking approximate integral values we
SBIPO EXAMS have,
1. (5) 2. (5) ?  448  28 × 5
3. (1) 4. (5)
5. (5) 6. (3) 448
= × 5 = 80
7. (5) 8. (4) 28
9. (2) 10. (5) 7. (3) (3.5)2 + 19.95 + ? = 275
11. (5) 12. (2)  12.25 × 19.95 + ? = 275
13. (3) 14. (1)
 ? = 275 - 235.81
15. (3) 16. (3)
= 39.18  40
17. (3) 18. (2)
8. (2) ? = 85% of 225 + 32.91 × 5.01
19. (4) 20. (5)
 85% of 225 + 33 × 5
21. (1) 22. (5)
23. (4) 24. (5) 85  225
25. (2) 26. (2)  + 33 × 5
100
27. (1) 28. (4) = 191.25 + 165
= 356.25  355
RBI GRADE-B OFFICER EXAMS 9. (5) ? = (15.96)2 + 75% of 285
1. (5) 2. (3)
75  285
3. (1) 4. (2)  (16) 2 +
5. (4) 100
= 256 + 213.75
INSURANCE EXAMS  469.75 = 470
1. (2) 2. (3) 10. (4) ? = 1679  14.95 × 5.02
3. (2) 4. (3)  1680  15 × 5
5. (4) 6. (5) 1680
7. (1) 8. (3)  × 5 = 560  565
15
9. (2) 10. (1)
11. (3) (?)2 = 63.9872 × 9449. 8780  243.0034
11. (3) 12. (2)
Taking approximate integral values,
13. (3) 14. (4)
(?) = 64 × 9450  240
15. (1) 16. (2)
17. (4) 64  9450
 240
= 2520  2500
EXPLANATIONS
 ? = 2500 = 50
NATIONALISED BANKS
& IBPS PO/MT/SO 12. (4) ? = 5237.897 - 6629.010 + 7153.999 -
1. (5)  95? = 953.7  950.9989 2205.102
 95? = 953.7-0.9989 = 952.7011  5238 - 6629 + 7154 - 2205
 (5238 + 7154) - (6629 + 2205)
 ?  2.7  12392 - 8834 = 3558
3 13. (2)? = 4985.0346  215.987 - 3768.112 
2. (2) ?  10000 + 5 × 1892 206.868
= 100 + 1135.2  4985  216 - 3768  207
= 23. 078 - 18.202
LEARN MATHS FROM S.K. RAJU (9811549822, 9811649822)
18
= 4.876  5  27 × 0.6 + 14.38 [0.56  0.6]
14. (1) ? 956240  977.8  979  16.2 + 14.38
 30.58  30
15. (5) ? = 459% of 849.947 + 266% of 6284.012
26. (2) ?  459 + 3 × 89
- 1486.002
[459.008  459, 88.862  89]
460  850 260  6280  459 + 267 = 726  725
 + - 1486 27. (1) ? = (621.52)2
100 100
= 3910 + 16328 - 1486 = 18752  622 × 622 = 386884  386300
This can be treated approximate to 19130. We have taken 622 > 621.52 here.
No te : H ere numbe rs h ave be en  Required answer = 386300.
approximated as multiples of 10. It makes 28. (5) 561204 × 58 = ? × 55555
simplification easy. The other point is the 561204  58
numbers in answer choices have larger gap. ?= = 585.90  586
55555
Hence, this approximation makes answer
correct.  444 
(3) ? =  531   972
100  
29.
623898  99 
16. (2) ? = = 1029.43  1030  2357.64  972 = 2.42  2.5
60000
30. (4) ? = (9321 + 5406 + 1001)  (498 + 929 +
4 3 6 5 660)
17. (3) ? = ×  
3 7 7 9 = 15728  2087 = 7.53  7.5
31. (3) ? =(11.49)4 = 17429.30  17430
4 3 7 9 18
= × × × =
5 7 6 5 25 2198  1347  403
32. (2) ? =
18. (1) (399.98)2 = ? 159  113  27
 ?  (400)2 = 160000 448
= = 23.58  24
19. (3) 624.9995 + (4.9989) 2 19
Taking approximate values,  825 
33. (1) ? =  330    507
1  100 
 625 + (5)2  ?  
5
2722.50
 25 + 25  ? × 5 = = 5.369  5
507
50
? = = 10 34. (4) ? = 888888 × 1.486
5
 943 × 1.5 = 1414.5
20. (3) 989.001 + 1.00982 × 76.792 = ?
 Nearest answer = 1400
 ?  989 + 1 × 77 35. (5) ? = 564.666 + 82.5091 × 44.581 - 34.111
= 989 + 77= 1066  1065  565 + 82.5 × 45 - 34
3 4 2 = 565 + 3712.5 - 34
21. (2) ? = × × × 3719  283 = 4243.5
7 9 5
 Approximate answer = 4210
22. (3) ? = 0.008 + 6.009  (0.7)2
 47 1412  36 
6.009 36. (3) ? = 1442     63
= 0.008 +
0.7  0.7  100 100 
= 0.008 + 12.26 = 12.27  12 = (677.74 - 508.32)  63
23. (5) ?  (92 × 7)  (3.8 × 5.5) 169.42
= 644  20.9 = 30.81  31 == = 2.689  3
63
[ 3 95657  92]
Hence, we can choose 31 as our answer. 37. (1) ? =  7921  2070.25 × 1
4
98  785
24. ?= = 0.94  0.9 1
285  285 = (89 - 45.5) ×
4
25. (1) ? = 749 × 0.56 + 14.38
43.5
= 10.875  11
=
4
LEARN MATHS FROM S.K. RAJU (9811549822, 9811649822)
19
38. (2) ? = (341789 + 265108)  (8936 - 3578) 56. (1) ? = 3 1500  11.4
= 606897  5358 = 113.27  113
Illustration:
725  29 315  60 11× 11 × 11
39. (5) = +? = 1331 ; 150 - 1331 = 169
100 100
 210.25 = 189 + ? 12 × 12 × 12
= 1728 ; 1728 - 1500 = 228
 ? = 210.25 - 189
= 21.25  21 8 15 22
57. (5) ? = × × = 25.142  25
40. (2) ? = 1595  25 × 36.5 5 7 3
1595 58. (4) ? = 18.999 × 12.005 × 25.998
= × 36.5 = 2328.7  2329  19 × 12 × 26 = 5928  5930
25
41. (2) 63251 × 82 = ? × 42105 11.5 18.3
59. (2) ? = 666 × × 888 ×
63251  82 100 100
? = = 123.182  123 = 12446.18  12446
42105
42. (4) ? = 84111  84100 = 290.01  290 2898
60. (3) ? = = 65.863  66
22  2
43. (1) ? = (54.78)2  (55)2 = 3025
 Approximate answer = 3000 61. (2) ? = 175 × 28 + 275 × 27.98
44. (5) ? = (7171 + 3854 + 1195)  (892 + 214 +  175 × 28 + 275 × 28
543) = 28 (175 + 275)
= 12220  1649 = 7.41  7 = 28 × 450 = 12600
62. (5) ?  325 × 16  4 + 37
 816  562 
45. (3) ? =   + 1449 325  16
 100  = + 37
4
= 4585.92 + 1449 = 6034.92  6035
= 1300 + 37 = 1337  1340
888888 63. (3) ? = 1164 × 128  8.008 + 969.007
46. (3) ? = =1262.625  1263
88  8 1164  128
47. (5)193.999 + 228.008 + ? + 422.005  8
+ 969
= 1168.01
= 18624 + 969 = 19593  19600
 194 + 228 + ? + 422  1168
 844 + ?  1168 64. (3) ? = 624.98 + 729.25
 ?  1168 - 844 = 324  625 + 729
48. (1) ? = 27.8 × 28.74 × 17.3 = 25 + 27 = 52
= 13822.2156  13822
700  69 400  33
12 90 53 65. (1) ?  +
49. ? = × × = 69.89  70 100 100
7 13 9
= 483 + 132 = 615
17 12 66. (4) ?  8000 + 72 × 50
50. (4) ?  222 × × 923 × = 8000 + 3600 = 11600
100 100
= 4180.08  4165 67. (3)?  (25)2 - (16)2
51. (4) ? = (4576 + 3286 + 5639)  (712 + 415 + = (25 + 16) (25 - 16) = 41 × 9 = 369
212) = 13501  1339 = 10.08  10 365
52. (5) ? = 675.456 + 12.492 × 55.671 68. (5) ? = 380 × 12.25 -
15
 675 + 12.5 × 56 = 4655 - 24.33 = 4630.67  4630
= 675 + 700  1375  1371
53. (1) ?  (447)2 = 199809  200000 180  25501 28999  50
69. (3) ?  +
4374562  64 100 100
54. (3) ? = = 35999.99  36000 = 45901.8 + 14499.5
7777
= 60401.3  60400
659  872  45900 + 14500  60400
55. (2) ? =
100
 543 = 10.58  11 70. (1) ?  172 × 15  25
172  15
= = 103.2  105
25
LEARN MATHS FROM S.K. RAJU (9811549822, 9811649822)
20
71. (4) ?  1580 × 24 = 37920
8400  15
 Approximate answer = 37900 = = 336  335
375
77 92. (5) ?  37 × 37 = 1369  1370
72. (5) ?  × 6 = 66
7
620  15
73. (1) ?  (16) 2- (9)2 93. (1) ?  = 93  95
= (16 + 9) (16 - 9) = 25 × 7 = 175 100
94. (4) ?  11 × 20 × 9 = 1980
170  400 95. (2) ? = 1088.88 + 1800.08 + 1880.80
74. (3) ?  = 680
100 = 4769.76  4770
75. (2) ?  225 × 784 =15 × 28 = 420 1
76. (4) ? = 23.999 × 9.004 × 16.997 96. (4) ?  1548 + 3065 ×
15
 24 × 9 × 17 = 3672  3700 = 1548 + 204.33 = 1752.33  1750
7 4 2 32
77. (1) ? = 5 ×8 ×9 97. (5)  250 ×  2400 × ?
9 5 3 5
= 6 × 9 × 9 = 486  490
78. (2) ? = 5940  28  6 1600 2
?  =
2400 3
5940
= = 35.35  35
28  6 695  39  10
98. (1) ? = = 2710.5  2800
100
850  15.5 650  24.8
79. (1) ? = + 99. (3) 6 × 1.414 + 14.275 = 196.35 × ?
100 100
 22.759 = 196.35 × ?
= 131.75 + 161.20
= 292.95  295 22.759 1
80. (5) 47 × 47 = 2209  ?= 
196.35 8
 2230  47 100. (3) ?  1525 × 20 + 495
= 30500 + 495 = 30995  31000
323 20.8
81. (5) ? = 15.5 × + 198 × 394  57 996  2.5
100 100 101. (3) ? = -
100 100
= 50.06 + 41.18 = 91.24  90
= 224.58 - 24.9
3058 = 199.68  200
82. (3) ? = × 3  340
27 102. (4) ?  97 × 10 + 1 = 971  940
83. (2) ? = (3.55)2 × (1.75)2 3 1125
= 12.81 × 3.06 = 39.23  40 103. (4) ? = × × 7 = 3.847  4
5 1228
72
84. (3) ?  = 12 339  25
6 104. (2) ? = = 15.34  15
30
37.5  34.9
85. (1) ? = = 475.90  476 638  9709  216
2.75 105. (5) ? = = 389.65  390
26
18  609 27.5  450 106. (4) ?  95 × 6 × 6 = 3420  3400
86. (2) ? = +
100 100 107. (1) 4735 - 3454 - 1613
= 109.62 + 123.75 = 233.37  233 = ? - 1611
3942  ? = -332 + 1611 = 1279  1280
87. (3) ? = = 20.53  21 6
64  3 323 971
108. (5) ? = × × = 20.856  21
53 251 61
23 34 15
88. (4) ? = × × = 83.785  84 109. (3) ?  133 × 3 - 112 + 74 = 361  357
10 7 2
110. (2) ?  32 × 2800  550 + 120
89. (4) ?  12.6 × 22 × 18 = 4989.6  4959
= 282.9  284
90. (3) ?  17 + 27 + 37 - 13 - 9  59  60
91. (3) ?  8400  375 × 15 515  22 ?
[  8399.999  8400, 14.996  15] 111. (4) - 43 =
100 5.5
LEARN MATHS FROM S.K. RAJU (9811549822, 9811649822)
21
? 150  150
 113.3 - 43 = 132. (1) ?  + 150
5.5 100
 ? = 70.3 × 5.5 = 386.65  375 = 225 + 150 = 375
1600  200 3000  750
112. (2) ?  - 1400 + 3900 133. (4) ?  - 1400
50 1000
= 6400 - 1400 + 3900 = 8900  9000 = 2250 - 1400 = 850
113. (1) ?  4434 - 2212 - 1134 + 3377 134. (3) ?  51 - 34 + 21= 38  40
= 4465  4466
900 90 600
114. (2) ?  (14)2 - (15)2 + (18)2 - 33 135. (5) ?  × × = 108  110
= 196 - 225 + 324 - 33 = 262  264 30 300 50
115. (4) ? = 8 × 6  9 = 5.33  5 550  19 10450
116. (4) ?  9230 - 5022 + 1500 = 5708  5700 136. (3) ?  = = 104.50  100
100 100
Required answer = 5700
137. (3) ?  970 + 997 + 967 = 2934  2930
1000
117. (1) ?  × 100 -1300 138. (1) ? = 1599  1600 = 40
50
139. (2) ?  25 × 13 × 17 = 5525
= 2000 - 1300 = 700
8600
260  30 510  60 140. (4) ?  × 15 = 307  300
118. (4) ?  + - 100 420
100 100
141. (1) ?  32  4 × 15
= 78 + 306 - 100 = 284  280
119. (1) ?  (22)2 - (25)2 + (13)2 32
= × 15 = 120
= 484 - 625 + 169 = 28  25 4
(5) ?  142. (3) ?  5 × 13 + 600 = 665
120. 2500 × 625  100
63
1 143. (4) ? = 21 + = 24.70  25
= 50 × 25 × = 125 17
10
356  24
150 199 91 144. (2) ? = 1562 
121. (3) ? = × × 100
17 13 16
= 1562  85.44 = 18.28  18
= 768.19  770
122. (1) ?  151 - 420 + 650 = 382  380 145. (2) ? = 5986  364 × 220

1300 5986
123. (1) ?  × 25 + 400  × 15 = 246.67  245
20 364
= 1625 + 400 = 2025 5282
146. (1) × 36 = ? × 19
30  500 40  800 63
124. (4) ?  +
100 100
5682  36
= 150 + 320 = 470 ?= = 170.88  170
63  19
125. (4) ?  152 - 72 + 53
= 225 - 49 + 125 = 301  300 340 510 113
147. (1) ? = × ×
3.5  1.35  4.5 33 43 93
126. (3) ? = = 42.525  40 = 148.47  150
0.5
148. (2) ? = (31)2 + (4)3 - (12)2
175 175 = 961 + 64 - 144 = 881  900
127. (4) = 12  ? = = 14.58  14
? 12
3178  1330
128. (4) ?  572  42 × 12 = 163.42  165 149. (2) ? =
360
129. (1) ? = 925 + 3 350
 30  7 = 4.285  4 56  36
130. (2) ?  53 + 13 = 66  = 106.105 = 110
19
131. (4) ?  466 + 765 - 212 = 1019  1020

LEARN MATHS FROM S.K. RAJU (9811549822, 9811649822)


22
405  39 610  62 701 699 112
150. (5) ? = + - 183.57 164. (3) ? = × ×
100 100 52 11 107
 158 + 378 - 184 = 352  350 700 700 110
600  20 900  10  × × = 915.88  900
50 11 100
151. (2) ?  +
100 100
= 120 + 90 = 210 6378  3330
165. (4) ? =
360
249 299 99
152. (3) ? = × ×
15 19 14 80  58
 19
= 244.21  250
250 300 98
?  × × = 1750 166. (3) ?  500 + 2000  40 × 50
15 20 14
153. (3) ?  (12)2 - (8)2 + (6)3 2000
= 500 + × 50
= 144 - 64 + 216 = 296  300 40
154. (5) ?  1200  15 × 20 + 400 = 500 + 2500 = 3000
1200 167. (4) ?  [82 - (13)2 + 43]2
= × 20 + 400 = (64 - 169 + 64]2
15
= (-41)2 = 1681  1680
=1600 + 400 = 2000
155. (1) ?  251 - 430 + 550 = 371  370 600 400 30
168. (5) ?  × 
156. (5) ? = 12959.998  18.010 50 80 200
12960 600 400 200
 = 720 = × × = 400  420
18 50 80 30
440  40 655  ? 169. (2) 441 - 233 + 1650  ? + 1226
157. (1) +  228  1858  ? + 1226
100 100
 ? = 1858 - 1226 = 632  630
655  ?
 176 +  228 1 1
100  1000  21.5  3  600  43  3
170. (2) ?    + 
655  ?  100   100 
  228 - 176 = 52 = (215)1/3 +(258)1/2  6 + 16 = 22
100
5555
52  100 171. (1) ?  = 111.1  110
?= = 7.93  8 50
655
172. (1) ?  (18)3 = 5832
158. (5) ?  6895 + 5025 + 600 = 12520
173. (3) ?  23 × 19 × 8 = 3496  3500
159. (2) ?  32 × 12 × 17.5 = 6720
160.   (3) ?  (11)3 = 11 × 11 × 11 = 1331  1330 9999
161. (1) ?  (41)2 + (8)2 - (22)2 174. (4) ? = = 11.22  11
99  9
= 1681 + 64 - 484 = 1261  1280
450  22
600  40 900  80 175. (2) ?  = 99  100
162. (3) - 250  ? - 100
100 100
 240 - 250  ? - 720 1300  74 1900  10
176. (3) ?  +
 ? = 720 + 240 - 250 = 710  700 100 100
163. (2) 52000  60 × 30  ? × 40 = 962 + 190 = 1152  1150
177. (1) ?  5900  15 + 589 - 112
52000 = 393 + 589 - 112 = 870
 × 30  ? × 40
60 178. (5) ?  (10)3 - (24)2 + (2)5
 26000  ? × 40 = 1000 - 576 + 32 = 456  450
2
26000  18  455 799
? = = 650  600 179. (1) ? =   × ×
40  4  19 61
 (4.5) × 24 × 13 = 6318  6320
2

LEARN MATHS FROM S.K. RAJU (9811549822, 9811649822)


23
180. (1) 2440 - 1234 + 402  ? + 990 193. (5) ?  (32)2 + (24)2 - (17)2
 1608  ? + 990 =1024 + 576 - 289 = 1311  1310
 ? = 1608 - 990 = 618  620 194. (3) ? = 5456 × 2120  460
8
2
400 900  74 × 46  21 = 162.09  160
181. (1) ?    × × = 1600
3 40 40 800  67 800  23
195. (1) - 231  ? -
100 100
1400  68 1300  14
182. (3) ?  -  536 - 231  ? - 184
100 100
 305  ? - 184
= 952 - 182 = 770
 ?  305 + 184 = 489  490
183. (4) 5466.97 - 3245.01 + 1122.99
= ? + 2309.99 40 59
196. (3) ?  4330 × + 5000 ×
 3344.95 = ? + 2309.99 100 100
 ? = 3344.95 - 2309.99 = 1732 + 2950 = 4682  4700
= 1034.96  1030 197. (5) ?  44000  2100 × 400
184. (4) ?  6000  10 + 671 - 140
= 600 + 671 - 140 = 1131  1130 44000
185. (5) ?  - (5)3 + (30)2 - (3)4 = × 400 = 8380.95  8300
2100
= -125 + 900 - 81 = 694  690
3178  1330
186. (2) 3100 × 567  250 198. (2) ? =
360
 ? + 8  56 × 24  16 = ?  8
56  24 ? 56  36
   = 106.105  110
16 8 19

?
199. (5) 4663 + 349 = ?  21.003
3

 84   17 + 349  ?  21
8
 ? = 8 × 84 = 672  670 ?
 366 
21
700  90 1000  50
187. (4) ?  + - 170  ?  366 × 21 = 7686  7680
100 100
= 630 + 500 - 170 = 960 5682
200. (1) × 36 = ? × 19
340 30 180 63
188. (4) ?   ×
20 510 60 5682  36
? = = 171.187  170
340 510 180 63  19
= × × = 867  870
20 30 60 201. (3) 8800  340 × 50
189. (1) 7000  70 × 95  ? × 20 [  8787  8800; 343  340 ]
7000  95   25.9 × 7 = 181.3  180
?
?= = 475
70  20 202. (2) ?2 = 3 54821 × (303  8)
19O. (1) ?  (50)2 - (9)2 - (16)2  3 55000 × 38
= 2500 - 81 - 256
= 2163  2165  38 × 38
 ? = 38
7441
191. (2) × 12 = ? × 9 + 110 4011.33  5 3411.22  7
34 203. (3) ? = +
 2626  ? × 9 + 110 8 10
 ? × 9  2516 4000  5 3400  7
 8
+
10
2516
 ? = 279.55  280 = 2500 + 2380 = 4880  4890
9
989 869 515 6783  23 8431  57
192. (3) ? = × × = 967.52  970 204. (5) ? = +
34 65 207 100 100

LEARN MATHS FROM S.K. RAJU (9811549822, 9811649822)


24
6800  23 8400  57 ?= 361 =19
= +
100 100 576  34 842  18 400  ?
= 1564 + 4788 = 6352  6360 224. (5) + = + 83.4
100 100 100
205. (1) ?  335 × 245  55 = 1492.27  1490
 195.84 + 151.56 = 4 × ? + 83.4
4568  24 100  347.4 = 4 × ? + 83.4
206. (5) × = 55.70  55
100 246  8  4 × ? = 347.4 - 83.4 = 264
207. (2) ? = (13.001)3  (13)3 = 2197  2200
64
208. (4) ?  55 × 55 + 5 ?= = 66
= 3025 + 5 = 3030 4

100  50 29241 47
209. (1) ?   50 = 1 225. (1) ? = ×
100 361 9
210. (1) ?  999 + 900 - 350 = 1549
171 47
211. (5) ?  23 × (2)-2  (4)-4 = × 9 = 47
19
2
=
4 = 2 × 2 8 = 29 = 512
2

13 44 3
212. (2) ? = (10.97)2 + (4.13)3 × 3.79 226. (2)  + + ? = 13 +
4 7 28
 (11) 2 + (4)3 × 4
= 121 + 256 = 377  376 91  176 3
 + ? = 13 +
936  12 26  1500 28 28
213. (1)?  +
100 100 3
= 112.32 + 390 = 502.32  500  + ? = 13 +
28
214. (2) ? = 3
65 × 23.93 - 31.04 264
 + ? = 13
 3 64 × 24 - 31 28
= 4 × 24 - 31 = 96 - 31 = 65
66
215. (5) ? = 1624.12 × 3.891  + ? = 13
7
 1624 × 4 = 6496  6500
216. (5) ?  3018  3 - 841 66 91  66
= 1006 - 841 = 209  200  ? = 13 - =
7 7
217. (1) ?  395 + 187 = 582
25 4
218. (2) ? = 3 3380 + 1300  ? = = 3
7 7
 3 3375 + 1296 227. (1) ?  21 + 4 × 3 + 9
= 5 + 36 = 51 = 21 + 12 + 9 = 42
219. (3) ?  (5) 2 + (21)3 + 1089 23
= 25 + 9261 + 33 = 9319
228. (2)  23
?
7020 13 23
220. (4) ?  × = 1048.96  1049 ?= =1
3 29 23
5000  25 3000  65 229. (3) ?  100000 = 1000
221. (5) ?  -
100 100 3900  134
= 1250 - 1950 = - 700 230. (3) ?  + 134 × 39
100
1 2
222. (3) ? =  81  2 -  64   3 = 2 × (39 × 134) = 10452
231. (2) 103 × 106 + 109 = 10? + 10?
1 2  109 + 109 = 2 × 10?
 1   1  2 3 1 1 16  9 7  2 × 109 = 2 × 10?
=   -   = - = =
 81   64  9 16 144 144
 ?=9
223. (4) (?) 2 = 331.8  23.7 + (-21)2 - 94
= 14 + 441 - 94 = 361

LEARN MATHS FROM S.K. RAJU (9811549822, 9811649822)


25
4 3 or, 727.95 + 177.87 + ? = 950.93
232. (1) ?  × 952 - 129 or, 905.82 + ? = 950.93
12
? = 45.11 = 45 (approx.)
= 952 - 129 = 823 5. (5) 39.05 × 14.95 - 27.99 × 10.12
8450  105 3 = (36 + ?) × 5
233. (2) ?  - 5006 × + 10 583.7975 - 283.2588
100 700
= 8872 - 21 + 10 = 8861 = (36 + ?) × 5
234. (2) 103 × 1003 + 109 = 10? + 10? 300.5387 = (36 + ?) × 5
 109 + 109 = 10? + 107 300.5387
= 36 + ?
?=9 5
235. (4) ? = 21 + 3.7 × 3 60.10774 = 36 +.?
= 21 + 11.1 = 32.1  32 60.10774 - 36 = ?
236. (1) 23 + 9 - ? = 23  ? = 9  ? = 24.10774  24 (approx.)
or 25 (approx.) as per the answer choice.
SBI PO EXAMS Approximate calculation :
39.05 × 14.95 - 27.99 × 10.12 = (36 + ?) × 5
This can be approximated as
1. (5) 39 × 15 - 28 × 10 = (36 + ?) × 5
585 - 280 = (36 + ?) × 5
305 = (36 + ?) × 5
305
= 36 + ?
5
61 = 36 + ?
? = 61 - 36
Volume of cylinder =  r 2h ? = 25 (approx.)
6. (3) 9876  24.96 + 215.006 - ? = 309.99
v =  r 2h (2r) = 2 r 3
 ? = 9876  24.96 + 215.005 - 309.99
Curve Surface area = 2 rh Approximating the terms to the nearest
= 2  r (2r) = 4 r 2 values, we have
? = 9900  25 + 215 - 310
4 r 2 = 616
r2 = 49 9900
 + 215 - 310
r=7 25
v = 2 (7)3 = 2156 cm3 = 396 + 215 - 310
= 2.156 litres = 301  300
 2.2 litres 7. (5) ? =125% of 4875 + 88.005 × 14.995
2. (5) 85.147 + 34.912 × 6.2 + ? = 802.293  125% of 4900 + 88 × 15
or, 85.147 + 216.4544 + ? = 802.293 125  4900
or, 301.6014 + ? = 802.293 = + 1320
100
 ? = 500.6916  500
= 6125 + 1320
Approximate calculation:
= 7445  7415
85.147 + 34.912 × 6.2 + ? = 802.293
8. (4) ? = 127.001 × 7.998 + 6.05 × 4.001
Approximate equation can be written as
85 + 35 × 6 + ? = 802  127 × 8 + 6 × 4
= 1016 + 24 = 1040
85 + 210 + ? = 802
9. (2) ? = 1010  36 + 187 × 20.05
295 + ? = 802
? = 507  500  1008  36 + 187 × 20
[1010  1008 which is a multiple of 36]
3. (1) 248.251  12.62 × 20.52
= 28 + 3740
Its approximate value may be written as
= 3768  3770
 250  12.5 × 20.5
= 20 × 20.5 = 410  400 1
10. (5) 33 % of 768.9 + 25% of 161.2 - 58.12
The answer choice nearest to 410 is 400. 3
4. (5) 158.25 × 4.6 + 21% of 847 + ? = 950.93 100 25
or, 727.95 + 0.21 × 847 + ? = 950.93 = 3  100 × 768.9 + × 161.2 - 58.12
100
LEARN MATHS FROM S.K. RAJU (9811549822, 9811649822)
26
= 256.3 + 40.3 - 58.12
3 2 1
= 238.48  240 = 1715 × × × = 85.75  85
5 3 8
11. (5)  95?  953.7  951.0
26. (2) ?  25 × 124 + 390 × 15
 95? = 953.7-1 = 952.7
= 3100 + 5850 = 8950
 ? = 2.7 27. (1) ?  560  35 × 20
3
12 (2) ?  10000 + × 1892 560
5 = × 20 = 320
35
= 100 + 1135.2 = 1235.2  1230
28. (4) ?  (15)2 × 729
0.0004
13. (3)  × 36 = 4 × 36 = 225 × 27 = 6075
0.0001
= 144  145
14. (1) ?  140% of 12300 RBI GRADE-B OFFICER EXAMS
1. (5) ? = (32.51 + 17.45) (32.51 - 17.45)
140  12300 = 49.96 × 15.06 = 752.39  750
=
100
450  88 530  ?
= 17220  17000 2. (3) 
15. (3) ? 3739 + 164 × 27 100 100
= 3739 + 4428 = 8167  8200 450  88
16. (3) ? = 6523  544 × 1.2 ? = = 74.71  75
530
6523
= × 1.2 = 14.4  14 3. (1) 900 × (12)2 + ?  5000
544
(898  900; 12.005  12)
17. (3) Amount received by each person  30 × 144 + ?  5000
74336  ?  5000 - 4320 = 680
= = Rs 495.57  Rs 496
150 3745
18. (2) ? = 2959.85  16.001 - 34.99 4. (2) ?  × 18 = 2808.75  2800
24
 2960  16 - 35
5. (4) ?  118 × 8 × 5 = 4720
2960
= - 35 = 185 - 35 = 150
16 INSURANCE EXAMS
19. (4) ? = (1702  68 ) × 136.05 1. (2) Let x will come in the place of question
mark (?)
1700
 68  136 = 3400 21.0091 - 6.085 + 13.24 = (3.5 + x) × 2
28.1641 = (3.5 + x) × 2
2950 28.1641
20. (5) ? = + 160 = 400.81  405 3.5 + x = = 14.082
12.25 2
21. (1) ? = 25.05 % 2845 + 14.95 × 2400  x = 14.082 - 3.5 = 10.582  10.5
25 2. (3) 9876  24.96 + 215.006 - ? = 309.99
 100 × 2845 + 15 × 2400  ? = 9876  24.96 + 215.005 - ? = 309.99
= 711.25 + 36000 Approximating the terms to the nearest
= 36711.25  36700 values, we have
? = 9900  25 + 215 - 310
186  271
22. (5) ?  = 1260.15  1260 9900
40 = + 215 - 310
25
425  24 = 396 + 215 - 310
23. (4) ?  = 1275
8 = 301  300
3. (2) ?  8538 - 2416 - 222 = 5900
260  32
24. (5) ?  = 83.2  83 [8537.986  8538]
100
1020
3 2 1 4. (3) ?  = 17
25. (2) ? = 1715 × × × 60
5 3 8

LEARN MATHS FROM S.K. RAJU (9811549822, 9811649822)


27
111111 80  800 66  500
5. (4) ? = = 9.09  9 12. (2) - 260  ? -
1111  11 100 100
6. (5) 3 5000  17  640 - 260 = ? - 330
7. (1) ?  16 × 31 × 9 = 4464  4450  ? = 640 - 260 + 330 = 710  700
8. (3) ? × 30  17000  80 × 20 25 16
13. (3) ?  × × 90 = 80
17000 9 50
 ? × 30 = × 20 = 4250
80
4 8
14. (4) ?  × 5670 - × 2520
4250 9 15
?= = 141.66  140
30 = 2520 - 1344 = 1176  1175
15. (1) ? = 4568.6531 + 134.675 - 2431.3178
900 1200 100 = 2272.0103  2272
9. (2) ?  × × = 360  350
50 20 300
25 45
10. (1) ?  (10) 4 + (30)2 - (17)2 16. (2)  5680 × + 4300 ×
= 1000 + 900 - 289 = 1611  1610 100 100
= 1420 + 1995 = 3355
58  46  10
11. (3) ?  = 808.48  810 17. (2) ?  72 + 82 - 81
33
= 49 + 64 - 9 = 104

LEARN MATHS FROM S.K. RAJU (9811549822, 9811649822)


1
AVERAGE
NATIONALISED BANKS 6. The average age of the family of five
& IBPS PO/MT/SO members is 24. If the present age of
1. The average of 5 numbers is 306.4. The youngest member is 8 years, then what was
average of the first tWO numbers is 431 and the average age of the family at the time of
the average of the last two numbers is the birth of the youngest member ?
214.5. What is the third number ? (1) 20 years (2) 16 years
(1) 108 (3) 12 years (4) 18 years
(2) 52 (5) 21 years
(3) 321 (Corporation Bank PO
(4) Cannot be determined Exam. 22.11.2009)
(5) None of these 7. The average of 5 numbers is 65. The average
(Bank Of Maharashtra of the first two numbers is 81 and the
PO Exam. 25.05.2008) average of the last two numbers is 38. What
2. Kamlesh bought 65 books for Rs. 1,050 is the third number ?
from one shop and 50 books for Rs. 1,020 (1) 63
from another. What is the average price he
(2) 87
paid per book ?
(3) 99
(1) Rs. 36.40 (2) Rs. 18.20
(4) Cannot be determined
(3) Rs. 24 (4) Rs. 18
(5) None of these
(5) None of these
(Indian Bank Rural Marketing
(Bank Of Maharashtra
Officer Exam. 03.01.2010)
PO Exam. 25.05.2008)
8. The total marks obtained by a student in
3. The sum of five numbers is 290. I The
Physics, Chemistry and Mathematics
average of the first two numbers is 48.5 and
together is 120 more than the marks
the average of last two numbers is 53.5.
obtained by him in Chemistry. What are the
What is the third number ?
average marks obtained by him in Physics
(1) 72 (2) 84
and Mathematics together ?
(3) 96 (4) 108
(1) 60
(5) None of these
(2) 120
(Indian Overseas Bank
(3) 40
PO Exam. 05.04.2009)
(4) Cannot be determined
4. The average weight of a group of 53 girls
(5) None of these
was calculated as 58 kgs. It was later
(Allahabad Bank PO Exam. 21.02.2010)
discovered that the weight of one of the girls
9. The average age of 80 boys in a class is 15
was read as 65 kgs., whereas her actual
years. The average age of a group of 15 boys
weight was 45 kgs. What is the actual
in the class is 16 years and the average age
average weight of the group of 53 girls ?
of another 25 boys in the class is 14 years.
(rounded off to two digits after decimal)
What is the average age of the remaining
(1) 58.62 kgs. (2) 58.37 kgs.
boys in the class ?
(3) 57.37 kgs. (4) 57.62 kgs.
(1) 15.25 years
(5) None of these
(2) 14 years
(Indian Overseas Bank
(3) 14.75 years
PO Exam. 05.04.2009)
(4) Cannot be determined
5. The average marks in Science subject of a
(5) None of these
class of 20 students is 68. If the marks of
(Corporation Bank PO
two students were misread as 48 and 65 of
Exam. 09.05.2010)
the actual marks 72 and 61 respectively,
10. The average age of seven boys sitting in a
then what would be the correct average ?
row facing North is 26 years. If the average
(1) 68.5 (2) 69
age of first three boys is 19 years and the
(3) 69.5 (4) 70
average age of last three boys is 32 years,
(5) 66
what is the age of the boy who is sitting in
(Corporation Bank PO
middle of the row ?
Exam. 22.11.2009)
(1) 28 years (2) 29 years

LEARN MATHS FROM S.K. RAJU (9811549822, 9811649822)


2
(3) 24 years (4) 31 years 15.
The sum of eight consecutive even numbers
(5) None of these of set-A is 376. What is the sum of different
(Punjab & Sind Bank PO set of five consecutive numbers whose
Exam. 16.05.2010) lowest number is 15 more than the mean
11. Ram’s present age is three times his son’s of set-A ?
present age and two-fifth of his father’s (1) 296 (2) 320
present age. The average of the present ages (3) 324 (4) 284
of all of them is 46 years. What is the (5) None of these
difference between the Ram’s son’s present (Union Bank Of India PO
age and Ram’s father’s present age ? Exam. 09.01.2001)
(1) 68 years 16. Average score of Rahul, Manish and Suresh
(2) 88 years is 63. Rahul’s score is 15 less than Ajay
(3) 58 years and 10 more than Manish. If Ajay scored
(4) Cannot be determined 30 marks more than the average score of
(5) None of these Rahul, Manish and Suresh, what is the sum
(Bank Of Baroda PO Exam. 30.05.2010) of Manish’s and Suresh’s scores ?
12. The average of five numbers is 56. If the (1) 120
average of first four numbers is 54, what is (2) 111
the value of the fifth number ? (3) 117
(1) 68 (2) 72 (4) Cannot be determined
(3) 56 (4) 64 (5) None of these
(5) None of these (Corporation Bank PO
(Bank Of India PO Exam. 31.10.2010) Exam. 16.01.2011)
13. 17.
The average weight of 45 students in a class The sum of 8 consecutive odd numbers is
was calculated as 36 kg. It was later found 656. Also average of four consecutive even
that the weight of two students in the class numbers is 87. What is the sum of the
was wrongly calculated. The actual weight smallest odd number and second largest
of one of the boys in the class was 32 kg., even number ?
but it was calculated as 34 kg. and the (1) 165
weight of another boy in the class was 45 (2) 175
kg. whereas it was calculated as 40 kg. (3) 163
What is the actual average weight of the 45 (4) Cannot be determined
students in the class ? (5) None of these
(Rounded off to two-digits after decimal) (Bank Of Baroda PO Exam. 13.03.2011)
(1) 36.07 kg. 18. The average marks in English subject of a
(2) 36.16 kg. class of 24 students is 56. If the marks of
(3) 35.84 kg. three students were misread as 44, 45 and
(4) Cannot be determined 61 of the actual marks 48,59 and 67
(5) None of these respectively, then what would be the correct
(PNB Management Trainee average ?
Exam. 28.11.2010) (1) 56 (2) 55
14. Seema’s present age is four times her son’s (3) 57.5 (4) 58.5
present age and four-seventh of her father’s (5) None of these
present age. The average of the present ages (IBPS Bank PO/MT CWE
of all three of them is 32 years. What is the Exam. 18.09.2011)
difference between the Seema’s son’s 19. Amit’s present age is three times Pratibha’s
present age and Seema’s father’s present present age and nine-thirteenth of his
age ? father’s present age. The sum of the present
(1) 44 years age of all of them is 150 years. What is the
(2) 48 years difference between Pratibha’s present age
(3) 46 years and Amit’s father’s present age ?
(4) Cannot be determined (1) 56 years
(5) None of these (2) 64 years
(Bank Of Maharashtra (3) 60 years
Exam. 19.12.2010) (4) Cannot be determined
(5) None of these
LEARN MATHS FROM S.K. RAJU (9811549822, 9811649822)
3
(IDBI Bank Officer Exam. 16.09.2012) 25. The average height of 16 students is 142
20. Farah was married 8 years ago. Today her cm. If the height of the teacher is included,
the average height increases by 1 cm. The
2
age is 1 times to that at the time of height of the teacher is
7 (1) 156 cm (2) 159 cm
marriage. At present her daughter’s age is (3) 158 cm (4) 157 cm
(5) 159.5 cm
1 (Corporation Bank Specialist
6 th of her age. What was her daughter’s Officer (Marketing Exam. 22.12.2014)
age 3 years ago ?
(1) 6 years
SBI PO EXAMS
(2) 7 years 1. If the ages of P and R are added to twice
(3) 3 years the age of Q, the total becomes 59; if the
(4) Cannot be determined ages of Q and R are added to thrice age of
(5) None of these P, the total becomes 68 and if the age of P
(IBPS Specialist Officer is added to thrice the age of Q and thrice
CWE 17.03.2013) the age of R, the total becomes 108. What
21. The average of four consecutive even is the age of P ?
numbers P, Q, R and S respectively (in (1)15 years (2) 19 years
increasing order) is 51. What is the product (3) 17 years (4) 12 years
of P and R ? (5) None of these
(1) 2592 (2) 2400 (SBI Associate Banks PO Exam.
(3) 2600 (4) 2808 14.02.1999)
(5) None of these 2. The product of the ages of Harish and
(IBPS RRBs Office Assistant Seema is 240. If twice the age of Seema is
CWE Exam. 09.09.2012 more than Harish’s age by 4 years, what is
22. The average height of 27 persons was Seema’s age in years ?
recorded as 162 cm. If the height of Shreyas (1) 12 years (2) 20 years
was deleted from the observation, the aver- (3) 10 years (4) 14 years
age height reduced by 1 cm. What was (5) Data inadequate
Shreyas’ height ? (SBI Associate Banks PO
(1) 184 cm. (2) 226 cm. Exam. 14.02.1999)
(3) 179 cm. (4) 186 cm. 3. Of the three numbers, the average of the
(5) None of these first and the second is greater than the
(IBPS RRBs Office Assistant average of the second and the third by 15.
CWE Exam. 09.09.2013) What is the difference between the first and
23. The average of the 9 consecutive positive the third of the three numbers ?
integers is 63. The product of the largest (1) 15
and smallest integers is (2) 45
(1) 3935 (2) 3953 (3) 60
(3) 3853 (4) 3835 (4) Data inadequate
(5) 3635 (5) None of these
(Indian Overseas Bank PO (SBI Banks PO Exam. 20.08.2000)
Online Exam. 01.09.2013 4. In Arun’s opinion his weight is greater than
24. The present age of Romila is one fourth of 65 kg but less than 72 kg. His brother does
that of her father. After 6 years the father’s not agree with Arun and he thinks that
age will be twice the age of Kapil. If Kapil Arun’s weight is greater than 60 kg but less
celebrated fifth birth day 8 years ago, What than 70 kg. His mother’s view is that his
is Romils’s present age ? weight cannot be greater than 68 kg. If all
(1) 7 years (2) 7.5 years of them are correct in their es­timation,
(3) 8 years (4) 8.5 years what is the average of different probably
(5) None of these weights of .Arun ?
(IBPS Bank PO/MT (1) 69 kg (2) 67 kg
CWE-III 26.10.2013) (3) 68 kg (4) Data inadequate
(5) None of these

LEARN MATHS FROM S.K. RAJU (9811549822, 9811649822)


4
(SBI Banks PO Exam. 20.08.2000) years. If Rohit is 54 years old, what is
5. Average weight of three boys P, T and R is Krishna’s age ?
(1) 45 years
1
54 kgs while the average weight of three (2) 24 years
3 (3) 36 years
boys, T, F and G is 53 kgs. What is the (4) Cannot be determined
average weight of P, T, R, F and H ? (5) None of these
(1) 53.8 kgs (2) 52.4 kgs (SBI PO Preliminary (Tire-I)
(3) 53.2 kgs (4) Data inadequate Exam. 27.07.2008)
(5) None of these 11. Father’s age is 30 years more than the son’s
(SBI Associate Banks PO age. Ten years hence the father’s age will
Exam. 21.07.2002) become three times the son’s age that time.
6. Present age of Rahul is 5 years less than What is son’s present age in years ?
Ritu’s present age. If 3 years ago Ritu’s age (1) 8
was x, which of the following represents (2) 7
Rahul’s present age ? (3) 5
(1) x + 3 (2) x - 2 (4) Cannot be determined
(3) x - 3 + 8 (4) x + 3 + 8 (5) None of these
(5) None of these (SBI Associate Banks PO
(SBI PO Exam. 26.11.2006) Exam. 07.08.2011)
7. The average weight of a group of 75 girls
was calculated as 47 kgs. It was later INSURANCE EXAMS
discovered that the weight of one of the girls 1. The number of heat wave days in the Indian
was read as 45 kgs., whereas her actual sub continent in 1995 was 29 and deaths
weight was 25 kgs. What is the actual due to heat wave were 406. What was the
average weight of the group of 75 girls ? average of deaths per heat wave day in 1995 ?
(Rounded off to two digits after decimal) (1) 12 (2) 13
(1) 46.73 kgs (3) 14 (4) 15
(2) 46.64 kgs (United India Insurance Co.
(3) 45.96 kgs AAO Exam. 21.04.2002)
(4) Cannot be determined 2. If twelve years hence a man is going to be
(5) None of these
(SBI PO Preliminary (Tire-I) 1
1 his age twelve years ago, What is his
Exam. 27.04.2008) 3
8. The difference between the present ages of present age ?
Arun and Deepak is 14 years. Seven years (1) 24 years (2) 72 years
ag o th e rati o of th ei r ag es w as 5:7 (3) 96 years (4) 84 years
respectively. What is Deepak’s present age ? (United India Insurance Co.
(1) 49 years (2) 42 years AAO Exam. 21.04.2002
(3) 63 years (4) 35 years 3. Average weight of three boys P, T and R is
(5) None of these
(SBI PO Preliminary (Tire-I) 1
54
kgs while the average weight of three
Exam. 27.04.2008) 3
9. The average age of 80 girls was 20 years, boys, T, F and G is 53 kgs. What is the
the average age of 20 of them was 22 years average weight of P, T, R, F and H ?
and that of another 20 was 24 years. Find (1) 53.8 kgs (2) 52.4 kgs
the average age of the remaining girls. (3) 53.2 kgs (4) Data inadequate
(1) 17 years (2) 19 years (5) None of these
(3) 21 years (4) 15 years (LIC Assitant Administrative
(5) None of these Officer (AAO) Exam. 2006)
(SBI PO Preliminary (Tire-I) 4. The mean temperature of Monday to
Exam. 27.07.2008) Wednesday was 37°C and of Tuesday to
10. The average of the ages of Sumit, Krishna Thursday was 34° C. If the temperature on
and Rishabh is 43 years and the average of Thursday was 4/5th that of Monday, then
the ages of Sumit, Rishabh and Rohit is 49 what was the temperature on Thursday ?
(1) 36.5°C (2) 36°C
LEARN MATHS FROM S.K. RAJU (9811549822, 9811649822)
5
(3) 35.5°C (4) 34°C (5) None of these
(United India Insurance Co. (United India Insurance AO
(AAO) Exam. 11.03.2007) Exam. 27.03.2011)
5. Three yeas ago the average age of A and B 11. Four numbers are written in a row. The
was 18 years. While C joining them now, average of first two numbers is 7, the
the average becomes 22 years. How old (in average of the middle two terms is 2.3 and
years) is C now ? the average of the last two numbers is 8.4.
(1) 24 (2) 27 The average of first number and the last
(3) 28 (4) 30 number is
(United India Insurance Co. (1) 5.9
(AAO) Exam. 11.03.2007) (2) 10.7
6. What fraction must be subtracted from the (3) 13.1
(4) cannot be determined
1 1 1
sum of and to have an average of of (New India Insurance AAO
4 6 12 Exam. 22.05.2011)
all the three fractions ? 12. The age of Mr. Bhagat in 2002 was 1 /90 of
1 1 his birth year. What is his age in 2006 ?
(1) (2) (1) 30 (2) 28
2 3
(3) 26 (4) 22
1 1 (General Insurance Corporation
(3) (6) AAO Exam. 11.12.2011)
4 6
(LIC Assistant Administrative 13. The average of marks obtained in 120
Officer (AAO) Exam. 07.06.2009) students was 35. If the average of passed
7. A team of 8 persons joins in a shooting candidates was 39 and that of failed candi-
competition. The best marksman scored 85 dates is 15, the number of candidates who
points. If he had scored 92 points, the passed the examination is:
average score for the team would have been (1) 100 (2) 110
84. The number of points, the team scored (3) 120 (4) 80
was : (General Insurance Corporation
(1) 645 (2) 665 AAO Exam. 11.12.2011)
(3) 588 (4) 672 14. The mean of n observations is x1 . If the first
(LIC Assistant Administrative observation is increased by 1, second by 2,
Officer (AAO) Exam. 07.06.2009)
8. A is as much younger to B as he is elder to and so on, then their mean is x2 . The value
C. If the sum of the ages of B and C is 48 of x2 - x1 is
years, what is the age of A in years ?
(1) 20 (2) 24 n
(3) 30 (4) 32 (1) n (2) +1
2
(LIC Assistant Administrative
Officer (AAO) Exam. 07.06.2009) n  n  1  n  1
(3) (4)
9. The ages of Melwyn and Louis are in the 2 2
ratio of 7 : 10 respectively. After 6 years (Oriental Insurance Company
the ratio of their ages will be 17 : 23. What AAO Exam. 08.04.2012)
is the difference in their ages ? 15. In a group of 5 friends, the sums of ages (in
(1) 8 years (2) 4 years years) of each group of 4 of them are 124,
(3) 12 years (4) 10 years 128, 130, 136 and 142. The age (in years)
(5) None of these of the youngest of them is
(New India Assurance (1) 18 (2) 21
AO Exam. 25.10.2009) (3) 23 (4) 27
10. Th e su m of seven co nsecutive even (United India Insurance AAO
numbers of a set is 532. What is the average Exam. 03.06.2012)
of first four consecutive even numbers of 16. The mean of 15 different natural numbers
the same set ? is 13. The maximum value of the second
(1) 76 (2) 75 largest of these numbers is
(3) 74 (4) 73
LEARN MATHS FROM S.K. RAJU (9811549822, 9811649822)
6
(1) 53 (2) 52 21. The average of 8 numbers is A and the
(3) 51 (4) 50 average of four of these is B. If the average
(United India Insurance AAO of remainig four is C, then
Exam. 03.06.2012 (1) 2A = B + C (2) A = B + 2C
17. What will be the average of the following (3) A = 2B + C (4) A = 2A + 2B
set of numbers ? (NICL (GIC) AO (Finance)
678, 193, 285, 494, 342, 762 Exam. 08.09.2013 (Paper-I)
(1) 395 (2) 359 22. Average of first five multiples of 5 is :
(3) 495 (4) 459 (1) 15 (2) 15.1
(5) None of these (3) 15.5 (4) 15.3
(LIC Apprentice Development (NICL (GIC) AO (Finance)
Officer (ADO) Online Exam. 03.02.2013 Exam. 08.09.2013 (Paper-I)
18. The average weight of M,N and O is equal 23. The sum of five numbers is 555. The average
to 65 kg. The average weight of M and N is of first two numbers is 75 and the third
equal to 69.5 kg. The average weight of N number is 115. What is the average of last
and O is 66.5 kg. What is the weight of N ? two-numbers ?
(1) 74 kg (2) 75 kg (1) 145 (2) 150
(3) 77 kg (4) 80 kg (3) 265 (4) 290
(5) None of these (NICL (GIC) Administrative
(LIC Assistant Administrative Officer Exam. 15.12.2013)
Office (AAO) Exam. 12.05.2013 24. The mean temperature of Monday and
19. The average temperature of Gwalior, Agra Thursday was 36°C. If the temperature on
an d Luckn ow is 37 o C. Th e averag e
4
temperature of Agra, Gwalior and Delhi is Thursday was th of that of Monday, then
38o C. If the temperature of Delhi is 39°C, 5
find the ratio of temperature of Delhi to that what was the temperature on Thursday ?
of Lucknow ? (1) 36.5°C (2) 32°C
(1) 12 : 11 (3) 35.5°C (4) 34°C
(2) 13 : 12 (NICL (GIC) Administrative
(3) 12 : 13 Officer Exam. 15.12.2013)
(4) Can’t be determined
(NICL (GIC) AO (Finance Exam.
08.09.2013 (Paper-I)
20. The average of the two-digit numbers,
which remain the same when the digits
interchange ther positions is:
(1) 55 (2) 55.5
(3) 56 (4) 56.5
(NICL (GIC) AO (Finance
Exam. 08.09.2013 (Paper-I)

LEARN MATHS FROM S.K. RAJU (9811549822, 9811649822)


7
SHORT ANSWERS 2070
NATIONALISED BANKS = = Rs. 18
115
& IBPS PO/MT/SO 3. (5) Third number
1. (5) 2. (4) = 290 - 2 × 48.5 - 2 × 53.5
3. (5) 4. (4) = 290 - 97 - 107 = 86
5. (2) 6. (2) 4. (4) Actual weight of 53 girls
7. (2) 8. (1) = 53 × 58 + 45 - 65
9. (1) 10. (2) = 3074 + 45 - 65 = 3054 kg.
11. (5) 12. (4)  Required actual average weight
13. (1) 14. (2)
15. (2) 16. (2) 3054
= = 57.62 kg.
17. (3) 18. (5) 53
19. (3) 20. (1) Or,
21. (5) 22. (5) Decrease in weight = 65-45
23. (2) 24. (2) = 20 kg
25. (2)  Required actual average weight
 20 
SBI PO EXAMS =  58   kg = 57.62 kg.
1. (4) 2. (1)  53 
3. (5) 4. (4) 5. (2) Difference of marks
5. (4) 6. (2) = 72 + 61 - 48 - 65 = 20
7. (1) 8. (5)  Correct average marks
9. (1) 10. (3) 20
11. (3) = 68 + = 69
20
6. (2) Total age of 5 member family
INSURANCE EXAMS = 24 × 5 = 120 years
1. (3) 2. (4) Total age 8 years ago
3. (4) 4. (2) = 120 - 5 × 8 = 80 years
5. (1) 6. (4)  Required average age
7. (2) 8. (2)
9. (3) 10. (4) 80
= = 16 years
11. (3) 12. (3) 5
13. (1) 14. (4) 7. (2) The third number
15. (2) 16. (3) = 5 × 65 - 2 × 81 - 2 × 38
17. (4) 18. (3) = 325 - 162 - 76 = 87
19. (2) 20. (1)
120
21. (1) 22. (1) 8. (1) Required average = = 60
23. (1) 24. (2) 2
9. (1) Tricky Approach
Let the required average age be x years.
EXPLANATIONS
 80 × 15 = 15 × 16 + 25 × 14 + 40 × x
NATIONALISED BANKS  40x =1200 - 240 - 350 = 610
& IBPS PO/MT/SO
1. (5) According to the question, 610
x = = 15.25 years
Sum of five numbers = 5 × 306.4 40
= 1532 10. (2) Age of the fourth boy
 Third number = (26 × 7 - 3 × 19 - 3 × 32) years
= 1532 - 2 × 431 - 2 × 214.5 = (182 - 57 - 96) years
= 1532 - 862 - 429 = 29 years
= 241 11. (5) Tricky approach
2. (4) Required average Present age of Ram’s son = x years
1050  1020  Ram’s present age = 3x years
=
65  50 5
Ram’s father’s present age = 3x ×
2
LEARN MATHS FROM S.K. RAJU (9811549822, 9811649822)
8
x + x + 2 + x + 4 + x + 6 = 87 × 4
15x
= years  4x + 12 = 348
2
 4x = 336
15x
 x + 3x + = 46 × 3 336
2  x= = 84
4
 23x = 46 × 3 × 2
 Required sum = 75 + (84 + 4) = 163
 x = 12 18. (5) Difference = 48 + 59 + 67 - 44 - 45 - 61
 Required difference = 24
15 x 13 x  Correct average
= - x =
2 2 24
= 56+ = 57
13  12 24
= = 78 years 19. (3) Pratibha’s present age = x years
2
12. (4) Fifth number  Amit’s present age = 3x years
= 5 × 56 - 4 × 54  Father’s present age
= 280 - 216 = 64 3 x  13 13 x
13. (1) Actual weight of all the students = = years
9 3
= 36 × 45 - 34 + 32 - 40 + 45
= 1620 + 3 = 1623kg. 13 x
 x + 3x + = 150
 Actual average weight 3
1623  3x + 9x + 13x = 150 × 3
= = 36.07kg.  25x = 150 × 3
45
14. (2) Let Seema’s son’s present age = x years 150  3
 Seema’s present age = 4x years = = 18
25
 Seema’s husband’s present age
 Required difference
7
= 4x × = 7x years 13 x 10 x
4 = -x=
3 3
 x + 4x + 7x = 32 × 3
10
96 = × 18 = 60 years
 12x = 96 x= =8 3
12
20. (3) Let Farah’s age 8 years ago be x years
 Required difference
= 7x - x = 6x Farah’s present age = (x + 8) years
= 6 × 8 = 48 years 9x
x + 8 = 7  7x + 56 = 9x
376
15. (2) Mean of set A = = 47  2x = 56  x = 28
8
The lowest number of second set Farah’s present age
= 47 + 15 = 62 = 28 + 8 = 36 years
 Required = 62 + 63 + 64 + 65 + 66 Her daughter’s age 3 years ago
= 320 1
16. (2) Ajay’s score = 63 + 30 = 93 = 36 × - 3 = 3 years
6
 Rahul’s score = 93 - 15 = 78 21. (5) x + x + 2 + x + 4 + x + 6
 Sum of Manish’s and Suresh’s scores = 4 × 51
= 3 × 63 - 78 = 189 - 78 = 111
17. (3) x + x + 2 + x + 4 + x + 6 + x + 8 + x + 10  4x + 12 = 204
+ x + 12 + x + 14 = 656  4x = 204- 12 = 192
 8x + 56 = 656  x =48 = P
 8x = 600  R = 48 + 4 = 52
 P × R = 48 × 52 = 2496
600 22. (5) Height of Shreyas
 x= = 75
8 = 162 × 27 - 161 × 26 = 188 cm
and,

LEARN MATHS FROM S.K. RAJU (9811549822, 9811649822)


9
23. (2) Average of 9 consecutive positive 5. (4) We are to determine the average weight
integers = 63 of P, T, R, F and H. Obviously, this can’t be
 Fifth number = 63 determined as we don’t know tin weight of H.
 Smallest number = 59 6. (2) 3 years ago Ritu’s age = x
Largest number = 67  Ritu’s present age = x + 3
 Required product Rahul’s present age = Ritu present age - 5
= 59 × 67 = 3953 = x + 3 - 5 = x - 2
24. (2) Kapil’s present age = 12 years 7. (1) Total actual weight of all gin
After 6 years, Kapil’s age = 18 years = 47 × 75 - 45 + 25
 Father’s present age = 18 × 2 - 6 = 3525- 20 = 3505 kg.
= 30 years  Average weight
1 3505
 Romlla’s present age = × 30 = 7.5 years = = 46.73 kg
4 75
25. (2) Height of teacher = 143 × 17 - 142 × 16 8. (5) Seven years ago, let Arun’s and Deepak’s
= 159 cm age be 5x and 7x years respectively.
 Arun’s present age = (5x + 7) years
SBI PO EXAMS Deepak’ present age = (7x + 7) years
1. (4) According to question, According to the question,
P + R + 2Q = 59 ....(i) 7x + 7 - 5x - 7 = 14
Q + R + 3P = 68 .... (ii)  2x = 14
P + 3Q + 3R = 108 ......(iii)
14
Subtract equation (iii) from thrice the  x= =7
equation (ii), we get 2
3Q + 3R + 9P - P - 3Q - 3R = 204 - 108  Deepak’s present age = 7x + 7
 8P = 96 = 7 × 7 + 7 = 56 years
9. (1) Total age of remaining 40 girls
96 = (80 × 20 - 20 × 22 - 20 × 24) years
 P= = 12 years
8 = (1600 - 440 - 480 ) years = 680 years
2. (1) Let the ages of Harish and Seema be x 680
and y years respectively.  Required average age = 40 = 17 years
 According to the question,
x × y = 240 10. (3) (Sumit + Krishna + RishabhJ’s age
and 2y = x + 4 = 43 × 3 = 129 years ....(i)
Now, (Sumit + Rishabh + Rohit)’s age
(2y - 4) × y = 240 = 3 × 49 = 147 years ....(ii)
By equation (i)-(ii), we have
 2y2 - 4y - 240 = 0
Krishna’s age - Rohit’s age = 129 - 147
 y2 - 2y - 120 = 0 = -18
 y2 - 12y + 10y - 120 = 0  Krishna’s age = 54 - 18 = 36 years
 y(y - 12) + 10(y - 12) = 0 11. (3) Let the son’s present age be x years.
 (y + 10) (y - 12) = 0  Father’s present age = (x + 30) years
 y = -10, 12 After 10 years,
 age of Seema = 12 years x + 40 = 3(x + 10)
3. (5) Let the three numbers be x, y and z.  x + 40 = 3x + 30
x y y z  2x = 10  x = 5 years
or, - = 15
2 2 INSURANCE EXAMS
x y yz
or, = 15 406
2 1. (3) Required answer = = 14
29
or, x - z = 30
4. (4) Arun’s opinion : 65 < W < 72 2. (4) Let present age of man be x years
Brother’s opinion : 60 < W < 70  According to the question,
Mother’s opinion : W not greater than 68 1
or W < 68 (x + 12) = 1 × (x -12)
3

LEARN MATHS FROM S.K. RAJU (9811549822, 9811649822)


10
4  170 x + 102 = 161 x + 138
or, x + 12 =
3
(x -12)  9x = 138 - 102 = 36
36
4x x= = 4
or, x + 12= -16 9
3
 Required difference = 10x - 7x = 3x
4x = 3 × 4 = 12 years
or, - x = 12 + 16
3 532
10. (4) Fourth even numbers = = 76
x 7
or, = 28 First four even numbers are 70, 72, 74 and
3
 x = 28 × 3 = 84 years 76.
3. (4) We are to determine the average weight  Required average
of P, T, R, F and H. Obviously, this can’t be 70  72  74  76
determined as we don’t know the weight of =
4
H.
4. (2) M + T + W = 3 × 37 = 111°C 292
= = 73
T + W + Th = 3 × 34 = 102° C 4
Subtracting equation (ii) from equation (i), 11. (3) Let the numbers be a, b, c and d
M - Th = (111 -102)°C respectively.
5 a + b = 14 ...(i)
 Th - Th = 9°C b + c = 4.6 ...(ii)
4
c + d = 16.8 ...(iii)
Th By equations (i) - (ii) + (iii)
 = 9 o C  Th = 9 × 4 = 36°C a + d = 14 - 4.6 + 16.8 = 26.2
4
5. (1) Three years ago, ad 26.2
Sum of the ages of A and B = 18 × 2  = = 13.1
2 2
= 36 years 12. (3) Let the birth year be x.
Sum of the present ages of A and B = 36 + 6
= 42 years 1
Sum of the present ages of A, B and C  90 x = 2002 - x
= 3 × 22 = 66 years  x = 2002 × 90 - 90x
 Present age of C = 66 - 42 = 24 years  91x = 2002 × 90
6. (4) Required fraction
 x = 1980
1 1 1  Bhagat’s age in 2006
= + -3 ×
4 6 12 = 2006 - 1980
= 26 years
1 1 1 1 13. (1) If the number of successful candidates
= + - =
4 6 4 6 be x , th en n umbe r o f un su ccessful
candidates = 120 - x
7. (2) The numbers of points term scored  x × 39 + (120 - x) 15 = 120 × 35
= 8 × 84 - 92 + 85 = 672 - 92 + 85 = 665  39x + 1800 - 15x = 4200
8. (2) According to the question,  24x = 4200 - 1800 = 2400
B - A = A - C  x = 100
 2A = B + C = 48
14. (4) New mean = x 2
48
A= = 24 years x1  x2  ......  x x 1  2  3  ....  n
2 = +
9. (3) Let the present ages of Melwyn and Louis n n
be 7x and 10x years respectively. n  n  1
After 6 years = x1 +
2n
7 x  6 17
=
10 x  6 23 n 1
= x1 +
2
LEARN MATHS FROM S.K. RAJU (9811549822, 9811649822)
11
20. (1) Required average
n 1
 x2 - x1 = x1 + - x1 11  22  33  ...  99
2
=
9
n 1
=
2 111  2  3  ....  9 
15. (3) a + b + c + d = 124 =
9
b + c + d + e = 128
c + d + e + a = 130 11  45
= = 55
d + e + a + b = 136 9
e + a + b + c = 142 21. (1) 8A = 4B + 4C
On adding,  2A= B + C
4(a + b + c + d + e) = 660 22. (1) Required average
 a + b + c + d + e = 165
 Age of the youngest of them 5  10  15  20  25
=
= d = 165- 142 = 23 years 5
16. (3) Sum of 15 numbers = 15 × 13 = 195
5  1  2  3  4  5
For maximum value, the other 13 numbers = =15
will be least. 5
 1 + 2 + ......+ 13 23. (1) Sum of last two numbers
= 555 - 2 × 75 - 115
13  14
= = 91 = 555 - 150 - 115 = 290
2
 Sum of last two numbers 290
 Required average = = 145
= 195 - 91 = 104 2
 Numbers = 51 and 53 24. (2) Monday + Thursday = 2 × 36 = 72°C
17. (4) Sum of numbers If the temperature on Thursday be T°C, then
= 678 + 193 + 285 + 494 + 342 + 762 5
= 2754 temperature on Monday = T
4
 Required average
2754 5
= = 459  T + T = 72
6 4
18. (3) M + N + O = 3 × 65 = 195 kg 9T
M + N = 2 × 69.5 = 139 kg.  = 72
4
N + O = 2 × 66.5 = 133 kg.
 Weight of N = (139 + 133 - 195) kg. 4  32
T= = 32°
= 77 kg. 9
19. (2) Gwalior + Agra + Lucknow
= 37 × 3 =111°C ...(i)
Agra + Delhi + Gwalior
= 38 × 3 = 114°C ...(ii)
By equation (i) - (ii)
Lucknow - Delhi = 111 - 114 = - 3°C
 Lucknow = -3 + 39 = 36°C
 Required ratio
= 39 : 36 = 13 : 12

LEARN MATHS FROM S.K. RAJU (9811549822, 9811649822)


12
MODEL EXERCISES
1. A batsman makes a score of 98 runs in the
F  M  20 F M
19th innings and thus increases his (1) (2)  20
average by 4. What is his average after 19th 2 2
innings ? F M
(1) 22 (2) 24 (3)  20 (4) F + M -10
2
(3) 28 (4) 26 (5) None of these
(5) None of these 7. The average of three numbers is 135. The
2. The average weight of 45 students in a class largest number is 180 and the difference
is 52 kg. 5 of them whose average weight is between the others is 25. The smallest
48 kg leave the class and other 5 students number is —
whose average weight is 54 kg join the (1) 130 (2) 125
class. What is the new average weight (in (3) 120 (4) 100
kg) of the class ? (5) None of these
2 8. Of the three numbers, the first is twice the
(1) 52.6 (2) 52 second and the second is twice the third.
3
The average of these numbers is 21. Find
1 3 the largest number out of these.
(3) 52 (4) 52
3 4 (1) 36 (2) 38
(5) None of these (3) 47 (4) 48
3. In three numbers, the first is twice the (5) None of these
second and thrice the third. If the average 9. There are 30 students in a class. The
of these three numbers is 44, then the first average age of the first 10 students is 12.5
number is — years. The average age of the next 20
(1) 72 (2) 24 students is 13.1 years. The average age of
(3) 36 (4) 44 the whole class is —
(5) None of these (1) 12.5 years (2) 12.7 years
4. The batting average of 40 innings of a (3) 12.8 years (4) 12.9 years
cricket player is 50 runs. His highest score (5) None of these
exceeds his lowest score by 172 runs. If 10. The weight in kilograms of 10 students are
these two innings are excluded the average 52, 45, 31, 35, 40, 55, 60, 38, 44, 36. If 44
of the remaining 38 innings is 48. His is replaced by 46 and 40 is replacec by 35,
highest score was : then the new median will be —
(1) 172 (2) 173 (1) 42 (2) 40.5
(3) 174 (4) 176 (3) 40 (4) 41.5
(5) None of these (5) None of these
5. Nine persons went to a hotel for taking their 11. In the first 10 overs of a cricket game, the
meals. Eight of them spent Rs 12 each over run rate was only 3.2. What should be the
their meals and the ninth spent Rs 8 more run rate in the remaining 40 overs to reach
than the average expending of all the nine. the target of 282 runs ?
Total money spent by them was : (1) 6.25 (2) 6.50
(1) Rs 104 (2) Rs 105 (3) 6.75 (4) 7
(3) Rs 116 (4) Rs 117 (5) None of these
(5) None of these 12. The average (arithmetic mean) of x and y is
6. A demographic survey of 100 families in 40. If z = 10, then what is the average of x,
which two parents were present revealed y and z ?
that the average age A, of the oldest child 2
(1) 16 (2) 30
1 3
is 20 years less than the sum of the ages
2
1
of the two parents. If F represents the age (3) 25 (4) 17
2
of one parents and M the age of the other
parents, then which of the following is (5) None of these
equivalent to A ?

LEARN MATHS FROM S.K. RAJU (9811549822, 9811649822)


13
13. The mean of 1, 7, 5, 3, 4 and 10 is m. The 20. Some students planned a picnic. The
mean of 10, 12, 3, 5, 1, 7 and p is m + 1 and budget for food was Rs 500. But 5 of them
their median is q. Then, q is equal to — failed to go and thus the cost of food for
(1) 4 (2) 5 each member increased by Rs 5. How many
(3) 10 (4) 12 students attend the picnic ?
(5) None of these (1) 15 (2) 25
14. In a class, the average score of girls in an (3) 20 (4) 30
examination is 73 and that of boys is 71. (5) None of these
The average score for the whole class is 21. The average score of boys in an examination
71.8. The percentage of the girls in the class in a school is 71 and that of the girls is 73.
is — The average score of the school is 71.8. The
(1) 60% (2) 40% ratio of the number of boys to that of the
(3) 1.8% (4) 18% girls that appeared in the examination is
(5) None of these —
15. The average of 10 numbers is 40.2. Later it (1) 1 : 2 (2) 3 : 2
is found that two numbers have been (3) 2 : 3 (4) 4 ; 2
wrongly added. The first is 18 greater than (5) None of these
the actual number and the second number 22. The mean monthly salary paid to 75
added is 13 instead of 31. Find the correct workers in a factory is Rs 5680. The mean
average. salary of 25 of them is Rs 5400 and that of
(1) 40.2 (2) 40.4 30 others is Rs 5700. The mean salary of
(3) 40.6 (4) 40.8 the remaining workers is —
(5) None of these (1) Rs 5000 (2) Rs 7000
16. The average of 11 numbers is 10.9. If the (3) Rs 6000 (4) Rs 8000
average of the first six numbers is 10.5 and (5) None of these
that of the last six numbers is 11.4, then 23. The average monthly expenditure of a
the middle number is — family was Rs 2200 during the first 3
(1) 11.5 (2) 11.4 months; Rs 2250 during the next 4 months
(3) 11.3 (4) 11.0 and Rs 3120 during the last 5 months of a
(5) None of these year. If the total savings during the year
17. The average temperature of the first four were 1260, then the average monthly
days of a week is 40.2°C and that of the income was—
last four days is 41.3°C. If the average (1) Rs 2605 (2) Rs 2805
temperature for the whole week is 40.6°C, (3) Rs 2705 (4) Rs 2905
then the temperature on the fourth day is (5) None of these
— 24. A monthly return railway ticket costs 25%
(1) 40.8°C (2) 38.5°C mo re than a si ng le ti cket. A we ek’s
(3) 41.3°C (4) 41.8°C extension can be had for the former by
(5) None of these paying 5% of the monthly ticket’s cost. If
18. The average salary of all the workers in a the money paid for the monthly ticket (with
workshop is Rs 8000. The average salary of extension) is 84, the price of the single
7 technicians is Rs 12000 and the average ticket is —
salary of the rest is Rs 6000. The total (1) Rs 64 (2) Rs 80
number of workers in the workshop is — (3) Rs 48 (4) Rs 72
(1) 21 (2) 20 (5) None of these
(3) 23 (4) 22 25. The average monthly salary of employees,
(5) None of these consisting of officers and workers of an
19. A number of friends decided to go on a organization is Rs 3000. The average salary
picnic and planned to spend Rs 96 on of an officer is Rs 10000 while that of a
eatables. Four of them, did not turn up. As worker is Rs 2000 per month. If there are
a consequence, the remaining ones had to total 400 employees in the organization,
contribute Rs 4 each extra. The number of find the number of officers and workers
those who attended the picnic was — separately.
(1) 8 (2) 16 (1) 50, 350 (2) 350, 450
(3) 12 (4) 24 (3) 50, 275 (4) 325, 350
(5) None of these (5) None of these
LEARN MATHS FROM S.K. RAJU (9811549822, 9811649822)
14
SHORT ANSWERS  x + y = 2000 - 1824
1. (4) 2. (2)  x + y = 176
3. (1) 4. (3) and x - y = 172
5. (4) 6. (3) Solving Eqs. (i) and (ii), we get
7. (4) 8. (1) x = 174
9. (4) 10. (4) 5. (4) Let the ninth person spent Rs x.
11. (1) 12. (2) Then, average of all the nine
13. (2) 14. (2)
12  8  x 96  x
15. (1) 16. (1) = =
17. (4) 18. (1) 9 9
19. (1) 20. (3) 96  x
21. (2) 22. (3) Given, x = +8
9
23. (3) 24. (1)
28. (1) 9x = 96 + x + 72
 8x = 168
EXPLANATIONS  x = 21
1. (4) Let us suppose that the average of 18 Hence, total money spent
innings be x. = 96 + 21 = Rs 117
6. (3) According to question
18 x  98
Then, =x+4 FM
19 A= - 20
2
 19x + 76 = 18x + 98
7. (4) According to question,
 x = 22
Thus, average after 19th innings  x  25   x   180
= x + 4 = 22 + 4 = 26 = 135
3
2. (2) According to the question,
 2x + 155 = 405
Total weight of 45 students
= 45 × 52 = 2340 kgs 250
= x = = 125
Total weight of 5 students who leave 2
= 5 × 48 = 240 kgs Hence, the smallest number
Total weight of 5 students who join = 125 - 25 = 100
= 5 × 54 = 270 kgs 8. (1) Let the numbers be a, b and c. Then,
Hence, new total weight of 45 students
b
= 2340 - 240 + 270 = 2370 a = 2b and b = 2c or c =
2
2370 2
 New Average = = 52 kg b
45 3 2b  b 
3.(1) Let the three numbers be a, b and c. 2
Then, = 21
a = 2b and 3c 3
a = 3c 7b
a  = 21  b = 18
a 6
 b = and c = 3
2  Numbers are 36, 18 and 9
 The largest number is 36.
a a
a  9. (4) Total age of 10 students = 125 years
Given, 2 3 = 44 Total age of 20 students = 262 years
3  Average age of 30 students
11a 125  262
 = 44 =
30
=12.9 years
18
 a = 72 10. (4) According to question, replacing the two
4. (3) According to total of 40 innings number and arranging them in ascending
= 40 × 50 = 2000 order, we get 31, 35, 35, 36, 38, 45, 46, 52,
Total of 38 innings = 38 × 48 = 1824 55, 60
 Sum of highest and lowest score 38  45 83
 Median = = = 41.5
2 2
LEARN MATHS FROM S.K. RAJU (9811549822, 9811649822)
15
11. (1) Total runs in the first 10 overs = 131.4 - 119.9 = 11.5
= 10 × 3.2 = 32 17. (4) Temperature on the fourth day
Runs rate in the remaining 40 overs = 40.2 × 4 + 41.3 × 4 - 40.6 × 7
= 160.8 + 165.2 - 284.2 = 41.8°C
282  32 250
= = = 6.25 18. (1) Let total no. of workers be x.
40 40 Then, 8000 x = 7 × 12000 + (x - 7) × 6000
12. (2) Total of x and y = 40 × 2 = 80  8000x = 84000 + 6000x - 42000
 Average of x, y and z
42000
80  10 90 x= = 21
= = = 30 2000
3 3
19. (1) Let there were x friends, then contribution
13. (2) According to question,
96
1  7  5  3  4  10 of one friend =
m = x
6
If four friends have left, then contribution
m=5 of each friend
and (m + 1)
96
10  12  3  5  1  7  p =
x4
=
7
96 96
38  p  - = 4
 =m +1 x4 x
7
 x = 12
38  p Hence, number of friends who attended the
 =5+1 picnic = 12 - 4 = 8
7
20. (3) Le t th ere we re x stu de nts, the n
 p = 42 - 38 = 4
 Numbers are 1, 3, 4, 5, 7, 10, 12. 500
 Median q = 5 contribution of one student =
x
14. (2) Let the number of boys be x and that of Contribution of each students where 5 of
girls be y.
Then, 500
total score of boys = 71 x them have left =
 x  5
total score of girls = 73 y
500 500
71x  73 y Given,  x  5  - = 5
  x  y  = 71.8 x
using options, we find x = 25 satisfies the
 71x + 73y = 71.8x + 71.8y equation. Therefore, number of students
x 1.2 3 who attended the picnic
0.8x = 1.2y y
= = = (25 - 5) = 20.
0.8 2
21. (2) Let the total number of boys and girls
 Percentage of girls in the class be B and G respectively.
2  Total score of boys = 71 B
= × 100 = 40% Total score of girls = 73G
5
Total score of the class = 71.8 (B+G)
15. (1) Sum of 10 numbers = 402
Correct sum of 10 numbers  71 B + 73 G = 71.8 (B+G)
402 - 13 + 31 - 18 = 402 B 1.2 3
 0.8B = 1.2G  = =
402 G 0.8 2
Hence, new average = = 40.2 22. (3) Total salary of 75 workers = 75 × 5680
10
= Rs 426000
16. (1) Sum of 11 numbers = 11 × 10.9
Total salary of 25 workers = 25 × 5400
Sum of first 6 numbers = 6 × 10.5
= Rs 135000
Sum of last 6 numbers = 6 × 11.4
Total salary of 30 workers = 30 × 5700
 The middle number
= Rs 171000
= 6 × 10.5 + 6 × 11.4 - 11 × 10.9
= 63 + 68.4 - 119.9  Total salary of remaining 20 workers
LEARN MATHS FROM S.K. RAJU (9811549822, 9811649822)
16
= 426000 - (135000 + 171000) = 120000 24. (1) Required price of the single ticket
120000 84 100 100
 Mean salary of 20 workers = = × ×
20 105 1 125
= Rs 6000
100 100
23. (3) Total expenditure for the year = 84 × × = Rs 64
= [2200 × 3 + 2250 × 4 + 3120 × 5) 105 125
= 6600 + 9000 + 15600 = Rs 31200 25. (1) Since, there are 400 employes.
Total saving = Rs 1260 hence option (1) is correct.
Total income = expenses + savings
= 31200 + 1260 = Rs 32460
32460
 Average income = = Rs 2705
12

LEARN MATHS FROM S.K. RAJU (9811549822, 9811649822)


1
COMPOUND INTEREST
NATIONALISED BANKS (1) Rs.5,420 (2) Rs.5,360
& IBPS PO/MT/SO (3) Rs.5,280 (4) Rs.5,140
1. Suresh invested a sum of Rs. 15000 at 9 (5) None of these
per cent per annum simple interest and Rs. (Oriental Bank of Commerce
12000 at 8 per cent per annum compound PO Exam. 21.12.2008)
interest for a period of 2 years. What 6. The compound interest accrued on an
amount of interest did Suresh earn in 2 amount of Rs. 25,500 at the end of three
years ? years is Rs. 8,440.5. What would be the
(1) Rs. 4096.60 (2) Rs. 4696.80 simple interest accrued on the same amount
(3) Rs. 4896.60 (4) Rs. 4698.60 at the same rate in the same period ?
(5) None of these (1) Rs. 4,650 (2) Rs. 5,650
(Syndicate Bank PO Exam. 10.10.2004) (3) Rs. 6,650 (4) Rs. 7,650
2. Satish invests Rs. 35,500 in a scheme (5) None of these
which earns him simple interest at the rate (Canara Bank PO Exam. 15.03.2009)
of 15 p.c.p.a. for two years. At the end of 7. What would be the compound interest
two years he reinvests the principal amount obtained on an amount of Rs. 20,000 at the
plus interest earned in another scheme rate of 15 p.c.p.a. after 4 years ?
which earns him compound Interest at the (1) Rs. 14,980.125
rate of 20 p.c.p.a. What will be the total (2) Rs. 19,680.125
interest earned by Satish over the principal (3) Rs. 16,780.125
amount at the end of 5 years ? (4) Rs. 18,980.125
(1) Rs. 30,956.80 (5) None of these
(2) Rs. 35,017.20 (Indian Overseas Bank PO
(3) Rs. 43,597.80 Exam. 05.04.2009)
(4) Rs. 44,247.20 8. Mr. Duggal invested Rs. 20,000 with rate
(5) None of these of interest at 20 p.c.p.a. The interest was
(Union Bank of India PO compounded half yearly for first year and
Exam. 27.11.2005 in the next year it was compounded yearly.
3. The simple interest accrued on an amount What will be the total interest earned at the
of Rs. 14,800 at the end of three years is end of two years ?
Rs. 6,216. What would be the compound (1) Rs. 8,800 (2) Rs. 9,040
interest accrued on the same amount at the (3) Rs. 8,040 (4) Rs. 9,800
same rate in the same period ? (5) None of these
(1) Rs. 6986.1142 (United Bank of India PO
(2) Rs. 7042.2014 Exam. 21.06.2009)
(3) Rs. 7126.8512 9. What will be the compoun d inte rest
(4) Rs. 8321.4166 accrued on an amount. Rs. 10,000 at the
(5) None of these rate of 20 p.c.p.a. in two years if the interest
(Corporation Bank PO is compounded half yearly ?
Exam. 29.07.2006) (1) Rs. 4,400 (2) Rs. 4,600
4. The simple interest accrued on an amount (3) Rs. 4,641 (4) Rs. 4,680
of Rs. 40,000 at the end of three years is (5) None of these
Rs. 12,000. What would be the compound (Andhra Bank PO Exam. 05.07.2009
interest accrued on the same amount at the 10. What would be the compound interest
same rate in the same period ? accrued on an amount of Rs. 7,400 at the
(1) Rs. 18,765 (2) Rs. 15,350 rate of 13.5 p.c.p.a. at the end of two years
(3) Rs. 21,555 (4) Rs. 13,240 ? (rounded off to two digits after decimal)
(5) None of these (1) Rs. 2,136.87
(Andhra Bank PO Exam. 14.09.2008) (2) Rs. 2,306.81
5. The compound interest accrued on an (3) Rs. 2,032.18
amount of Rs. 22,000 at the end of two (4) Rs. 2,132.87
years is Rs. 5,596.8. What would be the (5) None of these
simple interest accrued on the same amount (Indian Bank PO Exam. 17.10.2010)
at the same rate in the same period ?
LEARN MATHS FROM S.K. RAJU (9811549822, 9811649822)
2
11. What would be the compound interest 17. The difference between the amount of
accrued on an amout of Rs. 45,400 at the compound interest and simple interest
end of two years at the rate of 15 p.c.p.a ? accured on an amount of Rs 26000 at the
(1) Rs. 16411.5 (2) Rs. 14461.5 end of 3 years is Rs 2994.134. What is the
(3) Rs. 16461.5 (4) Rs. 14641.5 rate of interest p.c.p.a. ?
(5) None of these (1) 22
(Bank Of India Banking Officer (2) 17
Exam. 24.01.2010) (3) 19
12. Sonika invested an amount of Rs. 5,800 for (4) Cannot be determined
2 years. At what rate of compound interest (5) None of these
will she get an amount of Rs. 594.5 at the (Punjab & Sind Bank PO
end of two years ? Exam. 23.01.2011)
(1) 5 p.c.p.a. (2) 4 p.c.p.a. 18. The simple interest accrued on a sum of
(3) 6 p.c.p.a. (4) 8 p.c.p.a. certain principal is Rs 6500 in eight years
(5) None of these at the rate of 13 per cent per year. What
(Corporation Bank PO would be the compound interest accrued
Exam. 09.05.2010) on that principal at the rate of 8 per cent
13. A man gets a simple interest of Rs. 1,000 per year in 2 years?
on a certain principal at the rate of 5 p.c.p.a. (1) Rs 1040 (2) Rs 1020
in 4 years. What compound interest will (3) Rs 1060 (4) Rs 1200
the man get on twice the principal in two (5) None of these
years at the same rate ? (UCO Bank PO Exam. 30.01.2011)
(1) Rs. 1,050 (2) Rs. 1,005 19. The simple interest accrued on a sum of
(3) Rs. 11,025 (4) Rs. 10,125 certain principal is Rs 7,200 in six years at
(5) None of these the rate of 12 p.c.p.a. What would be the
(Punjab & Sind Bank Exam. compound interest accrued on that principal
16.05.2010) at the rate of 5 p.c.p.a. in 2 years ?
14. What will be the approximate difference (1) Rs 1,020 (2) Rs 1,055
in the simple and compound interest (3) Rs 1,050 (4) Rs 1,025
accrued on an amount of Rs 2600 at the rate (5) None of these
of 15 p.c. p.a. at the end of three years ? (Bank Of Baxoda PO Exam. 13.03.2011)
(1) Rs 167 (2) Rs 194 20. The simple interest accrued on an amount
(3) Rs 202 (4) Rs 172 of Rs 22,500 at the end of four years is Rs
(5) Rs 184 10,800. What would be the compound
(United Bank Of India PO interest accrued on the same amount at the
Exam. 14.11.2010) same rate at the end of two years ?
15. Pamela invested an amount of Rs 35,000 (1) Rs 14,908 (2) Rs 5,724
for two years at the rate of 5 p.c.p.a. What (3) Rs 26,234 (4) Rs 8,568
amount of compound interest would she (5) None of these
receive at the end of two years ? (IBPS Bank PO/MT CWE 18.09.2011)
(1) Rs 3587.50 (2) Rs 3500 21. What is the difference between the simple
(3) Rs 3580.50 (4) Rs 3565.50 and compound interest on Rs 7300 at the
(5) None of these rate of 6 p.c.p.a. in 2 years ?
(PNB Management Trainee (1) Rs 29.37 (2) Rs 26.28
Exam. 28.11.2010) (3) Rs 31.41 (4) Rs 23.22
16. The simple interest accrued on a sum of (5) Rs 21.34
certain principal is Rs 2,000 in five years (IBPS Bank PO/MT CWE 17.06.2012)
at the rate of 4 p.c.p.a. What would be the 22. The compound interest earned on a sum
compound interest accrued on same in 3 years at 15% per annum compounded
principal at same rate in two years ? annually is Rs 6500.52. What is the sum ?
(1) Rs 716 (2) Rs 724 (1) Rs 12480 (2) Rs 10500
(3) Rs 824 (4) Rs 816 (3) Rs 14800 (4) Rs 13620
(5) None of these (5) None of these
(Corporation Bank PO (IBPS Specialist Officer
Exam. 16.01.2011) CWE 17.03.2013)

LEARN MATHS FROM S.K. RAJU (9811549822, 9811649822)


3
23. What would be the compound interest (2) Rs. 8098.56
obtained on an amount of Rs. 7,790 at the (3) Rs. 8246.16
rate of 10 p.c.p.a. after two years ? (4) Rs. 8112.86
(1) Rs. 1532.60 (2) Rs. 1495.90 (5) None of these
(3) Rs. 1653.50 (4) Rs. 1635.90 (SBI PO Preliminary (Tire-I)
(5) None of these Exam. 27.07.2008)
(IBPS RRBs Office Assistant 6. The compound interest accrued on an
CWE Exam. 09.09.2012) amount at the end of three years at 15 p.c.p.a.
is Rs. 6500.52. What is the amount ?
SBI PO EXAMS (1) Rs. 12480 (2) Rs. 10500
1. The difference between the simple interest (3) Rs. 14800 (4) Rs. 13620
on a certain sum at the rate of 10% per (5) None of these
annum for 2 years and compound interest (SBI PO Preliminary (Tire-I)
which is compounded every 6 months is Exam. 27.07.2008)
Rs. 124.05. What is the principal sum ? 7. The compound interest earned by Suresh
(1) Rs. 10,000 (2) Rs. 6,000 on a certain amount at the end of two years
(3) Rs. 12,000 (4) Rs. 8,000 at the rate of 8 p.c.p.a was Rs. 1,414.4. What
(5) None of these was the total amount that Suresh got back
(SBI Banks PO Exam. 20.08.2000) at the end of two years in the form of
2. Sudharshan invested Rs. 15,000 at interest principal plus interest earned ?
@ 10 p.c. p.a. for one year. If the interest (1) Rs.9,414.4 (2) Rs.9.914.4
is compounded every six months what (3) Rs.9,014.4 (4) Rs.8,914.4
amount will Sudharshan get at the end of (5) None of these
the year ? (SBI & Rural Business PO
(1) Rs. 16,537.50 Exam. 18.04.2010)
(2) Rs. 16, 500
(3) Rs. 16, 525.50 RBI GRADE—B OFFICER EXAMS
(4) Rs. 18,150 1. The compound interest accrued on an
(5) None of these amount at the end of two years at the rate of
(SBI Associate Banks 12 p.c.p.a is Rs 2,862. What is the amount
PO Exam. 21.07.2002) ?
3. What will be the compound interest on a (1) Rs 11,250 (2) Rs 12,200
sum of Rs. 25,000 after three years at the (3) Rs 13,500 (4) Rs 10,000
rate of 12 per cent p.a. ? (5) None of these
(1) Rs. 10123.20 (RBI Grade-B Officer Exam.06.02.2011)
(2) Rs. 9000.30
(3) Rs. 10483.20
(4) Rs. 9720 INSURANCE EXAMS
(5) None of these 1.
Rajan invested an amount of Rs. 8,000/-
(SBI Banks PO Exam. 18.05.2003) in a fixed deposit scheme for 2 years at
4. Mr. Rao invests a sum of Rs. 41,250 at the compound interest rate 5 percent p.a. How
rate of 6 p.c.p.a. What approximate amount much amount will Rajan get on maturity of
of compound interest will he obtain at the the fixed deposit ?
end of 3 years ? (1) Rs. 8,620/- (2) Rs. 8,840/-
(1) Rs.8,100 (2) Rs.7,425 (3) Rs. 8,800/- (4) Rs. 8,600/-
(3) Rs.8,210 (4) Rs.7,879 (5) None of these
(5) Rs.7,295 (LIC Assistant Administrative
(SBI PO Preliminary (Tire-I) Officer (AAO) Exam. 24.04.2005)
Exam. 27.04.2008) 2. Sudharshan invested Rs. 15,000 at interest
5. The simple interest accrued on an amount @ 10 p.c. p.a. for one year. If the interest is
of Rs. 20,000 at the end of three years is co mpo unded every si x mon ths wh at
Rs. 7,200. What would be the compound amount will Sudharshan get at the end of
interest accrued on the same amount at the the year ?
same rate in the same period ? (1) Rs. 16,537.50 (2) Rs. 16,500
(1) Rs. 8342.36 (3) Rs. 16,525.50 (4) Rs. 18,150
(5) None of these
LEARN MATHS FROM S.K. RAJU (9811549822, 9811649822)
4
(LIC Assistant Administrative 7. The simple interest accrued on a sum of
Officer (AAO) Exam. 2006) certain principal is Rs 6,400 in four years
3. A sum of money becomes Rs. 13,380 after at the rate of 8 p.c.p.a. What would be the
3 years and Rs. 20,070 after 6 years on com- compound interest accrued on that principal
pound interest. The sum (in Rupees) is : at the rate of 2 p.c.p.a. in 2 years ?
(1) 8800 (2) 8890 (1) Rs 800 (2) Rs 808
(3) 8920 (4) 9040 (3) Rs 704 (4) Rs 700
(United India Insurance Co. (5) None of these
(AAO) Exam. 11.03.2007) (United India Insurance AO
4. The compound interest (in Rupees) on Rs. Exam. 27.03.2011)
8. A sum of Rs. 91,000 is borrowed at 20%
1
5,600 for 1 years at 10% per annum, per annum compounded annually. If the
2 amount is to be paid in three equal
compounded annually, is instalments,the annual instalment will be
(1) 882.70 (2) 873.50 (1) Rs. 43,200 (2) Rs. 42,800
(3) 868 (4) 840 (3) Rs. 42,500 (4) Rs. 42,300
(LIC Assistant Administrative (United India Insurance AAO
Officer (AAO) Exam, 07.06.2009) Exam. 03.06.2012)
5. The simple interest accrued on an amount 9. The compound interest accrued on an
of Rs. 27,500 at the end of three years is amount of Rs. 22,000 at the end of two
Rs. 10,230. What would be the approximate years is Rs 5,596.8. What would be the
compound interest accrued on the same simple interest accrued on the same amount
amount at the same rate in the same period ? at the same rate in the same period ?
(1) Rs. 11,550 (2) Rs. 12,620 (1) Rs.5,420 (2) Rs.5,360
(3) Rs. 10,950 (4) Rs. 11,900 (3) Rs.5,280 (4) Rs.5,140
(5) Rs. 13,500 (5) None of these
(New India Assurance AO (United India Insurance
Exam. 25.10.2009) AO Exam. 26.05.2013
6. A sum of money lent out at compound 10. The difference between the compound
interest increases in value by 50% in 5 interest and simple interest for the amount
years. A person wants to lend three different Rs. 5,000 in 2 years is Rs. 32. The rate of
sums x, y and z for 10, 15 and 20 years interest is
respectively at the above rate in such a way (1) 5% (2) 8%
that he gets back equal sums at the end of (3) 10% (4) 12%
their respective periods. The ratio x : y : z (NICL (GIC) AO (Finance)
is Exam. 15.12.2013)
(1) 6 : 9 : 4 (2) 9 : 4 : 6
(3) 9 : 6 : 4 (4) 6 : 4 : 9
(New India Assurance AO
Exam. 25.10.2009)

LEARN MATHS FROM S.K. RAJU (9811549822, 9811649822)


5
SHORT ANSWERS 35500 15  2
NATIONALISED BANKS = = Rs. 10650
100
& IBPS PO/MT/SO
Principal for another investment
1. (2) 2. (4)
= 35500 + 10650 = Rs. 46150
3. (3) 4. (4)
5. (3) 6. (4)  20 
3

7. (1) 8. (2)  C.I. = Rs. 46150  100   1
1 
9. (3) 10. (4)  
11. (4) 12. (1)
 6 3 
13. (5) 14. (5)
= Rs. 46150  5   1
15. (1) 16. (4)   
17. (3) 18. (1)
19. (4) 20. (2)  216  125 
= Rs. 46150  
21. (2) 22. (1)  125 
23. (4)
46150  91
= Rs.
SBI PO EXAMS 125
1. (4) 2. (1) = Rs. 33597.20
3. (1) 4. (4) Total interest earned
5. (2) 6. (1) = Rs (10650 + 33597.20) = Rs. 44247.20
7. (2) 3. (3) Let the rate p.c. p.a. be r.
6216  100
RBI GRADE—B OFFICER EXAMS r = = 14
14800  3
1. (1)
 r 
3

INSURANCE EXAMS  C.I = P  100   1
 1 
 
1. (5) 2. (1)
3. (3) 4. (3)  14 
3

5. (1) 6. (3) = Rs. 14800  100   1
 1 
  
7. (2) 8. (1)
9. (3) 10. (2)
= Rs. 14800 1.14   1
3

EXPLANATIONS = Rs. 14800 (1.481544- 1)


NATIONALISED BANKS = Rs. 14800 × 0.481544
& IBPS PO/MT/SO = Rs. 7126.8512

15000  9  2 SI  100
1. (2) SI = = Rs. 2700 4. (4) Rate = Principal × Time
100

 8 
2
 12000  100
= = 10%
CI = 12000  100   1
1  40000  3
  
 Rate Time 
 27    CI = Principal 1 100  1
2

= 12000  25   1  
  
 10 
3

 729  625  = 40000 1  100   1
= 12000     
 625 
3
= 40000 [(1.1) - 1]
104 = 40000 (1.331 - 1)
= 12000 × = Rs. 1996.8
625 = 40000 × 0.331 = Rs. 13240
 Total interest earned 5. (3) Let the rate of interest be R% per
= Rs. (2700 + 1996.8) = Rs. 4696.8 annum.
2. (4) Simple interest

LEARN MATHS FROM S.K. RAJU (9811549822, 9811649822)


6
 R 
T
 R 11 1

 CI = P  100   1

1 = -1=
100 10 10
 
1
 R 
2
  R = 100 × 10 = 10%
 5596.8 = 22000   1    1
 100   Principal × Time × Rate
 S.I =
2 100
5596.8  R 
 = 1   1 25500  3  10
22000  100 
= = Rs. 7650
2
100
 R  5596.8
 1   = 1
 100  22000  R 
T

7. (1) CI = P 1  100   1
 R 
2
22000  5596.8 27596.8   
 1   = =
 100  22000 22000  15 
4

= 20000  100   1
 1 
  
2
 R  275968 12544
 1   = =
 100  220000 10000
 3 
4

R 12544 112 = 20000  20   1
1 
1+ = =   
100 10000 100
 23  4 
112  110
= 20000  20   1
R 112 12
 = -1= =   
100 100 100 100
 R = 12%
 279841  160000 
Principal × Time × Rate = 20000  
 SI =  160000 
100
119841
22000  2  12 = 20000 × = Rs 14980.125
= = Rs 5280 160000
100
Time
6. (4) Amount  Rate 
8. (2) Amount = Principal  1  
= Rs. (25500 + 8440.5)  100 
= Rs. 33940.5
10   20 
2

= 20000  1    1  100 
T
 R 
A = P 1    100   
 100  (Rate of interest for the first year
 R 
3 = 10% , Time = 2 half years)
 33940.5 = 25500  1  
 100   11 11 6 
= Rs.  20000     = Rs. 29040
3  10 10 5 
33940.5  R 
 = 1    C.I. = Rs. (29040 - 20000) = Rs. 9040
25500  100  9. (3) Interest is compounded half yearly.
3  R = 20% p.a. = 10% /half year
339405  R 
 = 1   T = 2 years = 4 half years
255000  100 
 R 
T

 C.I = P  100   1
 
3
1331  R   1 
 = 1   
1000  100 
3 3  10 
4

 11   R 
   = 1   = 10000  100   1
 1 
 10   100    

R 11  11 4 
1+
= 10000  10   1
=
100 10
  
LEARN MATHS FROM S.K. RAJU (9811549822, 9811649822)
7
 121   121    Rate Time 
 1   1 
= 10000 
 100   100    CI = P 1 100  1
 
221 21
= 10000 × × = Rs.4641  5 
2

100 100 = 10000 1  100   1
  
 R 
T

10. (4) C.I. = P  100   1
1 
 21  2 
  
= 10000 ×  20   1
  
 13.5  2 
= 7400 1  100   1 41
   = 10000 × = Rs. 1025
400
= 7400 1.135  1
2
 R 
T

14. (5) A = P  100   1
1 
 
= 7400 × 0.288225 = Rs. 2132.87

 R 
T

11. (4) C.I. = P  100   1
 1   15 
3

   = 2600   1    1
 100  
 15 
2

= 45400  100   1

 1 4167
= 2600 × = Rs. 1354.275
   8000
P R T
= 45400 1.15   1
2
Simple interest =
100
= 45400 × 0.3225 = Rs. 14641.5
T
2600  15  3
 R  = = Rs. 1170
12. (1) Amount = P  1  
100
 100   Difference = Rs. (1354.275 - 1170)
2 = Rs. 184.275  Rs 184
 R 
 5800 + 594.5 = 5800  1   15. (1) Compound interest
 100   T

R 
2 = P  100   1
1 
63945  R    
 = 1  
58000  100 
2  5 
2

441  R    1    1
 = 1   = 35000
 100  
400  100 
2
 21   R 
2 41
   = 1   = 35000 ×
400
= Rs.3587.5
 20   100 
16. (4) Principal
R 21
1+ = SI  100 2000  100
100 20 = = = Rs 10000
Time × Rate 5 4
1
R= × 100  R 
T

20
 C.I = P  100   1
1 
= 5% per annum.  
SI  100
13. (5) Principal =  4 
2

Time  Rate = 10000 1  100   1
  
1000 100
= = Rs. 5000
45  26 2 
Case II = 10000  25   1
  
Principal = Rs. 10000

LEARN MATHS FROM S.K. RAJU (9811549822, 9811649822)


8
 676   21 2 
= 10000   1
 625  = 10000  20   1
  
10000  51
= = Rs 816  441 
625 = 10000   1
 400 
2
 r   300  r 
17. (3) CI - SI for 3 years = P     10000  41
 100   100  =
400
= Rs. 1025
2
 r   300  r  S.I × 100
 2994.134 = 2600     20. (2) Rate = Principal × Time
 100   100 

2994.134 300  r  r 2 =
10800  100
= 12% per annum
 = 22500  4
26000 1000000
2994134000  R 
T

 = 300r2 + r3  CI = P  100   1
 1 
26000  
 115159 = 300r2 + r3
Option (3) r = 19% satisfies the equation.  12 
2

= 22500  100   1
1 
SI ×100   
18. (1) Principal =
Time × Rate
 28 2 
6500  100 = 22500  25   1
= = Rs. 6250   
8  13

 T
  784  625 
R  = 22500  
CI = P  100   1
 1   625 
  
22500  159
 2
 = = Rs. 5724
8  625
= 6250  100   1
1 
   PR2
21. (2) Difference =
100 
2
 27  2 
= 6250  25   1
   7300  36
= = Rs. 26.28
10000
 729  625  22. (1) Let the principal be Rs. x, then
= 6250  
 625  3
 15  650052
6250  104 x 1   -x=
= = Rs 1040  100  100
625
23 23 23 650052
x× × × - x =
20 20 20 100
S.I × 100
19. (4) Principal = 650052  8000
Time × Rate
 12167x - 8000x =
100
7200  100
= = Rs 10000 650052  8000
6  12
x= = 156 × 80
100  4167
 R 
T

 C.I = P  100   1

 1 = Rs. 12480
 
 R 
T

23. (4) C.I. = P  100   1
 1 
 5 
2
   
= 10000  100   1
 1 
  

LEARN MATHS FROM S.K. RAJU (9811549822, 9811649822)


9
No. of periods = 2
 10 
2

 Amount
= 7790  100   1
1 
   2
 5 
 121   = 15000  1  
 1   100 
= 7790 
 100   21 21
= 15000 × ×
7790  21 20 20
= = Rs. 1635.90 = Rs. 16537.50
100
3. (1)
SBI PO EXAMS  12 
3

1. (4) Let the principal sum be Rs. P CI. = 25000  100   1
1 
  
PRT
SI =    
T
100 R 
 C.I.  P 1    1 
 nt
   100   
R 
CI = P  100   1
1 
= 25000 {(1.12)3 - 1}
  
= 25000 (1.404928- 1)
R = 10% , T = 2 years, n = 2 = 25000 × 0.404928
because CI is compounded half = Rs. 10123.2
P  10  2 P
SI = =  R 
T

100 5 4. (4) C.I. = P  100   1
1 
  
 10 
22

CI = P  100  2   1
 1   6 
3

  
= 41250  100   1
 1 
  
 21 4 
CI = P  20   1  148877  125000 
   = 41250  
 125000 
 194481 
CI = P   1 23877
160000  = 41250 ×
125000
= Rs 7879.41
34481  Rs. 7879
CI = P
16000 S.I.100
5. (2) Rate= Principal  Time
34481 P
CI - SI = P
160000 5 7200  100
= = 12% Per annum
34481  32000 20000  3
= P
160000
 R 
T

2481  C.I. = P  100   1
1 
=
160000
P  

2481  12 
2

 16000 P = 124.05 = 20000  100   1
 1 
  
124.05 160000
= 20000 1.12   1
3
P=
2481
= Rs. 8000 = 20000 × (1.404928 - 1)
2. (1) Interest is compounded half yearly. = Rs. 8098.56
Hence rate of interest 6. (1) Let the principal be Rs. P.
According to the question,
10
= = 5%
2

LEARN MATHS FROM S.K. RAJU (9811549822, 9811649822)


10
 
2
 105 
T
R 
C.I. = P  100   1
1  = 8000  
 100 
  
105 105
 15 
3
 = 8000 × × = Rs.8820
 6500.52 = P   1    1 100 100
 100   2. (1) Interest is compounded half yearly.
 6500.52 = P [1.520875 - 1] Hence rate of interest
10
6500.52 = = 5%
P= = Rs.12480 2
0.520875
No. of periods = 2
 R 
T
  Amount
7. (2) CI = P  100   1
 1 
    5 
2

= 15000  1  
 100 
 8 
2

 1414.4 = P  100   1
1  21 21
  = 15000 × × = Rs. 16537.50
20 20
 1414.4 = P × 0.1664 3. (3) Let the sum be Rs. x. Then,
1414.4 3
P = = Rs. 8500  R 
0.1664 x 1   =13380 ...(i)
 100 
 Amount = Rs. (8500 + 1414.4)
= Rs. 9914.40 and,
6
 R 
RBI GRADE-B OFFICER EXAMS x 1   = 20070 ...(ii)
 100 
1. (1) If the principal be Rs. P, then
Dividing equation (ii) by (i),
 R 
T
 3
 R 
C.I = P  100   1

 1 20070 3
1   = =
    100  13380 2
From equation (i),
 12 
2

 2862 = P  100   1
1  3
  x ×
2
= 13380

 28 2  2
 x = 13380 ×
 2862 = P  25   1
= 8920
3
 
Hence, the sum is Rs. 8920.
 784  625  4. (3) Amount
 2862 = P ×  625 
   Rate 
Time

= Principal  1  
159  100 
 2862 = P ×
625
 1 
1

5600  1  10  
2
2862  625
P = = Rs. 11250 = Rs.   100  
159  
INSURANCE EXAMS
1. (5) P = 8000   1 
 10  
t = 2 years  10   2
5600  1    1 
r = 5% = Rs.   100  

100  
t    
 r 
A = P 1  
 100   11 21 
= Rs.  5600    = Rs. 6468
2  10 20 
 5 
= 8000  1    Compound interest
 100 
= Rs. (6468 - 5600) = Rs. 868

LEARN MATHS FROM S.K. RAJU (9811549822, 9811649822)


11
SI  100  5 25 125 
5. (1) Rate =  91000 = x  6  36  216 
Principal × Time  
10230  100  180  150  125 
= = 12.4%  91000 = x  
27500  3  216 
 R 
T
 x  455
 C.I = P 1    1  91000 =
 100   216
91000  216
 12.4 3  x = = Rs. 43200
= 27500 1  100   1
455
   9. (3) Let the rate of interest be R% per
 27500 (1.42- 1) annum.
= 27500 × 0.42  T

R 
= Rs. 11550  CI = P  100   1
  1  
6. (3) Tricky approach  
According to the question,
 R 
2

 5596.8 = 22000  100   1
 
2 3 4
3  3 3 1
  x =   y =   z = k let  
2  2 2
2
2 3 5596.8  R 
2  2  = 1   1
 x =   k, y =   k 22000  100 
3  3
2
4  R  5596.8
2  1   = 1 + 22000
and z =   k  100 
3
2
2 3 4  R  22000  5596.8
2  2 2  1   =
x : y : z =   k :   k :   k  100  22000
3  3 3
2
2 2
2  R  27596.8
:   =9:6:4  1   = 22000
=1:
3 3  100 

R 12544 112
7. (2) P = S.I×100 1+ = =
100 10000 100
Rate × Time
6400  100 R 112 112  110 12
=  = -1= =
8 4 100 100 100 100
= Rs. 20000  R = 12%
 Rate Time  Principal × Time × Rate
C.I = P 1  100   1  SI = 100
  
22000  2  12
 2 
2
 = = Rs. 5280
= 20000  100   1

1 100
   10. (2) Difference
= 20000 × 0.0404 = Rs. 808  Rate 2 
8. (1) If each instalment be Rs. x,  Principal  10000 
then.  
91000 =
 r2 
x x x  32 = 5000  10000 
+ 2 + 3
 
r  r   r 
1  r2 = 32 × 2 = 64
100 1   1  
 100   100 
 r = 64 = 8% per annum
 5  5 2  5 3 
 91000 = x  6   6    6  

LEARN MATHSFROM S.K. RAJU (9811549822, 9811649822)
12
MODEL EXERCISES
1. A sum of money becomes eight times in 3 sum of money for 2 years at 15% per annum
years, if the rate is compounded annually. is Rs 45. Find the sum.
In how much time the same amount at the (1) Rs 7000 (2) Rs 2750
same compound interest rate will become (3) Rs 2000 (4) Rs 2250
sixteen times ? (5) None of these
(1) 6 years (2) 4 years 8. Two friends A and B jointly lent out Rs
(3) 8 years (4) 5 years 81600 at 4% per ann um co mpou nd
(5) None of these interest. After 2 years, A gets the same
2. If the difference between the simple and the amount as B gets after 3 years. The
compound interests on same principal investment made by B was
amount at 20% for 3 years is Rs 48, then (1) Rs 40000 (2) Rs 30000
the principal amount must be (3) Rs 45000 (4) Rs 38000
(1) Rs 650 (2) Rs 600 (5) None of these
(3) Rs 375 (4) Rs 400 9. A sum o f mone y is accumulatin g at
(5) None of these compound interest at a certain rate of
3. The difference between the simple interest interest. If simple interest instead of
and the compound interest compounded compound were reckoned, the interest for
annually at the rate of 12% per annum on the first two years would be diminished by
Rs 5000 for two years will be Rs 20 and that for the first three years by
(1) Rs 17.50 (2) Rs 36 Rs 61. Find the sum.
(3) Rs 45 (4) Rs 72 (1) Rs 7000 (2) Rs 8000
(5) None of these (3) Rs 7500 (4) Rs 6500
4. Sanjay borrowed a certain sum from Anil (5) None of these
at a certain rate of simple interest for 2 yr. 10. Rs 5887 is divided between Shyam and
He lent this sum to Ram at the same rate Ram, such that Shyam’s share at the end
of interest compounded annually for the of 9 years is equal to Ram’s share at the
same period. At the end of two years, he end of 11 years, compound annually at the
received Rs 4200 as compound interest but rate of 5% . The share of Shyam is
paid Rs 4000 only as simple interest, find (1) Rs 2088 (2) Rs 2000
the rate of interest. (3) Rs 3087 (4) Rs 2200
(1) 15% (2) 20% (5) None of these
(3) 35% (4) 10% 11. Amit borrowed a certain sum of money for
(5) None of these 2 years at 8% per annum on simple interest
5. Under the Rural Housing Scheme, the Delhi and immediately lent it to Ravi but at
Development Authority (DDA) allotted a compound interest and gained by Rs 16.
house to Kamal Raj for Rs 126100. This What amount did Amit borrow ?
payment is to be made in three equal (1) Rs 1600 (2) Rs 2500
annu al instalments. If the mone y is (3) Rs 24000 (4) Rs 1800
reckoned at 5% per annum compound (5) None of these
interest, then how much is to be paid by 12. A sum of Rs 30600 is divided between Anjali
Kamal Raj in each instalment ? and Arun, who are respectively 18 and 19
(1) Rs 45205 (2) Rs 47405 years old, in such a way that if their shares
(3) Rs 46305 (4) Rs 48505 are invested at 4% per annum compounded
(5) None of these annually, they shall receive the same
6. Ram takes a loan Rs 10000 and pays back amount on reaching 21 years of age. What
Rs 13310 after 3 years. The compound is the share of Anjali ?
in te re st rate per ann um w il l be (1) Rs 16000 (2) Rs 15000
approximately. (3) Rs 15600 (4) Rs 14600
(1) 8% (2) 9% (5) None of these
(3) 10% (4) 11% 13. A man borrows Rs 4000 from a bank at
(5) None of these 1
7. The difference between the compound 7 % compound interest. At the end of
2
interest and the simple interest on a certain
every year, he pays Rs 1500 as part

LEARN MATHS FROM S.K. RAJU (9811549822, 9811649822)


13
repayment of loan and interest. How much (1) 4 years (2) 2 years
does he still owe to the bank after three (3) 6 years (4) 8 years
such instalments ? (5) None of these
(1) Rs 123.25 (2) Rs 125 16. Equal sums of money are deposited in two
(3) Rs 400 (4) Rs 469.18 different banks by M/s Enterprise, one at
(5) None of these compound interest, compounded annually
14. Sanju puts equal amounts of money one and the other at simple interest, both at
at 10% per annum compound interest 5% per annum. If after two years, the
payable half yearly and the second at a difference in the amounts comes to Rs 200,
certain per cent per annum compound what are the amounts deposited with each
interest payable yearly. If he gets equal bank ?
amounts after 3 years, what is the value of (1) Rs 72000 (2) Rs 64000
the second per cent ? (3) Rs 80000 (4) Rs 8400
(5) None of these
1
(1) 10 % (2) 10% 17. A sum of money lent at compound interest
4 for 2 years at 20% per annum would fetch
1 1 Rs 482 more if the interest was payable half
(3) 9 % (4) 8 % yearly than if it was payable annually. The
2 4
(5) None of these sum is
15. Ravi Sh ankar take s mo ney from the (1) Rs 10,000 (2) Rs 20,000
employees cooperative society at lower rate (3) Rs 40,000 (4) Rs 50,000
of interest and saves in a scheme, which (5) None of these
gives him a compound interest of 20% ,
compounded annually. Finally the least
number of complete years after which his
sum will be more than double.

LEARN MATHS FROM S.K. RAJU (9811549822, 9811649822)


14
SHORTANSWERS 100  5
1. (2) 2. (3) Principal = × 48 = Rs 375
64
3. (4) 4. (4)
5. (3) 6. (3)  r 
2
7. (3) 8. (1) 3. (4) CI - SI for 2 years = P  
9. (2) 10. (3)  100 
11. (2) 12. (2) 2
13. (1) 14. (1)  12 
= 5000  
15. (2) 16. (3)  100 
17. (2)
33
= 5000 × = Rs 72
EXPLANATIONS 25  25
1. (2) Let the sum of money be Rs x. 4. (4) Let the money borrowed be Rs x and
3
rate be r% .
 r  and Time = 2 yr
Then. 8x = x  1  
 100  xr 2
3  4000 = 100
 r 
8 = 1    rx = 200000
 100 
2
3  r 
 r  and x  1   = x + 4200
 23 =  1    100 
 100 
3 xr 2 2 xr
 r  x + + = 4200 + x
 1   = 23 10000 100
 100 
 20r + 4000 = 4200
r  r = 10%
1+ =2
100 5. (3) Let the amount of each instalment be
Again, let the sum will become 16 times in Rs x.
n years Then,
Then,
x x x
 r 
n 126100 = + 2 + 3
r  r   r 
 16x = x  1   1 1  100  1  
 100  100    100 
 16 = 2n  24 = 2n
n = 4 years  20  20  2  20 3 
2. (3) Let principal amount be Rs 100.  126100 = x  21   21    21  
 
100  20  3
Then, S.I. = = Rs 60
100 20  20 400 
 126100 = x × 1  21  441 
3 21  
 20 
and C.I. = 100  1   - 100
 100  20  441  420  400 
 126100 = x ×  
3 21  441 
6 364
= 100 ×   -100 =
5 5 20 1261
 126100 = x × ×
21 441
364 64
 C.I. - S.I. = 5
- 60 =
5 126100  21 441
 x= = Rs 46305
20  1261
64
If differences is Rs , Principal 6. (3) Let the rate of compound interest be r.
5 3
= Rs 100  r 
Then, 10000 ×  1   =13310
If difference is if 48,  100 

LEARN MATHS FROM S.K. RAJU (9811549822, 9811649822)


15
3 9 11
 r  1331  5   5 
 1   = 1000 Then, x 1   = (5887 - x) 1  
 100   100   100 
3 3
 r   11  x
 1   =    = 1.1025
 100   10  5887  x
 x = 5887 × 1.1025 - 1.1025x
r 11
1+ =  2.1025x = 5887 × 1.1025
100 10
 x = Rs 3087
r 1 11. (2) Let Amit borrow Rs x.
 = According to the question,
100 10
 r = 10%  r 
2

CI - SI for 2 years = P  
 r 
2
 100 
7. (3) CI - SI for 2 years = P  
 100  2
 8 
 16 = x  
 15 
2
 100 
 45 = P  
 100  16  10000
x= = Rs 2500
45  100 100 64
 P= = Rs 2000 12. (2) Let the share of Anjali be x.
15  15
8. Let the investment made by  Then, the share of Arun is
(30600 - x).
B=x
3 2
Then, investment made by  4   4 
A = (81600 - x) x × 1   = (30600 - x)  1  
 100   100 
2 3
 4   4 
 (81600 - x)  1  100  = x  1  100  x×
104
    100
= 30600 - x
 81600 - x = 1.04x
204
81600  x = 30600
x= = Rs. 40000 100
2.04
9. (2) Let the principal be P and rate of interest 30600 100
x= = Rs 15000
be r% . Then, principal (when difference 204
between C.I. and SI is for 2 years) is given 13. (1) Amount remaining after
by
 7.5 
1 year = 4000  1   - 1500 = Rs 2800
20  100 
2
 100 
P= .......(i)
r2
 7.5 
and difference between CI and SI is for 3 2 years = 2800  1   - 1500 = Rs 1510
years is given by  100 

61  106  7.5 
3 years =1510  1   - 1500 = Rs 123.25
P=
r 2
 300  r  .......(ii)  100 
14. (1) Let the value of second rate of interest
From eqs. (i) and (ii), be x% and equal amounts be P each.
20  10 4 61106 6 3
 5   x 
r2
= r  300  r 
2
P × 1   = P × 1  
 100   100 
 r = 305 - 300 = 5%
 x 
2
 5 
20  10 4
 1   =  1  100 
From Eq. (i), P = = Rs 8000  100   
25
10. (3) Let Shyam’s share = x

LEARN MATHS FROM S.K. RAJU (9811549822, 9811649822)


16
105  105 100  x 441P 441P  400P 41P
 = = -P= =
100  100 100 400 400 400
 110.25 = 100 + x In IInd bank, we get
 = 10.25% P R T P 5 2 P
SI = = =
25 1 100 100 10
 x =10 = 10 %
100 4 41P P
15. (2) Let the sum of money be P.  100 - 100 = 200
 P = Rs 80000
T
 20 
 P  1  100   2P 17. (2) Let the sum be Rs. x.
 
4 2
T  10   20 
6 x 1   - x 1   = 482
   2  100   100 
5
 (1.2) T  2  11 
4
6
2

Hence, least value of T is 2 yr.  x   - x   = 482


 10  5
16. (3) Let P be the amount deposit in two
different banks.  x (1.4641 - 1.44) = 482
In 1st bank,  0.024lx = 482
2 482
 5  x= = 20000
CI = P  1   -P 0.0241
 100 
2
 21 
= P  - P
 20 

LEARN MATHS FROM S.K. RAJU (9811549822, 9811649822)


1
DATA SUFFICIENCY
NATIONALISED BANKS II. It takes 4 hours to cover distance
& IBPS SO/MT/SO between A and B upstreams.
Directions (1-5): Each of the questions Directions (6-10) : Each of the questions
below consists of a question and two statements below consists of a question and two statements
numbered I and II given below it. You have to numbered I and II given below it. You have have to
decide whether the data provided in the statements decide whether the data provided in the statements
are sufficient to answer the question. Read the are sufficient to answer the question. Read the
question and both the statements and— question and both the statements and.
Give answer (1) if the data in statement I Give answer (1) if the data in statement I
alone are sufficient to answer the question, while alone are sufficient to answer the question, while
the data in statement II alone are not sufficient the data in statement II alone are not sufficient
to ansswer the question. to answser the question.
Give answer (2) if the data in statement II Give answer (2) if the data in statement II
alone are sufficient to answer the question, while alone are sufficient to answer the question, while
the data in statement I alone are not sufficient the data in statement I alone are not sufficient
to answer the question. to answer the question.
Give answer (3) if the data either in Give answer (3) if the data either in
statement I alone or in statement II alone are statement I alone or in statement II alone are
sufficient to answer the question. sufficient to answer the question.
Give answer (4) if the data even in both Give answer (4) if the data in both the
the statements I and II together are not sufficient statements I and II together are not sufficient to
to answer the question. answer the question.
Give answer (5) if the data in both the Give answer (5) if the data in both the
statements I and II together are necessary to statements I and II together are necessary to
answer the question. answer the question.
Canara Bank PO Exam. 09.02.2003) (Syndicate Bank PO Exam. 10.10.2004)
1. Train ‘A’ running at a certain speed crosses 6. Wh at i s th e an nu al i ncome of M r.
another train ‘B’ running at a certain speed Krishnamurthy form April 2001 to March
in the opposite direction in 12 seconds. 2002 ?
What is the length of train ‘B’ ? I. The annual income of Mr. Krishnamurthy
I. The length of both the trains together is is 70% more than that of his boss.
450 metres. II . In A pril 2001 the i ncome of
II. Train ‘A’ is slower than train ‘B’. Krishnamurthy was Rs. 12000 and his
2. Area of a rectangle is equal to the area of a income increases by 10% per month.
right angled triangle. What is the length of 7. What is the area of circular field ?
the rectangle ? I. The circumference of the field is x metres.
I. The base of the triangle is 40 cms. II. The diameter of the field is y metres.
II. The height of the triangle is 50 cms. 8. What is the speed of boat in still water ?
3. What was the total compound interest on a I. The boat covers a distance of 35 kms in 5
sum after three years ? hours in downstream.
I. The interest after one year was Rs. 100/ II. The boat takes 7 hours to cover the same
and the sum was Rs. 1,000/-. distance in upstream.
II. The difference between simple and 9. How many students are there in the class ?
compound interest on a sum of Rs. I. The ratio between the number of boys
1,000/- at the end of two years was Rs. and girls is 5 : 6 respectively.
10/-. II. The difference between the numbers of
4. What is the two digit number where the boys and girls is 7.
digit at the unit place is smaller ? 10. What is the amount of earned profit ?
I. The difference between the two digits is 5. I. On selling the article for Rs. 1740, 20%
II. The sum of the two digits is 7. profit is earned.
5. What is the speed of the boat in still water ? II. The cost price ofthe article is Rs. 1400.
I. It takes 2 hours to cover distance between Directions (11-15) : In each of the following
A and B downstreams. questions, a question is followed by information
given in three statements. You have to study the
LEARN MATHS FROM S.K. RAJU (9811549822, 9811649822)
2
question alongwith the statements and decide, the III. An amount doubles itself in 5 years with
information given in which of the statement(s) is simple interest.
necessary to answer the question. (1) All the three
(Corporation Bank PO Exam. 29.07.2006) (2) Only III
11. In how many days can 16 men and 8 women (3) Either II or III
together complete the piece of work ? (4) Only III or I and II
I. 8 men complete the piece of work in 10 (5) Question cannot be answered even with
days. the information in all three statements
II. 16 women complete the piece of work in Directions (16-20): Each of these questions
10 days. consists of a question followed by information in
III. 5 women take 32 days to complete the three statements. You have to study the question
piece of work. and the statenents and decide that information in
(1) Only I and II which of the statements is/are necessary to answer
(2) Only I and III the question.
(3) Only II and III (Indian Overseas Bank PO
(4) Only I and either II or III Exam. 15.06.2008)
(5) Any two of the three 16. What is the capacity of the cylindrical tank ?
12. What is the speed of the train ? I. Radius of the base is half of its height ?
I. Train crosses a pole in 10 seconds II. Area of the base is 616 square metres.
II. Length of the train is 240 metres. III. Height of the cylinder is 28 metres.
III. Train crosses a platform of equal length (1) I and II Only
in 20 seconds. (2) II and III Only
(1) Only I and II (3) I and III Only
(2) Only II and III (4) All 1, II and III
(3) All I, II and III (5) Any two of the three
(4) Any two of the three 17. What is the speed of the train ?
(5) II and either I or III I. The train crosses a signal pole in 18
13. What is the area of the square ? seconds.
I. Measure of diagonal of the square is given. II. The train crosses a platform of equal
II. Measure of one side of square is given. length in 36 seconds.
III. Perimeter of the square is given. III. Length of the train is 300 metres.
(1) Only II (1) I and III Only
(2) Only III (2) II and III Only
(3) Only I and III (3) I and II Only
(4) Only II and III (4) III and either I or II only
(5) Any one of the three (5) Any two of the three
14. What is the two digit number ? 18. What is the staff strength of company ‘X’ ?
I. The number obtained by interchanging I. Male and female employees are in the
the digits of the number is greater than ratio of 2 : 3 respectively.
the original number by 18. II. Of the officer employees 80% are males.
II. Sum of the two digits of the number is III. Total number of officers is 132.
14. (1) I and III only
III. Difference between the two digits of the (2) II and either III or I only
number is 2. (3) All I, II and III
(1) Any two of the three (4) Any two of the three
(2) Only I and III (5) Question cannot be answered even with
(3) II and either I or III the information in all three statements
(4) All the three 19. What is the two digit number ?
(5) III and either I or II I. Number obtained by interchanging the
15. What is the rate of interest p.c.p.a. ? digits is more than the original number
I. Simple interest earned per annum is Rs. by 9.
5,300 II. Sum of the digits is 7.
II. The difference between the compound III. Difference between the digits is 1.
and simple interest on an amount is Rs. (1) 1 and III only
1,060 at the end of 2 years. (2) I and II only

LEARN MATHS FROM S.K. RAJU (9811549822, 9811649822)


3
(3) II and III only 25.
What is the profit earned by selling a printer
(4) All I, II and III for Rs. 3,000 ?
(5) Question cannot be answered even with A. The cost price of 6 such printers is equal
the information in all three statements to selling price of 5 such printers.
20. How many articles were sold ? B. 20% profit is earned by selling each
I. Total profit earned was Rs. 1,596 printer.
II. Cost-price per article was Rs. 632 Directions (26-30): Each of the questions
III. Selling price per article was Rs. 765
below consists of a question and two statements
(1) II and III only numbered I and II are given below it. You have to
(2) I and II only decide whether the data provided in the statements
(3) All I, II and III are sufficient to answer the question. Read both
(4) Any two of the three the statements and
(5) Question cannot be answered even with Give answer (1) if the data in Statement I
the information in all three statements
alone are sufficient to answer the question, while
Directions (21-25): Each question below
the data in Statement II alone are not sufficient to
is followed by two statements A and B. You have
answer the question.
to detemine whether the data given in the Give answer (2) if the data in Statement II
statement is sufficient for answering the question.
alone are sufficient to answer the question, while
You should use the data and your knowledge of the data in Statement I alone are not sufficient to
Mathematics to choose between the possible answer the question.
answers. Give answer (3) if the data either in
Give answer (1) if the statement A alone is
Statement I alone or in Statement II alone are
sufficie nt to answ er the question, but the sufficient to answer the question.
statement B alone is not sufficient. Give answer (4) if the data in both the
Give answer (2) if the statement B alone is
Statements I and II are not sufficient to answer
sufficie nt to answ er the question, but the the question.
statement A alone is not sufficient. Give answer (5) if the data in both the
Give answer (3) if both the statements A
Statements I and II together are necessary to
and B together are needed to answer the question.
answer the question.
Give answer (4) if either the statement A (Bank Of Baroda Specialist
alone or statement B alone is sufficient to answer Officer Exam. 05.10.2008)
the question. 26. What is the ratio of the number of freshers
Give answer (5) if you cannot get the to the number of seniors in a college ?
answer from the statements A and B together, but I. The ratio of males and females in the
need even more data. college is 2 : 3.
(Andhra Bank PO Exam. 14.09.2008) II. There are 1125 female freshers in the
21. Is A an odd number ? college.
A. A multiplied by an odd number is equal
27. What is Nidhi’s age ?
to an odd number. I. Nidhi is 3 times younger to Rani.
B. A is not divisible by 2. II. Surekha is twice the age of Rani and the
22. The ages of Pradumn and Gunit are in the sum of their ages is 72 years.
28.
ratio of 7 : 5 . What is the age of Pradumn. What is the ratio of the total number of girls
A. The ages of Pradumn and Nandini are in to the total number of boys in the school ?
the ratio of 3 : 1. I. The ratio of the total number of boys to
B. After 7 years the ratio of Pradumn’s and the total number of girls, last year was
Aviral’s ages will be 4 : 3. 4 : 5.
23. What is the salary of B, in a group of A, B, II. There are 3500 students in the school
C and D whose average salary is Rs. 62,880 ? out of which 60% are boys.
A. Total of the salary of A and C is exact
29. What is Mr. Mehta’s present income ?
multiple of 8. I. Mr. Mehta’s income increases by 10%
B. Average of the salary of A, C and D is Rs. every year.
61,665. II. His income will increase by Rs. 2500 this
24. What is the three digit number ? year.
A. The three digit number is divisible by 9.
30. What is the speed of the bus ?
B. The first and the third digit is 6. I. The bus covers a distance of 80 kms. in 5
hrs.
LEARN MATHS FROM S.K. RAJU (9811549822, 9811649822)
4
II. The bus covers a distance of 160 kms. Give answer (1) if the statement A alone is
in 10 hrs. sufficie nt to answ er the question, but the
Directions (31-35): Each question belowstatement B alone is not sufficient.
is followed by two statements A and B. You are to Give answer (2) if the statement B alone is
determine whether the data given in the statement
sufficie nt to answ er the question, but the
is sufficient for answering the question. You should
statement A alone is not sufficient.
use the data and your knowledge of Mathematics Give answer (3) if both statements A and
to choose between the possible answers. B together are needed to answer the question.
Give answer (1) if the statement A alone is Give answer (4) if either the statement A
sufficie nt to answ er the question, but the alone or statement B alone is sufficient to answer
statement B alone is not sufficient. the question.
Give answer (2) if the statement B alone is Give answer (5) if you cannot get the
sufficie nt to answ er the question, but the answer from the statement A and B together, but
statement A alone is not sufficient. need even more data.
Give answer (3) if both statements A and (Canara Bank PO Exam. 15.03.2009)
B together are needed to answer the question. 36. How many women can complete a piece of
Give answer (4) if neither the statement A work in 15 days ?
alone or statement B alone is sufficient to answer A. If 12 women can complete the same piece
the question. of work in 20 days.
Give answer (5) if you cannot get the B. If 10 men can complete the same piece
answer from the statement A and B together, but of work in 12 days.
need even more data. 37. What is the rate of interest p.c.p.a. on an
(Oriental Bank of Commerce amount of Rs. 12,000 deposited in a Bank ?
PO Exam. 21.12.2008) A. The difference between the simple
31. Is X an even number ? interest and compound interest is Rs.
A. X is divisible by 2. 172.8.
B. X + an odd number is an odd number. B. The simple interest for two years is Rs.
32. What is the three digit number ? 2,880.
A. One-third of that number is less by 27
38. What is the three digit number ?
of the half of that number. A. The three digit number is exact multiple
B. One-fifth of that number is 20% of that of 13.
number. B. The first and third digit are 7.
33. 39.
The ages of Tanish and Shivay are in the What is the age of C, in a group of A, B, C,
ratio of 6 : 5. What is the age of Shivay ? D and E whose average age is 45 years ?
A. The ages of Tanish and Danish are in A. Average of the ages of A and B is 53 years.
the ratio of 3 : 2. B. Average of the ages of D and E is 47 years.
B. After 6 years the ratio of Danish’s and
40. What is the profit earned by selling a Laptop
Shivay’s ages will be 6 : 7. of Rs. 26,250 ?
34. In how many days 15 men can complete a A. The cost price of 5 such Laptops is equal
piece of work ? to selling price of 4 such Laptops.
A. If 16 children can complete the same B. 25% profits is earned by selling each
piece of work in 48 days. Laptop.
B. If 9 men can complete the same piece of Directions (41-42) : Each question below
work in 27 days. is followed by two statements A and B. You are to
35. What is the salary of C, in a group of A, B,
determine whether the data given in the statement
C, D, E and F whose average salary is Rs.
is sufficient for answering the question. You should
60,000 ? use the data and your knowledge of Mathematics
A. Total of the salary of A and E is Rs. 64,500
to choose between the possible answers.
B. Total of the salary of B and F is Rs. 52,600 Give answer (1) if the Statement alone is
Directions (36 - 40): Each question below
sufficie nt to answ er the question, but the
is followed by two statements A and B. You are to
Statement B alone is not sufficient.
determine whether the data given in the statement Give answer (2) if the Statement B alone is
is sufficient for answering the question. You should
sufficient to answer the luestion, but the Statement
use the data and your knowledge of Mathematics A alone is not sufficient.
to choose between the possible answers. Give answer (3) if both statements A and
B together are needed to answer the question.
LEARN MATHS FROM S.K. RAJU (9811549822, 9811649822)
5
;
Give answer (4) if either the statement A (1) Only I and II
alone or statement B alone is sufficient to answer (2) Only II
the question. (3) Only II and III
Give answer (5) if you cannot get the (4) Any two of three
answer from the statements A and B together, but (5) None of these
needed even more data. 48. In how many days will B alone complete
(Indian Overseas Bank PO the work ?
Exam. 05.04.2009) I. A and B together can complete the work
41. What is the speed of the car ? in 8 days.
A. The car covers a distance of 135 kms in II. B and C together can complete the work
3 hours. in 10 days.
B. The car covers a distance of 270 kms. in III. A and C together can complete the work
6 hours. in 12 days.
42. What is the value of the two digit number ? (1) Only I and II
A. The product of the digits is 72 and the (2) Only II and III
difference between the digits is 1. (3) All I, II and III
B. The digit at the unit’s place is greater (4) Question cannot be answerd even with
than the other. the information in all the three statements
43. What is the ratio of the total number of girls(5) None of these
49.
to the total number of boys in a college ? What is the rate of interest p.c.p.a. ?
A. There are 2500 students in the college I. An amount doubles itself at simple
out of which 40% are boys. interest in 10 years.
B. The ratio of the total number of boys to II. Difference between the compound
the total number of girls in the last year
interest and simple interest on an amount
was 5 : 6. of Rs 15,000 in two years is Rs 150.
44. What is the number ? III. The compound interest accured in 8
A. 75% of that number is three- fourth of years is more than the amount (principal).
that number. (1) Only I
B. One-third of that number is less by 27 (2) Only II
of the half of that number. (3) Only II and III
45. What is Deepali’s age ? (4) Only I and III
A. Deepali is two times younger than Nisha. (5) Only either I or II
B. Supriya is twice the age of Nisha. 50. What are the marks scored by Abhijit in
Directions (46-50) : In each of these English ?
questions, a question is given followe d by I. Marks scored by Abhijit in Maths are more
information in three statements. You have to find than his marks in Science by 20.
out the data in which statement(s) is sufficient to II. Total marks scored by Abhijit in Maths,
answer the question and mark your answer Science and and English are 198.
accordingly. III. Marks scored by Abhijit in Science are
(United Bank of India PO more than his marks in English by 12.
Exam. 21.06.2009) (1) Any two of the three
46. What is the average age of the six members (2) Only II and III
A, B, C, D, E and F in the family ? (3) All I, II and III
I. Total age of D and E is 14 years. (4) Question cannot be answered even with
II. Average age of A, B, C and F is 50 years. the information in all the three statements
III. Average age of A, B, D and E is 40 years.(5) None of these
(1) Only I and II Directions (51-55): Each of the questions
(2) Only I and III below consists of a question and two statements
(3) Only II and III numbered I and II are given below it. You have to
(4) All I, II and III decide whether the data provided in the statements
(5) None of these are sufficient to answer question. Read both the
47. What is the area of the right angled triangle ?
statements and
I. Base of the triangle is X cms. Give answer (1) if the data in Statement I
II. Height of the triangle is Y cms. alone are sufficient to answer the question, while
III. Hypotenuse of the triangle is Z cms.
the data in Statement II alone are not sufficient to
answer the question.
LEARN MATHS FROM S.K. RAJU (9811549822, 9811649822)
6
Give answer (2) if the data in statement II Give answer (3) if the data in either
alone are sufficient to answer the question, while statement I alone or in statement II alone are
the data in Statement I alone are not sufficient to sufficient to answer the question
answer the question. Give answer (4) if the data in both the
Give answer (3) if the data in Statement I statements I and II are not sufficient to answer
alone or in Statement II alone sufficient to answer the question
the question. Give answer (5) if the data given in both
Give answer (4) if the data in both the the statements I and II are necessary to answer
Statements 1 and II are not sufficient to answer the question
the question. (Bank Of India Banking
Give answer (5) if the data in both the Officer Exam. 24.01.2010)
Statements I and II together are necessary to 56. What is the two digit number ?
answer the question. I. The difference between the two digits of
CAUTION: Do not mark your anawer unless the number is 0.
you consider both the statements carefully. II. The sum of the two digits of the number
(Andhra Bank PO Exam. 05.07.2009) is 18.
51. What is Nikita’s share in the profit of Rs. 57. What is Supriya’s present age ?
50,000 earned in the business run by her I. Supriya is three years older than Priya.
in partnership with Sharmila ? II. The ratio between Priya’s and Reshma’s
I. Nikita invested an amount 150% of the age is 3 : 4 respectively.
amount invested by Sharmila. 58. What is the speed of the boat in still water ?
II. Amount invested by Sharmila is two- I. The boat travels at the speed of 4 km/hr
third of the amount invested by Nikita. upstream
52. What is the two digit number ? II. The boat travels at the speed of 6 km/hr
I. Sum of the digits is 6. downstream.
II. Digit in the ten’s place is double the digit 59. How many marks did Nigam get in Biology ?
in the unit’s place. I. Nigam got 42 marks in English which
53. What is the rate of interest p.c.p.a. ? were half the marks he got in Biology
I. The amount doubles itself in 10 years. II. Nigam’s marks in Biology were 14 per
II. The simple interest accrued in 5 years is cent of the total marks he got in all the
Rs. 5,000. subjects together
54. In how many days can B alone complete 60. What is the rate of interest p.c.p.a. ?
the work ? I. An amount of Rs. 6200 fetches simple
I. B and C together can complete the work interest of Rs. 1736 in two years.
in 8 days. II . An amo un t of Rs. 4500 fetch es
II. A and B together can complete the work compound interest of Rs. 1348.2 in two
in 12 days. years.
55. What is the mother’s present age ? Directions (61-65): Each of the questions
I. Present ages of mother and child are in below consists of a question and two statements
the ratio of 5 : 1 respectively. numbered I and II given below it. You have to
II. Four years hence the ratio of the ages of decide whether the data provided in the statements
mo th er and chi ld wi ll be 17 : 5 are sufficient to answer the question. Read both
respectively. the statements and
Directions (56-60) : In each of the following Give answer (1) if the data in Statement I
questions, a question followed by two statements alone are sufficient to answer the question, while
numbered I and II are given. You have to read both the data in Statement II alone are not sufficient to
the statements and then answer the question.
Give answer (1) if the data given in Give answer (2) If the data in Statement II
statement I alone are sufficient to answer the alone are sufficient to answer the question, while
question whereas the data given in statement II the data in Statement I alone are not sufficient to
alone are not sufficient to answer the question. answer the question.
Give answer (2) if the data given in Give answer (3) if the data In Statement I
statement II alone are sufficient to answer the alone or in Statement II alone are sufficient to
question whereas the data given in statement I answer the question.
alone are not sufficient to answer the question.

LEARN MATHS FROM S.K. RAJU (9811549822, 9811649822)


7
Give answer (4) if the data in both the (Syndicate Bank PO Exam. 29.08.2010)
Statements I and II are not sufficient to answer 66. In how many days will five women complete
the question. a work ?
Give answer (5) if the data in both the I. Two women, five boys and three men
Statements I and II together are necessary to together complete the work in six days.
answer the question II. Eight women complete the same work
(Allahabad Bank PO Exam.21.02.2010) in twelve days.
61. What is the perimeter of a semicircle ? 67. What will be Shyam’s age after 6 years ?
I. The radius of the semicircle is equal to I. The ratio between Ram’s and Shyam’s
half the side of a square. present age is 4 : 3 respectively.
II. The area of the square is 196 sq.cm. II. Ram is seven years older than Shyam.
62. What is the exact average of n, 35, 39, 42, 68. What is the average speed of a car ?
p and w ? I. Average speed of the car is five times
I. n is six more than w. average speed of a truck whereas the
II. w is four less than p. average speed of a bus is 45 km/hr.
63. What was the percent profit/loss made/ II. Average speed of the truck is half the
incurred by selling an article for Rs.24,000 ? average speed of a train whereas the
I. The ratio between the selling price and average speed of a bus is 45 km/hr.
the cost price of the article is 5 : 3 69. What is the simple interest accrued on a
respectively. principal of Rs. 5,000 in six years ?
II. The difference between the cost price and I. The rate of the simple interest for the first
the selling price is Rs.9,600. three years is 5 p.c.p.a. ?
64. What will be the difference between two II. The rate of simple interest for the next
two-digit numbers ? three years is 8 p.c.p.a.?
I. The square of the first number is 9 times 70. What is the total cost of 4 kgs of apples and
the second number. 3 kgs of mangoes together ?
II. The ratio between the first number and I. Cost of 2 kgs of apples is Rs. 170 and one
the second number is 3 : 4 respectively. kg of mangoes is Rs. 50.
65. What is the ratio between two numbers x II. The total cost of 5 kgs of apples and 4
and y ? kgs of mangoes is Rs. 410.
I. 40% of x is 20% of 50 Directions (71-75): Each of the questions
II. 30% of y is 25% of 72. below consists of a question and two statements
Directions (66-70): Each of the questions numbered I and II given below it. You have to
below consists of a question and two statements decide whether the data provided in the statements
numbered I and II given below it. You have to are sufficient to answer the question. Read both
decide whether the data provided in the statement the statements and
are sufilcienit to answer the question. Read both Give answer (1) if the data in statement I
the statements and alone are sufficient to answer the question, while
Give answer (1) if the data in Statement I the data in statement II alone are not sufficient to
alone are sufficient to answer the question, while answer the question.
the data in Statement II alone are not sufficient to Give answer (2) if the data in statement II
answer the question. alone are sufficient to answer the question, while
Give answer (2) If the data in Statement II the data in statement I alone are not sufficient to
alone are sufficient to answer the question, while answer the question.
the data in Statement I alone are not sufficient to Give answer (3) if the data either in
answer the question. statement I alone or in statement II alone are
Give answer (3) if the data in Statement I sufficient to answer the question.
alone or in Statement II alone are sufficient to Give answer (4) if the data given in both
answer the question. the statements I and II together are not sufficient
Give answer (4) if the data in both the to answer the question, and
Statements I and II are not sufficient to answer Give answer (5) if the data in both the
the question. statements I and II together are necessary to
Give answer (5) if the data in both the answer the question.
Statements I and II together are necessary to (Punjab National Bank Specialist
answer the question. Officer Exam. 24.10.2010)

LEARN MATHS FROM S.K. RAJU (9811549822, 9811649822)


8
71. What is the circumference of the circle ? II. Sonu’s monthly income is Rs. 8,000 more
I. The diameter of the circle is 21 cm. than Sohan’s monthly income.
II. The area of the circle is 346.5 sq. cm. 78. What will be Pravin’s age after 4 years ?
72. What is the average age of the boys in the I. The ratio between Pravin’s and Shekhar’s
class ? present age is 2 : 3 respectively.
I. The average age of girls in the class is 14. II. Shekhar is six years older than Pravin.
II. The number of boys in the class is twice 79. What is the area of a circle ?
the number of the girls in the class. I. The radius of the circle is equal to length
73. How many pieces of 80 cm. each can be of a rectangle.
cut from the reel of cloth ? II. The breadth of the rectangle is equal to
I. The length of the reel of cloth is 900 cm. 22 cms.
II. After cutting all the pieces 20 cms. of 80. What is the average speed of a car ?
the cloth from the reel remains. I. Average speed of the car is four times
74. What is the selling price of the wrist watch ? average speed of a truck which covers
I. The cost price of the wrist watch is Rs 220 kms in 11 hours.
6,400. II. Average speed of the truck is half the
II. 31.25 per cent profit is earned after average speed of a train whereas the
selling the wrist watch. average speed of a jeep is 40 km/hr.
75. What is the two-digit number ? Directions (81-85): Each of the questions
I. The sum of the two digits of the number below consists of a question and two statements
is 6. numbered I and II given below it. You have to
II. The number is divisible by 7. decide Whether the data provided in the statements
Directions (76-80): Each of the questions are sufficient to answer the question. Read both
below consists of a questions and two statements the statements and
numbered I and II given below it. You have to Give answer (1) if the data in Statement I
decide whether the data provided in the statements alone are sufficient to answer the question, while
are sufficient to answer the question. Read both the data in Statement II alone are not sufficient to
the statements and — answer the question.
Give answer (1) if the data in Statement I Give answer (2) if the data in Statement II
alone are sufficient to answer the question, while alone are sufficient to answer the question, while
the data in Statement II alone are not sufficient to the data in Statement I alone are not sufficient to
answer the question. answer the question.
Give answer (2) if the data in Statement II Give answer (3) if the data in Statement I
alone are sufficient to answer the question, while alone or in Statement II alone are sufficient to
the data in Statement I alone are not sufficient to answer the question.
answer the question. Give answer (4) if the data in both the
Give answer (3) if the data in Statement I Statements I and II are not sufficient to answer
alone or in Statement II alone are sufficient to the question.
answer the question Give answer (5) if the data in both the
Give answer (4) if the data in both the Statements I and II together are necessary to
Statements I and II are not sufficient to answer answer the question.
the question. (Oriental Bank Of Commerce PO
Give answer (5) if the data in both the Exam. 26.12.2010 (1st Sitting)
Statements I and II together are necessary to 81. What is the area of the circle ?
answer the questions. I. The breadth of a rectangle is three-fourth
(Bank Of India PO Exam. 31.10.2010) of the radius of the circle.
76. In how many days will five women complete II. The radius of the circle is equal to the
a work ? side of a square of area 144 sq. cm.
I. Two women, five boys and three men 82. What is the cost of three tables and two
together complete the work in eight chairs ?
days. I. Cost of four chairs is twice the cost of
II. Six women complete the same work in three tables.
sixteen days. II. Cost of two tables is equal to cost of one
77. What is Sonu’s monthly income ? cot, i.e. Rs 500 ?
I. Rita’s monthly income is Rs 5000 more 83. What is Jyoti’s annual income ?
than Sonu’s monthly income.
LEARN MATHS FROM S.K. RAJU (9811549822, 9811649822)
9
I. Jyoti’s monthly income is Rs 8,500 more I. The difference between the two digits of
than Amit’s monthly income. the number is 9.
II. Rohit’s monthly income is Rs 3.5 II. The product of the two digits of the
th ousand wh ich is hal f o f A mit’s number is 0.
monthly income. 90. What is the perimeter of the rectangle ?
84. What will be Suraj’s age after eight years ? I. The area of the rectangle is 252 sq. m.
I. The ratio between Kamya’s and Suraj’s II. The ratio of length to breadth of the
present age is 4 : 7 respectively. rectangle is 9 : 7 respectively.
II. Kamya is 15 years younger than Suraj. Directions (91-95): Each of the questions
85. What is the minimum passing percentage below consists of a question and two statements
in a test ? numbered I and II given below it. You have to
I. Raju scored 162 marks in a test and failed decide whether the data provided in the statements
by 104 marks. are sufficient to answer the question. Read both
II. The maximum marks of the test are 538 the statements and
more marks obtained by Raju. Give answer (1) if the data in Statement I
Directions (86-90): Each of the questions alone are sufficient to answer the question, while
given below consists of a question and two the data in statement II alone are not sufficient to
statements numbered I and II given below it. You answer the question.
have to decide whether the data provided in the Give answer (2) if the data in Statement II
statements sufficient to answer the question. Read alone are sufficient to answer the question, while
both the statements and the data in statement 1 alone are not sufficient to
Give answer (1) if the data in statement I answer the question.
alone are sufficient to answer the question, while Give answer (3) if the data in either
the data in statement II alone are not sufficient to Statement I alone or in statement II alone are
answer the question. sufficient to answer the question.
Give answer (2) if the data in statement II Give answer (4) if the data in both the
alone are sufficient to answer the question, while Statements I and II are not sufficient to answer
the data in statement I alone are not sufficient to the question.
answer the question. Give answer (5) if the data in both the
Give answer (3) if the data in statement I Statements I and II together are necessary to
alone or in statement II alone are sufficient to answer the question.
answer the question. (IDBI Bank Officer Exam. 16.09.2012)
Give answer (4) if the data in both the 91. What is the minimum passing percentage
statements I and II are not sufficient to answer in a test ?
the question. I. Raman scored 25% marks in the test and
Give answer (5) if the data in both the Sunil scored 288 marks which is 128
statements I and II together are necessary to more than Raman.
answer the question. II. Raman scored 64 marks less than the
(Indian Bank PO Exam. 02.01.2011) minimum passing marks.
86. What is the salary of C, in a group of A, B, 92. What is the value of x2 + y + z ?
C, D and E whose average salary is Rs 48,250 ? I. 4x + 3y + 5z = 60 and
I. C’s salary is 1.5 times B’s salary. 2x = y, 2y = z
II. Average salary of A and B is Rs 23,500. II. 3x + 3y + 2z = 34,
87. What is the per cent profit earned by selling 2x + 5y + 6z = 72
a car for Rs 6,40,000 ? 93. Whose body weight is second highest
I. The amount of profit earned on selling among the five boys Arun, Vinay, Suraj,
the car was Rs 3,20,000. Raju and Pratap ?
II. The selling price of the car was twice the I. Average weight of Arun, Suraj and Vinay
cost price. is 68 kg and average weight of Raju and
88. What is the rate of interest p.c.p.a. ? Pratap is 72 kg. Also Suraj is 78 kg, Raju
I. An amount of Rs 14,350 gives a simple is 68 kg and Binay is 46 kg.
interest of Rs 11,480 in four years. II. Average weight of Arun, Suraj, Vinay and
II. The amount doubles itself in 5 years with Raju is 68 kg and also Suraj is 78 kg,
simple interest. Raju is 68 kg and Vinay is 46 kg. All of
89. What is the two digit number ? them have different weight.

LEARN MATHS FROM S.K. RAJU (9811549822, 9811649822)


10
94. What is the respective ratio between the 99. Sri Gupta borrowed a sum at compound
length of a rectangle and side of a square ? interest. What is the amount returned in 2
I. Area of the square is 576 sq. cm. and the years ?
area of the rectangle is 600 sq.cm. I. The rate of interest is 5% per annum.
II. Breadth of the rectangle is half the side II. The simple interest incurred on the sum
of the square. in 1 year is Rs. 600.
95. What is the smaller angle of a parallelogram ? III. The borrowed sum is ten times the
I. Ratio between the angles of a triangle is amount earned as simple interest in two
3 : 5 : 4 and the larger angle of the years.
parallelogram is 34° more than the (1) Only I
largest angle of the triangle. (2) Only III
II. Larger angle of the parallelogram is 38° (3) Both II and III
more than its smaller angle. (4) Either I or III
Directions (96-100) : Each of the following (5) All I, II and III
questions consists of a question followed by three 100. What is the area of the given right angled
statements I, II and III. You have to study the triangle ?
question and the statements and decide which of I. The length of hypotenuse is 5 cm.
the statement (s) is/are necessary to answer the II. The perimeter of triangle is four times of
question. its base.
(IBPS Bank PO/MT III. One of the angles of triangle is 60°
CWE-III 26.10.2013) (1) Only II
96. What is the speed of boat in still water ? (2) Only III
I. The boat covers 12 km in 2 hours in (3) Either II or III
downstream. (4) Both I and III
II. The boat covers same distance in 4 hours (5) Question cannot be answer even with
in upstream. the information in all three statements
III. The speed of stream is one third of that Directions (101 - 105): In each of the
of boat in still water. fo ll ow in g que stio ns, a que stion and three
(1) Both I and II statements following it have been given. You are
(2) I and either II or III required to study the question and statements and
(3) All I, II and III decide that the information given in which
(4) Question cannot be answered even with statement(s) is necessary to answer the question ?
the information in all three statements. (Corporation Bank Specialist Officer
(5) None of these (Marketing) Exam 22.12.2014)
97. What is the speed of train ? 101. What is the principal ?
I. The length of train is 240 metre. I. The simple interest accrued on that sum
II. The train crosses a pole in 24 seconds. at the rate of 12% per annum in 2 years
III. The train crosses a platform in 48 is Rs. 360 less than the compound
seconds. interest on the same sum at 12% per
(1) Both I and III annum in 2 years.
(2) Both I and II II. The sum doubles itself in 10 years at
(3) Both II and III 10% per annum rate of simple interest.
(4) Any two of three III. The compound interest on the sum in 2
(5) None of these years at the rate of 12% per annum is
98. What is the age of class teacher ? Rs 6360.
I. There are 11 students in the class. (1) I or II (2) I and III
II. The average age of students and the (3) I and II (4) II and III
teacher is 14 years. (5) None of these
III. The average age of the teacher and 102. What will be the area of a 2 metre wide
students is 3 years more than that of boundary around a rectangular field ?
students. I. The breadth of the field  is one-fourth of
(1) Both I and III its perimeter. Area of the field is 144
(2) Both I and II metre.
(3) II and either I or III II. The ratio of the length and breadth is
(4) All I, II and III respectively 3 : 2.
(5) None of these III. Area of the field is 216 sq. metre.
LEARN MATHS FROM S.K. RAJU (9811549822, 9811649822)
11
(1) I, II and III state-ments are sufficient to answer the question.
(2) only I or II and III Read the question and both the statements and —
(3) I and II (RBI Grade-B Officer Exam. 2008)
(4) Anyone of the three statements Give answer (1) if the data in statement I
(5) None of these alone are sufficient to answer the question, while
103. What are the marks obtained by Arnab in the data in statement II alone are not sufficient
History ? to answer the question.
I. The average marks obtained by Arnab in Give answer (2) if the data in statement II
History, Geography and Civics are 65. alone are sufficient to answer the question, while
II. Marks obtained by Arnab in Geography the data in statement I alone are not sufficient
is 6 more than that obtained in History. to answer the question.
III. Marks obtained in Geography is as Give answer (3) if the data either in
much more than that in Civics as the statement I alone or in statement II alone are
marks obtained in Civics is more than sufficient to answer the question.
that in History. (SBI Banks PO Exam. 11.02.2001)
(1) I and II (2) II and III 1. In h ow man y days can the w ork be
(3) I and III (4) I, II and III completed by A, B and C together ?
(5) None of these (I) A and B together can complete the work
104. What is the speed of train ? in 6 days.
I. Train crosses a signal post in 15 seconds. (II) B and C together can complete the work
II. Train crosses a 250 metre long platform 3
in 27 seconds. in 3 days.
4
III. Train crosses another train running in
the same direction on a parallel track (III) A and C together can complete the work
in 32 seconds. 1
(1) I and II in 3 days.
3
(2) I and III (1) Only I
(3) II and III (2) Only II
(4) Any two of the three (3) Only III
(5) None of these (4) Any one of the three
105. What percent of discount is offered ? (5) All the three statements are necessary
I. The profit earned on a article sold for Rs. to answer the question.
252 after discount is Rs. 52. 2. What is the cost of painting the two adjacent
II. If no discount were allowed, there would walls of a hall which has no windows or
have been a profit of Rs. 80. doors ?
III. If there were no discount, there would (I) The area of the hall is 24 sq. metres.
have been a gain of 40% . (II)
(1) I and II (III) Area of one wall is 30 square metres.
(2) II and either I or III (1) Only I
(3) I and III (2) Only II
(4) I and either II or III (3) Only III
(5) None of these (4) Either I or III
(5) Data inadequate
SBI PO EXAMS 3. What is the total compound interest earned
D irections (1 -5 ) : I n each o f th ese at the end of the three years ?
questions, a question is followed by information (I) Simple interest earned on that amount
given in three statements. You have to study the at the same rate and period is Rs. 4,500
question alongwith the information given in the (II) The rate of interest is 10 percent per
statements and decide the information in which of annum
the statement(s) is/are necessary and sufficient (III) Compound interest for three years is
to answer the question. more than the simple interest for that
Directions (1-5): Each of the questions period by Rs. 465.
below consists of a question and two statements (1) Only I and II
numbered I and II given below it. You have to (2) Only II and III
de ci de w he th er the data pro vi de d in the (3) Only I and III
LEARN MATHS FROM S.K. RAJU (9811549822, 9811649822)
12
(4) Any two of the three II. Had there been no discount the profit
(5) Either II or III only earned would have been Rs. 80.
4. What is the percent profit earned by the III. Had there been no discount the profit
shopkeeper on selling the articles in his earned would have been 40% .
shop ? (1) I and II only
(I) Labelled price of the articles sold was (2) II and either I or III only
130% of the cost price. (3) I and III only
(II) Cost price of each article was Rs. 550. (4) I and either II or III only
(III) A discount of 10% on labelled price was (5) None of these
offered. 8. What is the speed of the train ?
(1) Only I I. The train crosses a signal pole in 13 sec.
(2) Both I and II II. The train crosses a platform of length
(3) Only III 250 mts. in 27 seconds.
(4) All the three statements are necessary III. The train crosses another train running
to answer the question in the same direction in 32 seconds.
(5) Questions cannot be answered even with (1) I and II only
the information given in all the three (2) I and III only
statements. (3) II and III only
5. What is the average salary of 15 employees ? (4) Any two of the three
(I) Average salary of 7 Clerical cadre (out of (5) None of these
the 15 employees) employees is Rs. 9. What is the population of State A ?
8,500. I. After increasing the population of State
(II) Average salary of 5 Officer cadre (out of A by 15% it becomes 1.61 lakhs
the 15 employees) employees is Rs. II. Ratio of population of State A to that of.
10,000. State B is 7 : 8 respectively
(III) Average salary of the 3 substaff III. Population of State B is 1.6 lakhs
employees (out of the 15 employees) is (1) I only
Rs. 2,500. (2) II and III only
(1) None (3) I and II only
(2) Only I (4) Either only I or II and III
(3) Only II (5) All I, II and III
(4) Only III 10. Ho w many w orke rs are re qu ired for
(5) Question cannot be answered even with completing the construction work in 10
information given in all three statements days ?
Directions (6-10) : Each of the questions I. 20% of the work can be completed by 8
consists of a question followed by three statements. workers in 8 days
Yo u have to study the que stio ns and the II. 20 workers can complete the work in 16
statements and decide which of the statement(s) days
is/are necessary to answer the question. III. One eighth of the work can be completed
(SBI Associate Banks PO by 8 workers in 5 days
Exam. 21.07.2002) (1) I and II only
6. What is the area of the hall ? (2) II and III only
I. Material cost of flooring per sq metre is (3) I only
250 (4) III only
II. Labour cost of flooring the hall is Rs. (5) Any one of three
3,500 Directions (11-16) : Each of the questions
III. Total cost of flooring the hall is Rs. below consists of a question and two statements
14,500 numbered I and II given below it. You have to
(1) I and II only decide whether the data provided in the statements
(2) II and III only are sufficient to answer the question. Read both
(3) All I, II and III the statements and—
(4) Any two of the three Give answer (1) if the data in statement I
(5) None of these alone are sufficient to answer the question, while
7. What was the percentage of discount offered ? the data in statement II alone are not sufficient
I. Profit earned by selling the article for Rs. to answer the question.
252 after giving discount was Rs. 52.
LEARN MATHS FROM S.K. RAJU (9811549822, 9811649822)
13
Give answer (2) if the data in statement II (SBI PO Exam. 09.01.2005)
alone are sufficient to answer the question, while 17.
What is the average weight of girls in the
the data in statement I alone are not sufficient class ?
to answer the question. I. Average weight of all the 60 students is
Give answer (3) if the data either in 42 kg.
statement I alone or in statement II alone are II. Average weight of boys is 43 kg.
sufficient to answer the question. III. Total weight of all girls together is 1144
Give answer (4) if the data even in both kg.
statements I and II together are not sufficient (1) Any two of three
to answer the question. (2) All I, II & III
Give an swer (5) if the data in both (3) I & II only
statements I and II together are necessary to (4) II & III only
answer the question. (5) Question cannot be answer even with
(SBI Banks PO Exam. 18.05.2003) information in all three statements.
11. 18.
By selling a product with 20% profit, how What is the selling price of the T.V. set if no
much profit was earned ? discount is offers
I. The difference between, cost and selling I. Profit earned was 20% .
price is Rs. 40/-. II. Had 10% discount been offered on
II. The selling price is 120 per cent of the selling price the profit have been Rs.
cost price. 1,200.
12. A train crosses another train running in the III. Cost price is Rs. 15,000.
opposite direction in x seconds. What is the (1) Any two of the three
speed of the train ? (2) Only I & II
I. Both the trains have the same length and (3) Only I & III
are running at the same speed. (4) Only II & III
II. One train crosses a pole in 5 seconds. (5) None of these
13. What is the two digit number ? 19. What is the speed of the train ?
I. The difference between the two digits is I. Length of the platform is 150% of the
9. length of the train.
II. The sum of the digits is equal to the II. The train crosses the platform in 25
difference between the two digits. seconds.
14. A spherical ball of given radius x cm is Ill. The train crosses the signal pole in 19
melted and made into a right circular seconds.
cylinder. What is the height of the cylinder ?
(1) All I, II & III
I. The volume of the cylinder is equal to the(2) I and either II or III
volume of the ball. (3) Only II & III
II. The area of the base of the cylinder is (4) Any two of the three
given. (5) Question cannot be answered even with
15. Area of a square is equal to the area of a the information in all three statements.
20.
circle. What is the circumference of the How many children are there in the class ?
circle ? I. 20% children are there in the class speak
I. The diagonal of the square is x inches. Hindi.
II. The side of the square is y inches. II. 44 children can speak languages other
16. A, B and C are positive integers. Is their than Hindi.
product an even number ? III. There are 30 boys in the class.
I. A is an even number. (1) All I, II & III
II. The product of A and B is an even (2) Any two of the three
number and that of A and C is also an (3) II and either I or III
even number. (4) I and II only
Directions (17-21) : In each of the following(5) None of these
21.
questions, a question is followed by information What is the volume of the cylindrical tank ?
given in three statements. You have to study the I. Area of the base is X square metres.
question alongwith the statements and decide the II. Height of the tank is Y metres.
information given in which of the statement(s) is III. Diameter of the base is equal to height
necessary and sufficient to answer the question. of the tank.
(1) Only I & II
LEARN MATHS FROM S.K. RAJU (9811549822, 9811649822)
14
(2) Only II Directions : (27-31) : Each question below
(3) Only I & III is followed by two statements A and B. You are to
(4) All I, II & III. determine whether the data given in the statement
(5) Any two of the three is sufficient for answering the question. You
Directions (22-26) : Each of the questions should use the data and your knowledge of
below consists of a question and two statements Mathematics to choose the best possible answers.
numbered I and II given below it. You have to Give answer (1) if the statement A alone is
decide whether the data provided in the statements sufficie nt to answ er the question, but the
are sufficient to answer the questions. Read both statement B alone is not sufficient.
the statements and - Give answer (2) if the statement B alone is
Give answer (1) if the data in statement I sufficie nt to answ er the question, but the
alone are sufficient to answer the question, while statement A alone is not sufficient.
the data in statement II alone are not sufficient to Give answer (3) if both statements A and
answer the question. B together are needed to answer the question.
Give answer (2) if the data in statemeat II Give answer (4) either the statement A
alone are sufficient to answer the question, while alone or statement B alone is sufficient to answer
the data in statement I alone are not sufficient to the question
answer the question. Give answer (5) if you cannot get the
Give answer (3) if the data either in answer from the statements A and B together, but
statement I alone or in statement II alone are need even more data.
sufficient to answer the question. (SBI PO Preliminary (Tire-I)
Give answer (4) if the data even in both Exam. 27.07.2008)
the statements I and II together are not sufficient 27. Triangle ABC has angle BAC equal to 90°.
to answer the question. What is the measure of the angle ABC ?
Give answer (5) if the data in both  the A. The angle ACB is 35°.
statements I and II together are neccessary to B. The angle CBA is 55°.
answer the question. 28. X, Y and Z are three consecutive even
(SBI Associate Banks PO numbers (not necessarily in this order).
Exam. 07.01.2007) What is the sum of these numbers ?
22. By selling a product for Rs. 100/-how much A. The difference between X and Z is 4.
profit was earned ? B. One-third of Y is 14.
I. 20% profit would have been earned if it 29. What is the salary of P, in a group of P, Q,
had been sold for Rs. 90/- R, S, T and U, whose average salary is Rs.
II. The profit was one-third of the purchase 35,000 ?
price. A. Total of the salary of Q and S is Rs.
23. A train crosses another train running in the 54000.
opposite direction in x seconds. What is the B. Total of the salary of T and U is Rs. 58000.
speed of the train ? 30. What is the rate of p.c.p.a. on an amount of
I. Both the trains are running at the same Rs. 6,000 deposited in a Bank ?
speed. A. The simple interest for four years is Rs.
II. The first train is y cm. long. 36000.
24. The difference between the two digits of a B. The difference between the simple
number is 6. What is the number ? interest and compound interest is Rs.
I. The digit at the unit place is bigger than 894.0375.
the other digit. 31. What is the number ?
II. The sum of the two digits is 12. A. 20% of that number is one-fifth of that
25. X, Y and Z are integers. Is X an odd number ? number.
I. An odd number is obtained when X is 5
divided by 5. B. th of that number is less by 15 of that
6
II. (X + Y) is an odd number.
26. What is the capacity of a cylindrical tank ? number.
I. Radius of the base is half of its height Directions (32 - 36) : Each of the questions
which is 28 metres. below consists of a question and two statements
II. Area of the base is 616 sq. metres and numbered I and II given below it. You have to
its height is 28 metres. decide whether the data provided in the statements

LEARN MATHS FROM S.K. RAJU (9811549822, 9811649822)


15
are sufficient to answer the question. Read both II. The breadth of rectangle is 5 cm less than
the statements and— its length and its perimeter is 50 cm.
(SBI Management Executive
Exam. 23.02.2014) RBI GRADE-B OFFICER EXAMS
Give answer (1) if the data in statement I Directions (1-5): Each of the questions
alone are sufficient to answer the question, while below consists of a question and two statements
the data in statement II alone are not sufficient to numbered I and II given below it. You have to
answer the question. decide whether the data provided in the statements
Give answer (2) if the data in statement II are sufficient to answer the question. Read the
alone are sufficient to answer the question, while question and both the statements and —
the data in statement I alone are not sufficient to (RBI Grade-B Officer Exam. 2008)
answer the question. Give answer (1) if the data in statement I
Give answer (3) if the data either in alone are sufficient to answer the question, while
statement I alone or in statement II alone are the data in statement II alone are not sufficient
sufficient to answer the question. to answer the question.
Give answer (4) if the data given in both Give answer (2) if the data in statement II
the statements I and II together are not sufficient alone are sufficient to answer the question, while
to answer the question, and the data in statement I alone are not sufficient
Give answer (5) if the data in both the to answer the question.
statements I and II together are necessary to Give answer (3) if the data either in
answer the question. statement I alone or in statement II alone are
32. What will be the amount at the end of 2 sufficient to answer the question.
years, if the interest is compounded yearly. Give answer (4) if the data even in both
I. The simple interest on the same sum for the statements I and II together are not sufficient
a period of 2 years is Rs. 400 at the same to answer the question.
rate of 5% per annum. Give answer (5) if the data in both the
II. The difference between the simple statements I and II together are necessary to
interest and the compound interest for answer the question.
2 years at the rate of 5% per annum is 1. Train ‘A’ running at a certain speed crosses
Rs. 100. another train ‘B’ running at a certain speed
33. Find the average of five consecutive odd in the opposite direction in 12 seconds.
numbers. What is the length of train ‘B’ ?
I. The difference of fifth number and the I. The length of both the trains together is
first number is 7. 450 metres.
II. The sum of the first two numbers is 5 II. Train ‘A’ is slower than train ‘B’.
more than the fifth number. 2. Area of a rectangle is equal to the area of a
34. Find the ratio of the area of the bigger circle right angled triangle. What is the length of
and smaller circle. the rectangle ?
I. The base of the triangle is 40 cms.
II. The height of the triangle is 50 cms.
3. What was the total compound interest on a
sum after three years ?
I. The radius of the smaller circle is 24 cm. I. The interest after one year was Rs. 100/
II. The difference between the radii of bigger - and the sum was Rs. 1,000/-.
and the smaller circles is 3 cm. II. The difference between simple and
35. What is the length of the train ? compound interest on a sum of Rs.
I. The train crosses a signal post in 9 1,000/- at the end of two years was Rs.
seconds. 10/-.
II. If the train with speed x kmph crosses 4. What Is the two digit number where the
another train 100m long coming from digit at the unit place is smaller ?
the opposite direction at 60 kmph in 15 I. The difference between the two digits is
seconds. 5.
36. Find the radius of the semi-circle. II. The sum of the two digits is 7.
I. The area of semi-circle is equal to the area 5. What is the speed of the boat in still water ?
of the rectangle. I. It takes 2 hours to cover distance
between A and B downstreams.
LEARN MATHS FROM S.K. RAJU (9811549822, 9811649822)
16
II. It takes 4 hours to cover distance II. One of the variables is greater than 83.
between A and B upstreams.
Directions (6-10): Each of the questions INSURANCE EXAMS
given below consists of a question and two Directions (1-5) : In each of the following
statements numbered I and II given below it. You questions, a question is asked followed by three
have to decide whether the data provided in the statements. You have to the questions and all the
statements is sufficient to answer the question. three statements given and decide whether
Read both the statements. information provided in the statement(s) are
Give answer (1) if the data in statement I redundant and can be expensed with while
alone is sufficient to answer the question, while answering the questions ?
the data in statement II alone is not sufficient to (LIC Assistant Administrative
answer the question. Officer (AAO) Exam, 24.04.2005)
Give answer (2) if the data in statement II 1. What is the area of the given rectangle ?
alone is sufficient to answer the question, while (A) Perimeter of the rectangle is 60 cms.
the data in statement I alone is not sufficient to (B) Breadth of the rectangle is 12 cms,
answer the question. (C) Sum of two adjacent sides is 30 cms.
Give answer (3) if the data in statement I (1) A and B only
alone or in statement II alone is Sufficient to (2) A only
answer the question. (3) B only
Give answer (4) if the data in both the (4) C only
statements I and II is not stifficient to answer the (5) A or C only
question. 2. Who is the tallest among M, P, Q and R ?
Give answer (5) if the data in both the (A) P is taller than Q but not as R.
statements I and II together is necessary to answer (B) R is taller than M.
the questions. (C) M is taller than P but not as tall as R.
(RBI Grade-B Officer Exam.06.02.2011) (1) C only
6. What is the two digit positive number ? (2) B only
I. Sum of the two digits of the number is 8. (3) B or C only
II. Sum of the two digits is 3 more than the (4) A or B only
higher digit. (5) A or C only
7. The symbol  represents one of the the 3. What will be the ratio between ages of Samir
fo ll ow in g operati on s: additio n, and Anil after five years?
subtraction, multiplication or division. (A) Samir’s present age is more than Anil’s
What is the value of 6  4 ? present age by 4 years.
I. 0  5 = 5 (B) Anil’s present age is 20 years.
II. 5  0 = 5 (C) Anil and Samir’s present ages are in the
8. At what time did Poonam finish the job ? ratio of 3 : 4.
I. Poonam started working with out a break (1) A or B or C only
on the job at exactly 9 a.m. and by noon (2) B only
she had utilised exactly half the time (3) C only
that it took her to finish the job. (4) A or C only
II. Poonam took exactly 6 hours to finish (5) B or C only
the job. 4. Mr. X bo rrow d a sum o f mo ne y on
9. Last year an employee received an annual compound interest. What will be the
salary of Rs 6,18,000 which was paid in amount to be repaid if he is repaying entire
equal pay cheques throughout the  year. amount at the end of two years ?
What was the salary received in each of the (A) The rate of interest is 5 percent p.a.
paycheques ? (B) Simple Interest fetched on the same
I. The employee received a total of 24 amount in one year is Rs. 600/-.
paycheques durning the year ? (C) The amount borrowed is 10 times the
II. The employee received a paycheque twice simple interest in two years.
a month each month during the year. (1) C only
10. Is the sum of integers x and y greater than (2) A only
85 ? (3) A or B only
I. The product of x and y is greater than 85. (4) A or C only
LEARN MATHS FROM S.K. RAJU (9811549822, 9811649822)
17
(5) All A, B and C are required to answer (3) II and III only
the question. (4) Any two of the three
5. A boat will take how much time to cross (5) None of these
9.
the river against the stream of the river ? What is the population of State ‘A’ ?
(A) In still water the speed of the boat is 15 I. After increasing the population of State
km/hour. A by 15% it becomes 1.61 lakhs
(B) The width of the river is 8 km. II. Ratio of population of State A to that of
(C) The speed of the stream is 2 km/hr. State B is 7 : 8 respectively
(1) Only A III. Population of State B is 1.6 lakhs
(2) Only B (1) 1 only
(3) Only C (2) II and III only
(4) All A, B, C are required to answer the (3) I and II only
question (4) Either only I or II and III
(5) It is not possible to answer the question (5) All I, II and III
10.
with the help of statements all three A, Ho w many w orke rs are re qu ired for
B and C. completing the construction work in 10
Directions (6-10) : Each of the questions days ?
consists of a question followed by three statements. I. 20% of the work can be completed by 8
Yo u have to study the que stio ns and the workers in 8 days
statements and decide which of the statement(s) II. 20 workers can complete the work in 16
is/are necessary to answer the question. days
(LIC Assistant Administrative III. One eighth of the work can be completed
Officer (AAO) Exam. 2006) by 8 workers in 5 days
6. What is the area of the hall ? (1) I and II only
I. Material cost of flooring per sq metre is (2) II and III only
250 (3) I only
II. Labour cost of flooring the hall is Rs. (4) III only
3,500 (5) Any one of three
III. Total cost of flooring the hall is Rs. Directions (11-15): In each of the following
14,500 questions, a question and three statements I, II
(1) I and II only and III are given. You have to decide whether the
(2) II and III only data given in the statements are sufficient to
(3) All I, II and III answer the question or not.
(4) Any two of the three (LIC Assistant Administrative
(5) None of these Officer Exam. 2008)
7. 11.
What was the percentage of discount offered ? How much profit did the company earn in
I. Profit earned by selling the article for Rs.the year 2002 ?
252 after giving discount was Rs. 52. I. The company earned 40% more profit in
II. Had there been no discount the profit the year 2003 than that in the year
earned would have been Rs. 80. 2001.
III. Had there been no discount the profit II. The company earned a total profit of Rs.
earned would have been 40% . 20 crores in the years 2001 and 2002
(1) I and II only taken together,
(2) II and either I or III only III. In the year 2003, the company earned
(3) I and III only 80 per cent, profit of that in 2002.
(4) I and either II or III only (1) Any two of I, II and III are sufficient
(5) None of these (2) Either I and II or II and III are sufficient
8. What is the speed of the train ? (3) I and II or III are sufficient
I. The train crosses a signal pole in 13 sec. (4) All I, II and III are necessary to answer
II. The train crosses a platform of length the question
250 mts. in 27 seconds. 12. How many students secured at least 60 per
III. The train crosses another train running cent marks in Mathematics in a class of 240
in the same direction in 32 seconds. students ?
(1) I and II only I. 20 per cent of the students in the class
(2) I and III only secured 80 per cent and above marks
in Mathematics.
LEARN MATHS FROM S.K. RAJU (9811549822, 9811649822)
18
II. 80 students have secured more than 50 (3) Either I or II and III are sufficient
per cent but less than 60 per cent marks (4) All I, II and III are necessary to answer
in Mathematics. the question
III. The number of students who secured (5) The question cannot be answered even
marks between 60 and 79 per cent was with all I, II and III
equal to the number of students who Directions (16-20): Each question below
secured less than 50 per cent marks in is followed by two statements A and B. You are to
Mathematics. determine whether the data given in the statement
(1) All I, II and III are necessary to answer is sufficient for answering the question. You should
the question use the data and your knowledge of Mathematics
(2) Only I and III are sufficient to choose between the possible answers.
(3) Only II and III are sufficient (United India Insurance AO
(4) Only I and II are sufficient Exam. 26.05.2013
(5) The question cannot be answered even Give answer (1) if the statement A alone is
with all I, II and Ill sufficie nt to answ er the question, but the
13. In an institute employing managers, statement B alone is not sufficient.
observers and assistants what is the Give answer (2) if the statement B alone is
monthly salary of an assistant ? sufficie nt to answ er the question, but the
I. Each observer gets Rs. 12000 per month statement A alone is not sufficient.
more than an assistant. Give answer (3) if both statements A and
II. An observer and an assistant together B together are needed to answer the question.
get Rs. 32000 per month. Give answer (4) if either the statement A
III. The total salary per month of a manager alone or statement B alone is sufficient to answer
and an observer is Rs. 57000. the question.
(1) Either I or II and III are sufficient Give answer (5) if you cannot get the
(2) Only I and II are sufficient answer from the statements A and B together, but
(3) Any two of I, II and III are sufficient need even more data.
(4) All I, II and III are necessary to answer 16. Is A an odd number ?
the question A. A multiplied by an odd nmber is equal
(5) The question cannot be answered even to an odd number.
with all I, II and III B. A is not divisible by 2.
14. What is the circumference of semicircle in 17. The ages of Pradumn and C are in the ratio
cm ? of 7 : 5 . What is the age of Pradumn.
I. The area of semi-circle is half of the area A. The ages of Pradumn and Nandini are in
of parallelogram. the ratio of 3 :1.
II. The length of parallelogram is 1.5 times B. After 7 years the ratio of Pradumn’s and
the radius of the semicircle Aviral’s ages will be 4 : 3.
III. The difference between the length and 18. What is the salary of B, in a group of A, B,
breadth of parallelogram is 8 cm. C and D whose average salary is Rs. 62,880 ?
(1) Only II and III are sufficient A. Total of the salary of A and C is exact
(2) Only I and III are sufficient multiple of 8.
(3) Only I and II are sufficient B. Average of the salary of A , C and D is
(4) All I, II and III are necessary to answer Rs. 61,665.
the question 19. What is the three digit nunmber ?
(5) The question cannot be answered even A. The three digit number is divisible by 9.
with all I, II and III B. The first and the third digit is 6.
15. In a partnership business of A, B and C, 20. What is the profit earned by selling a printer
what profit did B get at the end of 2 years ? for Rs. 3,000 ?
I. A and B started the business by investing A. The cost price of 6 such printers is equal
in ratio 3 : 5 to selling price of 5 such printers.
II. C joined after 6 months with Rs. 4 lakhs B. 20% profit is earned by selling each
III. After two years, A’s share in profit was printer.
Rs. 60000.
(1) Only I and III are sufficient
(2) Only II and III are sufficient

LEARN MATHS FROM S.K. RAJU (9811549822, 9811649822)


19
SHORT ANSWERS SBI PO EXAMS
NATIONALISED BANKS 1. (5) 2. (5)
& IBPS PO/MT/SO 3. (5) 4. (2)
1. (4) 2. (4) 5. (1) 6. (3)
3. (3) 4. (5) 7. (4) 8. (1)
5. (4) 6. (2) 9. (4) 10. (5)
7. (3) 8. (5) 11. (1) 12. (4)
9. (5) 10. (1) 13. (1) 14. (2)
11. (4) 12. (5) 15. (3) 16. (3)
13. (5) 14. (3) 17. (2) 18. (1)
15. (2) 16. (5) 19. (5) 20. (4)
17. (4) 18. (5) 21. (5) 22. (3)
19. (2) 20. (3) 23. (4) 24. (5)
21. (4) 22. (5) 25. (1) 26. (3)
23. (2) 24. (3) 27. (4) 28. (3)
25. (4) 26. (4) 29. (5) 30. (1)
27. (5) 28. (2) 31. (2) 32. (3)
29. (5) 30. (3) 33. (4) 34. (5)
31. (4) 32. (1) 35. (5) 36. (5)
33. (3) 34. (2)
35. (5) 36. (1) RBI GRADE-B OFFICER EXAMS
37. (2) 38. (3) 1. (4) 2. (4)
39. (3) 40. (4) 3. (3) 4. (5)
41. (4) 42. (3) 5. (4) 6. (4)
43. (1) 44. (2) 7. (5) 8. (1)
45. (5) 46. (1) 9. (3) 10. (5)
47. (4) 48. (3)
49. (5) 50. (3) INSURANCE EXAMS
51. (3) 52. (5) 1. (1) 2. (4)
53. (1) 54. (4) 3. (3) 4. (5)
55. (5) 56. (2) 5. (4) 6. (3)
57. (4) 58. (5) 7. (4) 8. (1)
59. (1) 60. (3) 9. (4) 10. (5)
61. (5) 62. (4) 11. (4) 12. (5)
63. (3) 64. (4) 13. (2) 14. (5)
65. (5) 66. (2) 15. (1) 16. (4)
67. (5) 68. (4) 17. (5) 18. (2)
69. (5) 70. (1) 19. (3) 20. (4)
71. (3) 72. (4)
73. (1) 74. (5)
EXPLANATIONS
75. (5) 76. (2)
77. (4) 78. (5) NATIONALISED BANKS
79. (4) 80. (1) & IBPS PO/MT/SO
81. (2) 82. (5) 1. (4) Time taken in crossing each other
83. (5) 84. (5) Total length of trains
85. (5) 86. (4) =
Relative speed
87. (3) 88. (3)
89. (5) 90. (5) The information given in both statements
91. (5) 92. (1) is not sufficient as length of train A and
93. (5) 94. (5) individual speed of each train are required.
95. (3) 96. (2) 2. (4) Area of rectangle = Area of triangle.
97. (2) 98. (4) From the information given in both the
99. (4) 100. (4) statements, we can find area of triangle or
101. (1) 102. (2) area of rectangle. For finding length,
103. (4) 104. (1) breadth is required, which is not known.
105. (4)
LEARN MATHS FROM S.K. RAJU (9811549822, 9811649822)
20
3. (3) From the statement I, = Rs. (12000 + 1200) = Rs. 13200
100  100 Similarly, the income for other months can
r = =10% be calculated.
1000
7. (3) From statement I,
Thus we have, If the radius of the circular field be r metres,
P = Rs. 1000, r = 10% , t = 3years then 2  r = x
Hence, C.I. can be determined This relation gives us the value of radius.
From the statement II.
 Area =  × (Radius)2
1000  r  2 Hence, statement I alone is sufficient to
S.I = = 20r answer the question
100
From statement II,
 r 
2
 Diameter = y
C.I = 1000  100   1
 1 
    Diameter 
2

 Area =  
 C.I - S.I  2 
 200r  r 2  Hence, statement II alone is also sufficient
= 1000   - 20r to answer the question.
 10000  8. (5) Let the speed of boat in still water be x
 2000r + r2 - 200r = 100 kmph and that of current be y kmph
 r = 10 Rate upstream = (x - y) kmph
Hence, C.I. can be determined Rate downstream = (x + y) kmph
4. (5) Let the unit’s digit be x and ten’s digit From statement I,
be y and x < y. 35
 Number = 10y + x x+y=
5
From statement I,
y- x=5 ....(i)  x + y = 7 kmph ....(i)
From statement II, From statement II,
y+x=7 ....(ii) 35
From (i) and (ii), x, y can be calculated and x -y=
7
two digit number can be found.
 x - y = 5 kmph ...(ii)
5. (4) Let the distance between A and B be z
From combined statements I and II, we can
km.
get the required answer.
Again, let speed of boat in still water be x
9. (5) From statement I,
kmph and that of stream be y kmph.
Let the number of boys and girls be 5x and
 Rate downstream = (x + y) kmph 6x respectively.
Rate upstream = (x - y) kmph
From statement II,
From statement I,
6x - 5x = 7
z x=7
x y = 2 ...(i)
 5x = 35 and
From statement II 6x = 42
Clearly, both statement are required to
z answer the question.
x y = 4 ....(ii)
10. (1) From statement I,
6. (2) From statement I, SP = Rs. 1740
If the. annual income of boss be Rs. x, then % Profit = 20
Mr. Krishnamurthy’s annual income 100  1740
= 170% of x  CP = 120
= Rs 1450
But we donot know the value of. x. Hence
 Profit = Rs. (1740 - 1450)
this statement is not sufficient from
= Rs. 290
From statement II,
Information in statement II is not required.
Initial income = Rs. 12000
Hence, statement I alone is sufficient to
% Increase = 10% per month
answer the question.
Time = 12 months
11. (4) We need equivalence between one day’s
 Income in May = Rs. 12000 + 10% of work of man and woman.
12000
LEARN MATHS FROM S.K. RAJU (9811549822, 9811649822)
21
From statement I, we can get 1 man’s 1 Where r = radius of the base and h = height
day’s work. of the tank.
From statement II or III, we can get 1 From statement II,
woman’s 1 day’s work.  r 2 = 616
Hence, we can establish relation between
From statement III,
man’s and woman’s work and get the
h = 28 metre
required answer.
Hence, we can determine the capacity of
12. (5) When a train crosses a pole, then
the tank by any two of the statements
Speed of train
17. (4) Let the length of the train be x metre
Length of train From statements I and II,
=
Time taken x 2x
When a train crosses a platform, then Speed Speed of train = =
18 36
of train It gives no result.
Length of platform and train From statements I and III,
=
Time 300
Clearly, statement II and either I or III Speed of train =
18
supplement the required data to determine
the speed of train. 50
= metre/second
13. (5) Area of the square = (side)2 3
1 From statements II and III
= (diagonal)2 300  300
2
Speed of train =
Again, Perimeter = 4 × side 36
Cl earl y, from any o ne of the three 50
statements we can determine area of the = metre/second.
square.
3
14. (3) Let the number be 10x + y. 18. (5) Question cannot be answered even with
From statement I, the information in all three statements.
(10y + x) - (10x + y) = 18 19. (2) Let the two digit number be 10x + y.
From statement I,
 9 (y - x) = 18
 10y + x = 10x + y + 9
y - x =2 ...(i)
From statement II,  9 (y - x) = 9
x + y = 14 ...(ii) y - x = 1 ...(i)
From statement III, From statement II,
y- x =2 ...(ili) x+y=7 ...(ii)
Clearly, statement II and either I or III can From statement III,
supplement the data to know x and y and x - y = l or y - x = l ...(iii)
hence the number. From statements I and II,
15. (2) From statement I, The number obtained = 34
We do not know the principal. From statements II and III,
From statement II, Numbers = 34 or 43
Data are incomplete, Hence, we get no unique answer.
i.e. principal is unknown. 20. (3) From statements II and III
From statement III, Profit on each article = 765 - 632
We get the required data, i.e. From statements I, II and III
If principal be Rs. x. Number of articles sold
Interest = Rs. x 1596
Time = 5 years, = = 12
133
Interest × 100 21. (4) From statement A
Rate = Principe × time 3 × 5 = 15 ; 5 × 9 = 45 (An odd number)
It is also obvious from statement B.
16. (5) From statements I, 22. (5) The answer is not possible with the help
Volume of the cylindrical tank =  r2h of even both the statements. We need more

LEARN MATHS FROM S.K. RAJU (9811549822, 9811649822)


22
information like sum or average of their  x = 2500 × 10 = Rs. 25000
ages or ratio of their after some time or 30. (3) From statement I,
before sometime etc. Speed of the bus
23. (2) A + B + C + D
Distance covered
= Rs. (4 × 62880) =
From statement B, Time Taken
A + C + D = Rs.(3 × 61665) 80
 B’s salary = (A + B + C + D)’s = = 16 kmph
5
salary - (A + C + D)’s salary As per the information in statement II, the
24. (3) From statement A, speed ofthe bus can also be determined.
The three digit number is divisible by 9. 31. (4) From statement A,
From statement B, A number divisible by 2 is an even number.
Number = 6*6 From statement B,
A number is divisible by 9 if sum of its digits 2 + 3 = 5 (odd number)
is divisible by 9. 4 + 9 = 13 (odd number)
Clearly, * = 6 32. (1) From statement A,
because 666  9 = 74 Let the number be x.
25. (4) From statement A,
Let CP of 1 printer = Re. 1 x x 3x  2x
 - = 27  = 27
 CP of 5 printers = Rs. 5 2 3 6
and SP of 5 printers = Rs. 6  x = 27 × 6 = 162
1 Statement B is superfluous.
 Gain % = × 100 = 20% 33. (3) From both statements,
5
T:S=6:5
100 T:D = 3:2 = 6 :4
 CP = 120 × 3000 = Rs.2500
T : S : D = 6 : 5 : 4
 Gain = Rs. (3000 - 2500) = Rs. 500 Let the present ages of Danish and Shivay
From statement B, we can also find the be 4x and 5x years
answer. respectively.
26. (4) The given data are inadequate. 4x  6 6
27. (5) From statement II,  =
5x  6 7
If the age of Rani = x years,
then Surekha’s age = 2x years  30x + 36 = 28x + 42
 x + 2x = 72  2x = 6  x = 3
 3x = 72 years Hence, the age of Shivay can be determined.
34. (2) From statement B,
72 Men Days
 x= = 24 years
3 9 27
 Rani’s age = 24 years 15 x
As per the given information in statement  15 : 9 = 27 : x
I, Nidhi’s age can be determined.  15 × x = 9 × 27
28. (2) Statement I is superfluous.
From statement II, 9  27 81
x = = days
Number of boys in the school 15 5
35. (5) From statement A,
60
= 3500 × = 2100 A + E = 64500
100 From statement B,
Number of girls in the school = 3500 - 2100 B + F = 52600
= 1400.  C + D = (A + B + C + D + E + F) - (A + B +
 Required ratio = 2100 : 1400 = 3 : 2 E + F)
29. (5) Let Mr. Mehta’s present income be Rs. He nce, the sal ary of C can no t be
x. determined.
From statements I and II, 36. (1) From statement ‘A
10% of x = 2500 Days Women
10 20 12
x × = 2500 15 x
100
LEARN MATHS FROM S.K. RAJU (9811549822, 9811649822)
23
 15 : 20 = 12 : x 270
 15 × x = 12 × 20 Speed of car = = 45 kmph
6
12  20 42. (3) From statements A and B,
x= = 16 women
Let the number be 10y + x
15
Statement B is superfluous. where x > y
37. (2) From statement B, xy = 72 ....(i)
x - y=1 ...(ii)
Principal × time × Rate
SI =  (x + y) 2 = (x - y)2 + 4xy
100 = 1 + 4 × 72
12000  2  Rate = 1 + 288 = 289
 2800 =
100  x + y = 289 =17 .... (iii)
2880  100 From equations (ii) and (iii),
 Rate = 12000  2 x = 9 y= 8
 Number = 89
= 12% per annum
43. (1) From statement A,
38. (3) From both the statements,
Number of boys
Number = 7 * 7
 7 * 7 is exactly divisible by 13. 40
= 2500 × = 1000
 *= 6 100
 Number = 767 Number of girls
39. (3) From both the statements, = 2500 - 1000 = 1500
A + B + C + D + E = 5 × 45  Required ratio
= 225 years (i) = 1500 : 1000 = 3 :2
A + B = 106 years (ii) Statement B is superfluous.
D + E = 94 years (iii) 44. (2) Statement A is superfluous.
By Equation (i) - (ii) - (Iii), the age of C can From statement B,
be determined. Let the number be x.
40. (4) From statement A,
x x
CP of 5 laptops - = 27
= Rs. (4 × 26250) 2 3
= Rs. 105000 3x  2x
SP of 5 laptops  = 27
6
= Rs. (5 × 26250)
 x = 27 × 6 = 162
= Rs. 131250
45. (5) Both the statements together are not
 Gain/laptop
sufficient to answer the question.
Rs.(131250-105000) 46. (1) From statements (I) and (II),
=
5 D + E = 14
26250 and A+B + C + F = 4 × 50 = 200
= Rs. = Rs. 5250 A+B+C+D+E+F
5

From statement B, 6
CP of each laptop 14  200 214 2
100 = = = 35 years
6 6 3
= × 26250 = Rs. 21000
125 47. (4) Area of the right angled triangle
 Gain = Rs. (26250 - 21000) 1
= Rs. 5250 = × base × height
2
41. (4) From statement A,
48. (3) From all three statements,
Distance covered
Speed of car = 1
Time taken (A + B)’s 1 day’s work = ....(i)
8
135
= = 45 kmph 1
3 (B + C)’s 1 day’s work = ....(ii)
10
From statement B,
LEARN MATHS FROM S.K. RAJU (9811549822, 9811649822)
24
53. (1) From statement I alone,
1
(A + C)’s 1 day’s work = ...(iii) Principal = Rs. x, R = ?, S.I. = x
12 T = 10 years.
Adding all three equations,
2(A + B + C)’s 1 day’s work S.I × 100
 R = Principal × Time
1 1 1
= + + x  100
8 10 12 = = 10%
x  10
15  12  10 37 54. (4) Statement II gives no conclusion.
= =
120 120 From statement II,
37 1
 (A + B + C)’s 1 days work = 240 ....(iv) (A + B)’s 1 day’s work =
12
By equation (iv) - (iii), A’s 1 day’s work is not known.
55. (5) From both the statements,
37 1
B’sl day’s work = -
240 12 5x  4 17
=
x 4 5
37  20 17
= =  25x + 20 = 17x + 68
240 240
 8x = 48  x = 6
240  Mother’s present age = 5 × 8 = 40 years
 B will complete the work in days. 56. (2) From statement I,
17
49. (5) From statement (I), x - y = 0.
From statement II,
x  10  r x + y = 18
x =
100 It is possible only when x = y = 9.
 r = 10% / annum 57. (4) Data are inadequate.
From statement (II), 58. (5) From both statements,
2
Speed of boat in still water
 100 
Principal = Difference   1
 Rate  = (4 + 6) = 5 kmph
2
50. (3) From all three statements,
59. (1) From statement I
M + Sc. + E = 198
Nigam’s marks in Biology = 2 × 42 = 84
Let Abhijit get x marks in English.
60. (3) From statement I,
 Marks in science = x + 12 and
marks in Maths = x + 32 S.I × 100
Rate =
 x + x + 12 + x+ 32 = 198 Time × Principal
 3x+ 44 = 198
1736  100
 3x = 198 - 44 = 154 = = 14% Per annum
6200  2
154 By using C.I.
x =
3
51. (3) The data in statement I alone or in  R 
T

= P  100   1
 1 
statement II alone are sufficient. We can   
find the ratio of profit sharing as ratio of
amounts of investment can be determined we get the required rate of interest.
52. (5) Let the number be 10x + y. 61. (5) From statements I and II,
From statement I, Side of the square
x +y=6 = 196 = 14cm.
Many such combinations are possible.  Circumference of the semi-circle
From statement II = (   2) × radius = 7(   2) cm
x = 2y 62. (4) Data are inadequate.
Taking both the statement together, 63. (3) From statement I,
 3y = 6  y = 2 and hence x = 4 3
 Number = 42 Cost price of the article = × 24000
5
LEARN MATHS FROM S.K. RAJU (9811549822, 9811649822)
25
= Rs. 14400 71. (3) From statement I,
Gain = Rs. (24000 - 14400) = Rs 9600 Circumference of circle
9600 2 =  × diameter = 21  cm
 Gain per cent = × 100= 66 % From statement II,
14400 3
 r2 = 346.5
From statement II,
Cost price = Rs. (24000 - 9600) = Rs. 14400 22
 × r2 = 346.5
7
Hence, profit per cent can be determined. 346.5  7
64. (4) From statement I,  r2 = 22
= 110.25
x2 = 9y
Required answer is not possible From  r = 110.25 = 10.5
statement II,  Circumference = 2  r
4x - 3x = ? = 21  cm
65. (5) From statement I, 72. (4) Data in both th e state men ts are
40 50  20 inadequate
x× = 73. (1) From statement I,
100 100
Required number of pieces
 x = 25
From statement II, 900
=  11
30 72  25 80
y× = Data in statement II are inadequate.
100 100
74. (5) From statement I and II, Selling price of
 y = 60 wrist watch
 x : y = 25 : 60 = 5 : 12
66. (2) From statement II,  131.25 
= Rs.  6400 
M 1D 1 = M 2D 2  100 
 8 × 12 = 5 × D2 = Rs. 8400
8  12 96 1 75. (5) From statement I,
 D2 = = =19 days Numbers = 15, 51, 24, 42, 33, 60
5 5 5
Number divisible by 7 = 42
67. (5) From statement II,
76. (2) From statement I, no equivalence
If the present age of Shyam be x years then
relation among a woman, a boy and a man
Ram’s present age = (x + 7) years
can be established.
From statement I,
From statement II,
x 7 4 M 1D 1 = M 2D 2
=
x 3  6 × 16 = 5 × D2
 4x = 3x + 21 6  16 96
 x = 21  D2 = =
5 5
 Shyam’s age after 6 years
= 21 + 6 = 27 years 1
= 19 days
68. (4) Data from both the statements are 5
inadequate. 77. (4) From both statements,
69. (5) From statements I and II, Rita = Sonu + 5000
Simple interest Sonu = Sohan + 8000
Two equations and can’t established the
 5000  3  5 5000  3  8 
= Rs.    relation.
 100 100  Hence, answer can’t be determined.
78. (5) From statement I,
= Rs. (750 + 1200) = Rs. 1950 Pravin’s present age = 2x years
70. (1) From statement I, Shekhar’s present age = 3x years
Required CP. From statements I and II,
= Rs. (4 × 85 + 3 × 50) 3x = 2x + 6
= Rs. (340 + 150) x=6
= Rs. 490  Pravin’s age after 4 years

LEARN MATHS FROM S.K. RAJU (9811549822, 9811649822)


26
= 2x + 4 = 538 + 162 = 700
= 2 × 6 + 4 = 16 years If the minimum passing percentage be x,
79. (4) From both the statements answer can’t then
be determined. 700  x
Radius = Length of rectangle = 162 + 104 = 266
100
Breadth of rectangle = 22cm.
We get no information about the length of 266  100
rectangle. x= = 38%
700
80. (1) From statement I, 86. (4) We do not have the average salary of D
Average speed of truck and E.
Distance covered From both statements, A + B + C + D + E
= = 5 × 48250 ...(i)
Time taken
C = 1.5 B ...(ii)
220 A + B = 2 × 23500 ...(iii)
= = 20 kmph
11 Clearly, C’s salary cannot be determined.
 Average speed of car = (4 × 20) kmph 87. (3) From statement I,
= 80 kmph CP. = Rs. (640000 - 320000)
From statement II, = Rs. 3,20000
we get no result. 320000
81. (2) From statement I, we will not get radius  Profit per cent = 320000 × 100 = 100%
of circle.
From statement II, From statement II,
Radius of circle = Side of square If the CP. be Rs. x then
S.P. = Rs. 2x
= Area = 144 = 12 cm
Area of circle =  r2 x
Gain percent = × 100 = 100%
x
22 3168
= × 12 × 12 = sq.cm. 88. (3) From statement I,
7 7
S.I. ×100
82. (5) From statement II, Rate =
 CP of 2 tables = Rs. 500 principal × Time
500 11480  100
 CP of 3 tables = 2
× 3 = Rs. 750 =
14350  4
= 20% per annum

From statement I, From statement II,


 CP of 4 chairs = Rs. (2 × 750) If principal be Rs. x, then amount = Rs. 2x.
 CP of 2 chairs = Rs. 750  S.I. = Rs. x, Time = 5 years
 3 tables + 2 chairs S.I. × 100
= 750 + 750 = Rs. 1500 Rate =
Principal × Time
83. (5) From statement II,
Amit’s monthly income x  100
= 2 × 3.5 = Rs. 7 thousand = = 20% per annum
x 5
From statement I, 89. (5) From statement II,
Jyoti’s monthly income Unit digit = 0
= Rs. (7000 + 8500) = Rs. 15500 From statement I,
 Jyoti’s annual income ten’s digit = 9
= 12 × 15500 = Rs. 186000
 Number = 90
84. (5) From both statements, 90. (5) From statements I and II,
7x - 4x = 15 If the length of rectangle be 9x metre and
 3x = 15  x = 5 its breadth be 7x metre, then
 Suraj’s present age 9x × 7x = 252
= 7x = 7 × 5 = 35 years
252
 Suraj’s age after 8 years  x2 = =4
= 35 + 8 = 43 years 97
85. (5) From both statements, x = 4 =2
Maximum marks of the exam

LEARN MATHS FROM S.K. RAJU (9811549822, 9811649822)


27
 Perimeter of rectangle From statement II,
= 2 (length + breadth) x + x + 38 = 180°
= 2 (9x + 7x)  2x = 180°- 38° = 142°
= 32x = 32 × 2 = 64 metre  x = 71°
91. (5) From statements I and II, 96. (2) From statements I and II,
If the total marks be x, then Rate downstream
25 12
x× = 288-128 = 160 =u+v= = 6 kmph
100 2
 x = 640
If the minimum pass percentage be y, then 12
Rate upstream = u - v = = 3 kmph
4
y
640 × = 160 + 64 = 224 1
100
u= (u + v) + (u - v)
2
224  100
y= = 35 1 9
640
= (6 + 3) = kmph
92. (1) From statement I, 2 2
4x + 3y + 5z = 60 From statements I and III,
Again, 2x = y u + v = 6 kmph
2y = z  z = 4x
u
 4x + 6x + 20x = 60 u+ =6
3
 30x = 60  x = 2
 4u = 18
 y = 4; z = 8
 x2 + y + z = 4 + 4 + 8 = 16 18 9
From statement II, u= = kmph
4 2
We have three unknown quantities but two 97. (2) From statements I and II,
equations.
93. (5) From statement I, 240
Speed of train = = 10 m/sec
Arun + Vinay + Suraj + Raju + Pratap 24
= 68 × 3 + 72 × 2 98. (4) From all three statements,
= 204 + 144 = 348 kg Total age of 11 students + 1 teacher
 Arun + Pratap = 348 - 78 - 68 - 46 = 14 × 12 = 168 years
= 156 kg Average age of 11 students + teacher
From statement II, = Average age of 11 students + 3
Arun + Suraj + Vinay + Raju  Average age of 11 students = 14 - 3 = 11
= 68 × 4 = 272 kg years
 Pratap = 348 - 272 = 76 kg Their total age = 11 × 11 = 121 years
Arun = 156 - 76 = 80 kg Teacher’s age = 168 - 121 = 47 years
Clearly, weight of Suraj is the second 99. (3) From statements II and III,
biggest. SI for two years = Rs. 1200
94. (5) From statements I and II, Principal = 10 × 1200 = Rs. 12000
Side of square  576 = 24 cm Thus we can find C.I. and amount.
Area of rectangle = 600 100. (4) From statements I and III.
 l × b = 600
 l × 12 = 600
 l = 50 cm
 Required ratio = 50 : 24 = 25 : 12
95. (3) From statement I,
3x + 5x + 4x = 180
 12x = 180
 x = 15 AB 5
Larger angle of parallelogram = cos60°  BC =
AC 2
= 5x + 34 = 75 + 34 = 109°
 Second angle = 180 - 109 = 71°
LEARN MATHS FROM S.K. RAJU (9811549822, 9811649822)
28
5 Breadth = 2x metre
a = , b = 5 and  = 60°  3x × 2x = 216
2
316
1  x2 = = 36
 We get area by A = 2 absin C 32
x=6
101. (I) From statement I,
 Length =18 metre, Breadth = 12 metre
P  R2 Area of the field with boundary
Difference = = (18 + 2 × 2) (12 + 2 × 2)
10000
= 22 × 16 = 352 sq. metre
P  12  12  Area of boundary = 352 - 216 = 136 sq.
 360 =
10000 m.
360  10000 103. (4) From statement I,
P= H + G + C = 65 × 3 = 195 ...(i)
12  12
From statement II,
Rs. 25000 G=H+6
From statement II, From statement III,
Interest = 2 P - P = Rs, P G-C=C-H
= Principal
 2C = G + H
S.I 100  2C = H + 6 + H = 2H + 6
Principal = Time  Rate
C=H+3
From statement III,  H + G + C = 195
 H + H + 6 + H + 3=195
 Rate 
Time

   1  3H = 195 - 9 = 186
C.I = P 1 
 100   186
H = = 62
3
 12 
2

 6360 = P  100   1

1 104. (1) From statements I and II,
  If the length of the train be x metre, then
x x  250
 28 2  Speed of train = =
 6360 = P  25   1
15 27
 
x x  250
 =
 784  625   P  159 5 9
 6360 = P   =  9x = 5x + 1250
 628  625
 4x = 1250
6360  625
 P= = Rs 25000 1250
159 x= = 312.5 m/sec.
4
Note : You need not calculate during exam.
You are required to examine whether 105. (4) From statement I and II,
requirements of a formula are satisfied or CP. of article =252 - 52 = Rs. 200
not. Discount = 280 - 252 = Rs. 28
102. (2) From statement I, Marked price = Rs. 280
If length = x metre and breadth x
= y metre, then  280 × = 28
100
1 2  x = 10%
y= (x + y) Statement I and III will also give same
4
 2y = x + y  y = x result.
 x2 = 144
 x = 144 = 12 metre
SBI PO EXAMS
1. (5) The question has 3 variables, i.e., the
Area of field = (16)2 - (12)2 = 256 - 144
rate of work done per day by each A, B and
= 112 sq. metre
C. We need to have 3 equations to solve
From statement II and III,
the problem. This is possible only by using
Length 3x metre
LEARN MATHS FROM S.K. RAJU (9811549822, 9811649822)
29
all the 3 statements given. Let the rate of When statements I and II are used,
work done per day by A, B and C be denoted C.I. = S.I. + Difference in two interests.
by x, y and z respectively. = 4500 + 465 = Rs. 4965.
1 4. (2) Statement II can be dispensed with as
x + y = only percent profit is to be found.
6
Let the cost price be x
4  Labelled price = 1.30 x.
y+z= Selling price after 10% discount on labelled
15
price = 0.9 × 1.30 x = 1.17x
3
x+z= 1.17x  x
10 % profit = × 100
Adding above equations we get, x
= 17%
1 4 3 Thus, we see that the value of the cost price
2(x + y + z) = + +
6 15 10 is not required.
589 5. (1) In order to find the average salary of all
2(x + y + z) = the 15 employees,
30
we need to know the total salary of all the
22 15 employees. And for that the average
2 (x + y + z) =
30 salary and the number in each of the three
categories of employees is required.
11
x+y+z = Total salary of 15 employees
30 = (7 × 8500) (5 × 10000) + (3 × 2500)
Therefore, (A + B + C) together can complete = 59500 + 50000 + 7500 = 117000
30  Average salary of 15 employees
the work in
11 117000
= = Rs 7800
8 15
=2 days. 6. (3) Let the area of the hall be x sq. m.
11
Then total material cost
2. (5) The rate of painting has not been given
= Rs. 250 x
in any of the statements. Further, we need
Labour cost = 3500
to kow the exact dimensions of the hall for
which any two of the above statements will  250x + 3000 = 14,500 (total cost)
By this equation, we can find area = x
be required.
Hence, all three are required.
3. (5) Simple interest is the interest earned
7. (4) From statement I,
only on the Principal sum. But Compound
CP. of article = 252 - 52
interest is equal to the sum of the Simple
= Rs. 200
interest and the interest earned on this
From statement II.
Simple interest during this period. So, we
Marked price = 200 + 800 = Rs. 280
should know the amount of the Simple
interest and the rate of Interest to calculate  Discount = Rs. (280 - 252) = Rs. 28
the additional interest or the difference 28  100
between them or, we should know the  Discount % = 280
= 10%
difference in the two Interests and the From statement III, Profit % without
Simple Interest. Only then we can find the discount = 40%
amount of Compound interest during that  Profit = 80
period. Therefore, either statement II or III By this discount % can be calculated
can be dispensed with. 8. (1) When a train crosses a signal post it
When statements I and II are used, travels its own length.
 r 2  300r  30000  When it crosses a platform its crosses its
C.I = S.I   length and the length of platform.
 30000  Let the length of train = x m.
100  3000  30000  From statements (I) and (II) we can write,
= 4500   x  250
 30000 x
=
= Rs. 4965 13 27
LEARN MATHS FROM S.K. RAJU (9811549822, 9811649822)
30
Now x can be determined Speed of train Hence, its height can be obtain ed by
x 4 2
= m/s equating volume of the cylinda to r
13 3
9. (4) From statement I. 15. (3) From statement I,
Population of State A The diagonal of the square = x inches.
100 1 2
= × l.61 lakh  Area of the square = 2 x
115
From statements (II) and (III)
1 2
Population of State A Now, x = r 2
2
7 where r is the radius of the circle From this
= × 1.6 lakh
8 relation, r can be determined and hence the
10. (5) The no. of workers can be determined circumference.
by any of the three pieces of information. From statement II,
11. (1) From s ta te me n t I , the difference Area of the square = y2
between selling price and cost price gives Now, y2 =  r 2 gives us the value of radius
profit. The data in statement II alone are
and hence the circumference.
not sufficient to answer the question.
16. (3) From statement 1,
12. (4) From statement I.
A is an even number.
Let the speed of each train be u m/s and
Therefore, the product of A and any other
the length of each train be y metres.
positive number will be an even number.
2y From statement II,
 2u = x A × B = even number
A × C = even number
y
or, u = Obviously, at least one of these three is an
x even number. Hence their product will be
From statement II, an even number.
y 17. (2) From statement I,
=5 Total weight of 60 students
u
Obviously, the data even in both statements = 60 × 42 = 2520kg
I and II together are not sufficient to answer From statement III,
the question as in that case the length of Total weight of all the girls = 1144 kg
train is required.  Total weight of all the boys = 2520 - 1144
13. (1) We have the digit 0, 1,2, 3, 4, 5, 6, 7, 8 = 1376 kg.
and 9. From statement II,
From statement I. 1376
9 - 0=9 Number of boys = = 32
32
Hence, the number is 90.  Number of girls = 60 - 32 = 25
From statement II, From statement III,
9 + 0 = 9, 9 - 0 = 9
2 + 0 = 2, 2 - 0 = 2 etc. 1144
28 
Average weight of girls = 40.86 kg
Hence, many numbers are possible.
14. (2) When a spherical ball is melted and 18. (1) From statement I and III,
made into a right circular cylinder the S.P. can be obtained.
volume remains same, From statements II and III,
i.e., Volume of the cylinder = Volume of the Let the SP be Rs. x.
spherical ball. After 10% discount,
Again, volume of the cylinder = Area of the 9
base × height SP = 90% of x = x
From statement II, 10
Area of the cylinder is given. 9
 x - 15000 = 1200
10
 9x = 150000 + 12000

LEARN MATHS FROM S.K. RAJU (9811549822, 9811649822)


31
162000  Volume =  × (radius)2 × height
x= = Rs. 18000 From statements I and III,
9
Area of base = X sq. metres.
From statements I and II, Hence, radius can be determined.
Let the CP = Rs. x Height = 2 × radius.
120 6x Clearly, after knowing height and area of
 S.P. = x = Rs. base, volume can be determined.
100 5
Now, after 10% discount on 22. (3) S.P. of the product = Rs. 100
From statement I,
6x 90 6x
Rs. , New S.P. = × 100
5 100 5 CP. = × 90 = Rs. 75
120
54x
Rs.  Actual profit =100 - 75 = Rs. 25
50 From statement II,
54x Let the CP. be Rs. x
 - x = 1200
x
50
Then profit =
 54x - 50x = 50 × 1200 3
50  1200 x
x= = Rs. 15000 x + = 100
4 3
Hence S.P. can be calculated as S.P. Rs.  4x = 300
6x  x = Rs. 75
5  Profit = Rs. 25
19. (5) From statement I, 23. (4) Time taken in seconds to cross
Let the length of the train be x metre. Sum of length of both trains in metres
 Length of platform 3x =
Relative speed in m/ sec.
3x
= metre yz
2 x = u
From statement II,
where z m is the length of other train.
3x To know u, z is required.
x
24. (5) Let the digits at unit’s and ten’s
Speed of train = 2
25 places be x and y respectively.
When x > y then x - y = 6
5x x Then x - y = 6
= = ...(i)
50 10 and x + y = 12 gives
But x is not known. Hence, we proceed. x = 9 and y = 3
From statement III, x Again, where y > x
y-x=6
x
Speed of train = ...(ii) Hence, y - x = 6
19 and x + y = 12 gives
Clearly, we reach at no unique conclusion. y = 9 and x = 3
20. (4) From statement I, Hence, the numbers are 93 or 39.
80% children speak languages Obviously, when statement I is not taken
other than Hindi. into consideration, two numbers are
From statement II, possible.
 Total number of children 25. (1) From statements I,
44  100 X
= = 55 = odd number. It is possible only when
80 5
21. (5) From statements I and II, X is odd.
Volume of tank = XY cubic metres From statement II,
From statements II and III, X + Y = an odd number
Y But, X = 2 and Y = 5 gives
Radius of base = X + Y = 7, an odd number
2
LEARN MATHS FROM S.K. RAJU (9811549822, 9811649822)
32
Hence only statement 1 is sufficient to 5
answer the question. x - x = 15 x
6
26. (3) From statement I,
Capacity of tank x
=  × 14 × 14 × 28 = 17248 m3.  = 15  x = 90
6
From Statement II, 32. (3) From statement I,
Capacity of tank = 616 × 28 = 17248 m3.
Obviously, either statement I or statement S.I.× 100
P=
II is required. R×T
27. (4) In  ABC,  BAC = 90° 400  100
  BAC +  ABC +  ACB = 90° = =Rs 4000
52
From statement A, Using,
 ACB = 35° T
  ABC = 90° - 35° = 55°  R 
A = 1   Amount can be determined.
From the statement B,  100 
 CBA = 55° From statement II,
28. (3) From the statement A,
X - Z = 4 or Z - X = 4 PR 2
From the statement B,  
Difference = 100 2

Y We can find principal and hence amount.


= 14 33. (4) From both the statements, we get no
3
 Y = 14 × 3 = 42 result.
34. (5) From both statements,
Taking both statements together,
r1 = 24 cm
we have
r2 - r1 = 3 cm
X is the first number and Z is the last
number or vice versa as the difference  r2 = r1 + 3 = 24 + 3 = 27 cm
between the two consecutive even numbers  Required ratio
is 2.  r22 r22 272
 X = 40, Y = 42, Z = 44 =
 r12 =
r12 =
Or, Z = 40 , Y = 42, X = 44
242
29. (5) P + Q + R + S + T + U 84
= Rs. (6 × 35000) = = 21 : 16
64
= Rs. 210000 .....(i)
35. (5) From both the statements,
From the statements A and B,
If the length of the train be y metre,
Q + S = 54000 .....(ii)
T + U = 58000 .....(iii) y
Speed of train = m / sec
From all these equations, we can find 9
P + R i.e. total salary of P and R by equation
5y
(i) - (ii) - (iii) = kmph
30. (1) Principal = Rs. 6000 9  18
From the statement A,  5y  15 y  100
S.I. = Rs. 3600 Again   60  × =
 9  18  60  60 1000
T = 4 years
 5y  15 y  100
  162  60  ×
S.I ×100 = ....(i)
Rate = Principal × Time   36 10
will give the result. It is to be noted that when a train crosses a
pole, the distance covered = length of the
Statement B alone will give no result.
train.
31. (2) Statement A will give no result as it is
When it crosses another train, Relative
superfluous.
speed × time = sum of lengths of both trains.
From the statement B,
Hence, from equation (i), we can determine
If the number be x,
length of train.
5
 x = x - 15
6
LEARN MATHS FROM S.K. RAJU (9811549822, 9811649822)
33
36. (5) From statement II,  Number = 10y + x
length = x cm, breadth = (x - 5) cm From statement I,
 2 (x + x - 5) = 50 y- x=5 ...(i)
 2x - 5 = 25 From statement II,
 2x = 25 + 5 = 30 y+x=7 ...(ii)
 x = 15 cm = length From (i) and (ii), x, y can be calculated and
two digit number can be found.
 Breadth = 15 - 5 = 10 cm
From statement I, 5. (4) Let the distance between A ard B be z
km.
r 2 Again, let speed of boat in still water be x
= 15 ×10 kmph and that a stream be y kmph.
2
 Rate downstream = (x + y) kmph
  r 2 = 300 Rate upstream = (x - y) kmph
300
From statement I,
r=
 z
= =2
This gives us value of radius. x y
From statement II,
RBI GRADE-B OFFICER EXAMS z
1. (4) Time taken in crossing each other =4
x y
Total length of trains 6. (4) From both the statements,
=
Relative speed Number = 53 or 35
The information given in both statements 7. (1) From statements I,
is not sufficient length of train A and 0+5=5
individual speed of each train are required. From statement II,
2. (4) Area of rectangle = Area of triangle. 5 + 0 = 5 and 5 - 0 = 5
From the information given in both the  6  4 = 6 + 4 = 10
statements, we can find area of triangle or 8. (1) From statement I,
area of rectangle. For finding lenght breadth Total time = 6 hours
is required, which is not known.  The work was completed at 3 p.m.
3. (3) From the statement I, 9. (3) From statements I,
Salary recieved in each paycheques
100  100
r= = 10% 618000
1000 = Rs = Rs 27750
Thus we have, 24
P = Rs. 1000, r = 10% , From statements II,
t = 3 years Salary received in each paycheque
Hence, C.I. can be determined
 618000 
From the statement II. = Rs.   = Rs 25750
 24 
100  r  2
S.I = = 20r We get answer also from statement II.
100 10. (5) From both the statements,
 2
 if x  84, then y  2
r 
CI =1000 1  100   1  x + y > 85 ...(ii)
   INSURANCE EXAMS
 C.I. - S.I. 1. (1) Area of a rectangle = l × b
Where l = Length of the rectangle
 200r  r 2 
= 1000   - 20r b = Breadth of the rectangle
 10000  From the statement A
 2000r + r2 - 200r = IO0 Perimeter of the rectangle
 r = 10% = 2 (l + b) = 60
Hence, C.I. can be determined l + b = 30 — (1)
4. (5) Let the unit’s digit be x and ten’s digit From the statement B
be y and x < y. b = 12 cm —- (2)

LEARN MATHS FROM S.K. RAJU (9811549822, 9811649822)


34
l = 30 -12 = 18 cm. Distance 8
Area = 18 × 12 Time = =
Velocity 13
So for calculating the area of a rectangle
the statements A as well as B are necessary. Therefore, to answer the question all data
2. (4) From the statement A given in statements A, B and C are required.
R>P>Q 6. (3) Let the area of the hall be x sq. m.
From the statement B, Then total material cost
R>M = Rs. 250x
From the statements A and B it is clear that Labour cost = Rs 3500
the R is the tallest among M, P, Q, R  250x + 3500 = 14,500 (total cost)
3. (3) Let x is Samir’s present age and y is the By this equation, we can find area = x
Anil’s present age Hence, all three are required.
From the statement A 7. (4) From statement I,
x=y+4 CP. of article = 252 - 52
From the statement B = Rs. 200
y = 20 years From statement II.
From the statement C, Marked price = 200 + 80 = Rs. 280
y:x=3:4  Discount = Rs. (280 - 252) = Rs. 28
 x:y=4:3 28  100
Therefore, the only statement C is sufficient  Discount % = 280
= 10%
to answer the question. From statement III,
4. (5) From given statement, Profit % without discount = 40%
Sum of money borrowed by x
 Profit = 80
= unknown By this discount % can be calculated
C.I = ? T = 2 8. (1) When a train crosses a signal post it
To calculate C.I. the formula travels its own length. When it crosses a

t
r  platform its crosses its length and the
= P 1   1 length of platform.
 100 
Let the length of train = x m.
Where P  sum of money From statements (I) and (II) we can write,
r  rate of interest annually
t  time x x  250
=
From the statement A, 13 27
Annual rate = 5% Now x can be determined
From the statement B x
P r t Speed of train = m/s
13
S.I. =
100 9. (4) From statement I,
P 52 Population of State A
600 = 100
100 = × 1.61 lakh
115
600  100
P = From statements (II) and (III) Population of
10
State A
= Rs.6000
Now from the statement A, B and C, we have 7
= × 1.6 lakh
all data required to calculate the C.I. So 8
statements A, B and C are necessary to 10. (5) The no. of workers can be determined
solve the question statement. by any of the three pieces of information.
5. (4) In still water the velocity of boat is 11. (4) Taking all statements together,
= 15 km/hour Let the profit earned by company in 2001
Velocity of the stream = 2km/ hour = Rs. x and in 2002 = Rs. y
Relative velocity of the boat  Profit earned in 2003 = 1.4x,
=15 - 2 km/hour = 13 km/hour x + y = Rs. 20 crore ....(i)
Distance = 8 km From statement III,

LEARN MATHS FROM S.K. RAJU (9811549822, 9811649822)


35
80 18. (2) A + B + C + D = Rs. (4 × 62880)
1.4x = y × From statement B,
100
A + C + D = Rs.(3 × 61665)
4 1 4  B’s salary = (A + B + C + D)’s
x= × y = y ...(ii) salary - (A + C + D)’s salary
5 1.4 7
From equations (i) and (ii), we can get the 19. (3) From statement A,
required profit. The three digit number is divisible by 9.
12. (5) The equation cannot be answered even From statement B,
with all I, II and III. Number = 6 * 6
13. (2) From statement I, A number is divisible by 9 if sum of its digits
Observer = Assistant + Rs. 12000 ...(i) is divisible by 9.
From statement II, Clearly, * = 6
Observer + Assistant = Rs. 32000 ...(ii) because 666 * 9 = 74
From statement III, 20. (4) From statement A,
Manager + Observer = Rs. 57000 ...(iii) Let CP of 1 printer = Re. 1
We can find the salary of an assistant from  CP of 5 printers = Rs. 5
equations (i) and (ii) only. and SP of 5 printers = Rs. 6
14. (5) The question cannot be answered even 1
with all I, II, and III.  Gain% = × 100 = 20%
5
15. (1) From statements I and III we can get
the share of B in the profit. 100
 CP = 120 × 3000 = Rs.2500
16. (4) From statement A,
3 × 5 = 15 ; 5 × 9 = 45 (An odd number)  Gain = Rs. (3000 - 2500) = Rs. 500
It is also obvious from statement B. From statement B, we can also find the
17. (5) The answer is not possible with the help answer.
of even both the statements. We need more
information like sum or average of their
ages or ratio of their after some time or
before sometime etc.

LEARN MATHS FROM S.K. RAJU (9811549822, 9811649822)


36
MODEL EXERCISES
Directions (1-5): Each of these has a 5. Nitu and Arun start walking towards each
question followed by two statements numbered I other simultaneously. What is the distance
and II. Mark answer as between them when they start ?
(1) if statement I alone is sufficient but I. 30 minutes after crossing each other they
statement II alone is not sufficient to were 1200 m apart.
answer the question. II. After crossing each other, Nitu reaches
(2) if statement II alone is sufficient but the starting point of Arun in twice as much
statement I alone is not sufficient to time as Arun takes to reach the starting
answer the question. point of Nitu
(3) if both statements I and II together are Directions (6-10): Two quantities A and B
required to answer the question but are given in each of the following questions. Mark
neither statement alone is sufficient. answer as
(4) if statements I and II together are not (1) if Quantity A is greater than Quantity B.
sufficient to answer the question. (2) if Quantity B is greater than Quantity A.
1. If a certain village is losing 12% of its water (3) if Quantities A and B are equal.
supply each day because of a burst water (4) if comparison cannot be made.
pipe, then what is the loss in rupees per 6. A. The number of ways you can divide 6
day ? different candies to 4 different people.
I. The daily water supply to the village is B. The number of ways you can divide 7
700 m gallon. different candies to 3 different people.
II. The cost to the village for every 24000 7. Arun and Rakesh bought the same brand
gallons of water lost is Rs 25. of car. Arun got a 10% discount from the
2. Dipesh had a lawyer’s appointment on a dealer after which 7% sales tax was added.
certain day. Was the appointment on a For Rakesh 7% tax was added and only
Tuesday ? then a 10% discount was given.
I. Exactly 60 hours before the appointment, A. The amount Arun paid for the car.
it was Sunday. B. The amount Rakesh paid for the car.
II. The appointment was between 1.00 pm 8. The average height in the class didn’t
and 9.00 pm. change after Shaan and John left and Aarti
3. What is the weighted average of marks and Geeta came.
obtained by Ashokan ? A. The height of Shaan.
I. Math, Science and English have weights B. The height of Aarti.
5, 4 and 3 respectively. 9. Travis received an allowance. With a fifth
II. Simple arithmetic mean of Math and of the money, he bought a ticket to a movie,
Science is 140, which is twice his and with a quarter of what is left he bought
average of Science and English. a slice of pizza.
4. Total amount of Rs 38,500 was distributed A. The price of a movie ticket.
among Anuj, Sonam and Deepak. How B. The price of a slice of pizza.
much does each get ? 10. Mukesh and Vijay each have some stamps
2 while Sharon has none. They decided to give
I. Anuj gets of what the other two Sharon half of the number that Mukesh has
9
and half of the number that Vijay has.
together get. A. The average number of stamps that
3 Mukesh and Vijay have before giving
II. Sonam gets of what the other two Sharon any stamps.
11
together get. B. The number of stamps that Sharon gets
from Mukesh and Vijay.

LEARN MATHS FROM S.K. RAJU (9811549822, 9811649822)


37
SHORT ANSWERS 3
1. (3) 2. (3) y= [x + 38500 - x - y]
11
3. (4) 4. (3) Similarly as above, we can get the value of
5. (4) 6. (2) y from this equation. Hence, amount got
7. (3) 8. (4) by each can be determined to find which,
9. (3) 10. (3) both of the statements are required.
5. (4) Answer can’t be given using both the
EXPLANATIONS statements also
1. (3) Loss of water supply = 12% 6. (2) A Number of ways to divide 6 different
Therefore, using statement I, candies to 4 different people
Loss of water supply 6! 65
= 700 million gallon × 12% = 84000000 gallon = = =5
2!4! 2
Now, using statement II, we can find the
B. Number of ways to divide 7 different
loss in rupees
candies to 3 different people
Thus, both the statements are required
to answer the question. 7! 7 65
= 7 P3 = = = 35
2. (3) From statement I, it is not evident that 4!3! 32
appointment was on Tuesday or not. For A<B
ex ampl e, if w e assu me th at the 7. (3) Let marked price of the car be Rs. x.
appointment was at 10 am. then before 60 A. Amount given by Arun for the car
h. it is not Sunday but when we assume = 107% of 90% of x
that the appointment was at 2 pm, then = 107 × 0.9 × x = 0.963 x
before 60 h, it is Sunday. So, statement I B. Amount given by Rakesh for the car
alone is not sufficient to answer the = 90% of 107% of x = 0.963 x
question Thus, A = B
But with the help of statement II, we can 8. (4) From the given statement,
say that if the appointment was between Total height of Shaan and John = Total
1:00 pm and 9:00 am, then before 60 h, it height of Aarti and Geeta But we can’t find
is certainly Sunday. the separate heights of all the four students.
3. (4) From statement II, Thus, comparison between A and B can’t
M S 1 S  E  be made.
= 140 = 
2  2 
9. (3) Let allowance got by Travels be Rs. x.
2
Then, price of the ticket to the x movie
 M + S = 280  S + E = 560
From these equations, we can’t find the
x
= Rs.
values of his marks in Math, Science and 5
English. Then, price of the slice of pizza
In statement I, only weights of subjects are 1 x x
x   = Rs
4 
given. Thus, both the statements are not =
5 5
sufficient to answer the question.
4. (3) Let the amounts got by Anuj, Sonam and x
A. Price of the movie ticket = Rs.
Deepak be x, y and 5
{38500 - (x + y)}, respectively. x
From statement I, B. Price of the slice of pizza = Rs.
5
A = B
y  38500   x  y 
2
9
x= 10. (3) Let Mukesh and Vijay have x and y
number of stamps, respectively.
2
=
9
y  38500  x  y  A. Average number of stamps
x y
2 Mukesh and Vijay =
 x =  38500  x  2
9 B. Number of stamps that Sharon gets from
 9x = 77000 - 2x Mukesh and Vijay
 11x = 77000
x y x y
 x = 7000 + = A = B
From statement II, 2 2 2
LEARN MATHS FROM S.K. RAJU (9811549822, 9811649822)
1
DISCOUNT
NATIONALISED BANKS SBI PO EXAMS
& IBPS PO/MT/SO 1. A shopkeeper sells 25 articles at 45/- per
1. Deepa bought a calculator with 30% article after giving 10% discount and gains
discount on the listed price. Had she not 50% proflt. If the discount is not given what
got the discount, she would have paid Rs. is the proflt gained ?
82.50 extra. At what price did she buy the 2
calculator ? (1) 60% (2) 60 %
3
(1) Rs. 192.50
(2) Rs. 275 2
(3) Rs. 117.85 (3) 66 % (4) 66%
3
(4) Cannot be determined (5) None of these
(5) None of these (SBI Associate Banks
Canara Bank PO Ezam. 09.02.2003) PO Exam. 16.07.2000)
2. Naresh purchased a TV set for Rs. 11,250 2. Deepa bought a calculator with 30%
after getting discount of 10% on the labelled discount on the listed price. Had she not
price. He spent Rs. 150 on transport and got the discount, she would have paid Rs.
Rs. 800 on installation. At what price 82.50 extra. At what price did she buy the
should it be sold so that the proflt earned calculator ?
would have been 15% if no discount was (1) Rs. 192.50
offered ? (2) Rs. 275
(1) Rs. 12937.50 (2) Rs. 14,030 (3) Rs. 117.85
(3) Rs. 13,450 (4) Rs. 15467.50 (4) Cannot be determined
(5) None of these (5) None of these
(United Bank of India (SBI Associate Banks
PO Exam. 21.06.2009) PO Exam. 07.01.2007)
3. Manoj sold an article for Rs. 15,000. Had 3. While selling a watch, a shopkeeper gives
he offered a discount of 10% on the selling a discount of 15% . If he gives a discount of
price he would have earned a profit of 8% . 20% , he earns Rs. 51 less as proflt. What
What is the cost price ? is the original price of the watch ?
(1) Rs. 12,500 (2) Rs. 13,500 (1) Rs. 920 (2) Rs. 985
(3) Rs. 12,250 (4) Rs. 13,250 (3) Rs. 1125 (4) Rs. 1020
(5) None of these (5) None of these
Andhra Bank PO Exaxn. 05.07.2009) (SBI PO Preliminary (Tire-I)
4. A man buys a single apple for Rs. 25. If he Exam. 27.07.2008)
were to buy a dozen apples, he would have
to pay a total amount of Rs. 250. What
wo ul d be the approx imate pe rcen t RBI GRADE-B OFFICE EXAMS
discount he would get on buying a dozen 1. A shopkeeper sold a T.V. set for Rs. 17,940,
apples ? with a discount of 8% and gained 19.6% If
(1) 32 (2) 20 no discount is allowed, what will be his gain
(3) 12 (4) 8 per cent ?
(5) 17 (1) 25%
(Bank Of India Banking Officer (2) 26.4%
Exam. 24.01.2010) (3) 24.8%
5. A discount of 15% is given on the marked (4) Cannot be determined
price of an article. The shopkeeper charges (5) None of these
sales tax of 6% on the discounted price. If (RBI Grade-B Officer Exam.17.11.2002)
the selling price be Rs. 1081.20, what is
the marked price of the article ? INSURANCE EXAMS
(1) Rs. 1185.20 (2) Rs. 1250.20 1. A company decided to sell Rs. 50.000, T.V.
(3) Rs. 302 (4) Rs. 1200 set for Rs. 46,000/- as a world cup offer.
(5) None of these What is the percentage discount offered by
(Indian Overseas Bank PO the company ?
Online Exam. 01.09.2013)
LEARN MATHS FROM S.K. RAJU (9811549822, 9811649822)
2
(1) 7.5 (2) 7 5. A shopkeeper marks his goods at such a
(3) 6.5 (4) 8.5 price that after allowing a discount of 12.5%
(5) 8% on the marked price, he still earns a profit
(United India Insurance Co. of 10% . The marked price of an article
AAO Exam. 21.04.2002) which costs him Rs. 4,900 is
2. The labelled price of a cupboard is Rs. (1) Rs. 5,390 (2) Rs. 6,160
6,500/-. The shopkeeper sold it by giving (3) Rs. 5,490 (4) Rs. 6,260
5% discount on the labelled price and (New India Assurance
earn ed a pro fi t of 15% . Wh at AO Exam. 25.10.2009)
approximately is the cost price of the 6. A T-shirt marked at Rs. 400 is sold for Rs.
cupboard ? 360. The rate of discount is :
(1) Rs. 5,800/- (2) Rs. 5,600/- (1) 12 (2) 10
(3) Rs. 5,000/- (4) Rs. 6,000/- (3) 15 (4) 17
(5) Rs. 5,369/- (NICL (GIC) AÓ (Flnance)
(LIC Assistant Adminiatrative Exam. 08.09.2013 (Paper-I)
Officer (AAO) Exam. 24.04.2005) 7. The difference between a discount of 40%
3. On a Rs. 10,000 payment order, a person on Rs. 500 and two successive discounts
has choice between 3 successive discounts of 36% and 4% on the same amount is
of 10% , 10% and 30% , and 3 successive (1) Rs 0 (2) Rs 2
discounts of 40% , 5% and 5% . By choosing (3) Rs 1.93 (4) Rs 7.20
the better one he can save (in Rupees) : (NICL (GIC) AO Exam.
(1) 200 (2) 255 08.09.2013 (Paper-I)
(3) 400 (4) 433 8. A dealer offers a discount of 10% on the
(United India Insurance Co. marked price of an article and stili makes a
(AAO) Exam. 11.03.2007) profit of 20% . If its marked price is Rs. 800,
4. A book is listed at Rs. 150, with a discount then the cost price is:
of 20 per cent. What additional discount (1) Rs. 600 (2) Rs. 700
must be offered to bring the net price to (3) Rs. 800 (4) Rs. 900
Rs. 108 ? (NICL (GIC) Administrative
(1) 8 per cent (2) 10 per cent Offlcer Exam. 15.12.2013)
9. A seller allows a discount of 5% on a watch.
1
(3) 12 per cent (4) 15 per cent If he allows a discount of 7% , he earns Rs
2 15 less in the profit What is the marked
(LIC Assitant Administrative price ?
Offlcer (AAO) Exam. 07.06.2009) (1) Rs. 697.50
(2) Rs. 712.50
(3) Rs. 750
(4) Rs. 817.50
(NICL (GIC) AO (Finance)
Offlcer Exam. 15.12.2013)

LEARN MATHS FROM S.K. RAJU (9811549822, 9811649822)


3
SHORT ANSWERS  x = Rs. 12500
NATIONALISED BANKS 4. (5) C.P in first case
= Rs. (25 × 12) = Rs. 300
& IBPS PO/MT/SO
 Discount = Rs. 50.
1. (1) 2. (2)
If the discount be x % , then
3. (1) 4. (5)
5. (4) 300  x
= 50
100
SBI PO EXAMS
50  100
1. (3) 2. (1) x= = 16.67
3. (4) 300
5. (4) If the marked price of that article be Rs.
RBI GRADE-B OFFICER EXAMS x, then
Price after discount of 15%
1. (5)
85 x 17 x
= Rs. = Rs.
INSURANCE EXAMS 100 20
1. (5) 2. (5)
 17 x 106 
3. (2) 4. (2) Actual S.P. = Rs.   
5. (2) 6. (2)  20 100 
7. (4) 8. (1)
17 x 106
9. (3)   = 1081.20
20 100
EXPLANAT10NS 1081.20  20 100
NATIONALISED BANKS x = = Rs. 1200
17  106
IBPS & PO/MT/SO
1. (1) Let the listed price be Rs. x. SBI PO EXAMS
Discount = 30% of x 1. (3) Total marked Price of article
30 x 3x = 25 × 45 = Rs. 1125/-
= = Rs. Selling Price (Giving 10% discount)
100 10
According to the question, 90
= of 1125 = Rs 1012.5
3x 100
= 82.5
10 1012.50
CP = × 100 = Rs 675
82.5 10 150
x= = Rs. 275 Now the selling price is Rs. 1125 then profit
3
= 1125 - 675 = Rs 450
 Required cost price of calculator
= 70% of 275 450 2
% profit = × 100 = 66 %
70  275 675 3
= Rs = Rs. 192.50 2. (1) Let the listed price be Rs. x.
100
2. (2) Total cost of the TV set  Discount = 30% of x
= Rs. (11250 + 150 + 800) 30 x 3x
= Rs. 12200 = = Rs.
100 10
 Required SP According to the question,
 115 
= Rs. 12200  
3x
= 82.5
 100  10
= Rs. 14030
3. (1) Let the cost price be Rs. x. 82.5 10
x= = Rs. 275
 90% of 15000 = 108% of x 3
90  15000  Required cost price of calculator
x= = 70% of 275
108

LEARN MATHS FROM S.K. RAJU (9811549822, 9811649822)


4
4. (2) After a discount of 20% cost of the book
70  275
= Rs. = Rs. 192.50 = 80% of Rs. 150
100
3. (4) Let the marked price be Rs. x 150  80
= = Rs. 120
 First S.P. = 85% of Rs. x 100
= Rs. 0.85x Second discount
Second S.P. = 80% of Rs. x = 120 - 108 = Rs. 12
= Rs. 0.8x
12
According to the question,  additional discount% = × 100 = 10%
0.85x - 0.8x = 51 120
5. (2) Tricky approach
51 SP of the article
x= = Rs. 1020
0.05
110
= 4900 × = Rs. 5390
100
RBI GRADE-B OFFICER EXAMS
100
17940  Marked price = 87.5 × 5390
1. (5) MP = × 100 = Rs 19500
92
= Rs. 6160
17940
CP = × 100 = Rs 15000 400  360
119.6 6. (2) Discount% = × 100 = 10%
400
If no discount is given,
7. (4) Si ng le equi vale nt di scou nt for
Profit = 19500 - 15000 = Rs 4500
successive discounts of 36% and 4%
4500
 Profit% = 15000 × 100 = 30%  36  4 
=  36  4  %
 100 
= 38.56%
INSURANCE EXAMS  Difference
1. (5) Required percentage discount = 500 × (40 - 38.56)%
 50000  46000  500  1.44
= × 100 = = Rs. 7.20
50000 100
4 8. (1) Marked Price = Rs. 800
= × 100 = 8%
50 800  90
S.P = = Rs. 720
2. (5) According to question, 100
6500  95 100
Selling Price =
100
= Rs. 6175  CP = 120 × 720 = Rs. 600

6175 9. (3) If the marked price be Rs. x then,


 Cost Price = × 100 (7- 5)% of x = 15
115
= Rs. 5269.96  Rs. 5369 x2
 = 15
3. (2) Selling price in the first case 100
= 70% of 90% of 90% of Rs. 10000
1500
70 90 90 x= = Rs. 750
= × × × 10000 2
100 100 100
= Rs. 5670
Selling price in the second case
= 95% of 95% of 60% of Rs. 10000
95 95 60
= × × × 10000
100 100 100
= Rs. 5415
 Saving = Rs. (5670 - 5415) = Rs. 255

LEARN MATHS FROM S.K. RAJU (9811549822, 9811649822)


5
MODEL EXERCISES
1. What price should a shopkeeper mark on 6. A shopkeeper marks the prices of his goods
an article costing him Rs 153 to gain 20% at 25% higher than the original price. After
after allowing a discount of 15% ? that, he allows a discount of 12% discount.
(1) Rs 162 (2) Rs 184 What profit or loss did he get ?
(3) Rs 216 (4) Rs 224 (1) 15% profit (2) 10% profit
(5) None of these (3) 10% loss (4) 15% loss
2. Sunder purchased an office bag with a price (5) None of these
tag of Rs 600 in a sale where 25% discount 7. A shopkeepers announce the same price of
was being offered on the tag price. He was Rs 700 for a shirt. The first offers successive
given a further discount of 10% on the discounts of 30% and 6% while the second
amount arrived at after giving usual 25% offers successive discounts of 20% and
discount. What was the final amount paid 16% . The shopkeeper that offers better
by Sunder ? discount is more of
(1) Rs 210 (2) Rs 540 (1) Rs 22.40 (2) Rs 16.80
(3) Rs 405 (4) Rs 450 (3) Rs 9.80 (4) Rs 36.40
(5) None of these (5) None of these
3. A bicycle originally costs Rs 100 and was 8. A tradesman gives 4% discount on the
discounted 10% . After three months, it was marked price and 1 article free with every
sold after being discounted 15% . Howmuch 15 articles bought and still gains 35% . The
was the bicycle sold for ? marked price is more than the cost price
(1) Rs 55.5 (2) Rs 95.25 by-
(3) Rs 76.5 (4) None of these (1) 40% (2) 39%
(5) All of these (3) 20% (4) 50%
4. A shopkeeper sold a TV set for Rs 17940, (5) None of these
with a discount of 8% and gained 19.6% . If 9. What is the maximum percentage discount
no discount is allowed, then what will be that a merchant can offer on her marked
his gain per cent ? price so that she ends up selling at no profit
(1) 25% (2) 26.4 % or loss, if she had initially marked her goods
(3) 24.8% (4) 30% up by 50% ?
(5) None of these (1) 16.67% (2) 20%
5. A cash payment that wlll settle a bill for (3) 50% (4) 33.33%
250 chairs at Rs 50 per chair less 20% and (5) None of these
15% with a further discount of 5% on cash 10. An article is listed at Rs 65. A customer
payment is : bought this article for Rs 56.16 with two
(1) Rs 8075 (2) Rs 7025 successive discounts of which one is 10% .
(3) Rs 8500 (4) Rs 7125 The other discount of this discount scheme
(5) None of these that was allowed by the shopkeeper is
(1) 4% (2) 3%
(3) 6% (4) 2.5%
(5) None of these

LEARN MATHS FROM S.K. RAJU (9811549822, 9811649822)


6
SHORT ANSWERS 7. (3) According to the question,
1. (3) 2. (3) Selling price of first shopkeeper.
3. (3) 4. (4) 70 94
5. (1) 6. (2) = 700 × × = Rs 460.60
100 100
7. (3) 8. (4)
9. (4) 10. (1) Selling price of second shopkeeper
80 84
= 700 × × = Rs 470.40
EXPLANATIONS 100 100
1. (3) According to question, Required difference
CP = Rs 153, gain 20% = 470.40 - 460.60 = Rs 9.80
120 8. (4) According to the question,
 SP = 153 × = Rs 183.60 1
100
Discount on articles = ×100 = 6.25%
16
183.60
Now, MP = × 100 = Rs 216 4  6.25
85
Overall discount = -4-6.25 + = - 10%
2. (3) Final amount after giving successive 100
discounts of 25% and 10% Let cost price = Rs 100, then
= 600 × 0.75 × 0.9 = Rs 405 selling price = Rs 135
3. (3) According to question, So, 90% of marked price = 135
90 85 135 100
SP of bicycle 100 × × = Rs 76.50 Marked price = = Rs 150
100 100 90
4. (4) SP = 17940, Discount = 8% Marked price is increased by
17940 150  100
 MP = 0.92 = Rs 19500 =  100 = 50%
100
Gain = 19.6% (given) 9. (4) Let cost price = Rs 100
17940 Marked price = Rs150
 CP = 1.196 = Rs 15000  Discount per cent
New SP without discount = Rs 19500 50
= ×100 = 33.33%
Gain = (19500- 15000) = Rs 4500 150
4500 10. (1) Let the other discount be x%
 Gain percent = 15000 × 100 = 30% 90  100  x 
65 × ×  = 56.16
5. (1) By question, originai price of 250 chairs 100  100 
= 250 × 50 = Rs 12500
56.16 100  100
Price after discount  100 - x =
65  90
80 85 95  100 - x = 96
= 12500 × × × = Rs 8075
100 100 100  x = 4%
6. (2) By question, Profit per cent or loss per
cent.
25  12
= + 25 - 12 - = +10%
100
As the sign is +ve so, there is profit of 10% .

LEARN MATHS FROM S.K. RAJU (9811549822, 9811649822)


1
EQUATIONS AND INEQUATIONS
NATIONALISED BANKS 13. I. x2 + 91 = 20x
& IBPS SO/MT/SO II. 10y2 - 29y + 21 = 0
D irection s (1 -5): Fo r th e tw o gi ve n 14. I. 6x2 + 13x + 5 = 0
equations I and II. II. 9y2 + 22y + 8 = 0
Give answer (1) if p is greater than q. 15. I. (x + y)2 = 784
Give answer (2) if p is smaller than q. II. 92551 = 92567 - y
Give answer (3) if p is equal to q. 3x 21
Give answer (4) if p is either equal to or 16. If 4x + 5y = 83 and 2y = then what is
greater than q. 22
Give answer (S) if p is either equal to or the value of y - x ?
smaller than q. (1) 3 (2) 4
(Canara Bank PO Exam. 09.02.2003) (3) 7 (4) 11
1. 2
I. p + 5p + 6 = 0 (5) None of these
II. q2 + 3q + 2 = 0 (Indian Overseas Bank
2. 2
I. p = 4 PO Exam. 15.06.2008)
2
II. q + 4q = - 4 Directions (17-2 1): In the following
3. 2
I. p + p = 56 questions two equations numbered I and II are
2
II. q - 17q + 72 = 0 given. You have to solve both the equations and
4. I. 3p + 2q - 58 = 0 Give answer (1) if x > y
II. 4q + 4p = 92 Give answer (2) if x  y
5. 2
I. 3p + 17p + 10 = 0 Give answer (3) if x < y
II. 10q2 + 9q + 2 = 0 Give answer (4) if x  y
Directions (6-10): In each of the following Give answer (5) if x = y
questions two equations are given. You have to or the relationship cannot be established
solve the equations and (UCO Bank PO Exam. 22.03.2009)
Give answer (1) if x < y 17. I. x2 - 14x + 48 = 0
Give answer (2) if x  y II. y2 + 6 = 5y
Give answer (3) if x = y 18. I. x2 + 9x + 20 = 0
Give answer (4) if x  y II. y2 + 7y + 12 = 0
Give answer (5) if x > y 19. I. x2 = 529
(Syndicate Bank PO Exam. 10.10.2004) II. y = 529
6. I. 4x2 - 8x + 3 = 0 20. I. x2 + 13x = - 42
II. 2y2 - 7y + 6 = 0 II. y2 + 16y + 63 = 0
7. I. x 2 + x - 6 = 0 21. I. 2x + 3y = 14
II. 2y2- 13y + 21 = 0 II. 4x + 2y = 16
8. I. x2 - x - 6 = 0
II. 2y2 + 13y + 21 = 0 28x 140
22. If 3y + 9x = 54 and 13y = 39 then what is
9. I. x2 = 4
II. y2 + 6y + 9 = 0 the value of y - x ?
10. I. 2x + 3y = 4 (1) -1 (2) -2
II. 3x + 2y = 11 (3) 2 (4) 1
Directions (11-15): In each of the following (5) None of these
questions two equations I and II are given. You (Indian Overseas Bank PO
have to solve both the equations and - Exam. 05.04.2009)
Give answer if Directions (23-27) : In the following
(1) x > y (2) x < y questions two equations numbered I and II are
(3) x < y (4) x > y given. You have to solve both the equations and
(5) x = y or relationship betweei x and y Give answer (1) If x > y
cannot be established Give answer (2) If x  y
(Union Bank of India PO Give answer (3) If x < y
Exam. 7.11.2005 Give answer (4) If x  y
11. I. 4x + 2y = 51 Give answer (5) If x = y or the relationship
II. 15y + 13x = 221 cannot be established.
12. I. 8x2 + 3x = 38 (Corporation Bank PO
II. 6y2 + 34 = LEARN
29y MATHS FROM S.K. RAJU (9811549822, 9811649822)
2
Exam. 22.11.2009) Directions (38-42) : In the following
23. I. x2 - l = o questions two equations numbered I and II are
II. y2 + 4y + 3 = 0 given. You have to solve both the equations and
24. I. x 2 - 7x + 12 = 0 Give answer (1) if x > y
II. y2 - 12y + 32 = 0 Give answer (2) if x  y
25. I. x3 - 371 = 629 Give answer (3) if x < y
II. y3 - 543 = 788 Give answer (4) if x  y
26. I. 5x + 2y = 31 Give answer (5) if x = y or the relationship
II. 3x + 7y = 36 cannot be established
27. I. 2x 2 + 11x + 12 = 0 (Bank Of Baroda PO Exam. 30.05.2010)
II. 5y2 + 27y + 10 = 0 38. I. 3x2 + 8x + 4 = 0
Directions (28-32) : In the following II. 4y2 - 19y + 12 = 0
questions two equations numbered I and II are 39. I. x2 + x - 20 = 0
given. You have to solve both the equations and II. y2 - y - 30 = 0
Give answer (1) If x > y 40. I. x2 - 365 = 364
Give answer (2) If x  y
II. y - 324 = 81
Give answer (3) If x < y
Give answer (4) If x  y 4 7
41. I. + = x
Give answer (5) If x = y or the relationship x x
cannot be established. 5
(Indian Bank PO Exam. 17.10.2010)
2
11 2
28. I. 2x2 + 11x + 14 = 0 II. y - =0
y
II. 4y2 + 12y + 9 = 0
29. I. x2 - 4 = 0 42. I. 225x2 - 4 = 0
II. y2 + 6y + 9 = 0 II. 225y + 2 = 0
30. I. x 2 - 7x + 12 = 0
Directions (43-47) : In the following questions two
II. y2 + y - 12 = 0
equations numbered I and II are given. You have
31. I. x2 = 729
to solve both the equations and —
II. y = 729 Give answer (1) If x > y
32. I. x4 - 227 = 398 Give answer (2) If x  y
II. y2 + 321 = 346 Give answer (3) If x < y
Directions (33-37) : In the following Give answer (4) If x  y
questions two equations numred I and II are given. Give answer (5) If x = y or the relationship
You have to solve both the equations and cannot be established
Give answer (1) if x > y (Central Bank Of India
Give answer (2) if x  y PO Exam. 25.07.2010)
Give answer (3) if x < y 43. I. 5x2 - 18x + 9 = 0
Give answer (4) if x  y II. 20y2 - 13y + 2 = 0
Give answer (5) if x = y or the f.ationship 44. I. x 3 - 878 = 453
cannot be established. II. y2 - 82 = 39
(Punjab & Sind Bank PO 3 4
Exam. 16.05.2010) 45. I. + = x
x x
33. 1. x 2- x - 12 = 0
7
II. y2 + 5y + 6 = 0
34. I. x2 - 8x + 15 = 0
7  2
II. y3 - = 0
II. y2 - 3y + 2 = 0 y
35. I. x 2 - 32 = 112 46. I. 9x - 15.45 = 54.55 + 4x
II. y - l69 = 0 II. y  155 - 36 = 49
36. I. x - 121 = 0 47. I. x2 + 11x + 30 = 0
II. y2 - 121 = 0 II. y2 + 7y + 12 = 0
37. I. x2 - 16 = 0 Directions (48-5 2): In the following
II. y2 - 9y + 20 = 0 questions two equations numbered I and II are
given. You have to solve both the equations and
Give answer (1) if x > y
LEARN MATHS FROM S.K. RAJU (9811549822, 9811649822)
3
Give answer (2) if x  y Directions (59-63) : In the following
Give answer (3) if x < y questions two equations numbered I and II are
Give answer (4) if x  y given. You have to solve both the equations and
Give answer (5) if x = y or the relationship Give answer if
cannot be established (1) x > y
(Syndicate Bank PO Exam. 29.08.2010) (2) x  y
(3) x < y
6 (4) x  y
48. 1. x - = 0
x (5) x = y or the relationship cannot be
3 established
II. y3 - 6 2  = 0 (Bank Of Maharashtra
49. I. 3x - 2y = 10 Exam. 19.12.2010)
II. 5x - 6y = 6 59. I. 20x2 - x - 12 = 0
50. I. x2 + x - 12 = 0 II. 20y2 + 27y + 9 = 0
II. y2 - 5y + 6 = 0 60. I. x2 - 218 = 106
51. I. x2 + 9x + 18 = 0 II. y2 - 37y + 342 = 0
II. y2 - 13y + 40 = 0 7 5
52. I. x  6 = 121 - 36 61. I. + = x
x x
II. y2 + 112 = 473 5
Directions (53-5 7): In the following 12 2
questions two equations numbered I and II are II. y2 - =0
y
given. You have to solve both the equations and —
Give answer (1) if x > y 62. I. 361 x + 16 = 0
Give answer (2) if x  y II. 441 y + 4 = 0
Give answer (3) if x < y
Give answer (4) if x  y 15 2
63. I. - =6 x
Give answer (5) if x = y or the relationship x x
cannot be established. 1
y 7 y
(Bank Of India PO Exam. 31.10.2010) II. + = y
53. I. x2 - 1200 = 244 4 12
II. y + 122 = 159 Directions (64-68) : In the following
54. I. 14x - 25 = 59 - 7x questions two equations numbered I and II are
given. You have to solve both the equations and
II. y  222 - 36 = 81 Give answer If
2
55. I. 144x - 16 = 9 (1) x > y
II. 12y + 4 = 49 (2) x  y
56. I. x2 - 9x + 20 = 0 (3) x < y
II. y2 - 13y + 42 = 0 (4) x  y
(5) x = y or the relationship cannot be
x 3 x 1
57. I. + = established
5 10 x (Oriental Bank Of Commerce PO
10 2 Exam. 26.12.2010 (1st Sitting)
II. y - y = 4 y 64. I. x  18 = l44 - 49
58. If 2x + 3y = 78 and II. y2 + 409 = 473
3x + 2y = 72, what is the value of x + y ? 65. I. x 2 - 7x + 12 = 0
(1) 36 II. y2 - 9y + 20 = 0
(2) 32 66. I. y2- x 2 = 32
(3) 30 II. y - x = 2
(4) Cannot be determined 5
(5) None of these 67. I. x- =0
x
(PNB Management Trainee
Exam. 28.11.2010) II. y3 - 5(3/2) = 0
68. I. 3x + 5y = 28
II. 8x - 3y = 42
LEARN MATHS FROM S.K. RAJU (9811549822, 9811649822)
4
Directions (69-73) : In the following 78. I. (17)2 + 144  18 = x
questions two equations numbered I and II are II. (26)2- 18 × 21 = y
given. You have to solve both the equations and Directions (79-83) : In each of these
Give answer if questions, two equations are given. You have to
(1) x > y solve these equations and find out the values of x
(2) x  y and y and
(3) x < y Give answer If
(4) x  y (1) x < y
(5) x = y or the relationship cannot be (2) x > y
established (3) x  y
(Union Bank Of India (4) x  y
PO Exam. 09.01.2001) (5) x = y
69. I. (Punjab & Sind Bank
289x + 25 = 0
PO Exam. 23.01.2011)
II. 676y + 10 = 0 79. I. 16x2 + 20x + 6 = 0
70. I. 8x2 - 78x + 169 = 0 II. 10y2 + 38y + 24 = 0
II. 20y2 - 117y + 169 = 0 80. I, 18x2 + 18x + 4 = 0
II. 12y2 + 29y +14 = 0
15 9
71. I. + = 11 x 81. I. 8x2 + 6x = 5
x x II. 12y2 - 22y + 8 = 0
1 82. I. 17x2 + 48x = 9
y 5 y
II. + = y II. 13y2 = 32y- 12
4 12 83. I. 4x + 7y = 209
8 6 II. 12x - 14y = -38
72. I. + = x Directions (84-88) : In the following
x x
questions two equations numbered I and II are
7
given. You have to solve both the equations and
14  2 Give answer (1) if x > y
II. y3 - =0
y Give answer (2) if x  y
73. I. x 2 - 208 = 233 Give answer (3) if x < y
II. y2 - 47 + 371 = 0 Give answer (4) if x  y
Directions (74-78) : In the following Give answer (5) if x = y or the relationship
questions two equations numbered I and II are cannot be established
given. You have to solve both the equations and (UCO Bank PO Exam. 30.01.2011
Give answer if 25 12 9 4
(1) x > y 84. I. 2 - + 2= 2
x x x x
(2) x  y II. 9.84 - 2.64 = 0.95 + y2
(3) x < y
85. I. 901 x + 1295 = 0
(4) x  y
II. (257)1/4y + (217)1/3 = 0
(5) x = y or the relationship cannot be
3  7
5 3
established
(Corporation Bank 86. I. = x3
3
PO Exam. 16.01.2011) II. 7y3 = - (15 × 2) + 17y3
74. I. x 2 - 11x + 24 = 0
87. I. (x1/4  16)2 =144  x3/2
II. 2y2 - 9y + 9 = 0 II. y1/3 × y2/3 × 3104 = 16 × y2
75. I. x3 × 13 = x2 × 247
88. I. 3x2 - 19x + 28 = 0
II. y1/3 × 14 = 294  y2/3 II. 5y2 - 18y + 16 = 0
12  4 3  4 10 Directions (89-93) : In the following
76. I. 4 - 4 = x7 questions two equations numbered I and II are
x7 x7 given. You have to solve both the equations and
II. y3 + 783 = 999 Give answer If
77. I. 500 x + 402 = 0 (1) x > y
1/2 (2) x  y
II. 360 y + (200) = 0 (3) x < y

LEARN MATHS FROM S.K. RAJU (9811549822, 9811649822)


5
(4) x  y —
(5) x = y or the relationship cannot be Give answer If
established (1) x > y
(Bank Of Baroda PO Exam. 13.03.2011) (2) x  y
89. I. 1225 x + 4900 = 0 (3) x < y
1 1 (4) x  y
II.  81 4 y +  343  3 = 0 (5) x = y or the relation- ship cannot be
18 6 12 8 established
90. I. + - = (Indian Overseas Bank
x2 x x2 x2
PO Exam. 22.05.2011)
II. y2 + 9.68 + 5.64 = 16.95
99. I. x2 - 19x + 84 = 0
 2  11 = x3
5 3
II. y2 - 25y + 156 = 0
91. I. 100. I. x3 - 468 = 1729
6
II. 4y3 = - (589  4) + 5y3 II. y2 - 1733 + 1564 = 0
92. I. 12x2 + 11x + 12 = 10x2 + 22x 9 19
II. 13y2 - 18y + 3 = 9y2 - 10y 101. I. + = x
x x
 75  11
I.  x  9  = 169  x 5  2  14  2
3
93.
  II. y -5
=0
1
y
1 1
II. y 4 × y 4 × 7 = 273  y 2
102. I. 784 x + 1234 = 1486
Directions (94-98) : In the following II. 1089 y + 2081 = 2345
questions two equations numbred I and II are
given. You have to solved both the equations and 12 23
103. I. - =5 x
Give answer If x x
(1) x > y
y 5 y 1
(2) x  y II. - = y
(3) x < y 12 12
(4) x  y Directions (104-108): In the following
(5) x = y or the relationship cannot be questions two equations numbered I and II are
established given. You have to solve both the equations and
(Allahabad Bank PO Exam. 17.04.2011) (IDBI Bank Officer Exam. 16.09.2012)
Give answer if
94. 25x 2 - 125 = 0 (1) x > y
II. 361 y + 95 = 0 (2) x  y
(3) x < y
5 5 x (4) x  y
95. I. - =
7 21 42 (5) x = y or the relationship cannot be
established
y y 250
II. + = y 15 9 1

x x 
4 16 104. I. - = 2
x
1
96. (625)4 x + l225 = 155 II. y10 - (36)5 = 0
105. I. 5x + 2y = 96
II. 196 y + 13 = 279 II. 3(7x + 5y) = 489
97. I. 5x2 - 18x + 9 = 0 1
II. 3y2 + 5y - 2 = 0 106. I. (441)2 x2 - 111 = (15)2
13 9 II. 121 y2 + (6)3 = 260
98. I. + = x
x x
107.I. 17x = (13)2 + l96 + (5)2 + 4x
9
II. y4 - 13  2 2 II. 9y - 345 = 4y - 260
108. I. 3x2 - 13x + 14 = 0
Directions (99-103): In the following II. y2 - 7y + 12 = 0
questions, two equations numbred I and II are Directions (109-113): In each of the
given. You have to to solve both the equations and following questions two equations are given. Solve
LEARN MATHS FROM S.K. RAJU (9811549822, 9811649822)
6
these equations and give answer : 119. 1. 4x2 - 32x + 63 = 0
(1) if x  y, i.e., x is greater than or equal to y II. 2y2 - 11y + 15 = 0

 
(2) if x > y, i.e., x is greater than y 3
3

(3) if x  y, i.e., x is less than or equal to y 120. I. x3 = 216


(4) if x < y, i.e., x less than y II. 6y2 = 150
(5) x = y or no relation can be established 121. I. 12x2 + 17x + 6 = 0
between x and y II. 6y2 + 5y + l = 0
(IBPS Specialist Officer 122. I. 20x2 + 9x + 1= 0
CWE 17.03.2013) II. 30y2 + 11y + l = 0
109. I. x2 + 5x + 6 = 0 123. I. x2 + 17x +72 = 0
II. y2 + 7y + 12 = 0 II. y2 + 19y + 90 = 0
100. I. x2 + 20 = 9x
II. y2 + 42 = 13y SBI PO EXAMS
111. I. 2x + 3y = 14 Directions (1-3): In each question below
II. 4x + 2y = 16 one or more equation (s) is/are provided. On the
112. I. x = 625 basis of these you have to find out relation between
p and q.
II. y = 676
Give answer (1) if p = q
113. I. x2 + 4x + 4 = 0 Give answer (2) if p > q.
II. y2 - 8y + 16 = 0 Give answer (3) if q > p
Directions (114-118) : In each of the Give answer (4) if p  q and
following questions, two equations are given. You Give answer (5) if q  p
have to solve these equations and
(SBI Banks PO Exam. 20.08.2000)
(IBPS Bank PO/MT
1. I. 4q2 + 8q = 4q + 8
CWE-III 26.10.2013)
II. p2 + 9p = 2p -12
Give answer If
2. I. 2p2 + 40 = 18p
(1) x < y
II. q2 = 13q - 42
(2) x > y
(3) x = y 1 7
3. I. 6q2 + = q
(4) x  y 2 2
(5) x  y II. 12p2 + 2 = 10p
114. I. x2 - 24x + 144 = 0 Directions (4-8): In each of the questions
II. y2 - 26y + 169 = 0 a pair of equations is given. You have to find the
115. I. 2x2 + 3x - 20 = 0 values of x and y and give answer.
II. 2y2 + 19y + 44 = 0 (l) if x < y (2) if x  y
116. I. 6x2 + 77x + 121 = 0 (3) if x = y (4) if x > y
II. y2 + 9y - 22 = 0 (5) if x  y
117. I. x2 - 6x = 7 (SBI Associate Banks PO
II. 2y2 + 13y + 15 = 0 Exam. 21.07.2002)
118. I. 10x2 - 7x2 + 1 = 0 4. I. 2x2 - 7x + 6= 0
II. 35y2 - 12y + 1 = 0 II. 4y2 = 9
Directions (119- 123): In each of the 5. I. 4x2 - 4x - 3 = 0
following questions equation I and equation II have II. 4y2 + 12y + 5 = 0
been given. You have to solve both of these 6. I. 4x2 = 49
equations and II. 9y2 - 66y + 121 = 0
(Corporation Bank Specialist Officer 7. I. x2 + 9x + 14 = 0
(Marketing) Exam. 22.12.2014) II. y2 + y - 2 = 0
Give answer If 8. I. 9x2 - 18x + 5 = 0
(1) x < y II. 2y2 - 9y + 10 = 0
(2) x > y Directions (9-13) : For the two given
(3) x  y equations I and II.
(4) x  y Give answer (1) if p is greater than q
(5) x = y or no relation between two can be Give answer (2) if p is smaller than q
established. Give answer (3) if p is equal to q.

LEARN MATHS FROM S.K. RAJU (9811549822, 9811649822)


7
Give answer (4) if p is either equa to or 23. I. 2p2 + 12p + 16 = 0
greater than q. II. 2q2 + 14q + 24 = 0
Give answer (5) if p is either equa to or Directions (24-28): In each of the following
smaller than q. questions two equations are given. You have to
(SBI Banks PO Exam. 18.05.2003) solve them and
9. I. 6p2 + 5p + 1 = 0 Give answer (1) if p < q
II. 20p2 + 9q = - 1 Give answer (2) if p > q
10. I. 3p2 + 2p - 1 = 0 Give answer (3) if p  q
II. 2q2 + 7q + 6 = 0 Give answer (4) if p  q
11. I. 3p2 + 15p = - 18 Give answer (5) if p = q
II. q2 + 7q + 12 = 0 (SBI Associate Banks PO
Exam. 07.01.2007)
4
12. I. p = 24. I. p2 - 7p = - 12
9 II. q2 - 3q + 2 = 0
II. 9q2 - 12q + 4 = 0 25. I. 12p2 - 7p = - 1
13. I. p2 + 13p + 42 = 0 II. 6q2 - 7q + 2 = 0
II. q 2 = 36 26. I. p2 + 12p + 35 = 0
Directions (14-18) : In each of these II. 2q2 + 22q + 56 = 0
questions two equations numbered I & II are given. 27. I. p2 - 8p + 15 = 0
You have to solve both the equations and give II. q2 - 5q = - 6
answer 28. I. 2p2 + 20p + 50 = 0
(1) if a < b II. q2 = 25
(2) if a > b
(3) if relationship between a & b cannot be RBI GRADE-B OFFICER EXAMS
established 1. Which value of x does satisfy the inequality
(4) if a  b 2x2 + x - 3 < 0 ?
(5) if a  b
3 2
(SBI PO Exam. 09.01.2005) (1) - <x<1 (2) -l < x <
14. I. a2 + 5a + 6 = 0 2 3
II. b2 + 3b + 2 = 0 2
15. I. 2a2 + 3a + 1 = 0 (3) x > 1 (4) x <
5
II. 12b2 + 7b + 1 =0 (5) None of these
16. I. a2 = 4 (RBI Grade-B Officer Exam.17.11.2002
II. a2 = 9 Directions : (2 - 6) In each of the following
17. I. 6a2 - 25a + 25 = 0 questions, one or two equations (s) is/are given.
II. 15b2 - 16b + 4 = 0 On their basis you have to determine relation
18. I. 4a2 - 20a + 21 = 0 betweer x and y and then,
II. 2b2 - 5b + 3 = 0 (RBI Grade-B Officer Exam.17.11.2002)
Directions (19-23): In each question below Give answer (1) if x < y
one or two equation(s) is/ are provided. On the Give answer (2) if x > y
basis of these you have to find out relation between Give answer (3) if x < y
p and q. Give answer (4) if x > y
Give answer (1) if p = q Give answer (5) if x = y
Give answer (2) if p > q 2. I. x2 + 3x + 2 = 0
Give answer (3) if p < q II. 2y2 = 5y
Give answer (4) if p  q and 3. I. 2x2 + 5x + 2 = 0
Give answer (5) if q  p II. 4y2 = 1
(SBI PO Exam. 26.11.2006) j 4. I. y2 + 2y - 3 = 0
19. I. p2 + 24 = 10p II. 2x2 - 7x + 6 = 0
II. 2q2 + 18 = 12q 5. I. x2 + 2x - 8 = 0
20. pq + 30 = 6p + 5q II. y2 - 2 = 7
21. I. q2 + q = 2 6. I. x2 - 5x + 6 = 0
II. p2 + 7p + 10 = 0 II. y2 + y - 6 = 0
22. I. p2 + 16 = 8p Directions (7-11) : For the two given
II. 4q2 + 64 = 32q equations I and II.
LEARN MATHS FROM S.K. RAJU (9811549822, 9811649822)
8
Give answer (1) if p is greater than q. II. y2 = 64
Give answer (2) if p is smaller than q. 21. I. x2 - 463 = 321
Give answer (3) if p is equal to q. II. y2 - 421 = 308
Give answer (4) if p is either equal to or Directions (22-26) : In the fol lowi ng
greater than q. questions three equations numbered I, II and III
Give answer (5) if p is either equal to or are given. You have to solve all the equations either
smaller than q. together or separately, or two together and one
(RBI Orade-B Officer Exam. 2007) separately, or by any other method and—
7. I. p2 + 5p + 6 = 0 (RBI Grade ‘B’ Officer’s
II. q2 + 3q + 2 = 0 Exam. 18.12.2011)
8. I. p2 = 4 Give answer If
II. q2 + 4q = - 4 (1) x < y = z
9. I. p2 + p = 56 (2) x < y < z
II q2 - 17q + 72 = 0 (3) x < y > z
10. I. 3p + 2q - 58 = 0 (4) x = y > z
II. 4q + 4p = 92 (5) x = y = z or if none of the above
11. I. 3p2 + 17p + 10 = 0 relationship is established
II. 10q2 + 9q + 2 = 0 22. I. 7x + 6y + 4z = 122
Directions (12-16) : In each of these II. 4x + 5y + 3z = 88
questions, two equations are given. You have to III.9x + 2y + z = 78
solve these equations and find out the values of x 23. I. 7x + 6y = 110
and y and : II. 4x + 3y = 59
Give answer III. x + z = 15
(1) if x < y (2) if x > y
 1 1

(3) if x  y (4) if x  y 24. I. x =  36  2  1296  4

(5) if x = y
(RBI Grade-B Office Exam. 2008) II. 2y + 3z = 33
12. I. 8x2 + 6x = 5 III.6y + 5z = 71
II. 12y2 - 22y + 8 = 0 25. I. 8x + 7y = 135
13. I. 4x + 7y = 209 II. 5x + 6y = 99
II. 12x - 14y = -38 III. 9y + 8z = 121
14. I. 17x2 + 48x = 9 26. I. (x + y) 3 = 1331
II. 13y2 = 32y - 12 II. x - y + z = 0
15. I. 18x2 + 18x + 4 = 0 III. xy = 28
II. 12y2 + 29y + 14 = 0
16. I. 16x2 + 20x + 6 = 0 INSURANCE EXAMS
II. 10y2 + 38y + 24 = 0 Directions (1-5) : In each of the questions
Directions (17-2 1): In the following a pair of equations is given. You have to find the
questions two equations numbered I and II are values of x and y and give answer.
given. You have to solve both the equations and (l) if x < y (2) if x  y
Give Answer If (3) if x = y (4) if x > y
(1) x > y (5) if x  y
(2) x  y (LIC Assistant Administrative
(3) x < y Officer (AAO) Exam. 2006)
(4) x  y 1. I. 2x2 - 7x + 6= 0
(5) x = y or the relationship can not be II. 4y2 = 9
established 2. I. 4x2 - 4x - 3 = 0
(RBI Grade-B Officer Exam.06.02.2011) II. 4y2 + 12y + 5 = 0
17. I. x2 + 5x + 6 = 0 3. I. 4x2 = 49
II. y2 + 3y + 2 = 0 II. 9y2 - 66y + 121 = 0
18. I. x2 - 10x + 24 = 0 4. I. x2 + 9x + 14 = 0
II. y2 - 9y + 20 = 0 II. y2 + y - 2 = 0
19. I. (x)2 = 961 5. I. 9x2 - 18x + 5= 0
II. y = 961 II. 2y2 - 9y + 10 = 0
20. I. x2 - 72 = x
LEARN MATHS FROM S.K. RAJU (9811549822, 9811649822)
9
SHORT ANSWERS 105. (1) 106. (1)
NATIONALISED BANKS 107. (3) 108. (3)
& IBPS PO/MT/SO 109. (1) 110. (4)
1. (5) 2. (4) 111. (4) 112. (5)
3. (2) 4. (1) 113. (4) 114. (1)
5. (2) 6. (2) 115. (4) 116. (5)
7. (1) 8. (5) 117. (2) 118. (4)
9. (5) 10. (5) 119. (2) 120. (2)
11. (1) 12. (2) 121. (1) 122. (3)
13. (1) 14. (5) 123. (4)
15. (3) 16. (2)
17. (1) 18. (4) SBI PO EXAMS
19. (5) 20. (2) 1. (3) 2. (3)
21. (3) 22. (2) 3. (4) 4. (5)
23. (2) 24. (4) 5. (5) 6. (1)
25. (3) 26. (1) 7. (2) 8. (1)
27. (5) 28. (3) 9. (2) 10. (1)
29. (1) 30. (2) 11. (4) 12. (3)
31. (4) 32. (5) 13. (5) 14. (5)
33. (2) 34. (1) 15. (1) 16. (3)
35. (3) 36. (1) 17. (2) 18. (4)
37. (4) 38. (3) 19. (2) 20. (3)
39. (4) 40. (4) 21. (5) 22. (1)
41. (5) 42. (5) 23. (4) 24. (2)
43. (1) 44. (2) 25. (1) 26. (3)
45. (5) 46. (5) 27. (4) 28. (3)
47. (3) 48. (5)
49. (1) 50. (4) RBI GRADE-B OFFICER EXAMS
51. (3) 52. (2) 1. (1) 2. (1)
53. (5) 54. (1) 3. (3) 4. (2)
55. (5) 56. (3) 5. (1) 6. (4)
57. (5) 58. (3) 7. (5) 8. (4)
59. (2) 60. (4) 9. (2) 10. (1)
61. (5) 62. (3) 11. (2) 12. (3)
63. (1) 64. (5) 13. (5) 14. (1)
65. (4) 66. (3) 15. (4) 16. (2)
67. (5) 68. (1) 17. (4) 18. (2)
69. (3) 70. (2) 19. (5) 20. (2)
71. (1) 72. (5) 21. (5) 22. (1)
73. (5) 74. (2) 23. (3) 24. (2)
75. (3) 76. (5) 25. (4) 26. (5)
77. (3) 78. (3)
79. (2) 80. (2)
INSURANCE EXAMS
81. (3) 82. (1)
1. (5) 2. (5)
83. (5) 84. (2)
3. (1) 4. (2)
85. (1) 86. (1)
5. (1)
87. (3) 88. (1)
89. (1) 90. (3)
91. (1) 92. (2) EXPLANATIONS
93. (5) 94. (1) NATIONALISED BANKS
95. (3) 96. (1) & IBPS SO/MT/SO
97. (1) 98. (3) 1. (5) I.  p2 + 3p + 2p + 6 = 0
99. (4) 100. (2)  p (p + 3) + 2 (p + 3) = 0
101. (5) 102. (1)  (p + 3) (p + 2) = 0
l03. (1) 104. (5)
 p = -2 or -3
LEARN MATHS FROM S.K. RAJU (9811549822, 9811649822)
10
II.  q + q + 2q + 2 = 0
3
 2y2 - 4y - 3y + 6 = 0
 q (q + 1) + 2 (q + 1) = 0  2y (y - 2) - 3(y - 2) = 0
 (q + 1) (q + 2) = 0  (2y - 3) (y - 2) = 0
 q = - 1 or - 2 2
Obviously p  q  y = 2 or
3
2. (4) I.  p =  2 Hence x  y
II.  q2 + 2q + 2q + 4 = 0 7. (1) I. x2 + x - 6 = 0
 q (q + 2) + 2 (q + 2) = 0  x2 + 3x - 2x - 6 = 0
 (q + 2) (q + 2) = 0  x (x + 3) - 2 (x + 3) = 0
q= -2  (x + 3) (x - 2) = 0
Obviously, p  q  x = - 3 or 2
3. (2) I.  p5 + p - 56 = 0 II. 2y2 - 13y + 21 = 0
 p6 + 8p - 7p - 56 = 0  2y2 - 7y - 6y + 21 = 0
 p (p + 8) - 7 (p + 8) = 0  y (2y - 7) - 3 (2y - 7) = 0
 (p + 8) (p - 7) = 0  (2y - 7) (y - 3) = 0
 p = 7 or - 8 7
II.  q2 - 8q - 9q + 72 = 0  y = 2 or 3
 q (q - 8) - 9 (q - 8) = 0 Hence x < y
 (q - 8) (q - 9) = 0 8. (5)1. x 2 - x - 6 = 0
 q = 8 or 9  x 2 - 3x + 2x - 6 = 0
Obviously, p < q  x (x - 3) + 2 (x - 3) = 0
4. (1) We have,  (x + 2) (x - 3) = 0
3p + 2q = 58 ...(i)
 x = 2 or 3
4p + 4q = 92
II. 2y2 + 13y + 21 = 0
 2p + 2q = 46 ...(ii)
 2y2 + 7y + 6y + 21 = 0
By equation (i) - (ii) we get
p = 12  y (2y + 7) + 3(2y + 7) = 0
From equation (i), 3 × 12 + 2q = 58  (y + 3) (2y + 7) = 0
 2q = 58 - 36 = 22 7
 q = 11  y = -3 or 2
Hence, p > q Clearly, x > y
5. (2) I.  3p2 + 15p + 2p + 10 = 0 9. (5) I. x2 = 4
 3p (p + 5) + 2 (p + 5) = 0  x = ±2
 (p + 5) (3p + 2) = 0 II. y2 + 6y + 9 = 0
2  (y + 3)2 = o
 p = - 5 or - y+3 =0
3
II.  10q2 + 5q + 4q + 2 = 0  y = -3
Clearly, x > y
 5q (2q + 1) + 2 (2q + 1) = 0
10. (5) I. 2x + 3y = 4
 (2q + 1) (5q + 2) = 0 II. 3x + 2y = 11
1 2 Multiplying equation I by 3 and equation II
q = - or - by 2 and subtracting equation II from
2 5
Obviously, p < q equation I
6. (2) I. 4x2 - 8x + 3 = 0 6x + 9y - 6x - 4y = 10 - 22
5y = -10
 4x - 6x - 2x + 3 = 0
 y = -2
 2x(2x - 3) - l (2x - 3) = 0 Putting the value of y = -2 in equation I,
 (2x - 1) (2x - 3) = 0 2x + 3x - 2 = 4
1 2  2x = 6 + 4
 x = 2 or
3 10
II. 2y2 - 7y + 6 = 0 x = = 5
2
LEARN MATHS FROM S.K. RAJU (9811549822, 9811649822)
11
Clearly, x > y  9y2 + 18y + 4y + 8 = 0
11. (1) I. 4x + 2y = 51  9y (y + 2) + 4 (y + 2) = 0
II. 13x + 15y = 221  (y + 2) (9y + 4) = 0
Multiplying equation I by 15 an II by 2 and
by I - II, 4
 y = -2 or
60x + 30y - 26x - 30y = 765 - 442 9
34x = 323 Relationship can’t be established.
323
15. (3) II. y = 92567 - 92551 = 16
x= = 9.5 I. (x + y)2 = 784
34
 x + y = 28
From equation I,
 x = 28 - 16 = 12
4 × 9.5 + 2y = 51
Clearly, x < y
 38 + 2y = 51 16. (2) 4x + 5y = 83 ...(i)
 2y = 51 - 38
3x 21 x 21 2 7
2y = 22  y = 22 × 3 = 11
13
 y = 2 = 6.5
Clearly, x > y 7
12. (2) I. 8x2 + 3x - 38 = 0 x= y ...(ii)
11
 8x2 + 19x - 16x - 38 = 0 From equation (i),
 x (8x +19) -2 (8x + 19) = 0 7
 (8x + 19) (x - 2) = 0 4 × y + 5y = 83
11
19  28y + 55y = 913
 x = 2 or -
8  83y = 913
II. 6y2 - 29y + 34 = 0 913
 6y2 - 17y - 12y + 34 = 0 y= = 11
83
 y (6y -17) - 2 (6y - 17) = 0 From equation (ii),
 (y - 2) (6y - 17) = 0
7
17 x =
× 11 = 7
 y = 2 or 11
6  y - x = 11 - 7 = 4
Clearly, x  y. 17. (1) I. x2 - 14x + 48 = 0
13. (1) I. x2 - 20x + 91 = 0  x2 - 8x - 6x + 48 = 0
 x2 - 13x - 7x + 91 = 0  x(x - 8) - 6 (x - 8) = 0
 x (x - 13) - 7 (x - 13) = 0  (x - 6) (x - 8) = 0
 (x - 7) (x - 13) = 0  x = 6 or 8
 x = 7 or 13 II. y2 - 5y + 6 = 0
II. 10 y2 - 29y + 21= 0  y2 - 3y - 2y + 6 = 0
 10 y2 - 15y - 14y + 21 = 0  y (y - 3) - 2 (y - 3) = 0
 5y (2y - 3) - 7 (2y - 3) = 0  (y - 2) (y - 3) = 0
 (2y - 3) (5y - 7) = 0  y = 2 or 3
3
Clearly, x > y
7
 y= or 18. (4) I. x2 + 9x + 20 = 0
2 5
 x2 + 5x + 4x + 20 = 0
Clearly, x > y.
 x (x + 5) + 4 (x + 5) = 0
14. (5) I. 6x2 + 13x + 5 = 0
 (x + 4) (x + 5) = 0
 6x2 + 10x + 3x + 5 = 0
 x = -4 or -5
 2x (3x + 5) +1 (3x + 5) = 0 II. y2 + 7y + 12 = 0
 (3x + 5) (2x + l) = 0  y2 + 4y + 3y + 12 = 0
5 1  y (y + 4) + 3 (y + 4) = 0
x= or, -
3 2  (y + 3) (y + 4) = 0
II. 9y2 + 22y + 8 = 0  y = -3 or -4
Clearly, x  y
LEARN MATHS FROM S.K. RAJU (9811549822, 9811649822)
12
19. (5) I. x2 = 529  y - x = 3 - 5 = -2
23. (2) I. x2 - 1 = 0
x = 529 = ± 23
 (x + 1) (x - 1) = 0
II. y = 529 = ± 23  x = -1 or 1
Clearly, x = y II. y2 + 4y + 3 = 0
20. (2) I. x2 + 13x + 42 = 0  y2 + 3y + y + 3 = 0
 x2 + 7x + 6x + 42 = 0  y (y + 3) + 1 (y + 3) = 0
 x (x + 7) + 6 (x + 7) = 0  (y + 1) (y + 3) = 0
 (x + 6) (x + 7) = 0  y = -1 or - 3
 x = -6 or -7 Clearly, x  y
II. y2 + 16y + 63 = 0 24. (4) I. x2 - 7x + 12 = 0
 y2 + 9y + 7y + 63 = 0  x2 - 4x - 3x + 12 = 0
 y (y + 9) + 7 (y + 9) = 0  x (x - 4) - 3 (x - 4) = 0
 (y + 9) (y + 7) = 0  (x - 3) (x - 4) = 0
 y = -9 or -7  x = 3 or 4
Clearly, x  y
II. y2 - 12y + 32 = 0
21. (3) I. 2x + 3y =14
 y2 - 8y - 4y + 32 = 0
II. 4x + 2y = 16
By equation 1 × 2 - equation II, we have  y (y - 8) - 4 (y - 8) = 0
4x + 6y - 4x - 2y = 28 - 16  (y - 4) (y - 8) = 0
 4y = 12  y = 4 or 8
Clearly, x  y
12
y= =3 25. (3)1. x3 - 371 = 629
4  x3 = 371 + 629 = 1000
From equation I,  x = 3 1000 = 10
2x + 3 × 3 = 14
 2x = 5 II. y3 = 543 + 788 = 1331

5  y = 3 1331 = 11
 x= Clearly, x < y
2
26. (1) By equation 1 × 3 - equation
Clearly, x < y
II × 5, we have,
22. (2) 3y + 9x = 54
15x + 6y - 15x - 35y = 93 - 180
 3 (y + 3x) = 54
87
54  -29y = -87  y = =3
 3x + y = = 18 29
3
From equation I,
and, 5x + 2 × 3 = 31
28x 140  5x = 31 - 6 = 25
 13y = x=5
39
Clearly, x > y
x 140 13 x 5 27. (5)1. 2x 2 + 11x + 12 = 0
 y= ×
28
 y =
 2x2 + 8x + 3x + 12 = 0
39 3
 5y = 3x  2x (x + 4) + 3 (x + 4) = 0
 5y - 3x = 0 ...(ii)  (x + 4) (2x + 3) = 0
Adding equations (i) and (ii), we get 3
3x + y + 5y - 3x = 18  x = -4 or -
2
18 2
II. 5y + 27y + 10 = 0
 6y = 18  y = =3
6  5y2 + 25y + 2y + 10 = 0
From equation (ii),  5y (y + 5) + 2 (y + 5) = 0
5 × 3 - 3x = 0  (y + 5) (5y + 2) = 0
 3x = 15
2
15  y = -5 or -
x = = 5 5
3
LEARN MATHS FROM S.K. RAJU (9811549822, 9811649822)
13
Relationship can’t be established.  x (x - 5) - 3 (x - 5) = 0
28. (3) I. 2x2 + 7x + 4x + 14 = 0  (x - 3) (x - 5) = 0
 x (2x + 7) + 2 (2x + 7) = 0  x = 3 or 5
 (x + 2) (2x + 7) = 0 II. y2 - 3y + 2 = 0
7  y2 - 2y - y + 2 = 0
 x = -2 or - 2  y (y - 2)- 1 (y - 2) = 0
II. 4y2 + 2.2y.3 + 9 = 0  (y - 1) (y - 2) = 0
 (2y + 3)2 = 0  y = 1 or 2
Clearly, x > y
 2y + 3 = 0
35. (3) I. x2 = 32 + 112 = 144
3  x = ±12
y= -2
II. y = 169 = + 13
Clearly, x < y Clearly x < y
29. (1) I. (x - 2) (x + 2) = 0
x = 2 or -2 36. (1) I. x = 121 = + 11
II. y2 + 2.y.3 + 9 = 0 II. y2 = 121
 (y + 3)2 = 0  y = ± 11
y+3 =0 Clearly x  y
y = -3 37. (4) I. x2 = 16
Clearly, x > y x=±4
30. (2) I. x2 - 4x - 3x + 12 = 0 II. y2 - 9y + 20 = 0
 x (x - 4) - 3 (x - 4) = 0  y2 - 4y - 5y + 20 = 0
 (x - 3) (x - 4) = 0  y (y - 4) - 5 (y - 4) = 0
 x = 3 or 4  (y - 5) (y - 4) = 0
II. y2 + 4y - 3y - 12 = 0  y = 5 or 4
 y (y + 4) - 3 (y + 4) = 0 Clearly, x  y
 (y - 3) (y + 4) = 0 38. (3) I. 3x2 + 8x + 4 = 0
 y = 3 or -4  3x2 + 6x + 2x + 4 = 0
Clearly, x  y  3x (x + 2) + 2 (x + 2) = 0
31. (4) I. x2 = 729  (x + 2) (3x + 2) = 0
 x = ± 27
2
II. y = 729 = +27  x = -2 or -
3
Clearly, x  y II. 4y2 - 19y + 12 = 0
32. (5) I. x4 = 227 + 398 = 625 = 54  4y2 - 16y - 3y + 12 = 0
x=5  4y (y - 4) - 3 (y - 4) = 0
II. y2 = 346 - 321 = 25 = 52  (y - 4) (4y - 3) = 0
y=5
3
Clearly, x = y  y = 4 or
33. (2) I. x 2 - x - 12 = 0 4
 x2 - 4x + 3x - 12 = 0 Clearly, x < y
39. (4) I. x2 + x - 20 = 0
 x (x - 4) + 3 (x - 4) = 0
 x2 + 5x - 4x - 20 = 0
 (x - 4) (x + 3) = 0
 x (x + 5) - 4 (x + 5) = 0
 x = 4 or - 3
II. y2 + 5y + 6 = 0  (x + 5) (x - 4) = 0
 y2 + 3y + 2y + 6 = 0  x = - 5 or 4
II. y2 - y - 30 = 0
 y(y + 3) + 2(y + 3)= 0
 y2 - 6y + 5y - 30 = 0
 (y + 3) (y + 2) = 0
 y (y - 6) + 5 (y - 6) = 0
 y = -3 or - 2
Clearly, x  y  (y - 6) (y + 5) = 0
34. (1) I.x2 - 8x + 15 = 0  y = 6 or - 5
 x2- 5x - 3x + 15 = 0 Clearly, x  y

LEARN MATHS FROM S.K. RAJU (9811549822, 9811649822)


14
40. (4) I. x2 = 365 + 364 = 729 3 4
 x = ± 27 45. (5) I. + = x
x x
II. y - 324 = 81
3+4=x x= 7
 y - 18 = 9
7
 y =27 7  2
Clearly, x  y II. y3 - =0
y
41. (5) I. 4 + 7= x × x
1 7
3
 x = 11  y 2 - 72 = 0
5
7 7
112  y 2 = 72  y = 7
II. y2 - =0
y Clearly x = y
1 46. (5) I. 9x - 4x = 54.55 + 15.45
2 5
y 2 = 112  5x = 70
5
 x = 14
5
 y 2 = 112
iI I. y  155 = 7 + 6 =13
 y = 11  y + 155 = 169
Clearly, x = y
 y = 169 - 155 = 14
42. (3) I. 225x2 = 4 Clearly x = y
4 2 47. (3) I. x2 + 11x + 30 = 0
 x2 = x =±  x2 + 6x + 5x + 30 = 0
225 15
 x (x + 6) + 5 (x + 6) = 0
II. 225y + 2 = 0
 (x + 5) (x + 6) = 0
 225y = -2  x = -5 or -6
II. y2 + 7y + 12 = 0
Squaring on both sides, 225y = 4
 y2 + 4y + 3y + 12 = 0
4  y (y + 4) + 3 (y + 4) = 0
y=
225  (y + 3) (y + 4) = 0
Clearly x < y  y = - 3 or - 4
43. (1) I. 5x2 - 18x + 9 = 0 Clearly, x < y
 5x2 - 15x - 3x + 9 = 0
 5x (x - 3) - 3 (x - 3) = 0 6
48. (5) I. x - =0
 (x - 3) (5x - 3) = 0 x
3 x- 6=0  6
 x = 3 or
 6
5 3 3

II. 20y2 - 13y + 2 = 0 II. y3 = 6 2 =


 20y2 - 8y - 5y + 2 = 0 y = 6
 4y (5y - 2) -1 (5y - 2) = 0 Clearly x = y
 (4y - 1) (5y - 2) = 0 49. (1) By equation 1 × 3 - equation II
1 2 9x - 6y - 5x + 6y = 30 - 6
y = or  4x = 24  x = 6
4 5
Clearly, x > y From equation I,
44. (2) I. x3 = 878 + 453 = 1331 3 × 6 - 2y = 10
 2y = 18 - 10 = 8
 x = 3 1331 = 11
y=4
II. y2 = 82 + 39 = 121
Clearlr x > y
 y = ± 11 50. (4) I. x2 + x - 12 = 0
Clearly x  y
 X 2 + 4x - 3x - 12 = 0
 x (x + 4) - 3 (x + 4) = 0

LEARN MATHS FROM S.K. RAJU (9811549822, 9811649822)


15
 (x - 3) (x + 4) = 0  x2 - 5x - 4x + 20 = 0
 x = 3 or - 4 ]  x(x - 5) -4(x - 5) = 0
II. y2 - 5y + 6 = 0  (x - 5) (x - 4) = 0
 y2 - 3y - 2y + 6 = 0  x = 5 or 4
 y (y - 3) - 2 (y - 3) = 0 II. y2 - 7y - 6y + 42 = 0
 (y - 2) (y - 3) = 0  y (y - 7) - 6 (y - 7) = 0
 y = 2 or 3  (y - 6) (y - 7) = 0
Clearly x  y  y = 6 or 7
51. (3) I. x2 + 9x + 18 = 0 Clearly, x < y
 x 2 + 6x + 3x + 18 = 0 2 x 3 x 1
 x (x + 6) + 3 (x + 6) = 0 57. (5) I. =
10 x
 (x + 3) (x + 6) = 0
 5 x × x = 10
 x = -3 or -6
II. y2 - 13y + 40 = 0  5x = 10
 y2 - 8y - 5y + 40 = 0 x = 2
 y (y - 8) - 5 (y - 8) = 0 10  2
 (y - 5) (y - 8) = 0 II.
y
=4 y
 y = 5 or 8
 4y = 8
52. (2) I. x  6 = 11 - 6 = 5
8
 x + 6 = 25 y= =2
4
 x = 25 - 6 = 19
Clearly x = y
II. y2 = 473 - 112 = 361
58. (3) 2x + 3y = 78
 y = ±19
3x + 2y = 72
Cleary x  y On adding,
53. (5) I. x2 - 1200 + 244 = 1444 5x + 5y = 150  5(x + y) = 150
 x = ± 38
II. y = 159 - 122 = 37 150
Clearly, x > y or x < y x +y= = 30
5
54. (1) I. 14x + 7x = 59 + 25 59. (2) I. 20x2 - x - 12 = 0
 21x = 84  20x 2 - 16x + l5x - 12 = 0
84  4x(5x - 4) + 3(5x - 4) = 0
 x = 21 = 4  (5x - 4) (4x + 3) = 5
 5x - 4 = 0 or 4x + 3 = 0
II. y  222 = 36 + 81
4 3
 y  222 = 6 + 9 = 15 x= or
5 4
 y + 222 = 225 II. 20y2 + 27y + 9 = 0
 y = 225 - 222 = 3 20y2 + 15y + 12y + 9 = 0
Clearly, x > y  5y (4y + 3) + 3 (4y + 3) = 0
55. (4) I. 144x2 = 16 + 9 = 25  (5y + 3) (4y + 3) = 0
25 3 3
 x2 = y= or
144 5 4
5 Clearly, x  y
 x = ± 12 60. (4) I. x2 = 106 + 218 = 324
II. 12y = 49 - 4 = +5  x = 324 =  18
II. y2 - 37y + 342 = 0
5  y2 - 18y - 19y + 342 = 0
y =
12  y (y - 18) - 19 (y - 18) = 0
Clearly x  y
 (y - 19) (y - 18) = 0
56. (3) I. x2 - 9x + 20 = 0
LEARN MATHS FROM S.K. RAJU (9811549822, 9811649822)
16
 y = 19 or 18  y = 64 = ±8
Clearly x  y Relationship can’t be established.
7 5 65. (4) I. x2 - 7x + 12 = 0
61. (5) I.
x
+
x
= x  x2 - 4x - 3x + 12 = 0
 x (x - 4) - 3 (x - 4) = 0
7+5= x × x  (x - 3) - (x - 4) = 0
 x = 12  x = 3 or 4
5 II. y2 - 9y + 20 = 0
12 2  y2 - 5y - 4y + 20 = 0
II. =0
y  y (y - 5) - 4 (y - 5) = 0
1 5
 (y - 4) (y - 5) = 0
2
 y 2 - 12  2 = 0  y = 4 or 5
Clearly, x  y
 y5/2 = 125/2
66. (3) Dividing equation I by II,
 y = 12
62. (3) I. 19x + 4 = 0 y  x y  x  32
=
 19x = - 4 yx 2
4  y + x = 16 ....(i)
x= and y - x = 2 ....(ii)
19
II. 21y + 4 = 0 Adding both equations,
2y = 18  y = 9
4 From equation (i),
y=
21 x = 16 - 9 = 7
Clearly x < y Clearly, x < y
15 2 5
63. (1) I. =6 x - 67. (5) I. x - =0
x x x
 15 - 2 = 6x  x × x - 5= 0
 13 = 6x
x = 5
13 II. y3 = 53/2
x =
6
 5
3
 y3 =
y 7 y 1
II. + =
4 12 y y = 5
Clearly x = y
3 y 7 y 1 68. (1) By equation 1 × 3 + equation II × 5,
 =
y 9x + 15y = 84
12
40x - 15y = 210
10 y 1 49x = 294
 =
y
12 294
 10y = 12 x= =6
49
12 6 From equation I,
y= = 3 × 6 + 5y = 28
10 5
Clearly x > y  5y = 28 - 18 = 10
64. (5) 1. x  18 = 144 - 49 10
y= =2
5
 x  18 = ±(12 - 7) = ±5
Clearly, x > y
 x + 18 = 25
69. (3) I. 289x = - 25
 x = 25 - 18 = 7
II. y2 = 473 - 409 = 64 Squaring both sides,
289x = 25

LEARN MATHS FROM S.K. RAJU (9811549822, 9811649822)


17
25 7
x= 14  2
289  y3 - =0
y
II. 676y = -10
 
Squaring both sides, 7
676y = 100  y3 × y = 14  2
100 7 7
y=  y 2 = 14  2  y = 14
676
Clearly, x < y Clearly x = y
70. (2) I. 8x2 - 78x + 169 = 0 73. (5) I. x2 = 208 + 233 = 441
 8x2 - 26x - 52x + 169 = 0  x = 441 =  21
 2x (4x - 13) - 13 (4x - 13) = 0 II. y2 - 47 + 371 = 0
 (2x - 13) (4x - 13) = 0  y2 + 324 = 0
13
or 13  y = 324 = An imaginary number.
x =
2 4 Relationship can’t be established.
II. 20y2 - 117y + 169 = 0 74. (2) I. x2 - 11x + 24 = 0
 20y2 - 52y - 65y + 169 = 0  x2 - 8x - 3x + 24 = 0
 4y (5y - 13)- 13 (5y - 13) = 0  x (x - 8) - 3 (x - 8) = 0
 (4y - 13) (5y - 13) = 0  (x - 3) (x - 8) = 0
 x = 3 or 8
13
y = or - 13 II. 2y2 - 9y + 9 = 0
4 5  2y2 - 3y - 6y + 9 = 0
Clearly, x  y  y (2y - 3) - 3 (2y - 3) = 0
15  9  (2y - 3) (y - 3) = 0
71. (1) I. = 11 x
x 3
 y = 2 or 3
 11 x × x = 24
Clearly, x  y
24
 11x = 24  x = 75. (3) I. x3 × 13 = x2 × 247
11
x3 247
y 5 y 1  2 =  x = 19
II. + = y x 13
4 12
1 294
II. y 3 × 14 =
3 y 5 y 1 2

 = y y3
12
1 2 294
8 y 1  y3 × y3 =
 = y 14
12
1 2

 y 3 3 = 21  y = 21
8 y × y = 12
Clearly, x < y
12 3
y = = 76. (5) I.
48 12 10
8 2 4 - 4 = x 7
Clearly, x > y x7 x7
8 6 48  12 10 10 4
72. (5) I.
x
+
x
= x  4 =x 7  36 = x 7  7
7
x
86
 = x  x = 14  36 = x2  x = 36 = ±6
x
II. y3 = 999 - 783 = 216
7

3
14  2  y = 3 216 = 6
II. y - =0
y
LEARN MATHS FROM S.K. RAJU (9811549822, 9811649822)
18
Relationship can’t be established.  (2x - 1) (4x + 5) =0
77. (3) I. 500 x = - 402 1 5
 x = 2 or -
4
402 400
x= -  -0.9 II. 12y2 - 22y + 8 = 0
500 500
 6y2 - 11y + 4 = 0
II. 360 y = - 200  6y2 - 8y - 3y + 4 = 0
 2y (3y - 4) - 1 (3y - 4) = 0
200
y=- = -0.74  (3y - 4) (2y - 1) = 0
360
4 1
Clearly x < y y= or
3 2
1
78. (3) I. x = 172 + 144 × Clearly, x  y
18
82. (1) I. 17x2 + 48x - 9 = 0
= 289 + 8 = 297
 17x2 + 51x - 3x - 9 = 0
II. y = 262 - 18 × 21
= 676 - 378 = 298  17x (x + 3) - 3 (x + 3) = 0
Clearly, x < y  (x + 3)(17x - 3) = 0
79. (2) I. 16x2 + 20x + 6 = 0 3
 8x2 + 10x + 3 = 0  x = -3 or
17
 8x2 + 6x + 4x + 3 = 0 II. 13y2 - 32y + 12 = 0
 2x (4x + 3) + 1 (4x + 3) = 0  13y2 - 26y - 6y + 12 = 0
 (2x + 1) (4x + 3) = 0  13y (y - 2) - 6 (y - 2) = 0
1 3  (y - 2)(13y - 6) = 0
 x = - 2 or -
4 6
2
II. 10y + 38y + 24 = 0  y = 2 or
13
 5y2 + 19y + 12 = 0 Clearly, x < y
 5y2 + 15y + 4y + 12 = 0 83. (5) By equation I × 2 + equation II,
 5y (y + 3) + 4 (y + 3) = 0 8x + 14y = 418
 (y + 3) (5y + 4) = 0 4 12x -14y = -38
20x = 380
4
 y = -3 or - 380
5  x = 20 = 19
Clearly, x > y
80. (4) I. 18x2 + 18x + 4 = 0 From equation I,
 9x2 + 9x + 2 = 0 4 × 19 + 7y = 209
 9x2 + 6x + 3x + 2 = 0  7y = 209 - 76 = 133
 3x (3x + 2) + 1 (3x + 2) = 0 133
y = = 19
 (3x + 1) (3x + 2) = 0 7
1 2 Clearly x = y
x = - or - 25 9 4 12
3 3
84. (2) I. + - =
II. 12y2 + 29y + 14 = 0 x2 x2 x2 x
 12y2 + 21y + 8y + 14 = 0 25  9  4 12
 3y (4y + 7) + 2 (4y + 7) = 0  =
x2 x
 (3y + 2) (4y + 7) = 0
30
2 7  = 12
 y = - or - x
3 4  12x = 30
Clearly x  y
30 5
81. (3) I. 8x2 + 6x - 5 = 0  x = 12 = 2 = 2.5
 8x2 + 10x - 4x - 5 = 0
 2x (4x + 5) -1 (4x + 5) = 0 II. 9.84 - 2.64 = 0.95 + y2

LEARN MATHS FROM S.K. RAJU (9811549822, 9811649822)


19
 7.2 - 0.95 = y 2  (x - 4) (3x - 7) = 0
 6.25 = y2 7
 y = 6.25 = ± 2.5  x = 4 or
3
Clearly x  y II. 5y2 - 18y + 16 = 0
85. (1) I.  5y2 - 10y - 8y + 16 = 0
901 x = - 1295
 5y (y - 2) - 8 (y - 2) = 0
 901 x  - 1296  (5y - 8) (y - 2) =0
 30x  -36 8
36 6 y = or 2
5
x    - 1.2
30 5 Clearly x > y
1
II. 256 14 y  -  216  3 89. 1225 x +
(1) I. 4900 = 0
   35x + 70 = 0
 4y  - 6
70
6  35x = -70  x = = -2
y   - 1.5 35
4
1 1
Clearly x > y II.  3 4  4 y +  7 3  3 = 0
243  343
86. (1) I. = x3  3y + 7 = 0 [(am)n = amn]
3
7
586 y=
 = x3 3
3
Clearly, x > y
II. 7y3 = -30 + 17y3
 10y3 = 30 18 12 8 6
90. (3) I. 2 - 2 - 2 =
 y3 = 3 x x x x
Clearly x > y 18  12  8 6
 =
2 x2 x
 14 
x  144 2 6 2
87. (3) I.   = 3  =  = 6  6x = 2
 16  2 x2 x x
  x
2 1 1
1 x= =  x3 = = 0.037
x 1442 6 3 27
 = 3  y3 = 16.95 - 9.68 - 5.64 = 1.63
256
x2 Clearly x < y
1 3
 x 2 × x 2 = 256 × 144 32  1331 1363
91. (1) I. x3 = =  227
 x2 = 256 × 144 6 6

 x = 256  144 589


II. 4y3 - 5y3 = -
= ± 16 × 12 = ± 192 4
1 2 589
II. y 3 × y 3 × 3104 = 16 × y2  -y3 = -
4
 y × 3104 = 16 × y2
589
 3104 = 16y  y3 = = 147
4
3104 Clearly x > y
y= = 194
92. (2) I. 12x2 + 11x + 12 - 10x2 - 22x = 0
16
Clearly x < y  2x2 = -11x + 12 = 0
88. (1) I. 3x2 - 19x + 28 = 0  2x2 - 8x - 3x + 12 = 0
 3x2 - 12x - 7x + 28 = 0  2x (x - 4) - 3 (x - 4) = 0
 3x (x - 4) - 7 (x - 4) = 0  (x - 4) (2x - 3) = 0

LEARN MATHS FROM S.K. RAJU (9811549822, 9811649822)


20
3 y y 250
 x = 4 or II. + = y
2 4 16
II. 13y - 18y + 3 - 9y2 + 10y = 0
2

 4y2 - 8y + 3 = 0 4 y y 250
 = y
 4y2 - 6y - 2y + 3 = 0 16
 2y (2y - 3) - 1 (2y - 3) = 0
5 y × y = 250 × 16
 (2y - 1) (2y - 3) = 0
 5y = 250 × 16
1 3
 y = 2 or 2 250  16
y= = 800
5
Clearly x  y
clearly x < y
7
1
x 5 169 96. (1) I.  625  4 x + 1225 = 155
93. (5) I. = 3
9
x5 1

7 3   54  4 x + 35 = 155
 x × x 5 5 = 9 × 169
 5x = 155 - 35
7 3
 x 
= 9 × 169
5 5  5x = 120
 x2 = 9 × 169 120
x= = 24
 x = 3 × 13 = 39 5
1 1 273 II.196 y + 13 = 279
II. y 4 × y 4 × 7 = 1
2
 14y = 279 - 13 = 266
y
266
1 1 1273 y= = 19
  14
y =
4 4 2
7 Clearly x > y
 y = 39 97. (1) I. 5x2 - 18x + 9 = 0
Clearly x = y  5x2 - 15x - 3x + 9 = 0
94. (1) I.  5x (x - 3) - 3 (x - 3) = 0
25x 2 - 125 = 0
 (5x - 3) (x - 3) = 0
 25x 2 = 125 3
 25x2 = 125 × 125 x= or, 3
5
125  125 II. 3y2 + 5y - 2 = 0
 x2 = 25
= 625  3y2 + 6y - y - 2 = 0
 3y (y + 2) - 1 (y + 2) = 0
x = 625 = 25  (3y - 1) (y + 2) =0 1
II. 361 y + 95 = 0 1
 19y = - 95 y= or, -2
3
y= - 5 Clearly x > y
Clearly x > y
13 9
5 5 x 98. (3) I. + = x
x x
95. (3) I. - =
7 21 42
 13 + 9  x× x =x
15  5 x  x = 22
 =
21 42 9
26 2
10 II. y4 =
= x = 21 × 42 = 20 y

 x = 20 × 20 = 400 1 9
 y4 × y 2 =  26  2

LEARN MATHS FROM S.K. RAJU (9811549822, 9811649822)


21
9 9
6
  y  2 =  26  2  = x x=6
x
 y = 26
II. y10 = (36)5 = (62)5 = 610
Clearly x < y
99. (4) I. x2 - 19x + 84 = 0
y=6
Clearly x = y
 x2 - 12x - 7x + 84 = 0
105. (1) I. 5x + 2y = 96
 x (x -12) - 7 (x -12)= 0 II. 3 (7x + 5y) = 489
 (x - 7) (x - 12) = 0  7x + 5y = 489  3 = 163
 x = 7 or 12 By equation I × 5 - equation II × 2,
II. y2 - 25y + 156 = 0 25x + 10y = 480
 y2 - 12y - 13y + 156 = 0 14x + l0y = 326
 y (y - 12) - 13 (y - 12) = 0 11x = 154
 (y - 13) (y - 12) = 0 154
 y = 13 or 12 x=
11
Clearly x  y
100. (2) I. x3 = 1729 + 468 = 2197  x = 14
From equation I,
 x = 3 2197 = 13 14 × 5 + 2y = 96
II. y2 = 1733 - 1564 = 169  2y = 96 - 70 = 26
 y = 169 = ± 13  y = 13
Clearly x  y Clearly, x > y
1
101. (5) I. 9 + 19 = x × x 106. (1) I.  441 2 . x 2 - 111 = 225
 x = 28  21x2 = 225 + 111 = 336
1 11
II. y5 × y 2 =  28  2 336
 x2 = 21
= 16
11 11
 y 2 =  28  2 x=4
 y = 28 II.121 y2 + (6)3 = 260
Clearly x = y  11y2 + 216 = 260
102. (1) I. 784 x = 1486 - 1234 = 252  11y2 = 260 - 216 = 44

252 44
 x= =9  y2 = = 4
28 11
y=2
264 264 Clearly x > y
y= 1089
= =8
33 107. (3) I. 17x = 169 + 14 + 25 + 4x
103. (1) I. 12 - 23 = 5 x × x  13x = 208
 -11 = 5x 208
x= = 16
11 13
 x= II. 9y - 345 = 4y - 260
5
 9y - 4y = 345 - 260
y 5 y 1  5y = 85
II. = y
12  y = 17
Clearly, x < y
 -4 y × y = 12 108. (3) I. 3x2 - 13x + 14 = 0
 -4y = 12  3x 2 - 7x - 6x + 14 = 0
 y = -3  x (3x - 7) - 2 (3x - 7) = 0
Clearly x > y  (x - 2) (3x - 7) = 0
15 9 7
104. (5) I. - = x x = 2 or
x x 3

LEARN MATHS FROM S.K. RAJU (9811549822, 9811649822)


22
II. y2 - 7y + 12 = 0  (x - 12) (x - 12) = 0
 y2 - 4y - 3y + 12 = 0  x = 12
 y (y - 4) - 3 (y - 4) = 0 II. y2 - 26y + 169 = 0
 (y - 3) (y - 4) = 0  (y - 13)2 = 0
 y = 3 or 4  y - 13 = 0
Clearly x < y  y = 13
109. (1) I. x2 + 5x + 6 = 0 Clearly, x < y
 x 2 + 2x + 3x + 6 = 0 115. (4) I. 2x2 + 3x - 20 = 0
 x (x + 2) + 3 (x + 2) = 0  2x2 + 8x - 5x - 20 = 0
 (x + 3) (x + 2) = 0  2x (x + 4) - 5 (x + 4) = 0
 x = - 3 or -2  (x + 4) (2x - 5) = 0
II. y2 + 7y + 12 = 0 5
 y2 + 4y + 3y + 12 = 0  x = -4 or 2
 y (y + 4) + 3 (y + 4) = 0
II. 2y2 + 19y + 44 = 0
 (y + 3) (y + 4) = 0  2y2 + 11y + 8y + 44 = 0
 y = - 3 or -4
 y (2y + 11) + 4 (2y + 11) = 0
Clearly x  y
 (y + 4) (2y + 11) = 0
110. (4) I. x2 - 9x + 20 = 0
 x 2 - 5x - 4x + 20 = 0 11
 y = -4 or - 2
 x (x - 5) - 4 (x - 5) = 0
 (x - 4) (x - 5) = 0 Clearly, x  y
 x = 4 or 5 116. (5) I. 6x2 + 77x + 121 = 0
II. y2 - 13y + 42 = 0  6x2 + 66x + 11x + 121 = 0
 y2 - 7y - 6y + 42 = 0  6x (x + 11) + 11 (x + 11) = 0
 y (y - 7) - 6 (y - 7) = 0  (x + 11) (6x + 11) = 0
 (y - 6) (y - 7) = 0 11
 y = 6 or 7  x = - 11 or -
6
Clearly x < y 2
II. y + 11y - 2y - 22 = 0
111. (4) 2x + 3y = 14 ....I  y (y + 11) - 2 (y + 11) = 0
4x + 2y = 16 ....II
 (y - 2) (y + 11) = 0 .
By equation (I) × 2 - equation II,
4x + 6y - 4x - 2y = 28 - 16  y = 2 or -11
Clearly x  y
 4y = 12  y = 3
From equation I, 117. (2) I. x2 - 6x - 7 = 0
2x + 3 × 3 = 14  x 2 - 7x + x - 7 = 0
 x (x - 7) + 1 (x - 7) = 0
5
 2x =14 - 9 = 5  x = 2  (x + 1) (x - 7) = 0
 x = -1 or 7
Clearly x < y
II. 2y2 + 13y + 15 = 0
112. (5) I. x = 625 =  25  2y2 + 10y + 3y + 15 = 0
II. y = 676 = ±26  2y (y + 5) + 3 (y + 5) = 0
113. (4) I. x2 + 4x + 4 = 0  (y + 5) (2y + 3) = 0
(x + 2)2 = 0  x = -2 3
II. y2 - 8y + 16 = 0  y = -5 or -
2
 (y - 4)2 = 0 Clearly x > y
y=4 118. (4) I. 10x2 - 7x + 1 = 0
Clearly x < y  10x2 - 5x - 2x + 1 = 0
114. (1) I. x2 - 24x + 144 = 0  5x (2x - 1) - 1 (2x - 1) = 0
 x2 - 12x - 12x + 144 = 0  (5x - 1) (2x - 1) = 0
 x (x - 12) - 12 (x - 12) = 0
1 1
x= or
5 2
LEARN MATHS FROM S.K. RAJU (9811549822, 9811649822)
23
II. 35y2 - 12y + 1 = 0 II. 30y2 + 11y + 1 = 0
 35y2 - 7y - 5y + 1 = 0  30y2 + 6y + 5y + 1 = 0
 7y (5y - 1) - 1 (5y - 1) = 0  6y (5y + 1) + 1 (5y + 1) = 0
 (7y - 1) (5y - 1) = 0  (5y + 1) (6y + 1) = 0
1 1 1
 x = 1 or  y= - or -
7 5 5 6
Clearly x  y Clearly, x  y
119. (2) I. 4x2 - 32x + 63 = 0 123. (4) I. x2 + 17x + 72 = 0
 4x2 - 14x - 18x + 63 = 0  x2 + 8x + 9x + 72 = 0
 2x (2x - 7) - 9 (2x - 7) = 0  x (x + 8) + 9 (x + 8) = 0
 (2x - 7) (2x - 9) = 0  (x + 9) (x + 8) = 0
7 9  x = -9 or -8
 x = 2 or 2 II. y2 + 19y + 90 = 0
 y2 + 10y + 9y + 90 = 0
II. 2y2 - 11y + 15 = 0
 2y2 - 6y - 5y + 15 = 0  y (y + 10) + 9 (y + 10) = 0
 (y + 9) (y + 10) = 0
 2y (y - 3) - 5 (y - 3) = 0
 y = -9 or -10
 (y - 3) (2y - 5) = 0
Clearly, x > y
5
 y = 3 or 2
SBI PO EXAMS
Clearly, x > y 1. 2
(3) I. 4q + 8q = 4q + 8
1 4q2 + 4q - 8 = 0
120. (2) I. x3 =  216  3
3
= 216 q2 + q - 2 = 0
q2 + 2q - q - 2 = 0
 x = 3 216 = 6
q (q + 2) - 1 (q + 2) = 0
II. 6y2 = 150 (q + 2) (q - 1) = 0
150 q = -2, 1
 y2 = = 25 II. p2 + 9p = 2p - 12
6
p2 + 7p + 12 = 0
 y = 25 = ±5 p2 + 3p + 4p + 12 = 0
Clearly, x > y p (p + 3) + 4 (p + 3) = 0
121. (1) I. 12x2 + 17x + 6 = 0 (p + 3) (p + 4 ) = 0
 12x2 + 9x + 8x + 6 = 0 p = - 3, - 4
 3x (4x + 3) + 2 (4x + 3) = 0 q > p
 (4x + 3) (3x + 2) = 0 2. (3) I. 2p2 + 40 = 18p
p2 + 20 = 9p
3 2 p2 - 9p + 20 = 0
x =- or -
4 3 p2 - 4p - 5p + 20 = 0
2
II. 6y + 5y + 1 = 0 p (p - 4) - 5 (p - 4) = 0
 6y2 + 2y + 3y + 1 = 0 (p - 4) (p - 5) = 0
 2y (3y + 1) + 1 (3y + 1) = 0 p = 4, 5.
 (3y + 1) (2y + 1) = 0 II. q2 = 13q - 42
q2 - 13q + 42 = 0
1 1 q2 - 6q - 7q + 42 = 0
y = - or -
3 2 q (q - 6) - 7 (q - 6 ) = 0
Clearly, x < y (q - 6) (q - 7) = 0
122. (3) I. 20x2 + 9x + 1 = 0 q = 6, 7
 20x2 + 5x + 4x + 1 = 0 q >p
 5x (4x + 1) + 1 (4x + 1) = 0 1 7
 (4x + 1) (5x + 1) = 0 3. (4) I. 6q2 + = q
2 2
1 1
x=- or - 12q 2  1 7
4 5 = q
2 2
LEARN MATHS FROM S.K. RAJU (9811549822, 9811649822)
24
12q2 + 1 = 7q  9y2 - 33y - 33y + 121 =0
12q2 - 7q + 1 = 0  3y (3y - 11) - 11 (3y - 11) = 0
12q2 - 3q - 4q + 1 = 0  (3y - 11) (3y - 11) = 0
3q (4q - l) - 1 (4q - 1) = 0
(4q - 1) (3q - 1) = 0 11
y=
3
1 1
q= , Obviously, x < y
4 3
7. (2) Equation I
II. 12p2 + 2 = 10p x2 + 9x + 14 = 0
6p2 + 1 = 5p
 x2 + 2x + 7x + 14 = 0
6p2 - 5p + 1 = 0
6p2 - 3p - 2p + 1 = 0  x (x + 2) + 7 (x + 2) = 0
3p (2p - 1) - (2p - 1) = 0  (x + 2) (x + 7) = 0
(2p - 1) (3p - 1) = 0  x = - 2 or, x = - 7
Equation II
1 1
p= , y2 + y - 2 = 0
2 3
 y2 + 2y - y - 2 = 0
p  q  y (y + 2) - 1 (y + 2)= 0
4. (5) Equation I
 (y - 1)(y + 2) = 0
2x2 - 7x + 6 = 0
 y = 1 or, y = - 2
 2x2 - 4x - 3x + 6 = 0
Clearly, x  y
 (x - 2) (2x - 3) = 0
8. (1) Equation I
3 9x2 - 18x + 5 = 0
 x = 2 or x = 2
 9x2- 15x - 3x + 5 = 0
Equation II. 4y2 = 9  3x (3x - 5) - (3x - 5) = 0
3  (3x - 5) (3x - 1) = 0
9
 y2 = = 
2 5 1
4
x= or, x =
Obviously, y  x 3 3
5. (5) Equation I Equation II
4x2 - 4x - 3 = 0 2y2 - 9y + 10 = 0
 4x2 - 6x + 2x - 3 = 0  2y2 - 5y - 4y + 10 = 0
 2x (2x - 3) + 1 (2x - 3) = 0  y (2y - 5) - 2 (2y - 5) = 0
 (2x - 3) (2x + 1) = 0  (2y - 5) (y - 2) = 0
3 1 5
x= or, x = -  y = 2 or y = 2
2 2
Equation II. 4y2 + 12y + 5 = 0 Clearly, x < y
 4y2 + 10y + 2y + 5 = 0 9. (2) I. 6p2 + 3p + 2p + 1= 0
or, 3p (2p + 1) + 1 (2p + 1) = 0
 2y (2y + 5) + 1 (2y + 5) = 0
or, (2p + 1) (3p + 1) = 0
 (2y + 5) (2y + 1) = 0 Now, 2p + 1 = 0
5 1 1
 y = - 2 or, y = - 2 or, p = -
2
Obviously, x  y or, 3p + 1 = 0
6. (1) Equation I
1
4x2 = 49 or, p = -
3
49
 x2 = 1 1
4  p = - 2 or -
3
7 2
x=± 2 II. 20q + 9q + 1 = 0
or, 20q + 5q + 4q + 1 = 0
Equation II or, 5q (4q + 1) + 1 (4q + 1) = 0
9y2- 66y + 121 or, (4q + 1) (5q + 1) = 0
LEARN MATHS FROM S.K. RAJU (9811549822, 9811649822)
25
1 1 15. (1) I. 2a2 + 3a + 1 = 0
or, q = - or -  2a2 + 2a + a + 1 = 0
4 5
Obviously, p < q  2a(a + 1) + 1(a + 1) = 0
10. (1) I. 3p2 + 3p - p - 1 = 0  (a + 1) (2a + 1) = 0
or, 3p (p + 1) - 1 (p + 1) = 0 1
or, (p + 1) (3p - 1) = 0  a = -1 or -
2
1 2
II. 12b + 7b + 1 = 0
or, p = - 1 or
3  12b2 + 3b + 4b + 1 = 0
II. 2q2 + 4q + 3q + 6 = 0  3b(4b + 1) + 1(4b + 1) = 0
or, 2q (q + 2) + 3 (q + 2) = 0  (3b + 1) (4b + 1) = 0
or, (q + 2) (2q + 3) = 0
1 1
3 b=- or -
or, q = - 2 or - 3 4
2 Clearly, a < b
Obviously, p > q. 16. (3) I. a2 = 4  a = ±2
11. (4) I. 3p2 + 15p + 18 = 0 II. b2 = 9  b = + 3
or, 3p2 + 9p + 6p + 18 = 0
17. (2) I. 6a2 - 25a + 25 = 0
or, 3p (p + 3) + 6 (p + 3) = 0
 6a2 - 15a - 10a + 25 = 0
or, (p + 3) (3p + 6) = 0
 3a (2a - 5) - 5 (2a - 5) = 0
6
or, p = - 3 or - = -2  (3a - 5) (2a - 5) = 0
3
5 5
II. q2 + 4q + 3q + 12 = 0 a = or
or, q (q + 4) + 3 (q + 4) = 0 3 2
2
or, (q + 3) (q + 4) = 0 II. 15b - 16b + 4 = 0
or, q = - 3 or - 4  15b2 - 10b - 6b + 4 = 0
Obviously p  q  5b (3b - 2) - 2 (3b - 2) = 0
 (5b - 2) (3b - 2) = 0
4 2
12. (3) I. p = = 2 2
9 3 b = or
2
II. (3q - 2) = 0 5 3
or, (3q - 2) = 0 Clearly, a > b
18. (4) I. 4a2 - 20a + 21 = 0
2
q=  4a2 - 14a - 6a + 21 = 0
3  2a (2a - 7) - 3 (2a - 7) = 0
Obviously, p = q
 (2a - 7) (2a - 3) = 0
13. (5) p2 + 7p + 6p + 42 = 0
or, p (p + 7) + 6 (p + 7) = 0 7 3
or, (p + 6) (p + 7) = 0  a=
2
or
2
or, p = - 6 or - 7 II. 2b2 - 5b + 3 = 0
II. q2 = 36  2b2 - 3b - 2b + 3 = 0  
or, q = 6 or - 6
 b(2b - 3) - 1(2b - 3) = 0  
Obviously, p  q
 (b - 1) (2b - 3) = 0
14. (5) I. a2 + 5a + 6 = 0
 a2 + 2a + 3a + 6 = 0 3
 b = 1 or 2
 a (a + 2) + 3 (a + 2) = 0
 (a + 2) (a + 3) = 0 Clearly a  b
 a = -2 or -3 19. (2) I. p2 + 24 = 10p
II. b2 + 3b + 2 = 0 p2- 10p + 24 = 0
 b2 + 2b + b + 2 = 0 p2 - 6p - 4p + 24 = 0
p(p - 6) -4(p - 6) = 0
 b(b + 2) + 1(b + 2) = 0
(p - 6)(p - 4) = 0
 (b + 1) (b + 2) = 0 p = 4, 6
 b = -1 or -2 II. 2q2 + 18 = 12q
Clearly, a  b
LEARN MATHS FROM S.K. RAJU (9811549822, 9811649822)
26
q 2 + 9 = 6q  q (q - 2) - 1 (q - 2) = 0
q 2 - 6q + 9 = 0  (q - 2) (q - 1) = 0
(q - 3)2 = 0  q = 1 or 2
q=3 Obviously p > q
Thus, p > q 25. (1) (I) 12p2 - 7p = - 1
20. (3) I. pq + 30 = 6p + 5q
 12p2 - 7p + 1 = 0
6p + 5q - pq = 30
 12p2 - 4p - 3p + l = 0
p q pq  4p(3p - 1)- 1 (3p - 1) = 0
+ - =1
5 6 30  (3p - 1) (4p - 1) = 0
p = 5 and q = 6
q > p 1 1
p= or
21. 2
(5) I. q + q = 2 4 3
q2 + q - 2 = 0 (II) 6q2 - 7q + 2 = 0
q 2 - q + 2q - 2 = 0  6q2 - 4q - 3q + 2 = 0
q (q - 1) + 2 (q - 1) = 0  2q (3q - 2) - 1 (3q - 2) = 0
(q - 1) (q + 2) = 0  (3q - 2) (2q - 1) = 0
q = -2, 1
II. p2 + 7p + 10 = 0 2 1
 q = or 2
2
p + 5p + 2p + 10 = 0 3
p(p + 5) + 2(p + 5) = 0 Obviously, p < q
(p + 5)(p + 2) = 0 26. (3) (I) p2 + 12p + 35 = 0
p = -5, -2  p2 + 7p + 5p + 35 = 0
Thus, q  p  p (p + 7) + 5 (p + 7) = 0
22. (1) I. p2 +16 = 8p  (p + 7) (p + 5) = 0
p2 - 8p + 16 = 0  p = -5 or -7
(p - 4) 2 = 0
(II) 2q2 + 22q + 56 = 0
p=4
 2q2 + 14q + 8q + 56 = 0
II. 4q2 + 64 = 32q
q2 + 16 = 8q  2q (q + 7) + 8 (q + 7) = 0
2
q - 8q + 16 = 0  (q + 7) (2q + 8) = 0
2
(q - 4) = 0  q = -7 or -4
q=4 Obviously, p  q
Thus, p = q 27. (4) (I) p2 - 8p + 15 = 0
23. 2
(4) I. 2p + 12p +16 = 0  p2 - 3p - 5p + 15 = 0
 p (p - 3) - 5 (p - 3) = 0
2
p + 6p + 8 = 0
p2 + 2p + 4p + 8 = 0  (p - 3) (p - 5) = 0
p(p + 2) + 4(p + 2) = 0
 p = 3 or 5
(p + 2)(p + 4) = 0
(II) q2 - 5q = - 6
p = -2, -4
II. 2q 2 + 14q + 24 = 0  q2 - 5q + 6 = 0
q 2 + 7q + 12 = 0  q2 - 3q - 2q + 6 = 0
q 2 + 3q + 4q + 12 = 0  q (q - 3) - 2 (q - 3) - 0
q(q + 3) + 4(q + 3) = 0  (q - 3) (q - 2) = 0
(q + 3)(q + 4) = 0  q = 3 or 2
q = -3, -4 Obviously, p  q
Thus, p  q 28. (3) (I) 2p2 + 20p + 50 = 0
2
24. (2) (I) p - 7p = - 12  p2 + 10p + 25 = 0
 p - 7p + 12 = 0
2
 (p + 5)2 = 0
 p2 - 4p - 3p + 12 = 0 p+5 =0
 p (p - 4) - 3 (p - 4) = 0  p = -5
 (p - 4) (p - 3) = 0 (II) q2 = 25
 p = 3 or 4 q = 5
(II) q2 - 3q + 2 = 0 Obviously, p  q
 q2 - 2q - q + 2 = 0
LEARN MATHS FROM S.K. RAJU (9811549822, 9811649822)
27
RBI GRADE-B OFFICER EXAMS II.  y2 + 3y - 2y - 6 = 0
1. (1) 2
2x + x - 3 < 0  y (y + 3) - 2 (y + 3) = 0
 2x 2 - 2x + 2x - 3 < 0  (y - 2)(y + 3) = 0
 2x (x - 1) + 3 (x - 1) < 0  y = 2 or - 3.
 (2x + 3) (x - 1) < 0 Obviously, x > y
7. (5) I.  p2 + 3p + 2p + 6 = 0
3
 < x <1  p (p + 3) + 2 (p + 3) = 0
2
 (p + 3) (p + 2) = 0
2. (1)  x 2 + 2x +x + 2 = 0
 p = -2 or -3
 x (x + 2) + 1 (x + 2) = 0 II.  q2 + q + 2q + 2 = 0
 (x + 1) (x + 2) = 0
 q(q + 1) + 2(q + 1) = 0
 x = -1 or -2
 (q + 1) (q + 2) = 0
II.  2y2 - 5y = 0
 q = -1 or -2
 y(2y - 5) = 0
Obviously p  q
5
 y = 0 or 8. (4) I.  p = ± 2
2
II.  q2 + 2q + 2q + 4 = 0
Obviously, x < y
 q (q + 2) + 2 (q + 2) = 0
3. (3) I. 2x2 + 5x + 2 = 0
 2x2 + 4x + x + 2 = 0  (q + 2) (q + 2) = 0
 2x (x + 2) (x + 2) = 0 q=-2
Obviously, p  q
1 (2) I.  p2 + p - 56 = 0
 x=  or, -2 9.
2  p2 + 8p - 7p - 56 = 0
II. 4y2 = 1  p (p + 8) - 7 (p + 8) = 0
1 1  (p + 8) (p - 7) = 0
 y= =   p = 7 or - 8
4 2
Clearly, x  y II.  q2 + 17q + 72 = 0
4. (2) I.  y2 + 3y - y - 3 = 0  q - 8q - 9q + 72 = 0
 y (y + 3) -1 (y + 3) = 0  q (q - 8) - 9 (q - 8) = 0
 (y + 3) (y - 1) = 0.  (q - 8) (q - 9) = 0
 y = -3 or 1  q = 8 or 9
II.  2x2 - 4x - 3x + 6 = 0 Obviously, p < q
10. (1) We have,
 2x (x - 2) - 3 (x - 2) = 0
3p + 2q = 58 . ...(i)
 (2x - 3) (x - 2) 4p + 4q = 92
3  2p + 2q = 46 ...(ii)
 x = 2 or 2 By (i) - (ii) we get p = 12
Obviously, x > y From (i), 3 × 12 + 2q = 58
5. (1) I.  x2 + 4x - 2x - 8 = 0  2q = 58 - 36 = 22
 x (x + 4) - 2 (x + 4) = 0  q = 11
 (x - 2) (x + 4) = 0 Hence, p > q
 x = 2 or - 4 11. (2) I.  3p2 + 15p + 2p + 10 = 0
II.  y2 - 9 = 0  3p (p + 5) + 2 (p + 5) = 0
 (y + 3)(y - 3) = 0  (p + 5) (3p + 2) = 0
 y = 3 or - 3 2
Obviously, x < y.  p = - 5 or -
3
6. (4) I.  x2 - 3x - 2x + 6 = 0 II.  10q2 + 5q + 4q + 2 = 0
 x (x - 3) - 2 (x - 3) = 0  5q (2q + 1) + 2 (2q + 1) = 0
 (x - 2) (x - 3) = 0  (2q + 1) (5q + 2) = 0
 x = 2 or 3 12. (3) I. 8x2 + 6x - 5 = 0

LEARN MATHS FROM S.K. RAJU (9811549822, 9811649822)


28
 8x - 4x + 10x - 5 = 0
2
 (3y + 2) (4y + 7) = 0
 4x (2x - 1) + 5 (2x -1) = 0 2 7
 (4x + 5) (2x -1) = 0  y=  or 
3 4
5 1 Clearly, x  y
 x=  or
4 2 16. (2) I. 16x2 + 20x + 6 = 0
2
II. 12y - 22y + 8 = 0  8x2 + 10x + 3 = 0
 6y2 - 11y + 4 = 0  8x2 + 6x + 4x + 3 = 0
 6y2 - 8y - 3y + 4 = 0  8x2 + 6x + 4x + 3 = 0
 2y (3y - 4) - 1 (3y - 4) = 0  2x (4x + 3) + 1 (4x + 3) = 0
 (2y - 1) (3y - 4) = 0 1 3
1 4
 x=  or 
2 4
 y= or
2 3 II. 10y2 + 38y + 24 = 0
Clearly, x  y  5y2 + 19y + 12 = 0
13. (5) Equation (i) × 2 + Equation (ii),  5y2 + 15y + 4y + 12 = 0
8x + 14y + 12x - 14y = 418 - 38  5y (y + 3) + 4(y + 3) = 0
 20x = 380
4
380  y=  or -3
5
 x= = 19
20 Clearly, x > y
From (i), 17. (4) I. x2 + 5x + 6 = 0
4 × 19 - 7y = 209  x2 + 2x + 3x + 6 = 0
 7y = 76 - 209  x (x + 2) + 3 (x + 2) = 0
 7y = -133  (x + 3) (x + 2) = 0
 y = -19  x = -3 or -2
Clearly, x > y II. y2 + 3y + 2 = 0
14. (1) I. 17x2 + 48x - 9 = 0  y2 + 2y + y + 2 = 0
 17x2 + 51x - 3x - 9 = 0  y (y + 2) + 1 (y + 2) = 0
 17x (x + 3) - 3 (x + 3) = 0  (y + 1) (y + 2) = 0
 (17x - 3) (x + 3) = 0  y = -1 or - 2
3 Clearly, x  y
 x= or -3 18. (2) I. x2 - 10x + 24 = 0
17
 x2 - 6x - 4x + 24 = 0
II. 13y2 - 32y + 12 = 0
 x (x - 6) - 4 (x - 6) = 0
 13y2 - 26y - 6y + 12 = 0
 13y (y - 2) - 6(y - 2) = 0  (x - 4) (x - 6) = 0
 x = 4 or 6
 (13y - 6) (y - 2) = 0
II. y2 - 9y + 20 = 0
6  y2 - 5y - 4y + 20 = 0
 y= or 2
13  y (y - 5) - 4 (y - 5) = 0
Clearly, x < y  (y - 4) (y - 5) = 0
15. (4) I. 18x2 + 18x + 4 = 0  y = 4 or 5
 9x2 + 9x + 2 = 0 x  y
 9x2 + 6x + 3x + 2 = 0 19. (5) I. x2 = 961 = ± 31
 3x (3x + 2) + 1 (3x + 2) = 0
II. y = 961 = ± 31
 (3x + 1) (3x + 2) = 0
20. (2) I. x2 - x - 72 = 0
1 2  x2 - 9x + 8x - 72 = 0
 x =  or 
3 3  x(x - 9) + 8 (x - 9) = 0
II. 12y2 + 29y + 14 = 0  (x + 8) (x - 9) = 0
 12y2 + 21y + 8y + 14 = 0  x = -8 or 9
 3y (4y + 7) + 2 (4y + 7) = 0 II. y2 = 64

LEARN MATHS FROM S.K. RAJU (9811549822, 9811649822)


29
 y = 64 = ±8 By equation II × 3 - equation I
6y + 9z = 99
Clearly, x  y
6y + 5z = 71
21. (5) I. x2 = 463 + 321 = 784 - - - .
 x = 784 = ± 28 4z = 28
II. y2 = 308 + 421 = 729 z=7
From equation II,
 y = 729 = ± 27
2y + 3 × 7 = 33
22. (1)
 2y = 33 - 21 = 12
7x + 6y + 4z = 122 ... (i)
4x + 5y + 3z = 88 ... (ii) y=6
9x + 2y + z = 78 ... (iii) x<y<z
By equation (iii) × 3 - equation (ii), 25. (4) By equation I × 5 - II × 8
27x + 6y + 3z = 234 40x + 35y = 675
4x + 5y + 3z = 88 40x + 48y = 792
- - - - - - - .
23x + y = 146 ...(iv) -13y = - 117
By equation (iii) × 4 - equation (i),  y=9
From equation I,
36x + 8y + 4z = 312 8x + 7 × 9 = 135
7x + 6y + 4z =122  8x = 135 - 63 = 72
- - - - x= 9
29x + 2y = 190 ...(v) From equation III,
By equation (iv) × 2 - equation (v) 9 × 9 + 8z = 121
46x + 2y = 292  8z = 121 - 81 = 40
29x + 2y = 190 z =5
- - --___ Clearly,x = y > z
17x = 102 26. (5) I. (x + y) 3 = 1331
x = 6  x + y = 11
From equation (iv),  y = 11 - x
23 × 6 + y = 146 From equation III,
 y = 146 - 138 = 8 x (11 - x) = 28
From equation (iii),  11x - x 2 = 28
9 × 6 + 2 × 8 + z = 78  x 2 - 11x + 28 = 0
 54 + 16 + z = 78  x 2 - 7x - 4x + 28 = 0
 z = 78 - 70 = 8  x (x - 7) - 4 (x - 7) = 0
Clearly, x < y = z  (x - 7) (x - 4) = 0
23. (3) By equation II × 2 - equation (I)  x = 7, 4
8x + 6y = 118 From equation I
7x + 6y = 110 y = 4, 7
- - - . From equation II
x=8 7 - 4 + z = 0  z = -3
From equation (I),
or, 4 - 7 + z = 0  z = 3
7 × 8 + 6y = 110
 6y = 110 - 56 = 54 INSURANCE EXAMS
 y= 9
1. (5) Equation I
From equation (III),
2x2 - 7x + 6 = 0
8 + z =15
 2x2 - 4x - 3x + 6 = 0
z =7
 2x (x - 2) - 3 (x - 2) = 0
Clearly, x < y > z
 (x - 2) (2x - 3) = 0
1 1
24. (2) I. x = 36  2  1296  4 3
 x = 2 or x = 2
= 6  6 = ±6
LEARN MATHS FROM S.K. RAJU (9811549822, 9811649822)
30
Equation II. 4y2 = 9 4. (2) Equation I
9 x2 + 9x + 14 = 0
 y2 =  x2 + 2x + 7x + 14 = 0
4
 x (x + 2) + 7 (x + 2) = 0
3  (x + 2) (x + 7) = 0
 y=  2
 x = - 2 or, x = -7
Obviously, y  x Equation II
2. (5) Equation I y2 + y - 2 = 0
4x2 - 4x - 3 = 0  y2 + 2y - y - 2 = 0
 4x2 - 6x + 2x - 3 = 0  y(y + 2) -1(y + 2) = 0
 2x (2x - 3) + 1 (2x - 3) = 0  (y - 1) (y + 2) = 0
 (2x - 3) (2x + 1) = 0  y = 1 or, y = - 2
3 1 Clearly, y  x
 x = 2 or, x = - 2 5. (1) Equation I
Equation II. 4y2 + 12y + 5 = 0 9x2 - 18x + 5 = 0
 4y2 + 10y + 2y + 5 = 0  9x2 - 15x - 3x + 5 = 0
 2y (2y + 5) + 1 (2y + 5) = 0  3x (3x - 5) - 1 (3x - 5) = 0
 (2y + 5) (2y + 1) = 0  (3x - 5) (3x - 1) = 0

5 1 5 1
y= - or, y = - x = or, x =
2 2 3 3
Equation II
Obviously, x  y
2y2 - 9y + 10 = 0
3. (1) Equation I 4x2 = 49
 2y2 - 5y - 4y + 10 = 0
49  y (2y - 5) - 2 (2y - 5) = 0
 x2 =
4  (2y - 5) (y - 2) = 0
7 5
x=± 2  y = 2 or y = 2
Equation II Clearly, y > x
9y2 - 66y + 121
 9y2 - 33y - 33y + 121 =0
 3y (3y - 11) - 11 (3y - 11) = 0
 (3y - 11) (3y - 11) = 0
11
y=
3
Obviously, x < y

LEARN MATHS FROM S.K. RAJU (9811549822, 9811649822)


1
LCM & HCF
NATIONALISED BANKS INSURANCE EXAMS
& IBPS PO/MT/SO
1. Which of the following fractions are in 1 5 2 4
1. The L.C.M. of , , , is
descending order ? 3 6 9 27

5 2 3 2 1 1 10
(1) , , , , (1) (2)
12 7 11 9 5 54 27

2 2 3 1 5 27
(2) , , , , (3) 20 (4)
7 9 11 5 12 3 3
(LIC Assistant Administrative
1 2 3 2 1
(3) , , , , Officer (AAO) Exam. 07.06.2009)
5 9 11 7 5 2. The product of two 2-digit numbers is 2028
5 3 2 2 1 and their HCF is 13. The sum of the
(4) , , , , numbers is :
12 11 7 9 5
(1) 65 (2) 78
(5) None of these (3) 91 (4) 169
(Indian Bank Rural Marketing (New India Assurance
Officer Exam. 03.01.2010) AO Exam. 25.10.2009)
2. Amit, Sucheta and Neeti start running 3. When a number is divided by 2, 3, 4, 5 or
around a circular track and complete one 6, remainder in each case is 1. But the
ro un d in 18, 24 and 32 secon ds number is exactly divisible by 7. If the
respectively. In how many seconds will the number lies between 250 and 350, the sum
three meet again at the starting point if they of digits of the number will be
all have started running at the same time ? (1) 4 (2) 5
(1) 196 (3) 7 (4) 10
(2) 288 (New India Insurance
(3) 324 AAO Exam. 22.05.2011)
(4) Can not be determined 4. HCF of two numbers each of 4 digits is 103
(5) None of these and their LCM is 19261. Sum of the
(Bank Of India Banking Officer numbers is
Exam. 24.01.2010) (1) 2884 (2) 2488
3. Philip, Tom and Brad start jogging around (3) 4288 (4) 4882
a circular field and complete a single round (New India Insurance AAO
in 18, 22 and 30 seconds respectively. In Exam. 22.05.2011)
how much time will they meet again at the 5. HCF of three numbers 3240, 3600
starting point ? and p is 36. If their LCM is 24 × 35 ×5 2 × 72 ,
(1) 8 mins. 15 secs. then the number p is
(2) 21 mins. (1) 22 × 53 × 72 (2) 35 × 52 × 72
(3) 16 mins. 30 secs. (3) 22 × 35 × 72 (4) 23 × 35 × 73
(4) 12 mins. (Oriental Insurance Company
(5) None of these AAO Exam. 08.04.2012)
(Indian Bank PO Exam. 02.01.2011) 6. LCM of two numbers is 28 times their HCF.
The sum of HCF and LCM is 1740. If one of
RBI GRADE-B OFFICER EXAMS these numbers is 240, the sum of digits of
1. Punit, Vinit and Ajit begin to jog around a the other number is
stadium. They complete their revolutions (1) 4 (2) 5
in 45 seconds, 54 seconds and 36 seconds (3) 6 (4) 7
respectively. After how many seconds will (United India Insurance AAO
they be together at the starting point ? Exam. 03.06.2012)
(1) 360 (2) 600 7. L.C.M of two numbers is 120 and their
(3) 540 (4) 450 H.C.F. is 10. Which of the following can be
(5) None of these the sum of those two numbers ?
(RBI Grade-B Officer Exam. 2008 (1) 140 (2) 80
(3) 60 (4) 70
LEARN MATHS FROM S.K. RAJU (9811549822, 9811649822)
2
(NICL (GIC) AO (Finance) (NICL (GIC) AO (Finance) Exam.
Exam. 08.09.2013 (Paper-I) 08.09.2013 (Paper-I)
8. Find the maximum number of girls, among 9. The sum of two numbers is 45 and their
whom 2923 bags and 3239 eyeliners can product is 500. Their HCF is
be distributed in such a way that each girl (1) 5 (2) 9
ge ts the same n umbe r o f bags an d (3) 10 (4) 15
eyeliners. (NICL (GIC) AO Exam.
(1) 80 (2) 79 08.09.2013 (Paper-I)
(3) 78 (4) 81

LEARN MATHS FROM S.K. RAJU (9811549822, 9811649822)


3
SHORT ANSWERS 2028
NATIONALISED BANKS  xy = =12 = 3 × 4
13  13
& IBPS PO/MT/SO
1. (1) 2. (2)  Numbers = 13 × 3 = 39 and 13 × 4 = 52
3. (3)  Sum of numbers = 39 + 52 = 91
3. (1) LCM of 2, 3, 4, 5 and 6 = 60
RBI GRADE-B OFFICER EXAMS  Number = 60k + 1
It is exactly divisible by 7.
1. (3)
For k = 5. 60k + 1 is exactly divisibile by 7
 Number = 301
INSURANCE EXAMS  Required sum = 3 + 0 + 1 = 4
1. (3) 2. (3) 4. (1) Let the numbers be 103x and 103y
3. (1) 4. (1) where x and y are prime to each other.
5. (3) 6. (3)  LCM = 103 xy
7. (4) 8. (2)  103xy = 19261
9. (1)
19261
 xy = = 187
EXPLANATIONS 103
1. (1) The decimal equivalent of each fraction  x = 11 or 17 y = 17 or 11
 Numbers = 103 ×11 = 1133
5 2 and 103 × 17 = 1751
= 0.42; = 0.286;
12 7  Sum = 1751 + 1133 = 2884
3 5. (3) 3240 = 2 × 2 × 2 × 3 × 3 × 3 × 5
= 0.27 = 23 × 33 × 5
11
3600 = 2 × 2 × 2 × 2 × 3 × 3 × 5 × 5
2 1 = 24 × 32 × 52
= 0.2 ; = 0.2 LCM = 24 × 35 × 52 × 72
9 7
Clearly,  p = 22 × 35 × 72
because HCF = 36
5 2 3 2 1 6. (3) LCM = 28 × HCF
> > > >
12 7 11 9 5  L = 28 H
2. (2) Required time = LCM of 18, 24 and 32  L + H = 1740
seconds = 288 seconds.  28H + H = 1740
3. (3) Required time = LCM of 18, 22 and 30  29H = 1740
seconds = 990 seconds
1740
990  H= = 60
= minutes = 16 minute 30 seconds 29
60  L = 28 × 60 = 1680
 240 × second number
RBI GRADE-B OFFICER EXAMS 1680  60
1. (3) Time taken by Punit, Vinit and Ajit to be  Second number = = 420
240
together at the starting point
= LCM of 45, 54 and 36 seconds  Sum of digits = 4 + 2 + 0 = 6
7. (4) Numbers = 10x and 10y. Where x and y
= 3 × 3 × 2 × 5 × 3 × 2 = 540 seconds
are prime to each other.
 LCM = 10xy
INSURANCE EXAMS  10xy =120
1. (3) LCM
 xy = 12
LCM of 1, 5, 2, 4 20 Possible pair = (3, 4)
= =  Sum of numbers = 10 × 3 + 10 × 4 = 70
HCF of 3, 6, 9, 27 3
8. (2) Required answer
2. (3) Tricky approach
= HCF of 2923 and 3239 = 79
Let the number be 13x and 13y where x
and y are prime to each other.
 13x × 13y = 2028

LEARN MATHS FROM S.K. RAJU (9811549822, 9811649822)


4
 (x - y)2 = (x + y)2 - 4xy
2923)3239(1 = (45)2 - 4 × 500
2923 = 2025 - 2000 = 25
 x- y = 5 ... (iii)
316) 2923(9 From equations (i) and (iii),
2844 x = 25,y = 20
79 )316 (4  HCF = 5
316
x

9. (1) Let the numbers be x and y


Then
x + y = 45 ... (i)
xy = 500 ...(ii)

LEARN MATHS FROM S.K. RAJU (9811549822, 9811649822)


5
MODEL EXERCISES
1. The LCM and HCF of two numbers are 84 (1) 16 (2) 16000
and 21 respectively. If the ratio of the two (3) 160 (4) 1600
numbers is 1 : 4, then the larger of the two (5) None of these
numbers is 5. HCF of 3240, 3600 and a third number is
(1) 12 (2) 48 36 and their LCM is 24 × 35 × 52 × 72. The
(3) 84 (4) 108 third number is
(5) None of these (1) 2 2 × 3 5 × 72 (2) 22 × 53 × 72
3 5 2
2. A certain type of wooden board is sold only (3) 2 × 3 × 7 (4) 25 × 52 × 72
in lengths of multiples of 25 cm from 2 to (5) None of these
10 m. A carpenter needs a large quantity of 6. The HCF and LCM of two numbers are 21
this type of board in 1.65 m Ìengths. For and 4641 respectively. If on e of the
the minimum waste, the Ìengths to be numbers lies between 200 and 300, then
purchased should be the two numbers are
(1) 3.30 m (2) 6.60 m (1) 273, 357 (2) 273, 361
(3) 8.25 m (4) 9.95 m (3) 273, 359 (4) 273, 363
(5) None of these (5) None of these
3. The LCM of two numbers is 280 and their 7. Three different containers contain different
ratio is 7 : 8. The two numbers are quantities of a mixture of milk and water,
(1) 70, 80 (2) 42, 48 whose measurements are 403 kg, 434 kg
(3) 35, 40 (4) 28, 32 and 465 kg. What biggest measure be there
(5) None of these to measure all the different quantities
4. The LCM of two numbers is 4800 and their exactly ?
HCF is 160. If one of the numbers is 480, (1) 7 kg (2) 1 kg
then the other number is (3) 31 kg (4) 41 kg
(5) None of these

LEARN MATHS FROM S.K. RAJU (9811549822, 9811649822)


6
SHORT ANSWERS  Numbers are 35, 40.
4. (4) LCM × HCF = Product of two numbers
1. (3) 2. (3) 4800  160
3. (3) 4. (4)  Other number =
480
5. (1) 6. (1)
7. (3) = 1600
5. (1) 3600 = 4 × 9 × 100 = 22 × 32 × 52 × 22
=24 × 32 × 52
EXPLANATIONS 3240 = 810 × 4 = 32 × 32 × 2 × 5 × 22
1. (3) Let the numbers be x and 4x
= 34 × 23 × 5
According to the question,
Third number =22 × 35 × 72
84 × 21 = x × 4x
6. (1) Product of the numbers
 4x2 = 1764 = HCF × LCM = 21 × 4641
1764 = 21 × 3 × 7 × 13 × 17
 x2 = = 3 × 7 × 3 × 7 × 13 × 17.
4
 The numbers can be 3 × 7 × 13
 x = 441 = 21 and 3 × 7 × 17, i.e., 273 and 357.
Hence, the number be 21 and 84 Larger 7. (3) Required measure = HCF of Interference
number = 84 of 403, 434 and 465
2. (3) 1.65 m= 165 cm = 31 kg.
 Required length
= LCM of 25 and 165 = 825 cm = 8.25 m
3. (3) Let the number be 7x and 8x
 HCF = x
LCM × HCF = Product of numbers
 280 × x = 562x
x = 5

LEARN MATHS FROM S.K. RAJU (9811549822, 9811649822)


1
MENSURATION
NATIONALISED BANKS (5) None of these
& IBPS SO/MT/SO (Indian Overseas Bank PO
1. The area of a rectangular field is 460 square Exam. 15.06.2008)
metres. If the length is 15 per cent more 6. The circumference of a circle is equal to the
than the breadth, what is breadth of the side of a square whose area measures
rectangular field ? 407044 sq. cms. What is the area of the
(1) 15 metres circle ?
(2) 26 metres (1) 22583.2 sq. cms.
(3) 34.5 metres (2) 32378.5 sq. cms.
(4) Cannot be determined (3) 41263.5 sq. cms.
(5) None of these (4) 39483.4 sq. cms.
(Canara Bank PO Exam. 09.02.2003) (5) Cannot be determined
2. What will be the cost of gardening 1 metre (PNB Agriculture Officer
broad boundary around a rectangular plot Exam. 04.01.2009)
having perimeter of 340 metres at the rate 7. Wh at i s th e are a of a ci rcle w ho se
of Rs. 10 per square metre ? circumference is 1047.2 metres ?
(1) Rs. 3,400/- (1) 87231.76 sq.metres
(2) Rs. 1,700/­ (2) 85142.28 sq.metres
(3) Rs. 3,440/­ (3) 79943.82 sq.metres
(4) Cannot be determined (4) 78621.47 sq.metres
(5) None of these (5) 69843.23 sq.metres
(Cneara Bank PO Exam. 09.02.2003) (UCO Bank PO Exam. 22.03.2009)
3. What would be the cost of building a 7 8. An order was placed for supply of carpet of
metres wide garden around a circular field breadth 3 metres, the length of carpet was
with diameter equal to 280 metres if the 1.44 times of breadth. Subsequently the
cost per sq. metre for building the garden breadth and length were increased by 25
is Rs. 21 ? and 40 per cent respectively. At the rate of
(1) Rs. 1,56,242 Rs. 45 per square metre, what would be the
(2) Rs. 2,48,521 increase in the cost of the carpet ?
(3) Rs. 1,11,624 (1) Rs. 1020.6 (2) Rs. 398.8
(4) Rs. 2,06,118 . (3) Rs. 437.4 (4) Rs. 583.2
(5) None of these (5) None of these
(Corporation Bank PO (Indian Overseas Bank
Exam. 29.07.2006) PO Exam. 05.04.2009)
4. The ratio of length and breadth of a 9. The length of a rectangular plot is thrice
rectangular plot is 8 : 5 respectively. If the its breadth. If the area of the rectangular
breadth is 60 metre less than the length, what plot is 7803 sq. metre, what is the breadth
is the perimeter of the rectangular plot ? of the rectangular plot ?
(1) 260 metres (1) 51 metres (2) 153 metres
(2) 1600 metres (3) 104 metres (4) 88 metres
(3) 500 metres (5) None of these
(4) Cannot be determined (Indian Overseas Bank PO
(5) None of these Exam. 05.04.2009)
(Bank Of Maharashtra PO 10. The cost of fencing a circular plot at the
Exam. 25.05.2008) rate of Rs. 15 per metre is Rs. 3,300. What
5. What will be the area (in square metres) of will be the cost of flooring the plot at the
1.5 metre wide garden developed around rate of Rs. 100 per square metre ?
all the four sides of a rectangular field hav- (1) Rs. 3,85,000
ing area equal to 300 square metres and (2) Rs. 2,20,000
breadth equal to three-fourth of the length ? (3) Rs. 3,50,000
(1) 96 (4) Cannot be determined
(2) 105 (5) None of these
(3) 114 (United Bank of India
(4) Cannot be determined PO Exam. 21.06.2009)
LEARN MATHS FROM S.K. RAJU (9811549822, 9811649822)
2
11. If the area of a circle is 616 cm.2, what is its (2)
36 sq. cm.
circumference ? (3)
81 sq. cm.
(1) 76 cm. (2) 84 cm. (4)
Cannot be determined
(3) 96 cm. (4) 80 cm. (5)
None of these
(5) None of these (Punjab & Sind Bank PO
(Andhra Bank PO Exam. 05.07.2009) Exam. 16.05.2010)
12. 17.
The length of a rectangular floor is twice The total area of a circle and a square is
its breadth. If Rs. 256 is required to paint equal to 5450 sq.cm. The diameter of the
the floor at the rate of Rs. 2 per square circle is 70 cms. What is the sum of the
metres, then what would be the length of ci rcumfe re nce of th e ci rcle and the
floor ? perimeter of the square ?
(1) 16 metres (2) 8 metres (1) 360 cm
(3) 12 metres (4) 32 metres (2) 380 cm
(5) 20 metres (3) 270 cm
(Corporation Bank PO (4) Cannot be determined
Exam. 22.11.2009) (5) None of these
13. The circumference of two circles is 132 (Bank Of Baroda PO Exam. 30.05.2010)
metres and 176 metres respectively. What18. The area of a square is 1024 sq. cm. What
is the difference between the area of the is the respective ratio between the length
larger circle and the smaller circle ? and the breadth of a rectangle whose length
(1) 1048 sq. metres is twice the side of the square and breadth
(2) 1076 sq. metres is 12 cm. less than the side of the square ?
(3) 1078 sq. metres (1) 5 : 18 (2) 16 : 7
(4) 1090 sq. metres (3) 14 : 5 (4) 32 : 5
(5) None of these (5) None of these
(Indian Bank PO Exam. 17.10.2010) (Central Bank Of India PO
14. What would be the cost of building a fence Exam. 25.07.2010)
19.
around a circular field with area equal to There are two circles of different raddi. The
32378.5 sq. metres, if the price per metre area of a square is 196 sq.cm, whose side
for building the fence was Rs. 154 ? is half the radius of the larger circle. The
(1) Rs. 84,683 radius of the smaller circle is three-seventh
(2) Rs. 86,495 that of the larger circle. What is the circum-
(3) Rs. 79,326 ference of the smaller circle ?
(4) Rs. 98,252 (1) 12  cm (2) 16  cm
(5) None of these (3) 24  cm (4) 32  cm
(Bank Of India Banking Officer (5) None of these
Exam. 24.01.2010) (Syndicate Bank PO Exam. 29.08.2010)
15. 20.
The perimeter of a square is equal to twice The cost of building a fence around a
the perimeter of a rectangle of length 8 cms. circular field is Rs 7,700 at the rate of Rs
an d breadth 7 cms. What is the 14 per foot. What is the area of the circular
circumference of a semicircle whose field ?
diameter is equal to the side of the square (1) 24062.5 sq.ft.
? (Rounded off to the two decimal places) (2) 23864.4 sq.ft.
(1) 38.57 cms. (3) 24644.5 sq.ft.
(2) 23.57 cms. (4) Cannot be determined
(3) 42.46 cms. (5) None of these
(4) 47.47 cms. (United Bank Of India PO
(5) None of these Exam. 14.11.2010)
(Punjab & Sind Bank PO 21. The total area of a circle and a square is
Exam. 16.05.2010) equal to 2611 sq.cm. The diameter of the
16. The perimeter of a square is double the circle is 42 cms. What is the sum of the cir-
perimeter of a rectangle. The area of the cumference of the circle and the perimeter
rectangle is 240 sq. cm. What is the area of of the square ?
the square ? (1) 272 cms.
(1) 100 sq. cm. (2) 380 cms.
(3) 280 cms.
LEARN MATHS FROM S.K. RAJU (9811549822, 9811649822)
3
(4) Cannot be determined 27. The circumference of a semicircle of area
(5) None of these 1925 sq. cm is equal to the breadth of a
(Bank Of Maharashtra rectangle. If the length of the rectangle is
Exam. 19.12.2010) equal to the perimeter of a square of side
22. The area of a square is 196 sq. cm. whose 48 cm. What is the perimeter of the
side is half the radius of a circle. The rectangle ?
circumference of the circle is equal to (1) 734 cm
breadth of a rectangle. If perimeter of the (2) 754 cm
rectangle is 712 cm, what is the length of (3) 745 cm
the rectangle ? (4) Cannot be determined
(1) 196 cm (2) 186 cm (5) None of these
(3) 180 cm (4) 190 cm (UCO Bank PO Exam. 30.01.2011)
(5) None of these 28. The total area of a circle and a rectangle is
(Oriental Bank Of Commerce PO equal to 1166 sq.cm. The diameter of the
Exam. 26.12.2010 (1st Sitting) circle is 28 cms. What is the sum of the
23. What would be the cost of building a fence ci rcumfe re nce of th e ci rcle and the
around a square plot with area 462.25 sq. perimeter of the rectangle if the length of
ft. at the rate of Rs 34 per foot ? the rectangle is 25 cm ?
(1) Rs 2,924 (1) 186 cm
(2) Rs 2,682 (2) 182 cm
(3) Rs 2,846 (3) 184 cm
(4) Cannot be determined (4) Cannot be determined
(5) None of these (5) None of these
(Indian Bank PO Exam. 02.01.2011) (Bank Of Baroda PO Exam. 13.03.2011)
24. The length of a rectangle is twice the 29. The sum of the circumference of a circle
diameter of a circle. The circumference of and the perimeter of a square is equal to
the circle is equal to the area of a square of 272 cm. The diameter of the circle is 56 cm.
side 22 cm. What is the breadth of the What is the sum of the area of the circle
rectangle if its perimeter is 688 cm ? and the area of the square ?
(1) 24 cm (1) 2464 sq. cm.
(2) 26 cm (2) 2644 sq. cm.
(3) 52 cm (3) 3040 sq. cm.
(4) Cannot be determined (4) Cannot be determined
(5) None of these (5) None of these
(Union Bank Of India PO (Allahabad Bank PO Exam. 17.04.2011)
Exam. 09.01.2001) 30. The length of a rectangle is three-fifth of
25. The circumference of two circles is 88 the side of a square. The radius of a circle
metres and 220 metres respectively. What is e qu al to si de of the squ are. The
is the difference between the area of the circumference of the circle is 132 cm. What
larger circle and the smaller circle ? is the area of the rectangle if the breadth of
(1) 3422 sq. metre the rectangle is 8 cm ?
(2) 3242 sq. metre (1) 112.4 sq. cm.
(3) 3244 sq. metre (2) 104.2 sq. cm.
(4) 3424 sq. metre (3) 100.8 sq. cm.
(5) None of these (4) Cannot be determined
(Corporation Bank PO (5) None of these
Exam. 16.01.201l) (Indian Overseas Bank PO
26. What is the area of a circle whose radius is Exam. 22.05.2011)
equal to the side o f a square whose 31. Smallest side of a right angled triangle is 8
perimeter is 112 metres ? cm less than the side of a square of
(1) 176 sq. m. (2) 2504 sq. m. perimeter 56 cm. Second largest side of the
(3) 284 sq. m. (4) 1956 sc m. right angled triangle is 4 cm less than the
(5) None of these length of rectangle of area 96 sq. cm. and
(Punjab & Sind Bank PO breadth 8 cm. What is the largest side of
Exam. 23.01.2011) the right angled triangle ?
LEARN MATHS FROM S.K. RAJU (9811549822, 9811649822)
4
(1) 20 cm (2) 12 cm flooring the auditorium (only the floor of the
(3) 10 cm (4) 15 cm auditorium), if the cost of flooring is Rs. 12/
(5) None of these m2 ?
(Indian Overseas Bank PO (1) Rs. 87,954
Exam. 22.05.2011) (2) Rs. 91,236
32. The area of a square is 1444 square metre. (3) Rs. 94,284
1 (4) Rs. 75,490
The breadth of a rectangle is th of the (5) None of these
4
(IBPS RRBs Office Assistant
side of the square and the length of the CWE Exam. 09.09.2012)
rectangle is thrice the breadth. What is the 37. The circumference of a circular playground
difference between the area of the square is 308 metre. There is 7 metre wide path
and the area of the rectangle ? around the ground. The area of the path is
(1) 1152.38 sq.mtr. (1) 2130 sq. metre
(2) 1169.33 sq.mtr. (2) 2410 sq. metre
(3) 1181.21 sq.mtr. (3) 2510 sq. metre
(4) 1173.25 sq.mtr. (4) 2310 sq. metre
(5) None of these (5) None of these
(IBPS Bank PO/MT CWE 17.06.2012) (Indian Overseas Bank PO
4 Online Exam. 01.09.2013)
33. Circumference of a circle-A is l times 38. The edge of an ice cube is 14 cm. The
7
perimeter of a square. Area of the square is volume of the largest cylindrical ice cube
784 sq cm. What is the area of another that can be formed out of it is
circle-B whose diameter is half the radius (1) 2200 cu.cm
of the circle-A ? (2) 2000 cu.cm
(1) 38.5 sq.cm (3) 2156 cu.cm
(2) 156 sq.cm (4) 2400 cu.cm
(3) 35.8 sq.cm (5) None of these
(4) 616 sq.cm (IBPS Bank PO/MT
(5) None of these CWE-III 26.10.2013)
(IDBI Bank Officer Exam. 16.09.2012) Directions (39-40) : Study the following
34. A horse is tethered to a peg with a 14 metre diagram to answer the questions.
long rope at the corner of a 40 metre long (IBPS Bank PO/MT
and 24 metre wide rectangular grassfield. CWE-III 26.10.2013)
What area of the field will the horse graze ?
(1) 154 m2 (2) 308 m2
2
(3) 240 m (4) 480 m2
(5) None of these
(IBPS Specialist Officer 39. If the diameter of each circle is 14cm and
CWE 17.03.2013) DC = CE, the area of  BDE is
35. The area of circle is seven times the (1) 784 sq.cm (2) 748 sq.cm
numerical value of its circumference. What (3) 874 sq.cm (4) 441 sq.cm
is the circumference of the circle ? (5) None of these
(1) 616 units 40. The area of the shaded region of square
(2) 132 units ABCD is
(3) 88 units (1) 186 sq. cm (2) 168 sq. cm
(4) Cannot be determined (3) 188 sq. cm (4) 198 sq. cm
(5) None of these (5) None of these
(IBPS Specialist Officer
CWE 17.03.2013)
SBI PO EXAMS
36. Le ng th of the fl oo r of a re ctang ul ar
1.
If the height of a triangle is decreased 20%
auditorium is 6 metre more than the radius
and its base is increased by 20% what will
of a circle with a circumference of 572 m.
be the effect on its area?
The perimeter of the floor of the rectangular
(1) No change (2) 8% increase
auditorium is 356 m. What will be cost of
(3) 12% decrease (4) 16% increase
LEARN MATHS FROM S.K. RAJU (9811549822, 9811649822)
5
(5) None of these (SBI Associate Banks PO
(SBI Associate Banks PO Exam. 07.01.2007)
Exam. 14.02.1999) 7. What will be the cost of gardening 1 metre
2. A circular ground whose diameter is 35 broad boundary around a rectangular plot
metres, has a 1.4 metre broad garden having perimeter of 340 metres at the rate
around it. What is the area of the garden in of Rs. 10 per square metre ?
square metres ? (1) Rs. 3,400/-
(1) 160.16 (2) Rs. 1,700/-
(2) 6.16 (3) Rs. 3,440/-
(3) 1122.66 (4) Cannot be determined
(4) Data inadequate (5) None of these
(5) None of these (SBI Associate Banks PO
(SBI Associate Banks PO Exam. 07.01.2007)
Exam. 14.02.1999) 8. The ratio of the length and the breadth of a
3. 2 me tres bro ad path way is to be rectangular plot is 6 : 5 respectively. If the
constructed around a rectangular plot. The breadth of the plot is 34 metres less than
area of the plot is 96 sq.m. The cost of the length, what is the perimeter of the
construction is Rs. 50 per sq. metre. Then rectangular plot ?
find the total cost of production ? (1) 374 metres (2) 408 metres
(1) Rs. 4,800 (2) Rs. 4,000 (3) 814 metres (4) 748 metres
(3) Rs. 2400 (4) Data inadequate (5) None of these
(5) None of these (SBI PO Preliminary (Tire-I)
(SBI Associate Banks PO Exam. 27.04.2008)
Exam. 16.07.2000) 9. The diameter of a circle is equal to the
4. If length of the rectangle is increased by perimeter of a square whose area is 3136
50% and breadth is decreased by 20% . sq. cms. What is the circumference of the
Then what is the percentage change in the circle ?
area ? (1) 352 cms.
(1) by decrease 20% (2) 704 cms.
(2) 20% increase (3) 39424 cms.
(3) 80% increase (4) Cannot be determined
(4) 30% decrease (5) None of these
(5) None of these (SBI PO Preliminary (Tire-I)
(SBI Associate Banks PO Exam. 27.07.2008)
Exam. 16.07.2000) 10. If the length of a rectangular field is
5. The length and breadth of the floor of a room increased by 20% and the breadth is
are 20 feet and 10 feet respectively. Square reduced by 20% , the area of the rectangle
tiles of 2 feet length of three different will be 192 m2. What is the area of the
colours are to be laid on the floor. Black original rectangle ?
tiles are laid in the first row on all sides. If (1) 184 m2 (2) 196 m2
white titles are laid in the one-third of the (3) 204 m2 (4) 225 m2
remaining and blue tiles in the rest, how (5) None of these
many blue tiles will be there ? (SBI Associate Banks PO
(1) 16 (2) 32 Exam. 07.08.2011)
(3) 48 (4) 24 11. Inside a square plot a circular garden is
(5) None of these developed which exactly fits in the square
(SBI Banks PO Exam. 20.08.2000) plot and the diameter of the garden is equal
6. The area of a rectangular field is 460 square to the side of the square plot which is 28
metres. If the length is 15 per cent more metres. What is the area of the space left
than the breadth, what is breadth of the out in the square plot after developing the
rectangular field ? garden ?
(1) 15 metres (1) 98 m2 (2) 146 m2
(2) 26 metres (3) 84 m2 (4) 168 m2
(3) 34.5 metres (5) None of these
(4) Cannot be determined (SBI Associate Banks PO
(5) None of these Exam. 07.08.2007)
LEARN MATHS FROM S.K. RAJU (9811549822, 9811649822)
6
RBI GRADE-B OFFICER EXAMS 5. A rectangular carpet has an area of 120 sq.
1. The length of rectangular plot is thrice its metres and a perimeter of 46 metres. The
breadth. If the area of the rectangular plot length of its diagonal (in metres) is :
is 6075 sq. metres, what is the length ? (1) 11 (2) 13
(1) 145 metres (2) 130 metres (3) 15 (4) 17
(3) 75 metres (4) 45 metres (United India Insurance Co.
(5) None of these (AAO) Exam. 11.03.2007)
(RBI Grade-B Officcer Exam 2007) 6. The area of a square of side 8 cm is equal
2. A triangle has two of its angle in the ratio to a rectangle. Which of the following
of 1 : 2. If the measure of one of its angles statement/s is/ are definitely true about
is 30 degrees, what is the measure of the the rectangle ?
largest angle of the triangle in degrees ? (1) The length of rectangle is 16 times its
(1) 100 breadth
(2) 90 (2) The length of rectangle is 32 times its
(3) 135 breadth
(4) Cannot be determined 1
(5) None of these (3) The breadth of rectangle is of its
6
(RBI Grade-B Officer B Exam.06.02.2011)
length
1
INSURANCE EXAMS (4) The breadth of rectangle is of its
1. A wall 3m × 2.7m × 0.2m of concrete weight 9
320 kg per cubic meter. What will be the length
total weight of the wall in kg.? (5) None of these
(1) 5184.0 (2) 51.84 (LIC Assistant Administrative
(3) 518.4 (4) 51840 Officer Exam. 2008)
(United India Insurance Co. 7. An equilateral triangle and a regular
AAO Exam. 21.04.2002) hexagon have equal perimeters. If the area
2. The area of a square is 2.25 cm2. What is of the triangle is 2 cm2, then the area of the
its perimeter ? hexagon is
(1) 9.0 cms (2) 6.0 cms (1) 2 cm2 (2) 3 cm2
2
(3) 1.5 cms (4) 4.5 cms (3) 4 cm (4) 6 cm2
3. The length of a rectangle is 20% more than (New India Assurance AO
its breadth. What will be the ratio of the Exam. 25.10.2009)
area of this rectangle to the area of a square 8. A solid is in the form of a right circular
whose side is equal to the breadth of the cylinder with hemispherical ends. The total
rectangle ? length of the solid is 35 cm. The diameter
(1) 5 : 6 of the cylinder is 1/4 of its height. The
(2) 6 : 5 surface area of the solid is
(3) 2 : 1 22
(4) Data inadequate (Take  = )
7
(5) None of these (1) 462 cm2 (2) 693 cm2
LIC Assistant Administrative (3) 750 cm 2
(4) 770 cm2
Officer (AAO) Exam. 24.04.2005) (New India Assurance AO
4. A well with 14 m inside diameter is dug 10 Exam. 25.10.2009)
m deep. Earth taken out of it has been 9. A solid metalic sphere of radius r is
evenly spread all around it to a width of 21 converted into a solid right circular cylinder
m to form an embankment. The height (in of radius R. If the height of the cylinder is
metres) of the embankment is : twice the radius of the sphere, then
1 2
(1) (2) 2
2 3 (1) R = r (2) R = r
3
3 3
(3) (4) 2r 2r
4 5 (3) R = (4) R =
(United India Insurance Co. 3 3
(AAO) Exam. 11.03.2007)
LEARN MATHS FROM S.K. RAJU (9811549822, 9811649822)
7
(New India Assurance AO inscribed in a semicircle of the same circle,
Exam. 25.10.2009) in cm2, is
10. The radius of a circle is twice the side of a (1) 5 (2) 6
square of area 196 sq.cm. Length of a (3) 7.5 (4) 50
rectangle is twice the diameter of the circle.
(New India Insurance AAO
What is the perimeter of the rectangle if its
Exam. 22.05.2011)
breadth is half the length of the rectangle ?
15. Diameter of a cylindrical jar is increased
(1) 244 cm
by 25% . By what percent must the height
(2) 168 cm
be decreased so that there is no change in
(3) 336 cm
its volume ?
(4) Cannot be determined
(1) 18% (2)25%
(5) None of these
(3) 32% (4) 36%
(United India Insurance AO
(New India Insurance
Exam. 27.03.2011)
AAO Exam. 22.05.2011)
11. The smallest side of a right angled triangle
16. The area of a rectangle gets reduced by 9
is 6 cm. and second largest side is 8 cm.
sq. metre if its length is reduced by 5 m
Side of a square is thrice the largest side of
and breadth is increased by 3 m. If we
the triangle. What is the diagonal of the
increase the length by 2 m and breadth by
square ?
3 m, the area is increased by 67sq. metre.
(1) 30 2 cm The length of the rectangle is:
(1) 9 m (2) 15.6 m
(2)60 2 cm
(3) 17 m (4) 18.5 m
(3)30 cm. (General Insurance Corporation
(4)Cannot be determined AAO Exam. 11.12.2011)
(5)None of these 17. Height of a cylindrical jar is decreased by
(United India Insurance AO 36% . By what percent must the radius be
Exam. 27.03.2011) increased so that there is no change in its
12. In the figure, ABCDEFGH is a regular volume ?
octagon. What fraction of its area is shaded ? (1) 25 (2) 35
(3) 36 (4) 40
(Oriental Insurance Company AAO
Exam. 08.04.2012)
18. Suface area of a cuboid is 22 cm2 and the
1 1 sum of the lengths of all its edges is 24 cm.
(1) (2) Length of each diagonal of the cuboid (in
3 4
cm) is
1 3
(3) (4) (1) 11 (2) 13
5 8
(New India Insurance AAO (3) 14 (4) 15
Exam. 22.05.2011) (Oriental Insurance Company
13. In the figure there are two rectangles ABCD AAO Exam. 08.04.2012)
and DEBG, each of length 7 cm and width 19. An equilateral triangle and a regular
3 cm. The area of shaded region, in cm2, is hexagon have equal perimeters. The ratio
approximately of the area of the triangle and that of the
hexagon is
(1) 1 : 1 (2) 2 : 3
(3) 3 : 2 (4) 3 : 4
(Oriental Insurance Company
(1) 12 (2) 10 AAO Exam. 08.04.2012)
(3) 8 (4) 4 20. In the figure, ABCD is a parallelogram with
(New India Insurance AAO area 120 cm2, and BX : XC = 3 : 2, CY : YD
Exam. 22.05.2011) = 2 : 1 and AZ: ZD = 3 : 1. Area (in cm2) of
14. If the area of a square inscribed in a circle pentagon AXCYZ is
is 15 cm2, then the area of the square

LEARN MATHS FROM S.K. RAJU (9811549822, 9811649822)


8
4
(3)  (4) 7
3
(NICL (GIC) AO (Finance)
Exam. 08.09.2013 (Paper-I)
26. The areas of three consesutive faces of a
(1) 47 (2) 63 cuboid are 12 cm2, 20 cm2 and 15 cm2 ,
(3) 73 (4) 79 then the volume (in cm3) of the cuboid is
(United India Insurance AAO (1) 3600 (2) 100
Exam. 03.06.2012) (3) 80 (4) 60
21. A and B travel around a circular path at (NICL (GIC) AO Exam. 08.09.2013
uniform speeds in opposite directions, (Paper-I)
starting from diametrically opposite points, 27. The wheel of a motor car makes 1000
at the same time. They meet each other first revolutions in moving 440 m. The diameter
after B has travelled 100 metres and meet (in metre) of the wheel is
again 60 metres before A completed one (1) 0.44 (2) 0.14
round. The circumference of the park is (3) 0.24 (4) 0.34
(1) 240 m (2) 300 m (NICL (GIC) AO Exam. 08.09.2013
(3) 320 m (4) 480 m (Paper-I)
(United India Insurance AAO 28. The perimeters of a circular and a square
Exam. 03.06.2012) field are equal. What is the diameter of the
22. The perimetres of a square and a regular circular field if the area of the square field
hexagon are equal. The ratio of the area of 484 metre2 ?
the hexagon to the area of the square is (1) 14 metre (2) 21 metre
(1) 2 3 : 3 (2) (3) 28 metre (4) None of these
3 :1
(NICL (GIC) Administratis Officer
(3) 3 3 : 2 (4) 2 : 3 Exam. 15.12.2013
(United India Insurance AAO 29. For a sphere of radius 10 cm the numerical
Exam. 03.06.2012) value of the surface area is how many per
23. The surface of water in a swimming pool cent of the numerical value of its volume ?
forms a rectangle with length 40 m and (1) 24% (2) 26.5%
breadth 15 m. The depth of water increases (3) 30% (4) 45%
uniformly from 1.2 m to 2.4 m at the other (NICL (GIC) AO (Finance
end. The volume (in m3) of water in the pool Exam. 15.12.2013)
is 30. A circle and a rectangle have the same
(1) 500 (2) 540 perimeter. The sides of the rectangle are
(3) 720 (4) 1080 18 cm and 26 cm. What is the area of the
(United India Insurance AAO circle ?
Exam. 03.06.2012) (1) 88 cm2 (2) 154 cm2
2
24. The circumference of a park is 750 m. A (3) 616 cm (4) 1250 cm2
and B start walking from the same point in (NICL (GIC) AO (Finance Exam.
the same direction at 6.75 kmph and 4.75 15.12.2013)
kmph. In what time will they meet each 31. A rectangular courtyard 3.78 m long and
other again ? 5.25 m broad is to be paved exactly with
(1) 3 hours (2) 2.5 hours square tiles all of the same size. The
(3) 3.5 hours (4) 4 hours minimum number of such tiles is
(5) None of these (1) 430 (2) 440
(LIC Assistant Administrative (3) 450 (4) 460
Officer (AAO) Exam. 12.05.2013) (NICL (GIC) AO (Finance
25. A hollow speherical silver ball has an Exam. 15.12.2013)
external diameter 4 cm and internal
diameter 2 cm thick. Then the volume of
the silver used in the ball is :
28
(1)  (2) 1
3
LEARN MATHS FROM S.K. RAJU (9811549822, 9811649822)
9
SHORT ANSWERS EXPLANATIONS
NATIONALISED BANKS NATIONALISED BANKS
& IBPS PO/MT/SO & IBPS PO/MT/SO
1. (5) 2. (3) 1. (5) Let the breadth of rectangular field be x
3. (5) 4. (5) metres.
5. (3) 6. (2) 115 23x
7. (1) 8. (3)  Length = x × =
20
metres
100
9. (1) 10. (1) Now, Length × Breadth = Area
11. (5) 12. (1)
13. (3) 14. (4) 23x
 20 × x = 460
15. (1) 16. (4)
17. (2) 18. (5) 460  20
19. (3) 20. (1) x= 23
21. (1) 22. (3)
23. (1) 24. (2)  x 2
= 20 × 20
25. (5) 26. (5)  x = 20  20 = 20 metres
27. (5) 28. (2)
2. (3) Let the length of rectangular plot of x
29. (3) 30. (3)
and breadth be y.
31. (3) 32. (4)
Perimeter = 2 (x + y) = 340 metres
33. (5) 34. (1)
Area of boundary = (x + 2) + (y + 2) - xy
35. (3) 36. (3)
= xy + 2x + 2y + 4 - xy
37. (4) 38. (3)
= 2x + 2y + 4
39. (1) 40. (2)
= 2 (x + y) + 4
= 340 + 34 = 344
SBI PO EXAMS  Cost of gardening = 344 × 10 = Rs. 3440
1. (2) 2. (1) 3. (5)
3. (4) 4. (2)
5. (1) 6. (5)
7. (3) 8. (4)
9. (2) 10. (5)
11. (4)

RBI GRADE-B OFFICER EXAMS Radius of the field = 140 m


1. (5) 2. (4) Width of garden = 7 m
 Area of garden
INSURANCE EXAMS =  (1472 - 1402)
1. (3) 2. (2) 22
3. (2) 4. (2) = (147 + 140) (147-140)
7
5. (4) 6. (1) = 22 × 287 = 6314 m2
7. (2) 8. (4) Required cost = Rs. (21 × 6314)
9. (2) 10. (3) = Rs. 132594
11. (1) 12. (4) 4. (5) Let the length and breadth of the
13. (1) 14. (2) rectangular plot be 8x and 5x metres
15. (4) 16. (3) respectively,
17. (1) 18. (3) According to the question,
19. (2) 20. (4) 8x - 5x = 60
21. (4) 22. (1)
 3x = 60
23. (4) 24. (1)
25. (1) 26. (4) 60
27. (2) 28. (3) x = = 20
3
29. (3) 30. (3) Perimeter of the plot = 2 (8x + 5x)
31. (3) = 26x metre = 26 × 20 = 520 metre
5. (3) Let the length of the rectangular field
= x metre
LEARN MATHS FROM S.K. RAJU (9811549822, 9811649822)
10
3x  125   140 
 Its breadth = metre  3  100  ×  3  1.44  
4    100 
According to the question, = 22.68 sq. metre
3x  Increase in area = (22.68 -12.96) sq. metre
x× = 300 = 9.72 sq. metre
4
 Increase in the cost = Rs. (9.72 × 45)
4 = Rs. 437.4
 x 2 = 300 × = 400
3 9. (1) Let the breadth of the rectangular plot
 x = 400 = 20 be x metre.
 Length = 3x metre
 length = 20 metre
According to the question, 3x × x = 7803
3
and breadth = × 20 = 15 metre 7803
4  x2 = = 2601
3
 x = 2601 = 51 metre
10. (1) Total cost of fencing = Rs. 3300
Rate = Rs. 15 /metre
Length of the field with garden 3300
= 20 + 1.5 × 2 = 23 metre  Circumference of circular plot =
15
Breadth of the field with garden
= 15 + 1.5 × 2 = 18 metre = 220 metre
 Its area = 23 × 18 = 414 square metre  2  r = 220
 Area of the garden = 414 - 300 2  22
= 114 square metre  × r = 220
7
6. (2) Side of the square = Area 220  7
r= = 35 metre
= 407044 = 638 cm 2  22
 Circumference of the circle = 638 cm  Area of the circular plot =  r 2
 2  r = 638
22
22 = × 35 × 35 = 3850 sq.metre
 2× × r = 638 7
7
 Cost of flooring =3850 × 100 = Rs. 385000
638  7 29  7 22 2
r= = cm 11. (5)  r 2 = 616  r = 616
2  22 2 7
 Area of the circle
616  7
22 29  7  29  7  r2 = = 196
= × = 32378.5 sq. cm. 22
7 2 2
 r = 196 = 14 cm .
7. (1) 2  r = 1047.2
 Circumfernce of the circle = 2  r
22
2× × r = 1047.2 22
7 =2× × 14 = 88 cm
7
1047.2  7 12. (1) Rate of painting = Rs. 2/ sq.metre
r= 2  22
= 166.6 sq.metre
 Area of the rectangular floor
Area of the circle =  r2 256
22 = = 128 sq.m.
2
= × 166.6 × 166.6 = 87231.76 sq. metre
7 Let the breadth of floor be x metre
8. (3) Initial area of the carpet  length = 2x metre
= 3 × (3× l.44) sq. metre = 12.96 sq. metre  2x × x = 128
After corresponding changes in dimensions,
128
Area of the carpet =  x2 = 2
= 64

LEARN MATHS FROM S.K. RAJU (9811549822, 9811649822)


11
 x = 64 = 8 18. (5) Side of the square = 1024 = 32 cm.
 Length of floor = 2x = 2 × 8 = 16 metre  Length of rectangle = 2 × 32 = 64 cm.
13. (3) 2 r1 = 132 Breadth of rectangle
= 32 - 12 = 20 cm.
22  Required ratio = 64 : 20 = 16 : 5
2× × r1 = 132
7 19. (3) Radius of larger circle
132  7 =2× 196 = 28 cm
 r1 = 2  22 = 21 metre
3 
Similarly, Radius of smaller circle =   28  cm
7 
2 r2 = 176 = 12 cm
176  7 Circumference of smaller circle
 r2 = 2  22 = 28 metre
= 2 r = 2 × 12 = 24  cm
 Difference between their area 20. (1) Circumference of circular plot

=  r  r  =  (r
2 2 7700
2 1 2
+ r1)(r2 - r1) = = 550 feet
14
22
= × 49 × 7 = 1078 Sq. metre.  2 r = 550
7
550 550  7
14. (4)  r 2 = 32378.5 r= = = 87.5 feet.
2 2  22
32378.5  7
 r2 = 22
22  Area = × 87.5 × 87.5 = 24062.5 sq.feet.
7
32378.5  7 21. (1) If the side of square be x cm
r = = 101.5 metres then,
22
 Circumference of the circle  r 2 + x2 = 2611
22 22
= 2 r = 2 × × 101.5 = 638 metre  × 21 × 21 + x2 = 2611
7 7
 Expenditure on fencing = Rs. (154 × 638)  1386 + x2 = 2611
= Rs. 98252  x2 = 2611 - 1386 = 1225
15. (1) Perimeter of square
= 2 × Perimeter of rectangle  x = 1225 = 35 cm
= 2 × 2 (8 + 7) = 60 cm.  Circumference of circie + Perimeter of
60 square = 2 r + 4x
 Side of square = = l5 cm.
22
4
=2× × 21 + 4 × 35
 Diameter of semi-circle = 15 cm. 7
 Cicumference of semi-cricle = 132 + 140 = 272 cm
d 22 22. (3) Side of square
= +d = × 15 + 15 = 38.57 cm.
2 7 2 = Area = 196 = 14 cm
16. (4) Data are inadequate.  Radius of circle = 28 cm
17. (2) Tricky approach  Circumfence of circle
If the side of the square be x cm then, 22
 × 35 × 35 + x2 = 5450 =2× × 28 = 176 cm
7
22 If the lenght of rectangle be x cm
 × 35 × 35 + x 2 = 5450
7 then,
 x2 = 5450 - 3850 = 1600 2(x + 176) = 712
 x = 40 cm. 712
 Required sum =  × d + 4x  x + 176 = = 356
2
 22   x = 356 -176 = 180 cm
=   70  4  40  cm = 380 cm
 7 
LEARN MATHS FROM S.K. RAJU (9811549822, 9811649822)
12
23. (1) Side of the square plot = Area 22
= × 28 × 28 = 2464 sq. metre
= 7
462.25 = 21.5 feet
 Perimeter = 4 × Side r 2
= 4 × 21.5 = 86 feet 27. (5) = 1925
2
 Cost of building a fence
= Rs. (34 × 86) = Rs. 2924   r 2 = 1925 × 2
24. (2) Area of the square = 22 × 22 = 484 sq.cm 22
 Circumference of circle = 484 cm  × r2 = 1925 × 2
7
  × Dimater = 484
1925  2  7
22  r2 = = 1225
 × Dimater = 484 22
7
 r = 1225 = 35
484
 Dimater = 22 × 7 = 154 cm  Breadth of rectangle
=  r + 2r
 Lenght of rectangle = 2 × 154 cm
= 308 cm  22 
=    2 r =   2  × 35 = 180 cm
 2(le nght + breadh t) = Pe rimete r of  7 
rectangle Lenght of rectangle = Perimeter of square
 2(308 + x) = 668 [Breadht = x (let)] = 4 × 48 = 192 cm
668  Perimeter of recangle
 308 + x = 2
= 334 = 2 (lenght + breadht)
= 2 (192 + 180) = 744 cm
 x = 334 - 308 = 26 cm
(2) Area of circle =  r
2
25. (5) 2 r1 = 88 28.

22 22
= × 14 × 14 = 616 sq. cm.
2× × r1 = 88 7
7
 Area of rectangle = 1166 - 616
88  7 = 550 sq.cm.
 r1 = 2  22 = 14 metre
550
2  r = 220 Breadth of rectangle = = 22cm.
25
22  Circumference of circle =  × diameter
2× × r2 = 220
7 22
220  7 = × 28 = 88 cm.
7
 r2 = 2  22 = 35 metre
Perimeter of rectangle
Required diffrence = 2 (length + breadth)
= 2 (25 + 22) = 94 cm.
  r2  r1  =
22
2 2
r  r  r2  r1 
7 2 1
Required sum = 88 + 94 = 182 cm.
29. (3) Circumference of circle
22 =  × diameter

7
 35  14  (35 - 14)
22
= × 56 = 176 cm
22 7
 × 49 × 21 = 3234 sq.metre  Perimeter of square
7
= 272 - 176 = 96 cm
Perimeter
26. (5) Side of a square = 96
4  Side of square = = 24 cm
4
112  Area of square = 24 × 24 = 576 sq.cm.
= = 28 metre
4 Area of circle =  r 2
 Radius of circle = 28 metre
22
 Area of circle =  r 2 = × 28 × 28 = 2464 sq.cm
7

LEARN MATHS FROM S.K. RAJU (9811549822, 9811649822)


13
 Required sum = (576 + 2464) sq. cm. 22
= 3040 sq. cm.  2× × r = 16 × 11
7
30. (3) 2 r = 132
16  11  7
22 r= 2  22
= 28
2× × r = 132
7
28
132  7 Radius of circle-B = = 7 cm
 r= = 21 cm 4
2  22
 Side of square = 21 cm  Area =  r 2

3  21 22
 Length of the rectangle = cm = × 7 × 7 = 154 sq. cm.
5 7
34. (1)
3  21
 Area of the rectangle = ×8
5
= 100.8 sq.cm.
Perimeter
31. (3) Side of a square =
4
56 1
= = 14 cm Required area = ×  R2
4 4
 Smallest side of the right angled triangle 1 22
= 14 - 8 = 6 cm. = × × 14 × 14
4 7
Area = 154 sq. metre
Lenght of rectangle =
Breadht 35. (3)  R2 = 7 × 2  R
96  R = 14
= = 12 cm  Circumference of circle
8
 Second side of the triangle 22
= 2 R = 2 × × 14
= 12 - 4 = 8 cm 7
 Hypotenuse of the right angled tringle = 88 units
36. (3) Circumference of circle = 2  r
= 62  82
22
= 36  64 = 100 = 10 cm  572 = 2 × ×r
7
32. (4) Side of square = 1444 = 38 metre 572  7
r= = 91
1 19 2  22
Breadht of rectangle = × 38 = metre
4 2  Length of floor = 91 + 6 = 97 metre = l
 2(l + b) = 356
57  2 (97 + b) = 356
Lenght of rectangle = metre  97 + b = 178
2
 b = 178 - 97 = 81 metre
 Area of floor = 97 × 81 = 7857 sq. metre
19 57  Required cost = 12 × 7857 = Rs. 94284
Required difference = 1444 - ×
2 2 37. (4) 2 r1 = 308 where r1 = radius of circular
= 1444 - 270.75 = 1173.25 sq. metre playground
33. (5) Side of square = 784 = 28 cm
 Circumference of circle-A
11
= 4 × 28 ×
7
 2 r = 16 × 11

LEARN MATHS FROM S.K. RAJU (9811549822, 9811649822)


14
22 hb
2× × r1 = 308
7 25  100
change in area of triangle =
308  7 hb
 r1 = 2  22 = 49 metre 2
= -8 %
 Area of the path =  r2  r1 
2 2
Hence the area of triangle decreased by 8% .
2. (1)
22
= (56 + 49) (56 - 49)
7
= 22 × 105 = 2310 sq. m.
14
38. (3) Radius of cylinder = = 7 cm
2
Height of cylinder = 14 cm Required area
 Required volume =  r 2h =  [(18.9)2 - (17.5)2]
22 22
= × 7 × 7× 14 = 2156 cu.cm. = [(18.9 + 17.5) (18.9 - 17.5)]
7 7
39. (1) In  BDE, 22
DE = 28 + 28 = 56 cm = [36.4 × 1.4]
7
BC = 28 cm
22
1 = × 36.4 × 1.4
Area of  BDE = × DE × BC 7
2 = 22 × 36.4 × 0.2 = 160.16 sq. metres.
1 3. (4)
= × 56 × 28 = 784 sq.cm
2
40. (2) Area of square = 28 × 28 = 784 sq. cm.
Area of four circles = 4 r 2
22
=4 × × 7 × 7 = 616 sq.cm
7 (x + 4)(y + 4) = xy + 4(x + y) + 16
Area of shaded region = 96 + 4 (x + y) + 16
= 784 - 616 = 168 sq.cm New Area = 112 + 4 (x + y)
 We should have either x or y to complete
SBI PO EXAMS the question.
1. (2) Let height and base of triangle be h and xy
b respectively 4. (2) % change = x + y +
100
1
Area of triangle = hb 50  20
2 = + 50 - 20 +
100
Accordig this condition area of triangle
= 30 - 10 = 20% (increase)
1 5. (1) Area of the floor = 20 × 10
= (h - 20% of h) (b + 20% of b)
2 = 200 sq. ft.

1 h b
= h  5  b  5 
2  
12hb hb hb
Change in area = - =
25 2 25 Area covered by black tiles
= 2 (20 + 20 + 6 + 6) = 104 sq.ft.
Remaing in space = 200 - 104 = 96 sq.ft.
1
Area covered by white tiles = × 96
3

LEARN MATHS FROM S.K. RAJU (9811549822, 9811649822)


15
= 32 sq.feet 11. (4)
Area covered by blue tiles = 96 - 32
= 64 sq. feet
Area of 1 tile = (2)2 = 4 sq.ft.
64
No. of blue tile = = 16
4
6. (5) Let the breadth of rectangular field be x Radius of the circular garden
metres. 28
= = 14 metre
115 23x 2
 Length = x × = metres
100 20  Its area =  r 2
Now, Length x Breadth = Area
22
23x = × 14 × 14 = 616 sq. metre
 20 × x = 460 7
Area of the square plot = 28 × 28
460  20 = 784 sq. metre
 x2 = 23  Area of the shaded region
= 784 - 616 = 168 sq. metre
 x = 20 × 20
2

 x = 20  20 = 20 metres RBI GRADE-B OFFICER EXAMS


7. (3) Total area of rectangular plot to be 1. (5) Let the breadth of rectangular plot
gardened = 340 × 1 + 4 × 1 = 344 sq.m. = x m.
 Total cost = 344 × 10 = Rs. 3440  Length = 3x m
8. (4) Let the length and breadth of the According to the question,
rectangular plot be 6x and 5x metres 3x × x = 6075
respectively. 6075
According to the question,  x2 = = 2025
3
6x - 5x = 34
 x = 34  x = 2025 = 45
 Perimeter of the plot = 2 (length + breadth) 2. (4) If the angle be 30°, then according to
= 2 (6x + 5x) = 22x metre ratio, Second angle = 60°
= 22 × 34 = 748 metre  Third angle = 180° - 60° - 30° = 90°
9. (2) Area of the square = 3136 cm2 If the given ratio be of two other angles,
then
 Side of the square = 3136 = 56 cm.
x + 2x = 180° - 30° = 150°
 Its perimeter = 4 × 56 = 224 cm  3x = 150°  x = 50°
 Diameter of circle = 224 cm
 Second angle = 2 × 50 = 100°
 Circumference of circle =  × diameter
Hence, the largest angle = 90° or 100°
22
= × 224 = 704 cm
7 INSURANCE EXAMS
10. (5) Let the length and breadth of the original 1. (3) Required answer
rectangle be x and y metres respectively. = (3m × 2.7m × 0.2m) × 320 kg.
 Area of the original rectangle = 518.4 kg.
= xy square metre
According to the question,
120 80
x × ×y× = 192
100 100 2. (2)
 x × 1.2 × y × 0.8 = 192
192 According to the question,
 xy = = 200 square metre
1.2  0.8 x × x = 2.25 cm2
 x = 2.25 cm = 1.5 cm
 Perimeter = 4x = 4 × 1.5 cm = 6 cm

LEARN MATHS FROM S.K. RAJU (9811549822, 9811649822)


16
3. (2) Let the breadth of the rectangle be x
 According to question, 3
× side2 = Area of equilateral triangle
Length = 1.20 x 4
 Area of rectangle = 1.20 x × x = 1.20 x2
Area of square = x × x = x2 3
= × a2 = 2
4
1.20x 2
 Required ratio = 8
x2  a2 =
3
12 6
= = =6:5 Now, side of a regular hexagon
10 5
3 a
4. (2) Volume of earth dug out = × side =
6 2
 22 
=  7  7  10  m3 = 1540 m3  Area of regular hexagon
 7 
3 3 a2 3 3 8
22  = × = × = 3 cm2
 28    7   m2 34
2 2
Area of embankment = 2 4 2
7 
8. (4)
22
= × (28 + 7)(28 - 7) m2
7
22
= × 35 × 21 m2 = 2310 m2
7
Volume
 Height of embankment =
Area
Let radius of the cylinder be r cm and height
1540 2
= m= m = h cm.
2310 3
5. (4) Let the length of carpet h
Then, r = given
= a m and breadth = b m. 8
Then, h
2 (a + b) = 46 and h = 35 - 2r = 35 - 2 ×
8
 a + b = 23
Again, ab = 120 h
h+ = 35
4
 Diagonal = a 2  b 2
5h
 = 35
a  b   2ab
2
= 4
35  4
23   240
2
=  = 28 cm.
5
= 289 = 17 m h 28 7
6. (1) From statement (1), r = = = cm
2
8 8
If breadth of rectangle be x cm,
length = 16x 7
 Radius of the hemisphere = 2 cm
 16x × x = (8)2  x = 2
 Length = 32 cm. Total surface of area the solid
From statement (2), = (2  rh + 2 × 2  r 2 ) sq. cm.
If breadth = 2 cm = 2  r (h + 2r) sq. cm.
Length = 32 2 cm 22 7  7
=2× × ×  28  2   sq. cm.
Hence, area of rectangle = area of square. 7 2  2
Clearly, these two statements are definitely
true. 22 7
=2 × × × 35 sq. cm. = 770 sq. cm.
7. (2) Tricky approach 7 2

LEARN MATHS FROM S.K. RAJU (9811549822, 9811649822)


17
9. (2) Tricky approach Required area = 7 × 3 - 3 × 3 = 12 sq.cm.
In both cases, the volume will be the same. 14. (2)
4 2

3 r
3=
 R 2 .2r  3 r3= R2
I.
2
R = r
3
10. (3) Side of the square
= 196 = 14 cm
Radius of circle = 2 × 14 = 28 cm II.
Length of rectangle = 2 × 2 × 28 = 112 cm
112 DB =
Breadth = = 56 cm 2 × side
2
= 2 × 15 = 30
Perimeter = 2 (112 + 56) = (2 × 168) cm
= 336 cm 30
11. (1) For a right angled triangle,  Radius of Circle = cm
2
Hypotenuse
In second figure,
= 62  82 = 36  64
30
= 100 = 10 cm = Largest side OF =
2
 Side of square = 3 × 10 = 30 cm If FG = x, then
Dignonal of square
= 2 × 30 = 30 2 cm x2 5x 2
OF = x2  =
12. (4) 4 4

30 5x 2
 =
2 4

30 5 x 2
= +  x2 = 6 sq.cm
4 4
15. (4) Original volume of the cylindrical jar
3 2
Area of octagon = 8 × x = r h
2
4
2
= 2 3 x2 where, x = AB  125r 
   × h1 =  r h
2

Area of ABCH  100 


1 25
=
2
(2x + x) × AI  h =h
16 1
1 x2 h1 16
= × 3x × x 
2
 =
2 4 h 25
h  h1
1 3x 3 3 2  Required percentage =  100
= × 3x × = x h
2 2 4
Required shaded portion 25  16
= × 100 = 36%
25
3 3 2 3
= x  2 3x  = 16. (3) Let teh length of the rectangle be x metre
4 8 and breatdh be y metre.
13. (1)  xy - (x - 5)(y + 3) = 9
 xy - (xy - 5y + 3x - 15) = 9
 5y - 3x + 15 = 9
 3x - 5y - 6 = 0 ....(i)
LEARN MATHS FROM S.K. RAJU (9811549822, 9811649822)
18
Again, (x + 3) (y + 2) - xy = 67 BC = BX + XC = 3x + 2x = 5x cm
3y + 2x + 6 = 67 CD = CY + YD = 2y + y = 3y cm
 2x + 3y - 61 = 0 ....(ii)  5x × 3y = 120
By equation (i) × 3 + (ii) × 5  xy = 8 (= 2 × 4)
9x - I5y - 18 = 0 BC = 20 cm
10x + 15y -305 = 0 CD = 6 cm
19x = 323 3
 x = 17 metre BX = × 20 = 12 cm
5
17. (1) Volume of the cylindrical jar
1
= r h
2
YD = × 6 = 2 cm
New volume on reducing height 3

64 16 1
= r 2 × h=  r 2h ZD = × 20 = 5 cm
100 25 4
 Area of the shaded region
25 2
 If r1 =
2
R the volume will not change. = 120 -  ABX -  ZYD
16
1 1
5 1 = 120 - × 12 × 6 - ×2×5
 r1 = R = 1 + i.e. increase of 25% 2 2
4 4 = 120 - 36 - 5 = 79 sq.cm.
18. (3) Let the length, breadth and height of 21. (4)
the cuboid be a, b and c cm respectively.
2 (ab + bc + ca) = 22
and, 4 (a + b + c) = 24
a+b+c=6
 (a + b + c)2 = a2 + b2 + c2 + 2ab + 2ac + 2bc
 36 = a2 + b2 + c2 + 22 Distance covered in first meeting =  r
 a2 + b2 + c2 =14 Distance covered in second meeting = 3  r
BD = 100 metre
 a 2  b 2  c 2 = 14 AD = 60 metre
= Diagonal of cuboid If 2  r = 480 metre then,
19. (2) 3x = 6y where x = side of triangle; A’s speed : B’s speed = 7 : 5
y = side of regular hexagon. 22. (1) Side of the square = x units and side of
 x = 2y regular hexagon = y units.

x 3
 y=  4x = 6y  x = 2 y
2
9 2
3 2 Area of square = y
 Area of triangle = x 4
4
3 3
3 2 Area of hexagon = × y2
Area of hexagon = 6 × y 2
4
 Required ratio
3 x2 3 3 2
= 6 × × = x 3 3 9
4 4 8 = × y2 : y2 = 2 3 : 3
2 4
 Required ratio
23. (4) Volume of water in the pool
3 2 3 3 2
= x : x = 2 :3  1.2  2.4 
4 8 = 40 × 15 ×  
 2 
20. (4)
= 1080 cubic metre
24. (1) Time taken by A in completing one
revolution

LEARN MATHS FROM S.K. RAJU (9811549822, 9811649822)


19
750  750  18  29. (3) Surface area of the sphere = 4 r 2
= 6.75  5 sec =   sec = 4 × 102 = 400  sq. cm.
 6.75  5 
18 4
3 r
Volume o sphere = 3

 750  18 
Time taken by B =   sec
 4.75  5  4 4000
=  × 103 = cu. cm.
LCM of numerators 3 3
Required time =
HCF of denominators 400
 Required percentage = 4000  100
750  1800
= = 1800 × 6 seconds 3
125
= 30%
1800  6 30. (3) Perimeter of rectangle = 2(18 + 26)
= = 3 hours
3600 = 2 × 44 = 88 cm
25. (1) Volume of the silver used in the ball  Circumference of circle = 88 cm

 r2  r1 
4 22
2×
3 3
= × r = 88
3 7
88  7
 2  1 
4 3 3
=  r = 2  22 =14 cm
3
28  Area of circle =  r 2
= cu.cm
3 22
26. (4) Let the sides of the cuboid be a, b and c. = × 14 × 14 = 616 sq. cm.
7
 ab = 12 sq. cm.
31. (3) The number of tiles will be minimum if
bc = 20 sq.cm.
size of each marble is maximum.
ac = 15 sq.cm.
Size of each tile = HCF of 3.78 and 5.25
Volume of cuboid = abc
metre = 0.21 metre
= a 2b 2c 2 = 12  20  15
= 3600 = 60 cu.cm.
27. (2) Distance covered in 1 revolution
=  × diameter =  d
 1000 ×  d = 440
22
 1000 × × d = 440
7
440  7
 d = 1000  22 = 0.14 metre.

28. (3) Side of square field = 484 = 22 metre 3.78  5.25


 Number of tiles = 0.21  0.21 = 450
Circumference of circular field =
Perimeter of square field
 2 r = 4 × 22
22
2 × × r = 4 × 22
7
 r = 14 metre
 Diameter = 28 metre

LEARN MATHS FROM S.K. RAJU (9811549822, 9811649822)


20
MODEL EXERCISES
1. PQRS is a diameter of a circle whose radius immersed. The water level in the vessel will
is r. The lengths of PQ, QR and RS are equal. rise by —
Semicircles are drawn on PQ and QS to 9 9
create the shaded figure below (1) cm (2) cm
2 4
The perimeter of the shaded figure is —
4 2
(3) cm (4) cm
9 9
(5) None of these
7. A cyli nder is circumscribed about a
hemisphere and a cone is inscribed in the
cylinder so as to have its vertex at the centre
of one end and other end as its base. The
4 r volume of the cylinder, hemisphere and
(1) 2 r (2)
3 the cone are respectively in the ratio :
3 r (1) 2 : 3 : 2 (2) 3 : 2 : 1
5 r
(3) (4) (3) 3 : 1 : 2 (4) 1 : 2 : 3
3 2
(5) None of these
(5) None of these 8. A conical cavity is drilled in a circular
2. The Length of an edge of a hollow cube open cylinder of height 15 cm and base radius 8
at one face is 3 m. What is the length of cm. The height and the base radius of the
the largest pole that it can accommodate ? cone are also same. Then, the whole surface
of the remaining solid is —
(1) 3 m (2) 3 3 m
(1) 440  cm2 (2) 240  cm2
3 (3) 640  cm 2
(4) 960  cm2
(3) 3 m (4) m (5) None of these
3
9. A conical flask has base radius ‘a’ cm and
(5) None of these
height ‘h’ cm. It is completely filled with
3. A tank 30 m long 20 m wide and 12 m deep
milk. The milk is poured into a cylindrical
is dug in a field 500 m long and 30 m wide.
thermos flask whose base radius is ‘p’ cm.
By how much will the level of the field rise
What will be the height of the solution level
if the earth dug out of the tank is evenly
in the flask ?
spread over the field ?
(1) 0.33 m (2) 0.5 m a 2h 3hp 2
(3) 0.25 m (4) 0.4 m (1) cm (2) cm
3p2 a2
(5) None of these
4. A water tank in the form of a cuboid has its p2 3a 2
base 20 m long, 7 m wide and 10 m deep. (3)
2 cm (4) cm
3h hp 2
Initially, the tank is full but later when
(5) None of these
water is taken out of it, the level of water in
10. Th e he ig ht of a roo m is 40% of i ts
the tank reduces by 2 m. The volume of the
semiperimeter. It cost Rs 260 to paper the
water left in the tank is —
walls of the room with paper 50 cm wide at
(1) 1120 m3 (2) 400 m3
the rate of Rs 2 per m allowing an area of
(3) 280 m3 (4) 140 m3
15 m2 for doors and windows. The height of
(5) None of these
the room is —
5. The floor of a rectangular room is 15 m
(1) 2.6 m (2) 3.9 m
long and l2 m wide.The Room is surrounded
(3) 4 m (4) 4.2 m
by a verandah of width 2 m on its sides.
(5) None of these
The area of the verandah is —
11. The area of a figure formed by a square of
(1) 124 m2 (2) 120 m2
side 8 cm and an isosceles triangle with
(3) 108 m2 (4) 58 m2
base as on e side of the square and
(5) None of these
perimeter as 18 cm is —
6. A cylindrical vessel of radius 4 cm contains
(1) 80 cm2 (2) 84 cm2
water. A solid sphere of radius 3 cm is 2
(3) 104 cm (4) 76 cm2
lowered into the water until it is completely
(5) None of these
LEARN MATHS FROM S.K. RAJU (9811549822, 9811649822)
21
12. A toy is in the form of a cone mounted on a 100% , 200% and 200% respectively. Then,
hemisphere of radius 3.5 cm. The total the increase in the volume of the cuboid
height of the toy is 15.5 cm. Find the total will be —
(1) 5 times (2) 6 times
 22 
surface area.  use   (3) 12 times (4) 17 times
 7  (5) None of these
(1) 137.5 cm2 (2) 214.5 cm2 19. A sphere of radius 3 cm is dropped into a
2
(3) 154 cm (4) 291.5 cm2 cylindrical vessel partly filled with water.
(5) None of these The radius of the vessel is 6 cm. If the
13. A solid cylinder and a solid cone have equal sphere is submerged completely then the
base and equal height. If the radius and surface of the water is raised by-
the height be in the ratio of 4 : 3, the ratio
1 1
of the total surface area of the cylinder to (1) cm (2) cm
that of the cone is in the ratio of — 4 2
(1) 10 : 9 (2) 11 : 9 (3) 1 cm (4) 2 cm
(3) 12 : 9 (4) 14 : 9 (5) None of these
(5) None of these 20. If the length of the diagonal of a square and
14. The volume of a cube is numerically equal that of the side of another square are both
to the sum of its edges. What is its total 10 cm, the ratio of the area of the first
surface area in square units ? square to that of the second is —
(1) 66 (2) 183 (1) 1 : 2 (2) 1 : 3
(3) 36 (4) 72 (3) 1 : 4 (4) 2 : 3
(5) None of these (5) None of these
15. An ice-cream company makes a popular 21. If the circumference and the area of a circle
brand of ice-cream in rectangular shaped are numerically equal, then what is the
bar 6 cm long, 5 cm wide and 2 cm thick. numerical value of the diameter ?
To cut costs, the company has decided to (1) 1 (2) 2
reduce the volume of the bar by 20% the (3) 4 (4) 
thickness will remain the same, but the (5) None of these
length and wide will be decreased by the 22. A circular park of 20 m diameter has a
same percentage amount. The new length circular path just inside the boundary of
L will satisfy. — width 1 m. The area of the path is —
(1) 5.5 < L < 6 (2) 5 < L < 5.5 (1) 15  (2) 17 
(3) 4.5 < L < 5 (4) 4 < L < 4.5 (3) 19  (4) 25 
(5) None of these (5) None of these
16. From a square piece of paper having each 23. An edge of a cube measures 10 cm. If the
side equal to 10 cm, the largest possible largest possible cone is cut out of this cube,
circle is cut out. The ratio of the area of the then the volume of the cone is —
circle to the area of the original square is (1) 260 cm3 (2) 260.9 cm3
3
closest to — (3) 261.9 cm (4) 262.7 cm3
(5) None of these
4 3
(1) (2) 24. A square and an equilateral triangle have
5 5 the same perimeter. If the diagonal of the
5 6 square is 12 2 cm, then the area of the
(3) (4)
6 7 triangle is —
(5) None of these
(1) 24 3 cm2 (2) 24 2 cm2
17. The perimeter of a rectangular field is 480
m and the ratio between the length and the (3) 64 3 cm2 (4) 32 3 cm2
breadth is 5 : 3. The area is — (5) None of these
(1) 15500 m2 (2) 13500 m2 25. What is the area of the inner equilateral
2
(3) 1550 m (4) 1350 m2 triangle, if the side of the outermost square
(5) None of these is ‘a’ ?
18. The length breadth and height of a cuboid (ABCD is a square) —
are in the ratio 1 : 2 : 3. The length, breadth
and height of the cuboid are increased by
LEARN MATHS FROM S.K. RAJU (9811549822, 9811649822)
22
5 3a 2
(3) (4) None of these
32

5 3a 3
(5)
33

3 3a 2 3 3a 2
(1) (2)
32 64

LEARN MATHS FROM S.K. RAJU (9811549822, 9811649822)


23
SHORT ANSWERS 7200
1. (1) 2. (3) h= = 0.5 m
14400
3. (2) 4. (1)
4. (1) Given that,
5. (1) 6. (2)
Volume of the tank = l × b × h
7. (2) 8. (1)
= 20 × 7 × (10 - 2) = 1120 cm3
9. (1) 10. (3)
5. (1) Given that
11. (4) 12. (2)
Area of outer rectangle = 19 × 16 = 304 m2
13. (4) 14. (4)
15. (2) 16. (1)
17. (2) 18. (4)
19. (3) 20. (1)
21. (3) 22. (3)
23. (3) 24. (3) Area of inner rectangle = 15 × 12 = 180 m2
25. (4)  Required area = (304 - 180) = 124 m3
6. (2) According to question, volume of the
EXPLANATIONS sphere = Volume of the water displaced
1. (1) According to the question, Let the required height to which the water
Diameter PS = 2r rises be h.

2r Then,  r12h
 PQ = QR = RS =
3 4
 16 h = × 27
3
39 9
 h= = cm
16 4
7. (2) According to the information given in
Perimeter of shaded portion the question and the figure it is clear that
Radius of the hemisphere = radius of cone
1 r 1 2r  1
=  2 r  3  +  2 r  3  +  2 r  = height of cone = height of cylinder.
2  2  2 let it be r, then ratio of volume of cylinder,
 r 2 r hemisphere and cone
= + + r
3 3
 r  2 r  3 r
=
3
6 r
= = 2 r
3 2 1
= r 3 :  3 r h
3 : 2
2. (3) According to the question. 3 r
Largest pole can be accommodated
2 1
along the diagonal of cube. =1: : =3:2:1
3 3
 Diagonal of cube = 3 × side 8. (1) According to question,
= 3× 3 = 3m l= 152  82 = 17 cm
3. (2) Given that,
Volume of the earth taken out
= 30 × 20 × 12 = 7200 m3
Area of the remain ing portion
(leaving the area of dug out portion)
= 470 × 30 + 30 × 10
= 14100 + 300 = 14400m2 Total surface area of the remaining solid
Let hbe the height to which the field = 2 rl +  r 2 +  rl
is raised by spreading the earth = 2  × 8 × 15 +  (8)2 +  × 8 × 17
dugout Then, 14400 × h = 7200 = 240  + 64  + 136 
= 440  cm2

LEARN MATHS FROM S.K. RAJU (9811549822, 9811649822)


24
9. (1) According to question, Total surface area of the figure = Total
Vol ume of conical fl ask = volume of surface area of cone + Total surface area of
cylindrical flask up to the required height hemisphere
(x)
1 2 ha 2
a h =  p 2 × x = x =
3 3p2
10. (3) Let the length, breadth and height of
the room be l, b and h respectively.
Then, h = 0.4 ( l + b)
Area of the four walls = 2 (l + b) h
= 2 (l + b) × 0.4 (l + b) = 0.8 (l + b)2 =  rl + 2  r
Required area where paper has to be pasted
=  r × r 2  h 2 + 2  r2
4
= (l + b) 2 - 15 22 22
5
 3.5   12  + 2 × 7 × (3.5)
2
= × 3.5 2 2
Now, area of paper = area of wall Length × 7
4 44
breadth = (l + b) 2 - 15 = 11 12.25  144 + × 12.25
5 7
= 11 × 12.5 + 44 × 1.75
4
l  b 2  15 = 137.5 + 77 = 214.5 cm2
5 13. (4) Let the radius be 4x and height be 3x,
Length = 1 then required ratio
2
2 rh  2 r 2 2 h  r 
4  = =
 rl   r r2  h2 r
Given, 2  l  b   15  × 2 = 260
2 2

5  (where l is he slant height and


16 l= h2  r2
 (l + b)2 - 60 = 260
5
2  7x 
16 =
 (l + b)2 = 320 16x  9x 2  4x
2

5
 (l + b) 2 = 100 14x 14x
= = = 14 : 9
 (l + b) 2 = 10 5 x  4x 9x
 h = 0.4 × 10 = 4 m 14. (4) Let x be the side, (edge of the cube)
11. (4) By question, Given, a3 = 12a
AE + AB + EB = 18 cm  a2 = 12
AE + EB= 10 cm Total surface area = 6a2
AE = EB = 5 cm = 6 × 12 = 72
15. (2) According to question, volume of ice-
EF = 5    4  = 3 cm
2 2
cream = 6 × 5 × 2 = 60 cm2
Also, L × b × 2 = 48
 L × b = 24
Now, L = 6 - 6 × 10% = 5.4
b = 5 - 5 × 10% = 4.5
L × b = 5.4 × 4.5 = 24.3
Clearly, 5 < L < 5.5
Now, area of the required figure = area of 16. (1) By question, area of square =
square + area of isosceles triangle (10)2 = 100cm2
1 22 22  25
=8 × 8 + ×8 × 3 Area of circle = × (5)2 =
2 7 7
= 64 + 12 = 76 cm2
12. (2) According to the question,

LEARN MATHS FROM S.K. RAJU (9811549822, 9811649822)


25

22  25 22 11 =  (102 - 92) = 19 
 Required ratio = = =
7  100 28 14 23. (3) According to the question,
Height of the cone = 10 cm
4
= 0.785 which is closed to Radius = 5 cm
5
 Required volume
17. (2) Let the length and breadth be 5x and 3x
respectively, 1
= ×  × r2 × h
then 2 (length + breadth) = 480 3
 2 (5x + 3x) = 480 1 22
 8x = 240 =
3
×
7
× 25 × 10
 x = 30m = 261.9 cm3
 Sides are 150 m and 90 m 24. (3) Let the side of the square be x.
Hence, area = 150 × 90 = 13500 m2
 
2
18. (4) Let the length, breadh and height of the Then, 2x2 = 12 2
cube be x, 2x and 3x respectively.
Then, volume = x × 2x × 3x = 6x2  x = 12
New length, breadth and height Now, perimeter of equilateral triangle
= 2x, 6x and 9x respectively. = 12 × 4 = 48 cm
Now volume = 108x3 Side of equilateral triangle
Hence, increase in volume 48
= = 16 cm
 108  6  3
=   x3 = 17 times of original volume. Area of equilateral triangle
 6 
19. (3) Let the rise in the level of water be h cm. 3
= × (16)2 = 64 73cm2
Then, volume of the sphere = volume of the 4
water displaced 25. (4) According to the question,
4 BD || EF and BD = 2EF
   3  =  (6)2 × h
2

3
4
 27
3
 h = 36 = 1 cm
20. (1) Let the side of the 1st suare be x cm.
Then, for first square x2 + x2 = 100 1
x2 = 50 cm2  EF = 2 × a
Hence, area of 1st square is 50 cm2.  Area of equilateral triangle
Area of Ilnd square =100 cm2. 2
 Required ratio = 50 : 100 = 1 : 2 3 1a 
=  
21. (3) Let the radius of the circle be r cm. 4 2 
Then, 2  r =  r 2
 r = 2 cm 3 1 2 3 2
= × a = a
Diameter = 2 × 2 = 4 cm 4 4 16
22. (3) According to the question,
Area of circular path

LEARN MATHS FROM S.K. RAJU (9811549822, 9811649822)


1
MISCELLANEOUS
NATIONALISED BANKS (1) 48 (2) 36
& IBPS SO/MT/SO (3) 24 (4) 52

a  b   a  b 
2 2
x 6.
1. if = y
4a A B C
On si mplifyin g the above mentioned (1) 4 (2) 5
equation, what will be the equation ? (3) 5.5 (4) 6.5
(1) xy = b (2) bx = y (5) 9
(3) by = x (4) ab = x 7. Rachita enters a shop to buy icecreams,
(5) ay = x cookies and pastries. She has to buy atleast
(Corporation Bank PO 9 units of each. She buys more cookies than
Exam. 22.11.2009) ice-creams and more pastries than cookies.
2. Angle ‘A’ of the quadrilateral ABCD is 26° She picks up a total of 32 items. How many
less than angle B. Angle B is twice angle C cookies does she buy ?
and angle C is 10° more than the angle D. (1) Either 12 or 13
What would be the measure of angle A ? (2) Either 11 or 12
(1) 104° (2) 126° (3) Either 10 or 11
(3) 56° (4) 132° (4) Either 9 or 11
(5) 106° (5) Either 9 or 10
(Corporation Bank PO (IBPS Bank PO/MT CWE 17.06.2012)
Exam. 22.11.2009) 8. The fare of a bus is Rs x for the first five
3. In every 30 minutes the time of a watch ki lo me tres an d Rs13 per ki lo me tres
increases by 3 minutes. After setting the thereafter. If a passenger pays Rs 2,402 for
correct time at 5 a.m., what time will the a journey of 187 kilometres, what is the
watch show after 6 hours ? value of X ?
(1) 10 : 54 a.m. (2) 11 : 30 a.m. (1) Rs 29 (2) Rs 39
(3) 11: 36 a.m. (4) 11 : 42 a.m. (3) Rs 36 (4) Rs 31
(5) 11: 38 p.m. (5) None of these
(Corporation Bank PO (IBPS Bank PO/MT CWE 17.06.2012)
Exam. 22.11.2009) 9. Varun got a monthly increment of 12 pecent
Directions (4-6) : In the following questions of Puja’s monthly salary. Puja’s monthly
three squares A, B and C are given which contain salary is Rs 7800. Varun’s monthly salary
four figures/ numbers related to each other. The before increment was Rs 6400. What
figures/ numbers in the first two squares are given amount will he earn in four months after
as examples and bear a certain relationship to each his increment ?
other within the square. This relationship can be (1) Rs 29344 (2) Rs 29434
established/calculated vertically, horizontally or (3) Rs 28434 (4) Rs 28344
diagonally. Depending on this relationship between (5) None of these
the figures/numbers in the first two squares, find (IDBI Bank Officer Exam.16.09 2012)
out what should come in place of the question mark 10. In a schoo l some sweats were to be
in the third square - C. distributed among 420 children on the
(United Bank Of India occasion of Teacher’s day. But 140 children
PO Exam. 14.11.2010) remained absent on that particular day and
hence each child got one sweat extra. How
4. many sweats each child would have got
A B C originally ?
(1) 27 (2) -13 (1) Cannot be determined
(3) 6 (4) 13 (2) 2
(5) -27 (3) 5
(4) 4
5. (5) 1
A B C (IBPS Specialist Officer
CWE 17.03.2013)

LEARN MATHS FROM S.K. RAJU (9811549822, 9811649822)


2
11. The cost of 14 keyboards and 8 mouse-pads 2. The top of a 15 metre high tower makes an
is Rs. 26,240. What is the cost of 35 angle of elevation of 60° with the bottom of
keyboards and 20 mouse-pads ? an electric pole and an angle of elevation of
(1) Rs. 65,600 30° with the top of the pole. What is the
(2) Rs. 58,800 height of the electric pole ?
(3) Rs. 76,550 (1) 5 metres (2) 8 metres
(4) Cannot be determined (3) 10 metres (4) 12 metres
(5) None of these (5) None of these
(IBPS RRBs Office Assistant (SBI Associate Banks PO
CWE Exam. 09.09.2012) Exam. 14.02.1999)
12. A boat covers 20 km in 4 hours along the 3. Which of the following values of P satisfy
current and 9 km in 3 hours against the the inequality P (P - 3) < 4P - 12 ?
current. What is the speed of the current ? (1) P > 14 or P < 13
(1) 2 kmph (2) 1 kmph (2) 24  P< 71
(3) 1.5 kmph (4) 1.75 kmph (3) P > 13; P < 51
(5) None of these (4) 23 < P < 41
(Indian Overseas Bank PO (5) P = 4, P = +3
Online Exam. 01.09.2013) (SBI Associate Banks PO
13. A pump can fill a tank with water in 2 hours. Exam. 14.02.1999)
1 4. Four of the following five parts numbered
Because of a leak, it took 2 hours to fill (1), (2), (3), (4) and (5) in the following
3
equation are exactly equal. Which of the
the tank, The leak can drain all the water
parts is not equal to the other four ? The
of the tank in
number of that part is the answer.
1 (1) xy2 - x2y + 2x2y2 = xy2 (1 - 2x) + x2y
(1) 4 hours (2) 7 hours
3 (2) (2x - 1) = xy2
(3) 8 hours (4) 14 hours (3) [y (1 + x) - (1 - y)] = xy [(x + y)2 +y
(5) None of these (4) (1 + x) - x2y (1- y) = xy
(IBPS Bank PO/MT CWE-III (5) (1 - y) - x(l + x)
26.10.2013) 5. Four of the following five parts numbered
14. Pipe A can fill a tank In 30 minutes while (1), (2), (3), (4) and (5) are equal. Number of
pipe B can fill it in 45 minutes. An other the part which is not equal to the other four
pipe C can empty a full tank in 60 minutes. parts is the answer.
If all three pi pe s are open ed (1) 136 × 12  17 + 4 =
simultaneously, the empty tank will be (2) 152 × 12  19 + 48  12 =
filled in (3) 268  67 × 42 + 62 =
(4) 234  13 × 4 + 84  3 =
2 3
(1) hour (2) hour (5) 378  42 × 9 + 32 + 4
7 7 (SBI Associate Banks PO
4 5 Exam. 16.07.2000)
(3) hour (4) hour 6. In the following equation of equations four
7 7
(5) None of these out of the five parts numbered (1), (2), (3),
(Corporation Bank Specialist (4) and (5) are exactly equal. Which one part
Officer (Marketing) Exam. 22.12.2014) is not equal to the other four? The number
of that part is the answer.
(1) 120 × 12 - 22 × 20 =
SBI PO EXAMS
1. If 2x + y = 15, 2y + z = 25 and 2z + x = 26, 2
(2) 10% of 500 + of 1200 =
what is the value of z ? 5
(1) 4 (2) 7 (3) 80 × 40 - 20 × 110 =
(3) 9 (4) 12 (4) 8640  60 + 53.5 × 16 =
(5) None of these (5) 5314 - 3029- 1285 =
(SBI Associate Banks PO (SBI Banks PO Exam. 20.08.2000)
Exam. 14.02.1999) 7. In the following equation four out of the five
parts numbered (1), (2), (3), (4) and (5) are

LEARN MATHS FROM S.K. RAJU (9811549822, 9811649822)


3
exactly equal. Which part is not equal to (2)
6 kmph
the other four? The number of that part is (3)
8 kmph
the answer. (4)
Data inadequate
(1) 53 + 33 + 48 = (5)
None of these
(2) 52 × 33 - 475 = (SBI Associate Banks PO
(3) 35 - 44 = Exam. 21.07.2002)
(4) 43 + 2 × 17 × 4 = 13. If 2x + 3y + z = 55; x + z - y = 4 and y - x + z
(5) (6)3 - (2)4 = = 12 then what is the value of y ?
(SBI Banks PO Exam. 20.08.2000) (1) 7 (2) 8
8. If 3x + 7 = x2 + M = 7x + 5, (3) 12 (4) 9
what is the value of M ? (5) None of these
(1) 2 (SBI Associate Banks PO
1 Exam. 21.07.2002)
(2) 8 14. If 3y + 2x = 47 and 11x = 7y then what is
4
value of y - x ?
1 (1) 4 (2) 6
(3) 8 (3) 7 (4) 5
2
(4) Cannot be determined (5) None of these
(5) None of these (SBI Associate Banks PO
(SBI Banks PO Exam. 20.08.2000) Exam. 21.07.2002)
15. Four of the following five parts numbered
x  y 13 (1), (2), (3), (4) and (5) are exactly equal.
9. If 2x + 3y = 34 and y = , then 5y +
8 The number of the part which is not equal
7x = ? to the other four is the answer
(1) 82 (2) 175 (1) 136 × 12  9 × 3 =
(3) 178 (4) 185 (2) 17 × 64  6 × 3 =
(5) None of these (3) 36 × 17  9 × 8 =
(SBI Banks PO Exam. 11.02.2001) (4) 56 × 8  14 × 17 =
10. Taps A, B and C are connected to a water (5) 76 × 6  19 × 17
tank and the rate of flow of water is 42 SBI Associate Banks PO
litres/hr., 56 litres/ hr. and 48 litres/hr. Exam. 21.07.2002
respectively. Taps A and B fill the tank while 16. For which of the following values of x the
tap C empties the tank. If all the three taps inequality
are opened simultaneously, the tank gets x (x + 3) < 10 is satisfied ?
completely filled up in 16 hours. What is (1) x > 2, x < - 5
the capacity of the tank ? (2) - 5 < x < 2
(1) 960 litres (2) 2346 litres (3) - 2 < x < 5
(3) 1600 litres (4) 800 litres (4) x < - 2, x > 5
(5) None of these (5) None of these
(SBI Banks PO Exam. 11.02.2001) (SBI Associate Banks PO
11. Four of the following five parts numbered Exam. 21.07.2002)
(1), (2), (3), (4) and (5) are equal to other 17. A boat takes 6 hours to travel from place M
four parts is the answer. to N downstream and back from N to M
2
(1) (2x + 3y) = upstream. If the speed of the boat in still
2
(2) (2y - x) + y(16x + 5y) = water is 4 km./hr., what is the distance
(3) 4x (x + 2y) + y (4x + 9y) = between the two places ?
(4) (2x + 2y)2 + y (4x + 5y) = (1) 8 kms.
2
(5) (2x - y) + 8y(2x + y) (2) 12 kms.
(SBI Banks PO Exam. 11.02.2001) (3) 6 kms.
12. A boat running down stream covers a (4) Data inadequate
distance of 16 km in 2 hours while for (5) None of these
covering the same distance upstream it (SBI PO Exam. 28.11.2006)
takes 4 hours. What is the speed of the boat 18. The speed of a boat wh en trave lling
in still water ? downstream is 32 kmph whereas when
(1) 4 kmph travelling upstream it is 28 kmph. What is
the speed of the boat in still water ?
LEARN MATHS FROM S.K. RAJU (9811549822, 9811649822)
4
(1) 27 Kmph (1) 7 (2) 8
(2) 29 Kmph (3) 12 (4) 9
(3) 31 Kmph (5) None of these
(4) Cannot be determined (LIC Assistant Administrative
(5) None of these Officer (AAO) Exam. 2006)
(SBI & Rural Business PO 7. If 3y + 2x = 47 and 11x = 7y then what is
Exam. 18.04.2010) value of y - x ?
(1) 4 (2) 6
INSURANCE EXAMS (3) 7 (4) 5
1. A boat running down stream covers a (5) None of these
distance of 16 km in 2 hours while for (LIC Assistant Administrative
covering the same distance upstream it Officer (AAO) Exam. 2006)
takes 4 hours. What is the speed of the boat 8. Two straight lines can divide a circular disk
in still water ? into a maximum of 4 parts. Likewise into
(1) 4 kmph (2) 6 kmph how many parts can four straight lines
(3) 8 kmph (4) Data inadequate divide a circular disk ?
(5) None of these (1) 8 (2) 9
(LIC Assistant Administrative (3) 10 (4) 11
Officer (AAO) Exam. 2006) (General Insurance Corporation
2. River is running at 2 kmph. It took a man AAO Exam. 11.12.2011)
twice as long to row up as to row down the 9. A person can row a boat d km upstream
river. The rate (in km ph) of the man in still 1
water is : and the same distance downstream 5
4
(1) 8 (2) 10
hours. Also he can row the boat 2d km
(3) 4 (4) 6
upstream in 7 hours. He will row the same
(United India Insurance Co.
distance downstream in
(AAO) Exam. 11.03.2007)
3. Three pipes A, B and C can fill a cistern in 1 1
(1) 3 hours (2) 3 hours
6 hours. After working at it together for 2 2 4
hours, C is closed and A and B can fill the
1
remaining part in 7 hours. The number of (3) 1 hours (4) 4 hours
hours taken by C alone to fill the cistern is : 4
(1) 12 (2) 14 (Oriental Insurance Company
(3) 16 (4) 18 AAO Exam. 08.04.2012)
(United India Insurance Co. 10. The rate at which a river flows is one-third
(AAO) Exam. 11.03.2007) the speed of a boat in still water. If that
4. The angles of a convex hexagon in degrees boat travels down the river for 2 hours and
are integers and in arithmetic progression. then back up river for 2 hours, it will be 16
L and M denote the largest of these 6 angles. km short of its starting point. The speed
Then the maximum value that M can take (km/hour) of the boat in still water is
is : (1) 4 (2) 6
(1) 125 degree (2) 150 degree (3) 8 (4) 12
(3) 175 degree (4) 179 degree (United India Insurance AAO
(General Insurance Corporation Exam. 03.06.2012)
AAO Exam. 11.12.2011) b c 2z
5. The inequality 2x2 + 9x + 4 < 0 is satisfied 11. If ax = b y = cz and = then =?
a b x z
for which of the the following values of’x’?
y x
1 1 (1) (2)
(1) -4 < x - (2) < x < 4 x y
2 2
(3) 1 < x < 2 (4) -2 < x < 1 x z
(5) None of these (3) (4)
z x
(LIC Assistant Administrative (United India Insurance AAO
Officer (AAO) Exam. 24.04.2005) Exam. 03.06.2012)
6. If 2x + 3y + z = 55; x + z - y = 4and y - x + z
= 12 then what is the value of y ?
LEARN MATHS FROM S.K. RAJU (9811549822, 9811649822)
5
12. If f(x) = 2x 3 + 3x 2 + 5 then f(2) = ? (3) -2 (5) -3
(1) 31 (2) 32 (NICL (GIC) AO (Finance)
(3) 33 (4) 35 Exam. 18.12.2013)
(United India Insurance AAO 17. Each inch on ruler A is marked in equal
Exam. 03.06.2012) 1
13. For what value(s) of k, the roots of the - inch units, and each inch on ruler B is
8
equation 9x 2 + 2Kx + 4 = 0 will be equal ?
(1) 6 (2) -6 1
(3) ±6 (4) ±5 marked in equal - inch units. When ruler
12
(United India Insurance AAO A is used, a side of triangle measures 12 of
Exam, 03.06.2012)
14. A 10 metre, long ladder is placed against a 1
the - inch units. When ruler B is used,
wall. It is inclined at an angle of 30° to the 8
ground. The distance of the foot of the 1
ladder from the wall is how many - inch units will the same side
12
(1) 7.32 m (2) 8.26 m
(3) 8.66 m (4) 8.16 m measure ?
(NICL (GIC) Administrative (1) 8 (2) 12
Officer Exam. 15.12.2013) (3) 18 (4) 20
15. How many different triangles are there for (NICL (GIC) AO (Finance)
which the lengths of the sides are 3, 8 and Exam. 15.12.2013)
n, where n is an integer and 3 < n < 8 ? 18. Some students planned a picnic. The
(1) Two (2) Three budget for food was Rs 500. But, 5 of them
(3) Four (4) Five failed to go and thus the cost of food for
(NICL (GIC) AO (Finance) each member increased by Rs 5. How many
Exam. 15.12.2013) students attended the picnic ?
16. The value of k for which x - 1 is a factor of (1) 15 (2) 20
4x 3 + 3x2 - 4x + k, is (3) 25 (4) 30
(1) 3 (2) 1 (NICL (GIC) AO (Finance)
Exam. 15.12.2013)

LEARN MATHS FROM S.K. RAJU (9811549822, 9811649822)


6
SHORT ANSWERS 318
NATIONALISED BANKS x= = 106°
3
& IBPS PO/MT/SO
3. (3) Time gained in 6 hours
l. (3) 2. (5)
= 12 × 3 = 36 minutes
3. (3) 4. (5)
 Required time =11 : 36 a.m.
5. (1) 6. (2)
4. (5) Square A  92 - 32 = 60;
7. (3) 8. (3)
9. (1) 10. (2) 72 - 52 = 20
11. (1) 12. (2) Square B  76.5 - 16.5 = 60 ;
13. (4) 14. (2) 56.5 - 36.5 = 20
Square C  ? = 33 - 60 = - 27
SBI PO EXAMS 5. (1) Square A  9 × 5 = 45;
1. (5) 2. (3) 180 × 3 = 540
3. (5) 4. (2) Square B  5 × 5 = 25;
5. (5) 6. (2) 100 × 3 = 300
7. (3) 8. (2) Square C  2.4 × 5 = 12
9. (5) 10. (4)
144
11. (2) 12. (2) ?= = 48
13. (5) 14. (1) 3
15. (5) 16. (5) 6. (2) Square A  6 × 8 = 48
12 × 2 = 24
INSURANCE EXAMS Square B  4 × 8 = 32;
1. (2) 2. (4) 8 × 2 = 16
3. (2) 4. (4) Square C  2.5 × 8 = 20
5. (1) 6. (5) 10
7. (1) 8. (1) ? = = 5
2
9. (1) 10. (4)
11. (1) 12. (3) 7. (3) Look :
13. (3) 14. (3) Ice-creams + Cookies + Pastries = 32
15. (1) 16. (4)  9 + 11 + 12 = 32
17. (1) 18. (2) or, 9 + 10 + 13 = 32
10 + 11 + 12  32
EXPLANATIONS 8. (3) According to the question,
NATIONALISED BANKS x + 182 × 13 = 2402
& IBPS PO/MT/SO  x + 2366 = 2402
 x = 2402 - 2366
a  b   a  b 
2 2
x
1. (3) = y = Rs. 36
4a 9. (1) Monthly increase in salary of Varun
4ab x x 7800  12
 = y  y = b  x = by = = Rs. 936
4a 100
2. (5) Let  A = x o  New monthly salary of Varun
= 6400 + 936 = Rs. 7336
  B = x + 26  Required income = 4 × 7336
x  26 x = Rs. 29344
C = 2
=
2
+ 13 10. (2) If the number of all sweats be x, then

x x x
- =1
D = 2
+ 3 280 420

x x 3x  2x
 x + x + 26 + + 13 + + 3 = 360°  =1
2 2 840
 3x = 360 - 42 = 318°  x = 840
840
 Required answer = 420 = 2

LEARN MATHS FROM S.K. RAJU (9811549822, 9811649822)


7
11. (1) CP. of 1 key board = Rs. x. Now, according to question,
CP. of 1 mouse pad = Rs. y. AB = Height of Tower = 15 metres
 14x + 8y = 26240 CD = Height of Electric Pole
Dividing both sides by 2, = h metres
7x + 4y = 13120  ADE = 30°
Multiplying both sides by 5,  ACB = 60°
35x + 20y = 65600
 ACB = 60°
12. (2) Rate downstream AE = (15 - h) metres
20 Distance between B and C
= = 5 kmph = x metres
4
We know that
9
Rate upstream = = 3 kmph Perpendicular
3 Tan  =
 Speed of current Base
1 15  h
= (5 - 3) = 1 kmph  tan 30° =
2 x
13. (4) Part of tank emptied in 1 hour by the 1 15  h
leak or, =
3 x
1 3 1
= - = 15
2 7 14 or, 3= x
The leak will empty the tank in 14 hours.
14. (2) Part of tank filled by all three pipes in 1 x = 3 (15 - h) ....(i)
minute 15
Now, tan 60° =
1 1 1 643 7 x
= + - = =
30 45 60 180 180 15
180 x = 3
....(ii)
 Time taken = 7 minutes From equation (i) and (ii)
180 3 15
= = hours 3 (15 - h) =
7  60 7 3
15
SBI PO EXAMS or,15 - h =
3
1. (5) According to question,
or, h = 15 - 5 = 10 metres.
25  z 3. (5) P (P - 3) = 4 P - 12
y=
2 P2 - 3P = 4P - 12
x = 26 - 2z P2 - 7P + 12 = 0
25  z   7   49  4  12
 2(26 - 2z) + 2
=15 P =
2
25  z
 52 - 4z + = 15 7  1 7 1 7 1
2 = = , = 4, 3
2 2 2
 104 - 8z + 25 - z = 30 4. (2) Option (1)
 -9z = 30 - 129  xy2 - x2y + 2x2y2
 -9z = -99 Option (2)
 z = 11  xy2 (1 - 2x) + x2y (2y - 1)
2. (3)  xy2 - 2x 2y2 + 2x2y2 - x2y
= xy2 - x2y
Option (3)
xy2 (1 + x) - x2y (1 - y)
= xy2 + x2y2 - x2y + x2y2

LEARN MATHS FROM S.K. RAJU (9811549822, 9811649822)


8
= xy2 - x2y + 2x2y2 x  y 13
5. (5) (1) = 136 × 12  17 + 4 = 100 9. (5)
(2) = 152 × 12  19 + 48  12 y = 8
8x + 8y = 13y
152  12 48
= + = 100 8x - 5y = 0 ....(i)
19 12 Also, 2x + 3y = 34
(3) = 268  67 × 42 + 62 Multiplying this equation by 4 we can write,
268  16 8x + 12y = 136 ...(ii)
= + 36 = 100 Subtracting (i) from (ii) we get,
67
(4) = 234  13 × 4 + 84  3 17y = 102
y=6
234  4 84 Substituting this value in
= + = 100
13 3 2x + 3y = 34 we get,
(5) = 378  42 × 9 + 32 + 4 2x + (3 × 6) = 34
378  9 or, 2x = 16
= + 13 = 94 x = 8
42
 5y + 7x = 5 × 6 + 7 × 8 ,
6. (2) (1) 120 × 12 - 22 × 20 = 30 + 56
= 1440 - 440 = 1000 = 86
2 10. (4) Net amount of water filled in the tank in
(2) 10% of 5000 + of 1200 1 hour when all three taps are opened
5
simultaneously
10 2 = 42 + 56 - 48 litres
= × 5000 + × 1200
100 5 = 50 litres.
= 500 + 480 = 980 The tank gets completely filled in 16 hours.
(3) 80 × 40 - 20 × 110  Capacity of the tank
= 3200 - 2200 = 1000 = 16 × 50 = 800 litres.
(4) 8640  60 + 53.5 × 16 11. (2) (1) (2x + 3y)2 = 4x2 + 9y2 + 12xy
= 144 + 856 = 1000 (2) (2y - x)2 + y(16x + 5y)
(5) 5314 - 3029 - 1285 = 4y2 + x2 - 4xy + 16xy + 5y2
= 2285 - 1285 = 1000 = x2 + 9y2 + 12xy
7. (3) (1) 53 + 33 + 48 (3) 4x (x + 2y) + y (4x + 9y)
= 125 + 27 + 48 = 200 = 4x2 + 8xy + 4xy + 9y2
(2) 52 × 33 - 475 = 4x2 + 9y2 + 12xy
= 25 × 27 - 475 (4) (2x + 2y)2 + y (4x + 5y)
= 675 - 475 = 200 = 4x2 + 4y2 + 8xy + 4xy + 5y2
(3) 35 - 44 = 243 - 44 = 199 = 4x2 + 9y2 + 12xy
(4) 43 + 2 × 17 × 4 (5) (2x - y)2 + 8y (2x + y)
= 64 + 136 = 200 = 4x2 + y2 - 4xy + 16xy + 8y2
(5) (6)3 - (2)4 = 216 - 16 = 200 = 4x2 + 9y2 + 12xy
8. (2) 3x + 7 = x2 + M = 7x + 5 Thus, we see that except (2), other four
We can write . expressions are equal.
7x + 5 = 3x + 7 12. (2) Rate upstream
or, 4x = 2 16
1 = kmph = 8 kmph
2
or, x =
2
16
Now, 3x + 7 = x2 + M Rate downs tream =
or, M = 3x + 7 - x2
4
= 4 kmph
2
1 1  Rate In still water
= 3 × + 7 - 
2 2 1
= (8 + 4) kmph = 6 kmph
1 1 1 1 2
=1 +7- =1 +7=8 13. (5) Given,
2 4 4 4
2x + 3 + z = 55 ....(i)

LEARN MATHS FROM S.K. RAJU (9811549822, 9811649822)


9
x+z - y=4 ...(ii) or x = -3
y - x + z = 12 ...(iii)
From (ii) and (iii)
2z = 16  z = 8 The given inequality holds for
-3<x<0
From (ii), x + 8 - y = 4
17. (4) Total time = 6 hours.
x=4 -8 +y=y-4
Speed of the boat in still water = 4 km/hr.
Putting this value in (i) we get
Let the distance between M and N be D.
2 (y - 4) + 3y + 8 = 55
and the speed of the stream be x.
 2y - 8 + 3y + 8 = 55
 5y = 55  1 1 
D 
 4  x 4  x 
=6
 y = 11
14. (1) Given 3y + 2x = 47
7y - 11x = 0 ....(ii)  4x 4x 
D 4x 4x = 6
7     
From (ii), x = y
11
 8 
 Equation (i) reduces to D 2
 4  x 
2 = 6

7
3y + 2 × y = 47 8D
11 =6
16  x 2
33y  14y
 = 47 6 3
11 D= (16 - x2) = (16 - x2)
8 4
11  47
y= = 11 Since the speed of the stream (x) is not
47 given, the distance D cannot be determined.
7  11 18. (5) Tricky Approach
From (ii), x = =7 Speed of the boat in still water
11
 Required difference 1
=y-x = (Rate down stream + Rate upstream)
2
= 11 - 7 = 4
15. (5) 1
= (32 + 28) = 30 kmph.
Option (1) : 136 × 12  9 × 3 2
1
= 136 × 12 × × 3 = 544 INSURANCE EXAMS
9
1. (2) Rate upstream
Option (2) : 17 × 64  6 × 3
16
1 = kmph = 8 kmph
= 17 × 64 × × 3 = 544 2
6
Option (3) : 36 × 17  9 × 8 16
Rate downs tream =
1 4
= 56 × 8 × × 18 = 544 = 4 kmph
4
 Rate in still water
Option (4) : 56 × 8  14 × 17
1
1 = (8 + 4) kmph = 6 kmph
= 56 × 8 × × 18 = 544 2
4
2. (4) Let rate upstream be x kmph.
Option (5) : 76 × 6  19 × 17
Then, rate downstream
1 = 2x kmph
= 76 × 6 × × 17 = 408
19  Rate of current
16. (5) Given inequality : 1 x
x (x + 3) < 0 (2x - x) = kmph
2 2
Corresponding equation:
x (x + 3) = 0 x
 2=2 x=4
x=0
LEARN MATHS FROM S.K. RAJU (9811549822, 9811649822)
10
 Rate upstream = 4 kmph 7. (1) Given 3y + 2x = 47 ...(i)
Rate downstream = 8 kmph 7y - 11x = 0 ....(ii)
 Rate in still water 7
From(ii), x = y
1 11
= (8 + 4) = 6 kmph
2  Equation (i) reduces to
3. (2) Part of the tank filled by three pipes (A + 7
B + C) in 1 hour 3y + 2× y = 47
11
1
= 33y  14y
6  = 47
11
2 1
 Part filled in 2 hours = = 11  47
6 3 y= = 11
47
1 2
Remaining part = 1 - = 7  11
3 3 From(ii), x = =7
11
2  Required difference
Part filled by (A + B) in 7 hours =
3 =y-x
= 11 - 7 = 4
2
 (A + B)’s 1 hour’s work = 21 8. (1)

 Part filled by C in 1 hour


= Part filled by (A + B + C) in 1 hour - Part   8 parts
filled by (A + B) in 1 hour (4 parts) (8 parts)
1 2 14  8 6 1 9. (1) Let the speed of boat in still water be x
= - = = = kmph and that of current by y kmph.
6 21 84 84 14
According to the question,
C alone can fill the tank in 14 hours
4. (4) In convex polygon, each angle < 180° d d 21
5. (1) 2x2 + 9x + 4 = 0  x y + x y = ...(i)
4
 9  9 and,
2
 4 2 4
x=
2 2 2d d 7
x y = 7  x y 2 = ...(ii)
  9   81  32   9   49
= = By equation (ii) - (i),
4 4
d 21 7 21  14 7
 9  7 1 x y = 4 - 2 = 4
=
4
= = ,-4
4 2
Therefore, the required answer is 2d 7 1
 x  y = 2 = 3 2 hours
1
-4 < x < - 10. (4) Speed of boat in still water = x kmph.
2
6. (5) Given, x
Speed of current = kmph
2x + 3 + z = 55 ....(i) 3
x + z -y= 4 ...(ii)  Speed downstream
y - x + z = 12 ...(iii)
x 4x
From (ii) and (iii) = x + = kmph.
2z = 16  z = 8 3 3
From (ii), x + 8 - y = 4 Speed upstream
x=4 -8 +y=y -4 x 2x
=x - = kmph
Putting this value in (i) we get 3 3
2 (y - 4) + 3y + 8 = 55
4x 2x
 2y - 8 + 3y + 8 = 55  ×2- × 2 = 16
3 3
 5y = 55
 y = 11
LEARN MATHS FROM S.K. RAJU (9811549822, 9811649822)
11
4x AB is ladder.
 = 16  ABC = 30°
3
AC is a wall.
16  3
x = = 12 kmph BC
4 cos30° =
AB
b c
11. (1) = = b 2 = ca 3 BC
a a  =
Again, ax = by = cz = k 2 10
1 1 1
 a = k 2 , b = k y ,c = k 2 3
 BC = 10 ×
 b 2 = ac 2
2 1 1 = 5 3 metre
 ky = kx . kz = 5 × 1.732 = 8.66 metre
15. (1) The sum of two sides of a triangle is
2 1 1 z x
 y= + = greater than the third side.
x z zx When n = 6, 7
2z y 3+6>8
 = 3+7>8
x z x 16. (4) (x- 1) is a factor of 4x3 + 3x2 - 4x + k.
12. (3) f(2) = 2 × 23 + 3 × 22 + 5 On putting x = 1
= 16 + 12 + 5 = 33 4(1)3 + 3(1)2 - 4(1) + k = 0
2k 4+3 -4+k=0
13. (3) Sum of roots = 2  =
9  k = -3
k 1 1
 = 17. (1) of A  of B
9 8 12
4 1
Product of roots =  2
=  12 of A = 12 × 12 × 8 = 8
9
2 18. (2) Number of students who went on picnic
 k  4
=x
  =
 9  9 Students who planned = x + 5
k2 4 500 500
 =  k2 = 36  - =5
81 9 x x 5
k = ± 6 1 1 
 500  x  x  5  = 5
14. (3)  

x 5x 
 500  x  x  5   = 5
 
 x (x + 5) = 500
 x = 20

LEARN MATHS FROM S.K. RAJU (9811549822, 9811649822)


12
MODEL EXERCISES
1. The sides of a quadrilateral are extended (1) Only 0 (2) 0 and 3
to make the angles as shown below. 1
What is the value of x ? (3) - and 3 (4) 0 and 7
2
(5) None of these
1 1
7. If x + = 3, then the value of x + 2 is —
x x
(1) 9 (2) 10
(3) 27 (4) 7
(5) None of these
(1) 100° (2) 90°
(3) 80° (4) 75° 1 1
8. If log8x + log8 = , then the value of x is
(5) None of these 6 3
2. What is the area of the region in the —
cartesian plane whose points (x, y) satisfy (1) 18 (2) 24
|x| + |y| + |x + y)  2 ? (3) 16 (4) 12
(1) 2.5 (2) 3 (5) None of these
(3) 2 (4) 4 1 1
(5) None of these 9 If x + = 5, then the value of x3 + 3 is —
x x
3. From a horizontal distance of 50 m, the
(1) 125 (2) 110
angles of elevation of the top and the bottom
(3) 45 (4) 75
of a vertical cliff face are 45° and 30°
(5) None of these
respectively. The height of the cliff face (in
10. Of the following quadratic equations, which
m) is
is the one whose roots are 2 and -15 ?
50 50 (1) x2 - 2x + 15 = 0
(1) (2) (2) x2 + 15x - 2 = 0
3 2
(3) x2 + 13x - 30 = 0
50  1  (4) x2 - 30 = 0
(3) (4) 50 1  
2 3  3 (5) None of these
(5) None of these 11. The locus of a point equidistant from the
two fixed points is
4 (1) any straight line bisecting the segment
4. If G = H + then L equals joining the fixed points
L
(2) any straight line perpendicular to the
4 segment joining the fixed points
(1) (2) 4(G - H)2
G  H  (3) the straight line which is perpendicular
2

bisector of the segment joining the fixed


4 points
(3) G 2  H 2  (4) 4(G2 - H)2 (4) any straight line perpendicu­lar to the
line joining the fixed points
(5) None of these (5) None of these
1 1 1 12. Any cyclic parallelogram having unequal
5. If = y + , then z equals adjacent sides is necessarily a
x z
(1) square (2) rectangle
xy (3) rhombus (4) trapezium
(1)  x  y  (2) x - y (5) None of these
13. The lengths of three sides of a triangle are
xy x  y  known. In which of the cases given below,
(3)  y  x  (4) it is impossible to construct a triangle ?
xy
(1) 15 cm, 12 cm, 10 cm
(5) None of these (2) 3.6 cm, 4.3 cm, 5.7 cm
6. If (x - 3) (2x + 1) = 0, then possible value of (3) 17 cm, 12 cm, 6 cm
2x + 1 are — (4) 2.3 cm, 4.4 cm, 6.8 cm

LEARN MATHS FROM S.K. RAJU (9811549822, 9811649822)


13
(5) None of these (5) None of these
14. Two non-intersecting circles, one lying 20. In climbing a 21 m long round pole, a
inside another, are of diameters a and b. monkey climbs 6 m in the first minute and
The minimum distance between their slips 3 m in the next minute. What time (in
circumferences is c. The distance between minutes) the monkey would take to reach
their centres is the top of the pole ?
(1) a - b - c (2) a + b - c (1) 11 (2) 14
1 1 2
(3) (a - b - c) (4) (a - b) - c (3) 14 (4) 9
2 2 3
(5) None of these (5) None of these
15. A tin of oil was four fifths full. When six
bottles of oil were taken out and four bottles
21. The solution of the equation 25  x 2
of oil were poured into it, was three fourths = x -1 are
full. How many bottles of oil were contained (1) x = 3 and x = 4
by the tin ? (2) x = 5 and x = 1
(1) 10 (2) 20 (3) x = -3 and x = 4
(3) 30 (4) 40 (4) x = 4 and x  - 3
(5) None of these (5) None of these
16. A number consists of two digits. If the digits 22. Which one of the following is a factor of x3 -
in the unit’s place and the ten’s place are 7 19x + 30 ?
and x respectively, the number is — (1) x - 2 (2) x + 2
(1) x + 7 (2) 10 ( x + 7) (3) x -1 (4) x + l
(3) 70 + x (4) 10x + 7 (5) None of these
(5) None of these  1   1 
17. The sum of the digits of a three-digit 23. The value of  2  +  y 2  , where x = 2 +
number is 16. If the ten’s digit of the x   
number is 3 times the unit’s digit and the 3 and y = 2 - 3 ,is
unit’s digit is one-fourth of the hundredth (1) 14 (2) 12
digit, then what is the number ? (3) 10 (4) 16
(1) 446 (2) 561 (5) None of these
(3) 682 (4) 862 24. If x : y : z :: 1 : 3 : 5, then the value of
(5) None of these
18. If x11 = y° and x = 2y, then y is equal to — x 2  7y 2  9z 2
is
1 x
(1) (2) 1
(1) 7 (2) 17
2
(3) -1 (4) -2 (3) 13 (4) 1
(5) None of these (5) None of these
19. Pipes A and B can fill a tank in 5 and 6 h 25. If (x + 1) is factor of 2x3 - ax2 - (2a - 3) x + 2,
respectively. Pipe C can empty empty it in then the value of ‘a’ is
12 h. The tank is half full. All the three pipes (1) 3 (2) 2
are in operation simultaneously. After how 3 1
much time the tank will be full ? (3) (4)
2 2
9 (5) None of these
(1) 3 h (2) 11 h
17
8 13
(3) 2 h (4) l h
11 17

LEARN MATHS FROM S.K. RAJU (9811549822, 9811649822)


14
SHORT ANSWERS 1 x 50
1. (3) 2. (3)
3
= x= 3
50
3. (4) 4. (1)
5. (3) 6. (4) x  h 
7. (4) 8. (4) And tan 45° =
50
9. (2) 10. (3)
 x + h = 50
11. (3) 12. (2)
13. (4) 14. (4) 50  1 
15. (4) 16. (4)  h = 50 - =50 1  
3  3
17. (4) 18. (1)
19. (4) 20. (1) 4
21. (4) 22. (1) 4. (1) G = H +
L
23. (1) 24. (2)
25. (1) ~ 4
EXPLANATIONS  (G - H) =
L
1. (3) We know that sum of all the interior
angles of the polygon is 360°.
4
 L = G  H 2

1 1 1
5. (3) = y +
x z

1 1 1 y x
 = - y = xy
z x
xy
 (180° - x) + 105° + 65° + 90° = 360°.
z = y  x 
 x = 80°
2. (3) The region given by the inequality is 6. (4)(x - 3) (2x + 1) = 0
shown by the graph Then, (x - 3) = 0
x=3
and (2x + 1) = 0
If x = 3
Then, (2x + 1) = 7
 Possible values of (2x + 1) is 0 and 7.
2
 1 1
Area of one triangle 7. (4)  x   = x 2 + 2 + 2
1 1  x x
= × 1 × l= 1
2 2
 (9 - 2) = x 2 +
1 x2
Total area = 4 × =2 1
2
 x2 + =7
3. (4) Let the height of cliff PR is h m. x2
1 1
8. (4) log8x + log8 =
6 3

 1 1
 log8  x   =
 6 3

x 1
 log6   =
x 6 3
 tan 30° =
50 1 x
 83 =
6
[  if logay = x, then (a)x = y]

LEARN MATHS FROM S.K. RAJU (9811549822, 9811649822)


15
1 x 16. (4) Unit place = 7
 2 
3 3
= Tens place = 10
6
Hence, the number is 10x + 7.
 x = 12 17. (4) Let a, b and c be the digits at the
9. (2) hu ndre dth, ten ’s an d un it place
 3 1 
3
 1 respectively.
 1
 x   =  x  x3  + 3  x  x   a + b + c = 16 ...(i)
 x     b = 3c ...(ii)
 3 1  1
(5) 3 =  x  3  + 15 and c = a ...(iii)
 x  4
1 3
 x3 + = 125 - 15 = 110 From Eqs. (ii) and (iii), b =
4
a ...(iv)
x3
10. (3) Sum of roots = -13 From Eqs. (i), (iii) and (iv), we get
Product of roots = - 30 3 1
 Equation a+
4
a+
4
a = 16
x2 - x (sum of roots) + product of roots = 0
 a = 8, b = 6 and c = 2
 x2 + 13x - 30 = 0
Therefore, the three digit number is 862.
11. (3) Locus is the straight line which is
18. (1) If x11 = y° = 1
perpendicular bisector of the segment
joining the fixed points.  x11 =1  x = 1
12. (2) It is a rectangle. Given, x = 2y
13. (4) To construct a triangle sum of any two 1
sides is greater than the third side. Hence, 2y = 1  y =
2
option (d) is the correct answer. 19. (4) In one hour work done by all the pipes
14. (4) According to question, together
a 1 1 1
AC = = + -
2 5 6 12
b 17
BD = and CD = c = tank
2 60
In other work, tank is filled by all the pipes
60
working together in h.
17
1
 2 tank will be filled by all the pipes
Then, AB = AC - BC
60 1
a working together in ×
= - (BD + CD) 17 2
2
13
a b i.e. 1 h
= - -c 17
2 2 20. (1) Net distance travelled by monkey in 2
min = 3 m
1
= (a - b) - c  In 10 min, distance travelled (climbed)
2 = 15 m.
15. (4) Let the full capacity of the tin be x. As, it is given in the question that in first
4 3 minute it climbs 6 m and in other minute
Then, x-6+4= x it slips down 3 m.
5 4
And this happens alternately.
4 3 Hence, money will travel remaining 6 m
 x- x=2
5 4 in next minute.
 x = 40  Total time taken = (10 + 1) = 11 min

LEARN MATHS FROM S.K. RAJU (9811549822, 9811649822)


16
21. (4) 25  x 2 = x - 1 14
= = 14
49  16  3
 25 - x2 =  x  1
2

x y z
 25 - x2 = x2 + 1 - 2x 24. (2) Let = = =K
1 3 5
 2x2 - 2x - 24 = 0
 x2 - x - 12 = 0  x = K, y = 3K, z = 5K

  x  4  x  3  x 2  7y 2  9z 2

 x = 4, x = -3 x
x=4
K 2  7  9K 2  9  25K 2
 x  3 is not satisfy the give equation. =
K
22. (1) x3 - 19x + 30
= 289 = 17
(x - 2) is the fatctor of the above expression.
Because for x = 2 25. (1) Let f(x) = 2x3 -ax2 - (2a - 3) x + 2 = 0
(2)3 - 19 × (2) + 30 = 0 If (x + 1) is a factor of the avobe expression,
then f(-1) = 0
1 1
1 1  f(-) = 2(-1)3 - a(-1)2 - (2a - 3) × -1 + 2 = 0
   
2 2
23. (1) 2 + y 2 = 2  3 + 2 3  -2 -a +2a - 3 + 2 = 0
x
a-3 =0
1 1 a=3
  
= 74 3 + 74 3

LEARN MATHS FROM S.K. RAJU (9811549822, 9811649822)


1
NUMBER SERIES
NATIONALISED BANKS the number given, following the sequence of the
& IBPS SO/MT/SO original series and answer the questions that follow
Directions (1-5): In the following number the series.
series, a wrong number is given. Find out that (Union Bank of India
wrong number. PO Exam. 27.11.2005)
(Canara Bank PO Exam. 09.02.2003) 11. 12 30 120 460 1368 2730
1. 2 11 38 197 1172 8227 65806 16 (a) (b) (c) (d) (e)
(1) 11 (2) 38 What will come in place of (d) ?
(3) 197 (4) 1172 (1) 1384 (2) 2642
(5) 8227 (3) 2808 (4) 1988
2. 16 19 21 30 46 71 107 (5) None of these
(1) 19 (2) 21 12. 154 462 231 693 346.5 1039.5
(3) 30 (4) 46 276 (a) (b) (c) (d) (e)
(5) 71 What will come in place of (e) ?
3. 7 9 16 25 41 68 107 173 (1) 1746 (2) 621
(1) 107 (2) 16 (3) 1242 (4) 983
(3) 41 (4) 68 (5) None of these
(5) 25 13. 7 91 1001 7007 35035 105
4. 4 2 3.5 7.5 26.25 118.125 14.5 (a) (b) (c) (d) (e)
(1) 118.125 (2) 26.25 What will come in place of (c) ?
(3) 3.5 (4) 2 (1) 21132.5 (2) 14514.5
(5) 7.5 (3) 20020.5 (4) 13864.5
5. 16 4 2 1.5 1.75 1.875 (5) None of these
(1) 1.875 (2) 1.75 14. 582 574 601 537 662 446
(3) 1.5 (4) 2 204 (a) (b) (c) (d) (e)
(5) 4 What will come in place of (d) ?
Directions (6-10): What will come in place (1) 284 (2) 68
of the question mark (?) in the following number (3) 174 (4) 331
series ? (5) None of these
(Syndicate Bank PO Exam. 10.10.2004) 15. 85 43 44 67.5 137 345
6. 3 10 32 100 ? 125 (a) (b) (c) (d) (e)
(1) 345 (2) 460 What will come in place of (c) ?
(3) 308 (4) 440 (1) 86 (2) 107.5
(5) None of these (3) 112.5 (4) 97.5
7. 5 3 4 ? 38 (5) None of these
(1) 8.5 (2) 6 Directions (16-22) : What will come in
(3) 7.5 (4) 8 place of the question mark (?) in the following
(5) None of these number series ?
8. 5 6 ? 57 244 (Corporation Bank Po
(1) 21 (2) 16 Exam. 29.07.2006)
(3) 17 (4) 15 16. 1 ? 27 64 125
(5) None of these (1) 8 (2) 4
9. 3 10 21 ? 51 (3) 6 (4) 9
(1) 34 (2) 32 (5) None of these
(3) 33 (4) 37 17. 25 16 ? 4 1
(5) None of these (1) 3 (2) 6
10. 5 11 ? 55 117 (3) 12 (4) 18
(1) 21 (2) 27 (5) None of these
(3) 23 (4) 25 18. 1 6 36 240 1960 ?
(5) None of these (1) 19660 (2) 3680
Directions (11-15): In each of the following (3) 36800 (4) 19600
questions a number series is given. After the series (5) None of these
a number is given followed by (a), (b), (c), (d) and 19. 12 14 17 13 8 14 21 13 4 ?
(e). You have to complete the series starting with
LEARN MATHS FROM S.K. RAJU (9811549822, 9811649822)
2
(1) 14 (2) 13 (3) 1764 (4) 3136
(3) 15 (4) 2 (5) 6561
(5) None of these 31. 23 25 53 163 657 3291 ?
20. 2 5 7 12 19 31 50 ? (1) 16461 (2) 13169
(1) 53 (2) 81 (3) 9877 (4) 23045
(3) 69 (4) 74 (5) 19753
(5) None of these 32. 13 13 65 585 7605 129285 ?
21. 15 12 17 10 ? 8 21 6 (1) 2456415 (2) 2235675
(1) 3 (2) 7 (3) 2980565 (4) 2714985
(3) 21 (4) 19 (5) 2197845
(5) None of these Directions (33-37) : What should come in
22. 4 6 12 30 90 315 ? place of question mark (?) in the following number
(1) 945 (2) 1102 series ?
(3) 1260 (4) 1417.5 (Andhra Bank PO Exam. 14.09.2008)
(5) None of these 33. 40280625 732375 16275 465 18.6
Directions (16-22) : What should come in 1.24 ?
place of the question mark (?) in the following (1) 0.248 (2) 0.336
number series ? (3) 0.424 (4) 0.512
(Bank Of Maharashtra PO (5) 0.639
Exam. 29.07.2006) 34. 14 12 21 59 231 1149 ?
23. 1548 516 129 43 ? (1) 6987 (2) 6787
(1) 11 (2) 10.75 (3) 6887 (4) 6687
(3) 9.5 (4) 12 (5) 6587
(5) None of these 35. 1728 2744 4096 5832 8000 10648
24. 949 189.8 ? 22.776 11.388 6.8328 ?
(1) 48.24 (2) 53.86 (1) 12167 (2) 13824
(3) 74.26 (4) 56.94 (3) 15625 (4) 9261
(5) None of these (5) 17576
25. 121 144 190 259 ? 466 36. 120 15 105 17.5 87.5 ?
(1) 351 (2) 349 (1) 18.5 (2) 19.5
(3) 374 (4) 328 (3) 21.875 (4) 17.5
(5) None of these (5) 90
26. 14 43.5 264 ? 76188 37. 3 6 21 28 55 66 ? 120
(1) 3168 (2) 3176 (1) 103 (2) 104
(3) 1587 (4) 1590 (3) 108 (4) 106
(5) None of these (5) 105
27. 41 164 2624 ? 6045696 Directions (38-42) : In each of the following
(1) 104244 (2) 94644 questions a number se­ries is given which has only
(3) 94464 (4) 102444 one wrong number. You have to find out the wrong
(5) None of these number.
Directions (28-32): What should come In (Bank Of Baroda Specialist
place of question mark (?) in the following number Officer Exam. 05.10.2008)
series ? 38. 7.25 47.5 87.5 157.5 247.5 357.5
(Indian Overseas Bank 487.5
PO Exam. 15.06.2008) (1) 357.5 (2) 87.5
28. 12 12 18 45 180 1170 ? (3) 157.5 (4) 7.5
(1) 12285 (2) 10530 (5) 47.5
(3) 11700 (4) 12870 39. 13 16 21 27 39 52 69
(5) 7605 (1) 21 (2) 39
29. 444 467 513 582 674 789 ? (3) 27 (4) 52
(1) 950 (2) 904 (5) 16
(3) 927 (4) 881 40. 1500 1581 1664 1749 1833 1925 2016
(5) 973 (1) 1581 (2) 1664
30. 1 16 81 256 625 1296 ? (3) 1833 (4) 1925 (5) 1749
(1) 4096 (2) 2401 41. 66 91 120 153 190 233 276

LEARN MATHS FROM S.K. RAJU (9811549822, 9811649822)


3
(1) 120 (2) 233 (1) 708 (2) 3534
(3) 153 (4) 276 (3) 14136 (4) 42405
(5) 190 (5) None of these
42. 1331 2197 3375 4914 6859 9261 Directions (53-57): What should come in
12167 place of question mark (?) in the following number
(1) 4914 (2) 6859 series ?
(3) 9261 (4) 2197 (Canara Bank PO Exam. 15.03.2009)
(5) 12167 53. 5 9 18 34 59 95 ?
Directions (43-47): What should come in (1) 272 (2) 168
place of the question mark (?) in the following (3) 116 (4) 148
number series ? (5) 144
(Oriental Bank of Commerce 54. 1200 480 192 76.8 30.72 12.288 ?
PO Exam. 21.12.2008) (1) 4.9152 (2) 5.8192
43. 20 24 33 49 74 110 ? (3) 6.7112 (4) 7.6132
(1) 133 (2) 147 (5) 8.5172
(3) 159 (4) 163 55. 963 927 855 747 603 423 ?
(5) 171 (1) 209 (2) 208
44. 529 841 961 1369 1681 1849 ? (3) 207 (4) 206
(1) 2809 (2) 2601 (5) 205
(3) 3249 (4) 3481 56. 841 961 1089 1225 1369 1521 ?
(5) 2209 (1) 1581 (2) 1681
45. 16 24 48 120 360 1260 ? (3) 1781 (4) 1881
(1) 3780 (2) 4725 (5) 1981
(3) 5355 (4) 5040 57. 18 20 44 138 560 2810 ?
(5) 4410 (1) 16818 (2) 16836
46. 8 31 122 485 1936 7739 ? (3) 16854 (4) 16872
(1) 30950 (2) 46430 (5) 16890
(3) 34650 (4) 42850 Directions (58-62) : In the following
(5) 38540 number series only one number is wrong. Find
47. 499 622 868 1237 1729 2344 ? out the wrong number.
(1) 3205 (2) 3082 (UCO Bank PO Exam. 22.03.2009)
(3) 2959 (4) 3462 58. 4 6 18 49 201 1011
(5) 2876 (1) 1011 (2) 201
Directions (48-52) : In the following (3) 18 (4) 49
number series only one number is wrong. Find (5) None of these
out the wrong number. 59. 48 72 108 162 243 366
(PNB Agriculture Officer (1) 72 (2) 108
Exam. 04.01.2009) (3) 162 (4) 243
48. 1 4 27 256 3125 46658 (5) None of these
(1) 46658 (2) 4 60. 2 54 300 1220 3674 7350
(3) 27 (4) 3125 (1) 3674 (2) 1220
(5) None of these (3) 300 (4) 54
49. 18000 3600 720 142.2 28.8 5.76 (5) None of these
(1) 28.8 (2) 3600 61. 8 27 64 125 218 343
(3) 5.76 (4) 142.2 (1) 27 (2) 218
(5) None of these (3) 125 (4) 343
50. 12 237 406 527 604 657 (5) None of these
(1) 237 (2) 406 62. 19 68 102 129 145 154
(3) 527 (4) 657 (1) 154 (2) 129
(5) None of these (3) 145 (4) 102
51. 3 35 226 1160 4660 13998 (5) None of these
(1) 13998 (2) 4660 Directions (63-67): What should come in
(3) 226 (4) 1160 place of the question mark (?) in the following
(5) None of these number series ?
52. 18 119 708 3534 14136 42405 (Indian Overseas Bank
PO Exam. 05.04.2009)
LEARN MATHS FROM S.K. RAJU (9811549822, 9811649822)
4
63. 0 5 18 43 84 145 ? 75. 9 17 ? 65 129
(1) 220 (2) 240 (1) 32 (2) 24
(3) 260 (4) 280 (3) 35 (4) 33
(5) None of these (5) None of these
64. 10 17 48 165 688 3475 ? 76. 7 13 ? 49 97
(1) 27584 (2) 25670 (1) 27 (2) 25
(3) 21369 (4) 20892 (3) 23 (4) 29
(5) None of these (5) None of these
65. 1 3 24 360 8640 302400 ? 77. 5 3 6 ? 64.75
(1) 14525100 (2) 154152000 (1) 15 (2) 15.5
(3) 14515200 (4) 15425100 (3) 17.5 (4) 17.25
(5) None of these (5) None of these
66. 12 14 32 102 416 2090 ? Directions (78-82) : What will come in
(1) 15522 (2) 12552 place of the question mark (?) in each of the
(3) 13525 (4) 17552 following number series ?
(5) None of these (PNB Specialist Officer’s
67. 10 15 15 12.5 9.375 6.5625 ? Exam. 16.08.2009)
(1) 4.375 (2) 3.2375 78. 16 8 12 30 ?
(3) 4.6275 (4) 3.575 (1) 75 (2) 105
(5) None of these (3) 95 (4) 115
Directions (68-72) : What will come in (5) None of these
place of the question mark (?) in each of the (United Bank of India PO
following series ? Exam. 21.06.2009)
68. 17 52 158 477 ? 4310 79. 5 6 14 45 ?
(1) 1433 (2) 1432 (1) 138 (2) 154
(3) 1435 (4) 1434 (3) 118 (4) 184
(5) None of these (5) None of these
69. 3 22 ? 673 2696 8093 80. 7 12 32 105 ?
(1) 133 (2) 155 (1) 428 (2) 214
(3) 156 (4) 134 (3) 218 (4) 416
(5) None of these (5) None of these
70. 6 13 38 ? 532 2675 81. 11 23 47 95 ?
(1) 129 (2) 123 (1) 189 (2) 193
(3) 172 (4) 164 (3) 181 (4) 195
(5) None of these (5) None of these
71. 286 142 ? 34 16 7 82. 9 17 33 65 ?
(1) 66 (2) 72 (1) 113 (2) 131
(3) 64 (4) 74 (3) 129 (4) 118
(5) None of these (5) None of these
72. 17 9 ? 16.5 35 90 Directions (83-84) : In the following
(1) 5 (2) 15 number series only one number is wrong. Find
(3) 10 (4) 20 out the wrong number.
(5) None of these (Corporation Bank PO
Directions (73-77): What will come in place Exam. 22.11.2009)
of the question mark (?) in each of the following 83. 8 11 17 47 128 371 1100
rtamber series ? (1)11 (2)47
(Andhra Bank PO Exam 05.07.2009) (3) 17 (4) 371
73. 2 8 26 ? 242 (5) 128
(1) 78 (2) 72 84. 1 5 13 31 61 125 253
(3) 82 (4) 84   (1) 1 (2) 5
(5) None of these (3)31 (4)61
74. 3 4 12 ? 196 (5) 125
(1) 45 (2) 40 Directions (85-89) : In the following
(3) 41 (4) 49 number series a wrong number is given. Find out
(5) None of these the wrong number.

LEARN MATHS FROM S.K. RAJU (9811549822, 9811649822)


5
(Indian Bank Rural Marketing (1) 149 (2) 146
Officer Exam. 03.01.2010) (3) 142 (4) 152
85. 150 290 560 1120 2140 4230 (5) None of these
8400 96. 7 4 5 9 ? 52.5 160.5
(1) 2140 (2) 560 (1) 32 (2) 16
(3) 1120 (4) 4230 (3) 14 (4) 20
(5) 290 (5) None of these
86. 10 8 13 35 135 671 4007 97. 6 42 ? 1260 5040 15120 30240
(1) 8 (2) 671 (1) 546 (2) 424
(3) 135 (4) 13 (3) 252 (4) 328
(5) 35 (5) None of these
87. 80 42 24 13.5 8.75 6.375 5.1875 98. 4 10 40 190 940 ? 23440
(1) 8.75 (2) 13.5 (1) 4690 (2) 2930
(3) 24 (4) 6.375 (3) 5140 (4) 3680
(5) 42 (5) None of these
88. 125 75 45 25 99.
16.2 9.72 5.832 2 9 30 ? 436 2195 13182
(1) 25 (2) 45 (1) 216 (2) 105
(3) 9.72 (4) 16.2 (3) 178 (4) 324
(5) 75 (5) None of these
89. 29 37 21 43 13 53 5 Directions (100-104): In each question
(1) 37 (2) 53 below, a number series is given in which one
(3) 13 (4) 21 number is wrong. Find out the wrong number.
(5) 43 (Allahabad Bank PO Exam. 21.02.2010)
100. 484 240 120 57 26.5 11.25 3.625
Directions (90-9 4): In the following
number series only one number is wrong. Find (1) 240 (2) 120
out the wrong number. (3) 57 (4) 26.5
(Indian Bank PO Exam. 17.10.2010) (5) 11.25
90. 13 25 40 57 79 103 130 101. 3 5 13 43 176 891 5352
(1) 25 (2) 40 (1) 5 (2) 13
(3) 57 (4) 79 (3) 43 (4) 176
(5) None of these (5) 891
91. 850 600 550 500 475 102. 6
462.5 7 16 41 90 154 292
456.25 (1) 7 (2) 16
(1) 600 (2) 550 (3) 41 (4) 90
(3) 500 (4) 462.5 (5) 154
(5) None of these 103. 5 7 16 57 244 1245 7506
92. 2 10 18 54 162 486 1458 (1) 7 (2) 16
(1) 18 (2) 54 (3) 57 (4) 244
(3) 162 (4) 10 (5) 1245
(5) None of these 104. 4 2.5 3.5 6.5 15.5 41.25 126.75
93. 8 12 24 46 72 108 152 (1) 2.5 (2) 3.5
(1) 12 (2) 24 (3) 6.5 (4) 15.5
(3) 46 (4) 72 (5) 41.25
(5) None of these Directions (105-109) : What should come
94. 142 119 100 83 65 59 52 in place of the question mark (?) in the following
(1) 65 (2) 100 number series.
(3) 59 (4) 119 (Corporation Bank PO
(5) None of these Exam. 09.05.2010)
Directions (95-99) : What should come in
105. 325 314 292 259 215 ?
place of the question mark in the following number (1) 126 (2) 116
series ? (3) 130 (4) 160
(Bank Of India Banking (5) None of these
Officer Exam. 24.01.2010) 106. 45 46 70 141 ? 1061.5
(1) 353 (2) 353.5
95. 5 54 90 115 131 140 ? (3) 352.5 (4) 352
(5) None of these
LEARN MATHS FROM S.K. RAJU (9811549822, 9811649822)
6
107. 620 632 608 644 596 ? (3) 486 (4) 1657
(1) 536 (2) 556 (5) None of these
(3) 656 (4) 646 119. 656 432 320 264 236 (?)
(5) None of these (1) 222 (2) 229
108. 15 25 40 65 ? 195 (3) 232 (4) 223
(1) 115 (2) 90 (5) None of these
(3) 105 (4) 120 Directions (120-124): What will come in
(5) None of these place of the question mark (?) in the following
109. 120 320 ? 2070 5195 13007.5 number series ?
(1) 800 (2) 920 (Central Bank Of India
(3) 850 (4) 900 PO Exam. 25.07.2010)
(5) None of these 120. 7 20 46 98 202 (?)
Directions (110-114): In the following (1) 420 (2) 410
number series only one number is wrong. Find (3) 310 (4) 320
out the wrong number. (5) None of these
(Punjab & Sind Bank PO 121. 210 209 213 186 202 (?)
Exam. 16.05.2010) (1) 138 (2) 77
110. 32 34 37 46 62 87 123 (3) 177 (4) 327
(1) 34 (2) 37 (5) None of these
(3) 62 (4) 87 122. 27 38 71 126 203 (?)
(5) 46 (1) 212 (2) 202
111. 7 18 40 106 183 282 403 (3) 301 (4) 312
(1) 18 (2) 282 (5) None of these
(3) 40 (4) 106 123. 435 354 282 219 165 (?)
(5) 183 (1) 103 (2) 112
112. 850 843 829 808 788 745 703 (3) 120 (4) 130
(1) 843 (2) 829 (5) None of these
(3) 808 (4) 788 124. 4 200 369 513 634 (?)
(5) 745 (1) 788 (2) 715
113. 33 321 465 537 573 590 600 (3) 734 (4) 755
(1) 321 (2) 465 (5) None of these
(3) 573 (4) 537 Directions (125-129) : What will come in
(5) 590 place of the question mark (?) in the following
114. 37 47 52 67 87 112 142 number series ?
(1) 47 (2) 52 (Syndicate Bank PO Exam. 29.08.2010)
(3) 67 (4) 87 125. 495 485 465 425 345 ?
(5) 112 (1) 195 (2) 165
Directions (115-119) : What will come in (3) 185 (4) 175
place of the question mark (?) in the following (5) None of these
number series ? 126. 16 22 33 49 70 ?
(Bank Of Baroda PO Exam. 30.05.2010) (1) 95 (2) 96
115. 13 16 22 33 51 (?) (3) 85 (4) 91
(1) 89 (2) 78 (5) None of these
(3) 102 (4) 69 127. 32 36 52 88 152 ?
(5) None of these (1) 266 (2) 232
116. 39 52 78 117 169 (?) (3) 242 (4) 256
(1) 246 (2) 182 (5) None of theses
(3) 234 (4) 256 128. 17 289 425 493 527 ?
(5) None of these (1) 534 (2) 542
117. 62 87 187 412 812 (?) (3) 544 (4) 594
(1) 1012 (2) 1437 (5) None of these
(3) 1337 (4) 1457 129. 13 27 55 97 153 ?
(5) None of these (1) 243 (2) 265
118. 7 8 24 105 361 (?) (3) 215 (4) 223
(1) 986 (2) 617 (5) None of these

LEARN MATHS FROM S.K. RAJU (9811549822, 9811649822)


7
Directions (130-134) : What should come (1) 716 (2) 788
in place of the question mark (?) in the following (3) 348 (4) 689
number series ? (5) 780
(Punjab National Bank Specialist 141. 4444 2224 1114 556 281.5 142.75
Officer Exam. 24.10.2010) 73.375
130. 50 60 75 97.5 ? 184.275 267.19875 (1) 2224 (2) 281.5
(1) 120.50 (2) 130.50 (3) 1114 (4) 556
(3) 131.625 (4) 124.25 (5) 142.75
(5) None of these 142. 4.5 16 25 33 38.5 42 43.5
131. 12 15 36 ? 480 2415 14508 (1) 33 (2) 38.5
(1) 115 (2) 109 (3) 42 (4) 43.5
(3) 117 (4) 121 (5) 25
(5) None of these 143. 6 49 305 1545 6196 18603 37218
132. 1 2 6 21 88 445 ? (1) 6196 (2) 49
(1) 2230 (2) 2676 (3) 305 (4) 1545
(3) 2580 (4) 2670 (5) 18603
(5) None of these 144. 8 5 6.5 11 26 68 207.5
133. 20 21 25 34 50 ? 111 (1) 68 (2) 6.5
(1) 70 (2) 65 (3) 11 (4) 26
(3) 60 (4) 75 (5) 207.5
(5) None of these Directions (145-149) : What should come
134. 600 125 30 ? 7.2 6.44 6.288 in place of the question mark (?) in the following
(1) 6 (2) 10 number series ?
(3) 15 (4) 12 (PNB Management Trainee
(5) None of these Exam. 28.11.2010)
Directions (135-139): What will come in 145. 586 587 586 581 570 ? 522
the place of the question mark (?) in the following (1) 545 (2) 543
number series ? (3) 551 (4) 557
(Bank Of India PO Exam. 31.10.2010) (5) None of these
135. 11 15 31 67 131 (?) 146. 64 54 69 49 74 44 ?
(1) 233 (2) 221 (1) 89 (2) 69
(3) 243 (4) 231 (3) 59 (4) 99
(5) None of these (5) None of these
136. 483 471 435 375 291 (?) 147. 4000 2008 1012 ? 265 140.5 78.25
(1) 183 (2) 184 (1) 506 (2) 514
(3) 185 (4) 186 (3) 520 (4) 512
(5) None of these (5) None of these
137. 5 7 13 25 45 (?) 148. 5 5 15 75 ? 4725 51975
(1) 67 (2) 75 (1) 520 (2) 450
(3) 65 (4) 55 (3) 525 (4) 300
(5) None of these (5) None of these
138. 4 11 25 53 109 (?) 149. 52 26 26 39 78 ? 585
(1) 221 (2) 234 (1) 195 (2) 156
(3) 212 (4) 222 (3)234 (4)117
(5) None of these (5) None of these
139. 15 21 33 51 75 (?) Directions (150-154) .-What will come in
(1) 113 (2) 103 place of question mark (?) in the following number
(3) 105 (4) 115 series ?
(5) None of these (Bank Of Maharashtra
Directions (140-144): In the following Exam. 19.12.2010)
number series only one number is wrong. Find 150. 10 14 25 55 140 (?)
out the wrong number. (1) 386 (2) 398
(United Bank Of India (3) 388 (4) 396
PO Exam. 14.11.2010) (5) None of these
140. 5 348 564 689 716 780 788 151. 119 131 155 191 239 (?)
LEARN MATHS FROM S.K. RAJU (9811549822, 9811649822)
8
(1) 289 (2) 290 (5) None of these
(3) 279 (4) 280 163. 5 12 36 123 ? 2555 15342
(5) None of these (1) 508 (2) 381
152. 11 57 149 333 701 (?) (3) 504 (4) 635
(1) 1447 (2) 1347 (5) None of these
(3) 1368 (4) 1437 164. 8 11 17 ? 65 165.5 498.5
(5) None of these (1) 27.5 (2) 32
153. 697 553 453 389 353 (?) (3) 28 (4) 30.5
(1) 328 (2) 337 (5) None of these
(3) 362 (4) 338 Directions (165-169) :What will come in
(5) None of these place of the question mark (?) in the following
154. 336 224 168 140 126 (?) number series ?
(1) 119 (2) 118 (Union Bank Of India PO
(3) 116 (4) 121 Exam. 09.01.2001)
(5) None of these , 165. 117 389 525 593 627 (?)
Directions (155-159): What will come in (1) 654 (2) 640
place of the question mark (?) in the following (3) 634 (4) 630
number series ? (5) None of these
(Oriental Bank Of Commerce PO 166. 7 11 23 51 103 (?)
Exam. 26.12.2010 (1st Sitting) (1) 186 (2) 188
155. 9 15 27 51 99 ? (3) 185 (4) 187
(1) 165 (2) 195 (5) None of these
(3) 180 (4) 190 167. 18 27 49 84 132 (?)
(5) None of these (1) 190 (2) 183
156. 13 21 36 58 87 ? (3) 180 (4) 193
(1) 122 (2) 128 (5) None of these
(3) 133 (4) 123 168. 33 43 65 99 145 (?)
(5) None of these (1) 201 (2) 203
157. 7 9 19 45 95 ? (3) 205 (4) 211
(1) 150 (2) 160 (5) None of these
(3) 145 (4) 177 169. 655 439 314 250 223 (?)
(5) None of these (1) 205 (2) 210
158. 14 15 23 32 96 ? (3) 195 (4) 190
(1) 121 (2) 124 (5) None of these
(3) 152 (4) 111 Directions (170-174): What will come in
(5) None of these place of the question mark (?) in the following
159. 20 24 36 56 84 ? number series ?
(1) 116 (2) 124 (Corporation Bank PO
(3) 120 (4) 128 Exam. 16.01.2011)
(5) None of these 170. 15 21 39 77 143 (?)
Directions (160-164) : What should come (1) 243 (2) 240
in place of the question mark (?) In the following (3) 253 (4) 245
number series ? (5) None of these
(Indian Bank PO Exam. 02.01.2011) 171. 33 39 57 87 129 (?)
160. 3 732 1244 1587 1803 1928 ? (1) 183 (2) 177
(1) 2144 (2) 1992 (3) 189 (4) 199
(3) 1955 (4) 2053 (5) None of these
(5) None of these 172. 15 19 83 119 631 (?)
161. 16 24 ? 210 945 5197.5 33783.75 (1) 731 (2) 693
(1) 40 (2) 36 (3) 712 (4) 683
(3) 58 (4) 60 (5) None of these
(5) None of these 173. 19 26 40 68 124 (?)
162. 45030 9000 1795 355 68 ? 1.32 (1) 246 (2) 238
(1) 11.6 (2) 12.2 (3) 236 (4) 256
(3) 10.4 (4) 9.8 (5) None of these

LEARN MATHS FROM S.K. RAJU (9811549822, 9811649822)


9
174. 43 69 58 84 73 (?) Directions (185-189): What will come in
(1) 62 (2) 98 place of the question mark (?) in the following
(3) 109 (4) 63 number series.
(5) None of these (Bank Of Baroda PO Exam.13.03.2011)
Directions (175-179): What should come 185. 1 7 49 343 (?)
in place of the question mark (?) in the following (1) 16807 (2) 1227
number series ? (3) 2058 (4) 2401
(Punjab & Sind Bank PO (5) None of these
Exam. 23.01.2011) 186. 13 20 39 78 145 (?)
175. 15 18 16 19 17 20 ? (1) 234 (2) 244
(1) 23 (2) 22 (3) 236 (4) 248
(3) 16 (4) 18 (5) None of these
(5) None of these 187. 12 35 81 173 357 (?)
176. 1050 420 168 67.2 26.88 10.752 ? (1) 725 (2) 715
(1) 4.3008 (2) 6.5038 (3) 726 (4) 736
(3) 4.4015 (4) 5.6002 (5) None of these
(5) None of these 188. 3 100 297 594 991 (?)
177. 0 6 24 60 120 210 ? (1) 1489 (2) 1479
(1) 343 (2) 280 (3) 1478 (4) 1498
(3) 335 (4) 295 (5) None of these
(5) None of these 189. 112 119 140 175 224 (?)
178. 32 49 83 151 287 559 ? (1) 277 (2) 276
(1) 1118 (2) 979 (3) 287 (4) 266
(3) 1103 (4) 1120 (5) None of these
(5) None of these Directions (190-194): What will come in
179. 462 552 650 756 870 992 ? place of the question mark (?) in the following
(1) 1040 (2) 1122 number series ?
(3) 1132 (4) 1050 (Allahabad Bank PO Exam.17.04.2011)
(5) None of these 190. 958 833 733 658 608 (?)
Directions (180-184): What will come in (1) 577 (2) 583
place of the question mark (?) in the following (3) 567 (4) 573
number series ? (5) None of these
(UCO Bank PO Exam. 30.01.2011) 191. 11 10 18 51 200 (?)
180. 28 39 63 102 158 (?) (1) 885 (2) 1025
(1) 232 (2) 242 (3) 865 (4) 995
(3) 233 (4) 244 (5) None of these
(5) None of these 192. 25 48 94 186 370 (?)
181. 7 16 141 190 919 (?) (1) 738 (2) 744
(1) 1029 (2) 1019 (3) 746 (4) 724
(3) 1020 (4) 1030 (5) None of these
(5) None of these 193. 14 24 43 71 108 (?)
182. 12 17 32 57 92 (?) (1) 194 (2) 154
(1) 198 (2) 195 (3) 145 (4) 155
(3) 137 (4) 205 (5) None of these
(5) None of these 194. 144 173 140 169 136 (?)
183. 19 25 45 87 159 (?) (1) 157 (2) 148
(1) 254 (2) 279 (3) 164 (4) 132
(3) 284 (4) 269 (5) None of these
(5) None of these Directions (195-199): What will come in
184. 83 124 206 370 698 (?) place of the question mark (?) in the following
(1) 1344 (2) 1324 number series ?
(3) 1364 (4) 1334 (Indian Overseas Bank PO
(5) None of these Exam. 22.05.2011)
195. 8 10 18 44 124 (?)
(1) 344 (2) 366
LEARN MATHS FROM S.K. RAJU (9811549822, 9811649822)
10
(3) 354 (4) 356 (3) 4621 (4) 5135
(5) None of these (5) 5506
196. 13 25 61 121 205 (?) 206. 6 7 9 13 26 37 69
(1) 323 (2) 326 (1) 7 (2) 26
(3) 324 (4) 313 (3) 69 (4) 37
(5) None of these (5) 9
197. 656 352 200 124 86 (?) 207. 1 3 10 36 152 760 4632
(1) 67 (2) 59 (1) 3 (2) 36
(3) 62 (4) 57 (3) 4632 (4) 760
(5) None of these (5) 152
198. 454 472 445 463 436 (?) 208. 4 5 13 40 105 229 445
(1) 436 (2) 456 (1) 4 (2) 13
(3) 454 (4) 434 (3) 105 (4) 445
(5) None of these (5) 229
199. 12 18 36 102 360 (?) 209. 157.5 45 15 6 3 2 1
(1) 1364 (2) 1386 (1) 1 (2) 2
(3) 1384 (4) 1376 (3) 6 (4) 157.5
(5) None of these (5) 45
Directions (200-204): In the following Directions (210-215): What will come in
number series only one number is wrong. Find place of the question mark (?) in the following
out the wrong number. number series ?
(IBPS Bank PO/MT CWE (IDBI Bank Officer Exam. 16.09.2012)
Exam. 18.09.2011) 210. 123 277 459 669 907 ?
200. 7 12 40 222 1742 17390 208608 (1) 1179 (2) 1173
(1) 222 (2) 12 (3) 1167 (4) 1169
(3)40 (4) 1742 (5) None of these
(5) 208608 211. 456.5 407 368.5 341 324.5 ?
201. 6 91 584 2935 11756 35277 (1) 321 (2) 319
70558 (3) 317 (4) 323
(1) 6 (2) 70558 (5) None of these
(3) 584 (4) 2935 212. 23 42.2 80.6 157.4 311 ?
(5) 35277 (1) 618.2 (2) 623.2
202. 9050 5675 3478 2147 1418 1077 (3) 624.2 (4) 616.6
950 (5) None of these
(1) 950 (2) 1418 213. 36 154 232 278 300 ?
(3) 5675 (4) 2147 (1) 304 (2) 313
(5) 1077 (3) 308 (4) 307
203. 1 4 25 256 3125 46656 (5) None of these
823543 214. 24 536 487 703 678 ?
(1) 4 (2) 823543 (1) 768 (2) 748
(3) 46656 (4) 25 (3) 764 (4) 742
(5) 256 (5) None of these
204. 8424 4212 2106 1051 526.5 263.25 215. 224 576 752 840 884 ?
131.625 (1) 960 (2) 890
(1) 526.5 (2) 1051 (3) 906 (4) 908
(3) 4212 (4) 8424 (5) None of these
(5) 263.25 Directions (216-220) : What should come
Directions (205-209): In each of thesein place of the question mark (?) in the following
questions a number series is given. In each series
series ?
only one number is wrong. Find out the wrong (IBPS RRBs Office Assistant CWE
number. Exam. 09.09.2012)
(IBPS Bank PO/MT CWE 17.06.2012) 216. 5 6 16 57 ? 1245
205. 5531 5506 5425 5304 5135 4910 (1) 244 (2) 148
4621 (3) 296 (4) 271
(1) 5531 (2) 5425 (5) None of these
217. 12 ? 168 504 1260 2520
LEARN MATHS FROM S.K. RAJU (9811549822, 9811649822)
11
(1) 96 (2) 59 (1) 22.4 (2) 59.5
(3) 61 (4) 48 (3) 11.8 (4) 38.3
(5) None of these (5) 87.3
218. 4 9 29 ? 599 3599 229. 1 2 4 9 23 69 186
(1) 117 (2) 347 (1) 2 (2) 9
(3) 258 (4)174 (3) 23 (4) 4
(5) None of these (5) 69
219. 177 170 159 146 ? 110 230. 250 239 216 181 136 75 4
(1) 132 (2) 106 (1) 239 (2) 181
(3) 129 (4) 127 (3) 75 (4) 216
(5) None of these (5) 136
220. 2 3 11 38 102 ?
(1) 402 (2) 182 SBI PO EXAMS
(3) 227 (4) 168 Directions (1-5): One number is wrong in
(5) None of these each of the number series given in each of the
Directions (221-225): What will come in following questions. You have to identify that
place of the question mark (?) in the following number and assuming that a new series starts with
number series ? that number following the same logic as in the
(Indian Overseas Bank PO given series, which of the numbers given in (1),
Online Exam. 01.09.2013) (2), (3), (4) and (5) given below each series will be
221. 21 10.5 ? 15.75 31.5 78.75 the third number in the new series ?
(1) 10.5 (2) 11.5 (SBI Associate Banks PO
(3) 12.5 (4) 10.25 Exam. 14.02.1999)
(5) None of these 1. 3 5 12 38 154 914 4634
222. 6 19 58 ? 214 331 (1) 1636 (2) 1222
(1) 113 (2) 123 (3) 1834 (4) 3312
(3) 133 (4) 143 (5) 1488
(5) None of these 2. 3 4 10 34 136 685 4116
223. ? 16 28 58 114 204 (1) 22 (2) 276
(1) 7 (2) 9 (3) 72 (4) 1374
(3) 14 (4) 6 (5) 12
(5) 10 3. 214 18 162 62 143 90 106
224. 13. 76 14.91 17.21 20.66 ? 31.01 (1) 34 (2) 110
(1) 25.66 (2) 24.36 (3) 10 (4) 91
(3) 24.26 (4) 25.26 (5)38
(5) 25.36 4. 160 80 120 180 1050 4725
225. 15 ? 24 33 97 122 25987.5
(1) 20 (2) 19 (1) 60 (2) 90
(3) 17 (4) 18 (3) 3564 (4) 787.5
(5) 16 (5) 135
Directions (226-230) : In each of the 5. 2 3 7 13 25 47 78
following number series, a number is wrong. Find (1) 11 (2) 13
out that wrong number. (3) 15 (4) 18
(Corporation Bank Specialist Officer (5) 20
(Marketing) Exam 22.12.2014) Directions (6-8): In each of the following
226. 2 6 15 30 45 43.5 22.5 questions, a number series is given. After the
(1) 6 (2) 30 series, below it, a number alongwith (a), (b), (c), (d)
(3) 45 (4) 15 and (e) is given. You have to complete the series
(5) 43.5 following the same sequence as that of the given
227. 950 661 436 269 146 65 16 series. Then answer the question that follows.
(1) 436 (2) 65 (SBI Associate Banks PO
(3) 269 (4) 661 Exam. 16.07.2000)
(5) 146 6. 2 3 10 29 172 885
228. 6.5 11.8 22.4 38.3 59.5 87.3 1 (a) (b) (c) (d) (e)
117.8 What will come in place of (b) ?
LEARN MATHS FROM S.K. RAJU (9811549822, 9811649822)
12
(1) 11 (2) 7 2 (a) (b) (c) (d) (e) (f)
(3) 9 (4) 8 What will come in place of (d) ?
(5) None of these (1) 156 (2) 164
7. 5 7 10 36 136 690 (3) 168 (4) 152
2 (a) (b) (c) (d) (e) (5) None of these
What will come in place of (e) ? 13. 2 4.5 11 30 93 312 1136
(1) 310 (2) 330 1 (a) (b) (c) (d) (e) (i)
(3) 110 (4) 64 What will come in place of (b) ?
(5) None of these (1) 6 (2) 81
8. 8 4 6 15 52.5 236.25 (3) 16.75 (4) 18.75
4 (a) (b) (c) (d) (e) (5) None of these
14.
Which of the following will come in place of 2 14 18 46 82 176 338
(d) ? 4 (a) (b) (c) (d) (e) (i)
(1) 36.25 (2) 33.25 What will come in place of (e) ?
(3) 26.75 (4) 32.75 (1) 238 (2) 338
(5) None of these (3) 218 (4) 318
Directions (9-10) : In each of the following (5) None of these
questions, a number series is established if the15. 1 3 7 11 21 43 85
positions of two out of the five marked numbers 4 (a) (b) (c) (d) (e) (f)
are in terchanged. The position of the first What will come in place of (f) ?
unmarked number remains the same and it is the (1) 275 (2) 279
beginning of the series. The earlier of the two (3) 277 (4) 273
marked numbers whose positions are interchanged (5) None of these
is the answer. For example, if an interchange of Directions (16-20) : Find out the wrong
number marked ‘ 1’ and the number marked ‘4’ is number in the following given sequence.
required to establish the series, your answer is T. (SBI Associate Banks PO
If it is not necessary to interchange the position of Exam. 21.07.2002
16.
the numbers to establish the series, give 5 as your 7 4 6 9 20 52.5 160.5
answer. Remember that when the series is (1) 6 (2) 4
established, the numbers change from left to right (3) 20 (4) 9
(i.e. from the unmarked number to the last marked (5) 52.5
number) in a specific order. 17. 4 6 12 30 75 315 1260
(SBI Banks PO Exam. 20.08.2000) (1) 315 (2) 75
9. 40 14 60 24 80 19 (3) 12 (4) 6
(1) (2) (3) (4) (5) (5) 30
10. 120 15 105 21.875 87.5 17.5 18. 3 4 13 38 87 166 289
(1) (2) (3) (4) (5) (1) 38 (2) 13
Directions (11-15) : In each of the following(3) 87 (4) 166
number-series only one number is wrong. If the (5) 4
wrong number is corrected, the series gets 19. 4 5 9 29 111 556 3325
established following a certain logic. Below the (1) 5 (2) 9
series a number is given followed by (a), (b), (c), (3) 29 (4)111
(d), (e) and (f). You have to complete the series (5) 556
20.
following the same logic as in the given series after 2 6 16 38 84 176 368
correcting the wrong number, now answer the (1) 6 (2) 16
following questions giving the correct values for (3) 38 (4) 84
the letter in the questions. (5) 176
(SBI Banks PO Exam. 11.02.2001) Directions (21-26): In each of thefollowing
11. 2 3 2 15 76 254 1434 number series, a wrong number is given. Find out
3 (a) (b) (c) (d) (e) the wrong number.
(f)
What will come in place of (c) ? (SBI Banks PO Exam. 18.05.2003)
(1) 18 (2) 22 21. 2 3 6 18 109 194 209952
(3) 24 (4) 21 (1) 3 (2) 6
(5) None of these (3) 18 (4) 109
12. 1 2 8 33 148 740 4626 (5) 1944
22. 1 3 6 11 20 39 70
LEARN MATHS FROM S.K. RAJU (9811549822, 9811649822)
13
(1) 3 (2) 39 (3) 22.5 (4) 20.75
(3) 11 (4) 20 (5) None of these
(5) 6 Directions (32-36): In each of the following
23. 2 13 27 113 561 3369 23581 questions a number series is given. After the series,
(1) 13 (2) 27 a number is given followed by (a), (b), (c), (d) and
(3) 113 (4) 561 (e). You have to complete the series starting with
(5) 3369 the number given following the sequence of the
24. 50 51 47 56 42 65 29 given series. Then answer the question given below
(1) 51 (2) 47 it.
(3) 56 (4) 42 (SBI PO Exam. 26.11.2006)
(5) 65 32. 9 19.5 41 84.5
25. 3 9 23 99 479 2881 20159 12 (a) (b) (c) (d) (e)
(1) 9 (2) 23 Which of the following numbers will come
(3) 99 (4) 479 in place of (c) ?
(5) 2881 (1) 111.5 (2) 118.5
26. 2 4 5 8 13 21 34 (3) 108.25 (4) 106.75
(1) 4 (2) 5 (5) None of these
(3) 8 (4) 13 33. 4 5 22 201
(5) 21 7 (a) (b) (c) (d) (e)
Directions (27-31) : In each of the following Which of the following number will come
questions a number series is given. After the series in place of (d) ?
a number is given followed by (a), (b), (c), (d) and (1) 4948 (2) 4840
(e). You have to complete the series starting with (3) 4048 (4) 4984
the given number, following the sequence of (5) None of these
original series and answer the questions that follow 34. 5 5.25 11.5 36.75
the series. (a) (b) (c) (d) (e)
(SBI PO Exam. 09.01.2005) Which of the following number will come
27. 3 19 103 439 1381 2887 in place of (c) ?
5 (a) (b) (c) (d) (e) (1) 34.75 (2) 24.75
What will come in place of (b) ? (3) 24.5 (4) 34.5
(1) 139 (2) 163 (5) None of these
(3) 161 (4) 157 35. 38 19 28.5 71.25
(5) None of these 18 (a) (b) (c) (d) (e)
28. 4 13 40 135 552 2765 Which of the following number will come
2 (a) (b) (c) (d) (e) in place of (d) ?
What will come in place of (c) ? (1) 118.75 (2) 118.25
(1) 123 (2) 133 (3) 108.25 (4) 118.125
(3) 127 (4) 131 (5) None of these
(5) None of these 36. 25 146 65 114
29. 5 12 4 10 3 8 39 (a) (b) (c) (d) (e)
6 (a) (b) (C) (d) (e) Which of the following number will come
What will come in place of (d) ? in place of (e) ?
(1) 3 (2) 5 (1) 122 (2)119
(3) 4 (4) 7 (3) 112 (4) 94
(5) None of these (5) None of these
30. 3 13 37 87 191 401 Directions (37-41) : In each of these
1 (a) (b) (c) (d) (e) questions a number series is given. Only one
What will come in place of (d) ? number is wrong in each series. You have to find
(1) 169 (2) 161 out the wrong number.
(3) 171 (4) 159 (SBI Associate Banks PO
(5) None of these Exam. 07,01.2007)
31. 8 4 6 15 52.5 236.25 37. 10 15 24 35 54 75 100
12 (a) (b) (c) (d) (e) (1) 35 (2) 75
What will come in place of (c) ? (3) 24 (4) 15
(1) 18.25 (2) 19 (5) 54
LEARN MATHS FROM S.K. RAJU (9811549822, 9811649822)
14
38. 1 3 4 7 11 18 27 47 (5) None of these
(1) 4 (2) 11 49. 4 7 11 18 28 ? 76 12
(3) 18 (4) 7 (1) 59 (2) 38
(5) 27 (3) 46 (4) 53
39. 3 2 3 6 12 37.5 115.5 (5) None of these
(1) 37.5 (2) 3 50. 3 10 ? 172 886 5346 3747
(3) 6 (4) 2 299832
(5) 12 (1) 39 (2) 27
40. 2 8 32 148 765 4626 32431 (3) 24 (4) 34
(1) 765 (2) 148 (5) None of these
(3) 8 (4) 32 51. 15 22 57 183 ? 748 3751 22542
(5) 4626 (1) 709 (2) 698
41. 2 3 11 38 102 229 443 (3) 748 (4) 800
(1) 11 (2) 229 (5) None of these
(3) 102 (4) 38 Directions (52-56) : In each o these
(5) 3 questions a number series is given. In each series
Directions (42-46): What should come in only one number is wrong. Find out the wrong
place of the question mark(?) in the following number.
number series ? (SBI Associate Banks PO
(SBI PO Preliminary (Tire-I) Exam. 07.08.2011)
Exam. 27.04.2008) 52. 3601 3602 1803 604 154 36 12
42. 7413 7422 7440 ? 7503 7548 (1) 3602 (2) 1803
(1) 7464 (2) 7456 (3) 604 (4) 154
(3) 7466 (4) 7477 (5) 36
(5) None of these 53. 4 12 42 196 1005 6066 42511
43. 4 16 36 64 100 ? (1) 12 (2) 42
(1) 120 (2) 180 (3) 1005 (4) 196
(3) 136 (4) 144 (5) 6066
(5) None of these 54. 2 8 12 20 30 42 56
44. 12 33 96 ? 852 2553 (1) 8 (2) 42
(1) 285 (2) 288 (3) 30 (4) 20
(3) 250 (4) 384 (5) 12
(5) None of these 55. 32 16 24 65 210 945 5197.5
45. 70000 14000 2800 ? 112 22.4 (1) 945 (2) 16
(1) 640 (2) 420 (3) 24 (4) 210
(3) 560 (4) 540 (5) 65
(5) None of these 56. 7 13 25 49 97 194 385
46. 102 99 104 97 106 ? (1) 13 (2) 49
(1) 96 (2) 95 (3) 97 (4) 194
(3) 100 (4) 94 (5) 25
(5) None of these Directions (57-61): In each of the following
Directions (47-51) : What will ome in place questions, a number series is given. After the series
of the question mark (?) in the following number a number is given followed by (a), (b), (c), (d) and
series which as only one number wrong by a margin (e). You have to complete the ‘series starting with
of + 1 or - 1 ? The first and last number in the the number given, following the sequence of the
series are correct ? original series and answer the questions that bllow
(SBI PO Preliminary (Tire-I) the series.
Exam. 27.07.2008) (SBI Management Executive
47. 93 95 99 ? 110 121 134 Exam. 23.02.2014)
(1) 104 (2) 96 57. 37 19 20 31.5 65 165
(3) 82 (4) 103 21 (a) (b) (c) (d) (e)
(5) None of these What will come in the place of (e) ?
48. 8 12 18 26 40.5 60.75 136.6875 (1) 105 (2) 41
(1) 104.125 (2) 121.125 (3) 110 (4) 108
(3) 96.125 (4) 83.125 (5) 116

LEARN MATHS FROM S.K. RAJU (9811549822, 9811649822)


15
58. 5 6 16 57 244 1245 (1) 29 (2) 30
9 (a) (b) (c) (d) (e) (3) 26 (4) 28
What will come in the place of (d) ? (5) None of these
(1) 366 (2) 364 5. 5 8 6 10 7 12
(3) 368 (4) 378 7 (a) (b) (c) (d) (e)
(5) 382 What will come in place of (c) ?
59. 7 5 11 49 335 3005 (1) 14 (2) 16
13 (a) (b) (c) (d) (e) (3) 9 (4) 11
What will come in the place of (b) ? (5) None of these
(1) 31 (2) 27 Directions (6-10) : What should come in
(3) 29 (4) 28 place of the question mark (?) in the following
(5) 30 number series ?
60. 12 47 152 467 1412 4247 (RBI Grade-B Officer Exam. 2007)
33 (a) (b) (c) (d) (e) 6. 104 109 99 114 94 9
What will come in the place of (d) ? (1) 69 (2) 124
(1) 3131 (2) 1133 (3) 120 (4) 78
(3) 3311 (4) 3113 (5) None of these
(5) 3123 7. 980 392 156.8 ? 25.088 10.0352
61. 54 50 84 188 496 1456 (1) 65.04 (2) 60.28
42 (a) (b) (c) (d) (e) (3) 62.72 (4) 63.85
What will come the in the place of (d) ? (5) None of these
(1) 304 (2) 286 8. 14 16 35 109 441 ?
(3) 293 (4) 281 (1) 2651 (2) 2205
(5) 301 (3) 2315 (4) 2211
(5) None of these
RBI GRADE-B OFFICER EXAMS 9. 1331 2197 4913 6859 ? 24389
Directions (1-5) : In each of the following (1) 12167 (2) 13824
questions a number series is given. After the series (3) 9261 (4) 15625
a number is given followed by (a), (b) (c), (d) and (5) None of these
(e). You have to complete the series starting with 10. 3600 725 150 35 12 ?
the number given, following the sequence of the (1) 8 (2) 7.4
original series and answer the questions that follow (3) 10.5 (4) 10
the series. (5) None of these
(RBI Grade-B Officer Exam.17.11.2002) Directions (11-15) : What should come in
1. 5 6 16 57 244 1245 place of quesbon mark (?) in the following number
2 (a) (b) (c) (d) (e) series ?
What will come in place of (d) ? (RBI Grade-B Officer Exam. 2008)
(1) 46 (2) 39 11. 13 14 30 93 376 1885 ?
(3) 156 (4) 172 (1) 10818 (2) 10316
(5) None of these (3) 11316 (4) 11318
2. 3 5 9 17 33 65 (5) None of these
7 (a) (b) (c) (d) (e) 12. 4 6 9 13.5 20.25 30.375
What will come in place of (d) (1) 40.25 (2) 45.5625
(1) 95 (2) 51 (3) 42.7525 (4) 48.5625
(3) 99 (4) 49 (5) None of these
(5) None of these 13. 400 240 144 86.4 51.84 31.104
3. 7 4 5 9 20 52.5 ?
3 (a) (b) (c) (d) (e) (1) 19.2466 (2) 17.2244
What will come in place of (c) ? (3) 16.8824 (4) 18.6625
(1) 4.5 (2) 2 (5) None of these
(3) 6 (4) 7 14. 9 4.5 4.5 6.75 13.5 33.75 ?
(5) None of these (1) 101.25 (2) 103.75
4. 3 10 32 111 460 2315 (3) 99.75 (4) 105.50
2 (a) (b) (c) (d) (e) (5) None of these
What will come in place of (b) ? 15. 705 728 774 843 935 1050 ?
LEARN MATHS FROM S.K. RAJU (9811549822, 9811649822)
16
(1) 1190 (2) 1180 (3) 412 (4) 2075
(3) 1185 (4) 1187 (5) 12460
(5) None of these 24. 144 215 540 1890 8505 46777.5
Directions (16-20) : In each of these 304053.75
questions a number series is given. Below the (1) 215 (2) 540
series one number is given followed by (a), (b), (c), (3) 1890 (4) 8505
(d) and (e) You have to complete this series (5) 46777.5
following the same logic as in the original series 25. 2222 1879 1663 1538 1474 1447
and answer the question that tollows. 1440
(RBI Grade-B Officer Exam.11.10.2009) (1) 1879 (2) 1538
16. 5 9 25 91 414 2282 5 (3) 1474 (4) 1447
3 (a) (b) (c) (d) (e) (5) 1440
What will come in place of (c) ? Directions (26 - 30) : What will come in
(1) 63.25 (2) 63.75 place of the question mark (?) in the following
(3) 64.25 (4) 64.75 number series ?
(5) None of these (RBI Grade ‘B’ Officer’s
17. 15 9 8 12 36 170 Exam. 18.12.2011)
19 (a) (b) (C) (d) (e) 26. 9 31 73 141 (?)
What will come in place of (b) ? (1) 164 (2) 280
(1) 18 (2) 16 (3) 239 (4) 241
(3) 22 (4) 24 (5) None of these
(5) None of these 27. 35 256 451 620 763 (?)
18. 7 6 10 27 104 515 (1) 680 (2) 893
9 (a) (b) (c) (d) (e) (3) 633 (4) 880
What will come in place of (d) ? (5) None of these
(1) 152 (2) 156 28. 130 139 155 180 216 (?)
(3)108 (4)112 (1) 260 (2) 290
(5) None of these (3) 265 (4) 996
19. 6 16 57 244 1245 7506 (5) None of these
4 (a) (b) (c) (d) (e) 29. 2890 (?) 1162 874 730 658
What will come in place of (d) ? (1) 1684 (2) 1738
(1) 985 (2) 980 (3) 1784 (4) 1672
(3) 1004 (4) 1015 (5) None of these
(5) None of these 30. 14 1004 1202 1251.5 1268 (?)
20. 8 9 20 63 256 1285 (1) 1267.5 (2) 1276.25
5 (a) (b) (c) (d) (e) (3) 1324.5 (4) 1367.25
What will come in place of (e) (5) None of these
(1) 945 (2) 895 Directions (31-35) : What will come in
(3) 925 (4) 845 place of the question mark (?) in the following
(5) None of these number series ?
Directions (21-2 5): In the following (RBI Officer Grade ‘B’ Online
number series only one number is wrong. Find Exam. 25.08.2013
out the wrong number. 31. 224 576 752 840 884 ?
(RBI Grade-B Officer Exam.06.02.2011) (1) 960 (2) 890
21. 4 3 4.5 8.5 20 53 162.5 (3) 906 (4) 908
(1) 3 (2) 4.5 (5) None of these
(3) 8.5 (4) 20 32. 55 66.15 88.45 121.9 166.5 ?
(5) 53 (1) 212.25 (2) 322.25
22. 12000 2395 472 89.8 12.96 - (3) 224.25 (4) 222.25
2.408 -5.4816 (5) None of these
(1) -5.4816 (2) 472 33. 36 49 75 88 114 (?)
(3) 12.96 (4) - 2.408 (1) 130 (2) 140
(5) 2395 (3) 132 (4) 128
23. 1 8 28 99 412 2075 12460 (5) 127
(1) 28 (2) 99

LEARN MATHS FROM S.K. RAJU (9811549822, 9811649822)


17
(1) 315 (2) 75
(3) 12 (4) 6
INSURANCE EXAMS (5) 30
1. What will come in place of the question 10. 3 4 13 38 87 166 289
mark (?) in the following series ? (1) 38 (2) 13
3 7 18 26 ? 53 64 96 (3) 87 (4) 166
(1) 34 (2) 37 (5) 4
(3) 32 (4) 38 11. 4 5 9 29 111 556 3325
(United India Insurance Co. AAO (1) 5 (2) 9
Exam. 21.04.2002) (3) 29 (4) 111
2. What will come in place of the question (5) 556
mark (?) in the following series ? 12. 2 6 16 38 84 176 368
1.7 3.2 2.7 4.2 3.7 ? 4.7 6.2 (1) 6 (2) 16
(1) 6.2 (2) 5.5 (3) 38 (4) 84
(3) 5.2 (4) 4.3 (5) 176
(United India Insurance Co. Directions (13 - 17) : What should come
AAO Exam. 21.04.2002) in place of the question mark (?) in the following
Directions (3-7) : In each of the following number series ?
questions, a number series in given. Only one (New India Assurance AO
number is wrong in this series. Find out that Exam. 25.10.2009)
wrong number, and taking this wrong number as 13. 3 52 88 113 129 ?
the first term of the second series formed following (1) 128 (2) 142
the same logic, find out the fourth term of the (3) 133 (4) 145
second series. (5) None of these
(LIC Assistant Administrative 14. 2 3 8 ? 112 565
Officer (AAO) Exam. 24.04.2008) (1) 36 (2) 14
3. 8 4 4 6 12 28 90 (3) 27 (4) 45
(1) 18 (2) 42 (5) None of these
(3) 21 (4) 24 15. 6 4 8 23 ? 385.25
(5) None of these (1) 84.5 (2) 73
4. 17 17.25 18.25 20.75 24.5 30.75 (3) 78.5 (4) 82
(1) 23.25 (2) 24.25 (5) None of these
(3) 24,5 (4) 24,75 18. 8 84 216 512 ? 1728
(5) None of these (1) 729 (2) 1331
5. 438 487 447 476 460 469 (3) 684 (4) 1000
(1) 485 (2) 425 (5) None of these
(3) 475 (4) 496 17. 5 11 32 108 444 ?
(5) None of these (1) 1780 (2) 2230
6. 2 7 18 45 99 209 431 (3) 1784 (4) 2225
(1) 172 (2) 171 (5) None of these
(3) 174 (4) 175 18. If S = l 2 - 22 + 32 - 42 + ....+ 1992 - 2002, then
(5) None of these the value of S is
7. 6 8 10 42 146 770 4578 (1) 19900 (2) 20100
(1) 868 (2) 8872 (3) -20100 (4) -19900
(3) 858 (4) 882 (New India Assurance AO
(5) None of these Exam. 25.10.2009)
Directions (8-12) : Find out the wrong 3 5 7 17
number in the following given sequence. 19. Theexpression + + +.... +
4 36 144 5184
(LIC Assistant Administrative
Officer (AAO) Exam. 2006) 19
is equal to
8. 7 4 6 9 20 52.5 160.5 8100
(1) 6 (2) 4 (1) 0.9 (2) 0.95
(3) 20 (4) 9 (3) 0.99 (4) 1.91
(5) 52.5
9. 4 6 12 30 75 315 1260
LEARN MATHS FROM S.K. RAJU (9811549822, 9811649822)
18
Directions (20- 24) : What will come in 28. 1050 510 242 106 46 16 3
place of the question mark (?) in the following (1) 3 (2) 106
number series ? (3) 242 (4) 510
(United India Insurance AO (5) None of these
Exam. 27.03.2011) 29. 550 546 537 521 494 460 411
20. 8 14 32 70 136 (?) (1) 494 (2) 546
(1) 248 (2) 247 (3) 521 (4) 460
(3) 237 (4) 238 (5) None of these
(5) None of these 30. 8 21 47 86 140 203 281
21. 25 41 89 169 281 (?) (1) 47 (2) 86
(1) 425 (2) 415 (3) 140 (4) 203
(3) 409 (4) 419 (5) None of these
(5) None of these 31. 4 24 161 965 4795 19176 57525
22. 461 474 465 478 469 (?) (1) 161 (2) 965
(1) 460 (2) 482 (3) 57525 (4) 19176
(3) 456 (4) 478 (5) None of these
(5) None of these 32. 1 2 8 24 120 720 5040
23. 980 516 284 168 110 (?) (1)120 (2) 24
(1)73 (2)71 (3) 8 (4) 720
(3) 83 (4) 91 (5) None of these
(5) None of these Directions (33-38) : What should come in
24. 4 4 10 34 94 (?) place of the question mark (?) in the following
(1) 230 (2) 214 number series ?
(3) 220 (4) 209 (United India Insurance AO
(5) None of these Exam. 26.05.2013
25. The sum 1 + 3 - 5 + 7 + 9 - 11 +13 + 15- 17 33. 1548 516 129 43 ?
+.....+ 61 + 63 - 65 is equal to (1) 11 (2) 10.75
(1) 319 (2) 330 (3) 9.5 (4) 12
(3) 341 (4) 451 (5) None of these
(New India Insurance 34. 949 189.8 ? 22.776 11.388 6.8328
AAO Exam. 22.05.2011) (1) 48.24 (2) 53.86
1 1 1 1 1 1 1 1 (3) 74.26 (4) 56.94
26. If x = + + + + + + + (5) None of these
2 6 12 20 30 42 56 63
35. 121 144 190 259 ? 466
1 (1) 351 (2) 349
then value of is closest to (3) 374 (4) 328
x
(1) 1.1 (2) 1 (5) None of these
(3) 0.9 (4) 0.8 36. 14 43.5 264 ? 76188
(Ntw India Insurance AAO (1) 3168 (2) 3176
Exam. 22.05.2011) (3) 1587 (4) 1590
(5) None of these
37. 41 164 2624 ? 6045696
 1  1  1   1  (1) 104244 (2) 94644
27. If 1  2  1  2  1  2  .... 1  =
 2  3  4   20112  (3) 94464 (4) 102444
(5) None of these
x 38. Find the missing number in the series :
then the value of x is
2  2011 2, 5, 9, ?, 20, 27
(1) 1 (2) 2010 (1) 14 (2)16
(3) 2011 (4) 2012 (3)18 (4)24
(United India Insurance AAO (NICL (GIC) Administrative
Exam. 03.06.2012) Officer Exam. 15.1.2.2013)
Directions (28 - 32) : Find the wrong
number in the following number series .
(LIC Assistant Administrative Officer
(AAO) Exam. 12.05.2013)

LEARN MATHS FROM S.K. RAJU (9811549822, 9811649822)


19
SHORT ANSWERS 105. (4) 106. (2)
NATIONALISED BANKS 107. (3) 108. (5)
109. (5) 110. (1)
& IBPS PO/MT/SO
111. (3) 112. (4)
1. (4) 2. (1)
113. (5) 114. (1)
3. (4) 4. (3)
115. (2) 116. (3)
5. (2) 6. (3)
117. (2) 118. (1)
7. (5) 8. (2)
119. (1) 120. (2)
9. (1) 10. (4)
121. (2) 122. (5)
11. (3) 12. (5)
123. (3) 124. (3)
13. (2) 14. (1)
125. (3) 126. (2)
15. (4) 16. (1)
127. (5) 128. (3)
17. (5) 18. (1)
129. (4) 130. (3)
19. (1) 20. (2)
131. (3) 132. (2)
21. (4) 22. (3)
133. (4) 134. (5)
23. (2) 24. (4)
135. (4) 136. (1)
25. (1) 26. (5)
137. (2) 138. (1)
27. (3) 28. (1)
139. (3) 140. (1)
29. (3) 30. (2)
141. (4) 142. (5)
31. (5) 32. (4)
143. (3) 144. (3)
33. (1) 34. (3)
145. (3) 146. (5)
35. (2) 36. (3)
147. (2) 148. (3)
37. (5) 38. (5)
149. (1) 150. (3)
39. (3) 40. (3)
151. (5) 152. (4)
41. (2) 42. (1)
153. (2) 154. (1)
43. (3) 44. (5)
155. (2) 156. (4)
45. (4) 46. (1)
157. (4) 158. (1)
47. (2) 48. (1)
159. (3) 160. (2)
49. (4) 50. (5)
161. (4) 162. (1)
51. (3) 52. (2)
163. (1) 164. (4)
53. (5) 54. (1)
165. (5) 166. (4)
55. (3) 56. (2)
167. (4) 168. (2)
57. (4) 58. (3)
169. (5) 170. (4)
59. (5) 60. (1)
171. (1) 172. (1)
61. (2) 62. (4)
173. (3) 174. (5)
63. (5) 64. (4)
175. (4) 176. (1)
65. (3) 66. (2)
177. (5) 178. (3)
67. (1) 68. (3)
179. (2) 180. (3)
69. (4) 70. (1)
181. (5) 182. (3)
71. (5) 72. (3)
183. (4) 184. (5)
73. (5) 74. (1)
185. (4) 186. (4)
75. (4) 76. (2)
187. (1) 188. (5)
77. (3) 78. (2)
189. (3) 190. (2)
79. (4) 80. (1)
191. (4) 192. (1)
81. (5) 82. (3)
193. (2) 194. (5)
83. (3) 84. (3)
195. (2) 196. (4)
85. (3) 86. (2)
197. (1) 198. (3)
87. (3) 88. (1)
199. (2) 200. (4)
89. (5) 90. (3)
201. (3) 202. (5)
91. (1) 92. (4)
203. (4) 204. (2)
93. (3) 94. (1)
205. (1) 206. (2)
95. (5) 96. (4)
207. (4) 208. (3)
97. (3) 98. (1)
209. (1) 210. (2)
99. (2) 100. (2)
211. (2) 212. (1)
101. (4) 102. (5)
213. (5) 214. (4)
103. (1) 104. (3)
LEARN MATHS FROM S.K. RAJU (9811549822, 9811649822)
20
215. (3) 216. (1) 61. (5)
217. (4) 218. (5)
219. (3) 220. (3) RBI GRADE-B OFFICER EXAMS
221. (1) 222. (2) 1. (4) 2. (5)
223. (3) 224. (4) 3. (3) 4. (2)
225. (5) 226. (5) 5. (1) 6. (5)
227. (3) 228. (5) 7. (3) 8. (4)
229. (5) 230. (5) 9. (1) 10. (2)
11. (3) 12. (2)
SBI PO EXAMS 13. (4) 14. (1)
1. (3) 2. (3) 15. (5) 16. (4)
3. (4) 4. (5) 17. (2) 18. (1)
5. (1) 6. (4) 19. (5) 20. (3)
7. (2) 8. (5) 21. (3) 22. (2)
9. (3) 10. (3) 23. (5) 24. (1)
11. (4) 12. (5) 25. (5) 26. (4)
13. (5) 14. (1) 27. (4) 28. (3)
15. (3) 16. (1) 29. (2) 30. (2)
17. (2) 18. (4) 31. (3) 32. (4)
19. (3) 20. (5) 33. (5)
21. (4) 22. (2)
23. (1) 24. (4) INSURANCE EXAMS
25. (3) 26. (1) 1. (2) 2. (3)
27. (2) 28. (1) 3. (3) 4. (2)
29. (3) 30. (4) 5. (1) 6. (5)
31. (3) 32. (3) 7. (4) 8. (1)
33. (1) 34. (2) 9. (2) 10. (4)
35. (4) 36. (3) 11. (3) 12. (5)
37. (1) 38. (5) 13. (5) 14. (3)
39. (4) 40. (4) 15. (1) 16. (4)
41. (2) 42. (5) 17. (2) 18. (3)
43. (4) 44. (1) 19. (3) 20. (4)
45. (3) 46. (2) 21. (1) 22. (2)
47. (4) 48. (5) 23. (5) 24. (5)
49. (5) 50. (1) 25. (1) 26. (1)
51. (3) 52. (4) 27. (4) 28. (2)
53. (2) 54. (1) 29. (1) 30. (3)
55. (3) 56. (4) 31. (2) 32. (3)
57. (1) 58. (2) 33. (2) 34. (4)
59. (3) 60. (4) 35. (1) 36. (5)
37. (3) 38. (1)

LEARN MATHS FROM S.K. RAJU (9811549822, 9811649822)


21
EXPLANATIONS 8. (2) The given series is based on the following
NATIONALISED BANKS pattern:
5 × 1 + (1)2 = 6
& IBPS PO/MT/SO
6 × 2 + (2)2 = 16
1. (4) The series is based on the following
16 × 3 + (3)2 = 57
pattern:
57 × 4 + (4)2 = 244
11 = 2 × 3 + 5
Hence, 16 will come in place of question
38 = 11 × 4 - 6
mark.
197 = 38 × 5 + 7
9. (1) The given series is based on the following
1172  197 × 6 - 8
patterns.
 1172 is wong and it should be replaced
by 197 × 6 - 8 = 1174
2. (1) The series is based on the following
pattern:
107 - 71 = 36 = 62
71 - 46 = 25 = 52 Hence, 34 will come in place of question
46 - 30 = 16 = 42 mark.
30 - 21 = 9 = 32 10. (4) The given series is based on the following
21 - 19 = 2  22 pattern:
 19 I should be replaced by 17 for which 5 × 2 + 1 = 11
21 - 17 = 22 11 × 2 + 3 = 25
3. (4) The series is based on the following 25 × 2 + 5 = 55
pattern: 55 × 2 + 7 = 117
16 = 9 + 7 11. (3) The given series is based on the following
25 = 16 + 9 pattern:
41 = 25 + 16 30 = 12 × 6 - 7 × 6
68  41 + 25 120 = 30 × 5 - 6 × 5
4. (3) The series is based on the following 460 = 120 × 4 - 5 × 4
pattern: 1368 = 460 × 3 - 4 × 3
2730 = 1368 × 2 - 3 × 2
Similarly,
(a) = 16 × 6 - 7 × 6 = 96 - 42 = 54
Obviously, 3.5 Is the wrong number which (b) = 54 × 5 - 6 × 5 = 240
should be replaced by 3. (c) = 240 × 4 - 5 × 4 = 940
5. (2) The series is based on the following (d) = 940 × 3 - 4 × 3 = 2808
pattern: Hence, 2808 will come in place of (d).
12. (5) The given series is based on the following
pattern:

Obviously, 1.75 is the wrong number which


should be replaced by 1.5. Similarly,
6. (3) The given series is based on the following
pattern:

Hence, 1863 will come in place of (e).


13. (2) The given series is based on the following
Hence, 308 will come in place of question pattern:
mark.
7. (5) The given series is based on the following
pattern:
Similarly,

Hence, 10 will come in place of question


mark.
LEARN MATHS FROM S.K. RAJU (9811549822, 9811649822)
22
Hence, 14514.5 will come in place of (c). Hence, 81 will come in place of the question
14. (1) The given series is based on the following mark.
pattern : 21. (4) The given series is based on the following
pattern:

Similarly,
Hence, 19 will come in place of the question
mark.
22. (3) The given series is based on the following
pattern :
Hence, 284 will come in place of(d).
15. (4) The given series is based on the following
pattern: Hence, 1260 will come in place of the
question mark.
23. (2) The given number series is based on the
Similarly, following pattern :

Hence, 10.75 will replace the quesdon


Hence, 97.5 will come in place of (c). mark.
16. (1) The given series is based on the following 24. (4) The given number series is based on the
pattern : following pattern :
1 = l3 ? = 23= 8
3
27 = 3 64 = 43
3
125 = 5
Hence, 8 will come in place of the question Hence, 56.94 will replace the question
mark. mark.
17. (5) The given series is based on the following 25. (1) The given number series is based on the
pattern : following pattern :
25 = 52 16 = 42 121 + 23 × 1 = 144
?=3 =92
4 = 22 and 144 + 23 × 2 = 190
1=1 2 190 + 23 × 3 = 259
Hence, 9 will come in place of the question  ? = 259 + 23 × 4
mark, = 259 + 92 = 351
18. (1) The given series Is based on the following Hence, 351 will replace the question mark.
pattern: 26. (5) The given number series is based on the
1 × 2 +2 × 2 = 6 following pattern :
6 × 4 + 4 × 3 = 36 14 × 3 + 1.5 = 43.5
36 × 6 + 6 × 4 = 240 43.5 × 6 + 1.5 × 2 = 264
240 × 8 + 8 × 5 = 1960 264 × 12 + 1.5 × 4= 3174
1960 × 10 + 10 × 6 = 19660 3174 × 24 + 1.5 × 8 = 76188
Hence, 19660 will come in place of the Hence, 3174 will replace the question mark.
quesdon mark. 27. (3) The given number series is based on the
19. (1) The given series is based on the following following pattern :
pattern : 41 × 22 = 164
164 × 42 = 2624
2624 × 62 = 94464
94464 × 82 = 6045696
Hence, 14 will come in place ol the question
Hence, 94464 will replace the question
mark.
mark.
20. (2) The given series is based on the following
28. (1) The given number series is based on the
pattern :
following pattern :
2+5=7 7 + 5 = 12
12 × 1 = 12
12 + 7 = 19 19 + 12 = 31
12 × 1.5 = 18
31 + 19 = 50 50 + 31 = 81

LEARN MATHS FROM S.K. RAJU (9811549822, 9811649822)


23
18 × (1 + 1.5) = 18 × 2.5 = 45 34. (3) The given number series is based on the
45 × (1.5 + 2.5) = 45 × 4 = 180 following pattern :
180 × (4 + 2.5) = 180 × 6.5 = 1170 14 × 1 - 2 = 14 - 2 = 12
 ? = 1170 × (4 + 6.5) = 12285 12 × 2 - 3 = 24 - 3 = 21
Hence, 12285 will replace the quesdon 21 × 3 - 4 = 63 - 4 = 59
mark. 59 × 4 - 5 = 236 - 5 = 231
29. (3) The given number series is based on the 231 × 5 - 6 = 1155 - 6 = 1149
following pattern :  ? = 1149 × 6 - 7
467 - 444 = 23 = 23 × 1 = 6894 - 7 = 6887
513 - 467 = 46 = 23 × 2 Hence, 6887 will replace the question mark.
582 - 513 = 69 = 23 × 3 35. (2) The given number series is based on the
674 - 582 = 92 = 23 × 4 following pattern :
789 - 674 = 115 = 23 × 5 12 × 12 × 12 = 1728
 ? = 789 + 23 × 6 14 × 14 × 14 = 2744
= 789 + 138 = 927 16 × 16 × 16 = 4096
Hence, 927 will replace the question mark. 18 × 18 × 18 = 5832
30. (2) The given number series is based on the 20 × 20 × 20 = 8000
following pattern : 22 × 22 × 22 = 10648
1 = l4 ; 16 = 24;  ? = 24 × 24 × 24 = 13824
81 = 34; 256 = 44; Hence, 13824 will replace the question
625 = 54; 1296 = 64; mark.
 ? = 7 4
= 7 × 7 × 7 × 7 36. (3) The given number series is based on the
= 2401 following pattern :
Hence, 2401 will replace the question mark. 120  8 = 7
31. (5) The given number series is based on the 15 × 7 = 105
following pattern : 105  6 = 17.5
23 × 1 + 2 = 25 17.5 × 5 = 87.5
25 × 2 + 3 = 53  = 87.5  4 = 21.875
53 × 3 + 4 = 163 Hence, 21.875 will replace the question
163 × 4 + 5 = 657 mark.
657 × 5 + 6 = 3291 37. (5) The given number series is based on the
 ? = 3291 × 6 + 7 following pattern :
= 19746 + 7 = 19753
Hence, 19753 will replace the question
mark.
32. (4) The given number series is based on the Hence, 105 will replace the question mark.
following pattern : 38. (5) The given number series is based on the
13 × 1 = 13 following pattern :
13 × 5 = 65 487.5 - 357.5 = 130
65 × 9 = 585 357.5 - 247.5 = 110
585 × 13 = 7605 247.5 - 157.5 = 90
7605 × 17 = 129285 157.5 - 87.5 = 70
 ? = 129285 × 21 = 2714985 87.5 - 47.5 = 40
Hence, 2714985 will replace the question 87.5 - 37.5 = 50
mark. 37.5 - 7.5 = 30
33. (1) The given number series is based on the Clearly, 47.5 is the wrong number. It
following pattern : should be replaced by 37.5.
40280625  55 = 732375 39. (3) The given number series is based on the
732375  45 = 16275 following pattern :
16275    35 = 465 13 + 3 = 16
465  25 = 18.6 16 + 5 = 21
18.6  15 = 1.24 21 + 7 = 28  27
 ? = 1.24  5 = 0.248 28 + 11 = 39
Hence, 0.248 will replace the question 39 + 13 = 52
mark. 52 + 17 = 69
LEARN MATHS FROM S.K. RAJU (9811549822, 9811649822)
24
Clearly, 27 is the wrong num­ber. It should 48 × 2.5 = 120
be replaced by 28. 120 × 3 = 360
40. (3) The given number series is based on the 360 × 3.5 = 1260
following pattern :  ? = 1260 × 4 = 5040
1500 + 81 = 1581 46. (1) The given number series is based on the
1581 + 83 = 1664 following pattern :
1664 + 85 = 1749 8 × 4 - 1= 32 - 1 = 31
1749 + 87 = 1836  1833 31 × 4 - 2 = 124 - 2 = 122
1836 + 89 = 1925 122 × 4 - 3 = 488 - 3 = 485
1925 + 91 = 2016 485 × 4 - 4 = 1940 - 4 = 1936
Clearly, 1833 is the wrong number. It 1936 × 4 - 5 = 7744 - 5 = 7739
should be replaced by 1836.  ? = 7739 × 4 - 6
41. (2) The given number series is based on the = 30956 - 6 = 30950
following pattern : 47. (2) The given number series is based on the
66 + 25 = 91 following pattern :
91 + 29 = 120 499 + 1 × 123 = 622
120 + 33 = 153 622 + 2 × 123 = 868
153 + 37 = 190 868 + 3 × 123 = 1237
190 + 41 = 231  233 1237 + 4 × 123 = 1729
231 + 45 = 276 1729 + 5 × 123 = 2344  
Clearly, 233 is the wrong number. It should  ? = 2344 + 6 × 123
be replaced by 231. = 2344 + 738 = 3082
42. (1) The given number series is based on the 48. (1) The given number series is based on the
following pattern : following pattern
11 × 11 × 11 = 1331 l 1 = 1; 22 = 4
13 × 13 × 13 = 2197 33 = 27; 44 = 256
15 × 15 × 15 = 3375 55 = 3125; 66 = 46656
17 × 17 × 17 = 4913  4914 Hence 46658 is the wrong number.
19 × 19 × 19 = 6859 49. (4) The given number series is based on the
Clearly, 4914 is the wrong number. It following pattern  
should be replaced by 4913. 18000  5 = 3600
43. (3) The given number series is based on the 3600  5 = 720
following pattern : 720  5 = 144  142.2
20 + 22 = 24 144  5 = 28.3
24 + 32 = 33 28.8  5 = 5.76
33 + 42 = 49 Hence 142.2 is the wrong number.
49 + 52 = 74 5O. (5) The given number series is based on the
74 + 62 = 110 following pattern :
 ? = 110 + 72 12 + 152 = 12 + 225 = 237
= 110 + 49 = 159 237 + 132 = 237 + 169 = 406
44. (5) The given number series is based on the 406 + 112 = 406 + 121 = 527
following pattern : 527 + 81 = 608
529 = 23 × 23 608 + 72 = 608 + 49 = 657
841 = 29 × 29 Hence 604 is the wrong number.
961 = 31 × 31 51. (3) The given number series is based on the
1369 = 37 × 37 following pattern :
1681 = 41 × 41 3 × 7 + 2 × 7 = 21 + 14 = 35
1849 = 43 × 43 35 × 6 + 3 × 6 = 210 + 18
 ? = 47 × 47 = 2209 = 228  226
Here, the numbers are formed by squaring 228 × 5 + 4 × 5 = 1140 + 20 = 1160
the prime numbers greater than 23. 1160 × 4 + 5 × 4 = 4640 + 20 = 4660
45. (4) The given number series is based on the 4660 × 3 + 6 × 3 = 13980 + 18 = 13998
following pattern : Hence 226 is the wrong number
16 × 1.5 = 24 52. (2) The given number series i based on the
24 × 2 = 48 following pattern :

LEARN MATHS FROM S.K. RAJU (9811549822, 9811649822)


25
18 × 7 - 7 = 126 - 7 = 119 = 7350
119 × 6 - 6 = 714 - 708 61. (2) 23 = 8 : 33 = 27
708 × 5 - 5 = 3540 - 5 = 3535  3534 43 = 64 : 53 = 125
3535 × 4 - 4 = 14140 - 4 = 14136 63 = 216  218
Hence 3534 is the wrong number. 73 = 343
53. (5) 5 + 22 = 5 + 4 = 9 62. (4) 19 + 72 = 19 + 49 = 68
9 + 32 = 9 + 9 = 18 68 + 62 = 68 + 36 = 104  102
18 + 42 = 18 + 16 = 34 104 + 52 = 104 + 25 = 129
34 + 52 = 34 + 25 = 59 129 + 42 = 129 + 16 = 145
59 + 62 = 59 + 36 = 95 145 + 32 = 145 + 9 = 154
 ? = 95 + 72 = 95 + 49 = 144 63. (5)
54. (1) 1200  2.5 = 480 0 +5 = 5
480  2.5 = 192 5 + 13 = 18
192  2.5 =76.8 18 + 25 = 43
76.8  2.5 = 30.72 43 + 41 = 84
30.72  2.5 = 12.288 84 + 61 = 145
 ? = 12.288  2.5 = 4.9152  ? = 145 + 85 = 230
55. (3) 963 - 1 × 36 = 963 - 36 = 927 64. (4) 10 × 1 + 1 × 7 = 10 + 7 = 17
927 - 2 × 36 = 927 - 72 = 855 17 × 2 + 2 × 7 = 34 + 14 = 48
855 - 3 × 36 = 855 - 108 = 747 48 × 3 + 3 × 7 = 144 + 21 = 165
747 - 4 × 36 = 747 - 144 = 603 165 × 4 + 4 × 7 = 660 + 28 = 688
603 - 5 × 36 = 603 - 180 = 423 688 × 5 + 5 × 7 = 3440 + 35 = 3475
 ? = 423 - 6 × 36 = 423 - 216 = 207  ? = 3475 × 6 + 6 × 7
56. (2) 29 × 29 = 841 = 20850 + 42 = 20892
31 × 31 = 961 65. (3) 1 × 3 = 3
33 × 33 = 1089 3 × 8 = 24
35 × 35 = 1225 24 × 15 = 360
37 × 37 = 1369 360 × 24 = 8640
39 × 39 = 1521 8640 × 35 = 302400
 ? = 41 × 41 = 1681  ? = 302400 × 48
57. (4) 18 × 1 + 2 = 18 + 2 = 20 = 14515200
20 × 2 + 4 = 40 + 4 = 44 66. (2) 12 × 1 + 2 × 1 = 12 + 2 = 14
44 × 3 + 6 = 132 + 6 = 138 14 × 2 + 2 × 2 = 28 + 4 = 32
138 × 4 + 8 = 552 + 8 = 560 32 × 3 + 2 × 3 = 96 + 6 = 102
560 × 5 + 10 = 2800 + 10 = 2810 102 × 4 + 2 × 4 = 408 + 8 = 416
 ? = 2810 × 6 + 12 = 16860 + 12 = 16872 416 × 5 + 2 × 5 = 2080 + 10
58. (3) 4 × 1 + 2 = 4 + 2 = 6 = 2090
6 × 2 + 3 = 12 + 3 = 15  18  ? = 2090 × 6 + 2 × 6
15 × 3 + 4 = 45 + 4 = 49 = 12540 + 12 = 12552
49 × 4 + 5 = 196 + 5 = 201 3
201 × 5 + 6 = 1005 + 6 = 1011 67. (1) 10 × = 15
2
3 3
59. (5) 48 × = 72; 72 × = 108 4
2 2 15 × = 15
4
3 3
108 × = 162: 162 × = 243 5
2 2 15 × = 12.5
6
3
243 × = 364.5  366 6
2 12.5 × = 9.375
8
60. (1) 2 × 6 + 7 × 6 = 12 + 42 = 54
54 × 5 + 6 × 5 = 270 + 30 = 300 7
9.375 × = 6.5625
300 × 4 + 5 × 4 = 1200 + 20 = 1220 10
1220 × 3 + 4 × 3 = 3660 + 12
8
= 3672  3674  ? = 6.5625 × 12 = 4.375
3672 × 2 + 3 × 2 = 7344 + 6
LEARN MATHS FROM S.K. RAJU (9811549822, 9811649822)
26
68. (3) The pattern of the number series is : 6 × 2.5 + 2.5 = 15 + 2.5 = 17.5
17 × 3 + 1 = 51 + 1 = 52 17.5 × 3.5 + 3.5 = 61.25 + 3.5 = 64.75
52 × 3 + 2 = 156 + 2 = 158 78. (2) The pattern is :
158 × 3 + 3 = 474 + 3 = 477 16 × 0.5 = 8 8 × 1.5 = 12
477 × 3 + 4 = 1431 + 4 = 1435 12 × 2.5 = 30 30 × 3.5 = 105
69. (4) The pattern of the number series is : 79. (4) The pattern is :
3 × 7 + 1 = 21 + 1= 22 5 × 1+1=6
22 × 6 + 2 = 132 + 2 = 134 6 × 2 + 2 = 14
134 × 5 + 3 = 670 + 3 = 673 14 × 3 + 3 = 45
673 × 4 + 4 = 2692 + 4 = 2696 45 × 4 + 4 = 184
70. (1) The pattern of the number series is : 80. (1) The pattern is :
6 × 1 + 1 × 7 = 6 + 7 = 13 7 ×1 + 1 × 5 = 12
13 × 2 + 2 × 6 = 26 + 12 = 38 12 × 2 + 2 × 4 = 32
38 × 3 + 3 × 5 = 114 + 15 = 129 32 × 3 + 3 × 3 = 105
129 × 4 + 4 × 4 = 516 + 16 = 532 105 × 4 + 4 × 2 = 428
71. (5) The pattern of the number series is : 81. (5) The pattern is :
286 11 × 2 + 1 = 23
- 1 = 143 - 1 = 142 23 × 2 + 1 = 47
2
47 × 2 + 1 = 95
142 95 × 2 + 1 = 191
- l = 71 - l = 70 82. (3) The pattern is :
2
9 × 2 - 1 = 17
70 17 × 2 - 1 = 33
- 1 = 35 - 1 = 34
2 33 × 2 - 1 = 65
65 × 2 - 1 = 129
34
- 1 = 17- 1 = 16 83. (3) The pattern of the number series is :
2 8 + 3 =11
72. (3) The pattern of the number series is : 11 + 32 = 11 + 9 = 20  17
17 × 0.5 + 0.5 = 9 20 + 33 = 20 + 27 = 47
9 × 1 + 1 = 10 47 + 34 = 47 + 81 = 128
10 × 1.5 + 1.5 = 16.5 128 + 35 = 128 + 243 = 371
16.5 × 2 + 2 = 35 84. (3) The pattern of the number series is :
73. (5) The pattern is : 1 + 22 = 1 + 4 = 5
2 × 3 + 2= 6 + 2 = 8 5 + 23 = 5 + 8 = 13
8 × 3 + 2 = 24 + 2 = 26 13 + 24 = 13 + 16 = 29  31
26 × 3 + 2 = 78 + 2 = 80 29 + 25 = 29 + 32 = 61
80 × 3 + 2 = 240 + 2 = 242 61 + 26 = 61 + 64 = 125
74. (1) The pattern is : 85. (3) The pattern is :
3 × 1 + l2 = 3 + 1 = 4 150 × 2 - 1 × 10
4 × 2 + 22 = 8 + 4= 12 = 300 - 10 = 290
12 × 3 + 32 = 36 + 9 = 45 290 × 2 - 2 × 10
45 × 4 + 42 = 180 + 16 = 196 = 580 - 20 = 560
75. (4) The pattern is : 560 × 2 - 3 × 10 = 1120 - 30
9 × 2 - 1 = 18 - 1 = 17 = 1090  1120
17 × 2 - 1 = 34 - 1 = 33 1090 × 2 - 4 × 10 = 2180 - 40 = 2140
33 × 2 - 1 = 66 - 1 = 65 2140 × 2 - 5 × 10 = 4280 - 50 = 4230
65 × 2 - 1 = 130 - 1 = 129 86. (2) The pattern is : 10 × 1 - 2 = 8
76. (2) The pattern is : 8 × 2 - 3 = 13
7 × 2 - 1 = 14 - 1 = 13 13 × 3 - 4 = 35
13 × 2 - 1 = 26 - 1 = 25 35 × 4 - 5 = 135
25 × 2 - 1 = 50 - 1 = 49 135 × 5 - 6 = 675 - 6
49 × 2 - 1 = 98 - 1 = 97 = 669  671
77. (3) The pattern is : 669 × 6 - 7 = 4014 - 7 = 4007
5 × 0.5 + 0.5 = 2.5 + 0.5 = 3 87. (3) The pattern is :
3 × 1.5 + 1.5 = 4.5 + 1.5 = 6 (80  2) + 2 = 40 + 2 = 42

LEARN MATHS FROM S.K. RAJU (9811549822, 9811649822)


27
(42  2) + 2 = 21 + 2 = 23  24 131 + 32= 140
(23  2) + 2 = 11.5 + 2 = 13.5 140 + 22 = 140 + 4 = 144
(13.5  2) + 2 = 6.75 + 2 = 8.75 96. (4) The pattern is :
(8.75  2) + 2 = 4.375 + 2 = 6.375 7 × 0.5 + 0.5 = 3.5 + 0.5 = 4
88. (1) The pattern is : 4 × 1 + 1= 4 + 1 = 5
3 5 × 1.5 + 1.5 = 7.5 + 1.5 = 9
125 × = 75 9 × 2 + 2 = 18 + 2 = 20
5
97. (3) The pattern is :
3 6 × 7 = 42
75 × = 45 42 × 6 = 252
5
252 × 5 = 1260
3 98. (1) The pattern is:
45 × = 27  25
5 4 × 5 - 10 = 10
3 10 × 5 - 10 = 40
27 × = 16.2 40 × 5 - 10 = 190
5
190 × 5 - 10 = 940
3 940 × 5 - 10 = 4700 - 10
16.2 × = 9.72
5 = 4690
89. (5) The pattern is : 99. (2) The pattern is :
29 + 1 × 8 = 37 2 × 1 +1 × 7 = 9
37 - 2 × 8 = 37 - 16 = 21 9 × 2 + 2 × 6 = 30
21 + 3 × 8 = 21 + 24 = 45  43 30 × 3 + 3 × 5 = 105
45 - 4 × 8 = 45 - 32 = 13 105 × 4 + 4 × 4 = 436
13 + 5 × 8 = 13 + 40 = 53 436 × 5 + 5 × 3 = 2195
53 - 6 × 8 = 53 - 48 = 5 100. (2) The pattern of the numbe series is :
90. (3) The pattern is: (484  2) - 2 = 242 - 2 = 240
13 + 12 = 25 ; 25 + 15 = 40 (240  2) - 2 = 120 - 2 = 118 = 120
40 + 18 = 58  57 (118  2) - 2 = 59 - 2 = 57
58 + 21 = 79 (57  2) - 2 = 28.5 - 2 = 26 5
91. (1) The pattern is : 101. (4) The pattern of the number series is :
850 - 200 = 650  600 3 × 1 + 2 = 5
650 - 100 = 550 5 × 2 + 3 = 13
550 - 50 = 500 13 × 3 + 4 = 43
500 - 25 = 475 43 × 4 + 5 = 177  176
475 - 12.5 = 462.5 177 × 5 + 6 = 891
92. (4) The pattern is: 102. (5) The Pattern of the number series is :
2 × 3 = 6  10 6 + l2 = 6 + 1 = 7
6 × 3 = 18 ; 18 × 3 = 54 7 + 32 = 7 + 9 = 16
54 × 3 = 162 16 + 52 = 16 + 25 = 41
93. (3) The pattern is: 41 + 72 = 41 + 49 = 90
8 + 4 × 1 = 12; 12 + 4 × 3 = 24 90 + 92 = 90 + 81 = 177  154
24 + 4 × 5 = 44  46 171 + 11 2= 171 + 121 = 292
44 + 4 × 7 = 72 103. (1) The pattern of the number series is :
72 + 4 × 9 = 108 5 × 1 + 12 = 6  7
94. (1) The pattern is : 6 × 2 + 22 = 16
142 - 23 = 119 ; 119 - 19 = 100 16 × 3 + 32 = 57
100- 17 = 83 57 × 4 + 42 = 228 + 16 = 244
83 - 13 = 70  65 244 × 5 + 52 = 1220 + 25 = 1245
70 - 11 = 59 104. (3) The pattern of the number series is :
59 - 7 = 52 4 × 0.5 + 0.5 = 2 + 0.5 = 2.5
95. (5) The pattern is : 2.5 × 1 + 1 = 3.5
5 + 72 = 54 3.5 × 1.5 + 1.5 = 6.75 =  65
54 + 62 = 90 6.75 × 2 + 2 = 15.5
90 + 52 = 115 15.5 × 2.5 + 2.5 = 38.75 + 25 = 41.25
115 + 42 = 131 41.25 × 3 + 3 = 123.75 + 3 = 126.75
LEARN MATHS FROM S.K. RAJU (9811549822, 9811649822)
28
105. (4) The pattern of the number series is : 114. (1) The pattern of the number series is :
325- 1 × 11 = 314 37 + 1 × 5 = 42  47
314 - 2 × 11 = 292 42 + 2 × 5 = 52
292 - 3 × 11 = 259 52 + 3 × 5 = 67
259 - 4 × 11 = 215 67 + 4 × 5 = 87
215 - 5 × 11 =  160 87 + 5 × 5 = 112
106. (2) The pattern of the number series is : 112 + 6 × 5 = 142
45 × 1 + 1 = 46 115. (2) The pattern of the number series is :
46 × 1.5 + 1 = 70 13 + 3 = 16
70 × 2 + 1 = 141 16 + (3 + 3) = 22
141 × 2.5 + 1 22 + (6 + 5) = 33
= 352.5 + 1 = 353.5 33 + (11 + 7) = 51
107. (3) The pattern of the number series is : 51 + (18 + 9) = 78
620 + 1 × 12 = 632 116. (3) The pattern of the number series is :
632 - 2 × 12 = 608 39 + 1 × 13 = 52
608 + 3 × 12 = 644 52 + 2 × 13 = 78
644 - 4 + 12 = 596 78 + 3 × 13 = 117
596 + 5 × 12 = j 656 j 117 + 4 × 13 = 169
108. (5) The pattern of the number series is : 169 + 5 × 13 = 234
15 × 2 - 1 × 5 = 25 117. (2) The pattern of the number series is :
25 × 2 - 2 × 5 = 40 62 + 52 = 62 + 25 = 87
40 × 2 - 3 × 5 = 65 87 + 102 = 87 + 100 = 187
65 × 2 - 4 × 5 = 110 187 + 152 = 187 + 225 = 412
110 × 2 - 5 × 5 = 195 412 + 202 = 412 + 400 = 812
109. (5) The pattern of the number series is : 812 + (25)2 = 812 + 625 = 1437
120 × 2.5 + 20 = 320 118. (1) The pattern of the number series is :
320 × 2.5 + 20 = 820 7 + l2 = 8
820 × 2.5 + 20 = 2070 8 + 42 = 24
2070 × 2.5 + 20 = 5195 24 + 92 = 105
110. (1) The pattern of the number series is : 105 + 162 = 361
32 + l 2 = 32 + 1 = 33  34 361 + 252 = 986
33 + 22 = 33 + 4 = 37 119. (1) The pattern of the number series is :
37 + 32 = 37 + 9 = 46 656 - 224 = 432
46 + 42 = 46 + 16 = 62 432 - 112 = 320
62 + 52 = 62 + 25 = 87 320 - 56 = 264
111. (3) The pattern of the number series is : 264 - 28 = 236
7 + 1 × 11 = 7 + 11 = 18 236 - 14 = 222
18 + 3 × 11 = 18 + 33 = 51  40 120. (2) The pattern of the number series is :
51 + 5 × 11 = 51 + 55 = 106 7 × 2 + 6 = 20
106 + 7 × 11 = 106 + 77 = 183 20 × 2 + 6 = 46
183 + 9 × 11 = 183 + 99 = 282 46 × 2 + 6 = 98
112. (4) The pattern of the number series is : 98 × 2 + 6 = 202
850 - 1 × 7 = 843 202 × 2 + 6 = 404 + 6 = 410
843 - 2 × 7 = 829 121. (2) The pattern of the number series is :
829 - 3 × 7 = 808 210 - l 3 = 209
808 - 4 × 7 = 780  788 209 + 22 = 213
780 - 5 × 7 = 745 213 - 33 = 186
745 - 6 × 7 = 703 186 + 42 = 202
113. (5) The pattern of the number series is : 202 - 53 = 202 - 125 = 77
33 + 288 = 321 122. (5) The pattern of the number series is :
321 + 144 = 465 27 + 11 = 38
465 + 72 = 537 38 + 33 = 71
537 + 36 = 573 71 + 55 = 126
573 + 18 = 591  590 126 + 77 = 203
591 + 9 = 600 203 + 99 = 302

LEARN MATHS FROM S.K. RAJU (9811549822, 9811649822)


29
123. (3) The pattern of the number series is : 1 × 1 + 1 = 2
435 - 9 × 9 = 354 2 × 2 + 2 = 6
354 - 9 × 8 = 282 6 × 3 + 3 = 21
282 - 9 × 7 = 219 21 × 4 + 4 = 88
219 - 9 × 6 = 165 88 × 5 + 5 = 445
165 - 9 × 5 = 120 445 × 6 + 6 = 2676
124. (3) The paatem of the number series is : 133. (4) The pattern of the number series is :
4 + 142 = 4 + 196 = 200 20 + 12 = 21
200 + 132 = 200 + 169 = 369 21 + 22 = 25
369 + 122 = 369 + 144 = 513 25 + 32 = 34
513 + 112 = 513 + 121 = 634 34 + 42 = 50
634 + 102 = 634 + 100 = 734 50 + 52 = 75
125. (3) The pattern of the number series is : 134. (5) The pattern of the number series is :
495 - 1 × 10 = 485 600
485 - 2 × 10 = 465 + 5 = 125
5
465 - 4 × 10 = 425
425 - 8 × 10 = 345 125
345 - 16 × 10 = 185 + 5 = 30
5
126. (2) The pattern of the number series is :
16 + 6 = 22 30
+ 5 = 11
22 + 11 = 33 5
33 + 16 = 49 11
49 + 21 = 70 + 5 = 7.2
5
70 + 26 = 96
135. (4) The pattern of the number series is :
127. (5) The pattern of the number series is :
11 + 22 = 11 + 4 = 15
32 + 22 = 36
15 + 42 = 15 + 16 = 31
36 + 42 = 52
31 + 62 = 31 + 36 = 67
52 + 62 = 88
67 + 82 = 67 + 64 = 131
88 + 82 = 152
131 + 102 = 131 + 100 = 231
152 + 102 = 252
136. (1) The pattern of the number series is :
128. (3) The pattern of the number series is :
483 - 1 × 12 = 483 - 12 = 471
17 + 272 = 289
471 - 3 × 12 = 471 - 36 = 435
289 + 136 = 425
435 - 5 × 12 = 435 - 60 = 375
425 + 68 = 493
375 - 7 × 12 = 375 - 84 = 291
493 + 34 = 527
291 - 9 × 12 = 291 - 108 = 183
527 + 17 = 544
137. (2) The pattern of the number series is :
129. (4) The pattern of the numbe series is :
5 + 1 × 2 = 7
13 + 1 × 14 = 27
7 + 2 × 3 = 13
27 + 2 × 14 = 55
13 + 3 × 4 = 25
55 + 3 × 14 = 97
25 + 4 × 5 = 45
97 + 4 × 14 = 153
45 + 5 × 6 = 75
153 + 5 × 14 = 223
138. (1) The pattern of the number series is :
130. (3) The pattern of the number series is :
4 +1 × 7 = 11
50 × 1.2 = 60
11 + 2 × 7 = 25
60 × 1.25 = 75
25 + 4 × 7 = 53
75 × 1.3 = 97.5
53 + 8 × 7 = 109
97.5 × 1.35 = 131.625
109 + 16 × 7 = 109 + 112 = 221
131.625 × 1.4 = 184.275
139. (3) The pattern of the number series is :
131. (3) The pattern of the number series is :
15 + 6 × 1 = 21
12 × 1 + 3 × 1 = 15
21 + 6 × 2 = 33
15 × 2 + 3 × 2 = 36
33 + 6 × 3 = 51
36 × 3 + 3 × 3 = 117
51 + 6 × 4 = 75
117 × 4 + 3 × 4 = 480
75 + 6 × 5 = 105
480 × 5 + 3 × 5 = 2415
140. (1) The pattern of the number series is :
132. (2) The pattern of the number series is :
LEARN MATHS FROM S.K. RAJU (9811549822, 9811649822)
30
5 + 73 = 5 + 343 = 348 5 × 3 = 15
348 + 63 = 348 + 216 = 564 15 × 5 = 75
564 + 53 = 564 + 125 = 689 75 × 7 = 525
689 + 43 = 689 + 64 = 753, not 716 525 × 9 = 4725
753 + 33 = 753 + 27 = 780 149. (1) The pattern of the number series is :
141. (4) The pattern of the number series is : 1
4444 52 × = 26
2
+ 2 = 2224
2 26 × 1 = 26
2224 3
+ 2 = 1114 26 × = 39
2 2
1114 39 × 2 = 78
+ 2 = 559 not 556 5
2
78 × = 195
2
559
+ 2 = 281.5 150. (3) The pattern of the number series is :
2
14 - 10 = 4
142. (5) The pattern of the number series is : 25 - 14 = 11 = 4 × 3 -1
4.5 + 11.5 = 16 55 - 25 = 30 = 11 × 3 - 3
16 + 9.5 = 25.5, not 25 140 - 55 = 85 = 30 × 3 - 5
25.5 + 7.5 = 33  ? = 140 + 85 × 3 - 7
33 + 5.5 = 38.5 = 140 + 248 = 388
143. (3) The pattern of the number series is : 151. (5) The pattern of the number series is :
6 × 7 + 1 × 7 = 49 119 + 1 × 12 =131
49 × 6 + 2 × 6 = 306, not 305 131 + 2 × 12 = 155
306 × 5 + 3 × 5 = 1545 155 + 3 × 12 = 191
1545 × 4 + 4 × 4 = 6196 191 + 4 × 12 = 239
6196 × 3 + 5 × 3 = 18603 239 + 5 × 12 = 299
144. (3) The pattern of the number series is : 152. (4) The pattern of the number series is :
8 × 0.5 + 1 = 5 11 + 1 × 46 = 11 + 46 = 57
5 × 1 + 1.5 = 6.5 57 + 2 × 46 = 57 + 92 = 149
6.5 × 1.5 + 2 = 9.75 + 2= 11.75, not 11 149 + 2 × 92 = 149 + 184 = 333
11.75 × 2 + 2.5 = 23.5 + 2.5 = 26 333 + 2 × 184 = 333 + 368 = 701
26 × 2.5 + 3 = 68 701 + 2 × 368 = 701 + 736 = 1437
145. (3) The pattern of the number series is : 153. (2) The pattern of the number series is :
586 + 1 = 587 697 - 553 = 144 = 122
587 + (1 - 2) = 587 - 1 = 586 553 - 453 = 100 = 102
586+ (-1 - 4) = 586 - 5 = 581 453 - 389 = 64 = 82
581 + (-5 - 6) = 581 - 11 = 570 389 - 353 = 36 = 62
570+ (-11 -8) = 570 -19 = 551  ? = 353 - 42 = 353 - 16 = 337
551 + (-19 _ 10) = 551- 29 = 522 154. (1) The pattern of the number series is :
146. (5) The pattern of the number series is : 336 - 224 = 112
64 - 10 = 54 224 - 168 = 56
54 + 15 = 69 168 - 140 = 28
69 - 20 = 49 140 - 126 = 14
49 + 25 = 74  ? = 126 - 7 = 119
74 - 30 = 44 155. (2) The pattern of the number series is :
44 + 35 = 79 9 × 2 - 3 = 18 - 3 = 15
147. (2) The pattern of the number series is : 15 × 2 - 3 = 30 - 3 = 27
(4000  2) + 8 = 2008 27 × 2 - 3 = 54 - 3 = 51
(2008  2) + 8 = 1012 51 × 2 - 3 = 102 - 3 = 99
(1012  2) + 8 = 514 99 × 2 - 3 = 198 - 3 = 195
(514  2) + 8 = 265 156. (4) The pattern of the number series is :
148. (3) The pattern of the number series is : 13 + 8 = 21
5×1=5 21 + 8 + 7 = 21 + 15 = 36

LEARN MATHS FROM S.K. RAJU (9811549822, 9811649822)


31
36 + 15 + 7 = 36 + 22 = 58  ? = 627 + 17 = 644
58 + 22 + 7 = 58 + 29 = 87 166. (4) The pattern of the number series is :
87 + 29 + 7 = 87 + 36 = 123 7 + 1 × 4 = 11
157. (4) The pattern of the number series is : 11 + (1 + 2) 4 = 11 + 3 × 4 = 23
7+2+0=9 23 + (3 + 4) 4 = 23 + 7 × 4 = 51
9 + (2 + 8) = 19 51 + (7 + 6) 4 = 51 + 13 × 4 = 103
19 + (10 + 16) = 45 103 + (13 + 8) 4 = 103 + 21 × 4 = 187
45 + (26 + 24) = 95 167. (4) The pattern of the number series is :
95 + (50 + 32) = 177 18 + 9 = 27
158. (1) The pattern of the number series is : 27 + (9 + 13) = 49
14 + l2 = 15 49 + (9 + 26) = 84
15 + 23 = 23 84 + (9 + 39) = 132
23 + 32 = 32 168. (2) The pattern of the number series is :
32 + 43 = 96 33 + 10 = 43
96 + 52 = 96 + 25 = 121 43 + (10 + 12) = 65
159. (3) The pattern of the number series is : 65 + (10 + 24) = 99
20 + 1 × 4 = 20 + 4 = 24 99 + (10 + 36) = 145
24 + 3 × 4 = 24 + 12 = 36 145 + (10 + 48) = 203
36 + 5 × 4 = 36 + 20 = 56 169. (5) The pattern of the number series is :
56 + 7 × 4 = 56 + 28 = 84 655 - 439 = 216 = 63
84 + 9 × 4 = 84 + 36 = 120 439 - 314 = 125 = 53
160. (2) The pattern of the number series is : 314 - 250 = 64 = 43
732 - 3 = 729 = 93 250 - 223 = 27 - 33
1244 - 732 = 512 = 83  ? = 223 - 23 = 223 - 8
1587 - 1244 = 343 = 73 = 215
1803 - 1587 = 216 = 63 170. (4) The pattern of the number series is :
1928 - 1803 = 125 = 53 15 + 6 = 21
 ? = 1928 + 43 = 1928 + 64 = 1992 21 + 18 (= 6 + 12) = 39
161. (4) The pattern of the number series is : 39 + 38 (= 18 + 20) = 77
16 × 1.5 = 24 77 + 66 (= 38 + 28) = 143
24 × 2.5 = 60 143 + 102 (= 66 + 36) = 245
60 × 3.5 = 210 171. (1) The pattern of the number series is :
210 × 4.5 = 945 33 + 6 = 39
162. (1) The pattern of the number series is : 39 + 18 (= 6 + 12) = 57
(45030  5) - 6 = 9000 57 + 30 (= 18 + 12) = 87
(9000  5) - 5 = 1795 87 + 42 (= 30 + 12) = 129
(1795  51 - 4 = 355 129 + 54 (= 42 + 12) = 183
(355  5) - 3 = 68 172. (1) The pattern of the number series is :
(68  5) - 2 = 13.6 - 2 = 11.6 19 - 15 = 4 = 22
163. (1) The pattern of the number series is : 83 - 19 = 64 = 43
5 × 1 + 1 × 7 = 12 119 - 83 = 36 = 62
12 × 2 + 2 × 6 = 36 631 - 119 = 512 = 83
36 × 3 + 3 × 5 = 123  ? = 631 + 102 = 631 + 100 = 731
123 × 4 + 4 × 4 = 492 + 16 = 508 173. (3) The pattern of the number series is :
508 × 5 + 5 × 3 = 2540 + 15 = 2555 19 + 1 × 7 = 19 + 7 = 26
164. (4) The pattern of the number series is : 26 + 2 × 7 = 26 + 14 = 40
8 × 0.5 + 7 = 4 + 7=11 40 + 4 × 7 = 40 + 28 = 68
11 × 1 + 6 = 17 68 + 8 × 7 = 68 + 56 = 124
17 × 1.5 + 5 = 25.5 + 5 = 30.5 124 + 16 × 7 = 124 + 112
30.5 × 2 + 4 = 61 + 4 = 65 = 236
165. (5) The pattern of the number series is : 174. (5) The pattern of the number series is :
389 - 117 = 272 69 - 43 = 26
525 - 389 = 136 58 - 69 = - 11
593 - 525 = 68 84 - 58 = 26
627 - 593 = 34 73 - 84 = -11
LEARN MATHS FROM S.K. RAJU (9811549822, 9811649822)
32
 ? = 73 + 26 = 99 17 + 5 × 3 = 32
175. (4) The pattern of the numbe series is : 32 + 5 × 5 = 57
15 + 3 = 18 57 + 5 × 7 = 92
18 - 2 = 16 92 + 5 × 9 = 137
16 + 3 = 19 183. (4) The pattern of the number series is :
19 - 2 = 17 19 + 2 × 3 = 19 + 6 = 25
17 + 3 = 20 25 + 4 × 5 = 25 + 20 = 45
20 - 2 = 18 45 + 6 × 7 = 45 + 42 = 87
176. (1) The pattern of the number series is : 87 + 8 × 9 = 87 + 72 = 159
2 159 + 10 × 11 = 159 + 110 = 269
1050 × = 420 184. (5) The pattern of the number series is :
5
83 + 41 × 1 = 124
2 124 + 41 × 2 = 124 + 82 = 206
420 × = 168 206 + 41 × 4 = 206 + 164 = 370
5
370 + 41 × 8 = 370 + 328 = 698
2 698 + 41 × 16 = 698 + 656 = 1354
168 × = 67.2
5 185. (4) The pattern of the number series is :
2 1 × 7=7
 10.752 × = 4.3008 7 × 7 = 49
5
49 × 7 = 343
177. (5) The pattern of the number series is :
343 × 7 = 2401
0 + 1 × 6 = 6
186. (4) The pattern of the number series is :
6 + 2 × 9 = 24
13 + 7 = 20
24 + 3 × 12 = 60
20 + 19 (= 7 + 12) = 39
60 + 4 × 15 = 120
39 + 39 (=19 + 20) = 78
120 + 5 × 18 = 210
78 + 67 (= 39 + 28) = 145
210 + 6 × 21 = 210 + 126 = 336
145 + 103 (= 67 + 36) = 248
178. (3) The pattern of the number series is :
187. (1) The pattern of the number series is :
32 + 1 × 17 = 32 + 17 = 49
12 + 1 × 23 = 35
49 + 2 × 17 = 49 + 34 = 83
35 + 2 × 23 = 35 + 46 = 81
83 + 4 × 17 = 83 + 68 = 151
81 + 2 × 46 = 81 + 92 = 173
151 + 8 × 17 = 151 + 136 = 287
173 + 2 × 92 = 173 + 184 = 357
287 + 16 × 17 = 287 + 272 = 559
357 + 2 × 184 = 357 + 368 = 725
559 + 32 × 17 = 559 + 544 = 1103
188. (5) The pattern of the number series is :
179. (2) The pattern of the number series is :
3 + 97 = 100
552 - 462 = 90
100 + 197 = 297
650 - 552 = 98
297 + 297 = 594
756 - 650 = 106
594 + 397 = 991
870 - 756 = 114
991 + 497 = 1488
992 - 870 = 122
189. (3) The pattern of the number series is :
 ? = 992 + 130 = 1122 112 + 1 × 7 = 119
180. (3) The pattern of the number se ries is :
119 + 3 × 7 = 119 + 21 = 140
28 + 11 = 39
140 + 5 × 7 = 140 + 35 = 175
39 + 24(= 11 + 13) = 63
175 + 7 × 7 = 175 + 49 = 224
63 + 39 (= 24 + 15) = 102
224 + 9 × 7 = 224 + 63 = 287
102 + 56 (= 39 + 17) = 158
190. (2) The pattern of the number series is :
158 + 75 (= 56 + 19) = 233
958 - 833 = 125
181. (5) The pattern of the number series is :
833 - 733 = 100
7 + 32 = 7 + 9 = 16
733 - 658 = 75
16 + 53 = 16 + 125 = 141
658 - 608 = 50
141 + 72 = 141 + 49 = 190
 ? = 608 - 25 = 583
190 + 93 = 190 + 729 = 919
919+ 112 = 919 + 121 = 1040 191. (4) The pattern of the number series is :
11 × 1 - 1 = 10
182. (3) The pattern of the number series is :
10 × 2 - 2 = 18
12 + 5 × 1 = 17
18 × 3 - 3 = 51

LEARN MATHS FROM S.K. RAJU (9811549822, 9811649822)


33
51 × 4 - 4 = 200 18 × 4 - 36 = 72 - 36 = 36
200 × 5 - 5 = 995 36 × 4 - 42 = 144 - 42 = 102
192. (1) The pattern of the number series is : 102 × 4 - 48 = 408 - 48 = 360
25 × 2 - 2 = 50 - 2 = 48 360 × 4 - 54 = 1440 - 54 = 1386
48 × 2 - 2 = 96 - 2 = 94 200. (4) The pattern of the number series is :
94 × 2 - 2 = 188 - 2 = 186 7 × 2 - 2 = 12
186 × 2 - 2 = 372 - 2 = 370 12 × 4 - (2 + 6) = 48 - 8 = 40
370 × 2 - 2 = 740 - 2 = 738 40 × 6 - (8 + 10) = 240 - 18 = 222
193. (2) The pattern of the number series is : 222 × 8 - (18 + 14) = 1776 - 32
14 + 10 = 24 = 1744  1742
24 + 19 (=10 + 9) = 43 1744 × 10 - (32 + 18) = 17440 - 50 = 17390
43 + 28 (= 19 + 9) = 71 201. (3) The pattern of the number series is :
71 + 37 (= 28 + 9) = 108 6 × 7 + 72 = 42 + 49 = 91
108 + 46 (=37 + 9) = 154 91 × 6 + 62 = 546 + 36 = 582
194. (5) The pattern of the number series is : 582 × 5 + 52 = 2910 + 25 = 2935
144 + 29 = 173 2935 × 4 + 42 = 11740 + 16 = 11756
173 - 33 = 140 11756 × 3 + 32 = 35268 + 9 = 35277
140 + 29 = 169 202. (5) The pattern of the number series is :
169 - 33 = 136 9050 - 153 = 9050 - 3375 = 5675
136 + 29 = 165 5675- 133 = 5675 - 2197 = 3478
195. (2) The pattern of the number series is : 3478 - 113 = 3478 - 1331 = 2147
8 + 2 = 10 2147 - 93 = 2147 - 729 = 1418
10 + 8 (= 2 × 3 + 2) = 18 1418 - 73 = 1418 - 343 = 1075  1077
18 + 26 (= 3 × 8 + 2) = 44 203. (4) The pattern of the number series is :
44 + 80 (=3 × 26 + 2) = 124 1=1
124 + 242 (= 3 × 80 + 2) = 366 22 = 4
196. (4) The pattern of the number series is : 33 = 27  25
13 + 1 × 12 = 13 + 12 = 25 44 = 256
25 + 3 × 12 = 25 + 36 = 61 55 = 3125
61 + 5 × 12 = 61 + 60 = 121 66 = 46656
121 + 7 × 12 = 121 + 84 = 205 204. (2) The pattern of the number series is :
205 + 9 × 12 = 205 + 108 8424  2 = 4212
= 313 4212  2 = 2106
197. (1) The pattern of the number series is : 2106  2 = 1053  1051
656 1053  2 = 526.5
+ 24 = 328 + 24 = 352 526.5  2 = 263.25
2
205. (1) The pattern is :
352 5531 - 5506 = 25 = 52
+ 24 = 176 + 24 = 200 5555 - 5506 = 49 = 72
2
5506 - 5425 = 81 = 92
200 5425 - 5304 = 121 = 112
+ 24 = 100 + 24 = 124
2 5304 - 5135 = 169 = 132
5135 - 4910 = 225 = 152
124
+ 24 = 62 + 24 = 86 4910 - 4621 = 289 = 172
2 Clearly, 5531 is wrong which should be
86 substituted by 5555.
+ 24 = 43 + 24 = 67 206. (2) The pattern is :
2
6 +1 = 7
198. (3) The pattern of the number series is :
7 + 1 × 2 = 9
454 + 18 = 472
9 + 2 × 2 = 13
472 - 27 = 445
13 + 8 = 21  26
445 + 18 = 463
21 + 16 = 37
463 - 27 = 436
37 + 32 = 69
436 + 18 = 454
207. (4) The pattern is :
199. (2) The pattern of the number series is :
1 × 1 + 2 = 3
12 × 4 - 30 = 48 - 30 = 18
LEARN MATHS FROM S.K. RAJU (9811549822, 9811649822)
34
3 × 2 + 4 = 10 840 - 752 = 88
10 × 3 + 6 = 36 884 - 840 = 44
36 × 4 + 8 = 152  ? = 884 + 22 = 906
152 × 5 + 10 = 770  760 216. (1) The pattern is :
770 × 6 + 12 = 4632 5 × 1 + l2 = 5 + 1 = 6
208. (3) The pattern is : 6 × 2 + 22 = 12 + 4 = 16
4 + 13 = 5 16 × 3 + 32 = 48 + 9 = 57
5 + 23 = 13 57 × 4 + 42 = 228 + 16 = 244
13 + 33 = 40 217. (4) The pattern is :
40 + 43 = 104  105 12 × 4 = 48
104 + 53 = 229 48 × 3.5 = 168
229 + 63 = 445 168 × 3 = 504
209. (1) The pattern is : 504 × 2.5 = 1260
157.5  3.5 = 45 1260 × 2 = 2520
45  3 = 15 218. (5) The pattern is :
15  2.5 = 6 4 × 2 + 1= 8 + 1 = 9
6  2=3 9 × 3 + 2 = 27 + 2 = 29
3  1.5 = 2 29 × 4 + 3 = 116 + 3 = 119
2  1=2 1 119 × 5 + 4 = 595 + 4 = 599
210. (2) The pattern is : 599 × 6 + 5 = 3594 + 5 = 3599
123 + 11 × 14 = 123 + 154 = 277 219. (3) The pattern is :
277 + 13 × 14 = 277 + 182 = 459 177 - 7 = 170
459 + 15 × 14 = 459 + 210 = 669 170 - 11 = 159
669+ 17 × 14 = 669 + 238 = 907 159 - 13 = 146
907 + 19 × 14 = 907 + 266 146 - 17 = 129
= 1173 129 - 19 = 110
211. (2) The pattern is : Note : Consecutive prime numbers have
456.5 - 407 = 49.5 been subtracted.
407 - 368.5 = 38.5 220. (3) The pattern is :
368.5 - 341 = 27.5 2 + l3 = 2 + 1 = 3
341 - 324.5 = 16.5 3 + 23 = 3 + 8 = 11
 ? = 324.5 - 5.5 = 319 11 + 33 = 11 + 27 = 38
212. (1) The pattern is : 38 + 43 = 38 + 64 = 102
23 + 1 × 19.2 = 42.2 102 + 53 = 102 + 125 = 227
42.2 + 2 × 19.2 = 80.6 221. (1) The pattern of the number series is :
80.6 + 4 × 19.2 = 157.4 21 × 0.5 = 10.5
157.4 + 8 × 19.2 = 311 10.5 × 1 = 10.5
311 + 16 × 19.2 = 311 + 307.2 10.5 × 1.5 = 15.75
= 618.2 15.75 × 2 = 31.50
213. (5) The pattern is : 31.50 × 2.5 = 78.75
154 - 36 = 118 222. (2) The pattern of the number series is :
232 - 154 = 78 6 + 1 × 13 = 6 + 13 = 19
278 - 232 = 46 19 + 3 × 13 = 19 + 39 = 58
300 - 278 = 22 58 + 5 × 13 = 58 + 65 = 123
 ? - 300 = 6 123 + 7 × 13 = 123 + 91 = 214
 ? = 306 214 + 9 × 13 = 214 + 117 = 331
214. (4) The pattern is ; 223. (3) The pattern of the number series is :
24 + 83 = 24 + 512 = 536 14 + 1 × 2 = 16
536 - 72 = 536 - 49 = 487 16 + 3 × 4 = 16 + 12 = 28
487 + 63 = 487 + 216 = 703 28 + 5 × 6 = 28 + 30 = 58
703 - 52 = 703 - 25 = 678 58 + 7 × 8 = 58 + 56 = 114
678 + 43 = 678 + 64 = 742 114 + 9 × 10 = 114 + 90 = 204
215. (3) The pattern is : 224. (4) The pattern of the number series is :
576 - 224 = 352 13.76 + 1 × 1.15 = 14.91
752 - 576 = 176 14.91 + 2 × 1.15 = 14 + 2.30 = 17.21

LEARN MATHS FROM S.K. RAJU (9811549822, 9811649822)


35
17.21 + 3 × 1.15 = 17.21 + 3.45 = 20.66 772 × 6 + 2 = 4634
20.66 + 4 × 1.15 = 20.66 + 4.60 = 25.26 Therefore, the number 914 is wrong.
25.26 + 5 × 1.15 = 25.26 + 5.75 = 31.01  According to question, the new series is
225. (5) The pattern of the number series is : as follows:
15 + 12 = 16 914 × 1 + 2 = 916
16 + 23 = 16 + 8 = 24 916 × 2 + 2 = 1834
24 + 32 = 24 + 9 = 33 1834 × 3 + 2 = 5504
33 + 43 = 33 + 64 = 97 Therefore, the required number is 1834.
97 + 52 = 97 + 25 = 122 2. (3) The series is based on following pattern :
226. (5) The pattern is : 3×1+1=4
2×3=6 4 × 2 + 2 = 10
6 × 2.5 = 15 10 × 3 + 3 = 33
15 × 2 = 30 33 × 4 + 4 = 136
30 × 1.5 = 45 136 × 5 + 5 = 685
45 × 1 = 45  43.5 685 × 6 + 6 = 4116
45 × 0.5 = 22.5 Therefore, the number 34 is wrong.
227. (3) The pattern is :  According to question, the new series
950 - 661 = 289 = 172 starts from the number 34 in the same
661 - 436 = 225 = 152 pattern.
436 - 269 = 167  132 34 × 1 + 1 = 35
 436 - 267 = 169 = 132 35 × 2 + 2 = 72
267 - 146 = 121 = 112 Hence, the number 72 is required answer.
146 - 65 = 81 = 92 3. (4) The series is based on following pattern :
228. (5) The pattern is : 214 - (14)2 = 18
6.5 + 5.3 = 11.8 18 + (12)2 = 162
11.8 + 2 × 5.3 = 11.8 + 10.6 = 22.4 162 - (10)2 = 62
22.4 + 3 × 5.3 = 22.4 + 15.9 = 38.3 62 + (8)2 = 126
38.3 + 4 × 5.3 = 38.3 + 21.2 = 59.5 126 - (6)2 = 90
59.5 + 5 × 5.3 = 59.5 + 26.5 = 86 90 + (4)2 = 106
 87.3 Therefore the number 143 is wrong.
86 + 6 × 5.3 = 86 + 31.8 = 117.8  According to question, the new series
229. (5) The pattern is : starts from the number 143 in
1×3-1=2 143 - (14)2 = -53
2×3-2=4 -53 + (12)2 = 91
4×3-3=9 Hence, the number 91 is required answer.
9 × 3 - 4 = 23 4. (5) The series in based on following pattern:
23 × 3 - 5 = 69 - 5 = 64  69 160 × 0.5 = 80
64 × 3 - 6 = 192 - 6 = 186 80 × 1.5 = 120
230. (5) The pattern is : 120 × 2.5 = 300
250 - 11 = 239 300 × 3.5 = 1050
239 - (11 × 2 + 1) = 239 - 23 = 216 1050 × 4.5 = 4725
216 - (11 × 3 + 2) = 216 - 35 = 181 4725 × 5.5 = 25987.5
181 - (11 × 4 + 3) Therefore, the number 180 is wrong.
= 181 - 47 = 134  136  According to question, the new series
134 - (11 × 5 + 4) starts from the number 180 in the same
= 134 - 59 = 75 pattern:
75 - (11 × 6 + 58) 180 × 0.5 = 90
= 75 - 71 = 4 90 × 1.5 = 135
SBI PO EXAMS Hence, the number 135 is required answer.
1. (3) The series is based on following pattern: 5. (1) The series is based on following pattern:
3×1+2=5 2 + l2 - 0 = 3
5 × 2 + 2 = 12 3 + 22 - 1 = 6
12 × 3 + 2 = 38 6 + 32 - 2 = 13
38 × 4 + 2 = 154 13 + 42 - 3 = 26
154 × 5 + 2 = 772 26 + 52 - 4 = 47
LEARN MATHS FROM S.K. RAJU (9811549822, 9811649822)
36
47 + 62 - 5 = 78 3 × 2 - (2) 2 = 2
Th erefore, the nu mber 7 i s wron g. 2 × 3 + (3) 2 = 15
According to question, the new series starts 15 × 4 - (4) 2 = 44
from the number 7 in the same pattern. 44 × 5 + (5)2 = 245
7 + l1 - 0 = 8 245 × 6 - (6) 2 = 1434
8 + 22 - 1 = 11 Similarly,
Hence, the number 11 is required answer. 3 × 1 + (1)2 = 4 ....(a)
6. (4) The series is based on following pattern : 4 × 2 - (2) 2 = 4 ....(b)
2 × 1 + l2 = 3 4 × 3 + (3) 2 = 21 ....(c)
3 × 2 + 22 = 10 21 × 4 - (4)2 = 68 ....(d)
10 × 3 + 32 = 39 Therefore, the 21 will come in place of (c).
39 × 4 + 42 = 172 12. (5) The series is based on following pattern
172 × 5 + 52 = 885 1 × 1+ (1) 2 = 2
Similarly, the new series is as follows : 2 × 2 + (2) 2 = 8
1 × 1 + 12 = 2.....(a) 8 × 3 + (3) 2 = 33
2 × 2 + 22 = 8......(b) 33 × 4 + (4) 2 = 148
8 × 3 + 32 = 33 .....(c) 148 × 5 + (5) 2 = 765
Therefore, the number 8 will come in place 765 × 6 + (6) 2 = 4626
of (b). Similarly,
7. (2) The series is based on the following 2 × 1 + (1)2 = 3 ....(a)
pattern: 3 × 2 + (2) 2 = 10 ....(b)
5 × 1 + 2 = 7 10 × 3 + (3) 2 = 39 .....(c)
7 × 2 - 4 = 10 39 × 4 + (4) 2 = 172 ....(d)
10 × 3 + 6 = 36 Therefore, the number 172 will come in
36 × 4 - 8 = 136 place of (d).
136 × 5 + 10 = 690 13. (5) The series is based on following pattern :
Similarly, the new series is as follows: 2 × 2 + 0.5 = 4.5
2 × 1 + 2 = 4 ....(a) 4.5 × 2 + (0.5) × 4 = 11
4×2-4=4 ....(b) 11 × 2 + 2 × 4 = 30
4 × 3 + 6 = 18 ....(c) 30 × 2 + 8 × 4 = 92
18 × 4 - 8 = 64 ....(d) 92 × 2 + 32 × 4 = 312
64 × 51 + 10 = 330 .....(e) 312 × 2 + 128 × 4 = 1136
Therefore, the number 330 will come in Similarly,
palce of (e). 1 × 2 + 0.5 = 2.5 ....(a)
8. (5) The series is based on following pattern: 2.5 × 2 + (0.5) × 4 = 7 ....(b)
8 × 0.5 = 4 Therefore, the number 7 wil come in place
4 × 1.5 = 6 of (b)
6 × 2.5 = 15
15 × 3.5 = 52.6
52.5 × 4.5 = 236.25 14. (1)
Therefore, the number 236.25 will come in
place of (d).
9. (3) Interchanging (3) and (5)

In the given series 176 should be replace


10. (3) Interchanging (3) and (5) br 174.238 will come in place of (e)

15. (3)
11. (4) The series is based on following pattern
2 × 1 + (1) 2 = 3

LEARN MATHS FROM S.K. RAJU (9811549822, 9811649822)


37
22. (2) The given series is based on the following
pattern :

In the given series 7 should be replaced by


5. and 277 should come in place of (f).
16. (1) The given number series is based on the
Obviously, 39 is the wrong number and it
following pattern
should be replaced with 37.
23. (1) The given series is based on the following
pattern :
2 × 2 + 7 = 11 (not 13)
11 × 3 - 6 = 27
27 × 4 + 5 = 113
Hence the wrong number is 6 113 × 5 - 4 = 561
17. (2) The given number series is based on the Obviously the number 13 is wrong and it
following pattern: should be replaced with 11.
24. (4) The given series is based on the following
pattern.
50 + (12) = 51
51 - (22) = 47
47 + (32) = 56
Hence, the wrong number is 75
56 - (42) = 40 (not 42)
18. (4)The given number series is basei on the
40 + (52) = 65
following pattern
Obviously, the number 42 is wrong and it
4 - 3 =12
should be replaced with 40.
13 - 4 = 9 = 32
25. (3) The given series is based on the following
38 - 13 = 25 = 52
pattern :
87 - 38 = 49 = 72
3 × 2+3=9
168 - 87 = 81 = 92
9 × 3 - 4 = 23
289 - l68 = 121 = 112
23 × 4 + 5 = 97 (not 99)
Obviously, 166 is the wrong number.
97 × 5 - 6 = 479
19. (3) The number series follows the rule as
Obviously, the number 99 is wrong and it
mentioned below:
should be replaced with 97.
26. (1) The given series is based on the following
pattern:
2+3=5
5+3=8
Hence 29 is the wrong number. 8 + 5 = 13
20. (5) The followed pattern is: 13 + 8 = 21
21 + 13 = 34
Obviously, the number 4 is wrong and it
should be replaced with 3.
27. (2) The given series is based on the following
Hence the wrong number is 176 pattern :
21. (4) The given series is based on tne following
pattern
2 × 3=6
6 × 3 = 18
18 × 6  109 but 108 Similarly,
108 × 18 = 1944
1944 × 108 = 209952
Obviously, 109 is the wrong number and
it should be replaced with 108.
LEARN MATHS FROM S.K. RAJU (9811549822, 9811649822)
38

Hence, 22.5 will come in place of (c).


Hence, 163 will come in place of (b). 32. (3) The given series is based on the following
28. (1) The given series is based on t h e pattern :
following pattern 9 × 2 + 1.5 = 19.5
13 = 4 × 1 + 1 × 9 19.5 × 2 + 2 = 41
40 = 13 × 2 + 2 × 7 41 × 2 + 2.5 = 84.5
135 = 40 × 3 + 3 × 5 Therefore, the new series is as follows :
552 = 135 × 4 + 4 × 3 12 × 2 + 1.5 = 25.5 ....(a)
2765 = 552 × 5 + 5 × 1 25.5 × 2 + 2 = 53 ....(b)
Similarly, 53 × 2 + 2.5 = 108.5 ....(c)
(a) = 2 × l + 1 × 9 = 11 108.5 × 2 + 3 = 220 ....(d)
(b) = 11 × 2 + 2 × 7 = 36 220 × 2 + 3.5 = 443.5 ....(e)
(c) = 36 × 3 + 3 × 5 = 123 Therefore, the number 108.5 will come in
Hence, 123 will come in place of (c). place of (C) in the new series.
29. (3) The given series is based on the following 33. (1) The series is based on following pattern:
pattern: 4 × 1 + 1 = 5
 +3
5 × 4 + 2 = 22
 +5
Similarly, 22 × 9 + 3 = 201
Similarly the new series is as follows :
7 × 1+1 = 8 ....(a)
8 × 4+2 =4 ....(b)
34 × 9 + 3 = 309 ....(c)
309 × 16 + 4 = 4948 ....(d)
Hence, 4 will come in place of (d).
Therefore, the number 4948 will come in
30. (4) The given series is based on the following
place of (d) in the new series.
pattern :
34. (2) The series is based on following pattern :
5 × 1+ 0.25 × 1 = 5.25
 +3
5.25 × 2 + 0.25 × 4 = 11.5
7, 11, 13, 17, 19, ..... are consecutive prime  +5
numbers) 11.5 × 3 + 0.25 × 9 = 36.75
Similarly, Similarly, the new series is as follows.
3 × 1 + 0.25 × 1 = 3.25 ....(a)
3.25 × 2 + 0.25 × 4 = 7.5 ....(b)
7.5 × 3 + 0.25 × 9 = 24.75 ....(c)
Therefore, the number 24.75 will come in
place of (c) in the new series.
Hence, 159 will come in place of (d).
35. (4) The series is based on following pattern :
31. (3) The given series is based on the following
38 × 0.5 = 19
pattern :
19 × 1.5 = 28.5
28.5 × 2.5 = 71.25
Similarly, the new series is as follows :
18 × 0.5 = 9 ....(a)
9 × 1.5 = 13.5 .....(b)
Similarly,
13.5 × 2.5 = 33.75 ....(c)
33.75 × 3.5 = 118.125.....(d)
Therefore, the number 118.125 will come

LEARN MATHS FROM S.K. RAJU (9811549822, 9811649822)


39
in place of (d) in the new series. 8 = 2 × 2 + 22
36. (3) The series is based on following pattern: Obviously 32 is the wrong number.
25 + (11)2  25 + 121 = 146 41. (2) The sequence is based on following
146- (9)2  146 - 81 = 65 pattern:
65 + (7)2  65 + 49 = 114 3 - 2 = 13
Similarly, the new series is as follows : 11 - 3 = 8 = 23
38 - 11 = 27 = 33
39 + (11)2  39 + 121
102 - 38 = 64 = 43
= 190......(a)
But,
160 - (9)2  160 - 81 229 -102 = 127  53
= 79 ........(b) 227 - 102 = 125 = 53
79 + (7)2  79 + 49 443 - 227 = 216 = 63
= 128 ......(c) Obviously 229 is the wrong number.
128 + (5)2  128 - 25 42. (5) The given number series is based on the
= 103 .......(d) following pattern :
103 + (3)2  103 + 9 7413 + 9 × 1 = 7422
= 111 ....(e) 7422 + 9 × 2 = 7440
Therefore, the number 112 will come in 7440 + 9 × 3 = 7467
place of (e) in new series. 7467 + 9 × 4 = 7503
37. (1) The given series is based on following Hence, 7467 will replace the question mark.
pattern 43. (4) The given number series is based on the
15 - 10 = 5 following pattern :
24 - 15 = 9 4 = 22 ; 16 = 42;
37 - 24 = 13 36 = 62 ; 64 = 82 ;
54 - 37 = 17 100 = 102.
75 - 54 = 21  ? = 122 = 144
100 - 75 = 25 Hence, 144 will replace the question mark.
Obviously, 35 is wrong number. 44. (1) The given number series is based on the
38. (5) Here the middle number = difference of following pattern:
succeeding number and preceding number. 12 × 3 - 3 = 33
i.e., 4 - 1 = 3 33 × 3 - 3 = 96
7-3=4 96 × 3 - 3 = 285
11 - 4 = 7 285 × 3 - 3 = 852
18 - 7 = 11 Hence, 285 will replace the question mark.
27 - 11 = 16 45. (3) The given number series is based on the
Here the sequence gets disturbed following pattern :
 29 - 11 = 18 70000  5 = 14000
47 - 18 = 29 14000  5 = 2800
Hence, 27 is the wrong number. 2800  5 = 560
39. (5) The sequence is based on following 560  5 = 112
pattern: 112  5 = 22.4
3 × 0.5 + 0.5 = 2 Hence, 560 will replace the question mark.
2×1+1=3 46. (2) The given number series is based on the
3 × 1.5 + 1.5 = 6 following pattern :
6 × 2 + 2 = 14 102 - 3 = 99
14 × 2.5 + 2.5 = 37.5 99 + 5 = 104
37.5 × 3 + 3 = 115.5 104 - 7 = 97
Obviously, 12 is the wrong number. 97 + 9 = 106
40. (4) 32431 = 7 × 4626 + 72 106 - 11 = 95
4626 = 6 × 765 + 62 Hence, 95 will replace the question mark.
765 = 5 × 148 + 52 47. (4) The given number series is based on the
148 = 4 × 32 + 42 following pattern
But 148 = 4 × 33 + 42 93 + 2 (prime number) = 95
33 = 3 × 8 + 32 95 + 3 = 98  99
98 + 5 = 103
LEARN MATHS FROM S.K. RAJU (9811549822, 9811649822)
40
103 + 7 = 110 30 + 12 = 42
110 + 11 = 121 55. (5) The pattern of the number series is :
121 + 13 = 134 1
Hence, 103 will replace the question mark 32 × = 16
2
48. (5) The given number series is based on the
following pattern: 3
8 × 1.5 = 12 16 × = 24
2
12 × 1.5 = 18
18 × 1.5 = 27  26 5
24 × = 60  65
27 × 1.5 = 40.5 2
40.5 × 1.5 = 60.75
7
 ? = 60.75 × 1.5 = 91.125 60 × = 210
Hence, 91.125 will replace the question 2
mark. 9
49. (5) The given number series is based on the 210 × = 945
2
following pattern : 4 + 7 = 11
11 + 7 = 18 11
945 × = 5197.5
18 + 11 = 29  28 2
 ? = 29 + 18 = 47 56. (4) The pattern of the number series is :
Hence, 47 will replace the question mark. 7 × 2 - 1 = 14 - 1 = 13
50. (1) The given number series is based on the 13 × 2 - 1 = 26 - 1 = 25
following pattern: 25 × 2 - 1 = 50 - 1 = 49
3 × 2 + 22 = 10 49 × 2 - 1 = 98 - 1 = 97
10 × 3 + 32 = 39 97 × 2 - 1 = 194 - 1 = 193  194
39 × 4 + 42 = 172 193 × 2 - 1 = 386 - 1 = 385
172 × 5 + 52 = 885  886 57. (1) The pattern of the given series Is :
885 × 6 + 62 = 5346 37 × 0.5 + 0.5 = 18.5 + 0.5 = 19
Hence, 39 will replace the question mark. 19 × 1 + 1 = 19 + 1 - 20
51. (3) The given number series is based on the 20 × 1.5 + 1.5 = 30 + 1.5 - 31.5
following pattern : 31.5 × 2 + 2 = 63 + 2 = 65
15 × 1 + 1 × 7 = 22 65 × 2.5 + 2.5 = 162.5 + 2.5 - 165
22 × 2 + 2 × 6 = 56  57 Similarly,
56 × 3 + 3 × 5 = 183 21 × 0.5 + 0.5 = 10.5 + 0.5 = 11(a)
183 × 4 + 4 × 4 = 748 11 × 1 + 1 = 11 + 1 = 12 (b)
748 × 5 + 5 × 3 = 3755 12 × 1.5 + 1.5 = 18 + 1.5 = 19.5 (c)
3755 × 6 + 6 × 2 = 22542 19.5 × 2 + 2 = 39 + 2 = 41 (d)
Hence, 748 will replace the question mark. 41 × 2.5 + 2.5 = 102.5 + 2.5 = 105 (e)
52. (4) The pattern of the number series is : 58. (2) The pattern of the given series is :
3601  1 + 1 = 3602 5 × 1 + 12 = 5 + 1 = 6
3602  2 + 2- 1801 + 2 = 1803 6 × 2 + 22 = 12 + 4 = 16
1803  3 + 3 - 601 + 3 - 604 16 × 3 + 32 = 48 + 9 = 57
604  4 + 4 = 151 + 4 = 155  154 57 × 4 + 42 = 228 + 16 = 244
155  5 + 5 = 31 + 5 = 36 244 × 5 + 52 = 1220 + 25 = 1245
36  6 + 6 = 6 + 6 = 12 Similarly,
53. (2) The pattern of the number series is : 9 × 1 + 12 = 9 + 1 = 10 (a)
4 × 2 + 22 = 8 + 4 = 12 11 × 2 + 22 = 22 + 4 = 26 (b)
12 × 3 + 32 = 36 + 9 = 45  42 26 × 3 + 32 = 78 + 9 = 87 (c)
45 × 4 + 42 = 180 + 16 = 196 87 × 4 + 42 = 348 + 16 = 364 (d)
196 × 5 + 52 = 980 + 25 = 1005 59. (3) The pattern of the given series is :
1005 × 6 + 62 = 6030 + 36 = 6066 7 × 1 - 2= 7 - 2 = 5
54. (1) The pattern of the number series is : 5 × 3 - 4 = 15 - 4 = 11
2 + 4 = 6 8 11 × 5 - 6 = 55 - 6 = 49
6 + 6 = 12 49 × 7 - 8 = 343 - 8 = 335
12 + 8 = 20 335 × 9 - 10 = 3015 - 10 = 3005
20 + 10 = 30 Similarly,

LEARN MATHS FROM S.K. RAJU (9811549822, 9811649822)


41
13 × 1 - 2 = 13 - 2 = 11 (a) Hence, 119 will come in place of the
11× 3 - 4 = 33 - 4= 29 (b) question mark.
60. (4) The pattern of the given series is : 7. (3) The given series is based on the following
12 × 3 + 11 = 36 + 11 = 47 pattern :
47 × 3 + 11 = 141 + 11 = 152
152 × 3 + 11 = 456 + 11 = 467
467 × 3 + 11 = 1401 + 11 = 1412
1412 × 3 + 11 = 4236 + 11 = 4247
Similarly, Hence, 62.72 will come ir place of the
33 × 3 + 11 = 99 + 11 = 110 (a) question mark.
110 × 3 + 11 = 330 + 11 = 341 (b) 8. (4) The given series is based on the following
341 × 3 + 11 = 1023 + 11 = 1034(c) pattern :
1034 × 3 + 11 = 3102 + 11
= 3113 (d)
61. (5) The pattern of the given series is :
68 × 1 - 8 = 60 Hence, 2211 will come in place of the
60 × 1.5 + 14 = 90 + 14 = 104 question mark.
104 × 2 - 20 = 208 - 20 = 188 9. (1) The given series is based on the following
188 × 2.5 + 26 = 470 + 26 = 496 pattern:
496 × 3 - 32 = 1488 - 32 = 1456 Numbers are cubes of consecutive prime
Similarly, numbers. i.e.
42 × 1 - 8 = 42 - 8 = 34 (a) 113 = 1331
34 × 1.5 + 14 = 51 + 14 = 65 (b) 133 = 2197
65 × 2 - 20 = 130 - 20 = 110 (c) 173 = 4913
110 × 2.5 + 26 = 275 + 26  = 301 (d) 193 = 6859
233 = l2167
RBI GRADE-B OFFICER EXAMS 293 = 24389
l. (4)The given series is based on the following Hence, 12167 will come in place of the
pattern: question mark.
10. (2) The given series is based on the following
pattern

2. (5) The given series is based on the following


pattern:

Hence, 7.4 will come in place of the question


mark.
3. (3) The given series is based on the following 11. (3) The given number series is based on the
pattern: following pattern :
13 × 1 + 1 = 14
14 × 2 + 2 = 30
30 × 3 + 3 = 93
93 × 4 + 4 = 376
4. (2) 30 (According to question) 376 × 5 + 5 = 1885
5. (1) The given series is based on the following
 ? = 1885 × 6 + 6 = 11316
pattern: Hence, number 11316 will replace the
question mark.
12. (2)
6. (5) The given series is based on the following
pattern:

13. (4)

LEARN MATHS FROM S.K. RAJU (9811549822, 9811649822)


42
(d)  196 × 5 + 52 = 980 + 25 = 1005
20. (3) The pattern of the given series is :
8 × 1 + 1 = 9
9 × 2 + 2 = 20
20 × 3 + 3 = 63
14. (1) 63 × 4 + 4 = 256
Similarly,
(a)  5 × 1 + l = 6
(b)  6 × 2 + 2 = 14
15. (5)
(c)  14 × 3 + 3 = 45
705 + 1 × 23 = 728
728 + 2 × 23 = 774 (d)  45 × 4 + 4 = 184
774 + 3 × 23 = 843 (e)  184 × 5 + 5 = 925
843 + 4 × 23 = 935 21. (3) The pattern of the number series is :
935 + 5 × 23 = 1050 4 × 0.5 + 1 = 2 + 1 = 3
 ? = 1050 + 6 × 23 = 1050 + 138 = 1188 3 × 1 + 1.5 = 3 + 1.5 = 4.5
16. (4) The pattern of the given series is : 4.5 × 1.5 + 2 = 6.75 + 2
5 × 1.5 + 1.5 = 7.5 + 1.5 = 9 = 8.75  8.5
9 × 2.5 + 2.5 = 22.5 + 2.5 = 25 8.75 × 2 + 2.5 = 17.5 + 2.5 = 20
25 × 3.5 + 3.5 = 87.5 + 3.5 = 91 20 × 2.5 + 3 = 50 + 3 = 53
91 × 4.5 + 4.5 = 409.5 + 4.5 = 414 22. (2) The pattern of the number series is :
Similarly, 12000  5 - 5 = 2400 - 5 = 2395
(a)  3 × 1.5 + 1.5 = 4.5 + 1.5 = 6 2395  5 - 5 = 479 - 5
(b)  6 × 2.5 + 2.5 = 15 + 2.5 = 17.5 = 474  472
474  5 - 5 = 94.8 - 5 = 89.8
(c)  17.5 × 3.5 + 3.5 = 61.25 + 3.5 = 64.75
89.8  5 - 5= 17.96 - 5 = 12.96
17. (2) The pattern of the given se ries is :
23. (5) The pattern of the number series is :
15 × 1 - 1 × 6 = 15 - 6 = 9
l × 1 + 7 ×l = l + 7 = 8
9 × 2 - 2 × 5 = 18 - 10 = 8
8 × 2 + 6 × 2 = 16 + 12 = 28
8 × 3 - 3 × 4 = 24 - 12 = 12
28 × 3 + 5 × 3 = 84 + 15 = 99
12 × 4 - 4 × 3 = 48 - 12 = 36
99 × 4 + 4 × 4 = 396 + 16 = 412
36 × 5 - 5 × 2 = 180 - 10 = 170
412 × 5 + 3 × 5 = 2060 + 15 = 2075
Similarly,
2075 × 6 + 2 × 6 = 12450 + 12
(a)  19 × 1 - 1 × 6 = 19 - 6 = 13 = 12462  12460
(b)  13 × 2 - 2 × 5 = 26 - 10 = 16 24. (1) The pattern of the number series is :
18. (1) The pattern of the given series is : 144 × 1.5 = 216  215
7 × 1 - 1= 6 216 × 2.5 = 540
6 × 2 - 2 = 10 540 × 3.5 = 1890
10 × 3 - 3 = 27 1890 × 4.5 = 8505
27 × 4 - 4 = 104 8505 × 5.5 = 46777.5
104 × 5 - 5 = 515 25. (5) The pattern of the number series is :
Similarly, 2222 - 73 = 2222 - 343 = 1879
(a)  9 × 1 - 1 = 8 1879 - 63 = 1879 - 216 = 1663
(b)  8 × 2 - 2 =14 1663 - 53 = 1663 - 125 = 1538
(c)  14 × 3 - 3 = 39 1538 - 43 = 1538 - 64 = 1474
(d)  39 × 4 - 4 = 152 1474 - 33 = 1474 - 27 = 1447
19. (5) The pattern of the given series is : 1447 - 23 = 1447 - 8
6 × 2 + 22 = 12 + 4 = 16 = 1439  440
16 × 3 + 32 = 48 + 9 = 57 26. (4) The pattern is :
57 × 4 + 42 = 228 + 16 = 244 23 + 12 = 9
Similarly, 33 + 22 = 31
(a)  4 × 2 + 22 = 8 + 4 = 12 43 + 32 = 73
53 + 42 = 141
(b)  12 × 3 + 32 = 36 + 9 = 45
63 + 52 = 241
(c)  45 × 4 + 42 = 180 + 16 = 196 27. (4) The pattern is :

LEARN MATHS FROM S.K. RAJU (9811549822, 9811649822)


43
35 + 221 = 256 64 + 4 × (2)3 = 96
256 + (221 - 26) = 451 Therefore, the number 37 will come in place
451 + 169 (=195 - 26) = 620 of question mark (?) in the series.
620 + 143 (=169 - 26) = 763 2. (3) The series is based on following pattern :
763 + 117 = 880 1.7 + 1.5 = 3.2
28. (3) The pattern is : 3.2 - 0.5 = 2.7
130 + 32 = 139 2.7 + 1.5 = 4.2
139 + 42 = 155 4.2 - 0.5 = 3.7
155 + 52 = 180 3.7 + 1.5 = 5.2
180 + 62 = 216 5.2 - 0.5 = 4.7
216 + 72 = 265 4.7 + 1.5 = 6.2
29. (2) The pattern is : Therefore, the number 5.2 will come in
658 + 72 = 730 place of question mark (?) in the series.
730 + 144 = 874 3. (3) The original series is based on following
874 + 288 = 1162 pattern:
1162 + 576= 1738 1
30. (2) The pattern is : 8 × =4
2
14 + 990 = 1004
4 ×1 = 4
990 4 × 1.5 = 6
1004 + = 1202
5 6 × 2 = 12
198 12 × 2.5 = 30
1202 + = 1251.5 30 × 3 = 90
4
Therefore, the number 28 is wrong. Hence,
 49.5  the new series is as follows:
1251.5 + 16.5  
3 
= 1268
 1
28 × = 14 ....2nd term
1268 + 8.25 = 1276.25 2
31. (3) The pattern is : 14 × 1 = 14 ....3rd term
576 - 224 = 352 14 × 1.5 = 21 - 4th term
752 - 576 = 176 21 × 2 = 42
840 - 752 = 88 Therefore, the fourth term of new series is
884 - 840 = 44 21.
 ? = 884 + 22 = 906 4. (2) The original series is based on following
32. (4) The pattern is : pattern:
55 + 11.15 = 66.15 17 + 0.25 × (1)2 = 17.25
66.15 + 2 × 11.15 = 88.45 17.25 + 0.25 × (2)2 = 18.25
88.45 + 3 ×11.15 = 121.9 18.25 + 0.25 × (3)2
121.9 + 4 × 11.15 = 166.5 = 20.50
166.5 + 5 × 11.15 20.50 + 0.25 × (4)2 = 24.50
= 166.5 + 55.75 = 222.25 24.50 + 0.25 × (5)2 = 30.75
33. (5) The pattern is Therefore, the number 20.75 is wrong.
36 + 13 = 49 Hence, the new series is as follows:
49 + 2 × 13 = 75 20.75 + 0.25 × l2 = 21.00 .... 2nd term
75 + 13 = 88 21.00 + 0.25 × (2)2 = 22.00 .... 3rd term
88 + 2 × 13 = 114 22.00 + 0.25 × (3)2 = 24.25 ....4th term
114 + 13 = 127 Therefore, the fourth term of the new series
is 24.25.
INSURANCE EXAMS 5. (1) The original series is based on following
1. (2) The series is based on following pattern : pattern:
3 + 4 × (2)° = 7 438 + (7)2 = 487
7 + 11 = 18 487 - (6)2 = 451
18 + 4 × (2)1 = 26 451 + (5)2 = 476
26 + 11 = 37 476 + (4)2 = 460
37 + 4 × (2)2 = 53 460 + (3)2 = 469
53 + 11 = 64
LEARN MATHS FROM S.K. RAJU (9811549822, 9811649822)
44
Therefore, the number 447 is wrong. Hence Hence, the wrong number is 75.
the new series is as follows: 10. (4) The given number series is based on the
447 + (7)2 = 496 .....2nd term following pattern:
496 - (6)2 = 460 .....3rd term 4 - 3 = l2
460 + (5)2 = 485 - 4th term 13 - 4 = 9 = 32
485 - (4)2 = 469 38 - 13 = 25 = 52
Therefore, the fourth term of the new series 87 - 38 = 49 = 72
is 485. 168 - 87 = 81 = 92
6. (5) The original series is based on following 289 - 168 = 121 = 112
pattern: Obviously, 166 is the wrong number.
2 × 2 + 3 = 7 11. (3) The number series follows the rule as
7 × 2 + 5 = 19 mentioned below:
19 × 2 + 7 = 45
45 × 2 + 9 = 99
99 × 2 + 11 = 209
209 × 2 + 13 = 431
Therefore, the number 18 is wrong.
Hence, the new series is as follows: Hence 29 is the wrong number.
18 × 2 + 3 = 39 — 2nd term 12. (5) The followed pattern is :
39 × 2 + 5 = 83 — 3rd term
83 × 2 + 7 = 173 - 4th term
173 × 2 + 9 = 355
Therefore, the fourth term of the new series
is 173,
7. (4) The original series is based on following Hence the wrong number is 176.
pattern: 13. (5) The pattern of the number series is :
6 × 1 +1 × 2 = 8 3 + 72 = 3 + 49 = 52
8 × 2 - 2 × 3 = 10 52 + 62 = 52 + 36 = 88
10 × 3 + 3 × 4 = 42 88 + 52 = 88 + 25 = 113
42 × 4 - 4 × 5 = 148 113 + 42 = 113 + 16 = 129
148 × 5 + 5 × 6 = 770 129 + 32 = 129 + 9 = 138
770 × 6 - 6 × 7 = 4578 14. (3) The pattern of the number series is :
Therefore, the number 146 is wrong. 2 × 1 + 1 = 52
Hence, the new series is as follows: 3 × 2 + 2 = 8
146 × 1 + 1 × 2 = 148 8 × 3 + 3 = 27
= 2nd term 27 × 4 + 4 = 112
148 × 2 - 2 × 3 112 × 5 + 5 = 565
= 290 -- 3rd term 15. (1) The pattern of the number series is :
290 × 3 + 3 × 4 6 × 0.5 + 1 = 4
= 882 - 4th term 4 × 1.5 + 2 = 8
Therefore, the fourth term of the new series 8 × 2.5 + 3 = 23
is 882. 23 × 3.5 + 4 = 84.5
8. (1) The given number series is based on the 84.5 × 4.5 + 5 = 385.25
following pattern 16. (4) The pattern of the number series is :
23 = 8; 43 = 64
3
6 = 216; 83 = 512
3
10 = 1000 ; 123 = 1728
17. (2) The pattern of the number series is :
Hence the wrong number is 6. 5 × 1 + 1 × 6 = 11
9. (2) The given number series is based on the 11 × 2 + 2 × 5 = 32
following pattern : 32 × 3 + 3 × 4 = 108
108 × 4 + 4 × 3 = 444
444 × 5 + 5 × 2 = 2230
18. (3) S = (12 - 22) + (32 - 42) + (52 - 62) + .....to
100 terms
LEARN MATHS FROM S.K. RAJU (9811549822, 9811649822)
45
= -3 - 7 - 11 - 15 - .... to 100 terms  61 = l + (n - 1)d
= - (3 + 7 + 11 + 15 + ... to 100 terms)  61 - 1 = (n - 1)6
100  (n - 1)6 = 60
= [2 × 3 + (100 - 1)4]
2  n - 1 = 10
 n = 11
 n 
Sn  2 2a  n  1 d   n 11
Sn = a  l  = 1  61 = 341
2 2
= - 50 × 402 = - 20100
19. (3) Tricky approach  Expression = 341 - 22 = 319
3 5 7 17 19 1 1 1 1 1
+ + ..... + + 26. (1) x = + + +....+ +
4 36 44 5184 8100 1 2 2  3 3  4 7 8 7 9

 1 1 1 1 1  1 1 1 1 1 1 1 1
 1 1 
= 1   +    +    ..... +  81  100  = 1 - + - + - +.....+ - + -
2 2 3 3 4 6 7 7
 4   4 9   9 16 
1 1
+
1 99 8 7 9
=1- = = 0.99
100 100 1 1
20. (4) The pattern is : =1- +
8 63
8 + 6 = 14
504  63  8 449
14 + 18 (= 6 + 12) = 32 = =
32 + 38 (= 18 + 20) = 70 8  63 504
70 + 66 (= 38 + 28) = 136 1 504
136 + 102 (= 66 + 36 )  =  1.1
x 449
= 238
27. (4)
21. (1) The pattern is :
25 + 1 × 16 = 41  1  1   1   1 
1  22  1  2  1  2  .... 1  2  =
41 + 3 × 16 = 41 + 48 = 89    3   4   2011 
89 + 5 × 16 = 89 + 80 = 169
169 + 7 × 16 = 169 + 112 = 281 x
281 + 9 × 16 = 281 + 144 = 425 2  2011
22. (2) The pattern is :
461 + 13 = 474
 1  1  1  1  1  1
 1  2  1  2  1  3  1  3  1  4 1  4 
474 - 9 = 465
465 + 13 = 478       
478 - 9 = 469
 1  1  1   1 
469 + 13 = 482 1   1   .... 1   × 1  
23. (5) The pattern is :  5  5   2011   2011 
(980  2) + 26 = 516
x
(516  2) + 26 = 284 =
(284  2) + 26 = 168 2  2011
(168  2)+ 26 = 110 1 3 2 4 3 5 4 6
(110  2) + 26 = 81  × × × × × × × ....
2 2 3 3 4 4 5 5
24. (5) The pattern is :
4+0=4 2010 2012 x
4 + 6= 10 × =
2011 2011 2  2011
10 + 24 ( = 6 + 18) = 34
34 + 60 (= 6 + 54) = 94 1 2012 x
 × =
94 + 168 (= 6 + 162) = 262 2 2011 2  2011
25. (1) Expression =  x = 2012
(l + 7 + 13 + 19 + .... + 61) + (3 - 5 + 9 - 11 28. (2) The pattern is :
+ .... + 63 - 65)
= (1 + 7 +13 + ... + 61) - 2 ×11 1050  30
= 510
First Part = 1 + 7 + 13 +....+ 61 2
tn= a + (n- 1)d
LEARN MATHS FROM S.K. RAJU (9811549822, 9811649822)
46
33. (2) The given number series is based on the
following pattern :
1548  3 = 516
516  4 = 129
129  3 = 43
43  4 = 10.75
510  26 Hence, 10.75 will replace the question
= 242 mark.
2
34. (4) The given number series is ‘ based on
242  22 the following pattern :
= 100  106 949 × 0.2 = 189.8
2
189.8 × 0.3 = 56.94
110  18 56.94 × 0.4 = 22.776
= 46
2 22.776 × 0.5 = 11.388
46  14
11.388 × 0.6 = 6.8328
= 16. Hence, 56.94 will replace the question
2 mark.
29. (1) The pattern is 35. (1) The given number series is based on
550 - 22 = 550 - 4 = 546 the following pattern :
546 - 32 = 546 - 9 = 537 121 + 23 × 1 = 144
537 - 42 = 537 - 16 = 521 144 + 23 × 2 = 190
521 - 52 = 521 - 25 190 + 23 × 3 = 259
= 496  494  ? = 259 + 23 × 4 = 259 + 92 = 351
496 - 62 = 496 - 36 = 460 Hence, 351 will replace the question mark.
30. (3) The pattern is ; 36. (5) The given number series is based on the
8 + 1 × 13 = 21 following pattern :
21 + 2 ×13 = 21 +26 = 47 14 × 3 + 1.5 = 43.5
47 + 3 × 13 = 47 + 39 = 86 43.5 × 6+ 1.5 × 2 = 264
86 + 4 × 13 = 86 + 52 264 × 12 + 1.5 × 4 = 3174
= 138  140 3174 × 24 + 1.5 × 8 = 76188
138 + 5 × 13 = 138 + 65 = 203 Hence, 3174 will replace the question mark.
203 + 6 × 13 = 203 + 78 = 281 37. (3) The given number series is based on the
31 (2) The pattern is ; following pattern :
4 × 8 - 8 = 32 - 8 = 24 41 × 22 = 164
24 × 7 - 7 = 168 - 7 = 161 164 × 42 = 2624
161 × 6 - 6 = 966 - 6 2624 × 62 = 94464
= 960  965 94464 × 82 = 6045696
960 × 5 - 5 = 4800 - 5 = 4795 Hence 94464 will replace the question
32. (3) The pattern is : mark.
1 × 2=2 38. (1) The pattern is :
2 × 3 = 6 8 2+3=5
6 × 4 = 24 5+4=9
24 × 5 = 120 6 + 5 = 14
120 × 6 = 720 14 + 6 = 20
20 + 7 = 27

LEARN MATHS FROM S.K. RAJU (9811549822, 9811649822)


47
MODEL EXERCISES
1. The interior angles of a polygon are in AP, 10. Let Sn denote the sum of the first ‘n’ terms
the smallest angle is 120° and the common of an AP
difference is 5. Then, the number of sides
S 3n
of the polygon are — S2n = 3Sn. Then, the ratio S is equal to
(1) 16 (2) 9 n

(3) 8 (4) 12 (1) 4 (2) 6


(5) None of these (3) 8 (4) 10
2. A man arranges to pay off a debt of Rs 3600 (5) None of these
in 40 annual instalments which form an 11. The missing number in the series
AP. When 30 of the instalments are paid, 8, 24, 12, 36, 18, 54 is —
he dies leaving one-third of the debt unpaid. (1) 27 (2) 108
Find the value of the first instalment. (3) 68 (4) 72
(1) 55 (2) 53 (5) None of these
(3) 51 (4) 49 12. The sum of the 6th and 15th elements of
(5) None of these an arithmetic progression is equal to the
3. Find 13 + 23 + 33 + .... + 153 sum of 7th, 10th and 12th elements of the
(1) 11025 (2) 13400 same progression. Which element of the
(3) 900 (4) 14400 series should necessarily be equal to zero ?
(5) None of these (1) 10th (2) 8th
4. The value of (3) 1st (4) 9th
(13 + 23 + 33 +....... + 153) - (5) None of these
(1 + 2 + 3 +........ + 15) is — 13. If p, q, r, s are in harmonic progression and
(1) 14280 (2) 14400 p > s, then —
(3) 12280 (4) 13280
1 1
(5) None of these (1) ps < qr (2) q + r = p + s
5. What is the next number in the series given
below ?
1 1 1 1
53, 48, 50, 50, 47 (3) q + p = + (4) None of these
(1) 51 (2) 46 r s
(3) 53 (4) 52 (MAT Exam. Sept. 2003)
(5) None of these 14. What is the eighth term of the sequence
6. In a GP, the first term is 5 and the common 1, 4, 9, 16, 25 ..... ?
ratio is 2. The eighth term is — (1) 8 (2) 64
(1) 640 (2) 1280 (3) 128 (4) 200
(3) 256 (4) 160 (5) None of these
(5) None of these 15. In a geometric progression, the sum of the
7. If the arithmetic mean of two numbers is 5 first and the last term is 66 and the product
and geometric mean is 4, then the numbers of the second and the last but one term is
are — 128. Determine the first term of the series.
(1) 4, 6 (2) 4, 7 (1) 64 (2) 64 or 2
(3) 3, 8 (4) 2, 8 (3) 2 or 32 (4) 32
(5) None of these (5) None of these
8. What is the next number in the series given 16. A sequence is generated by the rule that
below ? the xth term is x 2 + 1 for each positive
2, 5, 9, 14, 20 integer x. In this sequence, for any value x
(1) 25 (2) 26 > 1, the value of (x + l)th term less the value
(3) 27 (4) 28 of xth term is —
(5) None of these (1) 2x2 > + 1 (2) x2+ 1
9. The sum of 40 terms of an AP whose first (3) 2x + 1 (4) x + 2
term is 4 and common difference is 4, will (5) None of these
be — 17. Four different integers form an increasing
(1) 3200 (2) 1600 AP. If one of these numbers is equal to the
(3) 200 (4) 2800 sum of the squares of the other three
(5) None of these numbers, then the numbers are —
LEARN MATHS FROM S.K. RAJU (9811549822, 9811649822)
48
(1) -2, -1, 0, 1 (2) 0, 1, 2, 3 (5) None of these
(3) -1, 0, 1, 2 (4) 1, 2, 3, 4 19. The first three numbers in a series are -3,
(5) None of these 0, 3, the 10th number in the series will be
18. How many terms are there in an AP whose —
fi rst an d fi fth terms are -14 and 2 (1) 18 (2) 21
respectively and the sum of terms is 40 ? (3) 24 (4) 27
(1) 15 (2) 10 (5) None of these
(3) 5 (4) 20

LEARN MATHS FROM S.K. RAJU (9811549822, 9811649822)


49
SHORT ANSWERS Here, n = number of terms = 15
1. (2) 2. (3)
 n n  1 
2 2
3. (4) 4. (1) 15  16 
  =  
5. (4) 6. (1)  2   2 
7. (4) 8. (3) = (120)2 = 14400
9. (1) 10. (2) 4. (1) According to question,
11. (1) 12. (2) (13 + 23 + 33 + ...... + 153) -
13. (4) 14. (1) (1 + 2 + 3 + .... + 15)
15. (4) 16. (2)
 n n  1   n n  1 
2
17. (2) 18. (3)
19. (3) =   -  
 2   2 
1. (4) 2. (1)
3. (4) 4. (3)
15  16 
2
5. (2) 6. (3) 15  16 
=  -  
7. (5) 8. (2)  2   2 
9. (1) 10. (4) = (120)2 - (120)
11. (3) 12. (5) =120 × 119 = 14280
13. (2) 14. (1) 5. (4) According to question,
15. (4) 16. (1) 53, 48, 50, 50, 47....
17. (5) 18. (1) The above series can be splitted into two
19. (1) series one in ascending order and other
EXPLANATIONS in descending order 53, 50, 47 and other
1. (2) Let the polygon has n sides. is 48, 50, 52.
Given, the smallest interior angle is 120°, Hence, 52 will be the next number.
hence the greatest exterior angle will be 6. (1) According to question,
(180° -120°) = 60° nth term of a GP = an-1
We know sum of exterior angles of a polygon  8th term = 5 × (2)8-1 = 5 × (2)7
= 360° = 5 × 128 = 640
60 + 55 + 50 + .... = 360 7. (4) Let the two numbers be x and y.
{Common difference = -5} Then, AM,
n x y
 2 [2a + (n - 1) d] = 360 =5
2
 x + y = 10
n
[120 + (n - 1) × -5] = 360 and GM, xy = 4 ...(i)
2
 n2 - 25n + 144 = 0  xy = 16
 n = 9, 16  (x - y) 2 = (x + y)2 - 4xy
Number of sides cannot be 16. 100 - 64 = 36
Hence, n = 9 x-y=6 ...(ii)
2. (3) According to question, Or
Sum of 40 instalments S40 Solving Eqs. (i) and (ii),
= 3600 = 20 (2a + 39d) x = 8 and y = 2
 2a + 39d = 180 ...(i) 8. (3) According to question,
Sum of 30 instalments 2 + 3 =5; 5 + 4 = 9;
S30 = 2400 = 15 (2a + 29d) 9 + 5 = 14; 14 + 6 = 20;
 2a + 29d = 160 ...(ii) 20 + 7 = 27
Solving Eqs. (i) and (ii), we get Hence, the next number of the series will
a = 51 and d = 2 be 27.
 The value of first instalment 9. (1) According to question,
= Rs 51 n
3. (4) According to question, we have, S40= [2a + (n - l)d]
2
 n  n  1 
2
= 20 [4 + 39 × 4]
13 + 23 + 33 +..... + n3 =   = 20 × 160 = 3200
 2 
LEARN MATHS FROM S.K. RAJU (9811549822, 9811649822)
50
10. (2) Let a be the first term and d be the
1 1 1 1
common difference.  q + = +
r s p
n
Then, Sn = (2a + (n - l)d] Hence the none of these be answer
2 14. (2) According to question,
2n 1, 4, 9, 16, 25
S 2= [2a+ (2n - l)d] (1) 2 (2)2 (3)2 (4)2 (5)2
2
Each term of the progression is the square
3n of a natural number.
and S3n = [2a + (3n - l)d] ] Hence, the eighth term of the sequence will
2
Given, S2n = 3Sn be (8)2 = 64
15. (2) Let the last term be n,
2n then a + arn-1 = 66
 2 [2a + (2n - 1)d] =
and ar. arn-2 = 128
a2rn-1’ = 128
n From Eqs. (i) and (ii),
2 [2a + (n - 1)d]
2 a (66 - a) = 128
 4a + (4n - 2)d = 6a + (3n - 3)d  a2 - 66a + 128 = 0
 d (4n - 2 - 3n + 3) = 2a  a = 64, 2
2a 16. (3) According to question,
d = (x + l)th term -xth term
n 1
= (x + 1)2 + 1 - (x2 + 1)
2an 2 = x2 + 2x + 1 + 1 - x2 - 1
 Sn = = 2x + 1
n 1
17. (3) By hit and trial or common sense, we
12an 2 have,
and S3n =
n 1 2 = (-1)2 + (0)2 + (1) 2
Hence the numbers are -1, 0, 1, 2
Sn 2an 2 n 1 1 S3n 18. (2) According to question,
S = × 2 = 6 = S =6
3n n  1 12 an n T5 = a + (n - 1).d
11. (1) According to question, 2 = - 14 + 4d
8, 24, 12, 36, 18, 54 16
d= = 4
4
n
 Sn = 2 [2a + (n - 1) × d]
Hence, 27 will come in the blank space.
12. (2) Let the first term and common term of n
the AP be a and d respectively. 40 = [-28 + (n- 1) × 4]
2
Then, (a + 5d) + (a + 14d) =
(a + 6d) + (a + 9d) + (a + 11d)  80 = - 28n + 4n2 - 4n
 2a + 19d = 3a + 26 d  4n2 - 32n - 80 = 0
 a + 7d = 0 n2 - 8n - 20 = 0
 8th term is 0.  (n - 10)(n + 2) = 0
13. (4) According to question,  n= 10 (  n  -2)
If p, q, r, s are in HP. 19. (3) According to question,
a = -3. d = 3
1 1 1 1
 p , q , , are in AP  T10= a + (10 -1). d
r s T10 = -3 + 9 × 3 = 24
1 1 1 1
 q -p= -
s r

LEARN MATHS FROM S.K. RAJU (9811549822, 9811649822)


1
NUMBER SYSTEM
NATIONALISED BANKS (Bank Of Maharashtra PO
& IBPS SO/MT/SO Exam. 25.05.2008)
1. The difference between a two digit number 5. In a two digit positive number, the digit in
and the number obtained by interchanging the unit’s place is equal to the square of
the positions of its digits is 36. What is the the digit in ten’s place, and the difference
difference between the two digits of that between the number and the number
number ? obtained by interchanging the digits is 54.
(1) 4 What is 40% of the original number ?
(2) 9 (1) 15.6 (2) 39
(3) 3 (3) 37.2 (4) 24
(4) Cannot be determined (5) None of these
(5) None of these (Indian Overseas Bank PO
(Canara Bank PO Exam. 09.02.2003) Exam. 15.06.2008)
2. If the numerator of a fraction is increased 6. If the numerator of a fraction is increased
by 20% and its denominator by 25% , then 1
by and the denominator is decreased by
3 4
the fraction so obtained is . What is the
5 1 33
original fraction ? , the new fraction obtained is . What
3 64
3 was the original fraction ?
(1)
5 9 5
(1) (2)
5 11 7
(2)
7 3 7
(3) (4)
5 7 9
(3) (5) None of these
8
(4) Cannot be determined (Indian Overseas Bank PO
(5) None of these Exam. 15.06.2008)
(Syndicate Bank PO Exam. 10.10.2004) 7. Twice the square of a number is more than
3. The number obtained by interchanging the eleven times the number by 21. The
two digits of a two digit number is less than number can have which of the following
the original number by 27. If the difference values ?
between the two digits of the number is 3, 7 3
what is the original number ? (1) 4 or - (2) 7 or -
2 2
(1) 74
(2) 63 7 9
(3) 3 or - (4) or - 4
(3) 85 2 2
(4) Cannot be determined (5) None of these
(5) None of these (Indian Overseas Bank PO
(Union Bank of India PO Exam. 15.06.2008)
Exam. 27.11.2005) 8. The product of two consecutive even
4. If the numerator of a fraction is increased numbers is 9408. Which is the greater of
by 200% and the denominator of the the two numbers ?
fraction is increased by 150%, the resultant (1) 96 (2) 98
7 (3) 94 (4) 92
fraction is . What is the original fraction ? (5) None of these
10
(Andhra Bank PO Exam. 14.09.2008)
3 7 9. If the numerator of a fraction is increased
(1) (2)
4 12 by 300% and the denominator is increased
11
(3) 7 (4) 9 by 100% , the resultant fraction is 1 .
11 11 19
(5) None of these What was the original fraction ?
LEARN MATHS FROM S.K. RAJU (9811549822, 9811649822)
2
(5) None of these
15 13
(1) (2) (Indian Overseas Bank PO
19 19 Exam. 05.04.2009)
5 4 3 4
(3) (4) 15. The difference between the th of th of
9 9 4 5
(5) None of these
(Andhra Bank PO Exam. 14.09.2008) 1 2
a number and th of th of the same
10. What will be the greater of two numbers 6 5
whose product is 640, if the sum of the two number is 648. What is the number ?
numbers, exceeds their difference by 32 ? (1) 1110 (2) 1215
(1) 45 (2) 50 (3) 1325 (4) 1440
(3) 55 (4) 40 (5) None of these
(5) None of these (Indian Overseas Bank PO
(Bank Of Baroda Specialist Exam. 05.04.2009)
Officer Exam. 05.10.2008) 16. If the positions of the digits of a two digit
11. The product of two successive numbers is number are interchanged, the number
4032. Which is the greater of the two obtained is smaller than the original
numbers ? number by 27. If the digits of the number
(1) 63 (2) 64 are in the ratio of 1 : 2, what is the original
(3) 65 (4) 66 number ?
(5) None of these (1) 36
(Oriental Bank of Commerce PO (2) 63
Exam. 21.12.2008) (3) 48
12. The product of two successive numbers is (4) Cannot be determined
9506. Which is the smaller of the two (5) None of these
numbers ? (United Bank of India PO
(1) 96 (2) 97 Exam. 21.06.2009)
(3) 98 (4) 99 17. If the numerator of a fraction is increased
(5) None of these by 20% and the denominator is increased
(Canara Bank PO Exam. 15.03.2009)
3
13. If the numerator of a fraction is increased by 25% , the fraction obtained is . What
by 200% and the denominator is increased 5
was the original fraction ?
1
by 400% , the resultant is 1 . What was 5
20 (1)
the original fraction ? 7

3 11 4
(1) 1 (2) (2)
4 10 7

6 1 3
(3) (4) 1 (3)
5 2 8
(5) None of these (4) Cannot be determined
(Canara Bank PO Exam. 15.03.2009) (5) None of these
14. If the numerator of a fraction is increased (United Bank of India PO
by 200% and the denominator is increased Exam. 21.06.2009)
18. Twice the square of a number is six times
5 the other number. What is the ratio
by 350% , the resultant fraction is . What
12 between the first number and the second
was the original fraction ? number ?
(1) 1 : 4
5 5 (2) 2 : 5
(1) (2)
9 8 (3) 1 : 3
7 11 (4) Cannot be determined
(3) (4) (5) None of these
12 12
(Andhra Bank PO Exam. 05.07.2009)
LEARN MATHS FROM S.K. RAJU (9811549822, 9811649822)
3
19. If the numerator of a fraction is increased (5) None of these
by 10% and the denominator is increased (Punjab & Sind Bank PO
Exam. 16.05.2010)
11
by 20% the resultant fraction is . What 24. Two-third of the first number is equal to
15 the cube of the second number. If the
was the original fraction ? second number is equal to twelve percent
7 4 of 50, what is the sum of the first and the
(1) (2) second numbers ?
12 5
(1) 330 (2) 360
3 5 (3) 390 (4) 372
(3) (4) (5) None of these
5 12
(5) None of these (Punjab & Sind Bank PO
(PNB Specialist Officer’s Exam. 16.05.2010)
Exam. 16.08.2009) 25. The sum of five consecutive numbers is
20. The sum of four consecutive even numbers 270. What is the sum of the second and
is 284. What would be the smallest number ? the fifth number ?
(1) 72 (2) 74 (1) 108
(3) 68 (4) 66 (2) 107
(5) 70 (3) 110
(Corporation Bank PO (4) Cannot be determined
Exam. 22.11.2009) (5) None of these
(Bank Of Baroda PO Exam.30.05.2010)
1 26. The sum of five consecutive even numbers
21. A number when subtracted by of itself
7 of set-A is 220. What is the sum of a different
gives the same value as the sum of all the set of five consecutive numbers whose
angles of a triangle. What is the number ? second lowest number is 37 less than
(1) 224 (2) 210 double of the lowest number of set -A ?
(3) 140 (4) 350 (1) 223 (2) 225
(5) 187 (3) 235 (4) 243
(Corporation Bank PO (5) None of these
Exam. 22.11.2009) (Central Bank Of India PO
22. If the numerator of a fraction is increased Exam. 25.07.2010)
by 150% and the denominator of the 27. The digit in the unit’s place of a three digit
fraction is increased by 350% , the resultant number is thrice the digit in the ten’s place
and the digit in the hundred’s place is two-
25
fraction is , what is the original fraction ? third of the digit in the ten’s place. If the
51 sum of the three digits of the number is
11 11 14, what is the three digit number ?
(1) (2) (1) 932
77 15
(2) 239
15 13 (3) 326
(3) (4) (4) Cannot be determined
17 15
(5) None of these (5) None of these
(Corporation Bank PO (United Bank Of India PO
Exam. 09.05.2010) Exam. 14.11.2010)
23. The sum of three consecutive odd numbers 28. What is the least number to be subtracted
and three consecutive even numbers from 945 to make it a perfect square ?
together is 231. Also, the smallest odd num- (1) 16 (2) 30
ber is 11 less than the smallest even (3) 24 (4) 45
number. What is the sum of the largest odd (5) None of these
number and the largest even number ? (PNB Management Trainee
(1) 82 Exam. 28.11.2010)
(2) 83 29. If the numerator of a fraction is increased
(3) 74 by 200% and the denominator of the
(4) Cannot be determined fraction is increased by 250% , the resultant

LEARN MATHS FROM S.K. RAJU (9811549822, 9811649822)


4
3 4 4
fraction is . What is the original fraction ? (1) (2)
14 9 5
1 3 8 8
(1) (2) (3) (4)
2 14 9 11
1 2 (5) None of these
(3) (4) (Punjab & Sind Bank PO
6 3
Exam. 23.01.2011)
(5) None of these 35. Sum of two numbers is equal to sum of
(PNB Management Trainee square of 11 and cube of 9. Larger number
Exam. 28.11.2010) is (5)2 less than square of 25. What is the
30. The sum of the digits of a two digit number value of the sum of twice of 24 percent of
is 15 and the difference between the two the smaller number and half of the larger
digits of the two digit number is 3. What is number ?
the product of the two digits of the two-digit (1) 415 (2) 400
number ? (3) 410 (4) 425
(1) 56 (5) None of these
(2) 63 (UCO Bank PO Exam. 30.01.2011)
(3) 42 36. The difference between the sum of four
(4) Cannot be determined co nse cuti ve odd nu mbe rs and three
(5) None of these consecutive even numbers together is 20.
(PNB Management Trainee Also, the largest even number is 5 more
Exam. 28.11.2010) than the largest odd number. What is the
31. The sum of the suqares of two consecutive sum of the smallest odd number and the
even numbers is 6500. Which is the smaller smallest even number ?
number ? (1) 75
(1) 54 (2) 52 (2) 77
(3) 48 (4) 56 (3) 85
(5) None of these (4) Cannot be determined
(PNB Management Trainee (5) None of these
Exam. 28.11.2010) (UCO Bank PO Exam. 30.01.2011)
32. The sum of five consecutive even numbers 37. Sum of square of first number and cube of
of set - A is 280. What is the sum of different second number together is 568. Also square
set of five consecutive numbers whose low- of the second number is 15 less than square
est number is 71 less than double the of 8. What is the value of three-fifth of the
lowest number of set-A ? first number ? (assuming both the numbers
(1) 182 (2) 165 are positive)
(3) 172 (4) 175 (1) 18 (2) 8
(5) None of these (3) 9 (4) 16
(Bank Of Maharashtra (5) None of these
Exam. 19.12.2010) (Bank Of Baroda PO Exam. 13.03.2011)
33. What is the least number that can be added 38. The sum of six consecutive even numbers
to the number 1020 to make it a perfect of set-A is 402. What is the sum of another
square ? set-B of four consecutive numbers whose
(1) 65 (2) 12 lowest number is 15 less than double the
(3) 59 (4) 4 lowest number of set-A ?
(5) None of these (1) 444 (2) 442
(Indian Bank PO Exam. 02.01.2011) (3) 440 (4) 446
34. If the numerator of a fraction is increased (5) None of these
by 150% and the denominator of the (Allahabad Bank PO Exam. 17.04.2011)
fraction is increased by 300% , the resultant 39. The sum of nine consecutive odd numbers
5 of set-A is 621. What is the sum of different
fraction is . What is the original fraction ? set of six consecutive even numbers whose
18

LEARN MATHS FROM S.K. RAJU (9811549822, 9811649822)


5
lowest number is 15 more than the lowest 1
number of set-A ?  17 3
(1) 498 (2) 468 (5)  of 3444 
 13 
(3) 478 (4) 488
(IDBI Bank Officer Exam.16.09.2012)
(5) None of these
45. The cost of 8 dozen of eggs is Rs 256. Which
(Indian Overseas Bank PO
calculation is needed to find the cost of 9
Exam. 22.05.2011)
eggs ?
40. The product of three consecutive even
(1) (9 × 256) × (8  12)
numbers is 4032. The product of the first
(2) (12 × 256)  (8 × 9)
and the third number is 252. What is five
(3) (8 × 256)  (9 × 12)
times the second number ?
(4) (9 × 256) × (8 × 12)
(1) 80 (2) 100
(5) (9 × 256)  (8 × 12)
(3) 60 (4) 70
(IDBI Bank Officer Exam.16.09.2012)
(5) 90
46. On a particular day, sweets were to be
(IBPS Bank PO/MT CWE 17.06.2012)
equally distributed among 495 students of
41. The sum of the ages of 4 members of a
a school. However, on that particular day
family 5 years ago was 94 years. Today,
396 students from a nearby school also
when the daughter has been married off
joined them. Hence each student present
and replaced by a daughter-in-law, the sum
on that day got four sweets less. How many
of their ages is 92. Assuming that there has
sweets were distributed ?
been no other change in the family struc-
(1) 5445 (2) 4950
ture and all the people are alive, what is
(3) 4455 (4) 3960
the difference in the age of the daughter
(5) None of these
and the daughter-in-law ?
(IBPS RRBs Office Assistant
(1) 22 years (2) 11 years
CWE Exam. 09.09.2012)
(3) 25 years (4) 19 years
47. A student requires 324 pencils in 6 years.
(5) 15 years
How many dozen pencils will he require in
(IBPS Bank PO/MT CWE 17.06.2012)
14 years ?
42. Which of the following is second largest ?
(1) 52 (2) 64
(1) 138.6 - 38.4 + 479.3
(3) 59 (4) 62
(2) 36.5 - 844.6 + 1289
(5) None of these
(3) 931 - 564 + 156
(IBPS RRBs Office Assistant
(4) 564 - 213 + 120
CWE Exam. 09.09.2012)
(5) 130 - 461 + 888
48. What is the least number to be added to
(IDBI Bank Officer Exam.16.09.2012)
4523 to make it a perfect square ?
43. Which of the following is the largest ? (You
(1) 101 (2) 34
are not expected to calculate the exact
(3) 238 (4) 121
value)
(5) None of these
(1) (56 × 15)  42
(IBPS RRBs Office Assistant
(2) (25 × 72)  62
CWE Exam. 09.09.2012)
(3) (6 × 441)  72
49. How many natural numbers are there lying
(4) (28 × 78)  56
between 134 and 467 which are divisible
(5) (32 × 48)  26
by 7 ?
(IDBI Bank Officer Exam.16.09.2012)
(1) 46 (2) 47
44. Which of the following is the smallest ? (You
(3) 49 (4) 51
are not expected to calculate the exact
(5) None of these
value)
1
(Indian Overseas Bank PO
1
 7 3 Online Exam. 01.09.2013)
5  2
(1)  of 1250  (2)  13 of 4112 
9   
SBI PO EXAMS
1 1 1. The denominator of fraction is 2 more than
 5  2  15  2
thrice its numerator. If the numerator as
(3)  of 3221 (4)  of 412 
 19   11  well as denominator is increased by one,

LEARN MATHS FROM S.K. RAJU (9811549822, 9811649822)


6
(3) 21
1
he fraction becomes . What was the (4) Cannot be determined
3 (5) None of these
original fraction ? (SBI Banks PO Exam. 20.08.2000)
4 3 6. If the digits of a two digit number are
(1) (2) interchanged the newly formed number is
13 11
more than the original number by 18, and
5 5 sum of the digit is 8 then what was the
(3) (4) original number ?
13 11
(5) None of these (1) 53
(SBI Associate Banks PO (2) 26
Exam. 14.02.1999) (3) 35
2. In a fraction, if numerator is increased by (4) Cannot be determined
40% , and denominator is increased by (5) None of these
80% . Then what fraction of the old is the (SBI Associate Banks PO
new fraction ? Exam. 21.07.2002)
7. By how much is three-fifth of 350 greater
1 7 than four-seventh of 210 ?
(1) (2)
2 9 (1) 120 (2) 210
(3) 95 (4) 110
7 (5) None of these
(3) (4) Data inadequate
18 (SBI Banks PO Exam. 18.05.2003)
(5) None of these 8. In a two digit number the digit in the unit’s
(SBI Associate Banks PO place is twice the digit in the ten’s place
Exam. 16.07.2000) and the number obtained by interchanging
4 5 4 the digits is more than the original number
3. of of a number is greater than of by 27. What is 50% of the original number ?
15 7 9
(1) 36 (2) 63
2 (3) 48 (4) 18
of the same number by 8. What is half of (5) None of these
5
(SBI PO Exam. 09.01.2005)
that number ?
9. What least number would be subtracted
(1) 630 (2) 315
from 427398 so that the remaining number
(3) 210 (4) 105
is divisible by 15 ?
(5) None of these
(1) 6 (2) 3
(SBI Banks PO Exam. 20.08.2000)
(3) 16 (4) 11
4. A classroom has equal number of boys and
(5) None of these
girls. Eight girls left to play Kho-Kho, leaving
(SBI PO Exam. 26.11.2006)
twice as many boys as girls in the class-
10. The difference between a two digit number
room. What was the total number of girls
and the number obtained by interchanging
and boys present initially ?
the positions of its digits is 36. What is the
(1) Cannot be determined
difference between the two digits of that
(2) 16
number ?
(3) 24
(1) 4
(4) 32
(2) 9
(5) None of these
(3) 3
(SBI Banks PO Exam. 20.08.2000)
(4) Cannot be determined
5. The sum of four numbers is 64. If you add
(5) None of these
3 to the first number, 3 is subtracted from
(SBI Associate Banks PO
the second number, the third is multiplied
Exam. 07.01.2007)
by 3 and the fourt is divided by three, then
11. By how much is two-fifth of 200 greater
all the results are equal. What is the
than three-fifth of 125 ?
difference between the largest and the
(1) 15 (2) 3
smallest of the original numbers ?
(3) 5 (4) 30
(1) 32
(5) None of these
(2) 27
LEARN MATHS FROM S.K. RAJU (9811549822, 9811649822)
7
(SBI Associate Banks PO (1) 48 (2) 44
Exam. 07.01.2007) (3) 52 (4) 54
12. If the numerator of a fraction is increased (5) None of these
by 200% and the denominator of the (SBI PO Preliminary (Tire-I)
fraction Is increased by 120% , the resultant Exam. 27.04.2008)
18. Two numbers are such that the sum of twice
4
fraction is . What is the original fraction ? the first number and thrice the second
11 number is 36 and the sum of thrice the first
4 3 number and twice the second number is 39.
(1) (2) Which is the smaller number ?
15 11
(1) 9 (2) 5
5 6 (3) 7 (4) 3
(3) (4) (5) None of these
12 11
(5) None of these (SBI PO Preliminary (Tire-I)
(SBI PO Preliminary (Tire-I) Exam. 27.04.2008)
Exam. 27.04.2008) 19. a, b, c, d and e are five consecutive even
numbers. If the sum of ‘a’ and ‘d’ is 162,
3 2 1 what is the sum of all the numbers ?
13. th of th of th of a number is 249.6.
4 9 5 (1) 400
What is 50% of that number ? (2) 380
(1) 3794 (2) 3749 (3) 420
(3) 3734 (4) 3739 (4) Cannot be determined
(5) None of these (5) None of these
(SBI PO Preliminary (Tire-I) (SBI PO Preliminary (Tire-I)
Exam. 27.04.2008) Exam. 27.04.2008)
14. The sum of the two digits of a two digit 20. What is 348 times 265 ?
number is 12 and the difference between (1) 88740 (2) 89570
the two digits of the two digit number is 6. (3) 95700 (4) 92220
What is the two-digit number ? (5) None of these
(1) 39 (SBI PO Preliminary (Tire-I)
(2) 84 Exam. 27.04.2008)
(3) 93 21. One-fifth of a number is 62. What will 73%
(4) Cannot be determined of that number be ?
(5) None of these (1) 198.7 (2) 212.5
SBI PO Preliminary (Tire-I) (3) 226.3 (4) 234.8
Exam. 27.04.2008) (5) None of these
15. There are 7 dozen candles kept in a box. If (SBI PO Preliminary (Tire-I)
there are 14 such boxes, how many candles Exam. 27.07.2008)
are there in all the boxes together ? 22. If the numerator of a fraction is increased
(1) 1176 (2) 98 by 300% and the denominator is increased
(3) 1216 (4) 168 5
(5) None of these by 500% , the resultant fraction is . What
12
(SBI PO Preliminary (Tire-I)
Exam. 27.04.2008) was the original fraction ?
16. Which of the following smallest numbers 8 5
should be added to 6659 to make it a perfect (1) (2)
5 11
square
(1) 230 (2) 65 12 5
(3) 98 (4) 56 (3) (4)
5 7
(5) None of these (5) None of these
(SBI PO Preliminary (Tire-I) (SBI PO Preliminary (Tire-I)
Exam. 27.04.2008) Exam. 27.07.2008)
17. What is the greater of two numbers whose
product is 1092 and the sum of the two 5 6 7 8 9
23. In the fractions , , , and are
numbers exceeds their difference by 42 ? 14 11 9 13 10

LEARN MATHS FROM S.K. RAJU (9811549822, 9811649822)


8
arranged in ascending order of their values, (RBI Officer Grade ‘B’ Online
which one will be the fourth ? Exam. 25.08.2013)
7 5
(1)
9
(2) INSURANCE EXAMS
14
1. If a number is divided by the other number,
8 9
(3) (4) the division is 3 , if their difference is 28,
13 10
5
(5) None of these what is the bigger number of the two ?
(SBI PO Preliminary (Tire-I) (1) 35 (2) 140
Exam. 27.07.2008) (3) 70 (4) 84
6 3 (5) -42
24. By how much is th of 506 lesser than (United India Insurance Co.
11 5
th of 895 ? AAO Exam. 21.04.2002)
(1) 178 (2) 219 2. In a pair of fractions, fraction ‘A’ is twice
(3) 143 (4) 261 the fraction ‘B’ and the product of two
(5) None of these 2
(SBI PO Preliminary (Tire-I) fractions is . What is the value of fraction
25
Exam. 27.07.2008) ‘A’ ?
25. In a three digit number the digit in the
unit’s place is twice the digit in the ten’s 1 2
(1) (2)
place and 1.5 time s the digit in the 5 5
hundred’s place. If the sum of all the three
digits of the number is 13, what is the 1
(3) (4) Data inadequate
number ? 25
(1) 364 (2) 436 (5) None of these
(3) 238 (4) 634 (LIC Assistant Administrative
(5) None of these Officer (AAO) Exam. 24.04.2005)
(SBI PO Preliminary (Tire-I) 3. In a two digit number the digit in unit’s
Exam. 27.07.2008) place is four times the digit in ten’s place
and sum of the digits is equal to ten. What
is the number ?
RBI GRADE-B OFFICER EXAMS (1) 14 (2) 41
1. Four of the following five parts numbered
(3) 82 (4) Data inadequate
1, 2, 3, 4 and 5 are equal. Which of the
(5) None of these
following is not equal to the other four.?
(LIC Assistant Administrative
(1) 242 - 122 + 112  14 =
Officer (AAO) Exam. 24.04.2005)
(2) 17 × 12 + 59 × 4 =
4. If the digits of a two digit number are
(3)15 × 28 + 20 =
interchanged the newly formed number is
(4) 27 × 16 + 56  8 =
more than the original number by 18, and
(5) 185 x 6 + 2 - 23 × 5
sum of the digit is 8 then what was the
(RBI Grade-B Officer
original number ?
Exam. 17.11.2002)
(1) 53
2. The denominators of two fractions are 5 and
(2) 26
7 respectively. The sum of these fractions
(3) 35
41 (4) Cannot be determined
is . On interchanging the numerators, (5) None of these
35
their sum becomes 43 . The fractions are (LIC Assistant Administrative
Officer (AAO) Exam. 2006)
2 4 3 4 5. For which of the following values of x the
(1) and (2) and
5 7 5 7 inequality x (x + 3) < 10 is satisfied ?
(1) x > 2, x < - 5
4 2 3 5
(3) and (4) and (2) - 5 < x < 2
5 7 5 7 (3) - 2 < x < 5
(5) None of these (4) x < - 2, x > 5

LEARN MATHS FROM S.K. RAJU (9811549822, 9811649822)


9
(5) None of these 11. Which of the following set of fractions is in
(LIC Assistant Administrative ascending order ?
Officer (AAO) Exam. 2006)
13 11 7 8 11 13 7 8
6. In a division sum, the divisor is 10 times (1) , , , (2) , , ,
the quotient and 5 times the remainder. If 15 13 8 9 13 15 8 9
the remainder is 46, the dividend is : 8 7 13 11 7 8 11 13
(1) 4236 (2) 4306 (3) , , , (4) , , ,
9 8 15 13 8 9 13 15
(3) 4336 (4) 5336
(5) None of these
(United India Insurance Co.
(LIC Assistant Administrative
(AAO) Exam. 11.03.2007)
Officer Exam. 2008)
7. A certain number of two digits is three times
12. Which of the following numbers is exactly
the sum of its digits. If 45 be added to it,
divisible by 99 ?
the digits are reversed. The number is
(1) 114345 (2) 135792
(1) 72 (2) 32
(3) 3572404 (4) 913464
(3) 27 (4) 23
(LIC Assistant Administrative
(United India Insurance Co.
Officer (AAO) Exam. 07.06.2009)
(AAO) Exam. 11.03.2007)
13. In a division sum, the divisor is 10 times
8. When a two-digit odd number is divided by
the quotient and 5 times the remainder. If
a two digit even number, the quotient is
the remainder is 46, the dividend is :
0.625. If the odd number is 5 less than the
(1) 4356 (2) 4816
even number, then which of the following
(3) 5096 (4) 5336
will definitely be the ratio of odd and even
(LIC Assistant Administrative
numbers respectively ?
Officer (AAO) Exam. 07.06.2009)
(1) 5 : 8
14. If r is the remainder when each of 7654,
(2) 8 : 5
8506 and 9997 is divided by the greatest
(3) 6 : 9
number d (d > 1), then d - r is equal to
(4) Cannot be determined
(1) 14 (2) 18
(5) None of these
(3) 24 (4) 28
(LIC Assistant Administrative
(New India Assurance AO
Officer Exam. 2008)
Exam. 25.10.2009)
9. Two third of one-fourth of a number is forty
15. For numbers a and b, define * as a a* b = (a
per cent of another number. Which of the
+ b) + (a - b). Then thevalue of (1 * 2) * 3 is
following statements is true about these
numbers ? 2 1
(1) The second number is 2.4 times the first (1) - (2) -
3 5
number
(2) The first number is 2.4 times the second 1
(3) 0 (4)
number 2
(3) The first number is forty per cent of the (New India Assurance AO
second number Exam. 25.10.2009)
(4) The first number is sixty per cent less 16. The number 2564 × 6425 is the square of a
than the first number. natural number n. The sum of digits of n is
(5) None of these (1) 7 (2) 14
(SBI PO Preliminary (Tire-I) (3) 21 (4) 28
Exam. 27.07.2008) (New India Assurance AO
10. Certain number of pieces of an article are Exam. 25.10.2009)
to be packed in boxes, such that each box 17. The unit’s digit of 132003 is
contains 145 pieces. If after packing them (1) 1 (2) 3
in 32 boxes 25 pieces are left out, what was (3) 7 (4) 9
the number of pieces to be packed ? (New India Assurance
(1) 4566 (2) 4655 AO Exam. 25.10.2009)
(3) 4465 (4) 4640 18. Let x be the product of two numbers 3, 659,
(5) None of these 893, 456, 789, 325, 678 and 342, 973, 489,
(LIC Assistant Administrative 379, 256. The number of digits in x is
Officer Exam. 2008) (1) 32 (2) 34
LEARN MATHS FROM S.K. RAJU (9811549822, 9811649822)
10
(3) 35 (4) 36 27. The prime number 1999 can be written as
(New India Assurance AO a2 - b2, where a and b are natural numbers.
Exam. 25.10.2009) Then the value of a2 + b2 is
19. The number (248 - 1) is exactly divisible by (1) 1998000 (2) 1998001
two numbers between 60 and 70. The (3) 1999000 (4) 1999001
numbers are : (New India Insurance AAO
(1) 63 and 65 (2) 63 and 67 Exam. 22.05.2011)
(3) 61 and 65 (4) 65 and 67 28. If a number of two digits is k times the sum
(New India Assurance of its digits, then the number formed by.
AO Exam. 25.10.2009) inter-changing the digits is the Sum of the
20. The six-digit number 5 A B B 7 A is a digits multiplied by:
multiple of 33 for digits A and B. Which of (1) 9 + k (2) 10 + k
the following could be possible value of A + (3) 11 - k (4) k - 1
B? (General Insurance Corporation
(1) 8 (2) 9 AAO Exam. 11.12.2011)
(3) 10 (4) 14 29. The first 44 positive integers are written in
(New India Assurance AO an order to form a larger number
Exam. 25.10.2009) N = 1 2 3 4 5 6 7 8 9 10 11 12 ............. 42
21. The remainder when 71987 is divided by 5 is 43 44
(1) 1 (2)2 when N is divided by 45, then the remainder
(3) 3 (4) 4 is
(New India Insurance AAO (1) 5 (2) 7
Exam. 22.05.2011) (3) 9 (4) 11
22. Unit’s digit in 132003 is (Oriental Insurance Company
(1) 1 (2) 3 AAO Exam. 08.04.2012)
(3) 7 (4) 9 30. When a natural number N is divided by 5,
(New India Insurance AAO the remainder is 2, when divided by 7, the
Exam. 22.05.2011) remainder is 3, when divided by 9, the
23. Which is the largest ? remainder is 4. If N is the smallest number,
(1) 1010 (2) (210)5 then the sum of the digits of N is
(3) (510)2 (4) (45)4 (1) 7 (2) 8
(New India Insurance (3) 11 (4) 13
AAO Exam. 22.05.2011) (Oriental Insurance Company
24. If (236 -1) = 68a19476735, where a is any AAO Exam. 08.04.2012)
digit, then the value of a is 31. In the sum below, F = 0, and the other
(1) 1 (2) 3 letters represent the digits 1, 2, 3, 4, 5, or
(3) 5 (4) 7 6, with each digit used exactly once and
(New India Insurance the two digit number AB is a prime num-
AAO Exam. 22.05.2011) ber.
25. Let x be an odd natural number. If x is
divided by 6, it leaves a remainder y. If y2 is AB
divided by 4, it leaves remainder of z. Which  CD
of the following must be true for z ? EFG
(1) z = 3 (2) z = 5
(3) z = 1 (4) z is even
(New India Insurance Then the value of A + 2B is
AAO Exam. 22.05.2011) (1) 10 (2) 8
26. Given the numbers : 25555 , 33333, 62222 These (3) 7 (4) 9
can be written in ascending order as (Oriental Insurance Company
(1) 25555 , 33333, 62222 AAO Exam. 08.04.2012)
(2) 33333 , 25555 , 62222 32. Let d be a two digit number. If half of d
(3) 25555 , 62222 , 33333 exceeds one third of d by the sum of the
(4) 62222 , 25555 , 33333 digits in, d, then the sum of the digits in d
(New India Insurance is
AAO Exam. 22.05.2011) (1) 6 (2) 8

LEARN MATHS FROM S.K. RAJU (9811549822, 9811649822)


11
(3) 9 (4) 15 37. What is the smallest number by which 4320
(United India Insurance AAO be divided to make it a perfect cube ?
Exam. 03.06.2012) (1) 15 (2) 20
33. Ho w many 9’s are th ere in the (3) 24 (4) 25
999998999992 ? (NICL (GIC) AO (Finance)
(1) 7 (2) 9 Exam. 08.09.2013 (Paper-I)
(3) 11 (4) 13 38. When ‘n’ is divided by 5 the remainder is 2.
(United India Insurance What is the remainder when n2 is divided
AAO Exam. 03.06.2012) by 5 ?
34. Which one of the following is the largest ? (1) 2 (2) 3
1
1
(3) 1 (4) 4
(1) 2 2 (2) (NICL (GIC) AO Exam. 08.09.2013
53
1 (Paper-I)
1
(3) 4
(4) 115 39. The least number which should be added
8
(United India Insurance AAO to 2497 so that the sum is exactly divisible
Exam. 03.06.2012) by 5, 6, 4 and 3 is:
(1) 3 (2) 13
1 (3) 23 (4) 33
35. By how much is th of 428 smaller than
4 (NICL (GIC) AO Exam. 08.09.2013
5 (Paper-I)
th of 216 ? 40. The sum of one-half, one-third and one-
6
fourth of a number exceeds the number by
(1) 61 (2) 67 12. The number is:
(3) 73 (4) 79 (1) 144 (2) 154
(5) None of these (3) 90 (4) 174
(United India Insurance AO (NICL (GIC) AO (Finance)
Exam. 26.05.2013 Exam. 15.12.2013)
36. The product of two successive numbers is 41. If the sum of two numbers is 42 and their
4032. Which is the greater of the two product is 437, then the absolute difference
numbers ? between the numbers is:
(1) 63 (2) 64 (1) 3 (2) 4
(3) 65 (4) 66 (3) 5 (4) 7
(5) None of these (NICL (GIC) AO (Finance)
(United India Insurance Exam. 15.12.2013)
AO Exam. 26.05.2013)

LEARN MATHS FROM S.K. RAJU (9811549822, 9811649822)


12
SHORT ANSWERS 13. (4) 14. (1)
NATIONALISED BANKS 15. (3) 16. (2)
& IBPS PO/MT/SO 17. (3) 18. (2)
1. (1) 2. (3) 19. (1) 20. (2)
3. (4) 4. (2) 21. (3) 22. (3)
5. (1) 6. (5) 23. (2) 24. (4)
7. (2) 8. (2) 25. (3) 26. (2)
9. (1) 10. (4) 27. (2) 28. (3)
11. (2) 12. (2) 29. (3) 30. (4)
13. (1) 14. (2) 31. (1) 32. (3)
15. (2) 16. (2) 33. (2) 34. (2)
17. (5) 18. (4) 35. (3) 36. (2)
19. (2) 20. (3) 37. (2) 38. (4)
21. (2) 22. (3) 39. (3) 40. (1)
23. (5) 24. (1) 41. (2)
25. (5) 26. (5)
27. (2) 28. (4) EXPLANATIONS
29. (5) 30. (5) NATIONALISED BANKS
31. (4) 32. (4) & IBPS PO/MT/SO
33. (4) 34. (1) 1. (l) Let the unit’s digit be x and ten’s digit be
35. (5) 36. (2) y.
37. (3) 38. (2) Also let y > x.
39. (5) 40. (1)  Number = 10y + x and number obtained
41. (1) 42. (5) by interchanging the digits = 10x + y
43. (5) 44. (4)  10y + x- 10x - y = 36
45. (5) 46. (3)
 9y - 9x = 36
47. (5) 48. (1)
49. (2)  9 (y - x) = 36
y-x=4
SBI PO EXAMS x
1. (2) 2. (2) 2. (3) Let the fraction by y .
3. (2) 4. (4)
After the respective increase in numerator
5. (1) 6. (3)
and denominator,
7. (5) 8. (1)
9. (2) 10. (1) 6
11. (3) 12. (1) x
5 6 4 x 24x
13. (5) 14. (4) = × × y = 25y
5 5 5
15. (1) 16. (2) y
4
17. (3) 18. (5)
19. (5) 20. (4) 24x 3
21. (3) 22. (5)  25y = 5
23. (1) 24. (4)
25. (2) 5
 Original fraction =
8
RBI GRADE-B OFFICER EXAMS 3. (4) Let the two-digit number be 10x + y
1. (4) 2. (2) where y > x.
New number = 10y + x
INSURANCE EXAMS  10y + x - 10x - y = 27
1. (5) 2. (2) y-x=3
3. (4) 4. (3) We reach at no unique conclusion, as
5. (5) 6. (4) several such numbers are possible. Hence,
7. (3) 8. (1) cannot be determined.
9. (2) 10. (5)
11. (2) 12. (1)

LEARN MATHS FROM S.K. RAJU (9811549822, 9811649822)


13
x x  400 30
4. (2) Let the original fraction be =
y y  200 19
According to the question
x 30  2 15
 y= =
x
300 4  19 19
100 7 12x 7
10. (4) Let one of the numbers be x.
250 10  10y
= =
10
y 640
100  Second number  
x
x 7 10 7 According to the question,
 y= × =
10 12 12 640 640
5. (1) Let the number = 10x + x2. x+ =x- + 32
x x
According to the question,
10x2 + x - 10x - x2 = 54 640
2 × = 32
 9(x 2- x) = 54 x
 x2 - x = 6  32x = 2 × 640
 x2 - 3x + 2x - 6 = 0
2  640
 x(x - 3) + 2(x - 3) = 0 =x= = 40
32
 (x - 3) (x + 2) =0
 x =3 or x = -2 640
 Second number =
 Number = 39 x
40 640
 40 % of 39 = 39 × = 15.6 = = 16
100 40
x  Larger number = 40
6. (5) Let the original fraction be y . 11. (2) From the given alternatives,
63 × 64 = 4032
According to the question, 12. (2) From the given alternatives,
97 × 98 = 9506
5
x  Smaller number = 97
4 33
2 = 64 x
y 13. (1) Let the original fraction be y .
3
According to the question,
x 33 4 2 11
 y= × × = x  300 21
64 5 3 40
=
7. (2) Let the number be x. y  500 20
According to the question,
3x 21
2x2 - 11x = 21  5y =
 2x2- 11x - 21 = 0 20
 2x2- 14x + 3x - 21 = 0 x 21 5 7 3
 2x(x - 7) + 3(x -7) = 0  y= × = =1
20 3 4 4
 (x - 7) (2x + 3) = 0
3
x = 7 or - x
2 14. (2) Let the original fraction be y .
8. (2) From the given alternatives,
98 × 96 = 9408 According to the question,

x 300
x
9. (1) Let the original fraction be y . 100 = 5
450 12
According to the question, y
100

LEARN MATHS FROM S.K. RAJU (9811549822, 9811649822)


14
x 5 45 5 6x 180  7
 y = × =  = 180  x = = 210
12 30 8 7 6
15. (2) Let the number be x. x
According to the question, 22. (3) Let the original fraction be .
y
4 3 2 1 According to the question,
x× × -x× × = 648
5 4 5 6
x  250 25
3x x =
y  450 51
 - = 648
5 15
x 25 450 15
9x  x 8x  y= × =
 = 648  = 648 51 250 17
15 15
23. (5) The smallest odd number = x
648  15  The smallest even number = x + 11
x= = 1215  x + x + 2 + x + 4 + x + 11 + x + 13 + x + 15
8
16. (2) Let the number be 10x + y where x > y = 231
 6 x + 45 = 231
 10x + y - 10y - x = 27  6x = 231 - 45 = 186
x-y=3 186
 2y - y = 3 x = 6
= 31
 y = 3 x = 6  Required sum
 Number = 63 = x + 4 + x + 15
x = 2x + 19 = 2 × 31 + 19
17. (5) Let original fraction be y , = 62 + 19 = 81
24. (1) Second number
According to the question,
50  12
120 = =6
x x  24 3
100
100 3
125 =  =
y  25 5 63  3
5
y  First number = = 324
100 2
 Required sum = 324 + 6 = 330
x 3 25 5
 y= × = 25. (5) x + x + 1 + x + 2 + x + 3 + x + 4
5 24 8 = 270
18. (4) Let the numbers be x and y respectively.  5x + 10 = 270
 2x2 = 6y  5x = 260
 x 2 = 3y
260
No ratio can be obtained between x and y. x= = 52
19. (2) Let the original fraction be x. 5
 x + 1 + x + 4 = 2x + 5
x  110 11 = 2 × 52 + 5 = 109
 y  120 =
15 26. (5) x + x + 2 + x + 4 + x + 6 + x + 8 = 220
 5x + 20 = 220
x 11 12 4
 y= × =  x = 40
15 11 5 Second lowest number of different set
20. (3) x + x + 2 + x + 4 + x + 6 = 284 = 2 × 40 - 37 = 43
 4x = 284 - 12 = 272  Required sum
= 42 + 43 + 44 + 45 + 46 = 220
272
x= = 68 27. (2) Let the ten’s digit be x.
4
 Unit’s digit = 3x
x 2x
21. (2) x - = 180 Hundred’s digit =
7 3

LEARN MATHS FROM S.K. RAJU (9811549822, 9811649822)


15
2x  x + y = 850 ...(i)
 x + 3x + = 14 x = 252 - 52 = (25 + 5) (25 - 5)
3
= 30 × 20 = 600
 3x + 9x + 2x = 14 × 3  y = 850 - 600 = 250
 14x = 14 × 3  Required sum
x=3
250  24  2 600
 Number = 239 = +
28. (4) 30 × 30 = 900 100 2
31 × 31 = 961 = 120 + 300 = 420
 Required least number = 945 - 900 = 45 36. (2) Let the consecutive odd numbers be :
x, x + 2, x + 4 and x + 6
x  Largest even number = x + 11
29. (5) Let Original fraction = y
 Other even numbers
= x + 7 and x + 9
x  300 3  x + x + 2 + x + 4 + x + 6-
 y  350 = (x + 7 + x + 9 + x + 11) = 20
14
 x - 15 = 20
x 3 35 1
 y= × =  x = 15 + 20 = 35
14 30 4  Required sum
30. (5) Let the number be 10x + y and x > y. =x+x + 7
 x + y = 15 ... (i) = 2x + 7 = 2 × 35 + 7 = 77
and, x - y = 3 ...(ii) 37. (3) Let the first number be x and the second
On adding x = 9 number be y.
From equation (i),  y2 = 82 - 15 = 64 - 15 = 49
9 + y = 15 y = 7
 y =15 - 9 = 6  x2 + 73 = 568
 xy = 9 × 6 = 54  x2 + 343 = 568
31. (4) 562 + 582 = 3136 + 3364 = 6500  x2 = 568 - 343 = 225
 Smaller number = 56
x = 225 = 15
32. (4) For set-A,
x + x + 2 + x + 4 + x + 6 + x + 8 = 280 3
 5x + 20 = 280  15 × 5 = 9
 5x = 280 - 20 = 260
402
260 38. (2) Third even number = -1
x= = 52 6
5 = 67 - 1 = 66
The lowest number of Set-B  Smallest even number = 62
2 × 52 - 71 = 33  Smallest number of Set - B
 Required sum = 33 + 34 + 35 + 36 + 37 = 2 × 62 - 15 = 109
= 175  Required sum = 109 + 110 + 111 + 112
33. (4) 32 × 32 = 1024 = 442
 Required number 39. (5) Fifth number of set-A
= 1024 - 1020 = 4
621
x = = 69
34. (1) Let the original fraction be y 9
Smallest number of Set-A = 61
x  250  Smallest number of Set- B
5
 y  400 = = 61 + 15 = 76 . 
18  Required sum = 76 + 78 + 80 + 82 + 84 +
86 = 486
x 5 400 4
 y= × = 40. (1) Second number
18 250 9
4032
35. (5) x + y = 112 + 93, = = 16
Where x = larger number 252
 x + y = 121 + 729  Required answer = 5 × 16 = 80
LEARN MATHS FROM S.K. RAJU (9811549822, 9811649822)
16
41. (1) Sum of the present ages of old family
891x  495 x
= 94 + 20 = 114 years  = 4
Father + Mother + Son + Daughter-in-law 891  495
= 92 years  396x = 4 × 891 × 495
Required difference 4  891  495
= 114 - 92 = 22 years x = = 4455
396
42. (5)
(1) 138.6 - 38.4 + 479.3 = 579.5 47. (5) 324 pencils = 27 dozens of pencils
(2) 36.5 - 844.6 + 1289 = 480.9  6 years  27 dozens
(3) 931 - 564 + 156 = 523 27
(4) 564 - 213 + 120 = 471  14 years  × 14 = 63
6
(5) 130 - 461 + 888 = 557
48. (1) 67 × 67 = 4489
56  15 68 × 68 = 4624
43. (5) (1) = 52.5
16  Required number
= 4624 - 4523 = 101
25  72 49. (2) Smallest number divisible by 7 = 140
(2) = 50
66 Largest number = 462
By an = a + (n - 1) d
28  78
(3) = 54 462 = 140 + (n - 1) × 7
56  (n - 1) × 7 = 462 - 140 = 322
28  78  n - 1 = 322  7 = 46
(4) = 39  n = 46 + 1 = 47
56
32  48 SBI PO EXAMS
(5) = 59.07
26
44. (4) x
1. (2) Let the original fraction be y
1
 5 2  According to question,
(1) 1250    26
 9 3x = y - 2 .... (i)
1 x 1 1
 7 3 and y  1 =
(2)  4112    28
3
 13  or, 3x + 3 = y + 1
1 3x = y - 2 ....(ii)
 5 2 Si nce both th e e qu ati on s are same,
(3)  3221    29
 19  therefore, we can’t calculate the required
answer. But, here in given options, the
1
option like Data inadequate or can not
 15  2
(4)  412    24 determined is not given. Hence, option (2)
 11  or (5) is required answer. Since option (2)
1 fulfills our conditions therefore this is most
 17  3 suitable choice.
(5)  3444    36
 13  x
45. (5)  CP. of 8 × 12 eggs 2. (2) Let the fraction = y
= Rs. 256
the new fraction
256
 C.P of one egg = 140
8  12 of x
7 x
100
= =  
256  9 180 9 y
 CP. of 9 eggs = of y
8  12 100
46. (3) Let the number of sweats be x.  The New fraction = g of original fraction.
3. (2) Let the number be x
x x
 - = 4
495 891
LEARN MATHS FROM S.K. RAJU (9811549822, 9811649822)
17
6. (3) Let the unit’s digit be y and ten’s digit
 4 5  4 2 
 of of x   of of x  = 8 be x
 15 7  9 5   Number = 10x + y
4 5 4 2  New number after interchange = 10y + x
or, × x - × x = 8 As given,
15 7 9 5
10y + x - 10x - y = 18
4 8  9 (y - x) = 18
or, x - x = 8 y-x=2 ....(i)
21 45
Again, x + y = 8 ...(ii)
60 x  56x From (i) and (ii)
or, = 8
315 2y = 10
4 y=5
or, x = 8  x = 3 [From(i)]
315
Number = 10x + y = 10 × 3 + 5 = 35
8 7. (5) The required difference
or, x = 315 ×
4 3 4
or, x = 630 = × 350 - × 210
5 7
x 630 = 210 - 120 = 90
 2 = 2 = 315 8. (1) Let the ten’s digit be x.
4. (4) Let the no. of boys and girls each be x.  Unit’s digit = 2x
2 (x - 8) = x  Original number
2x - 16 = x = 10x + 2x = 12x
x = 16 On interchanging the digits, the new
number
 Total no. of boys and girls present initially
in the classroom = 2x = 32 = 10 × 2x + x = 21 x
5. (1) Let the four numbers be a, b, c and d. According to question,
a + b + c + d = 64 21x - 12x = 27
 9x = 27
d
Also, a + 3 = b - 3 = 3c = 27
3 = = 3
b=a+6 9
 Original number = 12 x = 12 × 3 = 36.
a 9. (2) 427398 = 15 × 28493 + 3
c= +1
3 Th e le ast nu mber w hi ch sho ul d be
d = 3a + 9 subtracted from 427398 so that It becomes
divisible by 15 = 3.
a
a+a+6 + + l + 3a + 9 = 64 10. (1) Let the unit’s digit be x and ten’s digit
3 be y. Also let y > x.
a  Number  10y + x and number obtained
5a + + 16 = 64 by Interchanging the digits  10x + y
3
 10y + x - 10x - y = 36
16  9y - 9x = 36
a = 48
3  9 (y - x) = 36
y- x = 4
3
a = 48 × 11. (3) Required difference
16
a=9 2  3 
=  of 200  -  of 125 
b = a + 6 = 15 5  5 
9 = 80 - 75 = 5
c= +1=4
3 x
d = 3a + 9 - 36 12. (1) Let the original fraction be
y
Difference between the largest and the According to the question,
smallest numbers = 36 - 4 = 32

LEARN MATHS FROM S.K. RAJU (9811549822, 9811649822)


18
17. (3) Let the numbers be x and y.
300
x According to the question,
100 4 x + y - (x - y) = 42
220 = 11
y  2y = 42
100
42
30x 4
y= = 21
2
 22y =
11
1092
x = = 52
x 4 22 4 21
 y= × = 18. (5) Let the numbers be x and y.
11 30 15
13. (5) Let the number be x. According to the question,
According to the question, 2x + 3y = 36 ...(i)
3x + 2y = 39 ...(ii)
1 2 3 By equation (i) × 3 - (ii) × 2,
x × × × = 249.6
5 9 4 6x + 9y - 6x - 4y = 108 - 78
 x = 249.6 × 30 = 7488  5y = 30
 50% of 7488 30
y= =6
1 5
= 7488 × = 3744
2 From equation (i), we have,
14. (4) Let the two digit number be 2x + 3 × 6 = 36
= 10x + y where x > y  2x = 36 - 18 = 18
Accordig to the question, 18
x + y = 12 and x - y = 6 x= =9
2
Adding these equations, x + y
 The smaller number = 6
x  y  12 19. (5) Let the five consecutive even numbers
x y  6 be x, x + 2, x + 4, x + 6 and x + 8 respectively.
According to the question,
2x  18
x + x + 6 = 162  2x = 162 - 6 = 156
18 156
x= =9 x= = 78
2 2
 y = 12 - 9  = 3  Sum of all numbers
Number = 93 = 78 + 80 + 82 + 84 + 86 = 410
Again, let the number be 10 x + y 20. (4) 265 × 348 = 92220
where x < y. 21. (3) Let the number be x.
Then, According to the question,
x + y = 12
and x
= 62
y- x =6 5
Adding these equations, 2y = 18  x = 62 × 5 = 310
18 73
y= =9  73% of 310= 100 × 310 = 226.3
2
 From equation (i), x = 3
 Number = 39 x
22. (5) Let the original fraction be
Thus we get two numbers. This doesn’t y
satisfy the uniqueness of answer. According to the question,
15. (1) Total number of candles = 7 × 12 × 14
= 1176 x  400 5
=
16. (2) 81 × 81 = 6561 y  600 12
82 × 82 = 6724
To make 6659 a perfect square, the number x 5 6 5
to be added = 6724 - 6659 = 65  y= × =
12 4 8

LEARN MATHS FROM S.K. RAJU (9811549822, 9811649822)


19
23. (1) The decimal equivalent of fractions : or, 5x - 3y = 0 ...(i)
Again, x - y = 28
5 6 7 8
= 0.36 : = 0.545 : = 0.78 : = 0.62 or, 5x - 5y = 140
14 11 9 13 From equation (i) and (ii) we get y = - 70
9 and x = - 42
= 0.9
10 x
Clearly, 2. (2) Let the fraction A = y
0.36 < 0.545 < 0.62 < 0.78 < 0.9
 According to question,
5 6 8 7 9
i.e. < < < < x
14 11 13 9 10 B = 2y
24. (4) Required difference
3 6 x x 2
= × 895 - × 506  y × 2y = 25
5 11
= 537 - 276 = 261
x2 4 x 2
25. (2) Let the ten’s digit be x. 2 =  y=
y 25 5
 Unit’s digit = 2x
3. (4) Let the two digit number be 10x + y
2x
and hundred’s digits =  According to question, y = 4x
1.5 and 4x + y = 10
According to the question, or, 4x + 4x = 10
2x
x + 2x +
1.5
= 13 x = , y 10 = 5
8
1.5 x  3x  2x 4. (3) Let the unit’s digit be y and ten’s digit
 = 13 be x
1.5
 Number = 10x + y
 6.5x = 13 × 1.5  New number after interchange
13  15 = 10y + x
x= =3 As given,
6.5
10y + x - 10x - y = 18
 Unit’s digit = 6 and hundred’s digit
 9 (y - x) = 18
= 6 =4 y-x=2 ....(i)
1.5 Again, x + y = 8 ...(ii)
 Number = 436 From (i) and (ii)
Note : This question can be solved by oral 2y = 10
calculation, taking the alternatives into y= 5
consideration.  x = 3 [From (i)]
Numeber =10x + y = 10 × 3 + 5 = 35
RBI GRADE-B OFFICER EXAMS 5. (5) Given inequality :
1. (4) 27 × 16 + 56  8 = 439 x (x + 3) < 0
Corresponding equation:
3 4 21  20 41 4 3
2. (2) + = = and + x (x + 3) = 0
5 7 35 35 5 7  x = 0 or x = -3
28  15 43
= =
35 35 The given inequality holds for - 3 < x < 0
6. (4) The divisor is ten times the quotient and
INSURANCE EXAMS 5 times the remainder,
1. (5) Let two numbers be x and y. Here, remainder = 46
 According to the question,  Divisor = 5 × 46 = 230
 Quotient × 10 = 230
x 3
= 230
y 5  Quotient =
= 23
10
LEARN MATHS FROM S.K. RAJU (9811549822, 9811649822)
20
 Di vi de nd = D iviso r × Qu otie nt + 7 8
Remainder = 0.875 ; = 0.89
= 230 × 23 + 46 = 5290 + 46 = 5336 8 9
7. (3) Let the two digit number be Clearly,
= 10y + x. 0.85 < 0.87 < 0.875 < 0.89
According to the question, 11 13 7 8
10y + x = 3 (x + y)  < < <
13 15 8 9
 10y + x = 3x + 3y
12. (1) A number divisible by 99 must be
 10y - 3y + x - 3x = 0 divisible by 9 as well as 11.
 7y - 2x = 0 ....(i) Clearly, 114345 is divisible by both 9 and
and 10 y + x + 45 = 10 x + y 11 i.e. 99.
 9x - 9y = 45 13. (4) Let quotient = Q and remainder = R
 9 (x - y) = 45  Divisor = 10Q = 5R
Now, R = 46  10Q = 5 × 46
45
 x -y= = 5 Now, Q = 23, R = 46
9 and divisor = 5 × 46 = 230
x=y+5 ....(ii)  Di vi de nd = D iviso r × qu otie nt +
Now, 7y - 2x = 0 Remainder
 7y - 2 (y + 5) = 0 = 230 × 23 + 46 = 5290 + 46 = 5336
[From equation (ii)] 14. (1) Tricky approach
 7y - 2y - 10 = 0 Required number (d) =
 5y = 10 HCF of (7654 - r), (8506 - r) and (9997 - r)
= HCF of (8506 - 7654), (9997 - 8506) and
10 (9997 - 7654)
y= =2
5 = HCF of 852. 1491 and 2343 = 213
 From equation (ii), Remainder after dividing 9997 by 213 = 199
x=2+5=7  d - r = 213 - 199 = 14
 Number = 2 × 10 + 7 = 27 15. (3) 1 * 2 = (1 + 2)  (1 - 2) = -3
Note : This problem can be easily solved  (1 * 2) * 3 = - 3 * 3
by hit and trial method out of the given = (-3 + 3) + (-3 -3)
alternatives. 0
8. (1) Let the odd number be = =0
6
2n + 1, (n < 4).
 Even number = 2n - 4 16. (2) Tricky approach
n2 = 2564 × 6425
2n  1 625 = (52)64 × (26)25
Now, = 0.625 =
2n  4 1000 = 5128 × 2128 × 222
 n = 564 × 264 × 211
2n  1 5
= = (5 × 2)64 × 2048
2n  4 8 = 1064 × 2048
9. (2) Let the numbers be x and y.  Sum of digits
x 2 = 2 + 0 + 4 + 8= 14
Then, × = 40% of y 17. (3) Tricky approach
4 3
31 = 3; 32 = 9; 33 = 27, 34 = 81, 35 = 243
x 2 x 12  On dividing 2003 by 4, the remainder = 3
 = y  y=  Unit’s digit in the expansion of 32003
6 5 5
= Unit’s digit in the expansion of 33 = 7
10. (5) Number of pieces 18. (2) Tricky approach
= 32 × 145 + 25 3659893456789325678
= 4640 + 25 = 4665
 37 × 1017 and 342973489379256  3 ×
11. (2)Decimal equivalent of: 1014
13 11  37 × 1017 × 3 × 1014 = 111 × 1031
= 0.87; = 0.85  Number of digits = 31 + 3 = 34
15 13
19. (1) 248 - 1 = (224 + 1) (224 - 1)
= (224 + 1) (212 + 1) (212 - 1)
LEARN MATHS FROM S.K. RAJU (9811549822, 9811649822)
21
= (224 + 1) (212 + 1) (26 + 1) (26 - 1) 29. (3) Of the given alternatives,
 Required numbers = 26 + 1 and 26 - 1 Re main de r = 9 becau se , wh en i t is
= 65 and 63 subtracted from the number, the unit’s digit
20. (2) Tricky approach will be 5 and number is divisible by 45.
A number is divisible by 33 if it is divisible
by 3 and 11 both For divisibility by 11,
 5 + B +7 = A + B + A
 2A = 12 30. (4)
A=6
 For divisibility of 5 6 B B 7 6 by 3, B = 3
 Number = 563376
A + B = 6 + 3 = 9
21. (3) 71 =7, 72 = 49; 73 = 343;
74 = 2401; 75 = 16807
Hence, unit’s digit repeats after index 4.
Remainder when 1987 is divided by 4 = 3
Unit’s digit = 3
Hence, remainder on division by 5 = 3  B = 9 × 1 + 4 = 13
22. (3) 31 = 3, 32 = 9, 33 = 27, 34 = 81, 35 = 243 A = 7 × 13 + 3 = 94
i.e. the unit’s digit repeats itself after power N = 5 × 94 + 2 = 472
4.  Sum of the digits = 4 + 7 + 2 = 13
Remainder when 2003 is divided by 4 = 3 31. (1) AB = 43
 Required unit’s digit = Unit’s digit in the CD = 62
product 33 = 7 AB + CD = 105
23. (2) (2l10)5 = (25)10 = (32)10  A + 2B = 4 + 2 × 3 = 10
(510)2 = (52)10 = (25)10 32. (3) d = 10y + x
(45)4 = (42)10 = (16)10 10y  x 10y  x
and 1010  - =x+y
2 3
24. (1) 22 - 1 = 4 - 1 = 3
24 - 1 = 16 - 1 = 15 30y  3x  20y  2x
26 - 1 = 64 - 1 = 63  =x+y
6
28 - 1 = 256 - 1 = 255
Hence, if n = even number, then (2n - 1), is  10y + x = 6 (x + y)
divisible by 3. A number is divisible by 3, if If x + y = 9,
the sum of its digits is divisible by 3. a = 1 Number = 6 × 9 = 54
25. (3) x = 6 Q + y 54 54
y2 = 4Q1 + z  -
2 3
The value of z may be 1, 2 or 3.
= 27 - 18 = 9 ( = 5 + 4)
The value of y may be 1, 3, or 5 33. (2) The number of 9’s will be less by 1.
as if 2 or 4 be the value, y2 will be exactly Number of 9’s in 99999899999
divisible by 4. = 10
z = 1
26. (2) 25555 = (25)1111 = (32)1111 34. (2) 4
8 > 2
33333 = (33)1111 = (27)1111
32222 = (62)1111 = (36)1111
3
5= 12
54 =
12
625
33333 < 25555 < 62222 4
8= 12
83 =
12
512
27. (2) a2 - b2 = 1999
 (a + b) (a - b) = 1999
3
5> 4
8
 (1000 + 999)(1000 - 999) = 1999 Now,
 a + b2 = (1000)2 + (999)2
2
3
5= 15
55 =
15
3125
= 1000000 + 998001 = 1998001
28. (3) 10x + y = k (x + y) 5
11 = 15
113 = 15
1331
 10y + x = 11y + 11x - 10x - y
= 11 (x + y) - k (x + y)
 3 5 > 5 11
= (11 - k) (x + y)
LEARN MATHS FROM S.K. RAJU (9811549822, 9811649822)
22
35. (3) Required difference
 Required number = (60 - 37) = 23
 5 1
=  216   428   40. (1) Number = x.
 6 4
x x x
= 180 - 107 = 73  + + = x + 12
36. (2) From the given alternatives 2 3 4
63 × 64 = 4032 6x  4x  3x
 = x + 12
12
37. (2)
13x
 = x + 12
12
x
 x+ = x + 12
12
x
 = 12
12
 4320 = 2 × 2 × 2 × 2 × 2 × 5 × 3 × 3 × 3
= 22 × 33 × 22 × 5  x = 12 × 12 = 144
41. (2) If the numbers be x and y , then
 Required number
= 22 × 5 = 20 x + y = 42
38. (4) n = 5 × q + 2 xy = 437
where q = quotient (x - y) 2 = (x + y) 2 - 4xy
On squaring both sides, = (42)2 - 4 × 437
n2 = (5q + 2)2 = 1764 - 1748 = 16
= 25q2 + 20q + 4 x - y= 16 = 4
= 5(5q2 + 4q) + 4
 Remainder = 4
39. (3) LCM of 5, 6, 4 and 3 = 60

LEARN MATHS FROM S.K. RAJU (9811549822, 9811649822)


23
MODEL EXERCISES
1. There are two Examination rooms A and B. (1) 1225 (2) 24505
If 10 candidates are sent from A to B, the (3) 62525 (4)52650
number of students in each room is the (5) None of these
same. If 20 candidates are sent from B to 8. In a class, 50 students play Cricket, 20
A, the number of students in A is double students play Football and 10 students play
the number of students in B. Find the both Cricket and Football. How many play
number of students in each room. at least one of these two games ?
(1) 100 in A and 80 in B (1) 10 (2) 80
(2) 80 in A and 100 in B (3) 50 (4) 60
(3) 120 in A and 100 in B (5) None of these
(4) 100 in A and 120 in B 9. A clock strikes 4 takings 9s. In order to
(5) None of these strike 12 at the same rate, the time taken
2. A club consists of members whose ages are is
in AP, the common difference being 3 (1) 36 s (2) 27 s
months. If the youngest member of the club (3) 30 s (4) 33 s
is just 7 years old and the sum of the ages (5) None of these
of all the members is 250 years, then the 10. At the first stop on his route, a driver
number of members in the club are unloaded 2/5 of the packages in his van.
(1) 15 (2) 20 After a unloaded another three packages
(3) 25 (4) 30 at his next stop, 1/2 of the original num-
(5) None of these ber of packages remained. How many
3. Afte r be in g set u p, a company packages were in the van before the first
manufactured 6000 scooters in the third delivery ?
year and 7000 scooters in the seventh year. (1) 25 (2) 10
Assuming that the production increases (3) 30 (4) 36
uniformly by a fixed number every year, (5) None of these
what is the production in the tenth year ? 11. An Army Commander wishing to draw up
(1) 7850 (2) 7650 his 5180 men in the form of a solid square
(3) 7750 (4) 7950 found that he had 4 men less. If he could
(5) None of these get four more men and form the solid
4. Which of the following is true ? square, the number of men in the front row
A. 99/101<97/99<95/97 is
B. 95/97<97/99<99/101 (1) 72 (2) 68
C. (95/97)2>(97/99)2>(99/101)2 (3) 78 (4) 82
(D. (99/101)2>(97/99)2>(95/97)2 (5) None of these
(1) Only A (2) Only B 12. A tin of oil was 4/5 full. When six bottles of
(3) B and C (4) B and D oil were taken out and four botdes of oil
(5) None of these were poured into it, it was 3/4 full. How
5. Find the whole n umber which when many bottles of oil can it contain ?
increased by 20 is equal to one-sixth times H) 20 (2) 10
the new number (3) 30 (4) 40
(1) 7 (2) 5 (5) None of these
(3) 3 (4) 4 13. To win an election, a candidate needs 3/4
(5) None of these of the votes cast. If after 2/3 of the votes
6. A number when divided by 765 leaves a have been counted, a candidate has 5/6 of
remainder 42. What will be the remainder what he needs, then what part of the
if the number is divided by 17 ? remaining votes does he still need ?
(1) 8 (2) 5 (1) 1/8 (2) 7/12
(3) 7 (4) 6 (3) 1/4 (4) 3/8
(5) None of these (5) None of these
7. The natural numbers are divided into 14. The number of two-digit numbers exactly
groups as (1), (2, 3), (4, 5, 6), (7, 8, 9, 10) divisible by 3 is
and so on. The sum of the numbers in the (1) 33 (2) 32
50th group is (3) 31 (4) 30
LEARN MATHS FROM S.K. RAJU (9811549822, 9811649822)
24
(5) None of these 20. How many numbers are there between 500
and 600 in which 9 occurs only once ?
2
15. The highest score in an innings was of (1) 19 (2) 18
9 (3) 20 (4) 16
2 (5) None of these
the total score and the next highest was 21. If such numbers which are divisible by 5
9
and also those which have 5 as one of the
of the remainder. These scores differ by 8
digits are eliminated from the numbers 1
runs. What was the total score in the
to 60, how many numbers would remain ?
innings ?
(1) 40 (2) 47
(1) 152 (2) 162
(3) 53 (4) 45
(3) 142 (4) 132
(5) None of these
(5) None Of these.
22. The sum of two numbers, one of which is
16. A girl counted in the following way on the
one-third of the other is 36. The smaller
fingers of her left hand; she started by number is
calling the thumb 1, the index finger 2, the (1) 6 (2) 7
middle finger 3, the ring finger 4, the little (3) 8 (4) 9
fingers 5 and then reversed direction calling (5) None of these
the ring fingure 6, the middle finger 7 and 23. Two times a two-digit number is 9 times
so on. She counted upto 1994. She ended the number obtained by reversing the digits
counting on which finger ? and sum of the digits is 9. The number is
(1) The middle finger (1) 72 (2) 54
(2) The index finger (3) 63 (4) 81
(3) The thumb (5) None of these
(4) The ring finger 24. The students present in an auditorium are
(5) None of these asked to stand in rows. If there were 4
17. If 6 years are subtracted from the present students extra in a row there would be 4
age of Randheer and the remainder is less rows. However, if 2 students were less
divided by 18, then the present age of his in a row, there would be 4 more rows. The
grandson Anup is obtained. If Anup is 2 nu mber o f students presen t in the
years younger to Mahesh whose age is 5 auditorium is
years, what is the age of Randheer ? (1) 80 (2) 96
(1) 84 years (2) 48 years (3) 100 (4) 128
(3) 60 years (4) 96 years (5) None of these
(5) None of these 25. Out of 80 students in a class, 25 are
18. One of a group of swans, 7/2 times the studying Commerce, 15 Mathematics and
square root of the number are playing on 13 Physics. 3 are studying Commerce and
the shore of the pond. The two remaining Mathematics, 4 are studying Mathematics
are inside the pond. What is the total and Physics and 2 are studying Commerce
number of swans ? and Physics. 1 student is studying all the
(1) 10 (2) 14 th ree su bj ects tog ethe r. H ow man y
(3) 12 (4) 16 students are not studying any of the three
(5) None of these subjects ?
19. The fuel indicator in a car shows one-fifth (1) 35 (2) 40
of the fuel tank as full. When 22 more litres (3) 20 (4) 15
of fuel are poured into the tank, the (5) None of these
indicator rests at the three-fourth of the full
mark. Find the capacity of the fuel tank.
(1) 25 L (2) 35 L
(3) 30 L (4) 40 L
(5) None of these

LEARN MATHS FROM S.K. RAJU (9811549822, 9811649822)


25
SHORT ANSWERS  5x = 20  x = 4
1. (3) 2. (3) 6. (1) Let the number be (765x + 42).
3. (3) 4. (4) When this numbe is divided by 17, then
5. (4) 6. (1) quotient will be (45x + 2) and remainder
7. (3) 8. (4) will be 8.
9. (2) 10. (3) 7. (3) Let S = 1 + 2 + 4 + 7 +.. .+ Tn
11. (1) 12. (4) or S = 1 + 2 + 4 +... + Tn-1 + Tn
13. (2) 14. (4) Subtracting, we get
15. (2) 16. (4) 0 = 1 + [1 + 2 + 3 + ... (n - 1)] - Tn
17. (3) 18. (4) T = 1 + 2 + 3 + .... + (n - l) + l
19. (4) 20. (2)
n n  1
21. (1) 22. (4) = 1
23. (4) 24. (2) 2
25. (1)  First number of 50th term
50  49
EXPLANATIONS =
2
+ 1 = 1226
1. (3) Here, A - 10 = B + 10
 Sum of numbers of 50th term
 A - B = 20  ...(i)
= 1226 + 1227 + ..... upto 50th term
and A + 20 = 2(B - 20)
 A - 2B = -60 ...(ii) 50
= [2 × l226 + (50 - l) × 1]
From Eqs. (i) and (ii), A = 100, B = 80 2
2. (3) Let n be the number of members in the = 25 × 2501 = 62525
club. 8. (4) Number of students who play cricket
= 50
n 3
2  7  n  1  Number of students who play Football = 20
2 
Then, 250 =
12  Number of students who play both games
n 1 1 =10
 250 = 2 14  4n  4  Number of students who play only Cricket
  = 50 - 10 = 40
n2 n Number of students who play only Football
 250 = 7n + - = 20 - 10 = 10 .
8 8
Number of students who play at least one
 n = 25 game = 40 + 10 + 10 = 60
3. (3) Production in third year = 6000 9. (2) A clock strikes 4 taking 9 s,
Production in seventh year = 7000  Time taken to strike 12
 Production in fourthyear =1000
9  12
 Production increases every year = = 27s
= 250 scooters 4
 Production in tenth year 10. (3) Let the total number of packages be x.
= (7000 + 250 × 3) = 7750 2 x
2
5x+3= 2
99  99 
4. (4) = 0.9802,   = 0.9607
101  101  x 2x
 - =3
2 2 5
97  97 
= 0.9797,   = 0.9600 5 x  4x
99  99   =3
10
2
95  95 
= 0.9793,   = 0.9592 x
97  97   = 3  x = 30
10
Hence, onlv B and D are correct.
11. (1) Total number of men
5. (4) Let the whole number be x.
= 5180 + 4 = 5184
1  Number of men in first row
 x = 6 (x + 20)
= 5184 = 72
 6x = x + 20
LEARN MATHS FROM S.K. RAJU (9811549822, 9811649822)
26
12. (4) Let the tin oil contain x number of 16. (4)
bottles. Thumb Index Middle Ring Litde
3 1 2 3 4 6
4
x - 6 + 4 = 4x 9  8 7 6
5 10 11 12 13
17  16 15 14
4 x 3x 18 19 20 21
 - =2
5 4 25  24 23 22
16x  15 x 26 27 28 29
 =2 33  32 31 30
20
From the above counting pattern, we find
1 that every multiple of 8 comes on index
 x =2 finger and moves towards thumb therefore,
20
the last multiple of 8 which appears on
x = 40 bottles
13. (2) Let total number of votes cast be x. 1994
index finger will be  1992
2 8
Total number of counted votes = x Hence, 1994 will be on ring finger.
3
17. (3) According to question,
5 2 5
Votes that candidate got = × x = x R 6
6 3 9 =A
18
3 5 7 Given, Mahesh = 5 years
Votes still need to win = x- x= x
4 9 36  Anup = 3 years
 R = 18 × 3 + 6 = 60 years
1 18. (4) Let the total number of swans be x.
Remaining uncounted votes = x
3 The number of swans playing on shore
7 3 7 7
Required part = × = = x
36 1 12 2
14. (4) Required numbers are 12, 15, 18,.... 99 Number of remaining swans = 2
This is an AP with a =12 and d = 3
7
 Tn = a + (n - 1)d x = x +2
99 = 12 + (n - 1) × 3 2

99  12 7
n-1=  (x - 2) = x
3 2
n = 29 + 1 = 30 Now, among the options only
15. (2) Let the total score in the innings be x. x = 16 satisfies.
19. (4) Let the capacity of the fuel tank be x L.
2
Then, higher score = x x 3
9 Given, + 22 = x
5 4
2  2 
Next highest score = x  x  3 1
9  9    4  5  x = 22
 
2  2 
 2x - x  x  = 8 11
9 9  9   x = 22
20
2 89  x = 40L
x - x + x=
9 2 20. (2) Required numbers are 509, 519, 529,
539, 549, 559, 569, 579, 589, 590, 591, 592
899 ,.... 1598
x = = 162
2 2 So, there are 18 such numbers occurs only
once.

LEARN MATHS FROM S.K. RAJU (9811549822, 9811649822)


27
21. (1) Eliminated numbers are 5, 10, 15, 20,  4x - 2r = 8 ...(ii)
25, 30, 35, 40, 45, 50, 55, 60. Adding Eqs. (i) and (ii),
So, total eliminated numbers are 20. 2r = 24
 40 numbers would remain.  r =12
22. (4) Let the numbers be 3x and x.  from Eq. (ii),
 3x + x = 36 4x - 24 = 8
 4x = 36  4x = 32
36 x =8
x= = 9  Total number of students present in the
4
Hence, the smaller number = 9 auditorium = 8 × 12 = 96
23. (4) Let the two-digit number = xy
 2(10x + y) = 9(10y + x)
 88y - 11x = 0 ...(i)
Also, x + y = 9 ...(ii) 25. (1)
Solving Eqs. (i) and (ii), we get
x = 8 and y = 1
So, the number is 81.
24. (2) Let there are r rows and x students in Total students studying at least one subject
each row. =21 + 1 + 2 + 1 + 9 + 3 + 8 = 45 Students
 (x + 4) × (r - 4) = x × r who are not studying any of the three
 x × r - 4x + 4r - 16 = x × r subjects = 80 - 45 = 35
 - 4x + 4r = 16 ...(i)
and (x - 2) × (x + 4) = x + r
 x × r + 4x - 2r - 8 = x × r

LEARN MATHS FROM S.K. RAJU (9811549822, 9811649822)


1
PARTNERSHIP
NATIONALISED BANKS (Indiain Bank PO Exam. 17.10.2010)
& IBPS PO/MT/SO 6.
Gina invests Rs 48,000 to start a business.
1. A, B an d C starte d a busi ne ss wi th Four months later Shrayon joins her by
in ve stme nt in the ratio 5 : 6 : 8 investing Rs 62,000 and another two
respectively. After one year C withdrew 50% months later Deepika joins them both by
of his capital and A increased his capital investing Rs 80,000. At the end of one year
by 60% of his investment. After two years the business eams a profit of Rs 20,661.
in what ratio should the earned profit be What is Deepika’s share in the profit ?
distributed among A, B and C respectively ? (1) Rs 7,668 (2) Rs 6,603
(1) 2 : 3 : 3 (3) Rs 7,240 (4) Rs 6,390
(2) 4 : 3 : 2 (5) None of these
(3) 13 : 12 : 12 (United Bank Of India
(4) Cannot be determined PO Exam. 14.11.2010)
(5) None of these 7. Mrudul invested an amount of Rs 29,500
(Syndicate Bank PO Bxam. 10.10.2004} in order to start a business. Shalaka joined
2. Tanvi started a business investing Rs. her 4 months later by investing an amount
45,000. After 8 months Anisha joined her of Rs 33,500. If the business earned a profit
with a capital of Rs. 52,000. At the end of of Rs 1,20,575 at the end of two years, what
the year the total profit was Rs. 56,165. was Mrudul’s share of the profit ?
What is the share of profits of Anisha ? (1) Rs 60,725 (2) Rs 61,950
(1) Rs. 21,450 (2) Rs. 24,440 (3) Rs 59,250 (4) Rs 58,625
(3) Rs. 27,635 (4) Rs. 31,765 (5) None of these
(5) None of these (Indian Bank PO Exam. 02.01.2011)
(Orientai Bank of Commerce 8. Rs 73,689, are divided between A and B in
PO Exam. 21.12.2008) the ratio 4 : 7. What is the difference
3. Karan starts a business by investing Rs. between thrice the share of A and twice the
60,000. six months later Shirish joins him share of B ?
by investing Rs. 1,00,000. At the end of one (1) Rs 36,699 (2) Rs 46,893
year from commencement of the business, (3) Rs 20,097 (4) Rs 26,796
they eam a profit of Rs. 1,51,800. What is (5) Rs 13,398
the Shirish’s share in the profit ? (IBPS Bank PO/MT CWE 17.06.2012)
(1) Rs. 55200 (2) Rs. 82800 9. Three men A, B and C start a business
(3) Rs. 62500 (4) Rs. 96600 together. They invest Rs. 30000, Rs. 24000
(5) None of these and Rs. 42000 respectively in the begin-
(UCO Bank PO Exam. 22.03.2009) ning. After 4 months. B took out Rs. 6000
4. Prakash, Sunil and Anil started a business and C took out Rs. 10000. They get a profit
jointly investing Rs. 11 lakhs, Rs. 16.5 of Rs. 11960 at the end of the year. B’s share
lakhs and Rs. 8.25 lakhs respectively. The in the profit is
profit earned by them in the business at (1) Rs. 2700 (2) Rs. 2803
the end of three years was Rs. 19.5 lakhs. (3) Rs. 2900 (4) Rs. 2785
What will be the 50% of Anil’s share in the (5) None of these
profit ? (IBPS Bank PO/MT
(1) Rs. 4.5 lakhs (2) Rs.2.25 lakhs CWE-III 26.10.2013)
(3) Rs. 2.5 lakhs (4) Rs.3.75 lakhs 10. Geeta, Shweta and Sita started business
(5) None of these with investment of Rs 4200, Rs. 8400 and
(United Bank of India Rs. 5400 respectively. In the end of the year,
PO Exam. 21.06.2009) there was a profit of Rs 24000. Sita invested
5. Sonu invested 10% more than Mona. Mona 32% of her share in profit in the business.
invested 10% less than Raghu. If the total The remaining amount of profit with Sita is
sum of their investment is Rs. 5,780. How (1) Rs. 4986 (2) Rs. 4689
much amount did Raghu invest ? (3) Rs. 4886 (4) Rs. 4896
(1) Rs. 2010 (2) Rs. 2000 (5) None of these
(3) Rs. 2100 (4) Rs. 2210 (Corporation Bank Speciallist Officer
(5) None of these (Marketing) I Exam. 22.12.2014)
LEARN MATHS FROM S.K. RAJU (9811549822, 9811649822)
2
SBI PO EXAMS RBI GRADE-B
1. A man has divided his total money in his OFFICER EXAMS
will in such a way that half of it goes to his 1. Raksha invested an amount of Rs. 60,000
to start a software business. After six
2 months, Kamal joined her with an amount
wife, rd of the remaining among his three
3 of Rs. 90,000. After one year from the
sons equally and the rest among his four commencement of the business, Raksha
daughter equally. If each daughter gets Rs. put in an additional amount of Rs. 20,000.
20,000. how much money will each son At the end of three years, they earned a
get ? proflt of Rs. 71,20,000. What is Raksha’s
(1) Rs. 50,333.33 share in the proflt ?
(2) Rs. 48,233.33 (1) Rs. 35,20,000
(3) Rs. 53,333.33 (2) Rs. 26,40,000
(4) Data inadequate (3) Rs. 27,00,000
(5) None of these (4) Rs. 38,40,000
(SBI Banks PO Exam. 20.08.2000) (5) None of these
2. Sunetra invested an amount of Rs. 50,000 (RBI Grade-B Offìcer Exam. 2008)
to start a software business. After six
months Nikhil joined her with an amount ISURANCE EXAMS
of Rs 80,000. At the end of 3 years they 1. A and B started a partnership business
earned a profit of Rs. 24,500. What is investing some amount in the ratio of 3 : 5.
Sunetra’s share in in the profit ? C joined them after six months with an
(1) Rs, 14,000 (2) Rs. 9423 amount equal to that of B; proflt at the end
(3) Rs. 10,250 (4) Rs. 12,500 of one year should be distributed in what
(5) None of these proportion among A, B and C ?
(SBI Associate Banks PO Exam. (1) 6 : 10 : 5 (2) 3 : 5 : 5
21.07.2002) (3) 3 : 5 : 2 (4) Data inadequate
3. Mr. ‘X’ invested certain amounts in two (5) None of these.
different schemes ‘A’ & ‘B’ Scheme ‘A’ offers (LIC Assistant Administrative Offìcer
simple interest at 12 p.c.p.a. and Scheme (AAO) Exam. 24.04.2005)
‘B’ of offers compound interest at 10 p.c.p.a. 2. Sunetra invested an amount of Rs. 50,000
Interest accrued on the amount invested to start a software business. After six
in Scheme A in 2 years was Rs. 3,600 and months Nikhil joined her with an amount
the total amount invested was Rs 35,000. of Rs. 80,000. At the end of 3 years they
What was interest accrued on the amount eamed a proflt of Rs. 24,500. What is
invested in profìts of Anisha ? Sunetra’s share in the proflt ?
(1) Rs. 4,800 (1) Rs, 14,000 (2) Rs. 9423
(2) Rs. 4,200 (3) Rs. 10, 250 (4) Rs. 12,500
(3) Rs. 4,000 (5) None of these
(4) Cannot be determined (LIC Assistant Administrative
(5) None of these Officer (AAO) Exam. 2006)
(SBI PO Ezam. 09.01.2005) 3. A, B and C started a business with their
4. Ninad, Vikas and Manav enter into a investment in the ratio 1 : 3 : 5. After 4
partnership. Ninad invests some amount months, A invested the same amount as
at the beginning. Vikas invests doublé the before and B as well as C withdrew half of
amount after 6 months and Manav invests their investments. The ratio of their profits
thrice the amount invested by Ninad after at the end of the year was :
8 months. They eam a proflt of Rs. 45,000 (1) 5 : 6 : 10 (2) 6 : 5 : 10
at the end of the year. What is Manav’s (3) 10 : 5 : 6 (4) 4 : 3 : 5
share in the profit ? (United India Insurance Co.
(1) Rs. 25,000 (2) Rs. 15,000 (AAO) Exam. 11.03.2007)
(3) Rs. 12,000 (4) Rs. 9,000 4. Dheeraj distributed his total assets to his
(5) None of these wife, three sons, two daughters and five
(SBI PO Preliminary (Tire-I) grandchildren in such a way that each
Exam. 27.04.2008) grandchild got one-eighth of each son or

LEARN MATHS FROM S.K. RAJU (9811549822, 9811649822)


3
one-tenth of each daughter.His wife got 40 (General Insurance Corporation
per cent of the total share of his sons and AAO Exam. 11.12.2011)
daughters together. If each daughter 7. Tanvi started a business investing Rs.
received asset of worth Rs. 1.25 lakh, what 45,000. After 8 months Anisha joined her
was the total worth of the assets received with a capital of Rs. 52,000. At the end of
by his wife and the three grandchildren to- the year the total profit was Rs. 56,165.
gether ? What is the share of profits of Anisha ?
(1) Rs. 32,500 (1) Rs. 21,450 (2) Rs. 24,440
(2) Rs. 2,57,500 (3) Rs. 27,635 (4) Rs. 31,765
(3) Rs. 2,82,500 (5) None of these
(4) Cannot be determined (United India Insurance AO Exam.
(5) None of these 26.05.2013
(LIC Assistant Administrative 8. A, B, C started a shop by investing Rs.
Officer Exam. 2008) 27000, Rs. 81000 an d Rs. 72000
5. X, Y and Z were sharing profits in the ratio respectively. At the end of one year, B’s
4 : 3 : 2. Y retired from the firm and X and Z share in total profit was Rs. 36000. What
decide to share profits in the ratio 3 : 2. was the total profit ?
Calculate the gaining ratio. (1) Rs. 108000 (2) Rs. 116000
(1) 7 : 8 (2) 5 : 9 (3) Rs. 80000 (4) Rs. 92000
(3) 4 : 7 (4) 5 : 8 (NICL (GIC) Administrative
(LIC Assistant Administrative Officer Exam. 15.12.2013)
Officer (AAO) Exam. 07.06.2009)
6. A, B and C enter into a partnership. A
invests some money at the beginning, B
invests doublé the amount after 6 months
and C invests thrice the amount after 8
months. If the annual proflt be Rs. 27000
C’s share is:
(1) Rs. 9000 (2) Rs. 11250
(3) Rs. 10,800 (4) Rs. 8625

LEARN MATHS FROM S.K. RAJU (9811549822, 9811649822)


4
SHORT ANSWERS 4. (2) Ratio of the equivalent capitals of
NATIONALISED BANKS Prakash, Sunil and Anil
= 11 × 36 : 16.5 × 36 : 8.25 × 36
& IBPS PO/MT/SO
=4 : 6 : 3
Anil’s share in the proflt
1. (3) 2. (5)
3. (5) 4. (2)  3 
5. (2) 6. (4) = Rs.  4  6  3  19.5 lakh
8. (5) 9. (2)    
10. (4) = Rs. 4.5 lakh
50% of Rs. 4.5 lakh
SBI PO EXAMS = Rs. 2.25 lakh
1. (3) 2. (5) 5. (2) Let Mona’s investment = Rs.100
3. (2) 4. (2)  Sonu’s investment = Rs. 110 and Raghu’s
100 1000
investment =  100 = Rs.
RBI GRADE-B OFFICER EXAMS 90 9
1. (1)  Ratio of Mona’s, Sonu’s and Raghu’s
investments
INSURANCE EXAMS
1. (4) 2. (5) 1000
= 100 : 110 : = 90 : 99 : 100
3. (1) 4. (2) 9
5. (1) 6. (1) Sum of ratios = 90 + 99 + 100 = 289
7. (5) 8. (3)  Raghu’s investment
 100 
EXPLANATIONS = Rs.   5870  = Rs. 2000
 289 
NATIONALISED BANKS
6. (4) Ratio of the proflts of Gina, Shrayon and
& IBPS PO/MT/SO Deepika
1. (3) Total equivalent capital of A = 48000 × 12 : 62000 × 8 : 80000 × 6
= 5x × 12 + 8x × 12 = Rs. 156x = 48 × 12 : 62 × 8 : 80 × 6
Total equivalent capital of B for 1 month = 36 : 31 : 30
= 6x × 24 = Rs. 144x Sum of ratios
Total equivalent capital of C = 36 + 31 +30 = 97
= 8x × 12 + 4x × 12 = Rs. 144x
 Deepika’s share
 Required ratio = A : B : C
= 156x : 144x : 144x  30 
= Rs.   20661 = Rs. 6390
= 13 : 12 : 12  97 
2. (5) Ratio of the earned profit = Ratio of the 7. (2) Ratio of the equivalent capitals of Mrudul
equivalent capitate of Tanvi and Anisha and Shalaka
= 45000 × 12 : 52000 × 4 = 29500 × 24 : 33500 × 20 = 354 : 335
= 45 × 3 : 52 Sum of the ratios = 354 + 335 = 689
= 135 : 52  Mrudul’s share
Sum of ratios = 135 + 52 = 187
 Anisha’s share  354 
Rs   120575 
 689 
 52 
= Rs.   56165  = Rs. 156180.07 = Rs. 61950
 187 
8. (5) Required difference
3. (5) Ratio of the proflt = Ratio of the
equivalent capitals of Karan and Shirish 7 4 
=   2   3
= 60000 × 12 : 100000 × 6  11 11 
= 720000 : 600000 = 6 : 5
 Shirish’s share in the profit 2
= × 73689 = Rs. 13398
11
5 
= Rs.  151800  = Rs. 69000 9. (5) Ratio of equivalent capitals
 11  = 30000 × 12 : (24000 × 4 + 18000 × 8)

LEARN MATHS FROM S.K. RAJU (9811549822, 9811649822)


5
: (42000 × 4 + 32000 × 8) =3 :4
= 360000 : 240000 : 424000  Share of Sunetra out of
= 90 : 60 : 106
3
= 45 : 30 : 53 Rs. 24500 = Rs. × 24500
Sum of ratios = 45 + 30 + 53 = 128 7
 B’s share = Rs. 10,500
3. (2) Let the amount invested in scheme A
30 be Rs. x.
= × 11960 = Rs. 2803.125
128
x  12  2
10. (4) Ratio of share of proflt  = 3600
= Geeta : Shweta : Sita 100
= 4200 : 8400 : 5400 3600 100
= 7 : 14 : 9 x= = Rs. 15000
24
Sita’s share
Total investment = Rs. 35000
9  Amount invested in scheme B
=  7  14  9  × 24000 = Rs. (35000 - 15000) = Rs. 20000
 R 
T

 C.I = P  100   1

9  1
= × 24000 = Rs. 7200
30  
Remaining amount after investing 32%
= 7200 × (100 - 32)%  10 
2

= 20000   1    1
7200  68  100  
= = Rs.4896
100 = 20000 (1.21 - 1)
= 20000 × 0.21
SBI PO EXAMS = Rs. 4200
1. (3) Let the total money be Rs. x 4. (2) Let Ninad invest Rs.x
 Vikas’s investment = Rs.2x
x Manav’s investment = Rs. 3x
Half of the total money goes to wife =
2 Ratio of the equivalent capital of Ninad,
Vikas and Manav respectively
2 x x
of the remaining = is divided equally = x ×12 : 2x × 6 : 4 × 3x
3 2 3 =1 :1 :1
among three sons. Clearly, the proflt will be shared equally
 The share of each son  Manav’s share
1 x x 1
= × = = Rs. × 45000 = Rs. 15000
3 3 9 3
x x x
- = is divided equally among four RBI GRADE-B
2 3 6
daughters. OFFICER EXAMS
 The share of each daughter 1. (1) Equivalent capital of Raksha for 3 year
= Rs. (60000 × 1 + 80000 × 2)
x
= 20000 = Rs. (60000 + 160000) = Rs. 220000
24 Equivalent capital of Kamal for 3 year
x = 480000
The share of each son 1
= Rs. 90000 × 2
2
x 480000
= =  5
9 9
= Rs.  90000   = Rs 225000
= Rs.53333.33  2
2. (5) The ratio of capital invested by each Ratio of capitals = 220000 : 225000
partner i.e. Sunetra and Nikhil for in order = 44 : 45
to get the same proflt is : Sum of ratios = 44 + 45 = 89
36 × 50000 : 30 × 80000 Total profit = Rs. 7120000
LEARN MATHS FROM S.K. RAJU (9811549822, 9811649822)
6
 Raksha’s share 2
 Wife’s share = × 5.5
 44  5
= RS   7120000 
 89  = Rs. 2.2 lakhs
= Rs. 3520000 Share of three grandchildren
= 3 × 0.125 = Rs. 0.375 lakh
INSURANCE EXAMS  Required answer
= Rs. (2.2 + 0.375) lakh
1. (4) Let the amount invested by A and B is
= Rs. 2.575 lakhs
3x and 4x respectively after 6 month C
= Rs. 257500
joined amount equal to B.
5. (1) Gaining ratio
Then, Ratio of A, B and C in profit
= 3x × 12 : 5x × 12 : 5x × 6 = 6 : 10 : 5 3 4 2 2
2. (5) The ratio of capital invested by each =    :  
5 9 5 9
partner i.e. Sunetra and Nikhil for 1 month
in order to get the same profit is : 27  20 18  10
36 × 50000 : 30 × 80000 = 3 : 4 = : =7:8
45 45
 Share of Sunetra out of 6. (1) Ratio of profit
3 = x × 12 : 2x × 6 : 3x × 4
Rs. 24500 = Rs. × 24500 =1 :1 :1
7
= Rs. 10,500 1
3. ( 1) Let their initial investments be Rs. x,  C’s share = × 27000
3
Rs. 3x and Rs. 5x respectively. = Rs. 9000
Then, A : B : C 7. (5) Ratio of the earned profit = Ratio of the
3x equivalent capitals of Tanvi and Anisha
= (x × 4 + 2x × 8) : (3x × 4 + × 8) = 45000 × 12 : 52000 × 4
2
= 45 × 3 : 52 = 135 : 52
5x Sum of ratios = 135 + 52 = 187
: (5x × 4 + × 8)
2  Anisha’s share
= (4x + 16x) : (12x + 12x) : (20x + 20x)  52 
= 20x : 24x : 40x = 5 : 6 : 10 = Rs.   56165  = Rs. 15618.074
4. (2) Share of 1 grandchild  187 
8. (3) Ratio of their profit
1 = 27000 : 81000 : 72000 = 3 : 9 : 8
= × 1.25 = Rs. 0.125 lakh
10 If total profit be Rs. x,
 Each son gets = 8 × 0.125 = Rs. 1 lakh 9x
 Share of 3 sons = Rs. 3 lakhs then B’s share = = 36000
Share of 2 daughters = 2 × 1.25 = Rs. 2.5 20
lakhs 36000  20
Total share of sons and daughters = Rs. 5.5  x= = Rs. 80000
9
lakhs

LEARN MATHS FROM S.K. RAJU (9811549822, 9811649822)


7
MODEL EXERCISES 7. X and Y entered into partnership with Rs
1. Rs 770 nave been divided among A B and 700 and Rs 600 respectively. After 3
months, X withdrew 2/7 of his stock but
2 after 3 months, he puts back 3/5 of what
C such that A receives th of what B and
9 he had withdrawn. The profit at the end of
C together receive. Then, A’s share is the year is Rs 726. How much of this should
(1) Rs 140 (2) Rs 154 X receive ?
(3) Rs 165 (4) Rs 170 (1) Rs 336 (2) Rs 366
(5) None of these (3) Rs 633 (4) Rs 663
2. A sum of Rs 370 is to be divided among A, (5) None of these
B and C such that 8. A, B and C are three partners. They
A 's share B 's share
altogether invested Rs 14000 in business.
3
= = . At the end of the year, A got Rs 337.50, B
B 's share C 's share 4 Rs 1125 and C Rs 637.50 as proflt. The
Then, A’s share is difference between the investments of B
(1) RS 240 (2) Rs 120 and A was
(3) Rs 100 (4) Rs 90 (1) Rs 2200 (2) Rs 3200
(5) None of these (3) Rs 4200 (4) Rs 5250
3. A and B enter into a partnership with Rs (5) None of these
50000 and Rs 60000 respectively. C joins 10. Anu is a working partner and Bimla is a
them after x months contribuing Rs 70000 sleeping partner in a business. Anu puts
and B leaves x months before the end of in Rs 5000 and Bimla puts in Rs 6000. Anu
the year. If they share the profit in the ratio receives 12.5% of the proflt for managing
of 20 : 18 : 21, then the value of x is the business and the rest is divided in
(1) 3 (2) 9 proportion to their capital. What does each
(3) 6 (4) 8 get out of a proflt of Rs 880 ?
(5) None of these (1) Rs 400 and Rs 480
4. Ajay, Aman, Suman and Geeta rented a (2) Rs 460 and Rs 420
house and agreed to share the rent as (3) Rs 450 and Rs 430
follows (Ajay : Aman = 8 : 15, Aman : Suman (4) Rs 470 and Rs 410
= 5 : 8 and Suman : Geeta = 4 : 5. The part (5) None of these
of rent paid by Suman will be 10. X and Y are partners in a business. X
(1) 24/77 (2) 13/66 contributed 1/3 of the capital for 9 months
(3) 12/55 (4) 13/77 and Y received 2/5 of the profits. For how
(5) None of these long was Y’s money used in the business ?
5. Three partners invested capital in the rado (1) 4 months (2) 3 months
2 : 7 : 9. The time period for which each of (3) 2 months (4) 5 months
them invested was in the ratio of the (5) None of these
reciprocal of the amount invested. Find the 11. Two companies A and B quote for a tender.
share of the partner who brought in the On the tender opening day, A realizes that
highest capital, if profit is Rs 1080. the two quotes are in the ratio 7 : 4 and
(1) Rs 120 (2) Rs 360 he nce de crease s its price du ri ng
(3) Rs 540 (4) Rs 420 negotiations to make it Rs 1 lakh lower than
(5) None of these B’s quoted price. B then realizes that the
6. A man left one-half of the capital for his final quotes of the two were in the ratio 3 :
wi fe . On e-thi rd to hi s son and the 4. By how much did A decrease its price in
remainder to his daughter and her share order to win the bid ?
was worth Rs 45,000. Find the total capital. (1) Rs 7 lakhs (2) Rs 4 lakhs
(1) Rs 245000 (2) Rs 260000 (3) Rs 9 lakhs (4) None of these
(3) Rs 270000 (4) Rs 275000 (5) None of these
(5) None of these

LEARN MATHS FROM S.K. RAJU (9811549822, 9811649822)


8
SHORT ANSWERS  Share of each partner
1. (1) 2. (4) 1
3. (1) 4. (1) = × 1080 = Rs 360
3
5. (2) 6. (2) 6. (3) Let that man left Rs x.
7. (3) 8. (2)
9. (4) 10. (2) x x
 Share of daughter = x - -
11. (2) 12. (2) 2 3
x
EXPLANATIONS  45000 =
6
1. (1) According to question, x = Rs 270000
A + B + C = 770  ...(i) 7. (2) X’s investment = (700 × 3)
2
A= (B + C) ...(ii)  5   3
9 +  700   3  +  500  200   × 6
 7   5
From Eqs. (i) and (ii), we get
= Rs 7320
9 Y’s investment = 600 ×12 = Rs 7200
A  A = 770
2 X’s share from profit
 11A = 770 × 2 7320
 A = 140 =  7320  7200  × 726 = Rs 366
Thus, A’s share is Rs 140
2. (4) According to question, 8. (4) By question,
Ratio of investments of A, B and C
A 3  3 9 B 3  4 12 = 337.50 : 1125 : 637.50
= = : = :
B 4  3 12 C 4  4 16 = 27 : 90 : 51 : = 9 : 30 : 17
 A : B : C = 9 : 12 : 16 Let the investments of A, B and C be 9x,
9 30x and 17x respectively
A’s share = × 370 = Rs 90  9x + 30x + 17x = 14000
37
3. (1) According to question,  x = 250
Investment of A = 50000 ×12 = Rs 600000 So, required difference
Investment of B = 60000 × (12 - x) = 30x - 9x = 21x = Rs 5250
Investment of C = 70000 × (12 - x) 9. (2) By question, 2.5% of profit

600000 20 = 12.5 × 880 = Rs 110


 60000  12  x  = 18 100
Remaining = Rs 770 is divided in the ratio
 180 = 240 - 20x = 5000 : 6000 = 5 : 6 Proflt of Anu
x =3
4. (1) According to question, 5
= × 770 + 110 = Rs 460 Proflt of Bimla
Ajay : Aman = 8 : 15 11
Aman : Suman = 5 : 8 6
Suman : Geeta = 4 : 5 = × 770 = Rs 420
11
 Ajay : Aman : Suman : Geeta 10. (2) By question,
= 8 : 15 : 24 : 30
1 2
 Part of rent paid by Suman Ratio of capital of X and Y = : =1:2
3 3
24 24
= = 3 2
8  15  24  30 77 Ratio of their proflt = : =3:2
5. (2) According to question, 5 5
Ratio of capital = 2 : 7 : 9 Let Y’s money was used for n months.
 (1 × 9) : (2 × n) =3 : 2
1 1 1
Ratio of time = : :  n = 3 months
2 7 9 12. (2) A quotes Rs. 7x initially.
 Ratio of investment B quotes Rs. 4x initially.
1 1 1 The new ratio changes to 3 : 4.
=2× :7× :9× Change is decrease of Rs. 4 lakhs.
2 7 9
= 1 :1 :1

LEARN MATHS FROM S.K. RAJU (9811549822, 9811649822)


1
PERCENTAGE
NATIONALISED BANKS (Andhra Bank PO Exam. 14.09.2008)
& IBPS SO/MT/SO 6.
Vaish ali spen t Rs.31,897 on th e air
1. Vipul decided to donate 5% of his salary. conditioner for her home, Rs. 38,789 on
On the day of donation he changed his mind buying plasma television and the remaining
and donated Rs. 1687.50 which was 75% 23% of the total amount she had as cash
of what he had decided earlier. How much with her. What was the total amount ?
is Vipul’s salary ? (1) Rs. 74,625
(1) Rs. 37,500 (2) Rs. 86,750
(2) Rs. 45,000 (3) Rs. 91,800
(3) Rs. 33,750 (4) Cannot be determined
(4) Cannot be determined (5) None of these
(5) None of these (Bank Of Baroda Specialist
(Corporation Bank PO Officer Exam. 05.10.2008)
Exam. 29.07.2006) 7. In a class of 35 students and 6 teachers,
2. Two numbers are less than the third each student got sweets that are 20% of
number by 50% and 54% respectively. By the total number of students and each
how much per cent is the second number teacher got sweets that are 40% of the total
less than the first number ? number of students. How many sweets were
(1) 13 there ?
(2) 10 (1) 245 (2) 161
(3) 12 (3) 406 (4) 84
(4) Cannot be determined (5) None of these
(5) None of these (Oriental Bank of Commerce
(Bank Of Maharashtra PO PO Exam. 21.12.2008)
Exam. 25.05.2008) 8. Prithvi spent Rs. 89,745 on his college fees,
3. In an election between two candidates, one Rs.51,291 on Personality Development
got 52% of total valid votes. 25% of the totalClasses and the remaining 27% of the total
votes were invalid. The total number of amount he had as cash with him. What was
votes were 8400. How many valid votes did the total amount ?
the other person get ? (1) Rs. l,85,400 (2) Rs. 1,89,600
(1) 3276 (3) Rs. 1,91,800 (4) Rs. 1,93,200
(2) 3196 (5) None of these
(3) 3024 (Oriental Bank of Commerce
(4) Cannot be determined PO Exam. 21.12.2008)
(5) None of these 9. In a class of 65 students and 4 teachers,
(Bank Of Maharashtra each student got sweets that are 20% of
PO Exam. 25.05.2008) the total number of students and each
teacher got sweets that are 40% of the total
5
4. What is 27% of 36% of th of 4500 ? number of students. How many sweets were
9 there ?
(1) 239 (2) 241 (1) 845 (2) 897
(3) 243 (4) 245 (3) 949 (4) 104
(5) None of these (5) None of these
(Andhra Bank PO Exam. 14.09.2008) (Canara Bank PO Exam. 15.03.2009)
5. Vi sh akha spe nt Rs. 68,357 on the 10. Bovina spent Rs. 44,668 on her air tickets,
renovation for her home, Rs. 25,675 on Rs. 56,732 on buying gifts for the family
buying music system and the remaining members and the remaining 22% of the
28% of the total amount she had as cash total amount she had as cash with her.
with her. What was the total amount ? What was the total amount ?
(1) Rs. 94, 032 (1) Rs. 28,600
(2) Rs. 36,568 (2) Rs. 1,30,000
(3) Rs. 1,30,600 (3) Rs. 1,01,400
(4) Cannot be determined (4) Rs. 33,800
(5) None of these (5) None of these
LEARN MATHS FROM S.K. RAJU (9811549822, 9811649822)
2
(Canara Bank PO Exam. 15.03.2009) 16. Mr. X spends 20% of his monthly income
11. Rubina decided to donate 16% of her on household expenditure. Out of the
monthly salary to an NGO. On the day of remaining 25% he spends on children’s
donation she changed her mind and education, 15% on transport, 15% on
donated Rs. 6,567 which was 75% of what medicine and 10% on entertainment. He
she had decided earlier. How much is is left with Rs. 9,800 after incurring all these
Rubina’s monthly salary ? expenditures. What is his monthly income ?
(1) Rs. 8,756 (1) Rs. 35,000 (2) Rs. 28,000
(2) Rs. 54,725 (3) Rs. 65,333 (4) Rs. 48,400
(3) Rs. 6,56,700 (5) None of these
(4) Rs. 45,696 (PNB Specialist Officer’s
(5) None of these Exam. 16.08.2009)
(Canara Bank PO Exam. 15.03.2009) 17. A Man’s expense is 30% more than Vimal’s
12. A sum of Rs. 2,236 is divided among A, B expense and Vimal’s expense is 10% less
and C such that A receives 25% more than than Raman’s expense. If the sum of their
C and C receives 25% less than B. What is expenses is Rs. 6447, then what would be
A’s share in the amount ? the Aman’s expense ?
(1) Rs. 460 (2) Rs. 890 (1) Rs. 2,200 (2) Rs. 2,457
(3) Rs. 780 (4) Rs. 1280 (3) Rs. 1,890 (4) Rs. 2,100
(5) None of these (5) None of these
(Indian Overseas Bank (Corporation Bank PO
PO Exam. 05.04.2009) Exam. 22.11.2009)
13. Mr. Giridhar spends 50% of his monthly 18. A candidate appearing for an examination
income on household items and out of the has to secure 35% marks to pass. But he
remaining he spends 50% on transport, secured only 40 marks and failed by 30
25% on entertainment, 10% on sports and marks. What would be the maximum marks
remaining amount of Rs. 900 is saved. What of test ?
is Mr. Giridhar’s monthly income ? (1) 280 (2) 180
(1) Rs. 6,000 (3) 200 (4) 150
(2) Rs. 12,000 (5) 210
(3) Rs. 9,000 (Corporation Bank PO
(4) Cannot be determined Exam. 22.11.2009)
(5) None of these 19. The value of which of the following fractions
(United Bank of India PO is less than twenty per cent ?
Exam. 21.06.2009)
5 2
14. Income of A is 150% of the income of B (1) (2)
and income of C is 120% of the income of 6 3
A. If the total income of A, B and C together 2 1
is Rs. 86,000, what is C’s income ? (3) (4)
5 4
(1) Rs. 30,000
(2) Rs. 32,000 2
(3) Rs. 20,000 (5)
11
(4) Rs. 36,000 (Allahabad Bank PO Exam. 21.02.2010)
(5) None of these 20. Twenty per cent of Anuj’s annual salary is
(Andhra Bank PO Exam. 05.07.2009) equal to seventy five per cent of Raj’s annual
15. Population of a country increases every year salary. Raj’s monthly salary is 60% of Ravi’s
by 10% . If the population in January 2006 monthly salary. If Ravi’s annual salary is Rs.
was 15.8 lakhs, what was the population 1.44 lacs, what is Anuj’s monthly salary ?
in January 2008 ? (1) Rs. 2,70,000
(1) 19,11,800 (2) 18,96,000 (2) Rs. 27,000
(3) 19,11,600 (4) 18,94,000 (3) Rs. 3,24,000
(5) None of these (4) Rs. 5,400
(PNB Specialist Officer’s (5) None of these
Exam. 16.08.2009) (Bank Of Baroda PO Exam. 30.05.2010)

LEARN MATHS FROM S.K. RAJU (9811549822, 9811649822)


3
21. In a test, minimum passing percentage for percentage of the test is 35 per cent, what
girls and boys is 35% and 40% respectively. are the maximum marks of the test ?
A boy scored 483 marks and failed by 117 (1) 650 (2) 700
marks. What are the minimum passing (3) 750 (4) 600
marks for girls ? (5) None of these
(1) 425 (2) 520 (Union Bank Of India PO
(3) 500 (4) 625 Exam. 09.01.2001)
(5) None of these 27. In a school there are 2000 students out of
(Central Bank Of India whom 36 per cent are girls. Each boy’s
PO Exam. 25.07.2010) monthly fee is Rs 480 and each girl’s
22. Twelve percent of Kaushal’s monthly salary monthly fee is 25 per cent less than a boy.
is equal to sixteen percent of Nandini’s What is the total of the monthly fees of girls
monthly salary. Suresh’s monthly salary is and boys together ?
half that of Nandini’s monthly salary. If (1) Rs 8,73,400 (2) Rs 8,67,300
Suresh’s annual salary is Rs. 1.08 lacs, (3) Rs 8,76,300 (4) Rs 8,73,600
what is Kaushal’s monthly salary ? (5) None of these
(1) Rs. 20,000 (2) Rs. 18,000 (Union Bank Of India PO
(3) Rs. 26,000 (4) Rs. 24,000 Exam. 09.01.2001)
(5) None of these 28. A sum of Rs 731 is divided among A, B and
(Central Bank Of India C, such that ‘A’ receives 25% more than ‘B’
PO Exam. 25.07.2010) and ‘B’ receives 25% less than ‘C’. What is
23. In an exam Ritiz scored 52 per cent marks, C’s share in the amount ?
Sunil scored 64 percent marks and Ravi (1) Rs 172 (2) Rs 200
scored 74 percent of marks. The maximum (3) Rs 262 (4) Rs 258
marks of the exam are 750. What are the (5) None of these
average marks scored by all the three boys (Punjab & Sind Bank PO
together ? Exam. 23.01.2011)
(1) 475 (2) 485 29. In an examination Raman scored 25 marks
(3) 450 (4) 490 less than Rohit. Rohit scored 45 more
(5) None of these marks than Sonia. Rohan scored 75 marks
(Syndicate Bank PO Exam. 29.08.2010) which is 10 more than Sonia. Ravi’s score
24. Two candidates fought an election. One of is 50 less than maximum marks of the test.
them got 64% of the total votes polled and What approximate percentage of marks did
won with 992 votes. What was the total Ravi score in the examination if he gets 34
number of votes polled ? marks more than Raman ?
(1) 1500 (1) 90 (2) 70
(2) 1580 (3) 80 (4) 60
(3) 1550 (5) 85
(4) Cannot be determined (UCO Bank PO Exam. 30.01.2011)
(5) None of these 30. Raman scored 456 marks in an exam and
(PNB Management Trainee Sita got 54 percent marks in the same exam
Exam. 28.11.2010) which is 24 marks less than Raman. If the
25. In a test, minimum passing percentage for minimum passing marks in the exam is 34
girls and boys is 30% and 45% respectively. percent, then how much more marks did
A boy scored 280 marks and failed by 80 Raman score than the minimum passing
marks. How many more marks did a girl marks ?
require to pass in the test if she scored 108 (1) 184 (2) 196
marks ? (3) 190 (4) 180
(1) 132 (2) 140 (5) None of these
(3) 160 (4) 112 (Bank Of Baroda PO Exam. 13.03.2011)
(5) None of these 31. In a school there are 250 students out of
(Bank Of Maharashtra whom 12 percent are girls. Each girl’s
Exam. 19.12.2010) monthly fee is Rs 450 and each boy’s
26. Puneet scored 175 marks in a test and monthly fee is 24 percent more than a girl.
fail ed by 35 marks. I f the passi ng

LEARN MATHS FROM S.K. RAJU (9811549822, 9811649822)


4
What is the total monthly fee of girls and (2) 99.9
boys together ? (3) 66.6
(1) Rs 1,36,620 (2) Rs 1,36,260 (4) Cannot be determined
(3) Rs 1,32,660 (4) Rs 1,32,460 (5) None of these
(5) None of these (Indian Overseas Bank
(Indian Overseas Bank PO PO Exam. 22.05.2011)
Exam. 22.05.2011) 37. Dinesh’s monthly income is four times
32. An HR Company employs 4800 people, out Su re sh ’s mon th ly in co me . Su re sh ’s
of which 45 percent are males and 60 monthly income is twenty percent more
percent of the males are either 25 years or than Jyoti’s monthly income. Jyoti’s
older. How many males are employed in HR monthly income is Rs 22,000. What is
Company who are younger than 25 years ? Dinesh’s monthly income ?
(1) 2480 (2) 2320 (1) Rs 1,06,500 (2) Rs 1,05,600
(3) 1278 (4) 864 (3) Rs 1,04,500 (4) Rs 1,05,400
(5) None of these (5) None of these
(IBPS Bank PO/MT CWE (Indian Overseas Bank PO
Exam. 18.09.2011) Exam. 22.05.2011)
33. In a test, minimum passing percentage for 38. Ruby’s monthly income is three times
girls and boys is 30% and 45% respectively. Gayatri’s mon th ly in co me , G ay atri ’s
A boy scored 280 marks and failed by 80 monthly income is fifteen percent more
marks. How many more marks did a girl than Priya’s monthly income, Priya’s
require to pass in the test if she scored 108 monthly income is Rs 32,000. What is
marks ? Ruby’s Annual income ?
(1) 132 (2) 140 (1) Rs 1,20,300 (2) Rs 13,24,800
(3) 160 (4) 112 (3) Rs 38,800 (4) Rs 54,600
(5) None of these (5) None of these
(IBPS Bank PO/MT CWE (IBPS Bank PO/MT CWE
Exam. 18.09.2011) Exam. 18.09.2011)
34. Six-eleventh of a number is equal to twenty 39. Pradeep invested 20% more than Mohit.
two percent of second number. Second Mohit invested 10% less than Raghu. If the
number is equal to the one-fourth of third total sum of their investment is Rs 17,880,
number. The value of the third number is how much amount did Raghu invest ?
2400, What is the 45% of first number ? (1) Rs 6,000 (2) Rs 8,000
(1) 107.6 (2) 131.1 (3) Rs 7,000 (4) Rs 5,000
(3) 115.4 (4) 143.8 (5) None of these
(5) None of these (Corporation Bank PO
(IBPS Bank PO/MT CWE Exam. 16.01.2011)
Exam. 18.09.2011) 40. Sum of three consecutive numbers is 2262.
35. In an Entrance Examination Seema scored What is 41% of the highest number ?
56 percent marks, Nitya scored 92 percent (1) 301.51 (2) 303.14
marks and Meena scored 634 marks. The (3) 308.73 (4) 306.35
maximum marks of the examination are (5) 309.55
875. What are the average marks scored (IBPS Bank PO/MT CWE 17.06.2012)
by all the three girls together ? 41. Akash scored 73 marks in subject A. He
(1) 1939 (2) 817 scored 56% marks in subject B and x marks
(3) 680 (4)643 in subject C. Maximum marks in each
(5) None of these suject were 150. The overall percentage
(IBPS Bank PO/MT CWE marks obtained by Akash in all the three
Exam. 18.09.2011) subjects together were 54% . How many
36. Five-ninth of a number is equal to twenty marks did he score in subject C ?
five percent of second number. Second (1) 84 (2) 86
number is equal to one-fourth of third (3) 79 (4) 73
number. The value of third number is 2960. (5) None of these
What is 30 percent of first number ? (IBPS Bank PO/MT CWE 17.06.2012)
(1) 88.8

LEARN MATHS FROM S.K. RAJU (9811549822, 9811649822)


5
42. What value will you obtain if twenty five candidates got selected than state ‘A’. What
per cent of 2340 is subtracted from four- was the number of candidates appeared
ninth of square of 36 ? from each state ?
(1) 9 (2) - 18 (1) 8000 (2) 8400
(3) 18 (4) - 9 (3) 7600 (4) Data inadequate
(5) 3 (5) None of these
(IDBI Bank Officer Exam. 16.09.2012) (SBI Associate Banks PO
43. The product of two consecutive even Exam. 16.07.2000)
numbers is 7568. What is 150% of the sum 2. Sumitra has an average of 56% on her first
of the two numbers ? 7 examinations. How much she should
(1) 204 (2) 261 make on her eighth examination to obtain
(3) 304 (4) 198 an average of 60% on 8 examinations ?
(5) None of these (1) 88%
(IBPS RRBs Office Assistant (2) 78%
CWE Exam. 09.09.2012) (3) 98%
44. The difference between 20% of a number (4) Cannot be determined
(5) None of these
4
and th of the same number is 2499. What (SBI Banks PO Exam. 20.08.2000)
5 3. In a recent survey 40% houses contained
2 two or more people. Of those houses
is th of that number ? containing only one person 25% were
7
having only a male. What is the percentage
(1) 2156 (2)1190
of all houses, which contain exactly one
(3) 1090 (4) 1465
female and no males ?
(5) None of these
(1) 75
(IBPS RRBs Office Assistant
(2) 40
CWE Exam. 09.09.2012)
(3) 15
45. In order to pass in an examination, a
(4) Cannot be determined
student is required to get 342 marks out of
(5) None of these
the aggregate marks. Neha got 266 marks
(SBI Banks PO Exam. 20.08.2000)
and was declared fail by 8 per cent. What
4. The strength of a school increases and
is the minimum passing percentage of the
decreases every alternate year. It starts with
examination ?
increase by 10% and thereafter the percent-
(1) 28 (2) 36
age of increase/decrease is the same.
(3) 33 (4) 26
Which of the following is definitely true
(5) None of these
about the strength of the school in 2000 as
(IBPS RRBs Office Assistant
compared to that in 1996 ?
CWE Exam. 09.09.2012)
(1) Increase approximately by 2%
46. In an examination, 30% of total students
(2) Decrease approximately by 2%
failed in Hindi, 45% failed in English and
(3) Increase approximately by 20%
20% failed in both subjects. Find the per-
(4) Decrease approximately by 20% 0
centage of those who passed in both the
(5) None of these
subjects.
(SBI Banks PO Exam. 11.02.2001)
(1) 35.7% (2) 35%
5. 405 sweets were distributed equally among
(3) 40% (4) 45%
children in such a way that the number of
(5) 44%
sweets received by each child is 20% of the
(IBPS Bank PO/MT
total no. of children. How many sweets did
CWE-III 26.10.2013)
each child receive ?
(1) 15 (2) 45
SBI PO EXAMS (3) 9 (4) 18
1. In a competitive examination in State ‘A’, (5) None of these
6% candidates got selected from the total (SBI Associate Banks PO
appeared candidates. State ‘B’ had an equal Exam. 21.07.2002)
number of candidates appeared and 7% 6. The salary of an employee increases
candidates got selected with 80 more consistently by 50% every year. If his salary

LEARN MATHS FROM S.K. RAJU (9811549822, 9811649822)


6
today is Rs. 10,000, what will be the salary (3) Rs. 347424 (4) Rs. 173712
after another 4 years ? (5) None of these
(1) Rs. 62,500 (SBI PO Preliminary (Tire-I)
(2) Rs. 26,500 Exam. 27.07.2008)
(3) Rs. 50,625 12. In a class of 240 students, each student
(4) Rs. 33,750 got sweets that are 15% of the total number
(5) None of these of students. How many sweets were there ?
(SBI PO Exam. 09.01.2005) (1) 3000
7. Mr. Yadav spends 80% of his monthly (2) 3125
salary on consumable items and 50% of the (3) 8640
remaining on clothes and transport. He (4) Cannot be determined
saves the remaining amount. If his savings (5) None of these
at the end of the year are Rs. 5370, how (SBI PO Preliminary (Tire-I)
much amount per month he would have Exam. 27.07.2008)
spent on clothes and transport ? 13. Sonika spent Rs. 45,760 on the interior
(1) Rs. 4,037 decoration for her home, Rs. 27896 on
(2) Rs. 8,076 buying air conditioner and the remaining
(3) Rs. 9,691.20 28% of the total amount she had as cash
(4) Rs. 4,845.60 with her. What was the total amount ?
(5) None of these (1) Rs. 98540
(SBI PO Exam. 26.11.2006) (2) Rs. 102300
8. What is 170% of 1140 ? (3) Rs. 134560
(1) 1938 (2) 1824 (4) Cannot be determined
(3) 1995 (4) 1881 (5) None of these
(5) None of these (SBI PO Preliminary (Tire-I)
(SBI PO Preliminary (Tire-I) Exam. 27.07.2008)
Exam. 27.04.2008) 14. Product of one-third of a number and 150%
9. Mr. Sarang invests 6% of his monthly salary of another number is what percent of the
i.e. Rs.2,100 on insurance policies. Also he product of original number ?
invests 8% of his monthly salary on family (1) 80 (2) 50
mediclaim policies and another 9% of his (3) 75 (4) 120
salary on NSCs. What is the total annual (5) None of these
amount invested by Mr. Sarang ? (SBI Associate Banks PO
(1) Rs. 11,400 (2) Rs. 96,600 Exam. 07.08.2011)
(3) Rs. 8,050 (4) Rs. 9,500 15. Mr. Shamin’s salary increases every year
(5) None of these by 10% in June. If there is no other increase
(SBI PO Preliminary (Tire-I) or reduction in the salary and his salary in
Exam. 27.04.2008) June 2011 was Rs 22,385, what was his
salary in June 2009 ?
3
10. What is 25% of 75% of th of 4240 ? (1) Rs 18,650 (2) Rs 18,000
5 (3) Rs 19,250 (4) Rs 18,500
(1) 595 (2) 424 (5) None of these
(3) 348 (4) 477 (SBI Associate Banks PO
(5) None of these Exam. 07.08.2011)
(SBI PO Preliminary (Tire-I)
Exam. 27.07.2008)
RBI GRADE-B OFFICER EXAMS
11. Ms. Pooja Pushpan invests 13% of her
1. If the numerator of a fraction is increased
monthly salary, i.e. Rs. 8554 in Mediclaim
by 350% and the denominator of the
Policies. Later she invests 23% of her
fraction is increased by 300% , the resultant
monthly salary on Child Education Policies;
also she invests another 8% of her monthly 9
salary on Mutual Funds. What is the total fraction is . What is the original fraction ?
22
annual amount invested by Ms. Pooja
Pushpan ? 3 5
(1) (2)
(1) Rs. 28952 (2) Rs. 43428 4 12

LEARN MATHS FROM S.K. RAJU (9811549822, 9811649822)


7
In school ‘B’ the number of students
7
(3) (4) 4 appeared is 20% more than the students
9 11 appeared from school ‘A’ and the number
(5) None of these of students qualified from school ‘B’ is 50%
(RBI Grade-B Officer Exam. 2007) more than the students qualified from
2. Ms. Sujata invests 7% i.e. Rs. 2170 of her school ‘A’. What is the percentage of
monthly salary in mutual funds. Later she stu dents qu alified to th e nu mber of
invests 18% of her monthly salary in re- students appeared from school ‘B’ ?
curring deposits also, she invests 6% of her (1) 30% (2) 70%
salary on NSCs. What is the total annual (3) 87.5% (4) 78.5%
amount invested by Ms. Sujata ? (5) None of these
(1) Rs. 1,25,320 (LIC Assistant Administrative
(2) Rs. 1,13,520 Officer (AAO) Exam. 24.04.2005)
(3) Rs. 1,35,120 3. 405 sweets were distributed equally among
(4) Rs. 1,15,320 children in such a way that the number of
(5) None of these sweets received by each child is 20% of the
(RBI Grade-B Officer Exam. 2007) total no. of children. How many sweets did
3. Su jata scored 2240 marks i n an each child receive ?
examination that is 128 marks more than (1) 15 (2) 45
the minimum passing percentage of 64% . (3) 9 (4) 18
What is the percentage of marks obtained (5) None of these
by Meena if she scores 907 marks less than (LIC Assistant Administrative
Sujata ? Officer (AAO) Exam. 2006)
(1) 35 (2) 40 4. A money lender finds that due to fall in the
(3) 45 (4) 36
(5) 48 3
rate of interest from 8% to 7 % , his yearly
(RBI Officer Grade ‘B’ Online 4
Exam. 25.08.2013 income diminishes by Rs. 61.50. His capital
4. If tax on a commodity is reduced by 10% , (in Rupees) is :
total revenue remains unchanged. What is (1) 26000 (2) 24600
the percentage increase in its consumption ? (3) 23800 (4) 22400
(LIC Assistant Administrative
1
(1) 11 % (2) 20% Officer (AAO) Exam. 07.06.2009)
9 5. p is six times as large as q. The per cent
1 that q is less than p is
(3) 10% (4) 9 %
11 1 2
(1) 83 (2) 16
(5) None of these 3 3
(RBI Officer Grade B’ Online (3) 90 (4) 60
Exam. 25.08.2013 (LIC Assistant Administrative
Officer (AAO) Exam. 07.06.2009)
INSURANCE EXAMS 6. If the numerator of a fraction is increased
1. The number of Gypsy-cars sold in 1998 was by 400% and the denominator is increased
16,500 and that sold in 1997 was 16,580.
20
How much was the percentage decrease in by 500% , the resultant fraction is .
the sales of the Gypsy-cars form 1997 to 27
1998 ? What is the original fraction ?
(1) More than 1 per cent 9
(2) less than 1 per cent (1)
8
(3) Zero per cent
(4) Cannot be determined 11
(United India Insurance Co. (2)
12
AAO Exam. 21.04.2002) (3) 4
2. In an examination the percentage of (4) Cannot be determined
stu dents qu alified to th e nu mber of (5) None of these
students appeared from school ‘A’ is 70% .
LEARN MATHS FROM S.K. RAJU (9811549822, 9811649822)
8
(New India Assurance AO 12. In a market research project, 20% opted
Exam. 25.10.2009) for Nirma detergent whereas 60% opted for
7. A man spends 80% of his income. With an Surf Blue detergent. The rest were unsure.
in crease i n th e cost of l ivin g, h is If the difference between those who opted
for Surf Blue and those who were uncertain
1
expenditure increases by 37 % and his is 720, How many respondents were
2 covered in the survey ?
2 (1) 1800 (2) 1440
income increases by 16 % . His present (3) 3600 (4) Data Inadequate
3
(General Insurance Corporation
percent savings are
AAO Exam. 11.12.2011)
3 5 13. Fresh cherries contain 99% water. Suppose
(1) 5 % (2) 5 % you have 1 kg of fresh cherries. After a few
7 7
hours in the sun, some water evaporates
2 1 and the percentage of water in the cherries
(3) 6 % (4) 6 %
3 3 becomes 98% . The new weight (in g) of
(New India Assurance AO cherries is
Exam. 25.10.2009) (1) 750 (2) 700
8. Sunil scored 54 percent marks in a test. (3) 600 (4) 500
Ravi scored 450 marks in same test which (Oriental Insurance Company
is 300 less than Sonu. Sunil’s score is 60 AAO Exam. 08.04.2012
more marks than Sonu. If Ram scored 900 14. In a mathematics examination, the average
marks in the test. What is Ram’s percentage ? score of students who passed is x and those
(1) 80 (2) 65 who failed is y. If the average score of all
(3) 75 (4) 60 th e studen ts w ho appeare d in the
(5) None of these examination is z, then the percentage of
(United India Insurance AO students who failed was
Exam. 27.03.2011)
100xy 100  y  z 
9. In a school there are 800 students out of (1) (2)
whom 45 percent are girls. Monthly fee of z2 x y
each boy is Rs 600 and monthly fee of each
100  x  y  100  x  z 
girl is 30 percent less than each boy. What (3) (4)
is the total monthly fee of girls and boys x z x y
together ? (Oriental Insurance Company
(1) Rs 4,25,400 (2) Rs 4,14,600 AAO Exam. 08.04.2012)
(3) Rs 4,19,600 (4) Rs 4,23,400 15. Fresh grapes contain 80% water by weight,
(5) None of these whereas dried grapes contain 15% water
(United India Insurance AO by weight. How many kg of dried grapes can
Exam. 27.03.2011) be obtained from 3.4 kg of fresh grapes ?
10. Suppose x and y are inversely proportional (1) 0.51 (2) 0.6
and positive. If x increases by 10% , then y (3) 0.68 (4) 0.8
decreases by (United India Insurance AAO
10 Exam. 03.06.2012)
(1) 10% (2) % 16. p and q are inversely proportional to each
11
other and are positive. If p increases by
1 1 100% , then q increases by
(3) 9 % (4) % (1) 50% (2) 100%
11 11
(New India Insurance AAO (3) 150% (4) 200%
Exam. 22.05.2011) (United India Insurance AAO
11. (x % of y + y % of x) is : Exam. 03.06.2012)
(1) x % of y (2) y % of x 17. An interview panel found that a candidate
(3) 2% of xy (4) xy % of 3 has given a wrong date about his height,
(General Insurance Corporation while filling up his form he filled up 20%
AAO Exam. 11.12.2011) more than his actual height. His actual
he ig ht i s 5 fee t 2 in che s. By wh at

LEARN MATHS FROM S.K. RAJU (9811549822, 9811649822)


9
approximate percent should he reduce his (United India Insurance AO
height to get actual height ? Exam. 26.05.2013
(1) 15 (2) 14 21. If 90% of A = 30% of B and B = x % of A,
(3) 18 (4) 17 then the value of x is
(5) None of these (1) 800 (2) 300
(LIC Assistant Administrative (3) 700 (4) 400
Officer (AAO) Exam. 12.05.2013) (NICL (GIC) AO (Finance)
18. One-third of a number is 96. Whai will 67% Exam. 08.09.2013 (Paper-I)
of that number be ? 22. If 30% of (x - y) = 20% of (x + y), then what
(1) 192.96 (2) 181.44 percent of x is y ?
(3) 169.92 (4) 204.48 (1) 25 (2) 20
(5) None of these (3) 30 (4) 24
(United India Insurance AO (NICL (GIC) AO (Finance)
Exam. 26.05.2013 Exam. 08.09.2013 (Paper-I)
19. In a class of 35 students and 6 teachers, 23. A papaya tree was planted 2 years ago. It
each student got sweets that are 20% of increases at the rate of 20% every year. If
the total number of students and each at present, the height of the tree is 540 cm,
teacher got sweets that are 40% of the total what was it when the tree was planted ?
number of students. How many sweets were (1) 324 cm (2) 400 cm
there ? (3) 375 cm (4) 432 cm
(1) 245 (2) 161 (NICL (GIC) Administrative
(3) 406 (4) 84 Officer Exam. 15.12.2013)
(5) None of these 24. Fresh grapes contain 80% water while dry
(United India Insurance AO grapes contain 10% water. If the weight of
Exam. 26.05.2013 dry grapes is 250 kg, what was its total
20. Prithvi spent Rs. 89,745 on his college fees, weight when it was fresh ?
Rs. 51,291 on Personality Development (1) 1000 kg (2) 1100 kg
Classes and the remaining 27% of the total (3) 1125 kg (4) 1225 kg
amount he had as cash with him. What was (NICL (GIC) AO (Finance)
the total amount ? Exam. 15.12.2013)
(1) Rs. 1,85,400
(2) Rs. 1,89,600
(3) Rs. 1,91,800
(4) Rs. 1,93,200

LEARN MATHS FROM S.K. RAJU (9811549822, 9811649822)


10
SHORT ANSWERS EXPLANATIONS
NATIONALISED BANKS NATIONALISED BANKS
& IEPS PO/MT/SO & IBPS PO/MT/SO
1. (2) 2. (5) 1. (2) Let Vipul’s salary = Rs. x
3. (3) 4. (3)
5x x
5. (3) 6. (3) 5% of x = Rs. = RS.
7. (5) 8. (4) 100 20
9. (3) 10. (2) As given,
11. (2) 12. (3) 5 x 3x
13. (2) 14. (4) 1687.50 = × =
100 20 80
15. (1) 16. (1)
17. (2) 18. (3)  3x = 1687.50 × 80
19. (5) 20. (2) 1687.50  80
21. (5) 22. (4) x= = Rs. 45000
3
23. (1) 24. (5)
2. (5) Let the third number be 100.
25. (1) 26. (4)
27. (4) 28. (5)  First number = 50 and
Second number = 100 - 54 = 46
29. (2) 30. (1)
Decrease = 50 - 46 = 4
31. (2) 32. (4)
33. (1) 34. (5) 4
35. (4) 36. (2)  Required percentage = × 100 = 8%
50
37. (2) 38. (2) 3. (3) Number of valid votes
39. (1) 40. (5)
41. (2) 42. (4) 75
= 8400 × = 6300
43. (2) 44. (2) 100
45. (2) 46. (3) Number of valid votes got by other person
(defeated) = 48% of 6300
SBI PO EXAMS 6300  48
1. (1) 2. (4) = = 3024
100
3. (5) 4. (2)
5. (3) 6. (3) 4. (3) Required answer
7. (5) 8. (1) 5 36 27
9. (2) 10. (4) = 4500 × × × = 243
9 100 100
11. (3) 12. (3)
5. (3) Let the total amount be Rs. x.
13. (2) 14. (2)
According to the question,
15. (4)
Percentage of spent amount
= 100 - 28 = 72%
RBI GRADE-B OFFICER EXAMS Amount spent = Rs. (68357 + 25675)
1. (4) 2. (4) = Rs. 94032
3. (2) 4. (1)
72
 x × 100 = 94032
INSURANCE EXAMS
1. (2) 2. (3) 94032  100
3. (3) 4. (2) x= = Rs. 130600
72
5. (1) 6. (5)
7. (2) 8. (4) 6. (3) Let the total amount be Rs. x.
9. (5) 10. (3) Percentage of amount spent
11. (3) 12. (1) = 100 - 23 = 77%
13. (4) 14. (4)  77% of x = Rs. (31897 + 38789) = 70686
15. (4) 16. (1) 77
17. (4) 18. (1) x× = 70686
100
19. (5) 20. (4)
21. (2) 22. (2) 70686  100
23. (3) 24. (4) x = = Rs. 91800
77

LEARN MATHS FROM S.K. RAJU (9811549822, 9811649822)


11
7. (5) Number of sweets got by each According to the question,
35  20 15 x 3x
student = = 7 + + x = 2236
100 16 4
Number of sweets got by each
15 x  12x  16x
35  40  = 2236
16
teacher = = 14
100
43x
 Total number of sweets  = 2236
= 35 × 7 + 6 × 14 16
= 245 + 84 = 329 2236 16
8. (4) Let the total amount with Prithvi be Rs.  x= = 832
43
x.
Percentage of amount spent  A’s amount
= (100 - 27) = 73% 15
According to the question, = × 832 = Rs. 780
16
73% of x = (89745 + 51291) 13. (2) Let Giridhar’s monthly income = Rs. x.
x  73 x
 =141036
 Expenditure on household items = Rs.
100 2
141036  100 x
x= = Rs. 193200 Expenditure on transport = Rs.
73 4
9. (3) Number of total sweets
x
20 40 Expenditure on entertainment = Rs.
= 65 × 65 × + 4 × 65 × 8
100 100 Expenditure on sports
= 845 + 104 = 949
10. (2) Let the total amount with Bovina be Rs. x x  x
= Rs.    = Rs.
x.  2 10  20
According to the question,
78% of x = Rs. (44668 + 56732) x x x x
Savings = x - - - -
2 4 8 20
x  78
 = 101400
 40x  20x  10x  5x  2x 
100
= Rs  
 40 
 101400  100 
 x = Rs.   = Rs. 130000
 78  3x 3x
x =  40 = 900
11. (2) Let Rubina’s monthly salary be Rs. x. 40
According to the question,
900  40
75% of 16% of x = 6567 x= = Rs. 12000
3
16 75 14. (4) Let B’s income = Rs. x
x × × = 6567
100 100
150 3x
6567  100  100  A’s income = × x = Rs.
100 2
x= = Rs. 54725
16  75
120 3x 9x
12. (3) Let B receives Rs. x. C’s income = × = Rs.
100 2 5
 C’s amount = 75% of x
3x 3x 9x
= Rs.  x + 2 + 5 = 86000
4
3x 10 x  15 x  18 x
and, A’s amount = 125% of  = 86000
4 10
 43x = 860000
3x 125 15 x
= × = Rs. 860000
4 100 16
x= = 20000
43
LEARN MATHS FROM S.K. RAJU (9811549822, 9811649822)
12
 C’s income 70  100
9  x= = 200
35
= Rs.   20000  = Rs. 36000
5 
20 1
15. (1) Required population 19. (5) 20% = = = 0.2
100 5
T
 R 
= P 1   5 2 2
 100 
6
= 0.8; = 0.67; = 0.4
3 5
2
 10  1 2
= 15.8 1   lakh = 0.25; = 0.18
 100  4 11
20. (2) Tricky approach
11 11
= 15.8 × × lakh = 1911800 Monthly salary of Raj
10 10
1.44  60
16. (1) Let the monthly income of X be Rs. x. = = Rs. 0.072 lakh
Expenditure on household articles 12  100

x 1
= Rs.  Anuj’s monthly salarly ×
5 5

4x 3
Remaining amount = Rs. = Raj’s monthly salary ×
5 4
To tal percen tage ex penditu re in the  Anuj’s monthly salary
remaining amount  3 
= (25 + 15 + 15 + 10)% = 65% = Rs.  0.072   5  lakh = Rs. 27000
 4 
 Remaining amount
21. (5) If the maximum marks be x,
4x then
= 35% of Rs.
5 40% of x = 483 + 117

 35 4x  7x x  40
= Rs.    = Rs.  = 600
 100 5  25 100
600  100
7x x = = 1500
 25 = 9800 40
 Minimum passing marks for girls
 9800  25  1500  35
 x = Rs.   = Rs. 35000 = = 525
 7  100
17. (2) Let Vimal’s expense be Rs. 100. 22. (4) Suresh’s monthly salary
 Aman’s expense = Rs. 130 108000
and Raman’s expense = = Rs. 9000
12
100 1000 Nandini’s monthly salary = Rs. 18000
= × 100 = Rs.
90 9
12
 Ratio of the expenses of Vimal, Aman and  Kaushal’s monthly salary ×
100
Raman respectively
18000  16
1000 = = 2880
= 100 : 130 : 100
9
= 90 : 117 : 100  Kaushal’s monthly salary
 Aman expense  2880  100 
=   = Rs. 24000
117  12 
= × 6447 = Rs. 2457
90  117  100 23. (1) Average percentage of marks obtained
18. (3) If the maximum marks be x, 52  64  74 190
= = %
35 3 3
then x × = 40 + 30
100
LEARN MATHS FROM S.K. RAJU (9811549822, 9811649822)
13
 Average marks obtained 16
190  C’s share = × 731 = Rs. 272
43
= % of 750
3 29. (2 Rohan = 75
190 750 Sonia = 65
= × = 475 Rohit = 65 + 45 = 110
3 100
Raman = 110 - 25 = 85
24. (5) Number of votes cast = x (let) Ravi = 85 + 34 = 119
 (64 - 36)% of x = 992 Maximum Marks
x  28 = 119 + 50 = 169
 = 992
100 119
Required Percent = × 100 = 70
992  100 169
 = 3543 30. (1) If total maximum marks be x, then
28
25. (1) If the maximum marks of examination 54
be x, then x× = 456 - 24 = 432
100
x  45 432  100
= 280 + 80 = 360 x = = 800
100 54
360  100  Minimum passing marks
x= = 800
45 800  34
 30% of 800 = =272
100
800  30  Required answer = 456 - 272 = 184
= = 240 31. (2) Monthly fee of each boy
100
= Minimum marks to pass for girls 450  124
 Required difference = 240 - 108 = 132 = = Rs. 558
100
26. (4) If the maximum marks in the test be x,
then, 250  12
Number of girls = = 30
100
x  35
=175 + 35 = 210 Number of boys = Rs. 220
100
 Total monthly fee
210  100 = Rs. (220 × 558 + 30 × 450)
x= = 600 = Rs. (122760 + 13500) = Rs. 136260
35
32. (4) Number of males in company
2000  36
27. (4) Girls  = 720 4800  45
100 = = 2160
100
Boy  2000 - 720 = 1280
 Number of males younger than 25 years
75
Each girl’s fee = 480 × 2160  40
100 = = 864
100
= Rs. 360
33. (1) If the maximum marks of examination
 Total monthly fee
be x, then
= Rs. (1280 × 480 + 720 × 360)
= Rs. (614400 + 259200) x  45
= 280 + 80 = 360
= Rs. 873600 100
28. (5) Let amount received by B = Rs. 100
360  100
 Amount received by A = Rs. 125 x= = 800
 Amount received by C 45
 30% of 800
100  100 400
=
100  25
= Rs.
3
800  30
= = 240
100
400 = Minimum marks to pass for girls
 A : B : C = 125 : 100 : = 15 : 12 : 16
3  Required difference = 240 - 108 = 132
Sum of the ratios = 15 + 12 + 16 = 43

LEARN MATHS FROM S.K. RAJU (9811549822, 9811649822)


14
1  Raghu’s investment
34. (5) Second number = 2400 × = 600 50
4
= × 17880 = Rs 6000
If the first number be x, then 149
6 22 2262
x× = 600 × = 132 40. (5) Second number = = 754
11 100 3
132  11  Largest number = 755
x = = 242 755  41
6
 41% of 755 = = 309.55
100
45
 45% of 242 = 242 × = 108.9 41. (2) Marks obtained by Akash in subject B
100
35. (4) Total marks obtained by Seema 150  56
= = 84
100
875  56
= = 490 Total marks obtained by Akash
100
Total marks obtained by Nitya 450  54
= = 243
100
875  92
= = 805  Marks obtained in subject C
100 = 243 - 73 - 84 = 86
Required average marks 42. (4) Required answer
490  805  634 1929 4 25
= = = 643 = × (36)2 - 2340 ×
3 3 9 100
36. (2) Second number = 576 - 585 = - 9
1 43. (2) 7568 = 86 × 88.
= × 2960 = 740
 Sum = 86 + 88 = 174
4
5 740  25 174  150
 First number × =  150% of 174 = = 261
9 100 100
44. (2) Let the number be x.
740 9
 First number = × = 333 4x x
4 5  - = 2499
5 5
333  30
 30% of 333 = = 99.9 3x
100  = 2499
37. (2) Suresh’s monthly income 5

22000  120 2499  5


= = Rs. 26400 x= = 4165
100 3
 Dinesh’s monthly income 2
= Rs. (4 × 26400) = Rs. 105600  4165 × = 1190
7
38. (2) Gayatri’s monthly income 45. (2) Total marks of the examination = x
32000  115 x 8
= = Rs. 36800
100  = 342 - 266 = 76
100
 Ruby’s annual income
= Rs. (12 × 3 × 36800) = Rs. 1324800 76  100
x= = 950
39. (1) Let Raghu’s investment 8
= Rs. 100 342
 Mohit’s investment = Rs 90  Required percentage = × 100 = 36
950
90  120 46.
(3) n(A) = 35, n(B) = 45,
Pradeep’s investment = = Rs. 108
100 n (A  B) = 20
 Ratio of the investments of Pradeep, n(A  B) = n(A) + n(B) - n (A  B)
Mohit and Raghu respectively = 35 + 45 - 20 = 60
= 108 : 90 : 100 = 54 : 45 : 50  Percentage of failed students in Hindi or
Sum of ratios = 54 + 45 + 50 = 149 English or both = 60%
 Successful students = 40%
LEARN MATHS FROM S.K. RAJU (9811549822, 9811649822)
15
SBI PO EXAMS 5. (3) Let number of children be x
1. (1) No. of candidates appeared in state A  No. of sweets received by each
= No. of candidates appeared in state B = x 405
child =
7x 6 x
 - x = 80
100 100 405
x = 8000  = 20% of x
x
2. (4) The question cannot be answered
because total marks are not given. 405 x
 =
Note : The answer choice (1) i.e., 88% marks x 5
obtained by following method would be  x2 = 405 × 5
wrong, 60 × 8 - 56 × 7 = 88
For example, x= 405  5
Let the maximum marks in each of the first = 81  5  5 = 9 × 5 = 45
7 examinations be 100. and in the 8th exam  Required no. of sweets received by each
only 50.
Total maximum marks in first 7 exams 405
child = =9
= 7 × 100 = 700 45
Total marks obtained in first of exams 6. (3) Required salary
4
56  50 
= × 700 = 392 = 10000 1  
100  100 
Total maximum marks obtained in 8 exams = 10000 × (1.5)4 = Rs. 50625
= 700 + 50 = 750 7. (5) Let Mr. Yadav’s annual Salary be x.
For the average per centage in 8 exams to Amount spent on consumables = 0.80 x.
be 60% , the total marks obtained should Amount spent on clothes and transport
60 = 0.50 (x - 0.80x) = 0.50 × 0.20x = 0.10 x
be = × 750 = 450 Savings = x - 0.80x - 0.10x = 0.10x
100
 Marks to be obtained in the eighth exam  0.10x = Expenditure on clothes and
should be 450 - 392 = 58. But, 58 > 50 transport = 5370
It means the marks to be obtained would  Monthly expenditure
be greater than the maximum marks. This 5370
is impossible. = = Rs. 447.50
12
3. (5) 40% houses have two or more people. 8. (1) 170 % of 1140
 60% of all houses have only one person
of these 60% , 25% have only a male. 170
= 1140 × = 1938
25% of 60% = 0.25 × 0.60 100
= 0.15 = 15% 9. (2) 6% of Sarang’s monthly income
Rest of the houses have exactly one female = Rs. 2100
and no males = (60 - 15)% = 45% 2100  100
4. (2) Let the strength of the school in 1996  Sarang’s monthly income =
6
be 100
The strength increases and decreases every = Rs. 35000
alternate year by 10% .  Percentage of monthly investment = 23%
 Strength in 1997 = 110 Strength in 1998  Sarang’s annual investment
90 23
= 110 × = 99 = 12 × 35000 × = Rs. 96600
100 100
Strength in 1999 10. (4) Required answer

110 25 75 3
= 99 × = 108.9 = × × × 4240
100 100 100 5
Strength in 2000 90 = 477
11. (3) Let Ms. Pooja Pushpan’s monthly salary
90 = Rs. x
= 108.9 × = 98.01
100 According to the question,
100 - 98.01  2% discrease
LEARN MATHS FROM S.K. RAJU (9811549822, 9811649822)
16
13% of the x = Rs. 8554
9
x
 8554  100  2 9
 x = Rs.   = Rs. 65800  =
 13  y  4 22
Total monthly investment in percentage
= 13 + 23 + 8 = 44 x 98 4
 y= =
 Total monthly investment = 44% of Rs. 9  22 11
65800 2. (4) Sujata’s monthly salary
 44  65800   2170  100 
= Rs   = Rs. 28952 = Rs.   = Rs. 31000
 100   7 
 Total annual investment Percentage monthly investment by Sujata
= Rs. (12 × 28952) = Rs. 347424 = 7 + 18 + 6 = 31%
12. (3) Number of sweets received by each Sujata’s annual investment
student
31
= 15 % of 240 = 12 × × 31000 = Rs. 115320
100
15  240 3. (2) If total maximum marks be x,
= = 36
100 then,
 Total number of sweets = 240 × 36 = 8640
x  64
13. (2) Let the total amount be Rs. x. = 2240 - 128 = 2112
The amount spent = 100 - 28 = 72% 100
 72 % of x = Rs. (45760 + 27896 ) 2112  100
?= = 33
72  x 64
 = 73656 Marks obtained by Sujata
100
= 2240 - 907 = 1333
73656  100 Required percentage
x= 72
= Rs. 102300
1333
14. (2) Let the original numbers be x and y = × 100  40%
respectively. 3300
4. (1) Percentage increase
x y  150 xy
 × =
2
10
3 100 = × 100
= 50% of xy 100  10
15. (4) Let Shamin’s salary in June 2009 be Rs. 100 1
x. = = 11 %
9 9
2
 10 
 22385 = x 1  100  INSURANCE EXAMS
 
1. (2) Required answer
11  11
 22385 = x × 16580  16500 
100 = × 100
16580
22385  100
x= = Rs. 18500 80
11  11 = × 100 = 0.48
16580
i.e., less than 1 per cent
RBI GRADE-B OFFICER EXAMS 2. (3) Let the number of students appeared in
x school A = 100
1. (4)Let original fraction be y .  Number of students qualified in school A
= 70
According to the question,
 According to question,
450 Number of students appeared in School B
x 9 = 120
100
= Number of students qualified in School B
400 22
y = 70 + 50% of 70 = 70 + 35 = 105
100
 Required percentage ratio
LEARN MATHS FROM S.K. RAJU (9811549822, 9811649822)
17
105  100 Expenses = Rs. 80
= = 87.5%  Savings = Rs. 20
120
3. (3) Let the number of children be x 1 75
Increase in expenditure = 37 = %
 No. of sweets received by each 2 2
405  His present expenditure = Rs. (80 + 30)
child = = Rs. 110
x
2
405 Increase in his income = 16 %
 = 20% of x 3
x
350
405 x  His present income = Rs.
 = 3
x 5 His present savings
 x 2
= 405 ×5
 350  20
x= 405  5 =   110  = Rs.
 3  3
= 81  5  5 = 9 × 5 = 45  Required percentage savings
 Required no. of sweets received by each
20
405
child = = 9 3  100 = 20 × 3 × 100
45 350 3 350
4. (2) Let the capital be Rs. = x. 3
According to the question,
x  8 1 31 1 40 5
-x× × = 61.50 = =5 %
100 4 100 7 7
8. (4) Sonu’s score = 450 + 300
8x 31x = 750
 - = 61.50
100 400 Sunil’s score = 750 + 60
31x = 810
 8x - = 61.50 × 100  810  54%
4
54
32x  31x  900  × 900 = 60%
 = 6150 810
4
800  55
x 9. (5) Number of boys = = 440
 = 6150 100
4
Number of girls = 800 - 440 = 360
 x = 4 × 6150 = Rs. 24600
5. (1) p = 6q 600  70
Each girl’s fee =
 So, q is less than p by 5q. 100
Note that q has been compared with p. = Rs. 420
 Required percentage Total monthly fee
= Rs. (440 × 600 + 360 × 420)
 5q   5q  1
=  p  100  % =  6q  100  % = 83 %
= Rs. (264000 + 151200)
    3 = Rs. 415200
x 1
6. (5) Let the original fraction be y 10. (3) x  y

According to the question, x y = constant


New value of x
x  500 20
x  110 11x
y  600 = 27 = =
100 10
x 20 6 8 10y
 y = × =  New value of y =
27 5 9 11
7. (2) Let the original income be Rs. x.  Percentage decrease

LEARN MATHS FROM S.K. RAJU (9811549822, 9811649822)


18
y 100 1  q = 50%
= × 100 = =9 % 17. (4) Percentage decrease
11 11 11
y 20
= × 100
100  20
11. (3) x% of y + y% of x
xy xy 2 50 2
= + = xy × = = 16 %  17%
100 100 100 3 3
= 2% of xy 18. (1) Let the number be x.
12. (1) If the number of persons included in According to the question,
survey be x, then x
x × (60 - 20)% = 720 = 96
3
x  40  x = 96 × 3 = 288
= = 720
100  67% of 288
720  100 288  67
x= = 1800 =
100
=192.96
40
13. (4) Water Pulp 19. (5) Number of sweets got by each student
99x x 35  20
98y 2y = =7
100
x = 2y
Number of sweets got by each teacher
 99 × 2y + 2y = 1 kg
 200y = 1 kg 35  40
= = 14
100
1
 100y = 200 × 100  Total number of sweets
= 35 × 7 + 6 × 14
1 = 245 + 84 = 329
= kg = 500 g 20. (4) Let the total amount with Prithvi be Rs.
2
x.
14. (4) Let in 100 students, a students failed.
Percentage of amount spent
 (100 - a)x + a × y = 100z
= (100 - 27) = 73%
 100x - ax + ay = 100z
According to the question,
 100x - 100z = ax - ay 73% of x = (89745 + 51291)
 100 (x - z) = a(x- y)
x  73
100  x  z   = 141036
a = 100
x z
141036  100
15. (4) Water Pulp x=
4x x
73
15y 85y = Rs. 193200
 3y 17y A 30 30  B
Pulp remains same  x = 17y 21. (2) =
100 100
 Water Pulp
68y 17y Ax
 = 3A
3y 17y 100
Loss of water = 65y  B = 3A
 68y + 17y = 85y = 3.4 kg.
Ax
3.4  = 3A
 3y + 17y = 20y = × 20 100
85  x = 3 × 100 = 300
= 0.8 kg.
16. (1) pq = constant 30
22. (2) (x -y)×
100
 100q 
 0 = 100  q  100  % 20
  = (x + y)×
100
 2q =100
LEARN MATHS FROM S.K. RAJU (9811549822, 9811649822)
19
 3x - 3y = 2x + 2y 540  25
 x = 5y x= = 375 cm.
36
1 1  x 24. (4) Water Pulp
y= x =   100  × Fresh grapes 4x x
5  5  100
Dry grapes y 9y
= 20% of x
23. (3) If the height of the papaya tree two years 250  90
Pulp in dry grapes =
ago be x cm, then 100
2 = 225 kg.
 20 
540 = x 1    x = 9y = 225 kg.
 100   Weight of fresh grapes = 5x
36 = 5 × 225 = 1225 kg.
 540 = x × 25

LEARN MATHS FROM S.K. RAJU (9811549822, 9811649822)


20
MODEL EXERCISES
1. Which of the following multipliers will cause 8. Shatabdi Express has a capacity of 500
a number to be increased by 25.3% ? seats of which 10% are in the Executive
(1) 12.53 (2) 125.3 class and the rest being chair cars. During
(3) 1.253 (4) 1253 one journey, the train was booked to 85%
(5) None of these of its capacity. If Executive class was
2. When the price of a radio was reduced by booked to 96% of its capacity, then how
20% , its sale increased by 80% . What was many chair car seats were empty during
the net effect on the sale ? that journey ?
(1) 44% increase (1) 75 (2) 73
(2) 44 % decrease (3) 71 (4) None of these
(3) 66% increase (5) None of these
(4) 75% increase 9. p% of a number P is q% more than r% of
(5) None of these the number R. If the difference between P
3. The ratio of the prices of two house A and B and R is r% of R and if the sum of P and R is
was 4: 5 last year. This year, the price of A 210, then which of the following statements
is increased by 25% and that of B by Rs is always true ?
50000. if their prices are now in the ratio 9 (1) P = 110, R = 100
: 10, the price of A last year was (2) P = 220, R = 200
(1) Rs 360000 (2) Rs 450000 (3) P = 3300, R = 3000
(3) Rs 480000 (4) Rs 500000 (4) All of these
(5) None of these (5) None of these
4. In a co-educational school there are 15 10. A wants to secure an annual income of Rs
more girls than boys. If the number of girls 1500 by investing in 15% debentures of
is increased by 10% and the number of face value Rs 100 each and available for Rs
boys is also increased by 16% there would 104 each. If the brokerage is 1% , then the
be nine more girls than boys. What is the sum of money he should invest is
number of students in the school ? (1) Rs 19642 (2) Rs 10784
(1) 140 (2) 125 (3) Rs 10504 (4) Rs 15000
(3 265 (4) 255 (5) None of these
(5) None of these 11. The population of a variety of tiny bush in
5. The digit at unit’s place of a two-digit an experimental field increased by 10% in
number is increased by 100% and the ten’s the first year, increased by 8% in the
digit of the same number is increased by second year but decreased by 10% in the
50% . The new number thus formed is 19 third year. If the present number of bushes
more than the original number. What is the in the experimental field is 26730, then the
original number nu mber o f varie ty o f bushe s in the
(1) 22 (2) 63 beginning was
(3) 24 (4) None of these (1) 35000 (2) 27000
(5) None of these (3) 25000 (4) 36000
6. If 50% of the 2 : 3 solution of milk and water (5) None of these
is replace d wi th w ater, th en the 12. A worker earns a 5% raise. A year later,
concentration of the solution is reduced by the worker receives a 2.5% cut in pay and
(1) 25% (2) 33.33% now his salary is Rs 22702.68. What was
(3) 50% (4) 75% his salary to begin with ?
(5) None of these (1) Rs 22000 (2) Rs 22176
7. After having spent 35% of the money on (3) Rs 25000 (4) Rs 22193
machinery, 40% on raw material and 10% (5) None of these
on staff, a person is left with Rs 60000. The 13. A company has 6435 bars of soap. If the
total amount of money spent on machinery company has sold 20% of its stock, then
and raw material is how many bars of soap did it sell ?
(1) Rs 176000 (2) Rs 170000 (1) 1237 (2) 1257
(3) Rs 300000 (4) Rs 340000 (3) 1287 (4) 1300
(5) None of these (5) None of these

LEARN MATHS FROM S.K. RAJU (9811549822, 9811649822)


21
14. A screw driver and a hammer currently have bc
the same price. If the price of a screw driver (3) % (4) abc%
c
rises by 5% and the price of hammer goes
up by 3% , then how much more will it cost (5) None of these
to buy, 3 screw drivers and 3 hammers ? 20. A country follows a progressive taxation
(1) 3% (2) 4% system under which the income tax rate
(3) 5% (4) 8% applicable varies for different slabs of
(5) None of these in co me . To tal tax is compu te d by
15. Village A has a population of 6800, which calculating the tax for each slab and adding
is decreasing at the rate of 120 per year. them up. The rates applicable are as follows
Village B has a population of 4200, which Annual Income Tax Rate
is increasing at the rate of 80 per year. In 0 - 50000 0%
how many years will the population of the 50001 - 60000 10%
two villages be equal ? 60001 - 150000 20%
(1) 9 yr (2) 11 yr > 150000 30%
(3) 13 yr (4) 16 yr If any income is Rs 170000, then what is
(5) None of these the tax payable by me ?
16. x% of y is y% of (1) 51000 (2) 17000
(3) 34000 (4) 25000
y (5) None of these
(1) x (2)
100 21. The weight of an empty bottle is 20% of the
x weight of bottle when filled with some
(3) (4) 100x liquid. Some of the liquid has been removed.
100
Then, the bottle, along with the remaining
(5) None of these liquid, weighed half of the original weight.
17. Peter got 30% of the maximum marks in What fractional part of the liquid has been
an examination and failed by 10 marks. removed ?
Ho we ve r, Pau l who too k th e same
examination got 40% of the total marks and 2 3
(1) (2)
got 15 more than the passing marks in the 3 9
examination. What were the passing marks 2
in the examination ? (3) (4) None of these
(1) 35 (2) 250
5
(3) 75 (4) 85 (5) None of these
(5) None of these 22. There are five boxes in a cargo hold. The
18. In a facto ry , produ ci ng parts o f an weight of the first box is 200 kg and the
automobile, the parts manufactured on the weight of the second box is 20% higher than
shop floor are required to go through quality the weight of the third box, whose weight
checks, each conducted after specific part is 25% higher than the first box. The fourth
of the processing on the raw material is box at 350 kg is 30% higher than the fifth
completed. Only parts that are not rejected box. The difference in average weight of the
at one stage are put through subsequent four heaviest boxes and the four lightest
stages of production and testing. If average boxes is
rejection rates at these three testing (1) 51.5 kg (2) 75 kg
machines during a month are 10% , 5% and (3) 37.5 kg (4) 112.5 kg
2% respectively, then what is the effective (5) None of these
rejection rate for the whole plant ? 23. At what price should I buy a share, the
(1) 17% (2) 15.20% value of which is Rs 100, paying a dividend
(3) 84.80% (4) 16.21% of 8% , so that my yield is 11% ?
(5) None of these (1) Rs 70 (2) Rs 72.72
19. If a% of x is equal to b% of y, then c % of y (3) Rs 75 (4) Rs 84
is what per cent of x ? (5) None of these
24. 40% of the students in a college play Basket
ac ball, 34% of the students play Tennis and
(1) c% (2) %
b the number of students who play both the
games is 234. The number of students who

LEARN MATHS FROM S.K. RAJU (9811549822, 9811649822)


22
play neither Basket ball nor Tennis is 52% . value of each type of share being Rs 10.
Determine the student population in the The company had a total profit of Rs 180000
college. out of which Rs 30000 was kept in reserve
(1) 800 (2) 1050 fund and the remaining distributed to
(3) 900 (4) 850 share-holders. Find the dividend per cent
(5) None of these to the common shareholders.
25. The capital of a company is made up of (1) 20% (2) 35%
50000 preferred shares with dividend of (3) 25% (4) 30%
20% and 20000 common shares, the par (5) None of these

LEARN MATHS FROM S.K. RAJU (9811549822, 9811649822)


23
SHORT ANSWERS then the number of girls = (x + 15)
1. (3) 2. (1) New number of girls = (x + 15) × 1.1
3. (1) 4. (3) New number of boys = x × 1.16
5. (4) 6. (3) According to question,
7. (3) 8. (2) 1.1(x + 15) = 1.16x + 9
9. (1) 10. (3)  1.16x - 1.1x = 16.5 - 9 = 7.5
11. (3) 12. (2)  0.06x = 7.5
13. (3) 14. (2)
7.5
15. (3) 16. (1)  x= = 125
17. (4) 18. (4) 0.06
19. (2) 20. (4)  Number of boys = 125
21. (4) 22. (2) Number of girls = 140
23. (2) 24. (3)  Number of students = 265
25. (3) 5. (4) This question can be solved making use
of the options.
EXPLANATIONS Original New
Number Number Difference
1. (3) Let the number be 1 and multiplier be
Option (1) 22 34 12
x, then original number = 1
Option (2) 63 96 33
New number = 1 × x = x
Option (3) 44 68 24
Percentage increase
Therefore, none of the options (4) is correct.
x 1 6. (3) Let the quantity of milk and water be 40
= × 100 = 25.3
1 and 60 respectively. After removing 50% of
solution, Quantity of milk = 20 and quantity
25.3
 (x - 1) = of water = 30
100 Therefore, the concentration of the solution
 (x - 1) = 0.253 is reduced from 40 to 20. Hence the solution
 x = 1.253 is reduced by 50% .
2. (1) Let original price of radio be x and sale 7. (3) Let the original money be Rs 100,
be y. then money spent on machinery = Rs 35
Then, total revenue collected = x × y Money spent on raw material = Rs 40
New price = 0.8x, new sale = 1.8y Money spent on staff = Rs 10
New revenue collected = 1.44 xy Money left = 100 - 85 = Rs 15
Percentage increase in revenue If money left is Rs 15 original money =100
0.44xy If money left is 60000 original
= xy × 100 = 44% increase 100
money = × 60000 = 400000
3. (1) Let the prices of two houses be Rs 4x 15
and 5x respectively for A and B. Hence the total money spent on machinery
Then, current price of A and B this year are and raw material
1.25 × 4x and Rs (5x + 50000) respectively. = 400000 × 75% = Rs 300000
According to the question, 8. (2) Seats in executive class = 50
Seats in chair car = 450
1.25  4x 9 Booking seats in total = 425
=
5x  50000 10 Booking in executive class = 48
 50x - 45x = 450000  Booking in chair class
 5x = 450000 = (425 - 48) = 377
Empty seats in chair class
450000 = 450 - 377 = 73
 x=
5 9. (1) According to question,
 x = Rs 90000 P + R = 210
 The price of house A last year was 4x Using, options, we find that
= Rs 360000 P = 110 and R = 100 is correct.
4. (3) Let the number of boys = x, 10. (3) Let Rs x be the face value of debentures,
Then 15% of x = 1500

LEARN MATHS FROM S.K. RAJU (9811549822, 9811649822)


24
 x × 15% = 1500 Then, x × 30% + 10 = x × 40% - 15
 x = 10000  x × 10% = 25
 Available value of debentures  x = 250
 104  30
=   10000  = Rs10400 Passing marks = 250 ×
100
+ 10 = 85
 100 
18. (4) Let the parts produced at initial state
Now brokerage = 1% of Rs 10400
be 100.
1 Then, after three successive percentage
= × 10400 = Rs 104
100 rejection of 10% , 5% and 2%
 Sum of money invested = 100 × 0.9 × 0.95 × 0.98 = 83.79
= Rs (10400 + 104) = Rs 10504 Hence, single effective rejection rate for
11. (3) Let the beginning of the year variety of whole plant = 100 - 83.79 = 16.21 %
bushes is x, then 19. (2) According to question,
x × 1.1 × 1.08 × 0.9 = 26730 c a by
=
26730 100 100
 x= = 25000
1.1  1.08  0.9  ax = by
12. (2) Let the original salary of the worker be
c y a c
Rs x. Now, =
Then, x × 1.05 × 0.975 = 22702.68 100 100b
 x × 1.02375 = 22702.68 ac
 c% of y = of x
22702.68 b
x = = Rs 22176 20. (4) Total tax payable on Rs 170000
1.02375
= 50000 × 0% + 10000 × 10% + 90000 ×
13. (3) According to question,
20% + 20000 × 30%
Required number = 6435 × 20%
= 1000 + 18000 + 6000 = Rs 25000
20 21. (4) Let the weight of the filled bottle is 100 g,
= 6435 × = Rs 1287
100 then the weight of the empty bottle is 20 g.
14. (2) Let the original price of screw driver and 100
hammer be 100 each. Now, 100 - x =
2
According to question, price of 3 screw
driver and hammer = Rs 600  x = 50
Now, after increase of 5% price, the p r i c e  Fraction part of the liquid
of 3 screw drivers = Rs 315 50 1
And after increase of 3% price, the price of = =
100 2
3 hammers = Rs 309 22. (2) Weight of the 1st box = 200 kg
Price of 3 hammers and 3 screw drivers
 Weight of the 3rd box = 250 kg
= Rs 624 Weight of 2nd box = 300 kg
24 Weight of 4th box = 350 kg
Percentage increase in price = 600 × 100
 Weight of 5th box = 500 kg
= 4%  Average weight of four heaviest boxes
15. (3) Using options, we can solve this 1400
question, = = 350 kg and average weight of four
4
After 13 yr population of village
A = 6800 - 120 × 13 = 5240 1100
lightest boxes = = 275 kg
After 13 yr population of village 4
B = 4200 + 80 × 13 = 5240 350  200 150
16. (1) According to question,  Difference = 2
=
2
= 75 kg
x x 23. (2) Let the price of share be Rs x
×y=y×
100 100 Given, 11% of x = 8
 x% of y is y% of x. 800
17. (4) Let the maximum marks be x. x = = Rs 72.72
11

LEARN MATHS FROM S.K. RAJU (9811549822, 9811649822)


25
24. (3) Let the number of students in college 25. (3) Dividend for preferred share of company
be 100. 20
Then, number of students who play both = 50000 × 10 × = Rs 100000
100
games
= (34 + 40) - (48) = 26 Rest divident amount of company
If 26 plays both the games, then total = 180000 - 100000 - 30000 = Rs 50000
number of students =100  Dividend per cent for common
If 234 play both the games, then total 50000  100
share = = 25%
number of students 20000  10
100
= × 234 = 900
26

LEARN MATHS FROM S.K. RAJU (9811549822, 9811649822)


1
PROFIT AND LOSS
NATIONALISED BANKS 6. A shopkeeper bought 30 kg. of wheat at the
& IBPS PO/MT/SO rate of Rs.45 per kg. He sold forty percent
1. Sri Ganesh bought 40 kgs of wheat at Rs. of the total quantity at the rate of Rs. 50
12.50 per kg and 25 kgs of it at Rs. 15.10 per kg. Approximately, at what price per
per kg. He mixed them together. At what kg. should he sell the remaining quantity
rate should he sell the mixture to earn 10% to make 25 per cent overall proflt ?
proflt ? (1) Rs. 54 (2) Rs. 52
(1) Rs. 13.50 (2) Rs. 13.25 (3) Rs. 50 (4) Rs. 60
(3) Rs. 14.75 (4) Rs. 14.85 (5) Rs. 56
(5) None of these (Allahabad Bank PO Exam. 21.02.2010)
(Syndicate Bank PO Exam. 10.10.2004) 7. Meenal purchased a car for Rs. 2,50,000
2. The proflt earned after selling an article for and sold it for Rs. 3,48,000. What is the
Rs. 1,516 is the same as loss incurred after per cent profit she made on the car ?
selling the article for Rs. 1,112. What is the (1) 40 (2) 39.2
cost price of the article ? (3) 38.4 (4) 38
(1) Rs. 1,314 (2) Rs. 1,343 (5) None of these
(3) Rs. 1,414 (4) Rs. 1,434 (Corporation Bank PO Exam.
(5) None of these 09.05.2010)
(Orientai Bank of Commerce 8. Rajni purchased a mobile phone and a
PO Exam. 21.12.2008) refrigerator for Rs. 12,000 and Rs. 10,000
3. The proflt earned after selling an article for respectively. She sold the refrigerator at a
Rs. 1,754 is the same as loss incurred after loss of 12 per cent and mobile phone at a
selling the article for Rs. 1,492. What is the proflt of 8 per cent. What is her overall loss/
cost price of the article ? profit ?
(1) Rs. 1,623 (2) Rs. 1,523 (1) Loss of Rs. 280
(3) Rs. 1,689 (4) Rs. 1,589 (2) Profit of Rs. 2,160
(5) None of these (3) Loss of Rs. 240
(Canara Bank PO Exam. 15.03.2009) (4) Profit of Rs.2,060
4. Giridhar purchased 100 sarees @ Rs. 450 (5) None of these
per piece. While selling he offered 10% (Punjab & Sind Bank
discount on the labelled price and earned PO Exam. 16.05.2010)
a proflt of 20% . What was the labelled price 9. A shopkeeper sells notebooks at the rate of
of each saree ? Rs. 45 each and earns a commission of 4% .
(1) Rs. 540 (2) Rs. 650 He also sells pencil box at the rate of Rs.
(3) Rs. 590 (4) Rs. 600 80 each and earns a commission of 20% .
(5) None of these How much amount of commission will he
(PNB Specialist Officer’s earn in two weeks if he sells 10 notebooks
Exam. 16.08.2009) and 6 pencil boxes a day ?
5. Vinod makes a proflt of Rs. 110 if he sells a (1) Rs. 1,956 (2) Rs. 1,586
certain number of pencils he has at the (3) Rs. 1,496 (4) Rs. 1,596
price of Rs. 2.5 per pencil and incurs a loss (5) None of these
of Rs. 55 if he sells the same n umbe r of (Central Bank Of India
pencils for Rs. 1.75 per pencil. How many PO Exam. 25.07.2010)
pencils does Vinod have ? 10. A man sold a wristwatch for Rs. 2,400 at a
(1) 220 loss of twenty five percent. At what rate
(2) 240 should he have sold the wristwatch to earn
(3) 200 a proflt of twenty five per cent ?
(4) Cannot be determined (1) Rs. 3,600
(5) None of these (2) Rs. 4,000
(Indian Bank Rural Marketing (3) Rs. 3,500
Officer Exam. 03.01.2010) (4) Rs. 3,800
(5) None of these
(Bank Of India PO Exam. 31.10.2010)

LEARN MATHS FROM S.K. RAJU (9811549822, 9811649822)


2
11. What profit/loss per cent did Ravi earn if sold it at a gain of 12 percent. What was
he purchased an item of Rs 5,600 and sold her overall gain/loss ?
it at three-fourth of its cost price ? (1) Loss of Rs 662.40
(1) Loss of 20 per cent (2) Profit of Rs 662.40
(2) Gain of 25 per cent (3) Loss of Rs 642.80
(3) Neither gain nor loss (4) Profit of Rs 642.80
(4) Loss of 15 per cent (5) None of these
(5) None of these (Allahabad Bank PO Exam. 17.04.2011)
(Orientai Bank Of Commerce 17. An article was purchased for Rs 78,350. Its
PO Exam. 26.12.2010 (Ist Sitting) price was marked up by 30% . It was sold
12. Prasad sold his work tools for Rs 1,850 and at a discount of 20% on the marked up
earned a profit of 25 per cent. At what price price. What was the profit percent on the
did Prasad buy the work tools ? cost price ?
(1) Rs 1,360 (2) Rs 1,300 (1) 4 (2) 7
(3) Rs 1,240 (4) Rs 1,480 (3) 5 (4) 3
(5) None of these (5) 6
(Indian Bank PO Exam. 02.01.2011) (IBPS Bank PO/MT CWE 17.06.2012)
13. Rehaan purchased a bike for Rs 54,000. He 18. A man sells calculator at the rate of Rs 250
sold it at a loss of 8 per cent. With that each which includes a profit of 14 per cent.
money he again purchased another bike What amount of profit will he eam in 19
and sold it at a proflt of 10 per cent. What days if he sells seven calculators per day ?
is his overall loss/profit ? (1) Rs 4665 (2) Rs 4565
(1) Loss of Rs 657 (3) Rs 4545 (4) Rs 4655
(2) Profit of Rs 567 (5) None of these
(3) Loss of Rs 648 (IDBI Bank Officer Exam. 16.09.2012)
(4) Profit of Rs 648 19. Manish bought 25 kg of rice at Rs. 32 per
(5) None of these kg and 15 kg of rice at Rs. 36 per kg. What
(Corporation Bank PO profit did he get when he mixed the two
Exam. 16.01.2011) varieties together and sold it at Rs. 40.20
14. Meera purchased an item for Rs 62,000 and per kg ?
sold it at loss of 25 percent. With that (1) 25% (2) 40%
amount she purchased another item and (3) 30% (4) 20%
sold it at a gain of 30 percent. What was (5) None of these
her overall gain/loss ? 20. A shopkeeper sells two watches for Rs. 308
(1) Los of Rs 560 each. On one he gets 12% proflt and on
(2) Profit of Rs 1560 the other 12% loss. His profit or loss in
(3) Loss of Rs 1550 the entire transaction was
(4) Profit of Rs 1550
11
(5) None of these (1) 1 % loss
(UCO Bank PO Exam. 30.01.2011) 25
15. Seema purchased an item for Rs 9,600 and 11
sold it for a loss of 5 percent. From that (2) 1 % gain
25
money she purchased another item and
sold it for a gain of 5 percent. What is her 2
overall gain/loss ? (3) 3 % loss
25
(1) Loss of Rs 36
(2) Profit of Rs 24 2
(4) 3 % gain
(3) Loss of Rs 54 25
(4) Profit of Rs 36 (5) None of these
(5) None of these (IBPS Bank PO/MT
(Bank Of Baroda PO Exam. 13.03.2011) CWE-III 26.10.2013)
16. Kamya purchased an item of Rs 46,000 and 21. An article was sold for Rs. 5220 at a loss of
sold it at a loss of 12 percent. With that 42% of the cost price. What will be the selling
amount she purchased another item and price of the article for a proflt of 42% ?
(1) Rs. 12580 (2) Rs. 17280

LEARN MATHS FROM S.K. RAJU (9811549822, 9811649822)


3
(3) Rs. 12780 (4) Rs. 15280 5. A trader sells 145 metres of cloths for Rs.
(5) None of these 12,325 at the profit of Rs. 10 per metre of
(Corporation Bank Specialist Officer cloth. What is cost price of 1 metre of cloth ?
(Marketing) Exam. 22.12.2014) (1) Rs. 65 (2) Rs. 75
(3) Rs. 95 (4) Rs. 85
SBI PO EXAMS (5) None of these
1. Mr. X, a businessman had the income in (SBI PO Preliminary (Tire-I) Exam.
the year 1995, such that he earned a proflt 27.04.2008)
of 20% on his investment in the business. 6. The profit earned after selling an article for
In the year 1996 his investment was less Rs. 878 is the same as loss incurred after
by Rs. 5,000 but still had the same income selling the article for Rs. 636. What is the
(Income = Investment + Profit) as that in cost price of the article ?
1995. Thus the per cent profit earned in (1) Rs. 797 (2) Rs. 787
1996 increased by 6% . What was his (3) Rs. 767 (4) Rs. 757
investment in 1995 ? (5) None of these
(1) Rs. 1,02,000 (SBI PO Preliminary
(2) Rs. 1,50,500 (Tire-I Exam. 27.07.20O8)
(3) Rs. 1,05,000 7. The value of machine depreciates at the rate
(4) Data inadequate of 12 per cent per annum. It was purchased
(5) None of these three years ago. Its present value is Rs.
(SBI Banks PO Exam. 11.02.2001) 29644.032, what was the purchase price
2. Mr. Shivkumar started a businees investing of the machine ?
Rs. 25,000 in 1996. In 1997 he invested an (1) Rs. 48700 (2) Rs. 43500
additional amount of Rs. 10,000 and Mr. (3) Rs. 38900 (4) Rs. 39000
Rakesh joined him with an amount of Rs. (5) None of these
35,000. In 1998 Mr. Shivkumar invested (SBI PO Preliminary (Tire-I)
another additional amount of Rs. 10,000 Exam. 27.07.2008)
and Mr. Suresh joined them with an 8. Proflt e arne d by an o rg an izatio n is
amoun t of Rs. 35,000. Wh at wi ll be distributed among officers and clerks in the
Rakesh’s share in the profit of Rs. 1,50,000 ratio of 5 : 3 respectively. If the number of
earned at the end of three years from the officers is 45 and the number of clerks is
start of the business in 1996 ? 80 and the amount received by each officer
(1) Rs. 70,000 (2) Rs. 50,000 is Rs 25,000, what was the total amount of
(3) Rs. 45,000 (4) Rs. 75,000 proflt earned ?
(5) None of these (1) Rs 22 lakhs
(SBI Banks PO Exam. 11.02.2001) (2) Rs 18.25 lakhs
3. If on selling 12 notebook any seller makes (3) Rs 18 lakhs
a profit equal to the selling price of 4 (4) Rs 23.25 lakhs
notebooks. What is his per cent profit ? (5) None of these
(1) 50 (2) 25 (SBI Associate Banks
PO Exam. 07.08.2011)
2 9. A shopkeeper labelled the price of his
(3) 16 (4) Data inadequate
3 articles so as to earn a profit of 30% on the
(5) None of these cost price. He then sold the articles by
(SBI PO Exam. 26.11.2006) offering a discount of 10% on the labelled
4. The proflt after selling a pair trousers for price. What is the actual percent profit
Rs. 863 is the same as the loss incurred after earned in the deal ?
selling the same pair of trousers for Rs.631. (1) 18%
What is the cost price of the pair of trousers ? (2) 15%
(1) Rs. 750 (3) 20%
(2) Rs. 800 (4) Cannot be determined
(3) Rs. 763 (5) None of these
(4) Cannot be determined (SBI Associate Banks
(5) None of these PO Exam. 07.08.2011)
(SBI PO Preliminary
(Tire-I) Exam. 27.04.2008)
LEARN MATHS FROM S.K. RAJU (9811549822, 9811649822)
4
INSURANCE EXAMS (3) Rs. 550 (4) Rs. 600
1. If a shirt costing Rs. 385 is sold at a loss of (New India Assurance AO
5% of the cost price, flnd the selling price. Exam. 25.10.2009)
(1) Rs. 364 (2) Rs. 364.74 7. A vendor sells calculators at the rate of Rs
(3) Rs. 365 (4) Rs. 365.75 250 each and eams a commission of 20%
(United India Insurance Co. on each. He also sells pens at the rate of Rs
AAO Exam. 21.04.2002) 50 each and earns a commission of 10%
2. If the cost price of 24 articles is equal to on each. How much amount of commission
the selling price of 21 articles, what is the will he earn in three days if he sells 10
percentage gain or loss ? calcula-tors and 5 pens a day ?
(1) Rs 1,575 (2) Rs 1,445
2 (3) Rs 1,550 (4) Rs 1,450
(1) 14 % gain
7 (5) None of these
2 (United India Insurance
(2) 14 % loss AO Exam. 27.03.2011)
7
8. A person bought 864 articles and sold 800
(3) 12.5% loss of them for the price he paid for 864 articles.
(4) 12.5% gain He sold the remaining articles at the same
(United India Insurance Co. price per article as the other 800. The
AAO Exam. 21.04.2002) percentage gain on the entire transaction
3. A merchant has 1000 kg of sugar, part of is
which he sells at 8% proflt and the rest at (1) 7.5% (2) 8%
18% profit. He gains 14% on the whole. (3) 8.5% (4) 9%
The quantity (in kg.) sold at 18% profit is : (New India Insurance AAO
(1) 560 (2) 600 Exam. 22.05.2011)
(3) 400 (4) 640 9. The currencies in countries M and N are
(United India Insurance Co. denoted by m and n respectively. The
(AAO) Exam. 11.03.2007) exchange rate in 1990 was l m, 0.6 n the
4. A shopkeeper sold a T.V. set for Rs 17,940/ price level in 2006 in M and N are 150 and
-, with a discount of 8% and gained 19.6% 400 respectively with 1990 as a base of 100.
If no discount is allowed, what will be his The exchange rate in 2006 based solely on
gain per cent ? the purchasing power parity consideration
(1) 25% is 1 m :
(2) 26.4% (1) 0.225 n (2) 0.625 n
(3) 24.8% (3) 1.6 n (4) 3.6 n
(4) Cannot be determined (General Insurance Corporation
(5) None of these AAO Exam. 11.12.2011)
(LIC Assistant Administrative 10. Prof. Chatterjee bought a car and got 15%
Offlcer Exam. 2008) of its original price as a dealer’s discount.
5. The income of a broker remains unchanged He then sold it at 20% profit on his
though the rate of commission is increased purchase price. What percentage profit did
from 4 per ce nt to 5 per cen t. The he get on the original price ?
percentage of slump in business is : (1) 2% (2) 12%
(1) 10 per cent (3) 5% (4) 17%
(2) 15 per cent (General Insurance Corporation
(3) 20 per cent AAO Exam. 11.12.2011)
(4) 30 per cent 11. Cost of 4 fans and 3 blowers is Rs 16,500.
(LIC Assistant Administrative Also cost of 2 fans, 2 tables and 2 blowers
Offlcer (AAO) Exam. 07.06.2009) is Rs 12,000. Cost of one table is Rs 1,000.
6. A watch is sold at a proflt of 20% . If both What is the cost of 3 fans and one blower ?
the cost price and the selling price of the (1) Rs 8,000
watch are decreased by Rs. 100, the profit (2) Rs 7,500
would be 5% more. Originai cost price of (3) Rs 8,500
the watch is (4) Cannot be determined
(1) Rs. 450 (2) Rs. 500 (5) None of these

LEARN MATHS FROM S.K. RAJU (9811549822, 9811649822)


5
(United India Insurance AO 15. By selling a table for Rs. 350 instead of Rs.
Exam. 27.03.2011) 400, loss percent increases by 5% . The cost
12. By selling 18 chocolates, a vendor loses the price of the table is:
selling price of 2 chocolates. Find his loss (1) Rs. 435 (2) Rs. 417.50
percent. (3) Rs. 1000 (4) Rs. 1050
(1) 9% (2) 10% (NICL (GIC) Administrative
(3) 11% (4) 12% Officer Exam. 15.12.2013)
(United India Insurance 16. Ram bought 1600 eggs at Rs. 3.75 per
AAO Exam. 03.06.2012) dozen. He sold 900 of them at 2 for Re 1
13. The profit earned after selling an article for and the remaining at 5 for Rs. 2. His gain
Rs. 1,516 is the same as loss incurred after per cent is :
selling the article for Rs. 1,112. What is the (1) 40% (2) 45%
cost price of the article ? (3) 42% (4) 46%
(1) Rs. 1,314 (2) Rs. 1,343 (NICL (GIC) AO (Finance)
(3) Rs. 1,414 (4) Rs. 1,434 Exam. 15.12.2013)
(5) None of these 17. If the man ufacturer g ain s 10% , the
(United India Insurance AO Wholesale dealer 15% and the retailer 25% ,
Exam. 26.05.2013 then the cost of production of an article
14. If the cost price of 15 articles is equal to whose retail price is Rs 1265, is:
the selling price of 12 articles, find gain per (1) Rs. 632.50 (2) Rs. 800
cent. (3) Rs. 814 (4) Rs. 834.34
(1) 20 (2) 25 (NICL (GIC) AO (Finance) Exam.
(3) 18 . (4) 21 15.12.2013)
(NICL (GIC) AO Exam.
08.09.2013 (Paper-I)

LEARN MATHS FROM S.K. RAJU (9811549822, 9811649822)


6
SHORT ANSWERS 2628
NATIONALISED BANKS x= = Rs. 1314
2
& IBPS PO/MT/SO 3. (1) Let the CP of the article be Rs. x
1. (4) 2. (1) According to the question,
3. (1) 4. (4) 1754 - x = x - 1492
5. (1) 6. (4)  2x = 1754 + 1492 = 3246
7. (2) 8. (3)
9. (5) 10. (2) 3246
x = = Rs. 1623
11. (5) 12. (4) 2
13 (4) 14. (3) 4. (4) Let the labelled price of each sari be Rs.
15. (5) 16. (1) x.
17. (1) 18. (5) According to the question,
19. (4) 20. (1)
120  450
21. (3) 90% of x =
100
SBI PO EXAMS _ 90  x 120  450
1. (3) 2. (2)  =
100 100
3. (1) 4. (5)
5. (2) 6. (4) 120  450
x= = Rs. 600
7. (2) 8. (4) 90
9. (5) 5. (1) Let Vinod have x pencils.
 2.5 × x - 1.75 × x = 110 + 55
INSURANCE EXAMS  0.75 × x = 165
1. (4) 2. (1)
165
3. (2) 4. (5) x= = Rs 220
5. (3) 6. (2) 0.75
7. (1) 8. (2) 6. (4) Cost price of 30 kg of wheat
9. (1) 10. (1) = 30 × 45
11. (1) 12. (2) = Rs. 1350
13. (1) 14. (2) Total SP for an overall profit of
15. (3) 16. (4) 1350 125
17. (2) 25% = = Rs 1687.5
100
 30  40 
EXPLANATIONS Sp of   = 12 kg of wheat
 100 
NATIONALISED BANKS
& IBPS PO/MT/SO = 12 × 50 = Rs. 600
1. (4) Total weight of the mixture Expected SP of 18kg of remaining wheat
= 40 + 25 = 65 kg = 1687.5 - 600 = Rs. 1087.5
Total cost price of wheat Required selling price per kg
= Rs. (40 x 12.50 + 25 × 15.10) 1087.5
= Rs. (500 + 377.50) = Rs. 877.50 =
18
 Rs. 60
Total selling price of wheat
7. (2) Tricky Approach
877.50 110 Gain per cent
= Rs.
100
348000  250000
= Rs. 965.25 = × 100
250000
965.25 = 39.2%
 SP per kg = Rs. 65 8. (3) Total CP
= Rs. 14.85 = Rs. (12000 + 10000) = Rs. 22000
2. (1) Let the cost price of the artice be Rs. x.  12000 108 10000  88 
According to the question, Total S.P. =   
1516 - x = x - 1112  100 100 
 2x = 1516 + 1112 = 2628 = Rs. (12960 + 8800) = Rs. 21760
 Loss = Rs. (22000 - 21760) = Rs. 240
LEARN MATHS FROM S.K. RAJU (9811549822, 9811649822)
7
9. (5) Required amount of commission 40480 112
Second S.P. = = Rs. 45337.6
 10  45  4 80  20  6  100
= Rs.   
 100 100   Loss = Rs. (46000 - 45337.6) = Rs. 662.4
= Rs. (18 + 96) = Rs. 114 17. (1) Gain per cent
10. (2) CP. of wristwatch  30  20 
=  30  20   % = 4%
100  100 
= × S.P
100  loss% 18. (5) Profit on one calculator
 100   14 
= Rs.   2400  = Rs.  250  
 75   114 
= Rs. 3200 7 19  250  14
 Required S.P. of wristwatch  Total profit = = Rs. 4083.33
114
 125  19. (4) CP. of 40 kg of mixture
= Rs.   3200 
 100  = Rs [(25 × 32) + (15 × 36)]
= Rs. 4000 = Rs (800 + 540) = Rs 1340
S.P. of 40 kg of mixture
3 = Rs (40 × 40.2) = Rs 1608
11. (5) S.P. = 5600 ×
4 Profit = Rs (1608 - 1340) = Rs 268
= Rs. 4200
Loss = 5600 - 4200 268
Profit % = × 100 = 20%
= Rs. 1400 1340
 Loss percentage 20. (1) Loss per cent
2
Loss  Common gain or less 
= × 100 =  %
Cost price  10 
2
1400  12  36 11
= ×100 = 25% =   % = %= 1 %
5600  10  25 25
100 21. (3) CP. of article
12. (4) CP = 100  Profit per cent × S.P
100 5220 100
= 5220 × 100  42  = = Rs. 9000
100 58
= × 1850 = Rs. 1480
125 9000 142
 Required S.P. = = Rs 12780
54000  92 100
13. (4) First S.P. = = Rs 49680
100
110
SBI PO EXAMS
Second SP = 49680 × = Rs 54648 1. (3) Let the investment in 1995 be Rs. x
100
 Income in 1995 with 20% profit = 1.20x
 Profit = 54648 - 54000 = Rs 648 Also the income in 1996 = 1.20x
14. (3) Last selling price Investment in 1996 = Rs. (x - 5000)
75 130 Profit in 1996 = (20 + 6)% = 26%
= 62000 × × = Rs. 60450 Income in 1996 with 26% profit
100 100
= 1.26 (x - 5000)
 Loss = Rs. (62000 - 60450) = Rs. 1550
 1.26 (x - 5000) = 1.20x
9600  95 0.06 x = 1.26 × 5000
15. (5) First S.P. = = 9120
100
6300
9120  105 x=
0.06
Second S.P. = = Rs. 9576
100 x = Rs. 105000
 Loss = Rs. (9600 - 9576) = Rs. 24 2. (2) Shivkumar’s equivalent capital
= (25000 × 3) + (10000 × 2) + (10000 × 1)
46000  88
16. (1) First S.P. = = Rs. 40480 = 105000
100 Rakesh’s equivalent capital

LEARN MATHS FROM S.K. RAJU (9811549822, 9811649822)


8
= 35000 × 2 = 70000 = Rs. 117
Suresh’s equivalent capital  Actual gain per cent = 17
= 35000 × 1 = 35000
Total profit of Rs. 150000 will be distributed INSURANCE EXAMS
in the ratio of theii equivalent capitate. 1. (4) Required answer
Shivkumar : Rakesh : Suresh
= 105 : 70 : 35 = 3 : 2 : 1 95
= × 385 = Rs. 365.75
Rakesh’s share in the total profit 100
2 2. (1) Percentage gain
=
3 2 1
×150000 = Rs. 50000  24  21 100
= × 100 = %
3. (1) Profit = Selling price of 4 notebooks cost 21 7
price = selling price of(12 - 8) = 4 notebooks. 2
4 = 14 %
7
 % proflt = × 100 = 50
8 3. (2)
4. (5) Let the cost price of the trouser be Rs. Profit on first part Profit on second part
x.
According to the question,
863 - x = x - 631
 2x = 863 + 631 = 1494
1494
x= = Rs. 747
2
5. (2) Cost price of 145m long cloth Ratio of first and second parts
= Rs. (12325 - 145 × 10) = Rs. 10875 = 4 :6 = 2 :3
 Quantity of 2nd part
10875
 CP of 1 metre long cloth = 145 = Rs. 75 3 
=   1000  kg = 600 kg
6. (4) let the CP. of the article be Rs. x.  5 
According to the question, 4. (5)  If Rs. 92 is the S.P., the Marked Price
878 - x = x - 636 is Rs. 100.
 2x = 878 + 636 = 1514  If Rs. 17,940 is the S.P., the Marked Price
1514 100
x= = Rs. 757 = × 17940 =Rs. 19,500
2 92
3
Now, Cost Price
 12 
7. (2) 29644.032 = P  1   100
 100  =17940 ×
100  19.6
 29644.032 = P × (0.88)3
100
29644.032 = 17940 × = Rs. 15,000
P= = Rs. 43500 119.6
0.88  0.88  0.88
If there is no discount,
8. (4) Profit received by each officer
S.P. = Rs. 19,500
= Rs. 25000
 Proflt received by each clerk 3 19,500  15000
 % Profit = 15000
× 100
3
= × 25000 = Rs. 15000
5 4500
= × 100 = 30%
 Total earned profit 15000
= Rs. (45 × 25000 + 80 × 15000) 5. (3) Let the business value changes from Rs.
= Rs. (1125000 + 1200000) = Rs. 23.25 lakh x to Rs. y.
9. (5) Let the cost price of the article be Rs.  4% of x = 5% of y
100.
4x 5 y
 Marked price = Rs.130  =
100 100
130  90
 S.P. of the article = 100

LEARN MATHS FROM S.K. RAJU (9811549822, 9811649822)


9
4  4x + 3y = 16500 (i)
y= x 2x + 2y + 2 × 1000 = 12000
5
 2x + 2y = 12000 - 2000
Change in business
= 10000
 4  x  x + y = 5000 (ii)
=  x  x =
 5  5 Equation (ii) × 4 - (i),
y = 20000 - 16500 = 3500
 Percentage slump in business
 x = 5000 - 3500 = 1500
x 1  3x + y = 3 × 1500 + 3500
= × × 100 = 20%
5 x = Rs. 8000
6. (2) Let the CP of the watch be Rs. x 12. (2) If the S.P. of 1 chocolate be Re. 1, then,
120 x 6x S.P. of 18 chocolates = Rs. 18
 First SP = = Rs. C.P. = 18 + 2 = Rs. 20
100 5
Case II, 2
 Loss percent = × 100 = 10%
CP = Rs. (x - 100) 20
13. (1) Let the cost price of the artice be Rs. x.
 6x 
SP = Rs.   100  According to the question,
 5  1516 - x = x - 1112
 6x  x  2x = 1516 + 1112 = 2628
Profit =   x  = Rs.
 5  5
2628
x  x = = Rs. 1314
2
5 100
 x  100 = 25 14. (2) Gain percent
15  12
 25x - 2500 = 20x = 12 × 100 = 25%
= 5x = 2500
15. (3) If the C.P. of table be Rs. x,
2500
x= = Rs. 500 then
5
400  350
7. (1) Commission on 1 calculator ×100 = 5
x
250  20
= = Rs. 50  50 × 100 = 5x
100
 x = Rs. 1000
Commission on 1 pen 16. (4)  C.P. of 12 eggs = Rs. 3.75
50  10  C.P. of 1600 eggs
= = Rs. 5
100 3.75 1600
 Required amount of commissior = = Rs. 500
12
= Rs. 3 × (10 × 50 + 5 × 5) = Rs. 1575
8. (2) Profit percent 1
S.P. of 900 eggs = × 900 = Rs. 450
2
64
=  100 = 8% S.P. of remaining 700 eggs
800
2
1m 0.6 n = × 700 = Rs. 280
9. (1) = 5
150 400 Total S.P. = 450 + 280 = Rs. 730
0.6 Gain = 730 - 500 = Rs. 230
 1m = × 150n = 0.225n
400 230
Gain per cent = × 100 = 46%
10. (1) Gain percent 500
 20 15  17. (2) C.P. of article
=  20  15   % = 2%
 100  100 100 100
= 1265 × × ×
11. (1) Let C.P. of 1 fan = Rs. x 110 115 125
CP of 1 blower = Rs. y = Rs. 800
CP of 1 table = Rs. 1000

LEARN MATHS FROM S.K. RAJU (9811549822, 9811649822)


10
MODEL EXERCISES
1. By selling 12 marbles for a rupee, a pay Rs 2568, inclusive of sales tax. Find
shopkeeper loses 20% . In order to gain 20% the reduction needed in the price of the
in the transaction, he should sell the radio.
marbles at the rate of how many marbles (1) Rs 179.76 (2) Rs 170
for a rupee ? (3) Rs 168 (4) Rs 169
(1) 8 (2) 6 (5) None of these
(3) 4 (4) 3 8. A bought 4 bottles of beer and B bought
(5) None of these one bottle of lager, lager per bottle costing
2. In a certain store, the profit is 320% of the twice that of the beer. C bought nothing
cost. If the cost increase by 25% but the but paid Rs 50, for his share of the drink
selling price remains Constant, approxi- which they mixed together and shared
mately what percentage of the selling price equally. If C’s Rs 50 covered his share, then
is the profit ? what is the cost of the lager ?
(1) 30% (2) 70% (1) Rs 50 (2) Rs 75
(3) 100% (4) 250% (3) Rs 30 (4) Rs 46
(5) None of these (5) Noncof these
3. A horse and a carriage together cost Rs
1
8000. If by selling the horse at a profit of 9. Ravi sells an article at a gain of 12 % . if
10% and the carriage at a loss of 10% a 2
total profit of 2.5% is made, then what is he had sold it at Rs 22.50 more, he would
the cost price of the horse ? gained 25% . The cost price of the article is
(1) Rs 3000 (2) Rs 3500 (1) Rs 162 (2) Rs 140
(3) Rs 4000 (4) Rs 5000 (3) Rs 196 (4) Rs 180
(5) None of these (5) None of these
4. A piece of cloth costs Rs 35. If the length of 10. The sale price of an article including the
the piece would have been 4 m longer and sales tax is Rs 616. The rate of sales tax is
each metre costs Rs 1 less, the cost would 10% . If the shopkeeper has made a profit
have remained unchanged. How long is the of 12% , then the cost price of the article is
piece ? (1) Rs 500 (2) Rs 515
(1) 14 m (2) 10 m (3) Rs 550 (4) Rs 600
(3) 12 m (4) 9 m (5) None of these
(5) None of these 11. A reduction of 20% in the price of sugar
5. A house costs C rupees. Later it  was sold 1
for a profit of 25% . What is the capital gains enables a purchaser to obtain 2 kg more
2
tax if it is 50% ofthe profit? for Rs 160. Find the original price per kg of
C C sugar.
(1) (2) (1) Rs 12 (2) Rs 15
24 8
(3) Rs 16 (4) Rs 18
C C (5) None of these
(3) (4)
4 2 12. If the cost of 12 pencils is equal to the
(5) None of these selling price of 10 pencils, the profit per cent
6. If selling price is doubled, the profit triples. in the transactions is
Find the profit per cent. 2
2
(1) 16 % (2) 18%
3
(1) 66 % (2) 100%
3 (3) 20% (4) 25%
(5) None of these
1
(3) 105 % (4) 120% 13. Two motor cars were sold for Rs 9900 each,
3 gaining 10% on one and losing 10% on the
(5) None of these other. The gain or loss per cent in the whole
7. Vishal goes to a shop to buy a radio costing transaction is
Rs 2568. The rate of sales tax is 7% . He (1) neither loss no gain
tells the shopkeeper to reduce the price of (2) 1% gain
the radio to such an extent that he has to
LEARN MATHS FROM S.K. RAJU (9811549822, 9811649822)
11
much does he gain or loss ?
100
(3) % profit (1) Loss Rs 48 (2) Gain Rs 48
99 (3) Loss Rs 132 (4) Gain Rs 132
(4) 1% loss (5) None of these
(5) None of these 21. A video magazine distributor made 3500
14. By selling 33 m of cloth, a shopkeeper gains copies of the March issue of the magazine
the price of 11 m of cloth. His gain per cent at a cost of Rs 350000. He gave 500
is : cassettes free to some key video libraries.
(1) 7% (2) 50% He also allowed a 25% discount on the
(3) 20% (4) 22% market price of the cassette and gave one
(5) None of these extra cassette free with every 29 cassettes
15. On selling a pen at 5% loss and a book at bought at a time. In this manner, he was
15% gain, Karim gains Rs 7. If he sells the able to sell all the 3500 cassettes that were
pen at 5% gain and the book at 10% gain, produced. If the market price of a cassette
the he gains Rs 13. The actual price of the was Rs150, then what is his gain or loss
book is per cent for the March issue of video
(1) Rs 100 (2) Rs 80 magazine ?
(3) Rs 10 (4) Rs 400 (1) 25% loss (2) 10% gain
(5) None of these (3) 40% gain (4) 6.8% loss
16. My friend collects antique stamps. She (5) None of these
purchased two, but found that she needed 22. A firm of ready made garments makes both
to raise money urgently So, she sold them men’s and women’s shirts. Its average profit
for Rs 8000 each. On one she made 20% is 6% of the sales. Its profit in men’s shirts
profit and on the others she lost 20% . How average 8% of the sales and women’s shirts
much did she gain or loss in the entire comprise 60% of the output. The average
transaction ? proflt per sales rupee in women’s shirts is :
(1) No loss/profit (1) 0.0466 (2) 0.0666
(2) Rs 667 loss (3) 0.0166 (4) 0.0669
(3) Rs 667 profit (5) None of these
(4) Rs 658 profit 23. A manufacturer of a certain item can sell
(5) None of these all he can produce at the selling price of Rs
17. A man sells an article at 5% profit. If he 60 each. If costs him Rs 40 in materials
had bought it at 5% less and sold it for Rs and labour to produce each item and he
1 less, he would have gained 10% . The cost has overhead expenses of Rs 3000 per week
price of the article is in order to operate that plant. The number
(1) Rs 200 (2) Rs 150 of units he should produce and sell in order
(3) Rs 250 (4) Rs 240 to make a profit of at least 1000 per week
(5) None of these is —
18. A dairyman pays Rs 6.4 per litre of milk. (1) 250 (2) 300
He adds water and sells the mixture at Rs (3) 400 (4) 200
8 per litre, thereby making 37.5% profit. (5) None of these
The proportion of water to milk received by 24. If the selling price of a product is increased
the customers is by 162, then the business would make a
(1) 1 : 15 (2) 1 : 10 proflt of 17% instead of a loss of 19% . What
(3) 1 : 20 (4) 1 : 20 is the cost price of the product ?
(5) None of these (1) Rs 540 (2) Rs 450
19. The cost price of 20 articles is the same as (3) Rs 360 (4) Rs 600
the selling price of x articles. If the profit is (5) None of these
25% , then the value of x is
(1) 25 (2) 18 15
25. Sita buys a fridge at of its original value
(3) 16 (4) 15 16
(5) None of these and sells it for 10% more than its value.
20. A person had deposited Rs 13200 in a bank Then, gain per cent is
which pays 14% interest. He withdraws the (1) 15.55 (2) 11.67
money and invests in Rs 100 stock at Rs (3) 16.67 (4) None of these
110 which pays a dividend of 15% . How (5) 16.59
LEARN MATHS FROM S.K. RAJU (9811549822, 9811649822)
12
SHORT ANSWERS Cost price of house = Rs C
1. (1) 2. (2) C
3. (4) 4. (2) Profit = 25% of C =
4
5. (2) 6. (2)
7. (3) 8. (1) C C
Capital gain tax = 5% of = Rs
9. (4) 10. (1) 4 8
11. (3) 12. (3) 6. (2) Let CP be x and SP be y.
13. (4) 14. (2) Then, profit = (y - x)
15. (2) 16. (2) If SP = 2y, then profit = 3(y - x)
17. (1) 18. (2) Given 2y - x = 3(y - x)
19. (3) 20. (1)
 y = 2x
21. (4) 22. (1)
 Profit is 100%
23. (4) 24. (2)
7. (3) Let reduced price of the radio be Rs x.
25. (4)
 x + 7% of x = 2568
EXPLANATIONS 7x
x+ = 2568
1. (1) According to the question, 100
SP of 12 marbles = Rs 1, loss = 20%
256800
CP of 12 marbles x=  x = 2400
107
1
= Rs = Rs 1.25  Reduction needed in the price of radio
0.8 = (2568 - 2400) = Rs 168
Now, SP of 12 marbles at a gain of 20% 8. (1) According to the question,
CP × 1.2 = 1.25 × 1.2 = Rs 1.5 Total amount spent by them = Rs 150
It means in order to gain 20% , he should Let the cost of one beer be x, then the cost
sell 12 marbles for Rs. 5 Or for Rs 1, he of one lager will be 2x.
12 Total money spent on beer and larger
should sell = 8 marbles = (4x + 2x)
1.5
2. (5) Let the original cost price be Rs 100. 150
 6x = 150  x = = 25
Then, profit = Rs 320 and SP = Rs 420 6
New CP = Rs 125  The cost of the larger = 25 × 2 = 50
 New profit = Rs 295 9. (4) Let the CP of the article be Rs 100.
Required percentage
1
295 SP atagain of 12 %
2
= × 100 = 70.23%
420 = 100 × 1.125 = Rs 112.50
3. (4) Let the CP of horse be Rs x. SP at a gain of 25% = Rs 125
then CP of carriage would be Rs (8000 -x) If SP is Rs 12.5 more than CP = 100
SP of horse = 1.1x If SP Rs 22.50 more than CP
SP of carriage = 0.9 (8000 - x)
100
 1.1x × 0.9 (8000 - x) = × 22.5 = Rs 180
= 8000 × 1.025 = 8200 12.5
1.1x + 7200 - 0.9x = 8200 10. (1) Let the CP of the article be Rs x
 0.2x = 1000 Then, SP = x × 1.12 × 1.1
Given, x × 1.12 × 1.1 = 616
 x = Rs 5000
4. (2) Let the length of the piece be x m 616
Cost of price = Rs 35 x= = Rs 500
1.232
35 11. (3) According to question purchasing
Then, price per metre = Rs capacity = Rs 160
x
A reduction of 20% means, now a person
 35 
 (x + 4)  x  1 = 35 5
  gets
2
kg for Rs 32 and this is the present
 x = 10 m [from Eq. (i)] price of that commodity.
5. (2) According to the question,
LEARN MATHS FROM S.K. RAJU (9811549822, 9811649822)
13
y = Rs 80
32
 Present price per kg = ×2 16. (5) According to the question,
5 CP of stamp sold at a profit of 20%
= Rs 12.8
Let the originai price be Rs x, then new price 80000 20000
= = = 6666.66
is arrived after reduction 20% of it. 12 3
 x × 0.8 = 12.8  x = Rs 16 CP of stamp sold at a loss cf 20%
12. (3) Let CP of 12 pencils = SP of 10 pencils 8000
= Rs 1 = = 10000
0.8
1 Total SP 6666.66 + 10000 = 16666.66
Then, CP of 1 pencil = Rs
12 Total SP = 2 × 8000 = Rs16000
Loss = 16666.66 - 1600 = Rs 666.66
1
and SP of 1 pencil = Rs 17. (1) Let the CP of the article be Rs 100.
10 Then, SP = Rs 105
1 1 2 1 Now CP = Rs 95
 Profìt = - = = Rs New SP = Rs 104.5
10 12 120 60
If he sells it for Rs 0.5 less than CP
Profit per cent = = Rs 100
1 If he sells it for Rs 1 less than CP 100
60  100 1 100
1 = × 12 × 100 = 20% = × 1 = Rs 200
60 0.5
12 18. (2) Let the quantity of milk purchase be x
13. (4) According to question, where SP is same and quantity of water added be y.
and also gain % and loss % is same there Then, ratio of water to milk be y : x.
is always a loss and such loss % CP = 6.4x
2 2 SP = 8 (x + y)
 (Common gain or loss %   10  Profit per cent = 37.5%
  =  
 10   10   8 (x + y) = 6.4 x × 1.375
= 1% loss 8x + 8y = 8.8x
14. (2) SP of 33 m of cloth  8y = 0.8 x
= CP of 33m + gain
SP of 33 m = CP of 33 m + SP of 11 m x 80 10
 y= =
 SP of 22 m = CP of 33 m = 1 (say) 8 1

1 1  y : x = 1 : 10
 SP of l m = CP of l m 19. (3) According to question,
22 33
CP of 20 articles = SP of x articles = 1
1 1
Gain = - = 11 1
22 33 22  33  CP = of 1 articles = 20 ,

11 1
22  33  100 SP of 1 articles =
x
 Gain per cent = 1
33 1 1

x 20 25
11 Profit per cent = 1 = 100
=  33  100 = 50%
22  33 20
15. (2) Let the CP of the pen and book be Rs x
and Rs y respectively. 20  x 1
 =
 0.95x + 1.15y = (x + y) + 7 x 4
 0.15 y - 0.05 x = 7  80 - 4x = x
and 1.05x + 1.1y = (x + y) + 13  5x = 80
 0.05x + 0.1y = 13  x = 16
Solving Eqs. (i) and (ii), we get
LEARN MATHS FROM S.K. RAJU (9811549822, 9811649822)
14
20. (1) According to the question, 23. (4) Required number of items
Interest earned on Rs 13200 @ 14%
= Rs 1848
3000  1000  4000
 Number of shares purchased
=
 60  40  =
20
= 200

13200 24. (2) According to question,


= = 120 (17 + 19) = 36% of the cost price = Rs 162
110
100% of the cost price
Dividend earned by him on 120 shares Rs
15 per share = Rs 1800 162
= ×100 = Rs 450
 Net loss = Rs 48 36
21. (5) According to the question, 25. (4) Let the original value of fridge be Rs = x.
CP of 3500 cassettes = Rs 350000
The SP of one cassette after discount is Rs 15
Then, Cost price = x
112.50 16
SP of each set of 30 cassettes = (29 +1)
110
= Rs 29 × 112.50 = Rs 3262.50 Selling price = × x
100
 SP of 3500 cassettes including 500 free
cassettes = Rs 326250 110 15
 Overall loss on Rs 350000 is Rs 23750 x x
100 16  100
23750  Gain per cent = 15
 Loss percent = × 100 = 6.78%
350000 16
22. (1) According to the question, = 17.33%
Women’s shirts comprise 60% of the Hence, the answer is none of these.
output.
 Men’s shirts comprise 40% of the output.
 Average profit from men’s shirts
= 8% of 40 = 3.2 out of 40
Overall average profit = 6 out of 100
Average profit from women’s shirts = 2.8
out of 60,
i.e., 0.0466 out of each shirt

LEARN MATHS FROM S.K. RAJU (9811549822, 9811649822)


1
RATIO AND PROPORTION
NATIONALISED BANKS (1) 75 (2) 65
& IBPS PO/MT/SO (3) 70 (4) 60
1. Seats for Maths, Physics and Biology are (5) None of these
in the ratio of 5 : 7 : 8 respectively. There is (Bank Of Baroda Specialist
a proposal to increase these seats by 40% , Officer Exam. 05.10.2008)
50% and 75% respectively. What will be 6. A sum of money is divided among A, B, C
the respective ratio of increased seats ? an d D in the ratio o f 3 : 4 : 9 : 10
(1) 2 : 3 : 4 respectively. If the share of C is Rs. 2,580
(2) 6 : 7 : 8 more than the share of B, then what is the
(3) 6 : 8 : 9 total amount of money of A and D together ?
(4) Cannot be determined (1) Rs. 5,676 (2) Rs. 6,192
(5) None of these (3) Rs. 6,708 (4) Rs. 7,224
Canara Bank PO Exam. 09.02.2003) (5) None of these
2. Samira, Mahira and Kiara rented a set of (Canara Bank PO Exam. 15.03.2009)
DVDs at a rent of Rs. 578. If they used it 7. Production of company A is 120% of the
for 8 hours, 12 hours and 14 hours production of company B and 80% of the
respectively, what is Kiara’s share of rent production of company C. What is the ratio
to be paid ? between the productions of companies A,
(1) Rs. 238 (2) Rs. 204 B and C respectively ?
(3) Rs. 192 (4) Rs. 215 (1) 6 : 5 : 9 (2) 6 : 5 : 4
(5) None of these (3) 12 : 10 : 15 (4) 10 : 12 : 15
(Bank Of Maharashtra (5) None of these
PO Exam. 25.05.2008) (PNB Specialist Officer’s
3. A sum of money is to be divided among four Exam. 16.08.2009)
persons in the ratio of 2 : 3 : 4 : 5. Out of 8. Number of students in Arts and Science
the four, one person gets Rs. 200 more than faculties in an institute are in the ratio of 5
the other and Rs. 100 less than another. : 8 respectively. If 150 more students join
What is the sum ? Art faculty while 80 more students join
(1) Rs. 2800 Sci ence facul ty, th e respecti ve ratio
(2) Rs. 1400 becomes 3 : 4. Originally what was the total
(3) Rs. 4200 number of students in both faculties
(4) Cannot be determined together ?
(5) None of these (1) 1200
(Indian Overseas Bank (2) 1400
PO Exam. 15.06.2008) (3) 1150
4. In a college the number of students (4) Cannot be determined
studying Arts, Commerce and Science are (5) None of these
in the ratio of 3 : 5 : 8 respectively. If the (PNB Specialist Officer’s
num- ber of students studying Arts, Exam. 16.08.2009)
Commerce and Science is increased by 5
20% , 40% and 25% respectively, what will 9. 75% of a number is equal to th of another
8
be the new ratio of students in Arts,
number. What is the ratio between the first
Commerce and Science respectively ?
nu mber and the secon d nu mber
(1) 18 : 35 : 50 (2) 3 : 10 : 10
respectively ?
(3) 4 : 8 : 5 (4) 32 : 35 : 25
(1) 5 : 4 (2) 6 : 5
(5) None of these
(3) 4 : 5 (4) 5 : 6
(Indian Overseas Bank
(5) None of these
PO Exam. 15.06.2008)
(PNB Specialist Officer’s
5. 20 boys and 25 girls form a group of social
Exam. 16.08.2009)
workers. During their membership drive,
10. In a test, a candidate secured 336 marks
the same number of boys and girls joined
out of maximum marks ‘x’. If the maximum
the group (e.g. if 7 boys joined, 7 girls
marks ‘x’ were converted into 400 marks,
joined). How many members does the group
he would have secured 192 marks. What
have now, if the ratio of boys to girls is 7 : 8 ?
LEARN MATHS FROM S.K. RAJU (9811549822,were the 9811649822)
maximum marks of the test ?
2
(1) 700 (2) 750 quadrilateral is equal to the smallest angle
(3) 500 (4) 650 of a triangle. One of the angles of the
(5) 800 triangle is twice the smallest angle of the
(Corporation Bank PO triangle. What is the second largest angle
Exam. 22.11.2009) of the triangle ?
11. Which of the following represents ah = 64 ? (1) 80°
(1) 8 : a = 8 : b (2) a : 16 = b : 4 (2) 60°
(3) a : 8 = b : 8 (4) 32 : a = b : 2 (3) 120°
(5) None of these (4) Cannot be determined
(Indian Bank Rural Marketing (5) None of these
Officer Exam. 03.01.2010) (Central Bank Of India
12. The rati o of the n umbe r of stude nts PO Exam. 25.07.2010)
studying in schools A, B and C is 5 : 8 : 4 17.
Th e rati o be tw een the ang le s of a
respectively. If the number of students quadrilateral is 3 : 4 : 6 : 7. Half the second
studying in each of the schools is increased largest angle of the quadrilateral is equal
by 20% , 25% and 30% respectively, what to the smaller angle of a parallelogram.
will be the new respective ratio of the What is the value of adjacent angle of the
students in schools A, B and C ? parallelogram ?
(1) 13 : 25 : 15 (1) 136° (2) 126°
(2) 20 : 25 : 13 (3) 94° (4) 96°
(3) 15 : 25 : 13 (5) None of these
(4) Cannot be determined (Syndicate Bank PO Exam. 29.08.2010)
(5) None of these 18.
The ratio between the three angles of a
(Indian Bank Rural Marketing quadrilateral is 1 : 4 : 5 respectively. The
Officer Exam. 03.01.2010) valu e of the fou rth an gl e of the
13. When 30% of one number is subtracted quadrilateral is 60°. What is the difference
from another number, the second number between the value of the largest and the
reduces to its own four-fifth. What is the smallest angles of the quadrilateral ?
ratio between the first and the second (1) 120° (2) 90°
numbers respectively ? (3) 110° (4) 100°
(1) 4 : 7 (5) None of these
(2) 3 : 2 (Bank Of India PO Exam. 31.10.2010)
(3) 2 : 5 19.
Mr. Pandit owned 950 gold coins all of
(4) Cannot be determined which he distributed amongst his three
(5) None of these daughters Lalita, Amita and Neeta. Lalita
(Allahabad Bank PO Exam. 21.02.2010) gave 25 gold coins to her husband, Amita
14. The largest and the second largest angles donated 15 gold coins and Neeta made
of a triangle are in the ratio of 3 : 2 jewellery out of 30 gold coins. The new
respectively. The smallest angle is 20% of respective ratio of the coins left with them
the sum of the largest and the second was 20 : 73 : 83. How many gold coins did
largest angles. What is the sum of the Amita receive from Mr. Pandit ?
smallest and the second largest angles ? (1) 380 (2) 415
(1) 80° (2) 60° (3) 400 (4) 350
(3) 100% (4) 90° (5) None of these
(5) None of these (PNB Management Trainee
(Bank Of Baroda PO Exam. 30.05.2010) Exam. 28.11.2010)
15. Th e rati o be tw een the ang le s of a 20. The largest and the second largest angles
quadrilateral is 7 : 2 : 5 : 6 respectively. What
of a triangle are in the ratio of 13 : 12
is the sum of double the smallest angle and respectively. The smallest angle is 20% of
half the largest angle of the quadrilateral ? the sum of the largest and the second
(1) 162° (2) 198° largest angles. What is the sum of the
(3) 99° (4) 135° smallest and the second largest angles ?
(5) None of these (1) 120° (2) 108°
(Bank Of Baroda PO Exam. 30.05.2010) (3) 100° (4) 102°
16. The angles of a quadrilateral are in the ratio(5) None of these
of 2 : 4 : 7 : 5. The smallest angle of the
LEARN MATHS FROM S.K. RAJU (9811549822, 9811649822)
3
(Bank Of Maharashtra to the smallest angle of a quadrilateral.
Exam. 19.12.2010) Largest angle of quadrilateral is four times
21. Twenty five percent of Pranab’s annual its smallest angle. What is the sum of
salary is equal to eighty percent of Surya’s largest angle of quadrilateral and the
annual salary. Surya’s monthly salary is smaller angle of parallelogram
forty percent of Dheeru’s monthly salary. If (1) 252° (2) 226°
Dhreeu’s annual salary is Rs 6 lacs, what (3) 144° (4) 180°
is Pranab’s monthly salary ? (At some places (5) None of these
annual income and in some place monthly (Union Bank Of India PO
income are given) Exam. 09.01.2001)
(1) Rs 7.68 lacs (2) Rs 56,000 26. One of the angles of a triangle is two-third
(3) Rs 8.4 lacs (4) Rs 64,000 of sum of adjacent angles of parallelogram.
(5) None of these Remaining angles of the triangle are in ratio
(Bank Of Maharashtras 5 : 7 respectively. What is the value of
Exam. 19.12.2010) second largest angle of the triangle ?
22. The ratio between the three angles of a (1) 25°
quadrilateral is 1 : 6 : 2 respectively. The (2) 40°
value of the fourth angle of the quadrilateral (3) 35°
is 45°. What is the difference between the (4) Cannot be determined
value of the largest and the smallest angles (5) None of these
of the quadrilateral ? (Corporation Bank PO
(1) 165° (2) 140° Exam. 16.O1.2011)
(3) 175° (4) 150° 27. The largest and the smallest angles of a
(5) None of these triangle are in the ratio of 3 : 1 respectively.
(Bank Of Maharashtra The second largest angle of the triangle is
Exam. 19.12.2010) equal to 44°. What is the value of 150 per
23. Th e rati o betwe en the an gl es of a cent of the largest angle of the triangle ?
quadrilateral is 3 : 4 : 6 : 5. Two-third of the (1) 149 (2) 129
largest angle of the quadrilateral is equal (3) 153 (4) 173
to the smaller angle of a parallelogram. (5) None of these
What is the value of adjacent angle of the (UCO Bank PO Exam. 30.01.2011)
parallelogram ? 28. One of the angles of a quadrilateral is thrice
(1) 120° (2) 110° the smaller angle of a parallelogram. The
(3) 100° (4) 130° respective ratio between the adjacent
(5) None of these angles of the parallelogram is 4 : 5.
(Oriental Bank Of Commerce Remaining three angles of the quadrilateral
P0 Exam. 26.12.2010 (1st Sitting) are in ratio 4 : 11 : 9 respectively. What is
24. Rohit has some 50 paisa coins, some 2 the sum of the largest and the smallest
rupee coins, some 1 rupee coins and some angles of the quadrilateral ?
5 rupee coins. The value of all the coins is (1) 255° (2) 260°
Rs 50. Number of 2 rupee coins is 5 more (3) 265° (4) 270°
than that of the 5 rupee coins. 50 paisa (5) None of these
coins are double in number than 1 rupee (UCO Bank PO Exam. 30.01.2011)
coins Value of 50 paisa coins and 1 rupee 29. Smallest angle of a triangle is equal to two-
coins is Rs 26. How many 2 rupee coins third of the smallest angle of a quadrilateral.
does he have ? Th e ratio betw een the ang les of the
(1) 4 quadrilateral is 3 : 4 : 5 : 6. Largest angle of
(2) 2 the triangle is twice its smallest angle. What
(3) 7 is the sum of second largest angle of the
(4) Cannot be determined triang le and l arg est an gl e of the
(5) None of these quadrilateral ?
(Union Bank Of India PO (1) 160° (2) 180°
Exam. 09.01.2001) (3) 190° (4) 170°
25. The ratio between the adjacent angles of a (5) None of these
parallelogram is 2 : 3 respectively. Half the (Bank Of Baroda PO Exam. 13.03.2011)
smaller angle of the parallelogram is equal
LEARN MATHS FROM S.K. RAJU (9811549822, 9811649822)
4
30. The largest and the second largest angles (4) Cannot be detremined
of a triangle are in the ratio of 4 : 3 (5) None of these
respectively. The smallest angle is half the (UCO Bank PO Exam. 22.03.2009)
largest angle. What is the difference between 36. Ratio of Rani’s and Komal’s age is 3 : 5
the smallest and the largest angles of the respectively. Ratio of Komal’s and Pooja’s
triangle ? age is 2 : 3 respectively. If Rani is two-fifth
(1) 30° (2) 60° of Pooja’s age, what is Rani’s age ?
(3) 40° (4) 20° (1) 10 years
(5) None of these (2) 15 years
(Allahabad Bank PO Exam. 17.04.2011) (3) 24 years
31. The ratio between the three angles of a (4) Cannot be determined
quadrilateral is 13 : 9 : 5 respectively. The (5) None of these
value of the fourth angle of the quadrilateral (Indian Overseas Bank
is 36°. What is the difference between the PO Exam. 05.04.2009)
largest and the second smallest angles of 37. Present ages of Amit and his father are in
the quadrilateral ? the ratio of 2 : 5 respectively. Four years
(1) 104° (2) 108° hence the ratio of their ages becomes 5 :
(3) 72° (4) 96° 11 respectively. What was father’s age five
(5) None of these years ago ?
(Allahabad Bank PO Exam. 17.04.2011) (1) 40 years (2) 45 years
32. The ratio between the adjacent angles of a (3) 30 years (4) 35 years
parallelogram is 7 : 8 respectively. Also the (5) None of these
ratio between the angles of quadrilateral is (Andhra Bank PO Exam. 05.07.2009)
5 : 6 : 7 : 12. What is the sum of the smaller
3
angle of parallelogram and second largest 38. Four years ago Shyam’s age was times
angle of the quadrilateral ? 4
(1) 168° (2) 228° that of Ram. Four years hence, Shyam’s
(3) 156° (4) 224° 5
(5) None of these age will be times that of Ram. What is
6
(Indian Overseas Bank
the present age of Shyam ?
PO Exam. 22.05.2011)
(1) 15 years (2) 20 years
33. The ages of Sulekha and Arunima are in the
(3) 16 years (4) 24 years
ratio of 9 : 8 respectively. After 5 years the
(5) 8 years
ratio of their ages will be 10 : 9. What is the
(Corporation Bank PO
difference in years between their ages.
Exam. 22.11.2009)
(1) 4 years (2) 5 years
39. The ratio of the ages of Tina and Rakesh is
(3) 6 years (4) 7 years
9 : 10 respectively. Ten years ago the ratio
(5) None of these
of their ages was 4 : 5 respectively. What is
(Andhra Bank PO Exam. 14.09.2008)
the present age of Rakesh ?
34. The ages of Sonal and Nitya are in the ratio
(1) 25 years (2) 20 years
of 9 : 5 respectively. After 8 years the ratio
(3) 30 years (4) 24 years
of their ages will be 13 : 9. What is the
(5) None of these
difference in years between their ages ?
(Indian Bank PO Exam. 17.10.2010)
(1) 4 years (2) 12 years
40. The present ages of Vishal and Shekhar are
(3) 6 years (4) 14 years
in the ratio of 14 : 17 respectively. Six years
(5) None of these
from now, thier ages will be in the ratio of
(Oriental Bank of Commerce
17 : 20 respectively. What is Shekhar’s
PO Exam. 21.12.2008)
present age ?
35. The ratio of the ages of a father and son is
(1) 17 years (2) 51 years
17 : 7 respectively. 6 years ago the ratio of
(3) 34 years (4) 28 years
their ages was 3 : 1 respectively. What is
(5) None of these
the father’s present age ?
(Bank Of India Banking
(1) 64
Officer Exam. 24.01.2010)
(2) 51
41. The ratio between the ages of a father and
(3) 48
a son at present is 5 : 2 respectively. Four
LEARN MATHS FROM S.K. RAJU (9811549822, 9811649822)
5
years hence the ratio between the ages of 46. The respective ratio between the present
the son and his mother will be 1 : 2 ages of son, mother, father and grandfather
respectively. What is the ratio between the is 2 : 7 : 8 : 12. The average age of son and
present ages of the father and the mother mother is 27 years. What will be mother’s
respectively ? age after 7 years ?
(1) 3 : 4 (1) 40 years (2) 41 years
(2) 5 : 4 (3) 48 years (4) 49 years
(3) 4 : 3 (5) None of these
(4) Cannot be determined (UCO Bank PO Exam. 30.01.2011)
(5) None of these 47. The respective ratio between the present
(Allahabad Bank PO Exam. 21.02.2010) ages of Ram, Rohan and Raj is 3 : 4 : 5. If
42. Radha’s present age is three years less than the average of their present ages is 28 years
twice her age 12 years ago. Also the then what would be the sum of the ages of
respective ratio between Raj’s present age Ram and Rohan together after 5 years ?
and Radha’s present age is 4 : 9. What will (1) 45 years (2) 55 years
be Raj’s age after 5 years ? (3) 52 years (4) 59 years
(1) 12 years (5) None of these
(2) 7 years (Bank Of Baroda PO Exam. 13.03.2011)
(3) 21 years 48. The respective ratio between present age
(4) Cannot be determined of Manoj and Wasim is 3 : 11. Wasim is 12
(5) None of these years younger than Rehana. Rehana’s age
(Punjab & Sind Bank after 7 years will be 85 years. What is the
PO Exam. 16.05.2010) present age of Manoj’s father who is 25
43. The ratio of the present ages of Meena and years older than Manoj ?
Fiona is 16 : 13 respectively. Four years (1) 43 years (2) 67 years
ago the respective ratio of their ages was (3) 45 years (4) 69 years
14 : 11. What will be Fiona’s age four years (5) None of these
from now ? (Indian Overseas Bank
(1) 28 years (2) 32 years PO Exam. 22.05.2011)
(3) 26 years (4) 36 years 49. The respective ratio between the present
(5) None of these age of Aarti and Savita is 5 : x. Aarti is 9
(United Bank Of India years younger than Jahnavi. Jahnavi’s age
PO Exam. 14.11.2010) after 9 years will be 33 years. The difference
44. The respective ratio of the present ages of between Savita’s and Aarti’s age is same as
Swati and Trupti is 4 : 5. Six years hence the presen age of Jahnavi. What will come
the respective ratio of their ages will be 6 : 7. in place of x ?
What is the difference between their ages ? (1) 21
(1) 2 years (2) 37
(2) 3 years (3) 17
(3) 4 years (4) Cannot be determined
(4) Cannot be determined (5) None of these
(5) None of these (IBPS Bank PO/MT CWI
(PNB Management Trainee Exam.18.09.2011)
Exam. 28.11.2010) 50. An amount of money is to be divided among
45. The respective ratio between the present P, Q and R in in the ratio of 3 : 5 : 7
ages of Ram and Rakesh is 6 : 11. Four respectively. If the amount received by R is
years ago the ratio of their ages was 1 : 2 Rs 4,000 more than the amount received
respectively. What will be Rakesh’s age after by Q, what will be the be total amount
five years ? received by P and Q together ?
(1) 45 years (1) Rs. 8,000
(2) 29 years (2) Rs. 12,000
(3) 49 years (3) As. 16,000
(4) Cannot be determined (4) Cannot be determined
(5) None of these (5) None of these
(Corporation Bank PO (Allahabad Bank PO Exam. 21.02.2010)
Exam. 16.01.2011)
LEARN MATHS FROM S.K. RAJU (9811549822, 9811649822)
6
51. Rita invested 25% more than Sunil. Sunil Online Exam. 01.09.2013)
invested 30% less than Abhinav who 57. In two vessels A and B, there is mixture of
invested Rs 6,000. What is the respective milk and water. The ratio of milk and water
ratio between the amount that Rita invested in these vesse ls is 5 : 2 an d 8 : 5
and the total amount invested by all of them respectively. In what ratio these mixtures
together ? be mixed together so that the ratio of milk
(1) 35 : 104 (2) 13 : 29 and water in the new mixture becomes 9 :
(3) 101 : 36 (4) 35 : 103 4?
(5) None of these (1) 7 : 2 (2) 2 : 7
(Punjab & Sind Bank PO (3) 3 : 5 (4) 5 : 3
Exam. 16.05.2013 (5) 7 : 9
52. When X is subtracted from the numbers 9, (Indian Overseas Bank
15 and 27, the remainders are in continued PO Online Exam. 01.09.2013)
proportion. What is the value of X ?
3
(1) 8 (2) 6 58. If 50% of a certain number is equal to
(3) 4 (4) 5 4
(5) None of these th of another number, what is the ratio
(IBPS Bank PO/MT CWE 17.06.2012) between the numbers ?
53. A certain amount was to be distributed (1) 3 : 2 (2) 2 : 5
among A, B and C in the ratio 2 : 3 : 4 (3) 5 : 2 (4) 3 : 4
re specti ve ly , but w as e rron eo usly (5) 4 : 3
distributed the ratio 7 : 2 : 5 respectively. (Corporation Bank Specialist Officer
As a result of this, B got Rs 40 less. What is (Marketing) Exam. 22.12.2014)
the amount ? 59. The ratio of the present ages of Mahesh and
(1) Rs 210 (2) Rs 270 Ajay is respectively 3 : 2. After 8 years, ratio
(3) Rs 230 (4) Rs 280 of their ages will be 11 : 8. What will be the
(5) None of these present age of Mahesh’s son if his age is
(IBPS Bank PO/MT CWE 17.06.2012) half of the present age of Ajay ?
54. A particular sum was divided among A, B (1) 12 years (2) 24 years
and C in the ratio 2 : 6 : 7 respectively. If (3) 18 years (4) 9 years
the amount received by A was Rs. 4,908, (5) None of these
what was the difference between the (Corporation Bank Specialist
amounts received by B and C ? Officer (Marketing) Exam. 22.12.2014)
(1) Rs. 2,454 (2) Rs. 3,494
(3) Rs. 2,135 (4) Rs. 2,481 SBI PO EXAM
(5) None of these 1. When a number is added to a second
(IBPS RRBs Office Assistant
1
CWE Exam. 09.09.2012) number, the sum is 333 percent of the
55. The average age of a man and his son is 30 3
years. The ratio of their ages four years ago second number. What is the ratio between
was 10 : 3 respectivel y. What i s the the first number to the second number ?
difference between the present ages of the (1) 3 : 7 (2) 7 : 4
man and his son ? (3) 7 : 3 (4) Data inadequate
(1) 28 years (2) 16 years (5) None of these
(3) 26 years (4) 44 years (SBI Banks PO Exam. 20.08.2000)
(5) None of these 2. A sum of money is to be distributed among
(IBPS RRBs Office Assistant CWE P, Q and R in the ratio 6 : 19 : 7. If R gives
Exam. 09.09.2012) Rs. 200 from his share to Q, the ratio of P,
56. A sum of Rs. 221 is divided among X, Y and Q and R becomes 3 : 10 : 3, what is the
Z such that X gets Rs. 52 more than Y. Y total sum ?
gets Rs. 26 more than Z. The ratio of the (1) Rs. 6400 (2) Rs. 12800
shares of X, Y and Z respectively is : (3) Rs. 3200 (4) Data inadequate
(1) 9 : 5 : 3 (2) 9 : 3 : 5 (5) None of these
(3) 5 : 9 : 3 (4) 10 : 6 : 5 (SBI Banks PO Exam. 20.08.2000)
(5) None of these
(Indian Overseas Bank PO
LEARN MATHS FROM S.K. RAJU (9811549822, 9811649822)
7
3. In a school the number of boys and that of (5) None of these
the girls are in the respective ratio of 2 : 3. (SBI PO Exam. 09.01.2008)
If the number of boys is increased by 20% 8. Seats for Maths, Physics and Biology are
and that of girls is increased by 10% , what in the ratio of 5 : 7 : 8 respectively. There is
will be the new ratio of number of boys to a proposal to increase these seats by 40% ,
that of the girls ? 50% and 75% respectively. What will be
(1) 14 : 5 (2) 5 : 8 the respective ratio of increased seats ?
(3) 13 : 4 (4) Data inadequate (1) 2 : 3 : 4
(5) None of these (2) 6 : 7 : 8
(SBI Banks PO Exam. 11.02.2001) (3) 6 : 8 : 9
4. Income of two companies A and B are in (4) Cannot be determined
the ratio of 5 : 8. Had the income of (5) None of these
company ‘A’ been more by Rs. 25 lakhs, the (SBI Associate Banks
ratio of their incomes would have been 5 : PO Exam. 07.01.2007)
4 respectively. What is the income of 9. A sum of money is divided among A, B, C
company ‘B’ ? and D in the ratio of 3 : 5 : 9 : 13 respectively.
(1) Rs. 80 lakhs If the share of C is Rs. 2412 more than the
(2) Rs. 50 lakhs share of A, then what is the total amount of
(3) Rs. 40 lakhs money of B and D together ?
(4) Rs. 60 lakhs (1) Rs. 4422 (2) Rs. 7236
(5) None of these (3) Rs. 6030 (4) Rs. 4824
(SBI Banks PO Exam. 11.02.2001) (5) None of these
5. Ratio of the earnings of A and B is 4 : 7 (SBI PO Preliminary (Tire-I)
respectively. If the earnings of A increase Exam. 27.07.2008)
by 50% and the earnings of B decrease by 10. The ages of Khushi and Jagriti are in the
25% the new ratio of their earnings ratio of 5 : 8 respectively. After 8 years the
becomes 8 : 7 respectively. What are A’s ratio of their ages will be 3 : 4. What is the
earnings ? difference in their ages ?
(1) Rs 26,000 (1) 16 years (2) 8 years
(2) Rs 28, 000 (3) 10 years (4) 12 years
(3) Rs 21,000 (5) None of these
(4) Data inadequate (SBI PO Preliminary (Tire-I)
(5) None of these Exam. 27.07.2008)
(SBI Associate Banks 11. The respective ratio of the present ages of
PO Exam. 21.07.2002) a mother and daughter is 7 : 1. Four years
6. Salaries of A, B and C are in the ratio of 2 : ago the respective ratio of their ages was
3 : 5 respectively. If their salaries were 19 : l. What will be the mother’s age four
in crease d by 15% , 10% and 20% years from now ?
re specti ve ly. What wi ll be the n ew (1) 42 years (2) 38 years
respective ratio of their salaries ? (3) 46 years (4) 36 years
(1) 3 : 3 : 10 (5) None of these
(2) 23 : 33 : 60 (SBI & Rural Business
(3) 10 : 11 : 20 PO Exam. 18.04.2010)
(4) Can’t be determined 12. Mr. X invested a certain amount in Debit
(5) None of these and Equity funds in the ratio of 4 : 5
(SBI Associate Banks PO respectively. At the end of one year, he
Exam. 21.07.2002) earned a tolal dividend of 30% on his
7. Present ages of Seema and Naresh are in investment. After one year he reinvested the
the respective ratio of 5 : 7. Five years hence amount including dividend in the ratio of 6
the ratio of their ages becomes 3 : 4 : 7 in Debit and Equity Funds. If the amount
respectively. What is Naresh’s present age in reinvested in Equity Funds was f 94, 500,
years ? what was the original amount invested in
(1) 25 Equity Funds ?
(2) 40 (1) Rs 75,000 (2) Rs 81,0007
(3) 30 (3) Rs 60,000 (4) Rs 65,0007
(4) Can not be determined
LEARN MATHS FROM S.K. RAJU (9811549822, 9811649822)
8
(5) None of these (1) 40 : 11 (2) 10 : 11
(SBI Associate Banks PO (3) 11 : 10 (4) 11 : 40
Exam. 07.08.2011) (United India Insurance Co. AAO Exam.
21.04.2002
RBI GRADE-B OFFICER EXAMS 2. Ratio of the earnings of A and B is 4 : 7
1. The ages of Samir and Tanuj are in the ratio respectively. If the earnings of A increase
of 8 : 15 years respectively. After 9 years by 50% and the earnings of B decrease by
the ratio of their ages will be 11 : 18. 25% the new ratio of their earnings
What is the difference in years between their becomes 8 : 7 respectively. What are A’s
ages ? earnings ?
(1) 24 years (2) 20 years (1) Rs 26,000
(3) 33 years (4) 21 years (2) Rs 28, 000
(5) None of these (3) Rs 21,000
(RBI Grade-B Officer Exam. 2007) (4) Data inadequate
2. Ratio of earnings of A and B is 8 : 9 (5) None of these
respectively. If the earnings of A increase (LIC Assistant Administrative Officer
by 50% and the earnings of B decrease by (AAO) Exam. 2006
25% , the new ratio of their earnings 3. Salaries of A, B and C are in the ratio of 2 :
becomes 16 : 9 respectively. What are A’s 3 : 5 respectively.If their salaries were
earnings ? in crease d by 15% , 10% and 20%
(1) Rs. 37,000 re specti ve ly. What wi ll be the n ew
(2) Rs. 28,500 respective ratio of their salaries ?
(3) Rs. 22,000 (1) 3 : 3 : 10
(4) Cannot be determined (2) 23 : 33 : 60
(5) None of these (3) 10 : 11 : 20
(RBI Grade-B Officer Exam. 2008) (4) Can’t be determined
3. In a college, the ratio of boys to girls is 31 : (5) None of these
23 respectively. When 75 more girls join the (LIC Assistant Administrative Officer
college, this ratio becomes 124 : 107. How (AAO) Exam. 2006
many more girls should join the college to 4. The present ages of A, B and C are in the
make the number of boys and girls equal ? ratio of 8 : 14 : 22 respectively. The present
(1) 75 (2) 90 ages of B, C and D are in the ratio of 21 :
(3) 60 (4) 85 33 : 44 respectively. Which of the following
(5) None of these represents the ratio of the present ages of
(RBI Grade-B Officer Exam.06.02.2011) A, B, C and D respectively ?
4. The ratio of the salaries of A and B is 8 : 9. (1) 12 : 21 : 33 : 44
If A’s salary is increased by 50% and B’s (2) 12 : 22 : 31 : 44
salary is reduced by 25% , their ratio (3) 12 : 21 : 36 : 44
becomes 16 : 9. What is the salary of A ? (4) Cannot be determined
(1) Rs. 22000 (5) None of these
(2) Rs. 28500 (LIC Assistant Administrative
(3) Rs. 37000 Officer Exam. 2008.
(4) Cannot be determined 5. Three-fourth of a number is equal to 60%
(5) None of these of another number. What is the difference
(RBI Officer Grade ‘B’ between the numbers ?
Online Exam. 25.08.2013 (1) 18
(2) 32
INURANCE EXAMS (3) 24
l. The number of employees shed through (4) Cannot be determined
voluntary retirement in company A was (5) None of these
23,000 and that in company B was 6325. (LIC Assistant Administrative
What is the ratio of employees retired vol- Officer Exam. 2008)
untarily from company A to that retired from 6. The sum of three numbers is 136. If the
company B ? ratio between first and second be 2 : 3 and
that between second and third be 5 : 3,
then the second number is:
LEARN MATHS FROM S.K. RAJU (9811549822, 9811649822)
9
(1) 40 (2) 48
5
(3) 52 (4) 60 pieces is of the price of original diamond.
LIC Assistant Administrative 8
Officer (AAO) Exam. 07.06.2009) The masses (in carat) of two pieces are
(1) 3.5 and 2.5
1 (2) 5 and 1
7. The sides of a triangle are in the ratio :
2 (3) 4.5 and 1.5
1 (4) 3.2 and 2.8
1
: and its perimeter is 104 cm. The (Oriental Insurance Company
3 4 AAO Exam. 08.04.2012)
length of the longest side (in cm.) is : 12. A shopeeper placed on display some shirts
(1) 26 (2) 32 each with a marked price. He then posted
(3) 48 (4) 52
LIC Assistant Administrative 1
a board “ off on shirts”. If the cost of a
Officer (AAO) Exam. 07.06.2009) 4
8. Cost of a diamond varies directly as the
2
square of its weight. A diamond broke into shirt was of the price at which it was
four pieces with their weights in the ratio 1 3
: 2 : 3 : 4. If the loss in the total value of the actually sold, the ratio of cost and marked
diamond was Rs. 70,000, the price of the price of shirt was
original diamond was (1) 1 : 2 (2) 1 : 3
(1) Rs. 1,00,000 (3) 2 : 3 (4) 3 : 4
(2) Rs. 1,40,000 (United India Insurance AAO
(3) Rs. 1,50,000 Exam. 03.06.2012)
(4) Rs. 1,75,000 13. A camel pursues an elephant and takes 5
(New India Assurance leaps for every 7 leaps of the elephant, but
AO Exam. 25.10.2009) 5 leaps of elephant are equal to 3 leaps of
9. The angles of a quadrilateral are in ratio of camel. What is the ratio of speeds of camel
3 : 5 : 9 : 7. The second largest angle of the and elephant ?
quadrilateral is equal to the largest angle (1) 21 : 25 (2) 25 : 21
of a triangle. One of the angles of the (3) 23 : 21 (4) 21 : 23
triangle is 25°. What is the value of second (United India Insurance
largest angle of the triangle ? AAO Exam. 03.06.2012)
(1) 60° (2) 50° 14. The sum of the three consecutive odd
(3) 40° (4) 20° numbers is 285. What is the ratio of the
(5) None of these smallest and largest numbers respectively ?
(United India Insurance AO (1) 97 : 95 (2) 93 : 95
Exam. 27.03.2011) (3) 95 : 93 (4) 93 : 97
10. Abhijit invested in three schemes A, B and (5) None of these
C the amounts in the ratio of 2 : 3 : 4 (LIC Assistant Administrative
respectively. If the schemes offered interest Officer (AAO) Exam. 12.05.2013)
@ 20 p.c.p.a., 16 p.c.p.a. and 15 p.c.p.a. 15. The ages of Sonal and Nitya are in the ratio
respectively, what will be the respective of 9 : 5 respectively. After 8 years the ratio
ratio of the amounts after one year ? of their ages will be 13 : 9. What is the
(1) 10 : 8 : 5 difference in years between their ages ?
(2) 12 : 14 : 15 (1) 4 years (2) 12 years
(3) 12 : 15 : 22 (3) 6 years (4) 14 years
(4) Cannot be determined (5) None of these
(5) None of these (United India Insurance
(LIC Assistant Administrative AO Exam. 26.05.2013
Officer Exam. 2008) 16. If x : y = 4 : 5, then
11. The price of a diamond is proportional to (3x + y) : (5x + 3y) =
the square of its mass, which is measured (1) 3 : 5 (2) 5 : 3
in carats. A 6 carat diamond was broken (3) 17 : 35 (4) 35 : 17
into two parts and total price of the two (NICL (GIC) AO Exam.
08.09.2013 (Paper-I)
LEARN MATHS FROM S.K. RAJU (9811549822, 9811649822)
10
17. The ratio between two numbers is 2 : 3. If 23. The average score of a class of boys and
each number is increased by 4, the ratio girls in an examination is A. The ratio of
between them becomes 5 : 7, the difference boys and girls in the class is 3 : 1. If the
between the numbers is average score of the boys is (A + 1), the
(1) 8 (2) 6 average score of the girls is :
(3) 4 (4) 2 (1) (A - 1) (2) (A - 3)
(NICL (GIC) AO Exam. (3) (A + 1) (4) (A + 3)
08.09.2013 (Paper-I) (NICL (GIC) AO (Finance)
18. The ratio between the rates of travelling of Exam. 15.12.2013)
A and B is 2 : 3 and therefore A takes 10 24. Two trains running in opposite directions
minutes more than the time taken by B to cross a man standing on the platform in
reach a destination. If A had walked at 27 seconds and 17 seconds respectively
double the speed, he would have covered and they cross each other in 23 seconds.
the distance in The ratio of their speeds is:
(1) 30 min (2) 25 min (1) 1 : 3 (2) 3 : 2
(3) 20 min (4) 15 min (3) 3 : 4 (4) None of these
(NICL (GIC) Administrative Officer (NICL (GIC) AO (Finance)
Exam. 15.12.2013) Exam. 15.12.2013)
19. The price of sugar is increased by 20% . If 25. If A : B = 1 : 3, B : C = 5 : 7, C : D = 9 : 7,
the expenditure is not allowed to increase, then A : B : C : D = ?
the ratio between the reduction in (1) 15 : 45 : 63 : 49
consumption and the original consumption is (2) 15 : 49 : 45 : 63
(1) 1 : 3 (2) 1 : 4 (3) 45 : 15 : 63 : 49
(3) I : 6 (4) 1 : 5 (4) 49 : 15 : 45 : 63
(NICL (GIC) Administrative (NICL (GIC) AO (Finance)
Officer Exam. 15.12.2013) Exam. 15.12.2013)
20. The difference between the present ages of 26. The cost of making an article is divided
Anil and Sudhir is 6 years. The ratio between materials, labour and overheads
between their ages after 4 years will be 3 : in the ratio of 3 : 4 : 1. If the materials cost
4. What can be the present age of Sudhir ? Rs. 234, then the labour cost is:
(1) 15 years (2) 18 years (1) Rs. 176 (2) Rs. 312
(3) 20 years (4) 24 years (3) Rs. 78 (4) Rs. 390
(NICL (GIC) Administrative (NICL (GIC) AO (Finance)
Officer Exam. 15.12.2013) Exam. 15.12.2013)
21. In a ratio which is equal to 5 : 8, if the
antecedent is 40, then the consequent is:
(1) 25 (2) 64
(3) 48 (4) None of these
(NICL (GIC) Administrative
Officer Exam. 15.12.2013)
22. The rati o of the n umbe r of stude nts
studying in schools A, B and C is 5 : 6 : 8. If
the number of students in each of the
schools is increased by 30% , 25% and 25%
respectively. What will be the new ratio of
the students in schools A, B and C ?
(1) 14 : 15 : 20 (2) 13 : 15 : 20
(3) 13 : 14 : 15 (4) 15 : 17 : 19
(NICL (GIC) AO (Finance)
Exam. 15.12.2013)

LEARN MATHS FROM S.K. RAJU (9811549822, 9811649822)


11
SHORT ANSWERS 15. (5) 16. (3)
NATIONALISED BANKS 17. (1) 18. (4)
19. (3) 20. (3)
& IBPS PO/MT/SO
21. (2) 22. (2)
1. (1) 2. (1)
23. (2) 24. (4)
3. (2) 4. (1)
25. (1) 26. (2)
5. (1) 6. (3)
7. (3) 8. (5)
9. (4) 10. (1) EXPLANATIONS
11. (4) 12. (3) NATIONALISED BANKS
13. (5) 14. (4) & IBPS PO/MT/SO
15. (4) 16. (2) 1. (1) Let the inital seats for Maths Physics
17. (2) 18. (1) and Biology be 5x, 7x and 8x respectively.
19. (1) 20. (4)
5 x  140
21. (4) 22. (3) Now, new seats for Maths =
23. (3) 24. (3) 100
25. (5) 26. (3) 7 x  150
27. (3) 28. (2) For physics =
100
29. (2) 30. (3)
31. (4) 32. (1) 8 x  175
33. (2) 34. (5) and for Biology =
100
35. (2) 36. (4)  Required ratio
37. (4) 38. (3)
39. (2) 40. (3) 5 x  140 7 x  150 8 x  175
= : :
41. (4) 42. (5) 100 100 100
43. (5) 44. (2) = 5 × 140 : 7 × 150 : 8 × 175
45. (3) 46. (4) = 2 :3 :4
47. (4) 48. (1) 2. (1) Ratio of rent’s sharing
49. (5) 50. (3) = 8 : 12 : 14 = 4 : 6 : 7
51. (4) 52. (5) Total rent = Rs. 578
53. (1) 54. (1)
55. (1) 56. (1) 7
Share of Kiara = × 578 = Rs 238
57. (1) 58. (1) 17
59. (1) 60. (1) 3. (2) From the options,
2x + 3x + 4x + 5x = 1400
SBI PO EXAMS  14x = 1400  x = 100
1. (3) 2. (1)  shares are : Rs. 200, Rs. 300.
3. (5) 4. (3) Rs. 400 and Rs. 500.
5. (4) 6. (2) Hence, total sum = 200 + 300 + 400 + 500
7. (5) 8. (1) = Rs. 1400
9. (2) 10. (5) 4. (1) Let the number of students in Arts,
11. (3) 12. (1) Commerce and Science be 3x, 5x and 8x
respectively.
RBI GRADE-B OFFICER EXAMS On increasing their respective numbers,
1. (4) 2. (4) 120 140 125
3. (4) 4. (4) Required ratio = 3x × : 5x × : 8x ×
100 100 100
= 360 : 700 : 1000
INSURANCE EXAMS = 18 : 35 : 50
1. (1) 2. (4) 5. (1) Let x boys and x girls joined the group.
3. (2) 4. (1) According to the question,
5. (4) 6. (4)
20  x 7
7. (3) 8. (1) = =
9. (2) 10. (5) 25  x 8
11. (3) 12. (1)  160 + 8x = 175 + 7x
13. (2) 14. (4)  8x - 7x = 175 - 160
LEARN MATHS FROM S.K. RAJU (9811549822, 9811649822)
12
 x = 15 5  120 8  125 4  130
 New number of members 12. (3) Required ratio =
100
:
100
:
100
= 20 + x + 25 + x = 45 + 2x
= 45 + 2 × 15 = 75 = 5 × 120 : 8 × 125 : 4 × 130
6. (3) Let the amounts received by A, B, C and = 15 : 25 : 13
D be Rs. 3x, 4x, 9x, and Rs. 10x respectively. 13. (5) let the numbers be x and y respectively.
According to the question, According to the question,
9x - 4x = 2580 x  30 4
 5x = 2580 y- = y
100 5
2580 y 3x
x= = 516  =
5 5 10
Total amount of the money of A and D
= 3x + 10x x 10
 = = 2 :3
= 13x = 13 × 516 = Rs. 6708 y 3 5
7. (3) Let the production of company B = 100 14. (4) Tricky approach
units If the largest and the second largest angles
 Production of company A = 120 units be 3x° and 2x°. respectively then, third
Production of company C angle = x
120 100  x + 2x + 3x = 180°
= =150 units  x = 30°
80
 Required ratio = 120 : 100 : 150  Required sum = x + 2x = 3x = 90°
= 12 : 10 : 15 15. (4) Tricky approach
8. (5) Let the original number of students in 7x + 2x + 5x + 6x = 360°
Arts and Science faculties be 5x and 8x  20x = 360°
respectively. 360
According to the question, x= = 18
20
5 x  150 3
= 7x
8 x  80 4  Required answer = 2 × 2x +
2
 24x + 240 = 20x + 600
 4x = 360 15x 15 18
= = = 135o
2 2
360
x = = 90 16. (2) 2x + 4x + 7x + 5x = 360°
4  18x = 360°
 Original number of students
= 5x + 8x = 13x 360
=x= =20o
= 13 × 90 = 1170 18
9. (4) Let the number be x and y respectively.  Smallest angle of the triangle
75 x 5 y = 2 × 20° = 40°
 = Second angle = 2 × 40° = 80°
100 8
 Required angle = 180° - 80° - 40°
x 5 100 5 = 60°
 y= × = 17. (2) 3x + 4x + 6x + 7x = 360°
8 75 6
 20x = 360°  x = 18°
10. (1) x : 336 = 400 : 192
 x × 192 = 336 × 400 6x
 smaller angle of the parallelogram =
2
336  400
x = = 700 = 3x = 54°
192
 Adjacent angle of parallelogram
32 b = 180° - 54° = 126°
11. (4) =  ab = 64 18. (1) x + 4x + 5x + 60 = 360°
a 2
 10 x = 300°  x° = 30
 32 : a = b : 2
 Required difference = 5x - x = 4x

LEARN MATHS FROM S.K. RAJU (9811549822, 9811649822)


13
= 4 × 30 = 120°  The largest angle ofthe quadrilateral
19. (1) 20x + 73x + 83x = 950 - 25 - 15 - 30 = 6 × 20 = 120°
880  Smaller angle of parallelogram
 176x = 880  x = =5 2
176
= 120 × = 80°
 Number of coins got by Amita = 73x + 15 3
= 73 × 5 + 15 = 380  Its adjacent angle = 180 - 80 = 100°
20. (4) Sum of three angles of a triangle =180° 24. (3) If the number of 2-rupee coins be x, then
Largest angle = 13x° number of 5 rupee coins = x - 5
Second largest angle = 12x°  2x + 5 (x - 5) = 50 - 26
1  2x + 5x - 25 = 24
 Third angle = (13x + 12x) × = 5x°  7x = 24 + 25 = 49
5
 13x + 12x + 5x = 180° 49
x = = 7
 30x = 180° 7
180 25. (5) If the adjacent angles of parallelogram
x= = 6° be 2x° and 3x° respectively, then
30
2x° + 3x° = 180°
 Required sum = 5x + 12x = 17x  5x° = 180°
= 17 × 6 = 102°
 x° = 36°
25 80  Smaller angle of parallelogram
21. (4) Pranab × = Surya ×
100 100 = 2x = 72°
 Smallest angle of the quadrilateral = 36°
Pranab 80 16
 Surya = =  Its largest angle = 4 × 36 = 144°
25 5  Required sum = 144 + 72 = 216°
Pranab : Surya = 16 : 5 26. (3) Su m of adj ace nt ang le s of a
parallelogram = 180°
Surya 40 2
= =  One of the angles of triangle
Dheeru 100 5
2
 Surya : Dheeru = 2 : 5 = × 180° = 120°
3
Pranab : Surya : Dheeru
= 16 × 2 : 5 × 2 : 5 × 5 Sum of three angles of a triangle = 180°
= 32 : 10 : 25  5x + 7x = 180 - 120
Now,  12x = 60  x = 5
 25  600000 Second angle of triangle = 5 × 5 = 25°
Third angle of triangle = 7 × 5 = 35°
600000
 32  × 32  The second largest angle of triangle = 35°
25 27. (3) Sum of angles of a triangle = 180°
= Rs. 768000 = Pranab’ annual income  3x + x + 44 = 180°
 Pranab’s monthly salary  4x = 180 - 44 = 136°
768000 136
= = Rs. 64000 x= = 34°
12 4
22. (3) Sum ofthe angles of quadrilateral = 360°  Largest angle of triangle = 3 × 34 = 102°
 x + 6x + 2x + 45 = 360°
102  150
 9x = 360 - 45 = 315°  150% of 102 = = 153°
100
315 28. (2) For the Parallelogram,
x= = 35°
9 4x° + 5x° = 180°
 Required difference = 6x - x = 5x  9x = 180°
= 5 × 35 = 175°
23. (3) Sum of the angles of a quadrilateral 180
x=
= 20°
= 360° 9
 3x + 4x + 6x + 5x = 360°  Smaller angel of parallelogram
 8x = 360°  x = 20° = 4 × 20 = 80°
 One angle of the quadrilateral
LEARN MATHS FROM S.K. RAJU (9811549822, 9811649822)
14
= 3 × 80 = 240°
9 x  5 10
 4y + 11y + 9y = 360 - 240 = 120° = =
8x  5 9
120
 24y =120° y= = 5°  81x + 45 = 80x + 50
24  81x - 80x = 50 - 45
 Its smallest angle = 4 × 5 = 20° x=5
 Required sum = 240° + 20° = 260°
29. (2) Sum of the angles of quadrilateral = 360°  Required difference = 9x - 8x = x = 5 years
34. (5) Let the present ages of Sonal and Nitya
 3x + 4x + 5x + 6x = 360° be 9x and 5x year respectively
 18x = 360° According to the question,
360 9 x  8 13
x= = 20° = =
18 5x  8 9
 Smallest angle of quadrilateral = 3 × 20  81x + 72 = 65x + 104
= 60°
Largest angle of quadrilateral = 6 × 20 = 120°  81x - 65x = 104 - 72
 16x = 32
2
 Smallest angle of triangle = 60 × = 40° 32
3  x= =2
Largest angle of triangle = 2 × 40 = 80° 16
 Third angle of triangle = 180° - 40° - 80°  Required difference = 9x - 5x
= 60° = 4x = 4 × 2 = 8 years
 Required answer = 60 + 120 = 180° 35. (2) Let the present ages of father and son
30. (3) The smallest angle of triangle is half of be 17x years and 7x years respectively.
the largest angle. According to the question,
Ratio of three angles = 4 : 3 : 2 17 x  6 3
 4x + 3x + 2x = 180 = =
7x  6 1
 9x = 180
 21x - 18 = 17x - 6
 x = 20
 21x - 17x = 18 - 6
 Required difference = 4x - 2x = 2x
= 2 × 20 = 40°  4x = 12
31. (4) Let the three angles of quadrilateral be 12
13x°, 9x° and 5x° respectively. x= =3
4
 13x + 9x + 5x = 360 - 36
 Father’s present age = 17 × 3 = 51 years
324 36. (4) Rani : Komal = 3 : 5 = 6 :10
 27x = 324°  x = = 12° Komal : Pooja = 2 : 3 = 10 : 15
27
 Required difference = 13x - 5x = 8x  Rani : Komal : Pooja = 6 : 10 : 15
= 8 × 12 = 96° We have insufficient data to solve this
32. (1) Let the adjacent angles be 7x° and 8x°. question.
 7x + 8x = 180° 37. (4) Let the present ages of Amit and his
 15x = 180° fathe r be 2x y ears and 5 x ye ars
respectively.
 x = 12°
 Smaller angle = 7 × 12 = 84° 2x  4 5
Again, 5y + 6y + 7y + 12y = 360o  5x  4 =
11
 30y = 360°  25x + 20 = 22x + 44
360 24
 y= = 12°  3x = 24  x = =8
30 3
 Second largest angle of the quadrilaterals  Father’s age 5 years ago = 5x - 5
7 × 12 = 84° = 5 × 8 - 5 = 35 years
 Required sum = 84 + 84 = 168o 38. (3) Four years ago,
33. (2) Let the present ages of Sulekhaa and Shyam : Ram = 3 : 4
Arunima be 9x and 8x years respectively. After four years,
According to the question,
3x  8 5
After 5 years, =
4x  8 6
LEARN MATHS FROM S.K. RAJU (9811549822, 9811649822)
15
 20x + 40 = 18x + 48  176x - 44 = 182x - 56
 2x = 48 - 40 = 8  182x - 176x = 56 - 44
8  6x = 12
 x = = 4 x=2
2
 Fiona’s age after four years = 13x + 4
 Shyam’s present age = 3x + 4
= 3 × 4 + 4 = 16 years = 13 × 2 + 4 = 30 years
39. (2) Let the prestent ages of Tina and Rakesh 44. (2) Let Swati’s present age = 4x years
be 9x and 10x years respectively. Trupti’s present age = 5x years
10 years ago, 4x  6 6
 5x  6 = 7
9 x  10 4
=  30x + 36 = 28x + 42
10 x  10 5
 45x - 50 = 40x - 40  2x = 42 - 36 = 6
 x = 3 = difference of their ages
10 45. (3) Let Ram’s present age be 6x years and
 5x = 10  x = = 2
5 that of Rakesh be 11x years.
 Rakesh’s present age = 10x Four years ago,
= 10 × 2 = 20 years.
6x  4 1
40. (3) Let the present ages of Vishal and =
Shekhar be 14x and 17x years respectively. 11x  4 2
After 6 years,  12x - 8 = 11x - 4
14 x  6 17 x = 8 - 4 = 4
=  Rakesh’s age after five years = 11x + 5
17 x  6 20
= 11 × 4 + 5 = 49 years
 280x + 120 = 289x + 102 46. (4) According to the question,
 9x = 18
2x  7x
 x=2 = 27
2
 Shekhar’s present age = 17x
= 17 × 2 = 34 years  9x = 27 × 2 = 54
41. (4) Let the present ages of father and son 54
be 5x and 2x years respectively. =x= =6
9
After 4 years,
 Mother’s age after 7 years = 7x + 7
Son’s age = y years = 7 × 6 + 7 = 49 years
and mother’s age = 2y years 47. (4) Let the present ages of Ram, Rohan and
y4 Raj be 3x, 4x and 5x years respectively.
 y = 2x + 4  x =  3x + 4x + 5x = 3 × 28
2
 12x = 84
 y  4
 Father’s present age = 5 years 84
2 x= =7
Mother’s present age = (2y - 4) years 12
Clearly, data are inadequate. Sum of the ages of Ram and Rohan after 5
42. (5) Let Radha’s present age = x years. years = 3x + 4x + 10 = 7x + 10
 x = 2(x - 12) - 3 = 7 × 7 + 10 = 59 years
 x = 2x - 24 - 3 48. (1) Rehana’s present age = 85 - 7 = 78 years
Wasim’s present age = 78 - 12 = 66 years
 x = 27
3
4  Manoj’s present age = × 66 = 18 years
 Raj’s present age = 9 × 27 = 12 years 11
 Manoj’s father’s present age = 25 + 18
 Raj’s age after 5 years = 12 + 5 = 17 years = 43 years
43. (5) Let the present ages of Meena and Fiona 49. (5) Jahnavi’s present age = 33 - 9 = 24 years
be 16x and 13x years respectively.
 Aarti’s present age = 24 - 9 = 15 years
According to the question, Now, Aarti : Savita = 5 : x = 15 : 3x
16 x  4 14  Savita’s present age = 3x years
=
13 x  4 11
LEARN MATHS FROM S.K. RAJU (9811549822, 9811649822)
16
 3x - 15 = 24  13x = 60 - 8 = 52
 3x = 24 + 15 = 39 x = 4
39  Required difference = 7x = 7 × 4 = 28 years
x= = 13 56. (1) x = y + 52
3
z = y - 26
50. (3) Let the amount received by P, Q and R
 x + y + z = 221
be Rs. 3x, Rs. 5x and Rs. 7x respectively.
 y + 52 + y + y - 26 = 221
 7x - 5x = 4000
 3y = 221 - 26 = 195
4000
x = = 2000 195 = 65
2 y=
 Amount received by P and Q together 3
= 8x = 8 × 2000 = Rs. 16000  x = 65 + 52 = 117
51. (4) Abhinav’s investment = Rs. 6000  z = 65 - 26 = 39
 x : y : z = 117 : 65 : 39 = 9 : 5 : 3
70  6000 57. (1) Let. CP. of milk per litre be Rs 1
Sunil’s investment = = Rs. 4200
100
5
4200  125
Milk in 1 litre of A = litre
7
Rita’s investment = = Rs. 5250
100
8
 Required ratio Milk in 1 litre of B = litre
13
= 5250 : (6000 + 1200 + 5250)
= 5250 :15450 = 35 : 103 9
Milk in 1 litre of mixture = litre
9  x 15  x 13
52. (5) =
15  x 27  x
 243 - 9x - 27x + x2 = 225 - 30x + x2
 6x = 243 - 225 = 18
x = 3
53. (1) Let the amount be Rs. x.
3 2  1 2
  9  14  x = 40  Required ratio = : =7:2
  13 91
58. (1) First number = x
1 1
  3  7  x = 40 Second number = y
 
50 3
 7 3  x × 100 = y ×
4
  21  x = 40
  x 3
 =y×
40  21 2 4
x = = Rs. 210
4 x 3 3
54. (1) If the total amount be Rs. x, then  y = ×2 =
4 2
2x 59. (1) Mahesh = 3x years
= 4908
15 Ajay = 2x years
After 8 years,
4908  15
x= = Rs. 36810 3 x  8 11
2
=
2x  8 8
7 6
 Required difference = 15 × 36810  24x + 64 = 22x + 88
 2x = 88 - 64 = 24
= Rs. 2454
55. (1) Four years ago,  x = 12
Father’s age = 10x years  Ajay’s age = 2x = 2 × 12 = 24 years
Son’s age = 3x years 1
 10x + 3x + 8 = 60  Age of Mahesh’s son = × 24 = 12 years
2
LEARN MATHS FROM S.K. RAJU (9811549822, 9811649822)

17
SBI PO EXAMS 6. (2) Let the salaries of A, B and C be Rs. 2x,
1. (3) Let the first number be x and the second Rs. 3x and Rs. 5x respectively.
number be y. After respective increase of 15% 10% and
20% their salaries will be
1
x + y = 333
3
% of y 115  2 x 110  3 x 120  5 x
Rs. , and
100 100 100
1000
 x+y= y  Required ratio
3  100
115  2 x 110  3 x 120  5 x
10 = : : = 23 : 33 : 60
100 100 100
 x+y= y
3 7. (5) Let the present ages of Seema and
 3x + 3y = 10y Naresh be 5x and 7x years respectively.
 3x = 7y According to the question,

x 7 5x  5 3
 y= =7 :3 =
3 7x  5 4
2. (1) Let P’s share be 6x  21x + 15 = 20x + 20
Q’s share be 19x and  x = 20 - 15 = 5
R’s share be 7x.  Naresh’s present age = 7x years
Total sum = 6x + 19x + 7x = 32x = 7 × 5 = 35 years
6x : 19x + 200 : 7x - 200 = 3 : 10 : 3 8. (1) Let the inital seats for Maths, Physics
We can write and Biology be 5x, 7x and 8x respectively.
6x = 7x - 200 or, x = 200 5 x  140
 Total sum = 32x = Rs. 6400 Now, new. seats for Maths =
100
3. (5) Ratio of no. boys : girls = 2 : 3
Let the no. of boys = 2x
Then the no. of girls = 3x 7 x  150
No. of boys after 20% increase For Physics =
100
= 1.20 × 2x = 2.4x.
No. of girls after 10% increase 8 x  175
and for Biology =
= 1.10 × 3x = 3.3x 100
2.4 x 8 5 x  140 7 x  150 8 x  175
Required ratio = = = 8 : 11  Required ratio = : :
3.3 x 11 100 100 100
4. (3) A : B = 5 : 8 = 5 × 140 : 7 × 150 : 8 × 175
Let A’s income be 5x and B’s income be 8x. = 2 :3 :4
5 x  25 5 9. (2) Let the original sum be Rs.x.
or, = Sum of the Ratios = 3 + 5 + 9 + 13 = 30
8x 4
9x 3x
5  x  5
5 x5  C’s share = Rs. 30 = Rs. 10
or, = or, =1
4  2x 4 2x
 x + 5 = 2x 3x x
A’s share = Rs. = Rs.
 x=5 30 10
B’s income = 8x = 8 × 5 = Rs. 40 lakhs. According to the question,
5. (4) Let the earnings of A and B be Rs. 4x 3x x
and 7x respectively.  = 2412
10 10
After 50% increase,
A’s earnings = 150% of 4x 2x
After 25% decrease,  = 2412
10
B’s earning = 75% of 7x
 x = 2412 × 5 = Rs. 12060
Ratio = 150% of 4x : 75% of 7x = 8 : 7
 Amount received by B and D together
But their total earnings are unknown.
Hence A’s earnings can’t be known.   5  13 
Rs  30 12060  = Rs. 7236
 
LEARN MATHS FROM S.K. RAJU (9811549822, 9811649822)
18
10. (5) Let the present ages of Khushi and Tanuj’s present age = 15x = 45 years
Jagriti be 5x and 8x years respectively.  Required difference = 45 - 24 = 21 years
After 8 years, 2. (4) Let the earnings of A and B be Rs. 8x
and Rs. 9x respectively.
5x  8 3
= Now, after changes in their earnings,
8x  8 4
150
 24x + 24 = 20x + 32 A’s earning = 8x × =Rs. I2x
100
 4x = 32 - 24 = 8
8 75 27 x
x= =2 B’s earning = 9x × = Rs.
4 100 4
 Required difference = (8x - 5x) years Clearly, we need sum of their earnings etc.
= 3x = 3 × 2 = 6 years to get the desired result.
11. (3) Let the ages of the mother and daughter 3. (4) Original number of boys in the college
be 7x and x years respectively. = 31x
Original number of girls = 23x
7 x  4 19
 Four years ago, x  4 = 31x 124
1 = =
23 x  75 107
 19x - 76 = 7x - 4
 12x = 72  x = 6  2852x + 9300 = 3317x
 Mother’s age after four years  3317x - 2852x = 9300
= 7x + 4 = 7 × 6 + 4 = 46 years  465x = 9300
12. (1) Let the original amounts invested in 9300465
Debit and Equity funds be Rs. 4x and 5x x= = 20
465
respectively.  Number of boys = 31 × 20 = 620
Dividend at the end of the year
New number of girls = 23 × 20 + 75 = 535
9 x  30 27 x  Required answer = 620 - 535 = 85
= = Rs 4. (4) A’s salary = Rs. 8x and B’s salary
100 10
Total investment after one year = Rs. 9x

 27 x   117 x  8 x  150% 16
= Rs  9 x   = Rs   9 x  75%
=
9
 10   10 
7 117 x 8 x  150 16
 =
 13 × 10 = 94500 9 x  75 9

94500 13 10 12 x 16


x= = 15000  27 x =
7  117 9
4
 The original amount invested in Equity
funds = 5x = 5 × 15000 = Rs. 75000 48 16
 x= =
27 9
RBI GRADE-B OFFICER EXAMS Hence, A’s salary cannot be determined.
1. (4) Let the present ages of Samir and Tanuja
be 8x and 15x years respectively.
According to the question,
INSURANCE EXAMS
1. (1) Required answer
8 x  9 11
After 9 years, = 23000 40
15 x  9 18 = =
6325 11
 165x + 99 = 144x + 162
2. (4) Let the earnings of A and B be Rs. 4x
 165x - 144x = 162 - 99 and 7x respectively.
 21x = 63 After 50% increase,
63 A’s earnings = 150% of 4x
x= = 3 After 25% decrease,
21
B’s earning = 75% of 7x
 Samir’s present age = 8x = 24 years
LEARN MATHS FROM S.K. RAJU (9811549822, 9811649822)
19
Ratio = 150% of 4x : 75% of 7x = 8 : 7 = x + 2x + 3x + 4x = 10x units
But their total earnings are unknown. Price of the original diamond
Hence A’s earnings can’t be known. = k(100x2) where k is a constant.
3. (2) Let the salaries of A, B and C be Rs. 2x, Cost of four pieces of diamond
Rs. 3x and Rs. 5x respectively. = k (x2 + 4x2 + 9x2 + 16x2)
After respective increase of 15% 10% and = k.30x2
20% their salaries will be Loss in the cost of diamond
= k(100x2 - 30x2) = k.70x2
115  2 x 110  3 x 120  x
Rs , and  k.70x2 = 70000
100 100 100
 kx 2 = Rs. 1000
115  2 x 110  3 x 120  x  Cost of the original diamond
 Required ratio = : : = k (100x2)
100 100 100
= 1000 × 100 = Rs. 100000
= 23 : 33 : 60 9. (2) Sum of ratios
4. (1) A : B : C = 8 : 14 : 22 = 3 + 5 + 9 + 7 = 24
= 12 : 21: 33 Second largest angle of quadrilateral
B : C : D = 12 : 21 : 33 7
= × 360 = 105°
 A : B : C : D = 12 : 21 : 33 : 44 24
5. (4) Let the numbers be x and y respectively. = Largest angle of triangle
Then,  Third angle of triangle = 180° - 105° - 25°
3 = 50°= second largest angle of triangle
x = 60% of y 10. (5) Let Abhijit invested Rs. 2x, Rs. 3x and
4
Rs. 4x in th re e schemes A, B and C
3 60 3 3 respectively.
 x= y  x= y
4 100 4 5  The required ratio
3 5 5 120 116 115
y= × x= x = 2x × : 3x × : 4x ×
4 3 4 100 100 100
= 2 × 120 : 3 × 116 : 4 × 115
5 x = 60 : 87 : 115
 y-x= x-x=
4 4 11. (3) Actual price of diamond = k × 62
Clearly, no unique answer is possible. = 36k where k is constant of proportionality.
6. (4) Let the three numbers be a, b and c New price of diamond
respectively. = kx12 + kx22
Now,
5
a: b = 2 : 3  36k × = k( x12 + x22 )
b :c = 5 :3 8
a : b : c = 2 × 5 : 3 × 5 : 3 × 3 45
= 10 : 15 : 9  = x12 + x22
2
According to the question,
a + b + c = 136  22.5 = x12 + x22
 10x + 15x + 9x = 136
 (4.5)2 + (1.5)2 = x12 + x22
136
 34x = 136  x = =4 12. (1) S.P. of shirt = Rs. x
34
2x
 b = 15x = 15 × 4 = 60 CP. = Rs.
3
1 1 1
7. (3) a : b : c = : : =6:4:3 4 4x
2 3 4
 Marked price = 3 × x = Rs. 3
6
 The longest side = 13 × 104 = 48 cm. 2x 4x
 Required ratio = 3 : 3 = 1 : 2
8. (1) Tricky approach
Let the weights of the four prices of diamond
be x, 2x, 3x and 4x units respectively.
Total weight of the original diamond
LEARN MATHS FROM S.K. RAJU (9811549822, 9811649822)
20
5 7  Time taken by A when he doubles his
13. (2) Required ratio = : 30
3 5
speed = =15 minutes
2
5 7
= × 15 : × 15 = 25 : 21 19. (3) In the beginning.
3 5 The rate of sugar = Rs. 100/kg
14. (4) x + x + 2 + x + 4 = 285 Expenditure = 1 kg.
 3x = 285 - 6 = 279 Now,
Rate = Rs. 120 per kg
279
= = 93 100 5
3
 In Rs. 100, sugar bought = = kg
 Required ratio = 93 : 97 120 6
15. (5) Let the present ages of Sonal and Nitya
 5
be 9x and 5x years respectively.  Required ratio =  1  6  : 1
According to the question,  
9 x  8 13 1
= = = :1=1:6
5x  8 9 6
 81x + 72 = 65x + 104 20. (3) After four years,
 81x - 65x = 104 - 72 Anil = 3x years
 16x = 32 Sudhir = 4x years
In the present,
32 4x - 4 - 3x + 4 = 6
 x= =2
16 x=6
 Required difference = 9x - 5x = 4x  Sudhir’s present age = 4x - 4
= 4 × 2 = 8 years = 4 × 6 - 4 = 20 years
21. (2) Ratio = 5 : 8 = 5 × 8 : 8 × 8 = 40 : 64
x 4
16. (3) y = 22. (2) Number of students :
5 A = 5x
B = 6x
x
3 1 C = 8x
3x  y y
 5x  3 y = x 5 x  130 6 x  125 8 x  125
5 3  Required ratio = : :
y 100 100 100
= 650 : 750 : 1000 = 13 : 15 : 20
3 4 12  5 23. (2) Average marks of girls = x
1
= 5 =
5
=
17 3   A  1  x
4 7 35 A =
5  3 3 1
5
 4A = 3A + 3 + x
17. (1) Numbers = 2x and 3x  x = 4A - 3A - 3 = A - 3
2x  4 5 24. (4) Length of first train = x metre
 3x  4 = 7 Length of second train = y metre
 15x + 20 = 14x + 28  x 
 x = 28 - 20 = 8 Speed of first train =   m/sec
 27 
= Difference between numbers.
18. (4) Time taken by B = x minutes.  y
Speed of second train =   m/sec
Time taken by A = (x + 10) minutes.  17 
2 x  x y
 = Relative Speed =    m/sec
3 x  10
 27 17 
 2x + 20 = 3x
 x = 20 minutes x y
 Time taken by A = 30 minutes.  23 = x y

27 17

LEARN MATHS FROM S.K. RAJU (9811549822, 9811649822)


21
26. (2) Materials : labour : Overheads
x y x y
 + = + = 3 :4 :1
27 17 23 23 Sum of ratios = 3 + 4 +1 = 8
x x y y If the total cost be Rs. x, then
 - = -
23 27 17 23 3x
= 234
27 x  23 x 23 y  17 y 8
 =
23  27 23  17 234  8
x= = Rs. 624
4x 6 y 3
 =
27 17 4
 Labour cost = 8 × 624
x 6 27 81
 y= × = = Rs. 312
17 4 34
25. (1) A : B = 1 : 3 = 15 : 45
B :C =5 :7 = 5 × 7: 7 × 9
= 45 : 63
C : D = 9 : 7 =9 × 7 : 7 × 7 = 63 : 49
 A : B : C : D = 15 : 45 : 63 : 49

LEARN MATHS FROM S.K. RAJU (9811549822, 9811649822)


22
MODEL EXERCISES
1. M The total emoluments of A and B are
x6 x6
equal. However, A gets 65% of his basic (1) (2)
salary as allowances and B gets 80% of his x7 x7
basic salary as allowances. What is the ratio x7 x6
ofthe basic salaries of A and B ? (3) (4)
x6 x7
(1) 16 : 13 (2) 5 : 7
(5) None of these
(3) 12 : 11 (4) 7 : 9
8. If the ratio of boys to girls in a class is B
(5) None of these
and the ratio of girls to boys is G, then 3(B
2. A contractor employed 25 labourers on a
+ G) is
job. He was paid 275 for the work. After
(1) equal to 3 (2) less than 3
retaining 20% of this sum, he distributed
th e re main in g amou nt amo ng st the 1
labourers. If the number of men to women (3) more than 3 (4) less than
3
labourers was in the ratio 2 : 3 and their (5) None of these
wages in the ratio 5 : 4, what wages did a 9. The monthly incomes of two persons are
woman labourer get ? in the ratio of 4 : 5 and their monthly
(1) 10 (2) 8 expenditures are in the ratio of 7 : 9. If each
(3) 12 (4) 15 saves Rs 50 a month, then what are their
(5) None of these monthly incomes ?
3. The dimensions of a rectangular room when (1) Rs 100, Rs 125
increased by 4 m are in the ratio of 4 : 3 (2) Rs 300, Rs 375
and when decreased by 4 m are in the ratio (3) Rs 300, Rs 375
of 2 : 1. The dimensions of the room are (4) Rs 400, Rs 500
(1) 6 m and 4 m (5) None of these
(2) 12 m and 8 m 10. A bag contains Rs 216 in the form of one
(3) 16 m and 12 m rupee, 50 paise and 25 paise coins in the
(4) 24 m and 16 m ratio of 2 : 3 : 4. The number of 50 paise
(5) None of these coins is
4. If 1066 are divided among A, B, C and D (1) 96 (2) 144
such that A : B  = 3 : 4, B : C = 5 : 6 and C : (3) 114 (4) 141
D = 7 : 5, who will get the maximum ? (5) None of these
(1) B (2) A 11. Half the girls and one-third of the boys of a
(3) C (4) D college reside in the hostel. What fractional
(5) None of these part of the student body is hostel dwellers
5. Two numbers are such as the square of one if the total number of girls in the college is
is 224 less than 8 times the square of the
other. If the numbers be in the ratio of 3 : 4 1
100 and is of the total strength ?
their values are 4
(1) 12, 16 (2) 6, 8
2 5
(3) 9, 12 (4) 6, 9 (1) (2)
(5) All of these 5 12
6. If a : b = 2 : 5, then the value of (2a + 2b) : 1
(7a + 5b) is (3) (4) None of these
5
19 19 (5) All of these
(1) (2)
39 13 12. Given that 24 carat gold is pure gold, 18
31 19 3 5
(3) (4) carat gold is gold and 20 carat gold is
19 31 4 6
(5) None of these gold, the ratio of the pure gold in 18 carat
gold to the pure gold in 20 carat gold is
x 2  36 x6 (1) 5 : 8 (2) 9 : 10
7. P= and Q = then the value of
x  49
2
x7 (3) 15 : 24 (4) 8 : 5
(5) None of these
P
is
Q
LEARN MATHS FROM S.K. RAJU (9811549822, 9811649822)


23
(1) 16 (2) 36
y y x x (3) 75 (4) 90
13. If = = , then find x : y : z.
zx z y (5) None of these
(1) 1 : 2 : 3 (2) 3 : 2 : 1 20. A vessel contains liquid P and Q in the ratio
(3) 4 : 2 : 3 (4) 2 : 4 : 7 5 : 3. If 16 L of the mixture are removed
(5) None of these and the same quantity of liquid Q is added
14. The ratio of the rate of flow of water in pipes the ratio become 3 : 5. What quantity does
varies inversely as the square of the radius the vessel hold ?
of the pipes. What is the ratio of the rates (1) 35 L (2) 45 L
of flow in 2 pipes of diameter 2 cm and 4 (3) 40 L (4) 50 L
cm ? (5) None of these
(1) 1 : 2 (2) 2 : 1 21. A bottle is full of dettol. One-third of it is
(3) 1 : 8 (4) 4 : 1 taken out and then an equal amount of
(5) None of these water is poured into the bottle to fill it. This
15. Salaries of A, B and C were in the ratio 3 : operationis done four times. Find the final
5 : 7, respectively. If their salaries were ratio of dettol and water in the bottle.
in crease d by 50% , 60% and 50% (1) 13 : 55 (2) 20 : 74
respectively, what will be the new ratio of (3) 16 : 65 (4) 10 : 48
the their respective salaries ? (5) None of these
(1) 4 : 5 : 7 (2) 3 : 6 : 7 22. Divide Rs 671 among A, B and C such that
(3) 4 : 15 : 18 (4) 9 : 16 : 21 if their shares be increased by Rs 3, Rs 7
(5) None of these and Rs 9 respectively, the remainder shall
16. A sink contains exactly 12 L of water. If be in the ratio 1 : 2 : 3.
water is drained from the sink untill it holds (1) Rs 110, Rs 220 and Rs 336
exactly 6 L of water less than the quantity (2) Rs 112. Rs 223 and Rs 336
drained away, how many litres of water were (3) Rs 105, Rs 223 and Rs 330
drained away ? (4) None of these
(1) 2 L (2) 6 L (5) All of these
(3) 3 L (4) 9 L 23. The ratio of the ages of the father and the
(5) None of these son at present is 7 : 1. After 4 years, the
17. Railway fares of 1st, 2nd and 3rd classes ratio will become 4 : 1. What is the sum of
between two stations we are in the ratio of the present ages of the father and the son ?
8 : 6 : 3. The fares of 1st and 2nd class (1) 29 years (2) 35 years
were subsequently reduced by 1/6 and 1/ (3) 32 years (4) 36 years
12 respectively. If during a year, the ratio (5) None of these
between the passengers of 1st, 2nd and 3rd 24. Th e rati o be tw een the n umbe r of
classes was 9 : 12 : 26 and total amount passengers travelling by I and II classes
collected by the sale of tickets was Rs 1088, between the two railway stations is 1 : 50,
then find the collection from the passengers whereas the ratio of I and II classes fares
of 1st class. between the same stations is 3 : 1. If on a
(1) Rs 260 (2) Rs 280 particular day, Rs 1325 were collected from
(3) Rs 300 (4) Rs 320 the passengers travelling between these
(5) None of these stations, then what was the amount
18. At Narmada S arovar Bach ao ( NS B; collected from the II class passengers ?
demonstration, supporters of Ms Patkar (1) Rs 750 (2) Rs 1000
outnumbered the police by 9 : 1. The police (3) RS 850 (4) Rs 1250
arrested 135 NSB supporters averaging 5 (5) None of these
for even 3 policemen. How many supporters 25. Mira’s expenditure and savings are in the
of NSB were there in the demonstration ? ratio 3 : 2. Her income increases by 10% .
(1) 1215 (2) 665 Her expenditure also increases by 12% . By
(3) 405 (4) None of these how much per cent do her savings increase ?
(5) All of these (1) 7% (2) 9%
19. In a proportion the multiplication of 1st and (3) 10% (4) 13%
4th terms is 10 and that of 2nd and 3rd (5) None of these
terms is 2.5x, then the value of x is

LEARN MATHS FROM S.K. RAJU (9811549822, 9811649822)


24
SHORT ANSWERS B 20  7 140
1. (1) 2. (2) = = ;
C 24  7 168
3. (2) 4. (3)
5. (2) 6. (1) C 7  24 168
7. (1) 8. (3) = =
D 5  24 120
9. (4) 10. (2)
11. (4) 12. (2) A 15  7 105
 = =
13. (3) 14. (4) B 20  7 140
15. (4) 16. (4)
17. (4) 18. (4) A 105 B 140 C 168
Now, = , = , =
19. (1) 20. (3) B 140 C 168 D 120
21. (3) 22. (2)  A : B : C : D = 105 : 140 : 168 : 120
23. (3) 24. (4) Hence, C gets the maximum share.
25. (1) 5. (2) Let the numbers be 3x and 4x. Then,
16x2 = 8×(9x)2 - 224
EXPLANATIONS  16x2 = 72 × x2 - 224
1. (1) Let the basic salary of A be Rs x and  56x2 = 224
that of B be Rs y, then  x2 = 4
65 80 x= 2
A× =B× Hence, numbers are 6, 8.
100 100
6. (1) According to the question,
 A : B = 16 : 13
2. (2) According to the question, 2a
a 2 2 a  3b 3
2 = ; = b
Number of men = × 25 = 10 b 5 7 a  5b 7a
5 5
b
3
Number of women = × 25 = 15 2 19
5 2 3
5 5 19 5 19
Amount distributed among men and women = = =  =
2 39 5 39 39
= 275 × 80% = Rs 220 7  5
Let the wages paid to a man by 5x and to a 5 5
woman be 4x, then 7. (1) According to the question,
10 × 5x + 15 × 4x = 220
 50x + 60x = 220 x 2  36  x  6  x  6 
x 2  49  x  7  x  7 
P= =
x=2
 Wages received by a woman = 2 × 4 = Rs 8 x6
3. (2) According to the question, Q=
x7
L4 4
 =  3L + 12 = 4B + 16
B4 3  x  6  x  6 
 3L - 4B = 4 ...(i) P  x  7  x  7  x6
 Q =  x  6 =
L4 2 x7
and
B4 1
=
 x  7
 L - 4 = 2B - 8 8. (3) Let the number of boys be x and number
 L - 2B = - 4 ...(ii) of girls be y.
Solving Eqs. (i) and (ii), we get
L = 12 m and B = 8 m x y
Then, y = B and = G
4. (3) According to question, x
A 3 5 15  x y
 3(B + G) = 3  y  x 
= = ;
B 45 20
 
B 5  4 20
= = 3 x2  y2 
C 6  4 24 = >3
xy
LEARN MATHS FROM S.K. RAJU (9811549822, 9811649822)
25
9. (4) Let the income of two persons be 4x and 16. (4) Since, Water (W) + Drained (D) =12
5x and their expenses be 7y and 9y.  W + D = 12
Then, 4x - 7y = 50 ...(i) Now, W = D - 6
and 5x - 9y = 50 ...(ii)  D - 6 + D = 12
Solving Eqs. (i) and (ii), we get  D = 9L
x =100 and y = 50 17. (4) New ratio of fares (1st, 2nd and 3rd)
 The income of persons are Rs 400 and Rs
500. 5 11
=8× :6× :3×1
10. (2) By question amount of 50 paise coins 6 12
3 = 80 : 66 : 36 = 40 : 33 : 18
= × 216 = Rs 72 Ratio of passengers = 9 : 12 : 26
4
 Ratio of amount collected
 Number of 50 paise coins = 72 × 2 = 144
(As Rs 1 has 2 fifty paise coins) = 40 × 9 : 12 × 33 : 26 × 18
11. (4) By question total students = 400 = 90 : 99 : 117
No. of girls = 100 Amount collected from 1st class fares
 No. of boys = 300 99
 No. of hostel dwellers = × 1088 = Rs 320
306
1 1 18. (4) The number of police involved in the
= × 100 + × 300 = 150
2 3 3
security operation = × 135 = 3 × 27 = 81
150 15 5
Required fraction = = Thus, the required number of supporters
400 40
12. (2) According to the question = 81 × 9 = 729
19. (1) Acco rding to th e que stio n in a
3 3 proportion, multiplication of 1st and 4th
18 carat gold = pure gold = × 24 = 18
4 4 terms = Multiplication of 2nd and 3rd terms
40 = 2.5 x
5 5
20 carat gold = pure gold = × 24 = 20  x = 16
6 6
20. (3) Let the quantity of liquid P and Q be 5x
 Required ratio = 18 : 20 = 9 : 10 and 3x L respectively.
13. (3) According to the question,
5
y y x Quantity of P removed =
53
× 16 = 10 L
=
xz z
 yz = xy - x 2 - yz - xz ...(i) 3
Quantity of Q removed = × 16 = 6L
53
x y
Also,
y
=
xz 5 x  10 3
Now, =
3 x  6  16 5
 x2 - xz = y2 ...(ii)
Using Eqs. (i) and (ii), we get  25x - 50 = 9x + 30
yz = xy - y z + y2  16x = 80
 2yz = xy + y2 x= 5
 2z = x + y ...(iii)  Quantity that vessel hold = 8 × 5 = 40 L
Only option (3) satisfies the Eq. (iii). 21. (3) Amount of dettol water
14. (4) Radius of pipes are 1 cm and 2 cm. 2
First operation =
1 1 3
 Required ratio = 1  2 
2 : 2 =4 :1
2 2 4
Second operations = × =
15. (4) According to the question, 3 3 9
150 160 150 4 2 8
New ratio = 3 × :5× :7× Third operation = × =
100 100 100 9 3 27
9 21 8 2 16
= : 8 : = 9 : 16 : 21 Fourth operation = × =
2 2 27 3 81
LEARN MATHS FROM S.K. RAJU (9811549822, 9811649822)
26
Amount of water after fourth operation 24. (4) Let the number of passengers travelling
by Class I and Class II be x and 50x
16 65
= 1 = respectively.
81 81 Then amount collected from Class I and
16 65 Cl ass II wi ll be Rs 3x and Rs 50x
 Required ratio = : = 16 : 65 respectively.
81 81
Given, 3x + 50x = 1325
22. (2) Let the shares of A, B and C be (x - 3),
 53x = 1325
(2x - 7) and (3x - 9) respectively.
Then,  x = 25
(x - 3) + (2x - 7 ) + (3x - 9) = 671  Amount collected from Class II
= 50 × 25 = 1250
 6x = 690
25. (1) Let expenditure be Rs 60 and savings
 x = Rs 115
be Rs 40.
A = Rs 112
Total income = Rs 100
B = Rs 223
New income = Rs 110
and C = Rs 336
New expenditure = Rs 67.2
23. (3) Let the present age of father and son be
New saving = 110 - 67.2
7x and x years respectively.
= Rs 42.2
After 4 years,
age of father = (7x + 4) years  Percentage increase in saving
age of son = (x + 4) years 2.8
× 100 = 7%
7x  4 4 40
Given, =
x4 1
 7x + 4 = 4x + 16
 3x =12
 x = 4
 7x + x = 28 + 4 = 32 years

LEARN MATHS FROM S.K. RAJU (9811549822, 9811649822)


1
SIMPLE INTEREST
NATIONALISED BANKS (Indian Bank Rural Marketing Officer
& IBPS SO/MT/SO Exam. 03.01.2010)
1. A certain amount earns simple Interest of 6. Arun invested a sum of money at a certain
Rs. 1,750/- after 7 years. Had the interest rate of simple interest for a period of four
been 2% more, how mueh more interest years. Had he invested the same sum for a
would it have earned ? period of six years the total interest earned
(1) Rs. 35/- by him would have been fifty percent more
(2) Rs. 350/- than the earlier interest amount. What was
(3) Rs. 245/- the rate of interest per cent per annum ?
(4) Cannot be determined (1) 4
(5) None of these (2) 8
(Canara Bank PO Exam. 09.02.2003) (3) 5
2. Vishwas borrowed a total amount of Rs. (4) Cannot be determined
30,000 part of it on simple interest, rate of (5) None of these
12 p.c.p.a. and remaining on simple interest (Allahabad Bank PO Exam. 21.02.2010)
rate of 10 p.c.p.a. If at the end of 2 years he 7. The simple interest accrued on a sum of
paid in all Rs. 36,480 to settle the loan certain principal is Rs 1,200 in four years
amount, what was the amount borrowed at the rate of 8 p.c.p.a. What would be the
at 12 p.c.p.a.? simple interest accrued on thrice of that
(1) Rs. 16000 (2) Rs. 18000 principal at the rate of 6 p.c.p.a. in 3 years ?
(3) Rs. 17500 (4) Rs. 12000 (1) Rs 2,025 (2) Rs 3,025
(5) None of these (3) Rs 2,250 (4) Rs 2,150
(Indian Overseas Bank (5) None of these
PO Exam. 15.06.2008) (Oriental Bank Of Commerce
3. The simple interest accrued on an amount PO Exam. 26.12.2010 (Ist Sitting)
of Rs. 2,500 at the end of six years is Rs. 8. What total amount would Mithilesh get at
1,875. What would be the simple interest the end of three years if he invests an
accrued on an amount of Rs. 6,875 at the amount of Rs 11,200 in a scheme which
same rate and same period ? offers simple interest at the rate of 8.5 p. c.
(1) Rs. 4,556.5 (2) Rs. 5,025.25 p. a. for three years ?
(3) Rs. 4,895.25 (4) Rs. 5,245.5 (1) Rs 14,056 (2) Rs 14,348
(5) None of these (3) Rs 13,852 (4) Rs 15,064
(PNB Agriculture Officer (5) None of these
Exam. 04.01.2009) (Indian Bank PO Exam. 02.01.2011)
4. What amount a man would have received 9. Ravi borrowed some money at the rate of 4
on a principal of Rs. 4,000 after two years p.c.p.a. for the first three years, at the rate
at simple interest at the rate of 5 per cent of 8 p.c.p.a. for the next two years and at
per annum ? the rate of 9 p.c.p.a. for the period beyond
(1) RS 4,161 (2) RS 5,200 5 years. If he pays a total simple interest of
(3) RS 4,400 (4) RS 4,100 Rs 19,550 at the end of 7 years, how much
(5) None of these money did he borrow ?
(Corporation Bank PO Exam. 22.11.2009) (1) Rs 39,500 (2) Rs 42,500
5. Shamita took a loan at simple interest rate (3) Rs 41,900 (4) Rs 43,000
of 6 p.c.p.a in the year and it increased by (5) None of these
1.5 p.c.p.a every year. If she pays Rs. 8,190 (Punjab & Sind Bank
as interest at the end of 3 years, what was PO Exam. 23.01.2011)
her loan amount ? 10. A sum of Rs. 3200 becomes Rs 3456 in two
(1) Rs. 36000 years at a certain rate of simple interest.
(2) Rs. 35400 What is the rate of interest per annum ?
(3) Rs. 36800 (1) 5.5% (2) 6%
(4) Cannot be determined (3) 4% (4) 4.5%
(5) None of these (5) None of these
(Indian Overseas Bank PO
Online Exam. 01.09.2013)
LEARN MATHS FROM S.K. RAJU (9811549822, 9811649822)
2
11. A sum of Rs. 2200 is invested at two (1) Rs. 8,500/- (2) Rs. 15,000/-
different rates of interest. The difference (3) Rs. 7,500/- (4) Rs. 17,000/-
between the interests got after 4 years is (5) None of these
Rs. 202.40. What is the difference between SBI Associate Banks
the rates of interest ? PO Exam. 16.07.2000
(1) 3.3% (2) 2.3% 4. Rs. 800 becomes Rs. 956 in 3 years at
(3) 3.5% (4) 2.5% certain simple rate of interest. If the rate of
(5) None of these interest is increased by 4% , what amount
(Indian Overseas Bank will Rs. 800 become in 3 years ?
PO Online Exam. 01.09.2013) (1) Rs. 1020.8 (2) Rs. 1025
12. A sum of Rs. 16800 is divided into two parts. (3) Rs. 1052 (4) Data inadequate
One part is lent at the simple interest of (5) None of these
6% per annum and the other at 8% per SBI PO Exam. 26.11.2006
annum. After 2 years total sum received is 5. A certain amount earns simple interest of
Rs. 19000. The sum lent at 6% of simple Rs. 1,750/- after 7 years. Had the interest
interest is been 2% more, how much more interest
(1) Rs. 12200 (2) Rs. 12000 would it have earned ?
(3) Rs. 11000 (4) Rs. 10000 (1) Rs. 35/-
(5) None of these (2) Rs. 350/-
(IBPS Bank PO/MT CWE-III (3) Rs. 245/-
26.10.2013) (4) Cannot be determined
13. What will be the difference between the (5) None of these
interest accrued on a sum of Rs. 4500 at SBI Associate Banks PO
12% per annum for 2 years and that on a Exam. 07.01.2007
sum of Rs. 5600 at 9% per annum for 2 8. Veena obtained an amount of Rs. 8,376 as
years ? simple interest on a certain amount at 8
(1) Rs 75 (2) Rs 72 p.c.p.a. after 6 years. What is the amount
(3) Rs 69 (4) Rs 76 invested by Veena ?
(5) None of these (1) Rs.17,180 (2) Rs. 18,110
(Corporation Bank Specialist Officer (3) Rs. 16,660 (4) Rs. 17,450
(Marketing) Exam. 22.12.2014) (5) None of these
SBI PO Preliminary (Tire-I)
SBI PO EXAMS Exam. 27.04.2008
1. Mr. A lends 40% of sum at 15% p.a. 50%
of rest sum at 10% p.a. and the rest at 18% INSURANCE EXAMS
p.a. rate of interest. What would be the rate 1. A sum of money at simple interest amounts
of interest if the interest is calculated on to Rs. 14,160 in 3 years. If the rate of
the whole sum ? interest is increased to 25% , the same sum
(1) 13.4% p.a. (2) 14.33% p.a. amounts to Rs. 14,700 in the same time.
(3) 14.4% p.a. (4) 13.33% p.a. The rate of interest is
(5) None of these
SBI Associate Banks PO 1
(1) 5% (2) 5 %
Exam16.07.2000 2
2. A sum of Rs. 5000/- amounts to Rs. 6,050/ (3) 6% (4) 7%
- in two years.What is the rate of interest ? (New India Insurance
(1) 15% p.a. (2) 13% p.a. AAO Exam. 22.05.2011)
(3) 11% p.a. (4) 21% p.a. 2. Simple interest on a certain sum at a certain
(5) None of these annual rate of interest is 16% of the sum.
SBI Associate Banks If the numbers representing rate percent
PO Exam. 16.07.2000 and time in years be equal, then the rate
3. Srinivasan invests two equal amounts in of interest is
two banks giving 10% and 12% rate of (1) 4% (2) 6%
interest respectively. At the end of year the(3) 4.5% (4) 6.5%
interest earned is Rs. 1650/-. (5) None of these
Find the sum invested in each. (LIC Assistant Administrative
Officer (AAO) Exam. 12.05.2013)
LEARN MATHS FROM S.K. RAJU (9811549822, 9811649822)
3
SHORT ANSWERS 3. (5)  SI on Rs. 2500 for 6 years at a certain
NATIONALISED BANKS rate = Rs. 1875
& IBPS PO/MT/SO  SI on Rs. 6875 for the same period at
1. (4) 2. (4) the same rate
3. (5) 4. (3) 1875  6875
5. (5) 6. (4) =
2500
7. (1) 8. (1) = Rs. 5156.25
9. (2) 10. (3) 4. (3) S.I.
11. (2) 12. (1)
13. (2) Principal × Time × Rate
=
100
SBI PO EXAMS  4000  2  5 
1. (3) 2. (5) = Rs   = Rs 400
3. (3) 4. (3)  100 
5. (4) 6. (4)  Amount received
= Rs. (4000 + 400) = Rs. 4400
INSURANCE EXAMS 5. (5) Let the loan amount be Rs. x
1. (3) 2. (1) x  6  1 x  7.5  1 x  9  1
 8190 = 100 + 100 + 100
EXPLANAT10NS
NATIONALISED BANKS 22.5 x
 = 8190
& IBPS PO/MT/SO 100
1. (4) Let the principal be Rs. x and rate of 8190  100
interest be r% x = = Rs. 36400
22.5
xr 7 6. (4) According to the question,
Case I : = 1750
100 P×R×6 P×R×4 150
= ×
1750  100 100 100 100
 xr = This relation gives no result.
7
= Rs. 25000 SI×100
Case II : 7. (1) Principal =
Time×Rate
x   r  2  7
S.I. = 1200 100
100 = = Rs. 3750
4 8
Which cannot be determined with the help New principal = 3 × 3750
of given information. = Rs. 11250
2. (4) Let the sum borrowed at the rate of 12
p.c.p.a. be Rs. x. Principal × Time × Rate
 S.I =
 Sum borrowed at 10 p.c.p.a. 100
= Rs. (30000 - x)
11250  3  6
Simple interest = = Rs 2025
= Rs. (36480 - 30000) 100
= Rs. 6480 8. (1)
According to the question, Principal × Time × Rate
 S.I =
x  2  12  30000  x   2 10 100
+ = 6480
100 100 11200  3  8.5
24x + 600000 - 20x = 648000 = = Rs. 2856
100
 4x = 648000 - 600000
 Required amount
 4x = 48000 = Rs. (11200 + 2856)
= Rs. 14056
 x = 48000 = 12000 9. (2) If the amount borrowed be Rs. x, then
4
LEARN MATHS FROM S.K. RAJU (9811549822, 9811649822)
4
x  4 3 x 8 2 x 9 2 40
+ + = 19550 x at 15% p.a
100 100 100 100
12 x 16 x 18 x 50 60 x 30 x
 + + = 19550 of = at 10% p.a
100 100 100 100 100 100
 12x + 16x + 18x = 1955000 Rest amount
 46x = 1955000 40 x 30 x 30 x
=x- - = at 18% p.a
1955000 100 100 100
x= Interest eamed by each at end of 1 year
46
= Rs. 42500 15 40 x 60
10. (3) Interest = 3456 - 3200 = Rs. 256 By 1st  × = x
100 100 1000
S.I. × 100
Rate = Principal × Time 10 30 x 30
By 2nd  × = x
100 100 1000
256 100 18 30 x 54
= = 4% per annum By 3rd  x
3200  2 × =
100 100 1000
11. (2) If the rates of interest be r1 and r2% per
annum, then 144
Total interest = x
1000
2200  4  r1  r2 
= 202.40 144 x
100
1000 100 = 14.4%
202.40 100  Rate% =
 r1 - r2 = x
2200  4 2. (5) S.I = 6050 - 5000 = Rs 1050
= 2.3% per annum
12. (1) Sum lent at 6% rate of interest = Rs. x S .I 100 1050  100
R% = = = 10.5%
S.I. = 19000 - 16800 P T 5000  2
= Rs. 2200 3. (3) Let sum be Rs.x
According to the question,
x  6  2 16800  x   8  2
 100
+ = Rs. 2200 x  10  1 x  12  1
100 + = 1650
 12x + 16800 × 16 - 16x = 220000 100 100
 4x = 268800 - 220000 22x = 1650 × 100
 4x = 48800 1650 100
x= = Rs 7500
 x = Rs. 12200 22
13. (2) S.I. 4. (3) Increase is interest in 3 years due to
Principal × Time × Rate increase in rate by 4%
= 800  3  4
100
= = Rs. 96
 Diffrence 100
4500  2 12 5600  2  9 Total amount at the end of 3 years
= - = Rs. 956 + Rs. 96 = Rs. 1052
100 100
5. (4) Let the principal be Rs. x and rate of
= 1080 - 1008 interest be r%
= RS. 72
xr 7
Case I : = 1750
SBI PO EXAMS 100
1. (3) Let the amount be Rs. x 1750 100
Investment is done as given below.  xr =
7
40 60 x = Rs. 25000
Amount left = x - x=
100 100 Case II :

LEARN MATHS FROM S.K. RAJU (9811549822, 9811649822)


5
x   r  2  7  4P + 70800 - 5P = 58800
S.I. =  P = 70800 - 58800 = 12000
100
 From equation (i),
Which cannot be determined with the help
of given information. 12000  r  3
12000 + =14160
6. (4) Let the amount invested by Veena be 100
Rs. x.  120 × 3 × r = 14160 - 12000 = 2160
SI × 100 2160
 Principal = Time× Rate r= = 6%
120  3
2. (1) Let the sum be Rs. x.
8376  100
= = Rs. 17450
68 16 x
S.I. = Rs.
100
INSURANCE EXAMS Rate = Time = R
S.I. ×100
 Rate = Principal × Time
P r 3
1. (3) P + = 14160 .....(i)
100
16 x  100
R=
5r 100 x  R
P+P ×  3 = 14700 ....(ii)
400  R2 = 16
5  R = 4% Per annum
P+ (14160 - P) = 14700
4

LEARN MATHS FROM S.K. RAJU (9811549822, 9811649822)


6
MODEL EXERCISES 1
1. I derive an annual income of Rs 688.25 from 8 % per annum. In this transaction after
2
Rs 10000 invested partly at 8% per annum
a year, Mohit earned a proflt of Rs 350. Find
and partly at 5% per annum si mple
the sum of money lent by Sumit to Mohit.
interest. How much of my money is invested
(1) Rs 10000 (2) Rs 9000
at 5% ?
(3) Rs 10200 (4) Rs 8000
(1) Rs 3725 (2) Rs 4225
(5) None of these
(3) Rs 4800 (4) Rs 5000
7. A computer is available for Rs 39000 cash
(5) None of these
or Rs 17000 as cash down payment followed
2. Two equal sums of money were invested,
by five monthly instalments of Rs 4800
1 each. What is the rate of interest per annum
one at 4% and the other at 4 % . At the
2 under the instalment plant ?
end of 7 yr, the simple interest received from (1) 35.71% (2) 37.71%
the latter exceeded that received from the (3) 36.71% (4) 38.71%
former by Rs 31.50. Each sum was (5) None of these
(1) Rs 1000 (2) Rs 500 8. The ratio of the amount for two years under
(3) Rs 750 (4) Rs 900 CI annually and for one year under SI is 6
(5) None of these : 5. when the ratio of interest is same, then
3. What total amount would Mohan get at the the value of the rate of interest is
end of three years if he invests an amount (1) 12.5% (2) 18%
of Rs11,200 in a scheme which offers simple (3) 20% (4) 16.66%
interest at the rate of 8.5 per cent per (5) None of these
annum for three years ? 9. Consider the following statements If a sum
(1) Rs 14,348 (2) Rs 14,056 of money is lent at simple interest, then the
(3) Rs 13,852 (4) Rs 15,064 (I) money gets doubled in 5 yr, if the rate of
(5) None of these 2
4. Ravi gave Rs 1200 on loan. Some amount interest is 16 %.
3
he gave at 4% per annum simple interest
and remaining at 5% per annum simple (II) money gets doubled in 5 yr, if the rate
interest. After two years, he got Rs 110 as of interest is 20% .
interest. Then, the amounts given at 4% (IlI) money becomes four times in 10 yr, if
and 5% per annum simple interest are, it gets doubled in 5yr.
respectively (1) I and III are correct
(1) Rs 500, Rs 700 (2) II alone is correct
(2) Rs 400, Rs 800 (3) III alone is correct
(3) Rs 800, Rs 300 (4) II and III are correct
(4) Rs 1100, Rs 1100 (5) None of these
(5) None of these 10. Divide Rs 1586 in three parts in such a way
5. Two equal sums of money are lent at the that their amounts at the end of 2, 3 and 4
same time at 8% and 7% per annum simple yr respectively, at 5% per annum simple
interest. The former is recovered 6 months interest be equal
earlier than the latter and the amount in (1) Rs 552, Rs 528, Rs 506
each case is Rs 2560. The sum and the time (2) Rs 560, Rs 520, Rs 506
for which the sums of money are lent out (3) Rs 556, Rs 524, Rs 506
are : (4) Rs 548, Rs 528, Rs 510
(1) Rs 2000, 3.5 year and 4 year (5) None of these
(2) Rs 1500, 3.5 year and 4 year 11. Anu owes Biresh Rs 1120 payable 2 y ear
(3) Rs 2000, 4 year and 5.5 year hence, Biresh owes Anu Rs 1081.50
(4) Rs 3000, 4 year and 4.5 year payable 6 months. If they decide to settle
(5) None of these their accounts forthwith by payment of
6. Sumit lent some money to Mohit at 5% per ready money and the rate of interest be 6%
annum simple interest. Mohit lent the per annum, they who should pay and how
entire amount to Birju on the same day at much ?
(1) Anu, Rs 70 (2) Biresh, Rs 50
(3) Anu, Rs 50 (4) Biresh, Rs 70
LEARN MATHS FROM S.K. RAJU (9811549822, 9811649822)
7
(5) None of these (1) 12% (2) 12.5%
12. The present worth of bill due 7 months, (3) 6.67% (4) 8.33%
hence is Rs 1200. If the bill were due at the (5) None of these
19. If Rs 1000 be invested at interest rate of
1
end of 2 yr, its present worth would be Rs 5% and the interest be added to the
2 principal after 10 year, then the number of
1016. The rate per cent of bill is: years in which it will amount to Rs 2000 is
(1) 16% (2) 8%
(3) 10% (4) 18% 2 1
(1) 16 year (2) 16 year
(5) None of these 3 4
13. The simple interest and the true discount (3) 16 year (4) 11 year
on a certain sum of a given and at a given (5) None of these
rate are Rs 25 and Rs 20 respectively. The 20. A sum of Rs 7700 is to be divided among
sum is three brothers Sunil, Sumant and Surat in
(1) Rs 500 (2) Rs 200 such a way that simple interest on each
(3) Rs 250 (4) Rs 100 part at 5% per annum after 1, 2 and 3 yr
(5) None of these respectively remains equal. The share of
14. A trader owes a merchant Rs 10028 due 1 Sunil is more than that of Surat by :
yr, hence the trader wants to settle the (1) Rs 2800 (2) Rs 2500
account after 3 months. If the rate of (3) Rs 3000 (4) Rs 2700
interest is 12% per annum, how much cash (5) None of these
should he pay ? 21. What will be the ratio of simple interest
(1) Rs 9025 (2) Rs 9200 earned by a certain amount at the same
(3) Rs 9600 (4) Rs 9500 rate of interest for 6 years and 9 years ?
(5) None of these (1) 1:3
15. A person invested some amount at the rate (2) 1 : 4
of 12% simple interest and a certain (3) 2 : 3
amount at the rate of 10% simple interest. (4) Data inadequate
He received yearly interest of Rs 130. But (5) None of these
if he had interchanged th e amounts 22. An automobile flnancier claims to be
invested, he would have received Rs 4 more lending money at simple interest but he
as interest. How much did he invest at 12% includes the interest every six months for
simple interest ? calculating the principal. If he is charging
(1) Rs 700 (2) Rs 500 an interest of 10% , the effective rate of
(3) Rs 800 (4) Rs 400 interest becomes
(5) None of these (1) 10% (2) 10.25%
16. The effective annual rate of interest (3) 10.5% (4) 10.35%
corresponding to a nominal rate of 8% per (5) None of these
annum payable half yearly is 23. Sultan took a loan from the bank at 8%
(1) 8% (2) 8.01% per annum, and was supposed to pay a sum
(3) 8.13% (4) 8.16% of Rs 2240 at the end of 4 years. If the
(5) None of these same sum is cleared off in four equal annual
17. A sum was put at simple interest at a installments at the same rate, the amount
certain rate for 3 yr. Had it been put at 1% of annual installment will be
higher rate, it would have fetched Rs 5100 (1) Rs 500 (2) Rs 550
more. The sum is (3) Rs 600 (4) Rs 1000
(1) Rs 170000 (2) Rs 150000 (5) None of these
(3) Rs 125000 (4) Rs 120000
(5) None of these
18. Prem invested a certain sum of money in a
simple interest bond whose value grew to
Rs 300 at the end of 3 yr and to Rs 400 at
the end of another 5 yr. What was the rate
of interest in which he invested his sum ?

LEARN MATHS FROM S.K. RAJU (9811549822, 9811649822)


8
SHORT ANSWERS 5. (1) Let each amount be Rs x and time be t
1. (1) 2. (4) yr.
3. (2) 4. (1)  1 8
5. (1) 6. (1) Then x +  t   × x ×
7. (4) 8. (3)  2 100
9. (2) 10. (1) xt7
11. (2) 12. (3) =x+
100
13. (4) 14. (2)
15. (2) 16. (4) 2 xt x 7 xt
17. (1) 18. (4)  - =
25 25 100
19. (1) 20. (1)
21. (3) 22. (2) xt x
 =
23. (1) 100 25
 = 4 yr
t
EXPLANATIONS 1
1. (1) Let money invested at 5% be Rs x.  First money was lent for 4 - = 3.5 yr.
2
According to question,
3.5  x  8
x  1 5 10000  x   1 8  Amount = x + = 2560
+ = 688.25 100
100 100
x = Rs 2000
 5x - 8x - 80000 = 68825 6. (1) According to question,
 3x = 11175
1
 x = Rs 3725 8 % - 5% = 350
2. (4) Let each sum be Rs x, then 2

1 350
x4 7  100% = × 100 = Rs 10000
2 x  4 7 3.5
- = 31.50
100 100 7. (4) According to question cash price,
CP = Rs 39000
7 x 1 63 Cash down payment, DP = Rs 17000
  =
100 2 2 Balance due, after lst instalment,
 x = Rs 900 BD = Rs 22000
3. (2) S.I P = value of instalment = Rs 4800
n = no. of instalments = 5
Principal × Time × Rate R = rate of interest
100
 nR    n  1 R 
11200  3  8.5   1  1200  BD = 1  2400  nP
= = Rs 2856    
100
 5R   4R 
 Required Amount = 11200 + 2856 1   22000 = 1   5 × 4800
 1200   2400 
= Rs 14056
4. (1) Let the amount given at 4% per annum 55 R 24 R
be Rs x.  11 + = 12 +
1200 1200
 Amount given at 5% per annum
= Rs (1200 - x) 55 R 24 R 31R
 - =1  =1
1200 1200 1200
x  4  2 1200  x   5  2
 100
+ = 110 1200
100 R= = 38.71%
31
2 x  12000
 = 100
100
 x = Rs 500
Aslo, (1200 - x) = 1200 - 500 = Rs 700

LEARN MATHS FROM S.K. RAJU (9811549822, 9811649822)


9
8. (3) According to question, 11. (2) According to question, Present worth
of money for Anu
2
 r 
P 1   P  6 2
 100  6 1120 - P =  P = Rs 1000
= 100
 Pr  5
P  Present worth of money for Biresh
 100 
P  6 1
1081.50 - P =
 r  6 2  100
= 1  =  r = 20%
 100  5 108150 - 100 P = 3P
9. (2) Let the sum of money lent be Rs P. P = Rs 1050
Then,  Biresh should pay Rs 50.
I. SI received after 5 yr 12. (3) Let the rate of interest be r%
Then,
P  50  5 5
= = P 1200  r  7
3  100 6 A = 1200 +
12  100
Amount after 5 yr
 A = 1200 + 7r
5 11P Again 1200 + 7r =
= P + P =
6 6
1200  r  25
Therefore, I is not correct. 1016 +
II. SI after 5 yr = P + P = 2P 100
Therefore, II is correct.  1200 + 7r = 1016 + 25.4r
III. Amount after 10 yr. at the rate of 20%  18.4r = 184
P  20  10 184
= P +
100
= P + 2P = 3P  r = 18.4 = 10%
Therefore, III is not correct. 13. (4) Let that amount be Rs x and rate is r%
Hence, (2) is the correct option. Then, for simple interest
10. (1) Let the three parts be Rs x,
xr
Rs y, and Rs z. = 25
According to the question, 100
 x × r = 2500
x  2 5 y  3 5 z  45
x + = y+ =z+ For true discount
100 100 100
 1.1x = 1.15y =1.2z  x  20   r
= 20
100
x 1.15 23
 y= = x  r  20 r
1.1 22
 = 20
100
y 1.2 24
and = =  xr - 20r = 2000
z 1.15 23 From Eqs. (i) and (ii), we get
 x : y : z = 276 : 264 : 253 2500 - 20r = 2000
276  r = 25%
x= × l586 = Rs 552, From Eq. (i)
793
x × 25 = 2500
264  x = 100
 y= × 1586 = 528 14. (5) Let the present value of money be x.
793
253 x  12
and z = × 1586 = Rs 506 Then, + x = 10028
793 100
Hence, the required three parts are Rs 552,  0.12 x + x = 10028
Rs 528 and Rs 506. 10028
x =
1.12
This amount after 3 months

LEARN MATHS FROM S.K. RAJU (9811549822, 9811649822)


10
19. (1) According to question,
10028
12  3
112 1000  5  10
= + 10028 SI for 10 years = = Rs 500
12 100 100
Now,P = Rs 1500, A = Rs 2000
10028  3 10028
= +  SI = Rs 500
1.12  100 1.12
1500  5  T
10028 103  500 =
= = Rs 9222.17 100
112
15. (2) Amount invested at 12% = Rs x . 500  100 2
T= = 6 years
Amount invested at 10% = Rs y 1500  5 3
x  12  1 y  10  1 2
130 = +  Total time = 16 3 years
100 100
 13000 = 12x + 10y 20. (1) Let Sunil, Sumant and Surat get x, y
and z amount respectively.
x  10  1 y  12  1
and 134 =
100
+
100 x  5 1 y 5 2 z 53
= =
100 100 100
 13400 = 10x + 12y
Solving Eqs. (i) and (ii), we get x = Rs 500  x = 2y = 3z
So, amount invested at 12% is Rs 500. x: y: z=6:3:2
16. (4) Rate of 8% per annum payable half
6 2
yearly. So, effective rate Effective annual  Required amount × 7700
6  3 2
rate
= Rs 2800
4 4 21. (3) According to question, here, rate of
=4+4 + = 8.16%
100 interest is same and principal is also same.
5100  Required ratio = 6 : 9 = 2 : 3
17. (1) Simple interest for 1 yr = 22. (2) According to question, S.I. on Rs. 100
3 at 10% for six months =
= Rs 1700
1% of sum 1700 100 10  6
100  12
1700 100
 Sum = = Rs. 5
1 Principal for next six months = Rs. 105
= Rs 170000
18. (4) Let principle be P and rate of interest is 105 10  6
 SI = 100  12
r. Then,
Pr3 = Rs. 5.25
+ P = 300  Total interest = Rs. 10.25
100
23. (1) Let the annual payment be Rs. x. Since
P r 8 the first instalment is to be paid after 1 year,
and + P = 400 the sum of amounts of Rs. x for 3 years, of
100
Rs. x for 2 years, of Rs. x for 1 year and Rs.
Substracting Eq. (i) from Eq. (ii), we get
x, gives the amount of debt.
Pr5 Now, amount of Rs. x for three years
100 = 100
 x  8 3  31x 
= Rs  x  = Rs.  
 P × r = 2000  100   25 
From Eq. (i),
Amount of Rs. x for 2 years
2000  3
+ P = 300  P = Rs 240  x  8 2   29 x 
100 = Rs  x   = Rs.  
 100   25 
 240 × r = 2000
 r = 8.33% Amount of Rs. x for 1 year

LEARN MATHS FROM S.K. RAJU (9811549822, 9811649822)


11
 x  8 1  27 x 112 x = 2240
= Rs  x   = Rs. 
 100  25 25
 31x 29 x 27 x  2240  25
  25  25  25  x  = 2240  x = 112
= Rs. 500
 
31x  29 x  27 x  25 x
 = 2240
25

LEARN MATHS FROM S.K. RAJU (9811549822, 9811649822)


1
TIME AND DISTANCE
NATIONALISED BANKS is half that of the boat, what is the speed of
& IBPS PO/MT/SO that boat in kmph ?
1. A motor starts with the speed of 70 kmph (1) 15
with its speed increasing every two hours (2) 5
by 10 kmph. In how many hours will it cover (3) 10
345 kms ? (4) Cannot be determined
1 (5) None of these
(1) 2 hours (Indian Overseas Bank PO
4
Exam. 15.06.2008)
1 6. Samir drove at the speed of 45 kmph. from
(2) 4 hours home to a resort. Returning over the same
2
(3) 4 hours 5 minutes route, he got stuck in traffic and took an
(4) Cannot be determined hour longer, also he could drive only at the
(5) None of these speed of 40 kmph. How many kilometres
(Canara Bank PO Exam. 09.02.2003) did he drive each way ?
2. A train running at the speed of 20 metres/ (1) 250 kms. (2) 300 kms.
second crosses a pole in 24 seconds less (3) 310 kms. (4) 275 kms.
than the time it requires to cross a platform (5) None of these
thrice its length at the same speed. What (Bank Of Baroda Specialist
is the length of the train ? Officer Exam. 05.10.2008)
(1) 270 metres 7. A 240 metres long train crosses a platform
(2) 340 metres twice its lenght in 40 seconds. What is the
(3) 180 metres speed of the train ?
(4) Cannot be determined (1) 6 metres/sec.
(5) None of these (2) 28 metres/sec.
(Union Bank of India PO (3) 18 metres/sec.
Exam. 27.11.2005) (4) 16 metres/sec.
3. A man takes 6 hours 35 minutes in walking (5) None of these
to a certain place and riding back. He would (PNB Agriculture Officer
have taken 2 hours less by riding both Exam. 04.01.2009)
ways. What would be the time he would 8. A boat running at the speed of 34 kmph
take to walk both ways ? downstream covers a distance of 4.8 kms,
(1) 4 hours 35 minutes in 8 minutes. The same boat while running
(2) 8 hours 35 minutes upstream at same speed covers the same
(3) 10 hours distance in 9 minutes. What is the speed
(4) 8 hours 25 minutes of the current ?
(5) None of these (1) 2.4 kmph (2) 3 kmph
(Bank Of Maharashtra PO (3) 2 kmph (4) 3.2 kmph
Exam. 25.05.2008) (5) None of these
4. A train travelling at the speed of 60 kmph (PNB Specialist Officer’s
crosses a platform in 20 seconds. What is Exam. 16.08.2009)
the length of the train ? 9. A bus started its journey from Ramgarh and
(1) 333 metres reached Devgarh in 44 minutes with its
(2) 300 metres average speed of 50 km/hour. If the average
(3) 336 metres speed of the bus is increased by 5 km/hour,
(4) Cannot be determined how much time will it take to cover the same
(5) None of these distance ?
(Indian Overseas Bank PO (1) 40 minutes (2) 38 minutes
Exam. 15.06.2008) (3) 36 minutes (4) 31 minutes
5. A boat running downstreams covers a (5) 49 minutes
distance of 30 kms in 2 hours. While coming (Corporation Bank PO
back the boat takes 6 hours to cover the Exam. 22.11.2009)
same distance. If the speed of the current 10. A man walked at a speed of 4 km/hr from
point A to B and came back from point B to
LEARN MATHS FROM S.K. RAJU (9811549822, 9811649822)
2
A at the speed of 6 km/hr. What would be 15. The respective ratio between the speeds of
the ratio between the time taken by man in a car, a train and a bus is 5 : 9 : 4. The
walking from point A to B to point B to A average speed of the car, the bus and the
respectively ? train is 72 kmph together. What is the
(1) 5 : 3 (2) 2 : 3 average speed of the car and the train
(3) 2 : 1 (4) 4 : 3 together ?
(5) 3:2 (1) 82 kmph
(Corporation Bank P0 (2) 78 kmph
Exam. 22.11.2009) (3) 84 kmph
11. A bus covered a certain distance from (4) Cannot be determined
village A to village B at the speed of 60 km/ (5) None of these
hr. However on its return journey it got (Punjab & Slnd Bank PO
stuck in traffic and covered the same Exam. 16.05.2010)
distance at the speed of 40 km/hr. and took 16. The ratio between the speed of a train and
2 hours more to reach its destination. What a car is 16 : 15 respectively. Also, a bus
is the distance covered between village A covered a distance of 480 kms. in 8 hours.
and B ? The speed of the bus is three-fourth the
(1) 240 km. speed of the train. How much distance will
(2) 260 km. the car cover in 6 hours ?
(3) 200 km. (1) 450 km
(4) Cannot be determined (2) 480 km
(5) None of these (3) 360 km
(Indian Bank Rural Marketing (4) Cannot be determined
Officer Exam. 03.01.2010) (5) None of these
12. A train speeds past a pole in 20 seconds (Bank Of Baroda PO Exam. 30.05.2010)
and speeds past a platform 100 metres in 17. A man crosses a stationary train in 12
length in 30 seconds. What is the length of minutes. The same train crosses a man in
the train ? 54 seconds. What is the respective ratio be-
(1) 100 metre (2) 150 metre tween the speed of the train and the man ?
(3) 180 metre (4) 200 metre (1) 40 : 7
(5) None of these (2) 400 : 3
(Indian Bank Rural Marketing (3) 40 : 3
Officer Exam. 03.01.2010 (4) Cannot be determined
13. A 320 metre long train takes 80 seconds (5) None of these
more to cross a platform twice its length (Bank Of Baroda PO Exam. 30.05.2010)
than it takes to cross a pole at the same 18. Train-A crosses a stationary train -B in 50
speed. What is the speed of the train in seconds and a pole in 20 seconds with the
metre/second ? same speed. The length of the train-A is 240
(1) 16 metres. What is the length of the stationary
(2) 10 Train-B ?
(3) 6 (1) 360 metres
(4) Cannot be determined (2) 260 metres
(5) None of these (3) 300 metres
(Bank Of India Banking Officer (4) Cannot be determined
Exam. 24.01.2010) (5) None of these
14. A 180-metre long train crosses another (Central Bank Of India PO
270-metre long train running in the Exam. 25.07.2010)
opposite direction in 10.8 seconds. If the 19. The respective ratio between the speeds of
speed of the first train is 60 kmph., what is a car, a jeep and tractor is 3 : 5 : 2. The
the speed of the second train in kmph ? speed of the jeep is 250 percent of the speed
(1) 80 of the tractor which covers 360 km in 12
(2) 90 hours. What is the average speed of car and
(3) 150 jeep together ?
(4) Cannot be determined (1) 60 km/hr.
(5) None of these (2) 75 km/hr.
(Allahabad Bank PO Exam. 21.02.2010)
LEARN MATHS FROM S.K. RAJU (9811549822, 9811649822)
3
(3) 40 km/hr. (4) Cannot be determined
(4) Cannot be determined (5) None of these
(5) None of these (Bank Of Maharashtra
(Central Bank Of India Exam. 19.12.2010)
PO Exam. 25.07.2010) 25. A 300 metre long train moving with an
20. A man crosses a stationary bus in 18 average speed of 126 km/hr. crosses a
seconds. The same bus crosses a pole in 4 platform in 24 seconds. A man crosses the
seconds. What is the respective ratio Same platform in 5 minutes. What is the
between the speed of the bus and the speed speed of man in meter/second ?
of the man ? (1) 1.8 metre/second
(1) 9 : 2 (2) 1.2 metre/second
(2) 9 : 4 (3) 1.5 metre/second
(3) 18 : 5 (4) Cannot be determined
(4) Cannot be determined (5) None of these
(5) None of these (Bank Of Maharashtra
(Syndicate Bank PO Exam. 29.08.2010) Exam. 19.12.2010)
21. The speed of a car is 1.5 times the speed of 26. Train-A crosses a stationary train B in 35
a bus. If the car travels at the speed of 60 seconds and a pole in 14 seconds with the
km/ hr., what will be the difference in the same speed. The length of the train-A is 280
time taken by the bus and the time taken metres. What is the length of the stationary
by the car to cover 720 km ? train-B ?
(1) 5 hours (2) 6 hours (1) 360 metres
(3) 4 hours (4) 8 hours (2) 480 metres
(5) None of these (3) 400 metres
(Punjab National Bank (4) Cannot be determined
Specialist Officer Exam. 24.10.2010) (5) None of these
22. A 320 metre long train crosses a platform (Bank Of Maharashtra
thrice its length in 40 seconds. What is the Exam. 19.12.2010)
speed of the train in km/hour ? 27. A car covers a distance of 540 km in 9
(1) 120.6 hours. Speed of a train is double the speed
(2) 115.2 of the car. Two-third of the speed of the train
(3) 108.4 is equal of a bike. How much distance will
(4) Cannot be determined the bike cover in 5 hours ?
(5) None of these (1) 450 km (2) 360 km
(United Bank Of India PO (3) 400 km (4) 500 km
Exam. 14.11.2010) (5) None of these
23. A bike covers a certain distance at the speed (Oriental Bank Of Commerce PO
of 64 km/hr. in 8 hours. If the bike was to Exam. 26.12.2010 (1st Sitting)
cover the same distance is approximately 28. Train-A crosses a pole in 25 seconds and
6 hours, at what approximate speed another Train-B crosses a pole in 1 minute
should the bike travel ? and 15 seconds. Length of train-A is half
(1) 80 kmph (2) 85 kmph length of train-B. What is the respective
(3) 90 kmph (4) 75 kmph ratio between the speeds of Train-A and
(5) 70 kmph Train-B ?
(PNB Management Trainee (1) 3 : 2
Exam. 28.11.2010) (2) 3 : 4
24. The ratio between the speed of a train and (3) 4 : 3
a car is 18 : 13 respectively. Also, a bus (4) Cannot be determined
covered a distance of 480 kms. in 12 hours. (5) None of these
The speed of the bus is five-ninth the speed (Union Bank Of India PO
of the train. How much distance will the Exam. 09.01.2001)
car cover in 5 hours ? 4
(1) 250 kms. 29. The average speed of a car is l times the
5
(2) 280 kms.
(3) 260 kms. average speed of a bus. A tractor covers 575

LEARN MATHS FROM S.K. RAJU (9811549822, 9811649822)


4
km in 23 hours. How much distance will (2) 8 m/s
the car cover in 4 hours if the speed of the (3) 12 m/s
bus is twice the speed of the tractor ? (1) (4) Cannot be determined
340 km (2) 480 km (5) None of these
(3) 360 km (4) 450 km (Allahabad Bank PO Exam. 17.04.2011)
(5) None of these 3
(Corporation Bank PO 35. The average speed of a train is 1 times
7
Exam. 16.01.2011)
30. A car covers the first 39 km. of its journey the average speed of a car. The car covers a
in 45 minutes and covers the remaining 25 distance of 588 km in 6 hours. How much
km. in 35 minutes. What is the average distance will the train cover in 13 hours ?
speed of the car ? (1) 1750 km (2) 1760 km
(1) 40 kmph (2) 64 kmph (3) 1720 km (4) 1850 km
(3) 49 kmph (4) 48 kmph (5) None of these
(5) None of these (Indian Overseas Bank
(Punjab & Sind Bank PO PO Exam. 22.05.2011]
Exam. 23.01.2011) 36. Amit, Sucheta and Neeti start running
31. An aeroplane flies with an average speed of around a circular track and complete one
756 km/hr. A helicopter takes 48 hours to ro un d in 18, 24 and 32 secon ds
cover twice the distance covered by respectively. In how many seconds will the
aeroplane in 9 hours. How much distance three meet again at the starting point. if
will the helicopter cover in 18 hours ? they all have started running at the same
(assuming that flights are non-stop and time ?
moving with uniform speed) (1) 196
(1) 5014 km (2) 5140 km (2) 288
(3) 5130 km (4) 5103 km (3) 324
(5) None of these (4) Cannot be determined
(UCO Bank PO Exam. 30.01.2011) (5) None of these
32. A 320 metre long train moving with an (Bank Of India Banking
average speed of 120 km/hr crosses a Officer Exam. 24.01.2010
platform in 24 seconds. A man crosses the 37. Philip, Tom and Brad start jogging around
same platform in 4 minutes. What is the a circular field and complete a single round
speed of man in metre/second ? in 18, 22 and 30 seconds respectively In
(1) 2.4 (2) 1.5 how much time will they meet again at the
(3) 1.6 (4) 2.0 starting point ?
(5) None of these (1) 8 mins. 15 sees.
(Bank Of Baroda PO Exam. 13.03.2011) (2) 21 mins.
33. The ratio between the speed of a bus and (3) 16 mins. 30 sees.
train is 15 : 27 respectively. Also, a car (4) 12 mins.
covered a distance of 720 km. in 9 hours. (5) None of these
The speed of the bus is three-fourth of the (Indian Bank PO Exam. 02.01.2011)
speed of the car. How much distance will 38. A 320 metre long train crosses a pole in 50
the train cover in 7 hours ? seconds. What is the speed of train ?
(1) 760 km. (1) 4.6 m/sec
(2) 756 km. (2) 7.2 m/sec
(3) 740 km. (3) 5.4 m/sec
(4) Cannot be determined (4) 6.4 m/sec
(5) None of these (5) Cannot be determined
(Allahabad Bank PO Exam. 17.04.2011) (IBPS Specialist Officer
34. A 280 metre long train moving with an CWE 17.03.2013)
average speed of 108 km/hr crosses a 39. A car covers a distance of 525 km. in a
platform in 12 seconds. A man crosses the certain time at a speed of 66 km./hr. How
same platform in 10 seconds. What is the much distance would a truck cover at an
speed of the man in metre/ second ? average speed which is 24 km hr. less than
(1) 5 m/s that of the speed of the car in time which is
7 hours more than that taken by the car ?
LEARN MATHS FROM S.K. RAJU (9811549822, 9811649822)
5
(1) 336 km. (2) 682 km. Exam. 14.02.1999)
(3) 598 km. (4) 630 km. 4. A man starts going for morning walk every
(5) None of these day. The distance walked by him on the first
(IBPS RRBs Office Assistant day was 2 kms. Everyday he walks half of
CWE Exam. 09.09.2012 the distance walked on the previous day.
40. A person travels from P to Q at a speed of What can be the maximum total distance
40 kmph and returns to Q by increasing walked by him in his life time ?
his speed by 50% . What is his average (1) 4 kms. (2) 120 kms.
speed for both the trips ? (3) 18 kms. (4) Data inadequate
(1) 36 kmph (2) 45 kmph (5) None of these
(3) 48 kmph (4) 50 kmph (SBI Banks PO Exam. 11.02.2001)
(5) None of these 5. A car covers its journey at the speed of 80
(IBPS Bank PO/MT km/hour in 10 hours. If the same distance
CWE-III 26.10.2013) is to be covered in 4 hours, by how much
41. A 320 metre long train crosses a pole in 16 the speed of car will have to increase ?
seconds. It stops five times of duration 18 (1) 8 km/hr (2) 10 km/hr
minutes each. What time will it take in (3) 12 km/hr (4) 16 km/hr
covering a distance of 576 km ? (5) None of these
1 (SBI PO Exam. 26.11.2006)
(1) 9 hours (2) 9 hours 6. A motor starts with the speed of 70 kmph
4
with its speed increasing every two hours
1 1 by 10 kmph. In how many hours will it cover
(3) 9 hours (4) 8 hours 345 kms ?
2 2
(5) None of these 1
(Corporation Bank Specialist (1) 2 hours
4
Officer (Marketing) Exam. 22.12.2014)
1
(2) 4 hours
SBI PO EXAMS 2
1. A 260 metre long train crosses a 120 meter (3) 4 hours 5 minutes
long wall in 19 seconds. What is the speed (4) Cannot be determined
of the train ? (5) None of these
(1) 27 km/hr (2) 49 km/hr (SBI Associate Banks PO
(3) 72 km/hr (4) 70 km/hr Exam. 07.01.2007)
(5) None of these 7. Raman drove from home to a neighbouring
(SBI Associate Banks PO town at the speed of 50 kms./hr and on his
Exam. 14.02.1999) returning journey, he drove at the speed of
2. Two cars starts at the same time from A 45 kms./hr. and also took an hour longer
and B which is 120 km apart. If the two to reach home. What distance did he cover
cars tarvel in opposite direction they meet each way ?
after one hour and if they travel in same (1) 450 kms. (2) 225 kms.
direction (from A towards B) then A meet B (3) 900 kms. (4) 500 kms.
after 6 hours. What is the speed of car (5) None of these
starting from A ? (SBI PO Preliminary (Tire-I)
(1) 70 kmph (2) 120 kmph. Exam. 27.04.2008)
(3) 60 kmph (4) Data inadequate
(5) None of these RBI GRADE-B OFFICER EXAMS
(SBI Associate Banks PO 1. Nitin rides a bicycle at the speed of 15 kms/
Exam. 14.02.1999) hr., but stops for 10 minutes to take rest
3. Find the speed of train in km/hr whose every 20 kms. How much time will he take
length is 200 m and crosses a platform of to cover a distance of 150 kms. ?
length 240 m in 22 seconds ? (1) 12 hours 30 minutes
(1) 60 (2) 48 (2) 11 hours 10 minutes
(3) 53 (4) Data inadequate (3) 10 hours 20 minutes
(5) None of these (4) 12 hours 10 minutes
(SBI Associate Banks PO (5) None of these
LEARN MATHS FROM S.K. RAJU (9811549822, 9811649822)
6
(RBI Grade-B Officer Exam. 2008) (1)90 kms/hr.
2. A 280 metres long train, travelling at a (2)40 kms/hr.
uniform speed, crosses a platform in 60 (3)80 kms/hr.
seconds and a man standing on the (4)Data inadequate
platform in 20 seconds. What is the length (5)None of these
of the platform ? (LIC Assistant Administrative
(1) 640 metres Officer (AAO) Exam. 24.04.2005)
(2) 420 metres 4. An employee may claim Rs. 7.00 for each
(3) 280 metres km when he travels by taxi and Rs. 6.00 for
(4) Cannot be determined each km if he drives his own car. If in one
(5) None of these week he claimed Rs. 675 for travelling 90
(RBI Grade-B Officer Exam.06.02.2011) km, how many kms did he travel by taxi ?
3. Train A crosses a pole and platform in 18 (1) 135 (2) 155
secnds and 39 seconds respectively. The (3) 162 (4) 170
length of platform is 157.5 metre. What will (United India Insurance Co.
be the length of train B if it is equal to the (AAO) Exam. 11.03.2007)
sum of half of the length of train A and twice 5. A train B speeding with 120 kmph crosses
the length of the platform ? another train C running in the same
(1) 382.5 metre direction, in 2 minutes. If the lengths of the
(2) 328.5 metre trains B and C be 100 m and 200 m
(3) 238.5 metre respectively, what is the speed (in kmph)
(4) 315 metre of the train C ?
(5) None of these (1) 111 (2) 123
(3) 127 (4) 129
(RBI Officer Grade ‘B’ Online (United India Insurance Co.
Exam. 25.08.2013 (AAO) Exam. 11.03.2007)
6. A train crosses a 300 metre long platform
INSURANCE EXAMS in 38 seconds while it crosses a signal pole
1. A and B started towards each other at the in 18 seconds. What is the speed of the train
same time from places 114 kms. apart and in kmph ?
met in 8 hours. If ‘A’s rate was 6 kms. an (1) Cannot be determined
hour, what was ‘B’s ? (2) 72
(1) 14.25 km/hr. (2) 7 km./hr (3) 48
(4) 54
1 1 (5) None of these
(3) 7 km./hr (4) 8 km/hr.
8 2 (LIC Assistant Administrative
1 Officer Exam. 2008)
(5) 8 km/hr. 7. A man goes uphill with an average speed of
4
24 kmph. and comes down with an average
(United India Insurance Co. speed of 36 kmph. The distance travelled
AAO Exam. 21.04.2002) in both the cases being the same. The
2. Sunil left for city ‘x’ from city ‘y’ at 5.50a.m. average speed (in kmph) for the entire
He travelled at the speed of 80 kms/hr. for journey is :
2 hours 15 minutes. After that the speed (1) 30 (2) 28.8
was reduced to 60 kms/hour. If the dis- (3) 32.6 (4) 30.8
tance between two cities is 350 kms., at (LIC Assistant Administrative
what time did Sunil reach the city ‘Y? Officer (AAO) Exam. 07.06.2009)
(1) 10.05 a.m. (2) 10.35 a.m. 8. A train overtakes two persons who are
(3) 9.50 a.m. (4) 9.55 a.m. walking in the same direction in which the
(5) None of these train is going at the rate of 2 kmph and 4
(LIC Assistant Administrative kmph, and passes them completely in 9 and
Officer (AAO) Exam. 24.04.2005) 10 seconds respectively. The length of the
3. Two trains A and B of equal length of 200 train (in metres) is :
metres running in opposite direction cross (1) 72 (2) 54
each other in 16 seconds. What is the speed (3) 50 (4) 45
of train A ? (LIC Assistant Administrative
LEARN MATHS FROM S.K. RAJU (9811549822, 9811649822)
7
Officer (AAO) Exam. 07.06.2009) he take to walk twice the distance, twice as
9. Train-A crosses a pole in 33 seconds and fast and rests twice as long each day ?
another Train-B crosses a pole in 55 (1) 40 days (2) 60 days
seconds. Length of Train-A is three-fourth (3) 80 days (4) 100 days
of Train-B. What is the respective ratio (Oriental Insurance Company
between the speed of Train-A and Train-B ? AAO Exam. 08.04.2012)
(1) 5 : 11 15. A person leaves a place A to place B at 6
(2) 5 : 4 a.m. and reaches place B at 10 a.m. Another
(3) 11 : 3 person leaves B at 7.30 a.m. and reaches A
(4) Cannot be determined at 11 a.m. They will meet each other at
(5) None of these (1) 7.40 a.m. (2) 8.20 a.m.
(United India Insurance (3) 8.40 a.m. (4) 9.20 a.m.
AO Exam. 27.03.2011) (Oriental Insurance Company
10. There is a hill behind a person’s house. He AAO Exam. 08.04.2012)
walks up the top of the hill at a speed of 16. A can walk a certain distance in 10 days
1 1 when he rests 9 hours a day. How long will
1 km/hour, but walks down it at 4 km/ he take to walk twice the distance, twice as
2 2
fast and rest twice as long each day ?
hour. If it takes him 6 hours for the entire (1) 4 days (2) 10 days
journey, the distance, in km, from his house (3) 15 days (4) 25 days
to the top of the hill is (United India Insurance AAO
1 Exam. 03.06.2012)
(1) 5 (2) 6 17. A 222 metre long train crosses a pole in 6
4
seconds. The same train crosses a man
3 running in the same direction in 10
(3) 6 (4) 9
4 seconds. What will be the speed of man ?
(New India Insurance AAO (1) 15 m/sec (2) 17 m/sec
Exam. 22.05.2011) (3) 18 m/sec (4) 14 m/sec
Directions (11-13): The cost of fuel running (5) None of these
the engine of an army tank is proportional to the (LIC Assistant Administrative
square of the speed and Rs. 64 per hour for a speed Officer (AAO) Exam. 12.05.2013)
of 16 kmph. Other costs amount to Rs. 400 per 18. How long (time in seconds) will a 450 metre
hour. The tank has to make a journey of 400 km at long train take to cross a girl walking with
a constant speed. a speed of 5 km/hr in the direction of the
(General Insurance Corporation moving train. The speed of the train is 65
AAO Exam. 11.12.2011) km/hr ?
11. The most economical speed for this journey (1) 27 (2) 28
is : (3) 30 (4) 29
(1) 20 kmph (2) 30 kmph (NICL (GIC) AO (Finance
(3) 35 kmph (4) 40 kmph Exam. 08.09.2013 (Paper-I)
12. The total cost for the journey at this most 19. An express train travelled at an average
economical speed is : speed of 100 km/hr stopping for 3 minutes
(1) Rs. 6000 (2) Rs. 8000 after 75 km. A local train travelled at a speed
(3) Rs. 10000 (4) Rs. 11000 of 50 km/hr, stopping for 1 minute after
13. Ravi can walk a certain distance in 40 days every 25 km. If the trains began travelling
when he rests 9 hours a day. How long will at the same time, how many kms did the
he take to walk twice the distance, twice as local train travel in the time it took the
fast and rest twice as long each day ? express train to travel 600 km ?
(1) 40 days (2) 80 days (1) 307.5 km (2) 900 km
(3) 50 days (4) 100 days (3) 1000 km (4) 1200 km
(New India Assurance AO (NICL (GIC) Administrative
Exam. 25.10.2009) Officer Exam. 15.12.2013)
14. A can walk a certain distance in 20 days 20. If a student walks from his house to school
when he rests 8 hours a day. How long will at 5 km/hr, he is late by 30 minutes.
However, if he walks at 6 km/hr, he is late

LEARN MATHS FROM S.K. RAJU (9811549822, 9811649822)


8
by 5 minutes only. The distance of his 21. A plane left 30 minutes later than the
school from his house is. scheduled time and in order to reach the
(1) 2.5 km (2) 3.6 km destination 1500 km away in time, it had
(3) 5.5 km (4) 12.5 km to increase the speed by 250 km/hr from
(NICL (GIC) Administrative the usual speed. Its usual speed is:
Officer Exam. 15.12.2013) (1) 720 km/hr (2) 730 km/hr
(3) 740 km/hr (4) 750 km/hr
(NICL (GIC) AO (Finance)
Exam. 5.12.2013)

LEARN MATHS FROM S.K. RAJU (9811549822, 9811649822)


9
SHORT ANSWERS For next two hours speed of motor = 80
NATIONALISED BANKS kmph.
Distance covered in next 2 hours = 2 × 80
& IBPS PO/MT/SO
= 160 kms.
1. (2) 2. (5)
Distance covered in first 4 hours
3. (2) 4. (4)
= 140 + 160 = 300 Remaining distance =
5. (3) 6. (5)
345 - 300 = 45 km
7. (3) 8. (3)
This distance will be covered at the speed
9. (1) 10. (5)
of 90 kmph.
11. (1) 12. (4)
13. (5) 14. (2) 45 1
15. (3) 16. (1)  Time taken = =
2
hour
90
17. (3) 18. (1)  Total time
19. (1) 20. (1)
1 1
21. (2) 22. (2) =4+ = 4 hours
23. (2) 24. (3) 2 2
25. (1) 26. (5) 2. (5) Let the length of train be x metres.
27. (3) 28. (1) x
29. (3) 30. (4) Time taken in crossing the pole =
20
31. (4) 32. (4)
33. (2) 34. (2) Time taken in crossing the bridge
35. (5) 36. (2) 4x
37. (3) 38. (4) =
20
39. (4) 40. (3)
41. (3) 4x x
As given, - = 24
2 20
SBI PO EXAMS 3x 24  30
1. (3) 2. (1)  = 24  = 160 m
20 2
3. (5) 4. (1)
3. (2) Time taken in walking one way + riding
5. (5) 6. (2)
other way
7. (1)
= 6 hours 35 minutes ... (i)
Time taken in riding both ways
RBI GRADE-B OFFICER EXAMS = 4 hours 35 minutes ...(ii)
1. (2) 2. (5) By equation (i) × 2 - (ii),
3. (1) 2 × Time taken in walking one way
= 13 hours 10 minutes - 4 hours
INSURANCE EXAMS 35 minutes
1. (5) 2. (5) = 8 hours 35 minutes
3. (5) 4. (1) 4. (4) When a train crosses a platform, the
5. (1) 6. (4) distance covered
7. (2) 8. (3) = Length of platform and the train.
9. (2) 10. (3)
11. (4) 12. (2)
Length of (platform + train)
13. (4) 14. (1) Speed =
15. (3) 16. (4) Time taken
17. (1) 18. (1) Thus we have inadequate data.
19. (1) 20. (4) 5. (3) Let the speed of boat in still water = x
21. (4) kmph
x
EXPLANATIONS  Speed of current = 2 kmph
NATIONALISED BANKS When boat runs downstream,
& IBPS PO/MT/SO
30
1. (2) Initial speed of motor = 70 kmph.
x =2
Distance covered in first 2 hours = 2 × 70 x
= 140 kms. 2
LEARN MATHS FROM S.K. RAJU (9811549822, 9811649822)
10
30  2 110 60
 = 2  6x = 60 = × = 40 minutes
3x 3 55
60 10. (5) Required ratio = 6 : 4 = 3 : 2
 x= = 10 kmph as time taken is inversely proportional to
6
the speed.
6. (5) Let the required distance be x km. 11. (1) Let the distance between villages A and
According to the question, B be x km.
x x x x 3x  2x
- =1 =2 
40 45  -
120
=2
40 60
9x  8 x  x = 2 × 120 = 240 km
 =1
360 12. (4) Let the length of train be x metre.
x x x  100
 =1  Speed of train = 20 =
360 30
 x = 360 km.  3x = 2x + 200
7. (3) Length of the train = 240 metre  x = 200 m = length of train
 Length of the platform = 480 metre 13. (5) Let the speed of the train be x m/sec.
We know that when a train crosses a
320  3 320
platform, it covers a distance equal to the  - = 80
sum of its length and that of platform. x x
 Speed of train 320  2
 = 80
 240  480  x
=  m/sec
 40  320  2
x= = 8 m/sec = Speed of train
720 80
=18 m/sec 14. (2) Time taken by trains in crossing each
40
8. (3) Let the speed of boat in still water be x Sum of lengths of trains
kmph and that of current be y kmph. other =
Relative speed
4.8 4.8  60 60  5 50
x + y = 8 = 60 kmph = = m/sec.
8 18 3
60 If the speed of other train be x m/sec. then,
 x + y = 36 ....(i) 180  270
10.8 =
4.8 4.8  60 50
and, x - y = x
9 = 9 3
60  180 + 10.8x = 450
 x - y = 32 ....(ii)  10.8x = 450 - 180 = 270
By equation (i)-(ii), 270
x + y- x + y x= = 25 metre/sec.
10.8
= 36 - 32 = 4
4 18
= 25 × kmph = 90 kmph
 2y = 4  y = 2
= 2 kmph 5
15. (3) 5x + 9x + 4x = 72 × 3
9. (1) Distance between Ramgarh and Devgarh
 18x = 72 × 3
50  44 110
= = km 72  3
60 3 x = = 12 kmph
New speed = 55 kmph 18

55 5 x  9x
= km/minute  Average speed of car and train = 2
60
= 7x = 7 × 12 = 84 kmph
Distance 16. (1) Tricky approach
 Required time = Speed

LEARN MATHS FROM S.K. RAJU (9811549822, 9811649822)


11
Speed of the bus =
480
= 60 kmph  32  18 
8  5  kmph = 115.2 kmph
 
4 23. (2) Distance = 64 × 8 = 512km.
 Speed of the train = 60 × = 80 kmph
3 Distance
 Required speed =
15 Time
 Speed of the car = × 80 = 75 kmph
16 512
 Required distance = Speed × time = = 85.33  85 kmph.
6
= 75 × 6 = 450 km.
24. (3) Speed of bus
17. (3) Tricky approach
If the length of the train be l metre, then Distance covered 480
= = = 40 kmph
l Time taken 12
l
Required ratio = : = 40 : 3
54 12  60 9
 Speed of train = × 40 = 72 kmph
18. (1) If the length of train-B be x metre, then 5
240  x 240  18  72
Speed of train = = 72
50 20
1  =4
18
240  x
 = 12  4 × 13 = 52 kmph = speed of car
50
Distance covered by car in 5 hours
 240 + x = 600 = 5 × 52 = 260 km
 x = 360 metre 25. (1) Let the length of the platform be x metre.
360 Speed of train =126 kmph
19. (1) Speed of tractor = = 30 kmph
12  126  5 
=   m/sec. = 35 m/sec.
250  18 
Speed of jeep = 30 × = 75 kmph
100 x  300
 Speed of train = 24
3
 Speed of car = × 75 = 45 kmph
5 x  300
 Average speed of car and jeep together  35 = 24
 75  45   35 × 24 = x + 300
=   kmph = 60 kmph
 2   840 = x + 300
20. (1) If the length of the bus be x metre, then  x = 840 - 300 = 540 metre
x x Distance
Required ratio = : =9:2  Speed of man =
4 18 Time
60 540
21. (2) Speed of bus = = 40 kmph = =1.8 m/sec.
1.5 5  60
26. (5) Let the length of train B be x metre.
 720 720 
 Required difference =  40  60  hours  Speed of train
 
Sum of lengths of trains
= (18 - 12) hours = 6 hours =
Time taken
22. (2) Speed of train
Length of (train + platform)
Length of train - A
=
= Time taken
Time taken
280  x 280
 320  3  320   =
=   metre/second 35 14
 40 
280  x
= 32 metre/second  = 20
35
 280 + x = 700

LEARN MATHS FROM S.K. RAJU (9811549822, 9811649822)


12
 x = 700 - 280 = 420 metre = 283.5 kmph
 Distance covered by helicopter in 18
Distance
27. (3) Speed of car = hours = (283.5 × 18) km = 5103 km
Time 32. (4) Speed of train = 120 kmph
540 120  5 100
= = 60 kmph = = m/sec
9 18 3
Speed of train = 2 × 60
= 120 kmph 100 (Length of train and platform)
 =
Time taken in crossirg
 Speed of bike 3
2 100 320  x
=
3
× 120 = 80 kmph  =
3 24
 Distance covered by bike in 5 hours Where x = length of platform
= 5 × 80 = 400 km
24  100
28. (1) If the length of train A be x metre, then  = 320 + x
length of train B = 2x metre. 3
When a train crosses a pole, it covers a  800 = 320 + x
distance equal to its own length.  x = 800 - 320 = 480 m
x 2x  Man’s speed
 Required ratio = 25 : 480
75 = = 2 m/sec
4  60
1 2
= × 75 : × 75 = 3 : 2 33. (2) Speed of car
25 75
Distance covered
Distance =
29. (3) Speed of tractor = Time taken
Time
720
575 = = 80 kmph
= = 25 kmph 9
23
3
 Speed of bus = 50 kmph  Speed of bus = × 80 = 60 kmph
 Speed of car 4

9 27
= × 50 = 90 kmph  Speed of train = × 60 = 108 kmph
5 15
 Distance covered by car in 4 hours  Distance covered by train in 7 hours
= 4 × 90 = 360 km = 7 × 108 = 756 km.
30. (4) Average speed 34. (2) Speed of train =108 kmph

Total distance 108  5


= = = 30 m/second
Total time 18
If the length of platfrom be x metre, then
 39  25  km x  280
 
= 45  35 minute
12
= 30

64  x + 280 = 30 × 12 = 360
= kmph  x = 360 - 280 = 80 metre
80
60 Distance
 Man’speed =
Time
 64  60 
= 
 80  80
= = 8 m/second
= 48 kmph 10
31. (4) Distance covered by aeroplane in 9 35. (5) Average speed of car
hours Distance
= Speed × Time = 9 × 756 = 6804 km = = 98 kmph
Time
2  6804
 Speed of helicopter = 98  10
48 Average speed of train = = 140 kmph
7
LEARN MATHS FROM S.K. RAJU (9811549822, 9811649822)
13
Distance covered by train in 13 hours 380
= Speed × Time = 140 × 13 = 1820 km = m/sec. = 20 m/sec.
19
36. (2) Required time = LCM of 18, 24 and 32
seconds. = 288 seconds 20  18
37. (3) Required time = LCM of 18, 22 and 30 = km/hr = 72 km/hr
5
seconds 2. (1) Let speed of car starting at
990 A = x km/hr
= 990 seconds = minutes and speed of car starting at B = y km/hr
60
= 16 minutes 30 seconds  x + y = 120 .... eq (i)
6x - 6y = 120 ....eq (ii)
320  x = 70 km/hr. y = 50 km/hr.
38. (4) Speed of train = = 6.4 m/sec.
50 3. (5) Speed of train
528 length of (train + platform)
39. (4) Time taken by car = = 8 hours =
66 time taken to cross
Case II, 440
Time =15 hours = = 20 m/s
22
Speed = 66 - 24 = 42 kmph
 Distance covered by truck 18
= Speed × Time = 20 × = 72 km/hr
5
= 15 × 42 = 630 km 4. (1) The distance walked on the first day = 2
40  150 kms.
40. (3) Speed of return = The distance walked on subsequent days
100
= 60 kmph is half the distance walked on the previous
days.
2  40  60
Average speed = 1 1
40  60 Total distance walked = 2 + 1+ + + ...
2 2
4800 This is a Geometric series Whose
= = 48 kmph
100 First term, a = 2
41. (3) Speed of train
1
Length of train Common ratio, r =
2
=
Time taken in crossing Maximum total distance walked by the
320 person in his life-time means the number
= = 20m/sec. of terms in the series would be infinite.
16
Hence, the series would be an infinite
18 Geometric series. Sum of an infinite
= 20 × = 72 kmph Geometric series is given by
5
Total period of stoppage = 5 × 18 a 2
S= S=
3 1 r 1
1
= 90 minutes = hours
2 2
Total time taken in covering a distance of
2
576 km. S= , S = 4 kms.
1
 576 3  2
=    hours
 72 2  5. (5) Initial speed = 80 km/hr
3 1
Total distance = 80 × 10 = 800 km.
=8+ = 9 hours 800
2 2
New speed = = 200 km / hr.
4
SBI PO EXAMS Increase in speed
1. (3) We know that = 200 - 80 = 120 km/hr.
6. (2) Initial speed of motor = 70 kmph.
Distance  260  120  Distance covered in first 2 hours = 2 × 70
Speed = =   m/ sec.
Time  19 
LEARN MATHS FROM S.K. RAJU (9811549822, 9811649822)
14
= 140 kms.
 135 
For next two hours speed of motor  Length of train B =  2  2  157.5 
= 80 kmph.  
 Distance covered in next 2 hours = (67.5 + 315) metre = 382.5 metre
= 2 × 80 = 160 kms.
Distance covered in first 4 hours INSURANCE EXAMS
= 140 + 160 = 300 km 1. (5) According to the question, Distance
Remaining distance = 345 - 300 = 45 km covered by A in 8 hrs.
This distance will be covered at the speed = 6 × 8 = 48 km
of 90 kmph.  Distance covered by B in 8 hrs.
45 1 = (114 - 48) km = 66 km
 Time taken = = hour
90 2 66 1
 Required speed = kmph = 8 kmph
1 1 8 4
 Total time = 4 + 2 = 4 2 hours 2. (5) Distance between two cities = 350 km
Distance travelled by Sunil with the speed
7. (1) Let the required distance be = x km.
According to the question, 1
of 80 km/hr in 2 hr or 2.25 hr
x x 2
- =1 = 80 × 2.25 km = 180 km
45 50
Remaining distance
10x  9x = 350 - 180 = 170 km
 =1
450 Speed = 60 km/hr
 x = 450 km. Time taken to reach in city
170
RBI GRADE-B OFFICER EXAMS x= = 2.83 hr
60
1. (2)Nitin will take rest sever, times i.e. 70 Total time = 2.25 + 2.83
minutes in covering 150 km. = 5.08 hr = 305 minute
Time taken to cover 150 km at 15 kmph = 6 hr. 5 minute
150 Sunil started journey at 5:50 a.m. &
= = 10 hours finished the journey = 11:55 a.m.
15
3. (5) Distance = 200 metres
 Total time
= 10 hours + 70 minutes time =16 seconds
= 11 hours 10 minutes distance 200
2. (5) When a train crosses a platform it covers Speed = = = 12.5m/sec.
time 16
a distance equal to the sum of lengths of
platform and train. 12.5  1000
= km/hr = 3.47 Km/hr.
Let the length of platform be x metre. 3600
4. (1) Let the employee travelled x kms by taxi.
x  280 280
 Speed of train = = = 14  Distance covered by him by his own car
60 20 = (90 - x) km.
According to the question,
 x + 280 = 60 × 14 = 840 x × 7 + (90 - x) × 6 = 675
 x = 840 - 280 = 560 metre 7x + 540 - 6x = 675
3. (1) Length of train A = x metre x = 675 - 540 = 135
x x  157.5  Required distance = 135 km.
 = 5. (1) Let the speed of train C be x kmph.
18 39
Speed of train B relative to C
 13x = 6x + 157.5 × 6 = (120 - x) kmph
 7x = 945
 5
945 = 120  x    m/sec
x= = 135 metre  18 
7
 600  5x 
=  
 18 

LEARN MATHS FROM S.K. RAJU (9811549822, 9811649822)


15
Distance covered = 100 + 200 = 300m
 2 5  5
300 8. (3) 2 kmph =   metre/sec. = metre/
 18  9
 600  5x  = 120
 
sec.
 18 
45 10
and 4 kmph = metre/sec. = metre/
120  600  5x  18 9
 300 = sec.
18
Let the length of the train be x metre and
 10 × 9 = 2 (600 - 5x)
its speed be y metre/ sec. Then,
 90 = 1200 - 10x
 10x = 1200 - 90 x
5=9
1110 y
x= = 111 9
10
Hence, the speed of train C is 111 kmph.  9y - 5 = x
6. (4) When a train crosses a platform it covers  9y - x = 5 ........... (i)
a distance equal to the sum of lengths of x
platform and the train itself. If the length and = = 10
10
of train be x metres, then y
9
Speed of train
 10 (9y - 10) = 9x
x  300
= m/sec ...(i)  90y - 9x = 100 ......... (ii)
38 By equation (i) × 10 - equation (ii), we have
When the train crosses a signal post it
covers its own length. 90y  10x  50
 Speed of train x 90y  9x  100
x   
= m/sec ...(ii)
18  x   50
From equations (i) and (ii)
 x = 50 m
x  300 x 9. (2) Let the length of train B = x metre
=
38 18
3x
 38x - 18x = 300 × 18  Length of train A = metre
4
300  18
x= 3x x
20  Required ratio = : =5:4
4  33 55
= 270 metres
10. (3) If the distance be x km, then
270
 Speed of train = x x
18 3+9=6
= 15m/sec.
2 2
18
= 15 × = 54 kmph 2x 2x
5  + =6
3 9
2xy
7. (2) Average speed = x  y 6x  2x
 =6
9
(when the same distances are covered)
 8x = 9 × 6
 2  24  36  54 27 3
=   kmph x=
 24  36  8
=
4
= 6 km.
4
2  24  36 11. (4) C = kx2
= = 28.8 kmph When x = 16 kmph
60
C = Rs. 64
 64 = k × 162

LEARN MATHS FROM S.K. RAJU (9811549822, 9811649822)


16
1 7
k= = × 60 = 70 minutes
4 6
1  Requisite time = 8:40 a.m.
C = × x2 16. (4) Working hours per day = 24 - 9 = 15
4
Total working hours = 15 × 10 = 150
1 2 Distance is doubled and speed too. Both
Total expenditure per hour =x + 400 cancel each other.
4
When speed = 40 kmph, the expenditure New working hours = 24 - 18 = 6 hours
will be minimum. 150
12. (2) Total expenditure  Required time = = 25 days
6
1 222
= × 40 × 40 × 10 + 400 × 10 = Rs. 8000 17. (1) Speed of train = = 37m/sec.
4 6
13. (4) Tricky approach If the speed of man be x m/sec, then
Working hours per day = 24 - 9 = 15 hours
Total time = 15 × 40 = 600 hours 220
= 10
Decrease in walking hours per day after 37  x
increase in hours of rest = 24 - (2 × 9)  220 = 370 - 10x
= 6 hours  10x = 370 - 220 = 150
600  x = 15 m/sec
Required time = = 100 days 18. (1) Relative speed
6
14. (1) Working hours per day = 65 - 5 = 60 kmph
= 24 - 8 = 16 hours 60  5 50
Total working time = 16 × 20 = 320 hours = m/sec = m/sec
18 3
In case II,
Distance is doubled and speed too. 450
Working hours per day = 24 - 16 = 8 hours  Required time = 50
320 3
 Required time = = 40 days
8 450  3
15. (3) If AB = x km, then Speed of person = = 27 seconds
50
x 19. (1) Effective time taken by express train in
starting from A = kmph covering 75 km
4
Speed of person starting from B  60 
=   75  3  minutes = 48 minutes
x 2x  100 
= = kmph
7 7 Time taken in covering 600 km
2 = (48 × 7 + 45) minutes
Relative speed = 336 + 45 = 381 minutes
Effective time taken by local train in
x 2x 7x  8x 15 x
= + = = kmph covering 25 km = 31 minutes
4 7 28 28
Distance covered by first person till 7:30  31)381(12
x 3 3x 31
a.m. = × = km 71
4 2 8
3x 62
Remaining distance = x - = 5x km. 9
8 8
Remaining distance Distance covered in 9 minutes
Time of meeting =
Relative speed 50
= × 9 = 7.5 km
60
5x
Distance covered in (31 × 120 minutes
8 5x 28 7 = 12 × 25 = 300 km
= 15x = × = hours
8 15x 6
28 LEARN MATHS FROM S.K. RAJU (9811549822, 9811649822)
17
Total distance = 300 + 7.5 = 307.5 km 21. (4) Usual speed of plane = x kmph
20. (4) If the required distance be x km, then  New speed = (x + 250) kmph
x x 30  5 1500 1500 30
-
5 6
=
60  -
x  250
=
x 60
6x  5x 25 5  x  250  1  1
 = =
30 60 12  1500  x  x  250   = 2
 
30  5
x= = 12.5 km  x(x + 250) = 1500 × 500 = 750000
12  x (x + 250) = 750 (750 + 250)
 x = 750 kmph

LEARN MATHS FROM S.K. RAJU (9811549822, 9811649822)


18
MODEL EXERCISES
1. A train running between two stations A and 1
B arrives at its destination 10 min late when hour and increase the speed by km each
2
its speed is 50 km/h and 50 min late when
its speed is 30 km/h. What is the distance succeeding hour. After how many hours will
between the stations A and B ? the second car overtaken the first if both
(1) 40 km (2) 50 km go non stop ?
(3) 60 km (4) 70 km (1) 9 h (2) 5 h
(5) None of these (3) 7 h (4) 8 h
2. A student rides on a bicycles at 8 km/h (5) None of these
and reaches his school 2.5 min late. The 8. Two rockets approach each other, one at
next day he increases his speed to 10 km/ 42000 mile/h and the other at 18000 mile/
h and reaches the school 5 min early. How h. They start 3256 miles a part. How far
far is the school from his house ? are they a part (in miles) 1 min before im-
(1) 1.25 km (2) 8 km pact ?
(3) 5 km (4) 10 km (1) 1628 (2) 1000
(5) None of these (3) 826 (4) 1200
3. A small aeroplane can travel at 320 km/h (5) None of these
in still air. The wind is blowing at a constant 9. Dinesh travels 760 km to his home, partly
speed of 40 km/h. The total time for a jour- by train and partly by car. He takes 8 h if
ney against the wind is 135 min. What will he travels 160 km by train and the rest by
be the time in minutes for the return car. He takes 12 min more if he travels 240
journey with the wind? (Ignore take off and km by train and the rest by car. The speeds
landing time for the aeroplane) of the train and the car respectively are
(1) 94.5 (2) 105 (1) 80 km/h, 100 km/h
(3) 108.125 (4) 120 (2) 100 km/h, 80 km/h
(5) None of these (3) 120 km/h, 100 km/h
4. Two cyclists start on a circular track from (4) 100 km/h, 120 km/h
a given point but in opposite directions with (5) None of these
speeds of 7 m/s and 8 m/s respectively. If 10. In a flight of 3000 km an aircraft was slowed
the circumference of the circle is 300 m down by bad weather. If average speed for
after what time will they meet at the starting the trip was reduced by 100 km/h and the
point ? time increased by one hour, find the original
(1) 20 s (2) 100 s duration of the flight.
(3) 300 s (4) 200 s (1) 5 h (2) 6 h
(5) None of these (3) 4 h (4) None of these
5. During a journey of 80 km a train covers (5) None of these
first 60 km with a speed of 40 km/h and 11. The diameter of a cycle wheel is 70 cm. A
completes the remaining distance with a cyclist takes 30 h to reach a destination at
speed of 20 km/h. What is the average th e spee d o f 22 km/h. Ho w many
speed of the train during the whole journey ? revolutions will the whell make during this
(1) 30 km/h (2) 32 km/h journey ?
(3) 36 km/h (4) 40 km/h (1) 3 million (2) 3 lakh
(5) None of these (3) 4 lakh (4) 5 lakh
6. If a man travels at 30 km/h. he reaches his (5) None of these
destination late by 10 min but if he travels 12. It takes eight hours for a 600 km journey,
at 42 km/h, then he reaches 10 min earlier. if 120 km is done by train and the rest by
Therefore, the distance travelled by him is car. It takes 20 min more, if 200 km is done
(1) 36 km (2) 35 km by train and the rest by car. The ratio of
(3) 40 km (4) 45 km the speed of the train to that of the speed
(5) None of these of the car is
7. Two cars start tog ether in the same (1) 4 : 3 (2) 3 : 4
direction from the same place. The first goes (3) 3 : 2 (4) 2 : 3
with a uniform speed of 10 km/h. The sec- (5) None of these
ond goes at a speed of 8 km/h in the first
LEARN MATHS FROM S.K. RAJU (9811549822, 9811649822)
19
13. Points A and B are 70 km apart on a rate of 3.75 km/h and 4.5 km/h respec-
highway. One car starts from A and the tively. When will they meet for the first time ?
another one from B at the same time. If they (1) 5.5 min (2) 6.0 min
travel in the same direction, they meet in 7 (3) 5.28 min (4) 4.9 min
h. But if they travel towards each other, (5) None of these
they meet in one hour. The speeds of the 20. A gun is fired at a distance of 3.32 km from
two cars are Babar. He hears the sound 10 s later. Find
(1) 45 and 25 km/h the speed of the sound.
(2) 70 and 10 km/h (1) 301 m/s (2) 302 m/s
(3) 40 and 30 km/h (3) 332 m/s (4) 340 m/s
(4) 60 and 40 km/h (5) None of these
(5) None of these 21. The jogging track in a sports complex is 726
14. A journey of 192 km between two cities m in circumference. Suresh and his wife
takes two hours less by a fast train than by start from the same point and walk in
a slow train. If the average speed of the slow opposite direction at 4.5 km/h and 3.75
train is 16 km/h less than that of the fast km/h respectively. They will meet for the
train, then the average speed of the fast first time in
train is (1) 5.5 min (2) 6.0 min
(1) 36 km/h (2) 64 km/h (3) 5.28 min (4) 4.9 min
(3) 7 km/h (4) 48 km/h (5) None of these
(5) None of these 22. There are 20 poles with a constant distance
15. A train 100 m long passes a bridge at the between each pole. A car takes 24 s to reach
rate of 72 km/h in 25 s. The length of the the 12 th pole. How much more time will it
bridge is take to reach the last pole ?
(1) 150 m (2) 400 m (1) 25.25 s (2) 17.45 s
(3) 300 m (4) 200 m (3) 35.75 s (4) 41.45 S
(5) None of these (5) None of these
16. A train 100 m in length travels at 60 km/h. 23. A train can travel 20% faster than a car.
How much time does the train take in Both start from the point A at the same time
passing a man walking at 6 km/h against and reach point B, 75 km away from A at
the train the same time. On the way however the
(1) 6 s (2) 12 s train lost about 12.5 min while stopping at
(3) 10 s (4) 18 s the stations. The speed of the car is
(5) None of these (1) 50 km/h (2) 55 km/h
17. A car driver travels from the plains to the (3) 60 km/h (4) 65 km/h
hill station, which are 200 km a part at an (5) None of these
average speed of 40 km/h. In the return 24. Two trains of equal length are running on
trip he covers the same distance at an parallel lines in the same direction at 46
average speed of 20 km/h the car. The km/h and 36 km/h. The faster train passes
average speed of the car over the entire the slower train in 36 s. The length of each
distance of 400 km is train is
(1) 16.56 km/h (1) 50 m (2) 80 m
(2) 17.89 km/h (3) 72 m (4) 82 m
(3) 26.67 km/h (5) None of these
(4) 35 km/h 25. An aeroplane first flew with a speed of 440
(5) None of these km/h and covered a certain distance. It still
18. A man in a train notices that he can count had to cover 770 km less than what it had
21 telephone posts in one minute. If they already covered but it flew with a speed of
are known to be 50 m apart, then at what 660 km/h. The average speed for the entire
speed is the train travelling ? flight was 550 km/h. Find the total dis-
(1) 57 km/h (2) 60 km/h tance covered.
(3) 63 km/h (4) 55 km/h (1) 3250 km (2) 2750 km
(5) None of these (3) 4400 km (4) 1375 km
19. A circular running path is 726 m in (5) None of these
circumstance. Two men start from the same
point and walk in opposite directions at the
LEARN MATHS FROM S.K. RAJU (9811549822, 9811649822)
20
SHORT ANSWERS 4. (3) According to the question,
1. (2) 2. (3) Relative speed = (8 - 7) =1 m/s
3. (2) 4. (3) Distance = 300 m
5. (1) 6. (2)  Time taken to cover this distance
7. (1) 8. (2)
300
9. (1) 10. (1) = = 300 s
11. (2) 12. (2) 1
13. (3) 14. (4) 5. (1) According to the question,
15. (2) 16. (1) The clock gains 15 m in 24 h. Then, in 16 h
17. (3) 18. (2) it will gain 10 min
19. (3) 20. (3) Hence, the time shown by this clock will
21. (1) 22. (4) be 4 : 10 am.
23. (3) 24. (1) 6. (2) Let the distance be x km.
25. (2) Then, correct time at a speed of 30 km/h
x 10
EXPLANATIONS = -
30 60
1. (2) Let the distance between two stations Correct time at a speed of 42 km/h
be x km.
x 10
x 1 x 5 = +
Then, - = - 42 60
50 6 30 6
x 10 x 10
x x 2  - = +
42 60
 - = 30 60
30 50 3
x x 2
5x  3x 2  - =
 = 30 42 6
150 3
 6x = 300 x 12 2
 =
 x = 50 km 1260 6
2. (3) Let the distance be x km. 1260  2
x 2.5 x 5 x =
72
Then, - = +
8 60 10 60  x = 35 km
x x 5 2.5 7. (1) Let second car overtake the first car after
- = + n h.
8 10 60 60
 Distance covered by first car = Distance
2x 7.5 covered by second car
 =
80 60 n 1 
 1  1 
7.5  80  10n = 8 +  8  2  +  8  2  +  8  2 
     
x= = 5 km
2  60
1
3. (2) According to the question,  10n = 8n + 2 [l + 2 +....+(n - 1)]
Speed of aeroplane against the wind
= (320 - 40) = 280 km/h 1 n n  1
Let the distance be x km.  10n = 8n +
2 2
135 x
Then, = 1 2
60 280  2n = (n - n)
4
 x = 630 km
Speed of Aeroplane with the wind
n=9
= (320 + 40) as = n  0
Hence the answer is 9 h.
= 360 km/h
8. (2) Relative speed of rockets
 Time taken by aeroplane with the wind
= (42000 + 18000)
630 = 60000 mile/h
= × 60 min = 105 min
360

LEARN MATHS FROM S.K. RAJU (9811549822, 9811649822)


21
It means both of them together cover a x: y=3:4
distance of 60000 miles between then in 13. (3) Let the speed of the cars be x km/h and
60 min or 1000 miles 1 min. y km/h.
Hence, they should be 1000 miles apart, 1 Relative speed in same direction
min before impact. = (x - y) km/h
9. (1) Let the speed of the train be x km/h Relative speed in opposite direction
and that of car be y km/h. = (x + y) km/h
160 600 70
Then, + y = 8 ...(i) Now, x  y = 1
x

240 520 41  x + y = 70 ...(i)


+ y = ...(ii)
x 5 70
and x  y = 7
Solving Eqs. (i) and (ii), we get
x = 80 and y = 100  x - y = 10
10. (1) Let the original time taken by the plane Solving Eqs. (i) and (ii),
x h. x = 40 km/h and y = 30 km/h
The, original speed 14. (4) Let the speed of the faster train be x
3000 km/h.
= km/h
x  Speed of the slower train = (x -16) km/h
3000 3000 192 192
 = - 100  = -2
x 1 x x x  16
 x =5h  x = 48 km/h
11. (2) Acco rdi ng to th e que stio n, 15. (2) Let the length be x m.
Circumference of the wheel 5
Then, (x + 100) = 72 × 25 × = 500
22 18
= 2 r = 2 × × 35 = 220 m
7  x = 500 - 100 = 400 m
 Number of revolution made by the wheel 16. (1) According to the question,
during the journey 66  5
(  22 km = 2200000 cm) Relative =(60 + 6) = 66 km/h = m/s
18
= 300000 = 3 lakh Distance = 110 m
12. (2) Let the speed of the train be x km/h
and car be y km/h. 110  18
Time = =6s
66  5
120 400
Then, + y = 8 ...(i) 17. (3) According to the question,
x
2xy
200 400 25 Average speed = x  y where x and y are
and + y = ...(ii)
x 3 two different speeds covering same distance
From Eqs. (i) 120y + 480x = 8xy
2  40  20
From Eqs. (ii) 200y + 400x =
25
xy  Average speed =  20  40  = 26.67 km/h
3
18. (2)  Distance between 21 posts
120y  480x
 = (21/01) × 50 = 1000 m
8  Speed of train = 1 km/min = 60 km/h
3  200y  400x  19. (3)  Relative speed
= = (4.5 + 3.75) = 8.25 km/h
25
726
 25y + 60x = 24y + 48x  Distance = 726 m = = 0.726 km
1000
 12x = 9y
0.726
x 3  Required time = 8.25 × 60 = 5.28 min
 y=
4

LEARN MATHS FROM S.K. RAJU (9811549822, 9811649822)


22
20. (3)  Distance = 3.32 × 100 24. (1) Let the length of each train be xm.
Time = 10 s Then, total distance covered = (x + x)
3320 = 2x m
Speed = = 332 m/s Relative speed = (46 - 36)
10
21. (1)  Relative speed = (500 + 700) 10  5
= 10 km/h = m/s
= 1200 m/min 18
4200 2x  18
 Time taken = 1200 = 3.5 min Given (time) 36 =
50
22. (4) Let the distance between each pole be x  x = 50 m
m. 25. (2) Let the plane covers x km with 440 km/
Then, distance up to 12th pole = 11x m h and (x - 770) km at a speed of 660 km/h.
11x Hence, it covers a total distance of (2x - 770)
 Speed = 22 s km at a speed of 500 km/h

 Time to cover total distance up to 20th


19x  24
pole = = 41.45 s
11x
Total distance
23. (3) Let the speed of the car be x km/h. Average speed = Total time
6x
 Speed of train = x × 1.2x = km/h
2x  770
5
500 =
x x  770
5 25 75 
 × 75 +
2  60
= 440 660
6x x
2x  770 x x  770
75  2  60  = +
= = 60 km/h 500 440 660
6  25
 x = 1760
 Total distance covered
= 2x - 770 = 2 × 770 - 770 = 2750 km

LEARN MATHS FROM S.K. RAJU (9811549822, 9811649822)


1
TIME AND WORK
NATIONALISED BANKS can complete the same piece of work in 8
& IBPS PO/MT/SO days. 2 men, 8 children and 3 women work
1. 12 men can complete a piece of work in 36 together for 2 days. If only women were to
days. 18 women can complete the same finish the remaining work in 2 days, how
piece of work in 60 days. 8 men and 20 many total women would be required ?
women work together for 20 days. If only (1) 12 (2) 18
women were to complete the remaining (3) 24 (4) 20
piece of work in 4 days, how many women (5) None of these
would be required ? (PNB Agriculture Officer
(1) 70 (2) 28 Exam. 04.01.2009)
(3) 66 (4) 40 7. 18 children can do a piece of work in 12
(5) None of these days. How many children would be required
(Union Bank of India PO to do the same work in 8 days ?
Exam. 27.11.2005) (1) 12 (2) 18
2. 9 children can complete a piece of work in (3) 24 (4) 27
360 days. 18 men can complete the same (5) None of these
piece of work in 72 days and 12 women can (Canara Bank PO Exam. 15.03.2009)
complete the piece of work in 162 days. In 8. 8 men and 4 women together can complete
how many days can 4 men, 12 women and a piece of work in 6 days. Work done by a
10 children together complete the piece of man in one day is double the work done by
work ? a woman in one day. If 8 men and 4 women
(1) 124 (2) 81 started working and after 2 days, 4 men left
(3) 68 (4) 96 and 4 new women joined, in how many
(5) None of these more days will the work be completed ?
(Corporation Bank PO (1) 5 days (2) 8 days
Exam. 29.07.2006) (3) 6 days (4) 4 days
3. Fifty six men can complete a piece of work (5) 9 days
in 24 days. In how many days can 42 men (United Bank of India PO
complete the same piece of work ? Exam. 21.06.2009)
(1) 18 (2) 32 9. B and C together can complete a work in 8
(3) 98 (4) 48 days, A and B together can complete the
(5) None of these same work in 12 days and A and C together
(Bank Of Maharashtra PO can complete the same work in 16 days. In
Exam. 25.05.2008) how many days can A, B and C together
4. 42 women can do a piece of work in 18 days. complete the same work ?
How many women would be required do the
9 5
same work in 21 days ? (1) 3 (2) 7
(1) 36 (2) 24 13 13
(3) 30 (4) 44 5 5
(5) None of these (3) 7 (4) 3
12 12
(Andhra Bank PO Exam. 14.09.2008)
(5) None of these
5. 30 men can do a piece of work in 16 days.
(Andhra Bank PO Exam. 05.07.2009)
How many men would be required to do the
10. 10 men can complete a piece of work in 8
same work in 20 days ?
days. 20 women can complete the same
(1) 12 (2) 36
piece of work in 6 days. In how many days
(3) 48 (4) 24
16 men 18 women together can complete
(5) None of these
the same piece of work ?
(Oriental Bank of Commerce
PO Exam. 21.12.2008) 5 6
6. 8 men can complete a piece of work in 4 (1) 2 (2) 3
7 7
days. 12 women can complete the same
piece of work in 4 days whereas 8 children 3 3
(3) 3 (4) 2
7 7

LEARN MATHS FROM S.K. RAJU (9811549822, 9811649822)


2
(5) None of these 3 women and 4 children worked together
(PNB Specialist Officer for 1 day. If only men were to finish the
Exam. 16.08.2O09) remaining work in 1 day, how many total
11. Ayesha can complete a piece of work in 16 men would be required ?
days. Amita can complete the same piece (1) 4
of work in 8 days. If both of them work (2) 8
together in how many days can complete (3) 6
the same piece of work ? (4) Cannot be determined
(5)None of these
2
(1) 6 days (2) 4 days (United Bank Of India PO
5 Exam. 14.11.2010)
1 16. Vikas gets Rs 350 for every day that he
(3) 5 days (4) 12 days works. If he earns Rs 9,800 in a month of
3
31 days, for how many days did he work ?
(5) None of these
(1) 25 days (2) 30 days
(Indian Bank Rural Marketing
(3) 24 days (4) 28 days
Officer Exam. 03.01.2010)
(5) None of these
12. 6 men can complete a piece work in 12
(PNB Management Trainee
days. 8 women can complete the same piece
Exam. 28.11.2010)
of work in 18 days whereas 18 children can
17. A water tank has three taps A, B and C.
complete the piece of work in 10 days. 4
Tap A, when opened, can fill the water tank
men, 12 women and 20 children work
alone in 4 hours. Tap B, when opened, can
together for 2 days. If only men were to
fill the water tank alone in 6 hours and tap
complete the remaining work in 1 day how
C, when opened, can empty the water tank
many men would be required totally ?
alone in 3 hours. If taps A, B and C are
(1) 36
opened simultaneously how long will it take
(2) 24
to fill the tank completely ?
(3) 18
(1) 10 hours (2) 8 hours
(4) Cannot be determined
(3) 18 hours (4) 12 hours
(5) None of these
(5) None of these
(Bank Of India Banking Officer
(Indian Bank PO Exam. 02.01.2011)
Exam. 24.01.2010)
18. 4 girls can do a piece of work in 8 days, 3
13. 3 men can complete a piece of work in 6
boys can do the same piece of work in 9
days. 5 women can complete the same work
days, 7 men do the same piece of work in 2
in 18 days. In how many days will 4 men
days and 5 women can do the same piece
and 10 women together complete the same
of work in 4 days. Who is least efficient ?
work ?
(1) Boys (2) Girls
(1) 3 days (2) 5 days
(3) Women (4) Men
(3) 2 days (4) 4 days
(5) Boys and Men both
(5) None of these
(Union Bank Of India PO
(Bank Of Baroda PO Exam. 30.05.2010)
Exam. 09.01.2001)
14. 8 men can complete a piece of work in 20
19. Two men alone or three women alone can
days. 8 women can complete the same work
complete a piece of work in 4 days. In how
in 32 days. In how many days will 5 men
many days can 1 woman and one man
and 8 women together complete the same
together complete the same piece of work ?
work ?
(1) 6 days
(1) 16 days (2) 12 days
(3) 14 days (4) 10 days 24
(2) days
(5) None of these 5
(Central Bank Of India PO
Exam. 25.07.2010) 12
(3) days
15. 2 men can complete a piece of work in 6 1.75
days. 2 women can complete the same piece (4) Cannot be determined
of work in 9 days, whereas 3 children can (5) None of these
complete the same piece of work in 8 days.

LEARN MATHS FROM S.K. RAJU (9811549822, 9811649822)


3
(Corporation Bank PO 2. 12 men take 36 days to do a work while 12
Exam. 16.01.2011)
3
20. Four examiners can examine a certain women complete th of the same work in
number of answer papers in 10 days by 4
working for 5 hours a day. For how many 36 days. In how many days 10 men and 8
hours in a day would 2 examiners have to women together will complete the same
work in order to examine twice the number work ?
of answerpapers in 20 days ? (1) 6 (2) 27
(3)12 (4) Data inadequate
1 (5) None of these
(1) 8 hours (2) 7 hours
2 (SBI PO Exam. 26.11.2006)
3. ‘A’ can complete a piece of work in 12 days.
1
(3) 10 hours (4) 8 hours ‘A’ and ‘B’ together can complete the same
2 piece of work in 8 days. In how many days
(5) None of these can ‘B’ alone complete the same piece of
(Punjab & Sind Bank PO work ?
Exam. 23.01.2011) (1) 15 days (2) 18 days
21. 6 women and 6 men together can complete (3) 24 days (4) 28 days
a piece of work in 6 days. In how many days (5) None of these
can 15 men alone complete the piece of (SBI PO Preliminary (Tire-I)
work if 9 women alone can complete the Exam. 27.04.2008)
work in 10 days ? 4. 12 men can do a piece of work in 10 days.
(1) 6 How many men would be required to do the
(2) 5 same work in 8 days ?
(3) 7.2 (1) 14 (2) 18
(4) Cannot be determined (3) 16 (4) 12
(5) None of these (5) None of these
(Punjab & Sind Bank PO (SBI PO Preliminary (Tire-I)
Exam. 23.01.2011) Exam. 27.07.2008)
22. A and B together can complete a task in 20 5. A and B together complete a piece of work
days. B and C together can complete the in T days. If A alone completes the work in
same task in 30 days. A and C together can T +3 days and B alone completes the piece
complete the same task in 40 days. What of work in T +12 days, what is T ?
is the respective ratio of the number of days (1) 3 days
taken by A when completing the same task (2) 12 days
alone to the number of days taken by C (3) 9 days
when completing the same task alone ? (4) Cannot be determined
(1) 2 : 5 (2) 2 : 7 (5) None of these
(3) 3 : 7 (4) 1 : 5 (SBI PO Preliminary (Tire-I)
(5) 3 : 5 Exam. 27.07.2008)
(IBPS Bank PO/MT CWE 17.06.2012) 6. 4 men can complete a piece of work in 2
days. 4 women can complete the same piece
SBI PO EXAMS of work in 4 days whereas 5 children can
1. Three men, four women and six children complete the same piece of work in 4 days.
can complete a work in 7 days. A woman If 2 men, 4 women and 10 children work
does double the work a man does and a together, in how many days can the work
child does half the work a man does. How be completed ?
many women alone can complete this work (1) 1 day (2) 3 days
in 7 days ? (3) 2 days (4) 4 days
(1) 8 (5) None of these
(2) 7 (SBI & Rural Business PO
(3) 12 Exam. 18.04.2010)
(4) Cannot be determined 7. Amit and Sujit together can complete an
(5) None of these assignment of data entry in 5 days. Sujit’s
(SBI Banks PO Exam. 18.05.2003) speed is 80% of Amit’s speed and the total

LEARN MATHS FROM S.K. RAJU (9811549822, 9811649822)


4
key depressions in the assignment are (5) None of these
5,76,000. What is Amit’s speed in key (LIC Assistant Administrative
depressions per hour if they work for 8 Officer (AAO) Exams. 24.04.2005)
hours a day ? 3. A man’s basic pay for a 40 hours week is
(1) 4800 (2) 6400 Rs 200. Overtime is paid at 25% above the
(3) 8000 (4) 7200 basic rate. In a certain week, he worked
(5) None of these overtime and his total was Rs 300. He
(SBI Associate Banks PO therefore, worked for a total of (in hours):
Exam. 07.08.2011) (1) 52 (2) 56
(3) 58 (4) 62
RBI GRADE-B OFFICER EXAMS (United India Insurance Co.
1. 9 Men working for 7 hours a day can AAO Exam. 11.03.2007)
complete a piece of work in 15 days. In how 1
many days can 6 men working for 9 hours 4. If 9 men working 7 hours a day can finish
2
a day, complete the same piece of work ? a piece of work in 20 days will be taken
1 3 how many days will be taken by 12 men,
(1) 17 days (2) 16 days working 6 hours a day to finish the work ?
2 4
It is being given that 2 men of latter type
3 work as much as 3 men of the former type ?
(3) 16 days (4) 15 days
4
1
(5) None of these (1) 9 (2) 11
(RBI Grade-B Officer Exam. 2008) 2
2. If 12 boys or 15 girls can do a work in 48 1
(3) 12 (4) 13
days, in what time will 24 boys and 6 girls 2
do twice the work ? (United India Insurance (AAO)
(1) 42 days (2) 40 days Exam. 11.03.2007)
(3) 45 days (4) 30 days 5. Seven men, five women and eight children
(5) None of these were given an assignment of distributing
(RBI Officer Grade ‘B’ Online 2000 books to students in a school over a
Exam. 25.08.2013 period of three days. All of then distributed
books on the first day. On the second day
INSURANCE EXAMS two women and three children remained
1. If it takes A four days to dig a certain ditch, absent and on the third day three men and
whereas B can dig it in 8 days and A, B, C five children remained absent. If the ratio
of the number of books distributed in a day
2
together can dig it in 2 days, how long C by a man, a woman and a child was 5 : 4 : 2
7 respective, a total of approximately how
alone would take to dig it ? many books were distributed on the second
(1) 8 (2) 4 day ?
(3) 6 (4) 12 (1) 1000 (2) 800
(5) 16 (3) 650 (4) 900
(United India Insurance Co. (5) Cannot be determined
AAO Exam. 21.04.2002) (LIC Assistant Administrator
2. P can complete a work in 12 days working Officer Exam 2008)
8 hours a day. Q can complete the same 6. 15 men take 21 days of 8 hours each to do
work in 8 days working 10 hours a day. If P a piece of work. How many days of 6 hours
and Q work together, working 8 hours a each would 21 women take, if 3 wonen do
day, in how many days can they complete as much work as 2 men ?
the work ? (1) 18 (2) 20
6 5 (3) 25 (4) 30
(1) 6 (2) 6 (LIC Assistant Administrative
11 11
Officer (AAO) Exam. 07.06.20O9)
6 5 7. A is thrice as good a workman as B and so
(3) 5 (4) 5
11 11 takes 60 days less than B for doing a job.

LEARN MATHS FROM S.K. RAJU (9811549822, 9811649822)


5
The time in which they can do the job 13. ‘x’ number of men can finish a piece of work
together is in 30 days. If there were 6 men more, the
(1) 30 days (2) 60 days work could be finished in 10 days less. The
original number of men is
1
(3) 22 days (4) 45 days (1) 6 (2) 10
2 (3) 12 (4) 15
(New India Assurance AO (NICL (GIC) AO Exam.
Exam. 25.10.2009) 08.09.2013 (Paper-I)
8. A certain number of persons can complete 14. A man, a woman and a boy together
a work in 100 days. If there be 10 persons complete a piece of work in 3 days. If a man
less, it would have taken 10 days more for alone can do it in 6 days and a boy alone in
the work to be completed. The number of 18 days, how long will a woman take to
persons in the beginning was complete the work ?
(1) 90 (2) 105 (1) 9 days (2) 21 days
(3) 110 (4) 120 (3) 24 days (4) 27 days
(New India Insurance AA0 (NICL (GIC) Administrative
Exam. 22.05.2011) Officer Exam. 15.12.2013)
9. A is twice as good a workman as B and
together they finish a piece of work in 14 15. 8 men can finish a piece of work in 40 days.
days. The number of days taken by A alone If 2 more men join with them, then the work
to finish the work is : will be completed in
(1) 11 (2) 21 (1) 30 days (2) 32 days
(3) 28 (4) 42 (3) 36 days (4) 25 days
(General Insurance Corporation (NICL (GIC) Administrative
AAO Exam. 11.12.2011) Officer Exam. 15.12.2013)
10. M and N can do a work in 10 days and 15 16. 10 women can complete a work in 7 days
days respectively. If M starts on the work and 10 children take 14 days to complete
and both work alternately day after day, in the work. How many days will 5 women and
how many days will the work be completed ? 10 children take to complete the work ?
(1) 10 (2) 12 (1) 3
(3) 8 (4) 9 (2) 5
(5) None of these (3) 7
(LIC Assistant Administrative (4) Cannot be determined
Officer (AAO) Exam. 12.05.2013) (NICL (GIC) Administrative
11. 30 men can do a piece of work in 16 days. Officer Exam. 15.12.2013)
How many men would be required to do the 17. 9 children can complete a piece of work in
same work in 20 days ? 360 days, 18 men can complete the same
(1) 12 (2) 36 piece of work in 72 days and 12 women can
(3) 48 (4) 24 complete it in 162 days. In how many days
(5) None of these can 4 men, 12 women and 10 children
(United India Insurance AO together complete the piece of work ?
Exam. 26.05.2013 (1) 68 days (2) 81 days
12. A work can be completed by P and Q in 12 (3) 96 days (4) 124 days
days, Q and R in 15 days, R and P in 20 (NICL (GIC) Administrative
days. In how many days P alone can finish Officer Exam. 15.12.2013)
the work ?
(1) 10 (2) 20
(3) 30 (4) 60
(NICL (GIC) AO Exam.
08.09.2013 (Paper-I)

LEARN MATHS FROM S.K. RAJU (9811549822, 9811649822)


6
SHORT ANSWERS 20 7
NATIONALISED BANKS  Remaining work = 1 - =
27 27
& IBPS PO/MT/SO
 18 × 60 women do 1 work in 1 day.
1. (1) 2. (2) 1
3. (2) 4. (1)  1 woman does = Work in 1 day
80  60
5. (4) 6. (1)
 1 woman does in 4 days
7. (4) 8. (1)
9. (2) 10. (5) 4 1
= = Work
11. (3) 12. (1) 180  60 18 15
13. (1) 14. (1)
15. (2) 16. (4) 1
 18 15 work is done in 4 days by 1 woman
17. (4) 18. (2)
19. (2) 20. (3)
7 18  15  7
21. (5) 22. (4)  27 work is done in 4 days by = 27
SBI PO EXAMS = 70 days
2. (2) Clearly,
1. (2) 2. (2)
9 × 360 children = 18 × 72 men
3. (3) 4. (5)
= 12 × 162 women
5. (5) 6. (1)
7. (3)  45 children =18 men = 27 women
 5 children = 2 men = 3 women
RBI GRADE-B OFFICER EXAMS Now, 4 men +12 women +10 children
1. (5) 2. (4) = 4 men + 8 men + 4 men = 16 men
3. (2) 4. (3)  18 men can complete the work in 72 days.
5. (3) 6. (4)  16 men can complete the work
7. (3) 8. (3) 18  72
9. (2) 10. (2) in= = 81 days
16
11. (4) 12. (3)
3. (2)  56 men can complete 1 work in 24
13. (3) 14. (1)
days.
15. (2) 16. (3)
 1 man complete the work in 24 × 56 days.
17. (2)
 42 men will do the same work in
EXPLANATIONS 24  56
= = 32 days
NATIONALISED BANKS 42
& IBPS PO/MT/SO QuickerMethod
1. (1)  12 men can complete the work in 36
days.  M 1 D 1 = M2 D2
 12 × 36 men can complete the work in 1  56 × 24 = 42 × D2
day.
56  24
Again,  D2 = = 32 days
42
 18 women can complete the work in 60
days. 4. (1) Days Women
 18 × 60 women can complete the work in 18 42
1 day. 21  x  
12 × 36 men = 18 × 60 women
 21 : 18 : ; 42 : x
 2 men = 5 women
Now, 8 men + 20 women
= (4 × 5 + 20) women = 40 women 42  18
x= = 36
 18 women complete the work in 60 days. 21
 40 womens’ 20 days’ work 5. (4) Days Men
40  20

16 30

20
= =
18  60 27 20 x
 20 : 16 = 30 : x
LEARN MATHS FROM S.K. RAJU (9811549822, 9811649822)
7
 20 × x = 16 × 30 1 1 1 6  4  3 13
 + + = =
30 16 8 12 16 48 48
x= = 24 men
20 13
6. (1) As per given information,  (A + B + C)’s 1 day’s work =
96
8 × 4 men = 12 × 4 women
= 8 × 8 children  A, B and C together can complete the
 32 men = 48 women = 64 children work in
96 5
= 7 days
 2 men = 3 women = 4 children 13 13
2 men + 8 children + 3 women 10. (5)  10 men complete the work in 8 days.
= (3 + 6 + 3) women =12 women  80 men will complete the work in 1 day.
 20 women complete the work in 6 days.
1
12 women’s 2 day’s work =  120 women complete the work I in 1 day.
2  80 men = 120 women
1  2 men = 3 women
Remaining work =  16 men + 18 women = 16 men
2
 Required number of women = 12 2
7. (4) Days Children + 18 × men = 28 men
3
12 18  10 men can do the work in 8 days
8  x   28 men can do the work in
 8 : 12 = 18 : x 10  8 20 6
= = 2 days.
 8x = 12 × 18 28 7 7
12 18 11. (3) Ayesha’s and Amita’s 1 day’s work
x = = 27 1 1 1 2 3
8
= + = =
Note : you can use the formula : 16 8 16 16
M 1 D 1 = M2 D2 and solved it.  Ayesha and Amita together can complete
8. (1) According to the question,
16 1
1 man = 2 women the work in = 5 days
3 3
 8 men + 4 women
= (16 + 4) women = 20 women 12. (1) 72 men = 144 women = 180 children
4 men + 8 women = 16 women 2 men = 4 women = 5 children 4 men +12
20 women’s 2 days’ work women + 20 children
= 4 men+ 6 men+ 8 men = 18 men
2 1 When 6 men do 1 work in 12 days
= = Part
6 3 18 men will do the same in 4 days.
1 2  In 2 days half work will done.
Remaining work = 1 - = 1
3 3  Remaining work =
 20 women complete 1 work in 6 days. 2
2 M1D1 M2D2
 16 women will do work in  W = W
3 1 2

20  6 2 1
=
16
× = 5 days
3  × 6 × 12 = l × M 2
2
1  M2 = 36 = Number of men
9. (2) (B+C)’s 1 day’s work = ...(i) 13. (1) Tricky approach
8
18 men = 5 × 18 = 90 women
1  1 man = 5 women
(A+B)’s 1 day’s work = ...(ii)
12  4 men +10 women = 30 women
1  M 1 D 1 = M 2D 2
(A+C)’s 1 day’s work =
16
...(iii)  5 × 18 = 30 D2
On adding all these three equations, 5  18
2 (A + B + C)’s 1 day’s work  D2 = = 3 days
30
LEARN MATHS FROM S.K. RAJU (9811549822, 9811649822)
8
14. (1) 8 × 20 men = 8 × 32 women 21. (5)  9 women can complete the work in
 5 men = 8 women 10 days.
5 men + 8 women =16 women  6 women can complete the work in
M 1 D 1 = M 2D 2 10  9
 8 × 32 = 16 × D2 6
= 15 days.
8  32
 D 2= = 16 days 6 2
16 Part of work done by 6 women = =
15 5
15. (2) 2 × 6 men = 18 women = 24 children
2 men = 3 women = 4 children 2 3
3 women + 4 children = 4 men  Part of work done by 6 men = 1 - 5 = 5
1
part of work done in 1 day by 4 men = 3
3  5 part of work done by 6 men in 1 days.
2
 Remaining work will be finished by 8 6
3  1 part of work done by 6 men in 3 × 5
men in 1 day. Hence, 8 men would be
required. = 10 days.
10  6
16. (4) Number of days = 9800 = 28  15 men can complete the work in
15
350
17. (4) Part of the tank filled in 1 hour when all = 4 days
the taps are opened 1
22. (4) (A + B)’s 1 day’s work =
1 1 1 20
= + -
4 6 3 1
(B + C)’s 1 day’s work =
3 2  4 1 30
= =
12 12 1
Hence, the tank will be filled in 12 hours. (C + A)’s 1 day’s work =
40
18. (2) 4 × 8 girls = 3 × 9 boys
On adding,
= 7 × 2 men = 5 × 4 women
2(A + B + C)’s 1 day’s work
 32 girls = 27 boys = 14 men
= 20 women 1 1 1
= + +
19. (2) 2 men = 3 women 20 30 40
1 man + 1 woman
6 43 13
= =
3  5 120 120
=   1 women = women
2  2  (A + B + C)’s 1 day’s work
 M1Dl = M2D2 13
=
5 240
3 × 4 = × D2  A’s 1 day’s work
2
3  4  2 24 13 1 13  8
 D2 = = days = - =
5 5 240 30 240
20. (3) 5 1
Examiners work Days Hours/day = =
240 48
4 1 10 5 C’s 1 day’s work
2  2  20  x  13 1 13  12 1
M 1D 1H 1W 2 = M 2D 2H 2W 1 = - = =
240 20 240 240
 2 × 20 × x = 4 × 10 × 5 × 2  Required ratio = 48 : 240 = 1 : 5
4  10  5  2
x= = 10 hours
2  20

LEARN MATHS FROM S.K. RAJU (9811549822, 9811649822)


9
SBI PO EXAMS  B alone can do the work in 24 days.
1. (2) 2 men = 1 woman 4. (5) Days Men
10 12
 
1
 1 man = woman 8 x
2
Where x = number of men
3
 3 men = women  8 : 10 :: 12 : x
2
 8 × x = 10 × 12
Again, 2 children = 1 man
10 12
1 =x= = 15 men
= woman 8
2
1
1 5. (5) (A + B)’s 1 day’s work =
 1 child = woman T
4
1
6 3 A’s 1 day’s work =
 6 children = = women T3
4 2
Now, three men, four women and six 1
B’s 1 day’s work =
children T  12
3 3 1 1 1
= +4+ = 7 women   T  3 +  T  12  =
2 2 T
Hence, 7 women complete the work in 7
days. T  12  T  3 1
2. (2) In 36 days 12 men can do 1 complete   T  3 T  12  =
T
work.
2T  15 1
3  =
In 36 days 12 women can do th of the T 2  15T  36 T
4
 2T2 + 15T = T2 + 15T + 36
work
Since time and the no. of persons is the  T2 = 36
same is both cases,  T = 6 days
6. (1) Tricky Approach
3
1 woman’s daily work = th of 1 man’s daily
4 4 × 2 men = 4 × 4 women = 20 children
work  2 men = 4 women = 5 children
3  2 men + 4 women +10 children
8 women’s daily work= × 8 = 6 men’s = 20 children
4
daily work  M 1 D 1 = M 2D 2
(10 men + 8 womens daily work) = (10 men  5 × 4 = 20 × D2
+ 6 men)  D2 = 1 day
= 16 men’s daily work. 7. (3) Let Amit’s speed be x key depressions
12 men can do the work is 36 days per day.
 16 men can do the work is 4x
12  Sujit’s speed = key depressions per
5
36 × = 27 days.
16 day
1 4x
3. (3) A’s 1 day’s work =  5x + 5 × = 576000
12 5
1  9x = 576000
(A + B)’s 1 day’s work = 576000
8
x= = 64000
 B’s 1 day’s work 9
1 1 3 2 1  Amit’s speed per hour
= - = =
8 12 24 24 64000
= = 8000
8
LEARN MATHS FROM S.K. RAJU (9811549822, 9811649822)
10
RBI GRADE-B OFFICER EXAMS Similarly, Q’s 1 day work working 1 hour a
1. (1) Men Hours/day Days 1
day =
9 7 15 8  10
6  9 x 
Where x = number of days  (A + B)’s 1 day work working 1 hour a day
6 : 9 1 1
  ::15: x = +
9 : 7 8 12 8 10

 6 × 9 × x = 9 × 7 × 15 56 11
= =
60  8 60  8
9  7 15 35 1
 = = 17 days  (A + B)’s 1 day work working 8 hours a
69 2 2
11 8 11
day = =
2. (2)  12 boys = 15 girls 60  8 60
 24 boys = 30 girls  Required answer
 24 boys + 6 girls 60 5
= 36 girls = = 5 days
11 11
M1D1 M2D2 3. (2) Let the man worked overtime for x hours.
 W = W Basic pay of 40 hours = Rs. 200.
1 2

15  48 36  D 2 200
 =  Basic pay of 1 hour = = Rs. 5
1 2 40
According to the question,
15  48
 D2 = = 40 days 5  125
18 200 + x × = 300
100

INSURANCE EXAMS 25
 x = 300 - 200
1. (5) According to the question, 4
1 100  4
A’s one day work = x = = 16
4 25
1  Total hours of work = 40 + 16 = 56
B’s one day work = 4. (3) More men, less days (Indirect) Less hrs/
8
day, more days (Indirect)
(A + B)’s one day work Less efficiency, more days (Indirect)
1 1 3
= + = 
4 8 8
Men 12 : 9 
Similarly, 
15 
7 Hrs / Day 6: 
(A + B + C)’s one day work = 2
16 1 1
Efficiency :
C’s one day work 2 3 
7 3 1
= - = 1 15 1
16 8 16  12 × 6 × ×x=9× × × 20
2 2 3
Therefore, C can complete the whole work
in 16 days. 9 15  2  20
(4)  P can complete a work in 12 days x=
2. 2  3 12  6
working 8 hours a day
25 1
 P can do a work in 12 days working = = 12
2 2
1
1 hour a day = 1
8
 Required number of days = 12
 P’s 1 day work working 1 hour a day 2
5. (3) Let the number of books distributed by
1
= a man, a woman and a child per day be 5x,
8 12 LEARN MATHS FROM S.K. RAJU (9811549822, 9811649822)
11
4x and 2x respectively  x = 3 × 7 = 21 days
Books distributed on the first day
1 1
= 7 × 5x + 5 × 4x + 8 × 2x 10. (2) Work done in 1st two days = +
= 35x + 20x + 16x = 71x 10 15
Book distributed on the second day 3 2 1
= 7 × 5x + 3 × 4x + 5 × 2x = =
30 6
= 35x + 12x + 10x = 57x
 Number of days = 12
Books distributed on the third day 11. (4) Days Men
= 4 × 5x +5 × 4x + 3 × 2x = 46x
 71x + 57x + 46x = 2000 16 30
 174x = 2000 20  x 
 20 : 16 = 30 : x
2000
 57x = 174 × 57 = 655.172  650  20 × x = 16 × 30
6. (4) 3 women = 2 men 30 16
x= = 24
 21 women = 14 men 20
men working hours/day days 1
12. (3) (P + Q)’s 1 day’s work =
 
15 8 21 12
14  6 x 1
(Q + R)’s 1 day’s work =
14 :15 15
 6 : 8  ::21 : x
 1
(R + P)’s 1 day’s work =
 14 × 6 × x = 15 × 8 × 21 20
On adding,
15  8  21
x= = 30 days 2 (P + Q + R)’s 1 day’s work
14  6
7. (3) Tricky approach 1 1 1 5 4 3 1
= + + = =
12 15 20 60 5
A is three times as good a workman as B. (P + Q + R)’s 1 day’s work
 If A completes a work in 30 days, B will 1
complete it in 90 days. =
10
 (A + B)’ 1 day’s work
1 1
1 1 3 1 2  P’s 1 day’s work = 
= + = = 10 15
30 90 90 45
3 2 1
45 = =
 Hence, both will complete the work in 30 30
2
Hence, P alone will complete the work in
1 30 days.
= 22 day.
2 13. (3) M1D1 = M2D2
8. (3) Men Days  x × 30 = (x + 6) × 20
x 100  3x = 2x + 12
x  10  110  x = 12 men
14. (1) (1 man + 1 woman + 1 child)’s 1 day’s
x  10 100
 =
work =
1
x 110
3
 11x - 110 = 10x
 x = 110 men 1 1 1
 Woman’s 1 day’s work = - -
9. (2) If time taken by B be 2x days, then time 3 6 18
taken by A = x days. 6  3 1 1
= =
1 1 1 18 9
 + =
x 2x 14  The woman alone will complete the work
in 9 days.
2 1 1 3 1
 =  =
2x 14 x 7
LEARN MATHS FROM S.K. RAJU (9811549822, 9811649822)
12
15. (2) M1D1 = M2D2 17. (2) According to the question,
 8 × 40 = 10 × D2 9 × 360 children = 18 × 72 men
= 12 × 162 women
8  40
 D2 = = 32 days  5 children = 2 men = 3 women
10
 4 men + 12 women +10 children
16. (3) 10 women complete the work in 7 days. = 10 children + 20 children + 10 children
10 children complete the same work in 14 = 40 children
day’s
 M 1 D 1 = M 2D 2
 1 woman = 2 children  9 × 360 = 40 × D2
 5 women +10 children = 20 children
 By M1D1 = M2D2 9  360
10 × 14 = 20 × D2  D2 = = 81 days
40
10 14
 D2 = = 7 days
20

LEARN MATHS FROM S.K. RAJU (9811549822, 9811649822)


13
MODEL EXERCISES
1. A certain job was assigned to a group of 1
men to did it in 20 days. But 12 men did 7. rd o f th e co ntents of a con tain er
3
not turn up for the job and the remaining
men did the job in 32 days. The original 3
number of men in the group was evaporated on the 1st day. th of the
4
(1) 32 (2) 34 remaining evaporated on the second day.
(3) 36 (4) 40 What part of the contents of the container
(5) None of these is left at the end of the second day ?
2. A mother and a daughter working together
can complete a work in 4 days. But if the 1 1
(1) (2)
mother works alone she can complete the 4 2
work in 6 days. Both of them worked for 1 1
one day and then the mother had to leave. (3) (4)
18 6
How long will the daughter take to complete
(5) None of these
the remaining work ?
8. Two men and 7 children complete a certain
(1) 7 days (2) 8 days
piece of work in 4 days while 4 men and 4
(3) 9 days (4) 10 days
children complete the same work in only 3
(5) None of these
days. The number of days required by 1
3. Ramesh is twice as good workman as Sunil
man to complete the work is
and finished a piece of work in 3 h less than
(1) 60 days (2) 15 days
Sunil. In how many hours they together
(3) 6 days (4) 51 days
could finish that piece of work ?
(5) None of these
1 9. Mr. Ram is an tour and he has Rs 360 for
(1) 2 (2) 2
3 his expenses. If he exceeds his tour by 4
days he must cut down daily expenses by
2
(3) 1 (4) 4 Rs 3. The number of days of Mr. Ram’s tour
3 programme is
(5) None of these (1) 20 days (2) 24 days
4. A can do a work in 18 days, B in 9 days and (3) 40 days (4) 42 days
C in 6 days. A and B start working together (5) None of these
and after 2 days C joins them. What is the
total number of days taken to finish the 2
10. A worker makes a basket in of an hours.
work ? 3
(1) 4.33 (2) 4.0 1
(3) 4.66 (4) 5.33 If he works for 7 h, then how many
2
(5) None of these
baskets will he make ?
5. In a house, there are six 40 W lamps which
are on for 5 h a day and three 80 W fans 3 1
(1) 10 (2) 11
which are on for 10 h a days. If electricity 4 4
costs 2 per kilowatt hours what is the
1
monthly electricity bill ? (3) 12 (4) 13
(1) Rs 216 (2) Rs 280 2
(3) Rs 315 (4) Rs 400 (5) None of these
(5) None of these 11. If 5 men take an hour to dig a pitch, then
6. A worker is paid Rs 56 for 35 h week. Up to how long should 12 men take to dig a ditch
40 h, he is paid at the normal rate and on of the same type ?
overtime, 1.5 times the normal. How many (1) 25 min (2) 30min
hours did he work to get Rs 88 ? (3) 28 min (4) 20 min
(1) 48 (2) 50 (5) None of these
(3) 58 (4) 55 12. A and B can finish a job in 10 days while B
(5) None of these and C can do it in 18 days. A started the
job worked for 5 days, then B worked for
10 days and the remaining job was finished

LEARN MATHS FROM S.K. RAJU (9811549822, 9811649822)


14
by C in 15 days. In how many days could C of a boy, they finish it in 3 days. The boy’s
alone have finished the whole job ? share should be .
(1) 30 days (2) 15 days (1) Rs 75 (2) Rs 225
(3) 45 days (4) 24 days (3) Rs 300 (4) Rs 100
(5) None of these (5) None of these
13. A can do a piece of work in 10 days, while 18. 15 men can complete a work in 210 days.
B alone can do it in 15 days. They work They started the work but at the end of 10
together for 5 days and the rest of the work days 15 additional men, with double
is done by C in 2 days. If they get Rs 450 efficiency, were inducted. How many days
for the whole work, how should they divide in all did they take to finish the work ?
the money ? 1 3
(1) Rs 225, Rs 150, Rs 75 (1) 72 days (2) 84 days
2 4
(2) Rs 250, Rs 100, Rs 100
(3) Rs 200, Rs 150, Rs 100 2
(4) Rs 175, Rs 175, Rs 100 (3) 76 days (4) 70 days
3
(5) None of these (5) None of these
14. A can build up a wall in 8 days while B can 19. Tw o me n A an d B wo rkin g to ge th er
break it in 3 days. A has worked for 4 days complete a piece of work which it would
and then B joined to work with A for an- have taken th em 30 and 40 days
other 2 days only. In how many days will A respectively to complete if they worked
alone build up the remaining part of the separately. If they received a payment of
wall ? Rs 2100. B’s share is
1 1 (1) Rs 900 (2) Rs 800
(1) 13 days (2) 6 days (3) Rs 1200 (4) Rs 1300
3 3
(5) None of these
2 20. Five painters can paint a wall 100 m long
(3) 7 days (4) 7 days
3 in 10 days of 8 h each. How many days of 6
(5) None of these h each will it take for 8 men to paint a wall
15. 4 men and 10 women were put on a work. 30 m long ?
They completed 1/3 of the work in 4 days. 1
After this 2 men and 2 women were (1) 2 (2) 1
increased. They completed 2/9 more of the
2
work in 2 days. If the remaining work is to 3
be completed in 3 days, then how many (3) 3 (4) 1
4
more women must be increased ? (5) None of these
(1) 32 (2) 8 21. There are two types of workers-category I
(3) 50 (4) 55 and category II. A category I worker can
(5) None of these finish a piece of work in 2 m per 3 h and a
16. C is twice efficient as A. B takes thrice as category II worker can finish in m h. If one
many days as C. A takes 12 days to finish worker of category I and two workers of
the work alone If they work in pairs (i.e., category II are employed, the work can be
AB, BC, CA) starting with AB on the first finished in how many hours ?
day, BC on the second day and AC on the (1) 7 m/2 (2) 9 m/2
third day and so on, then how many days (3) 2 m/7 (4) 2 m/9
are required to finish the work ? (5) None of these
1 22. Two coal loading machines each working
(1) 6 days (2) 4.5 days 12 h per day for 8 days handle 9000 tonnes
5
of coal with an efficiency of 90% ; while 3
1 other coal loading machines at an efficiency
(3) 5 days (4) 8 days
9 of 80% are set to handle 12000 tonnes of
(5) None of these coal in 6 days. Find how many hours per
17. Two men undertake to do a piece of work day each should work ?
for Rs 600. One alone could do it in 6 days (1) 20 h/day (2) 18 h/day
and the other in 8 days. With the assistance (3) 16 h/day (4) 14 h/day
(5) None of these
LEARN MATHS FROM S.K. RAJU (9811549822, 9811649822)
15
23. If 36 men can dig a trench 200 m long, 3 m together. What time would they take if both
wide and 2 m deep in 6 days working 10 h Bhavika and Rita worked together ?
a day, in how many days, working 8 h a (1) 5 days (2) 4 days
day will 10 men dig a trench 100 m long, 4 (3) 3 days (4) 6 days
m wide and 3 m deep ? (5) None of these
(1) 15 days (2) 27 days 25. Ten men can finish a piece of work in 10
(3) 20 days (4) 54 days days, whereas it takes 12 women undertake
(5) None of these the work, how many days will they take to
24. Bhavika alone would take 8 h more to complete it ?
complete the job than when Bhavika and 1
Rita worked together. If Rita worked alone, (1) 4 days (2) 4 days
2
1 (3) 5 days (4) 6 days
she would take 4 h more to complete the
2 (5) None of these
job than when Bhavika and Rita worked

LEARN MATHS FROM S.K. RAJU (9811549822, 9811649822)


16
SHORT ANSWERS  They together could finish a piece of work
1. (1) 2. (3) in 2 h
3. (2) 4. (2) 4. (2) Let (A + B)’s 2 days work
5. (1) 6. (2)  1 1 2 1
7. (4) 8. (4) = 2   = =
9. (1) 10. (2)  18 9  6 3
11. (1) 12. (3) 1 2
13. (1) 14. (3) Remaining work = l - =
3 3
15. (2) 16. (3) (A + B + C)’s one day’s work
17. (1) 18. (3)
19. (1) 20. (1)  1 1 1 1
=    =
21. (3) 22. (3)  18 9 6  3
23. (2) 24. (4)
25. (3) 1
work is completed working all of them
3
EXPLANATIONS together in 1 day.
1. (1) Let originally there were x men in the 2
group. Hence, work will be completed in 2 days.
3
 (x - 12) men did the job in 32 days
 Total number of days taken to finish the
 20x = 32 (x -12) work is 4 days.
 x = 32 5. (1) By question,
2. (3) According to the question, Electricity consumption per day
1 = 6 × 40 × 5 + 3 × 80 × 10
(M + D)’s one day’s work = = 1200 + 2400 = 3600 watt
4
= 3.6 kW
Remaining work =
3  Consumption in a month
4 = 3.6 × 30 =108 kW
1  Monthly electricity bill
M’s one day’s work = = 108 × 2 = Rs 216
6 6. (2) Let the worker worked for (40 + x) h.
1 1 1 Then,
D’s one day’s work = - =
4 6 12 56 x 1.5  56
× 40 + = 88
1 35 35
 th of the work is done by the daughter  2.4x = 24
12
in 1 day  x = 10
3  The worker worked for (40 + 10) = 50 h
th of the work will be done by daughter 7. (4) According to question, after one day
4 contents left
12  3
in = 9 days  1 2
4 = 1   =
 3 3
3. (2) According to the question,
Second day contents evapored
Let Sunil finishes the job in x h.
2 3 6
x = × =
Then, Ramesh will finish the job in h. 3 4 12
2
Contents left after second day
x
Given, x - =3 2 6 86 2 1
2 = - = = =
3 12 12 12 6
x=6
8. (2) According to question,
 Sunil finishes a job in 6 h and Ramesh
does it in 3 h. 1
(2M + 7C)’s 1 day work =
Work done by both of them in hour 4
1 1 1 It means that 1 work will be finished by
+ = h (8M + 28C)
6 3 2
LEARN MATHS FROM S.K. RAJU (9811549822, 9811649822)
17
1  1 1 5
Again, (4M + 4C)’s 1 day’s work = =   × 5 =
3  10 15  6
or 1 work will be completed by 12M + 12C
 8M + 28C = 12M + 12C 5 1
Rest Work = 1- =
 M = 4C 6 6
 4M + 4C = 5M  C alone can do the work in 6 × 2 =12 days
Since, 5 M complete a work in 3 days. Then, 5 5 2
1 M will complete it in 15 days. Ratio of their work = : :
10 15 12
9. (1) Let Ram undertakes a tour of x days.
=3 : 2 : 1
360 Share of money = Rs 225, Rs 150 and Rs 75
Then, expenses for each day =
x 14. (3) According to question, work done by A
360 360 4 1
 = 3 in 4 days = =
x4 x 8 2
Net work done by (A + B) in 1 day
1 1 
 360  x  x  4  = 3
   1 1  5
 8  3=
  24
2
x + 4x - 480 = 0
 x = - 24 or x = 20
Work done by (A+B) in 2 days
Since, x  -24
Hence, x = 20 days 5 5
= ×2=
10. (2) By question, 24 12
2
In h he makes 1 basket. 1  5  1
3  Work done in 6 days = +  =
2  12  12
15 1 15
 In h he will make  baskets 11
2 2/3 2  Remaining of the wall is built by A in
12
45 1
= = 11 baskets 8 11 88 22 1
4 4 = = = 7 days
11. (1) According to question, 12 12 3 3
15. (2) Let one man takes x days to complete
5 the work and one woman takes y days to
Required time = h
12 complete the work independently.
5 4  4 10  4 1
= × 60 min = 25 min Then, + =
12 x y 3
12. (3) Let complete the work in x days.
6  2 12  2 2
1 and + =
Work done by (A + B) in 1 day = work x y 9
10
Solving above equations, we get
1 x = 108, y = 216
Work done by (B + C) in 1 day = work Let z women be added to complete the work
18
A’s 5 day’s work + B’s 10 day’s work + C’s in 3 days.
15 day’s work = 1 6  3 3 12  z 
(A + B)’s 5 day’s work + (B + C)’s 5 day’s Then, +
108 216
work + C’s 10 day’s work = 1
1 2 4
5 5 10
+ - =1 = 1 -  =
10 18 x 3 9 9
 x = 45 216  4
13. (1) According to question,  36 + 36 + 3z = = 96
9
Work done by A and B in 5 days  3z = 96 - 72 = 24  z = 8

LEARN MATHS FROM S.K. RAJU (9811549822, 9811649822)


18
16. (3) According to question, 30
A B C  Share of B = × 2100 = Rs 900
40  30
Efficiency 3 : 2 : 6
No. of days 2 : 3 : 1 20. (1) By question,
 Number of days taken by A = 12, M1D1T1 M 2 D 2T2
 W = W2
Number of days taken by B =18 1
and Number of days taken by C = 6
5  10  8 8  D 2  6
5  =
1 day’s work of (A + B) = 100 30
36
5 10  8  30 1
8  D2 = = 2 days
1 day’s work of (B + C) = 100  8  6 2
36
21. (3) Let one worker of category I can finish
9
1 day’s work of (C + A) = 2m
36 the work in h.
3
In 5 days total work done Two workers of category II can finish the
5 8 9 5 8 35 m
= + + + + = work in h.
36 36 36 36 36 36 2
Now, the rest of the work
2m m
 1  
 ie, 36  is done by AC 3 2
  so, required time = 2m m

Number of days taken by AC for the rest of 3 2
1 2m 2 2
36 1 = = mh
the work = = 7m 7
9 9
22. (3) According to question
36
 M1 × D1 × W1 = M2 × D2 × T2 × W1
Therefore, total time taken to complete the  2 × 12 × 8 × 12000 × 0.9
1 1 = 3 × 6 × x × 9000 × 0.8
work = 5 + = 5 days
9 9 2 12  8 12000  0.9
x=
17. (1) According to the question, 3  6  9000  0.8
Ratio of efficiencies of the three persons = 16 h/day
23. (2) According to question
24 24 24  24 24 
= : : -    = 4: 3:1  M1 × D1 × T1 × W2 = M2 × D2 × T2 × W1
6 8 3  6 8 
36 × 6 × 10 × 1200 = 10 ×D2 × 8× 1200
1 36  6  10  1200
 Boy’s share=  4  3  1 × 600  D 2= = 27 days
10  9  1200
1 24. (4) Let Bhavika and Rita working together
= × 600 = Rs 75 can finish a job in x days.
8
Then, in 1 day they will complete
18. (3) Total time taken to finish the work
1
15  210  10  15  200 =
x
work
 
= 10+ 15  15  2 = 10 +
45 Bhavika working alone do in 1 day
200 2
=10 + = 76 days 1
3 3
=  x  8  work
19. (1) A and B can complete the work in 30
and 40 days respectively. Rita working alone will do in 1 day
So, ratio of their wages
1
1 1 =
work
 9
= : = 40 : 30
30 40  x  
 2
LEARN MATHS FROM S.K. RAJU (9811549822, 9811649822)
19
1 1
25. (3) Work done by 1 man in 1 day
1
  x  8 +  2x  9 = 1
x =
100
 2 x  9   2  x  8 1 Work done by 15 men and 6 women
  x  8  2 x  9  = x 15 1
= +
100 120
4 x  25 1
  x  8 2 x  9  = 3 1
x = +
20 20
 x = 6 days
4 1
= = work
20 5
 15 men and women will take 5 days to
complete the work.

LEARN MATHS FROM S.K. RAJU (9811549822, 9811649822)

You might also like